You are on page 1of 377

Case List

Labor Relations Case Digest - Atty. Joyrich Golangco

Test to Determine ER-EE Relationship


1. Francisco v. NLRC
2. Sonza v. ABS CBN
3. Javier v. Flyace Corp
4. SMCEU v. Judge Bersamira
5. Locsin et al v. PLDT
6. People Broadcasting Service v. Secretary of Labor
7. Ymbong v. ABS CBN
8. Professional Services v. CA
9. South East International Rattan Inc v. Coming
10. Tenazas et al. v. Villegas
Article 212

23. Atlas Farms Inc v. NLRC


24. Perpetual Help Credit Cooperative Inc. v. Fuburada
25. Austria v. NLRC
26. DFA v. NLRC
27. PNB v. Cabansag
28. Banez v. Valdevilla
29. Santos v. Servier Philippines Inc
30. Pepsi Cola Distributors Phils v. Galang
31. 7K Corp v. Albanco

Article 218
32. Landbank of the Phil. V. Listana
33. Meralco v. Gala
Article 221

11. Citibank v. CA
12. PAL v. NLRC

34. Nationwide Security and Allied Services v. CA


35. Diamond Taxi v. Llmas
36. Islriz Trading v. Capade et al

Managerial Employee
13. Penarada v. Bagang Plywood Corp
14. SMCC v. Charter Chemical and Coating Corp
15. Jumuad v. Hi-Flyer Food Inc
Article 217
16. Ex-Bataan Veterans Security Agency v. Sec Laguesma
17. Locsin v. Nissan Leases Philippines
18. Reyes v. RTC Makati
19. Okol v. Slimmers World
20. Rural Bank of Coron v. Cortes
21. Halguena v. PAL
22. Santiago v. CF Sharp Crew Management

Article 223
37. Garcia et al v. KJ Commercial
38. Ong v. CA
39. Rosewood Processing Inc v. NLRC
40. Filipinas System Inc. (FSFI) v. NLRC
41. Buenaobra v. Lim King Guan
42. Lepanto Consolidated Mining Corp v. Icao
43. Bergonio v. SEAIR
44. Loon v. Power Master
45. Mcburnie v. Ganzon

3E Andaya Ching Espiritu Hefti Galvez Gammad Lainez Lui Madamba Nagera Narvasa
Ong Palangdao Rosales Sanchez Santos Satrain Tabo (2014-2013)

Labor Relations Case Digest - Atty. Joyrich Golangco

Reinstatement Aspect of LAs Decision

46. Pioneer Texturing Corporation v. NLRC


47. Roquero v. PAL
48. Air Phil Corp v. Zamora
49. Lansangan v. Amkor Technology Philippines
50. Genuino v. NLRC
51. Garcia et al v. PAL
52. Mt. Carmel College v. Resuena
53. Buenviaje v. CA
54. Pfizer Inc v. Velasco
55. Wenphil Corp v. Abing

Article 234 amended by RA 9481


66. Mariwasa Siam Ceramics Inc v. Sec of Labor
67. Electromat Manufacturing and Recording Corporation v.
Lagunzad
68. Eagle Ridge Golf and Country Club v. CA

Certificate of Registration of Labor Organization


69. Tagaytay Highlands International Golf Club Inc. v. Tagaytay
Highlands Employees Union PGTWO
70. SS Ventures International Inc v. SS ventures Labor Union
Article 238-239

Article 224
56. Sy et al. v. Fairland Knitcraft Co.
57. Yupangco Cotton mills v. CA
58. Ando v. Campo
Article 226
59. Employees Union of Bayer Phils v. Bayer Phils
60. Montano v. Verceles
61. Diokno et al v. Cacdac
Article 227

71. Heritage Hotel Manila v. NUWHRAIN-HHMSC


72. Republic of the Phils v. Kawashima Textile Manufacturing
Article 241
73. Del Pilar Academy et al v. Del Pilar Academys Employee Union
74. Edgardo Marino Jr. et al v. Gamilia
Article 242
75. Abaria v. NLRC
Nature of CBA

62. Magbuana v. UY
63. Solomon et al. v. Powertech Corp
64. Philippine Journalist Inc v. NLRC

76. Weslayan University Phils v. Weslayan University Philippines


faculty and Staff Association
Article 245

Article 232
65. Colegio de San Juan de Letran v. Association of Employees
and Faculty of Letran et al

77. Cathay Pacific Steel Corp v. CA


78. Chris Garments v. Sto Tomas

3E Andaya Ching Espiritu Hefti Galvez Gammad Lainez Lui Madamba Nagera Narvasa
Ong Palangdao Rosales Sanchez Santos Satrain Tabo (2014-2013)

Article 247-249

Labor Relations Case Digest - Atty. Joyrich Golangco


Article 255

79. General Santos Coca cola Plant Free Workers Union Tupas v.
Coca-cola Bottlers Phil Inc
80. UST Faculty Union v. UST
81. Phil Skylander Inc. v. NLRC
82. Tropical Hut Employee Union v. Tropical Hut Food Market Inc
83. Purefoods Corp v. Nagkakaisang Samahang Manggawa ng
Purefoods Rank and File
84. De la sale University v. DLSUEA-NAFTEU
85. MSMG_UWP v. UWP
86. Alabang Country Club v. NLRC
87. Standard Chartered Bank Employees Union v. Confesor
88. GMC v. CA
89. Hacienda Fatima v. NFSW Food and General Trade
90. St. John Colleges Inc v. St. John Academy Faculty and
Employees Union
91. Central Azucarera De Bias Employees Union NFL v. Central
Azucarera De Bias Inc
92. UFE-DFA-KMU v. Nestle Phil
93. Malayang Mangagawa ng Stayfast v. NLRC
94. Holy Child Catholic School v. Sec of labor
Article 252
95. Kiok Loy v. NLRC
96. PAL v. Calleya
Article 253-A
97. FVC Labor Union-Phil Transport and General Workers
Association v. SANAMA-FVC-SIGLO
98. SMCEU-PTGWO v. Confesor

99. International School Alliance of Educators v. Quisumbing


100. National Association of Free Trade Unions v. Mainit Lumber
Development Company Workers Union
Article 256
101. Picop Resources Inc v. Dequilla et al
102. National Union of Workers in Hotel, Restaurants and Allied
Industries-Manila Pavillon Chapter v. Sec of Labor
Article 260-261
103. Santuyo et al v. Ramerco Garmets Manufacturing Inc
104. Teng v. Pahagac
105. Samahan ng Mangagawa ng Hyatt (SAMASAH-NUWHRAIN)
v. Magsalin
Article 263-264
106. YSS Employees Union v. YSS Laboratories Inc
107. NUWHRAIN-APL-IUL Dusit Hotel Nikko Chapter v. CA
108. Jackbuilt Industries Inc., v. Jackbilt Employees Workers
Union-NAFLU-KMU
109. Airline Pilots Association of the Philippines v. PAL
110. Olisa et al v. Escario
111. Tabangao Shell Refinery Employees Association v. Pilipinas
Shell
112. Asia Brewery Inc v. TPMA
113. Escario et al v. NLRC
114. University of San Agustin Employees Union v. CA
115. Phil Diamond Hotel and Resort Inc v. Manila Diamond Hotel
Employees Union

3E Andaya Ching Espiritu Hefti Galvez Gammad Lainez Lui Madamba Nagera Narvasa
Ong Palangdao Rosales Sanchez Santos Satrain Tabo (2014-2013)

Labor Relations Case Digest - Atty. Joyrich Golangco

116. Sukhothai Cuisine and Restaurant v. CA


117. Biflex Phil Inc labor Union v. Filflex Industrial Manufacturing
Corporation
118. Sta Rosa Coca-cola Plant Employee Union v. Coca-Cola
Bottlers Phil Inc
119. Manila Hotel Employee Association v. Manila Hotel
Corporation
120. G&S Transport Corporation v. Infante
121. Steel Corporation of the Phils v. SCP employees Union
National Federation of Labor Unions
Article 277(b)
122. St. Lukes Medical Center Inc. v. Notario
123. Aliling v. World Express Corp
124. Perez v. PT&T
Article 279
125.
126.
127.
128.
129.
130.
131.
132.
133.
134.
135.
136.
137.
138.

Agabon v. NLRC
Jaka food Processing Corp v. Pacot
Culili v. Eastern Telecommunications Phils
Serrano v. Gallant Maritime
Yap v. Thenamaris Ship Management
Bank of Lucbao v. Manabat
St. Marys Academy v. Palacio et al
Toyota Motors Phils Corp Workers Association v. NLRC
Bristol Myers Squibb Inc v. Haban
Yrasuegui v. PAL
Dreamland Hotel Resort v. Johnson
Manila Water v. Del Rosario
Nacar v. Gallery Frames
Bani Rural Bank Inc v. De Guzman

139.
140.
141.
142.
143.

Universal Robina Corp v. Castillo


Baptista v. Villanueva
BPI employees Union Davao City v. BPI
Integrated Microelectronics v. Pionelles
Golden Ace Builders v. Talde

Article 280
144.
145.
146.
147.
148.
149.
150.
151.
152.
153.
154.
155.
156.
157.
158.
159.
160.

Lynvil Fishing Enterprises Inc v. Ariola


Consolidated Broadcasting Systems Inc v. Oberio
Orozco v. CA
William Uy Construction Corp v. Trinidad
DM Consunji Inc v. Jamin
Aro et al v. NLRC
Universal Robina Sugar Milling Corp v. Acibo et al
GMA Network Inc v. Pabriga
Pasos v. PNCC
Gapayao v. Fulo
Millennium Erectors Corp v. Magallanes
Caparoso et al v. CA
Sps Lim v. Legaspi Hope Christian School et al
DM Consunji v. Gobres et al
Mercado et al v. Ama Computer College
Brent School v. Zamora
Pure Foods Corporation v. NLRC

Management Prerogative
161. Zuellig Frieght and Cargo System v. NLRC
162. Peckson v. Robinsons Supermarket Corp
163. Gatbaton v. NLRC

3E Andaya Ching Espiritu Hefti Galvez Gammad Lainez Lui Madamba Nagera Narvasa
Ong Palangdao Rosales Sanchez Santos Satrain Tabo (2014-2013)

Article 281
164.
165.
166.
167.
168.
169.
170.
171.
172.

Labor Relations Case Digest - Atty. Joyrich Golangco

Tamsons Enterprise Inc. et al v. CA


Hacienda Primera Development Corporation v. Villegas
Universidad De Sta Isabel v. Sambajon
Univac Development v. Soriano
Abbott Laboratories v. Alcaraz
Colegio de Santisimo Rosario v. Rojo
Philippine Daily Inquirer v. Magtibay
Alcira v. NLRC
Mercado v. Ama Computer College

Article 282
173. Concepcion v. Minex Import Corporation
174. Grand Asian Shipping Lines Inc v. Galvez
175. Mirant Phil Corp v. Caro
176. Bluer than Blue Joint Ventures Co v. Esteban
177. Manila Jockey Club v. Trajano
178. Hormillosa v. Coca-cola
179. Chuanico v. Legacy COnsololidated Plans
180. Duncan Association of Detailman-PGTWO v. Glaxo Welcome
Phils
181. Starpaper Corporation et al v. Simbol
182. Ace Promotion and Marketing Corp v. Notario
183. The Coca-Cola Export Corporation v. Gacayan
184. Domingo v. Rayala
185. Phil Aeolus Auto Motive United Corp v. NLRC
186. Pharmacia and Upjohn Inc. v. Albayda Jr.
187. Jerusalem v. Hock et al
188. Reno Foods Inc. v. NLM-Katipunan
189. La rosa et al v. Ambassador Hotel
190. Maribago Resort v. Dual

191.
192.
193.
194.

Centrury Canning Corp v. Ramil


Tongko v. The Manfacturers Life Insurance Co. Inc
School of Holy Spirit of Quezon City v. Taguiam
John Handcock Life Insurance Co. v. Davis

Article 283
195.
196.
197.
198.
199.
200.

SPI Technologies Inc v. Mapua


Arabit v. Jardine Pacific Finance Inc
Phil Carpet Manufacturing Corp v. Tagyamon et al
Sanoh Fulton Phil v. Bernard
Andrada et al v. NLRC
Asufrin v. San Miguel Corporation

Article 284
201. Sy et al v. CA
202. Union Motors v. CA
203. Villaruel v. Yeo Han Guan
Article 285
204.
205.
206.
207.
208.
209.
210.
211.
212.

Malig-on v. Equitable General Servces Inc


Chang Kai Shek College v. Torres
Opilando v. Ravina
Willi Hann Enterprises v. Maghuyop
Skippers United Pacific et al v. Doza
Morales v. Harbour Centre Port Terminal Inc
SHS Perforated Material Inc v. Diaz
San Miguel Properties Phis v. Gucaban
BMG Records Phils Inc v. Aparecio

Article 286
213. SKM Art Craft Corporation v. Bauca

3E Andaya Ching Espiritu Hefti Galvez Gammad Lainez Lui Madamba Nagera Narvasa
Ong Palangdao Rosales Sanchez Santos Satrain Tabo (2014-2013)

Labor Relations Case Digest - Atty. Joyrich Golangco

214. Emeritus Security and Maintenance System Inc v. Dailig


215. Nippon Housing Phil Inc. et al v. Leynes
216. Mayon Hotel and Restaurant et al v. Adana et al

234. Fernandez et al v. New Field Staff Solution


Change of Equity Composition of Corporations

Article 287
217.
218.
219.
220.
221.
222.
223.
224.
225.

Corporate Liabilities

Serrano v. Severino Santos Transit


Daaboy v. Coca-Cola
Pinero v. NLRC
Sta Catalina College v. NLRC
Pantranco North Express Inc v. NLRC
R and E Transport v. Latag
Obusan v. PNB
Kimberly Clark Phils v. Dimayuga
Magdadaro v. PNB

235. SME Bank v. Peregrin

Article 291
226.
227.
228.
229.
230.

Serrano v. CA
IBC v. Panganiban
Accessories Specialist v. Albanza
Autobus Transport System v. Bautista
PLDT v. Pingol

Effect of Change of Ownership of Business


231. Panafrancia Tours and Travel Transport v. Sarmiento
Liability of Corporate Officers
232. Carag v. NLRC
Effect of Company Merger on Union Shop Clause
233. BPI v. BPI Employees Union Davao Chapter
3E Andaya Ching Espiritu Hefti Galvez Gammad Lainez Lui Madamba Nagera Narvasa
Ong Palangdao Rosales Sanchez Santos Satrain Tabo (2014-2013)

Labor Relations Case Digest - Atty. Joyrich Golangco


1: Francisco v NLRC
FACTS: In 1995, petitioner Angelina Francisco was hired by Kasei Corporation
during its incorporation stage. She was designated as Accountant and Corporate
Secretary and was assigned to handle all the accounting needs of the company.
She was also designated as Liaison Officer to the City of Makati to secure
business permits, construction permits and other licenses for the initial
operation of the company.
Although she was designated as Corporate Secretary, she was not
entrusted with the corporate documents; neither did she attend any board
meeting nor required to do so. She never prepared any legal document and never
represented the company as its Corporate Secretary. However, on some
occasions, she was prevailed upon to sign documentation for the company.
In 1996, petitioner was designated Acting Manager. The corporation also
hired Gerry Nino as accountant in lieu of petitioner. As Acting Manager,
petitioner was assigned to handle recruitment of all employees and perform
management administration functions; represent the company in all dealings
with government agencies, especially with the Bureau of Internal Revenue (BIR),
Social Security System (SSS) and in the city government of Makati; and to
administer all other matters pertaining to the operation of Kasei Restaurant
which is owned and operated by Kasei Corporation.
For five years, petitioner performed the duties of Acting Manager.
As of December 31, 2000 her salary was P27, 500.00 plus P3, 000.00 housing
allowance and a 10% share in the profit of Kasei Corporation.
In January 2001, petitioner was replaced by Liza R. Fuentes as
Manager. Petitioner alleged that she was required to sign a prepared resolution
for her replacement but she was assured that she would still be connected with
Kasei Corporation. Timoteo Acedo, the designated Treasurer, convened a
meeting of all employees of Kasei Corporation and announced that nothing had
changed and that petitioner was still connected with Kasei Corporation as
Technical Assistant to Seiji Kamura and in charge of all BIR matters.
Thereafter, Kasei Corporation reduced her salary by P2, 500.00 a
month beginning January up to September 2001 for a total reduction of
P22, 500.00 as of September 2001. Petitioner was not paid her mid-year
bonus allegedly because the company was not earning well. On October

2001, petitioner did not receive her salary from the company. She made
repeated follow-ups with the company cashier but she was advised that the
company was not earning well.
On October 15, 2001, petitioner asked for her salary from Acedo and the
rest of the officers but she was informed that she is no longer connected with
the company.
Since she was no longer paid her salary, petitioner did not report for work and
filed an action for constructive dismissal before the labor arbiter.

Argument of Private Respondent: Private respondents averred that petitioner


is not an employee of Kasei Corporation. They alleged that petitioner was hired
in 1995 as one of its technical consultants on accounting matters and act
concurrently as Corporate Secretary. As technical consultant, petitioner
performed her work at her own discretion without control and supervision
of Kasei Corporation. Petitioner had no daily time record and she came to the
office any time she wanted. The company never interfered with her work except
that from time to time; the management would ask her opinion on matters
relating to her profession. Petitioner did not go through the usual procedure of
selection of employees, but her services were engaged through a Board
Resolution designating her as technical consultant. The money received by
petitioner from the corporation was her professional fee subject to the 10%
expanded withholding tax on professionals, and that she was not one of those
reported to the BIR or SSS as one of the companys employees.
Petitioners designation as technical consultant depended solely upon
the will of management. As such, her consultancy may be terminated any time
considering that her services were only temporary in nature and dependent on
the needs of the corporation.
To prove that petitioner was not an employee of the corporation, private
respondents submitted a list of employees for the years 1999 and 2000 duly
received by the BIR showing that petitioner was not among the employees
reported to the BIR, as well as a list of payees subject to expanded withholding
tax which included petitioner. SSS records were also submitted showing that
petitioners latest employer was Seiji Corporation.
Labor Arbiter: Petitioner was illegally dismissed
NLRC: the decision of the Labor Arbiter was affirmed

3E Andaya Ching Espiritu Hefti Galvez Gammad Lainez Lui Madamba Nagera Narvasa
Ong Palangdao Rosales Sanchez Santos Satrain Tabo (2014-2015)

Labor Relations Case Digest - Atty. Joyrich Golangco

On appeal, the Court of Appeals reversed and set aside the decision of the NLRC.
The subsequent motion for reconsideration was also denied, hence, the present
recourse.

ISSUES: (1) whether there was an employer-employee relationship between


petitioner and private respondent Kasei Corporation; and if in the affirmative
(2) Whether petitioner was illegally dismissed.

HELD: (1) YES. By applying the control test, there is no doubt that petitioner is
an employee of Kasei Corporation because she was under the direct control and
supervision of Seiji Kamura, the corporations Technical Consultant. She reported
for work regularly and served in various capacities as Accountant, Liaison
Officer, Technical Consultant, Acting Manager and Corporate Secretary, with
substantially the same job functions, that is, rendering accounting and tax
services to the company and performing functions necessary and desirable for
the proper operation of the corporation such as securing business permits and
other licenses over an indefinite period of engagement.
Under the broader economic reality test, the petitioner can likewise be said to
be an employee of respondent corporation because she had served the company
for six years before her dismissal, receiving check vouchers indicating her
salaries/wages, benefits, 13th month pay, bonuses and allowances, as well as
deductions and Social Security contributions from August 1, 1999 to December
18, 2000. When petitioner was designated General Manager, respondent
corporation made a report to the SSS signed by Irene Ballesteros. Petitioners
membership in the SSS as manifested by a copy of the SSS specimen signature
card which was signed by the President of Kasei Corporation and the inclusion of
her name in the on-line inquiry system of the SSS evinces the existence of an
employer-employee relationship between petitioner and Respondent
Corporation. It is therefore apparent that petitioner is economically
dependent on Respondent Corporation for her continued employment in
the latters line of business.
Petitioner is an employee of respondent Kasei Corporation. She was
selected and engaged by the company for compensation, and is economically
dependent upon respondent for her continued employment in that line of
business. Her main job function involved accounting and tax services rendered to
Respondent Corporation on a regular basis over an indefinite period of

engagement. Respondent Corporation hired and engaged petitioner for


compensation, with the power to dismiss her for cause. More importantly,
Respondent Corporation had the power to control petitioner with the
means and methods by which the work is to be accomplished.
(2) YES. The corporation constructively dismissed petitioner when it
reduced her salary by P2, 500 a month from January to September 2001. This
amounts to an illegal termination of employment, where the petitioner is entitled
to full back wages. Since the position of petitioner as accountant is one of trust
and confidence, and under the principle of strained relations, petitioner is
further entitled to separation pay, in lieu of reinstatement.
2: Sonza vs ABS CBN
FACTS: In May 1994, respondent ABS-CBN Broadcasting Corporation (ABSCBN) signed an Agreement with the Mel and Jay Management and Development
Corporation (MJMDC). ABS-CBN was represented by its corporate officers
while MJMDC was represented by SONZA, as President and General Manager, and
Carmela Tiangco (TIANGCO), as EVP and Treasurer. Referred to in the
Agreement as AGENT, MJMDC agreed to provide SONZAs services exclusively
to ABS-CBN as talent for radio and television.
ABS-CBN agreed to pay for SONZAs services a monthly talent fee
of P310, 000 for the first year andP317, 000 for the second and third year
of the Agreement. ABS-CBN would pay the talent fees on the 10thand
25th days of the month.

On 1 April 1996, SONZA wrote a letter to ABS-CBNs President, Eugenio


Lopez III, stating in the letter that Mr. Sonza irrevocably resigned in view of
recent events** concerning his programs and career; that these were due to acts
of the station in violation and breach of their agreement; that the letter served as
notice of rescission of said agreement; and that he is waiving and renouncing
recovery of the remaining amount stipulated in paragraph 7 of the Agreement
but reserves the right to seek recovery of the other benefits under said
Agreement.

3E Andaya Ching Espiritu Hefti Galvez Gammad Lainez Lui Madamba Nagera Narvasa
Ong Palangdao Rosales Sanchez Santos Satrain Tabo (2014-2015)

Labor Relations Case Digest - Atty. Joyrich Golangco

**DIGESTERS NOTE: later on in the case, this apparently refers to


the cancellation of his programs.

On 30 April 1996, SONZA filed a complaint against ABS-CBN before the


Department of Labor and Employment, National Capital Region in Quezon
City. SONZA complained that ABS-CBN did not pay his salaries, separation pay,
service incentive leave pay, 13th month pay, signing bonus, travel allowance and
amounts due under the Employees Stock Option Plan (ESOP).
On 10 July 1996, ABS-CBN filed a Motion to dismiss on the ground that
no employer-employee relationship existed between the parties, to which SONZA
filed an opposition to the motion.
LA: The Labor Arbiter denied the motion to dismiss. The Labor Arbiter ruled:

In this instant case, complainant for having invoked a claim that he was
an employee of respondent company until April 15, 1996 and that he was not
paid certain claims, it is sufficient enough as to confer jurisdiction over the
instant case in this Office. And as to whether or not such claim would entitle
complainant to recover upon the causes of action asserted is a matter to be
resolved only after and as a result of a hearing. Thus, the respondents plea of
lack of employer-employee relationship may be pleaded only as a matter of
defense. It behooves upon it the duty to prove that there really is no employeremployee relationship between it and the complainant.
The Labor Arbiter then considered the case submitted for resolution.
The parties submitted their position papers on 24 February 1997. The Labor
Arbiter rendered his Decision dated 8 July 1997 dismissing the complaint for
lack of jurisdiction:

It must be noted that complainant was engaged by respondent by


reason of his peculiar skills and talent as a TV host and a radio
broadcaster. Unlike an ordinary employee, he was free to perform the services
he undertook to render in accordance with his own style Whatever benefits
complainant enjoyed arose from specific agreement by the parties and not by
reason of employer-employee relationship The fact that complainant was made
subject to respondents Rules and Regulations, likewise, does not detract from
the absence of employer-employee relationship.

NLRC: Affirmed the LAs decision of lack of jurisdiction. Motion for


reconsideration denied.

MJMDC is an agent of SONZA, not a mere labor-only contractor of ABSCBN such that there exists an employer-employee relationship between the
latter and SONZA. Jurisdiction over the instant controversy belongs to the
regular courts, the same being in the nature of an action for alleged breach of
contractual obligation on the part of respondent-appellee. The compensation and
bonuses for Mr. Sonzas services are not based on the Labor Code but rather on
the provisions of their agreement.
CA: Petitioner filed a special civil action for certiorari, to which the CA dismissed
the case.

The Court of Appeals affirmed the NLRCs finding that no employeremployee relationship existed between SONZA and ABS-CBN.
The Court of Appeals ruled that the existence of an employer-employee
relationship between SONZA and ABS-CBN is a factual question that is within the
jurisdiction of the NLRC to resolve. A special civil action for certiorari extends
only to issues of want or excess of jurisdiction of the NLRC. Such action cannot
cover an inquiry into the correctness of the evaluation of the evidence which
served as basis of the NLRCs conclusion.

ISSUES: Whether the Court of Appeals gravely erred in affirming the NLRCs
decision and refusing to find that an Employer-Employee relationship existed
between SONZA and ABS-CBN.

HELD: NO, the Court of Appeals did not. No convincing reason exists to
warrant a reversal of the decision of the Court of Appeals affirming the NLRC
ruling which upheld the Labor Arbiters dismissal of the case for lack of
jurisdiction.

SONZA maintains that all essential elements of an employer-employee


relationship are present in this case. Case law has consistently held that the
elements of an employer-employee relationship are: (a) the selection and
engagement of the employee; (b) the payment of wages; (c) the power of
dismissal; and (d) the employers power to control the employee on the means

3E Andaya Ching Espiritu Hefti Galvez Gammad Lainez Lui Madamba Nagera Narvasa
Ong Palangdao Rosales Sanchez Santos Satrain Tabo (2014-2015)

Labor Relations Case Digest - Atty. Joyrich Golangco

and methods by which the work is accomplished.[18]The last element, the socalled control test, is the most important element.

(a) On Selection: ABS-CBN engaged SONZAs services to co-host its

television and radio programs because of SONZAs peculiar


skills, talent and celebrity status. The specific selection and hiring
of SONZA, because of his unique skills, talent and celebrity
status not possessed by ordinary employees, is a circumstance
indicative, but not conclusive, of an independent contractual
relationship.

(b) On wages: All the talent fees and benefits paid to SONZA were the
result of negotiations that led to the Agreement. Whatever benefits
SONZA enjoyed arose from contract and not because of an
employer-employee relationship.

(c) On power of dismissal: For violation of any provision of the

Agreement,
either
party
may terminate
their
relationship. SONZA failed to show that ABS-CBN could terminate
his services on grounds other than breach of contract, such as
retrenchment to prevent losses as provided under labor laws.

(d) On control: Applying the control test to the present case, we find

that SONZA is not an employee but an independent contractor. ABSCBN was not involved in the actual performance that produced the
finished product of SONZAs work. ABS-CBN did not instruct SONZA
how to perform his job. ABS-CBN merely reserved the right to
modify the program format and airtime schedule for more effective
programming. ABS-CBNs sole concern was the quality of the shows
and their standing in the ratings. Clearly, ABS-CBN did not exercise
control over the means and methods of performance of SONZAs
work.

Although ABS-CBN did have the option not to broadcast


SONZAs show, ABS-CBN was still obligated to pay SONZAs talent
fees. Thus, even if ABS-CBN was completely dissatisfied with the
means and methods of SONZAs performance of his work, or even

with the quality or product of his work, ABS-CBN could not dismiss
or even discipline SONZA. All that ABS-CBN could do is not to
broadcast SONZAs show but ABS-CBN must still pay his talent fees
in full.

The present case does not call for an application of the Labor Code
provisions but an interpretation and implementation of the May 1994
Agreement. In effect, SONZAs cause of action is for breach of contract which
is intrinsically a civil dispute cognizable by the regular courts.
3: Javier vs Flyace Corp
Facts: On May 23, 2008, Javier filed a complaint before the NLRC for
underpayment of salaries and other labor standard benefits. He alleged that he
was an employee of Fly Ace since September 2007, performing various tasks at
the respondents warehouse such as cleaning and arranging the canned items
before their delivery to certain locations, except in instances when he would be
ordered to accompany the companys delivery vehicles, as pahinante; that during
his employment, he was not issued an identification card and pay slips by the
company; that on May 6, 2008, he reported for work but he was no longer
allowed to enter the company premises by the security guard upon the
instruction of Ruben Ong (Mr. Ong), his superior; that after several minutes
of begging to the guard to allow him to enter, he saw Ong whom he approached
and asked why he was being barred from entering the premises; that Ong replied
by saying, Tanungin mo anak mo; that he then went home and discussed the
matter with his family; that he discovered that Ong had been courting his
daughter Annalyn after the two met at a fiesta celebration in Malabon City; that
Annalyn tried to talk to Ong and convince him to spare her father from trouble
but he refused to accede; that thereafter, Javier was terminated from his
employment without notice; and that he was neither given the opportunity
to refute the cause/s of his dismissal from work.
To support his allegations, Javier presented an affidavit of one Bengie
Valenzuela who alleged that Javier was a stevedore or pahinante of Fly
Ace from September 2007 to January 2008. The said affidavit was subscribed
before the Labor Arbiter

3E Andaya Ching Espiritu Hefti Galvez Gammad Lainez Lui Madamba Nagera Narvasa
Ong Palangdao Rosales Sanchez Santos Satrain Tabo (2014-2015)

Labor Relations Case Digest - Atty. Joyrich Golangco

For its part, Fly Ace averred that it was engaged in the business of
importation and sales of groceries. Sometime in December 2007, Javier was
contracted by its employee, Mr. Ong, as extra helper on a pakyaw basis at an
agreed rate of 300.00 per trip, which was later increased to 325.00 in January
2008. Mr. Ong contracted Javier roughly 5 to 6 times only in a month whenever
the vehicle of its contracted hauler, Milmar Hauling Services, was not
available. On April 30, 2008, Fly Ace no longer needed the services of
Javier. Denying that he was their employee, Fly Ace insisted that there was
no illegal dismissal. Fly Ace submitted a copy of its agreement with Milmar
Hauling Services and copies of acknowledgment receipts evidencing payment to
Javier for his contracted services bearing the words, daily
manpower (pakyaw/piece rate pay) and the latters signatures/initials.
LA: the LA dismissed the complaint for lack of merit on the ground that Javier
failed to present proof that he was a regular employee of Fly Ace.

NLRC: LA Decision was reversed. On appeal with the NLRC, Javier was favored.
It ruled that the LA skirted the argument of Javier and immediately concluded
that he was not a regular employee simply because he failed to present proof. It
was of the view that a pakyaw-basis arrangement did not preclude the existence
of employer-employee relationship.
The NLRC held that substantial evidence was sufficient basis for
judgment on the existence of the employer-employee relationship. Finding
Javier to be a regular employee, the NLRC ruled that he was entitled to a security
of tenure.

CA: the CA annulled the NLRC findings that Javier was indeed a former
employee of Fly Ace and reinstated the dismissal of Javiers complaint as ordered
by the LA.
In an illegal dismissal case the onus probandi rests on the employer to
prove that its dismissal was for a valid cause. However, before a case for illegal
dismissal can prosper, an employer-employee relationship must first be
established. x x x it is incumbent upon private respondent to prove the
employee-employer relationship by substantial evidence.

Since no substantial evidence was presented to establish an employeremployee relationship, the case for illegal dismissal could not prosper.
ISSUES: Whether or not Petitioner was an employee of Fly Ace.

HELD: No, he was not an employee. The Court affirms the assailed CA decision.

As the records bear out, the LA and the CA found Javiers claim of
employment with Fly Ace as wanting and deficient. The Court is constrained
to agree. The petitioner needs to show by substantial evidence that he was
indeed an employee of the company against which he claims illegal dismissal.

Although substantial evidence is not a function of quantity but rather of


quality, the circumstances of the instant case demand that something more
should have been proffered. Had there been other proofs of employment,
such as inclusion in petitioners payroll, or a clear exercise of control, the
Court would have affirmed the finding of employer-employee relationship.
In this case, the LA and the CA both concluded that Javier failed to
establish his employment with Fly Ace. By way of evidence on this point, all that
Javier presented were his self-serving statements purportedly showing his
activities as an employee of Fly Ace. Clearly, Javier failed to pass the
substantiality requirement to support his claim. Hence, the Court sees no
reason to depart from the findings of the CA.

Javier was not able to persuade the Court that the elements of the
four-fold test exist in his case. He could not submit competent proof that Fly
Ace engaged his services as a regular employee; that Fly Ace paid his wages as an
employee, or that Fly Ace could dictate what his conduct should be while at
work. In other words, Javiers allegations did not establish that his relationship
with Fly Ace had the attributes of an employer-employee relationship on the
basis of the four-fold test.

4, SMCEU vs Judge Bersamira (of Pasig RTC)

3E Andaya Ching Espiritu Hefti Galvez Gammad Lainez Lui Madamba Nagera Narvasa
Ong Palangdao Rosales Sanchez Santos Satrain Tabo (2014-2015)

Labor Relations Case Digest - Atty. Joyrich Golangco

FACTS: Sometime in 1983 and 1984, SanMig entered into contracts for
merchandising services with Lipercon and D'Rite. These companies are
independent contractors duly licensed by the Department of Labor and
Employment (DOLE). In said contracts, it was expressly understood and agreed
that the workers employed by the contractors were to be paid by the latter and
that none of them were to be deemed employees or agents of SanMig. There was
to be no employer-employee relation between the contractors and/or its
workers, on the one hand, and SanMig on the other.

Petitioner San Miguel Corporation Employees Union-PTWGO


(the Union, for brevity) is the duly authorized representative of the monthly paid
rank-and-file employees of SanMig with whom the latter executed a Collective
Bargaining Agreement (CBA) effective 1 July 1986 to 30 June 1989. Section 1 of
their CBA specifically provides that "temporary, probationary, or contract
employees and workers are excluded from the bargaining unit and, therefore,
outside the scope of this Agreement."
In a letter, dated 20 November 1988, the Union advised SanMig
that some Lipercon and D'Rite workers had signed up for union membership and
sought the regularization of their employment with SMC. The Union alleged that
this group of employees, while appearing to be contractual workers supposedly
independent contractors, have been continuously working for SanMig for a
period ranging from six (6) months to fifteen (15) years and that their work is
neither casual nor seasonal as they are performing work or activities necessary
or desirable in the usual business or trade of SanMig. Thus, it was contended that
there exists a "labor-only" contracting situation. It was then demanded that the
employment status of these workers be regularized.

The Union filed two notices to strike and several conciliation


conferences were held to settle the dispute before the National Conciliation and
Mediation Board (NCMB) of DOLE. Beginning 14 February 1989 until 2 March
1989, series of pickets were staged by Lipercon and D'Rite workers in various
SMC plants and offices.
On 6 March 1989, SMC filed a verified Complaint for Injunction and
Damages before respondent Court to enjoin the Union from staging the strikes.

Respondent Court found the Complaint sufficient in form and substance


and issued a Temporary Restraining Order for the purpose of maintaining the
status quo, and set the application for Injunction for hearing.
In the meantime, the Union filed a Motion to Dismiss SanMig's Complaint
on the ground of lack of jurisdiction over the case/nature of the action, which
motion was opposed by SanMig. That Motion was denied by respondent Judge.

After several hearings on SanMig's application for injunctive relief,


respondent Court issued the questioned Order granting the application and
enjoining the Union from committing the acts complained of. Accordingly,
respondent Court issued the corresponding Writ of Preliminary Injunction. The
respondent court rationalized that the absence of an employer-employee
relationship negates the existence of labor dispute. Verily, this court (RTC) has
jurisdiction to take cognizance of plaintiff's grievance.

ISSUES: Whether, or not the case at bar involves, or is in connection with, or


relates to a labor dispute.

An affirmative answer would bring the case within the original and
exclusive jurisdiction of labor tribunals to the exclusion of the regular Courts.

HELD: YES, it does. That a labor dispute, as defined by the law in Article 212 (1)
of the Labor Code, does exist herein is evident. At bottom, what the Union seeks
is to regularize the status of the employees contracted by Lipercon and D'Rite in
effect, that they be absorbed into the working unit of SanMig. This matter
definitely dwells on the working relationship between said employees vis-a-vis
SanMig. Terms, tenure and conditions of their employment and the arrangement
of those terms are thus involved bringing the matter within the purview of a
labor dispute. Further, the Union also seeks to represent those workers, who
have signed up for Union membership, for the purpose of collective bargaining.
SanMig, for its part, resists that Union demand on the ground that there is no
employer-employee relationship between it and those workers and because the
demand violates the terms of their CBA. Obvious then is that representation and
association, for the purpose of negotiating the conditions of employment are also
involved. In fact, the injunction sought by SanMig was precisely also to prevent
such representation. Again, the matter of representation falls within the scope of

3E Andaya Ching Espiritu Hefti Galvez Gammad Lainez Lui Madamba Nagera Narvasa
Ong Palangdao Rosales Sanchez Santos Satrain Tabo (2014-2015)

Labor Relations Case Digest - Atty. Joyrich Golangco

a labor dispute. Neither can it be denied that the controversy below is directly
connected with the labor dispute already taken cognizance of by the NCMB-DOLE

The issues between the parties union demands; labor-only


contracting of Lipercon and Drite; the Union representing workers from
Lipercon and Drite are issues the resolution of which calls for the application
of labor laws.

As the case is indisputably linked with a labor dispute,


jurisdiction belongs to the labor tribunals. As explicitly provided for in Article
217 of the Labor Code, prior to its amendment by R.A. No. 6715 on 21 March
1989, since the suit below was instituted on 6 March 1989, Labor Arbiters have
original and exclusive jurisdiction to hear and decide the following cases
involving all workers including "1. Unfair labor practice cases; 2. Those that
workers may file involving wages, hours of work and other terms and conditions
of employment; ... and 5. Cases arising from any violation of Article 265 of this
Code, including questions involving the legality of strikes and lockouts. ..." Article
217 lays down the plain command of the law.
5. Locsin et. al. v. PLDT, October 2, 2009

Facts: On November 1, 1990, respondent PLDT and the Security and Safety
Corporation of the
Philippines (SSCP) entered into a Security Services Agreement whereby SSCP
would provide armed security guards to PLDT to be assigned to its various
offices. Petitioners Raul Locsin and Eddie Tomaquin, among other security
guards, were posted at a PLDT office. On August 30, 2001, respondent issued a
Letter terminating the Agreement effective October 1, 2001. Despite the
termination of the Agreement, however, petitioners continued to secure the
premises of their assigned office. They were allegedly directed to remain at their
post by representatives of respondent. In support of their contention, petitioners
provided the Labor Arbiter with copies of petitioner Locsin's pay slips for the
period of January to September 2002. Then, on September 30, 2002, their
services were terminated.
Petitioners filed a complaint before the Labor Arbiter for illegal dismissal and
recovery of money claims such as overtime pay, holiday pay, and premium pay

for holiday and rest day, service incentive leave pay, Emergency Cost of Living
Allowance, and moral and exemplary damages against PLDT. The Labor Arbiter
rendered a Decision finding PLDT liable for illegal dismissal. It held that
petitioners were employees of PLDT and not of SSCP for petitioners continued to
serve as guards of PLDT's offices. As such employees, they were entitled to
substantive and procedural due process before termination of employment.
PLDT appealed to NLRC which rendered a Resolution affirming in toto the
Arbiter's Decision. Thus, PDLT filed a Motion for Reconsideration of the NLRC's
Resolution which was also denied.

Hence, PLDT filed a Petition for Certiorari with the CA which rendered the
assailed decision granting PLDT's petition and dismissing petitioners' complaint.
The CA applied the fourfold
test in order to determine the existence of an employer employee relationship
between the parties but did not find such relationship. It determined that SSCP
was not a labor only
contractor and was an independent contractor having substantial capital to
operate and conduct its own business. The CA further bolstered its decision by
citing the Agreement whereby it was stipulated that there shall be no employer
employee relationship between the security guards and PLDT. Anent the pay
slips that were presented by petitioners, the CA noted that those were issued by
SSCP and not PLDT hence, SSCP continued to pay the salaries of petitioners after
the Agreement. This fact allegedly proved that petitioners continued to be
employees of SSCP albeit performing their work at PLDT's premises. Hence, this
petition.
ISSUES: Whether petitioners became employees of respondent after the
Agreement between SSCP and respondent was terminated. (yes)

Held: The SC held that there was no employer employee relationship between
the parties from the time of petitioners' first assignment to respondent by SSCP
in 1988 until the alleged termination of the Agreement between respondent and
SSCP. The petitioners were among those declared to be employees of their
respective security agencies and not of PLDT. However, the petitioners became

3E Andaya Ching Espiritu Hefti Galvez Gammad Lainez Lui Madamba Nagera Narvasa
Ong Palangdao Rosales Sanchez Santos Satrain Tabo (2014-2015)

Labor Relations Case Digest - Atty. Joyrich Golangco

the employees of respondent after the agreement between SSCP and respondent
was terminated.

While respondent and SSCP no longer had any legal relationship with the
termination of the Agreement, petitioners remained at their post securing the
premises of respondent while receiving their salaries, allegedly from SSCP.
Clearly, such a situation makes no sense, and the denials proffered by respondent
do not shed any light to the situation. It is but reasonable to conclude that, with
the directive of respondent, petitioners continued with their services. Evidently,
such are indicia of control that respondent exercised over petitioners. Such
power of control has been explained as the "right to control not only the end to
be achieved but also the means to be used in reaching such end." With the
conclusion that respondent directed petitioners to remain at their posts and
continue with their duties, it is clear that respondent exercised the power of
control over them thus, the existence of an employer employee relationship.

Evidently, respondent having the power of control over petitioners must


be considered as petitioners employerfrom the termination of the Agreement
onwardsas this was the only time that any evidence of control was exhibited
by respondent over petitioners. Thus, as aptly declared by the NLRC, petitioners
were entitled to the rights and benefits of employees of respondent, including
due process requirements in the termination of their services. Both the Labor
Arbiter and NLRC found that respondent did not observe such due process
requirements. Having failed to do so, respondent is guilty of illegal dismissal
6. PEOPLES BROADCASTING SERVICE vs SEC OF LABOR
GR NO. 179652 March 6, 2012

FACTS: The DOLE Regional Office No. VII conducted an inspection of Bombo
Radyos premises in response to Juezans money claims against the broadcasting
company, as a result an order for Bombo Radyo to rectify/restitute the labor
standards violations discovered during the inspection. Bombo Radyo failed to
make any rectification or restitution, prompting the DOLE to conduct a summary
investigation. Bombo Radyo reiterated its position, made during the inspection,
that Juezan was not its employee. Both parties submitted evidence to support
their respective positions.

DOLE Director Rodolfo M. Sabulao found Juezan to be an employee of


Bombo Radyo. Consequently, Director Sabulao ordered Bombo Radyo to pay
Juezan P203, 726.30 representing his demanded money claims. Bombo Radyo
moved for reconsideration and submitted additional evidence, but Director
Sabulao denied the motion. Bombo Radyo then appealed to the DOLE Secretary,
insisting that Juezan was not its employee as he was a drama talent hired on a
per drama basis. The Acting DOLE Secretary dismissed the appeal for nonperfection due to Bombo Radyos failure to put a cash or surety bond, as
required by Article 128(b) of the Labor Code.
Bombo Radyo went to the Court of Appeals (CA) through a petition for certiorari
under Rule 65 of the Rules of Court. But was dismissed for lack of merit, hence
this appeal. The Court found that there was no employer-employee relationship
between petitioner and private respondent. It was held that while the DOLE may
make a determination of the existence of an employer-employee relationship,
this function could not be co-extensive with the visitorial and enforcement
power provided in Art. 128(b) of the Labor Code, as amended by RA 7730. The
NLRC was held to be the primary agency in determining the existence of an
employer-employee relationship. This was the interpretation of the Court of the
clause in cases where the relationship of employer-employee still exists in Art.
128(b).
ISSUES: Whether the DOLE can determine existing EE-ER relationship.

HELD: YES, No limitation in the law was placed upon the power of the DOLE to
determine the existence of an employer-employee relationship. No procedure
was laid down where the DOLE would only make a preliminary finding, that the
power was primarily held by the NLRC. The law did not say that the DOLE would
first seek the NLRCs determination of the existence of an employer-employee
relationship, or that should the existence of the employer-employee relationship
be disputed, the DOLE would refer the matter to the NLRC. The DOLE must have
the power to determine whether or not an employer-employee relationship
exists, and from there to decide whether or not to issue compliance orders in
accordance with Art. 128(b) of the Labor Code, as amended by RA 7730.
The DOLE, in determining the existence of an employer-employee
relationship, has a ready set of guidelines to follow, the same guide the courts

3E Andaya Ching Espiritu Hefti Galvez Gammad Lainez Lui Madamba Nagera Narvasa
Ong Palangdao Rosales Sanchez Santos Satrain Tabo (2014-2015)

Labor Relations Case Digest - Atty. Joyrich Golangco

themselves use. The elements to determine the existence of an employment


relationship are: (1) the selection and engagement of the employee; (2) the
payment of wages; (3) the power of dismissal; (4) the employers power to
control the employees conduct. The use of this test is not solely limited to the
NLRC. The DOLE Secretary, or his or her representatives, can utilize the same
test, even in the course of inspection, making use of the same evidence that
would have been presented before the NLRC.
The present Resolution now recognizes that the determination of the
existence of an employer-employee relationship by the DOLE, in the exercise of
its visitorial and enforcement power under Article 128(b) of the Labor Code, is
entitled to full respect and must be fully supported. This means that the DOLE
has the full power to determine the existence of an employer-employee
relationship in cases brought to it under Article 128(b) of the Labor Code. This
power is parallel and not subordinate to that of the NLRC.
The Court, at the same time, confirms its previous finding that no
employer-employee relationship exists between Juezan and Bombo Radyo based
on the evidence presented, and that a Deed of Assignment of Bank Deposits can
be a substitute for a cash or surety bond in perfecting an appeal to the Labor
Secretary.
7. Ymbong vs. ABS-CBN

Facts: Petitioner Ernesto G. Ymbong started working for ABS-CBN Broadcasting


Corporation (ABS-CBN) in 1993 at its regional station in Cebu as a television
talent, co-anchoring Hoy Gising and TV Patrol Cebu. His stint in ABS-CBN later
extended to radio when ABS-CBN Cebu launched its AM station DYAB in 1995
where he worked as drama and voice talent, spinner, scriptwriter and public
affairs program anchor. Like Ymbong, Leandro Patalinghug also worked for ABSCBN Cebu. Starting 1995, he worked as talent, director and scriptwriter for
various radio programs aired over DYAB. On January 1, 1996, the ABS-CBN Head
Office in Manila issued Policy No. HR-ER-016 or the Policy on Employees
Seeking Public Office. Under this policy, employees who will be seeking public
office must file a letter of resignation, while those who will be joining a political
party or actively campaign for a candidate must file a request for leave of
absence subject to the managements approval. Because of the impending May

1998 elections and based on his immediate recollection of the policy at that time,
Dante Luzon, Assistant Station Manager of DYAB issued a memorandum stating
that employees who want to run for office should file for a leave of absence and
his services will be temporarily suspended during the campaign period. Luzon,
however, admitted that upon double-checking of the exact text of the policy and
subsequent confirmation with the ABS-CBN Head Office, he saw that the policy
actually required suspension for those who intend to campaign for a political
party or candidate and resignation for those who will actually run in the
elections. Ymbong informed Luzon that he will be taking a leave of absence to
campaign for the administration ticket, however it was found out after the
elections that Ymbong actually ran for councilor of Lapu-lapu city. Patalinghug ,
on the other hand tendered his resignation for he will be running as councilor at
Naga, Cebu. Both Ymbong and Patalinghug lost in the elections. They were not
allowed to come back to work for respondent, but because of liberality, they
were given a chance to wind up their participation in a radio drama entitle
Nagbabagang Langit. Both then filed an illegal dismissal complaint against
respondent. Respondent prayed for dismissal of the complaints claiming that
there is no E-E relationship since both are mere talents.
LA found the dismissal illegal and ordered the reinstatement of respondents as
well as the payment of back wages. It also declared that there exists an E-E
relationship between the parties. The Labor Arbiter noted particularly that the
appointment letters/talent contracts imposed conditions in the performance of
their work, specifically on attendance and punctuality, which effectively placed
them under the control of ABS-CBN.

NLRC modified the decision. In the case of Patalinghug, it found that he


voluntarily resigned from employment on April 21, 1998 when he submitted his
resignation letter. The NLRC noted that although the tenor of the resignation
letter is somewhat involuntary, he knew that it is the policy of the company that
every person connected therewith should resign from his employment if he
seeks an elected position in the government. As to Ymbong, however, the NLRC
ruled otherwise.
Issues: a. Whether or not there exists E-E relationship between the parties
(discussed by the CA, but not by the SC)

3E Andaya Ching Espiritu Hefti Galvez Gammad Lainez Lui Madamba Nagera Narvasa
Ong Palangdao Rosales Sanchez Santos Satrain Tabo (2014-2015)

Labor Relations Case Digest - Atty. Joyrich Golangco

b. Whether or not Ymbong was illegally dismissed

HELD: a) CA rendered the assailed decision reversing and setting aside


the March 8, 2004 Decision and June 21, 2004 Resolution of the NLRC. The CA
declared Ymbong resigned from employment and not to have been illegally
dismissed. The award of full back wages in his favor was deleted
accordingly. The CA ruled that ABS-CBN is estopped from claiming that Ymbong
was not its employee after applying the provisions of Policy No. HR-ER-016 to
him. It noted that said policy is entitled Policy on Employees Seeking Public
Office and the guidelines contained therein specifically pertain to employees
and did not even mention talents or independent contractors. It held that it is a
complete turnaround on ABS-CBNs part to later argue that Ymbong is only a
radio talent or independent contractor and not its employee. By applying the
subject company policy on Ymbong, ABS-CBN had explicitly recognized him to be
an employee and not merely an independent contractor.

b) We find no merit in Ymbongs argument that [his] automatic termination x x


x was a blatant [disregard] of [his] right to due process as he was never asked
to explain why he did not tender his resignation before he ran for public office as
mandated by [the subject company policy]. Ymbongs overt act of running for
councilor of Lapu-Lapu City is tantamount to resignation on his part. He was
separated from ABS-CBN not because he was dismissed but because he
resigned. Since there was no termination to speak of, the requirement of due
process in dismissal cases cannot be applied to Ymbong. Thus, ABS-CBN is not
duty-bound to ask him to explain why he did not tender his resignation before he
ran for public office as mandated by the subject company policy.
8: Professional Services Inc. vs. CA

FACTS:

Natividad Agana was admitted at Medical City because of difficulty of


bowel movement and bloody anal discharge. Dr. Ampil diagnosed her to
be suffering from "cancer of the sigmoid."
Dr. Ampil, assisted by the medical staff of Medical City, performed
surgery upon her. During the surgery, he found that the malignancy in
her sigmoid area had spread to her left ovary, necessitating the removal

of certain portions of it. Thus, Dr. Ampil obtained the consent of


Natividads husband, to permit Dr. Fuentes to perform another
operation.
However, the operation appeared to be flawed. After a couple of days,
Natividad complained of excruciating pain in her anal region. They told
her that the pain was the natural consequence of the surgical operation
performed upon her.
Natividad and her husband went to the United States to seek further
treatment of her cancerous nodes. After 4 months of consultations and
laboratory examinations, Natividad was told that she was free of cancer.
Still suffering from pains. 2 weeks after returning to the Philippines, her
daughter found a piece of gauze protruding from her vagina. Dr. Ampil
was immediately informed. He proceeded to Natividads house where he
managed to extract by hand a piece of gauze measuring 1.5 inches in
width. Dr. Ampil then assured Natividad that the pains would soon
vanish.
Despite that, the pains intensified, prompting Natividad to seek
treatment at another hospital. While confined thereat, Dr. Gutierrez
detected the presence of a foreign object in her vagina -- a foul-smelling
gauze measuring 1.5 inches in width. The gauze had badly infected her
vaginal vault, which forced stool to excrete through the vagina.
Natividad and her husband filed with the Regional Trial Court a
complaint for damages against PSI (owner of Medical City), Dr. Ampil
and Dr. Fuentes.
Pending the outcome of the above case, Natividad died. She was duly
substituted by her children (the Aganas).
RTC: PSI, Dr. Ampil and Dr. Fuentes jointly and severally liable.
CA: affirmed the RTC judgment but dismissed the complaint against Dr.
Fuentes.
PSI, Dr. Ampil and the Aganas filed with SC separate petitions for review
on certiorari.
SC (First Division): PSI is jointly and severally liable with Dr. Ampil for
the following reasons:
1. There is an employer-employee relationship between Medical
City and Dr. Ampil.
2. PSIs act of publicly displaying in the lobby of the Medical City

3E Andaya Ching Espiritu Hefti Galvez Gammad Lainez Lui Madamba Nagera Narvasa
Ong Palangdao Rosales Sanchez Santos Satrain Tabo (2014-2015)

Labor Relations Case Digest - Atty. Joyrich Golangco

the names and specializations of its accredited physicians,


including Dr. Ampil, estopped it from denying the existence of
an employer-employee relationship between them under the
doctrine of ostensible agency or agency by estoppel
3. PSIs failed to supervise Dr. Ampil to take an active step in order
to remedy their negligence rendered it directly liable under the
doctrine of corporate negligence.
PSIs contention: 1) there is no employer-employee relationship
between it and its consultant, Dr. Ampil. 2) the doctrine of ostensible
agency or agency by estoppel cannot apply because spouses Agana
failed to establish that Natividad relied on the representation of the
hospital in engaging the services of Dr. 3) PSI maintains that the
doctrine of corporate negligence is misplaced because the proximate
cause of Natividads injury was Dr. Ampils negligence

ISSUES: Whether or not there exists an employee-employer relationship, thus


making PSI jointly and severally liable.
HELD: Yes An employer-employee relationship "in effect" exists between the
Medical City and Dr. Ampil. Consequently, both are jointly and severally liable to
the Agana.
In the SC decision in Ramos vs. CA

Hospitals exercise significant control in the hiring and firing of


consultants and in the conduct of their work within the hospital
premises. Doctors who apply for "consultant" slots, visiting or
attending, are required to submit proof of completion of residency, their
educational qualifications; generally, evidence of accreditation by the
appropriate board, evidence of fellowship in most cases, and references.
These requirements are carefully scrutinized by members of the
hospital administration or by a review committee set up by the
hospital who either accept or reject the application. This is
particularly true with respondent hospital.
After a physician is accepted, either as a visiting or attending
consultant, he is still normally required to accomplish more tasks.
Further, physicians performance as a specialist is generally

evaluated by a peer review committee. A consultant remiss in his


duties, or a consultant who regularly falls short of the minimum
standards acceptable to the hospital or its peer review committee,
is normally politely terminated.
In other words, private hospitals hire fire and exercise real control
over their attending and visiting "consultant" staff. While
"consultants" are not, technically employees, the control exercised,
the hiring, and the rights to terminate consultants all fulfill the
important hallmarks of an employer-employee relationship, with
the exception of the payment of wages. In assessing whether such a
relationship in fact exists, the control test is determining.
Accordingly, on the basis of the foregoing, for the purpose of
allocating responsibility in medical negligence cases, an employeremployee relationship in effect exists between hospitals and their
attending and visiting physicians.
The basis for holding an employer solidarily responsible for the
negligence of its employee is found in Article 2180 of the Civil Code
which considers a person accountable not only for his own acts but also
for those of others based on the
Even assuming that Dr. Ampil is not an employee of Medical City, but an
independent contractor, still the said hospital is liable to the Aganas.

In Nograles, et al. v. Capitol Medical Center, et al., the Court HELD:

In general, a hospital is not liable for the negligence of an


independent contractor-physician. There is, however, an exception
to this principle. The hospital may be liable if the physician is the
"ostensible" agent of the hospital.
The doctrine of apparent authority essentially involves two factors to
determine the liability of an independent contractor-physician.
o First factor focuses on the hospitals manifestations
whether the hospital acted in a manner which would lead a
reasonable person to conclude that the individual who was
alleged to be negligent was an employee or agent of the hospital
o Second factor focuses on the patients reliancewhether the
plaintiff acted in reliance upon the conduct of the hospital or its

3E Andaya Ching Espiritu Hefti Galvez Gammad Lainez Lui Madamba Nagera Narvasa
Ong Palangdao Rosales Sanchez Santos Satrain Tabo (2014-2015)

Labor Relations Case Digest - Atty. Joyrich Golangco

agent, consistent with ordinary care and prudence.


Atty. Agana categorically testified that one of the reasons why he chose
Dr. Ampil was that he knew him to be a staff member of Medical City,
a prominent and known hospital.
Clearly, PSI is estopped from passing the blame solely to Dr. Ampil. Its
act of displaying his name and those of the other physicians in the
public directory at the lobby of the hospital amounts to holding out
to the public that it offers quality medical service through the listed
physicians.
Lastly, PSI had been remiss in its duty. It did not conduct an immediate
investigation on the reported missing gauzes to the great prejudice and
agony of its patient. This renders PSI, not only vicariously liable for the
negligence of Dr. Ampil under Article 2180 of the Civil Code, but also
directly liable for its own negligence under Article 2176.

9: South East International Rattan vs. Coming

FACTS:

South East International Rattan, Inc. (SEIRI) is a domestic corporation


engaged in the business of manufacturing and exporting furniture to
various countries while petitioner Estanislao Agbay, as per records, is
the President and General Manager of SEIRI.
Respondent Jesus Coming filed a complaint for illegal dismissal,
underpayment of wages, non-payment of holiday pay, 13th month pay
and service incentive leave pay, with prayer for reinstatement, back
wages, damages and attorneys fees.
Respondents allegations: He was hired by petitioners as Sizing
Machine Operator on March 17, 1984. His work schedule is from 8:00
a.m. to 5:00 p.m. Initially, his compensation was on "pakiao" basis but
sometime it was fixed at P150.00 per day which was paid weekly. In
1990, without any apparent reason, his employment was interrupted as
he was told by petitioners to resume work in two months time
Despite being an employee for many years with his work performance
never questioned by petitioners, respondent was dismissed without
lawful cause. He was told that he will be terminated because the

company is not doing well financially and that he would be called back to
work only if they need his services again. Respondent waited for almost
a year but petitioners did not call him back to work.
Petitioners contention: denied having hired respondent and that
respondent actually worked for SEIRIs furniture suppliers. They
stressed that respondent was not included in the list of employees
submitted to the Social Security System (SSS). Moreover, respondents
brother, Vicente Coming, executed an affidavit in support of petitioners
position while Allan Mayol and Faustino Apondar issued notarized
certifications that respondent worked for them instead.
Labor Arbiter: respondent is a regular employee of SEIRI and that the
termination of his employment was illegal
o Respondents work as sizing machine operator is usually
necessary and desirable to the rattan furniture business of
petitioners and their failure to include respondent in the
employment report to SSS is not conclusive proof that
respondent is not their employee.
NLRC (Fourth Division): SET ASIDE LAs decision and dismissed the
complaint
o First complainant alleged that he worked continuously from
March 17, 1984 up to January 21, 2002. Records reveal however
that South East (Intl.) Rattan, Inc. was incorporated only last
July 18, 1986 and they were engaged purely on "buying and
exporting rattan furniture" hence no manufacturing employees
were hired. Furthermore, from the last quarter of 1989 up to
August of 1992, the company suspended operations due to
economic reverses
o Second, for all his insistence that he was a regular employee,
complainant failed to present a single pay slip, voucher or a
copy of a company payroll showing that he rendered service
during the period indicated therein.
o Complainants name does not appear in the list of employees
reported to the SSS nor does it appear in the sample payrolls of
respondents employees.
CA: reversed the NLRC and ruled that there existed an employeremployee relationship between petitioners and respondent who was

3E Andaya Ching Espiritu Hefti Galvez Gammad Lainez Lui Madamba Nagera Narvasa
Ong Palangdao Rosales Sanchez Santos Satrain Tabo (2014-2015)

Labor Relations Case Digest - Atty. Joyrich Golangco

dismissed without just and valid cause.


o As to the "control test", the following facts indubitably reveal
that respondents wielded control over the work performance of
petitioner, to wit: (1) they required him to work within the
company premises; (2) they obliged petitioner to report every
day of the week and tasked him to usually perform the same
job; (3) they enforced the observance of definite hours of work
from 8am to 5pm; (4) the mode of payment of petitioners
salary was under their discretion, at first paying him on pakiao
basis and thereafter, on daily basis; (5) they implemented
company rules and regulations; (6) Agbay directly paid
petitioners salaries and controlled all aspects of his
employment and (7) petitioner rendered work necessary and
desirable in the business of the respondent company.

ISSUES: Whether or not Coming is an employee of South East Rattan


HELD: YES

To ascertain the existence of employer-employee relationship


jurisprudence has invariably adhered to the four-fold test, to wit: (1)
the selection and engagement of the employee; (2) the payment of
wages; (3) the power of dismissal; and (4) the power to control the
employees conduct, or the so-called "control test." Although no
particular form of evidence is required to prove the existence of the
relationship, and any competent and relevant evidence to prove the
relationship may be admitted, a finding that the relationship exists must
nonetheless rest on substantial evidence.
In Tan v. Lagrama, the Court held that the fact that a worker was not
reported as an employee to the SSS is not conclusive proof of the
absence of employer-employee relationship. Otherwise, an employer
would be rewarded for his failure or even neglect to perform his
obligation.
Nor does the fact that respondents name does not appear in the
payrolls and pay envelope records submitted by petitioners negate
the existence of employer-employee relationship. For a payroll to be
utilized to disprove the employment of a person, it must contain a true

and complete list of the employee. In this case, the exhibits offered by
petitioners before the NLRC consisting of copies of payrolls and pay
earnings records are only for the years 1999 and 2000; they do not
cover the entire 18-year period during which respondent
supposedly worked for SEIRI.
While they claim that respondent was the employee of their
suppliers Mayol and Apondar, they did not submit proof that the
latter were indeed independent contractors; clearly, petitioners
failed to discharge their burden of proving their own affirmative
allegation. In any controversy between a laborer and his master,
doubts reasonably arising from the evidence are resolved in favor
of the laborer.
As a regular employee, respondent enjoys the right to security of
tenure under Article 279 of the Labor Code and may only be dismissed
for a just or authorized cause, otherwise the dismissal becomes illegal.
10: Tenazas et al vs. R. Villegas Taxi

FACTS:

Tenazas and Francisco filed a complaint for illegal dismissal against


respondents. At that time, a similar case had already been filed by
Endraca against the same respondents. The 2 cases were subsequently
consolidated.
In their position paper, petitioners alleged that they were hired and
dismissed by the respondents.
Tenazas allegation: On July 1, 2007, the taxi unit assigned to him was
sideswiped by another vehicle, causing a dent on the left fender near the
driver seat. Upon reporting the incident to the company, he was scolded
by respondents and was told to leave the garage for he is already fired.
He was even threatened with physical harm should he ever be seen in
the companys premises again. Despite the warning, Tenazas reported
for work on the following day but was told that he can no longer drive
any of the companys units as he is already fired.
Franciscos allegation: His dismissal without the benefit of procedural
due process was brought about by the companys unfounded suspicion
that he was organizing a labor union.

3E Andaya Ching Espiritu Hefti Galvez Gammad Lainez Lui Madamba Nagera Narvasa
Ong Palangdao Rosales Sanchez Santos Satrain Tabo (2014-2015)

Labor Relations Case Digest - Atty. Joyrich Golangco

Endracas allegation: His dismissal was instigated by an occasion when


he fell short of the required boundary for his taxi unit because he needed
to bring his unit for an urgent repair. Upon returning to the company
garage and informing the management of the incident, his drivers
license was confiscated and was told to settle the deficiency in his
boundary first before his license will be returned to him. He was no
longer allowed to drive a taxi unit despite his persistent pleas.
Respondents contention: Tenazas and Endraca were employees of the
company, the former being a regular driver and the latter a spare driver.
The respondents, however, denied that Francisco was an employee of
the company or that he was able to drive one of the companys units at
any point in time. That Tenazas was never terminated by the company
instead he was just advised to wait for further notice from the company
if his unit has already been fixed. However, upon being informed that his
unit is ready for release, Tenazas failed to report back to work for no
apparent reason. As regards Endraca, he stopped reporting for work
without informing the company of his reason. Even then, they
expressed willingness to accommodate Endraca should he wish to work
as a spare driver for the company again since he was never really
dismissed from employment anyway.
Petitioners filed a Motion to Admit Additional Evidence. They submitted
(a) Joint Affidavit of the petitioners; (2) Affidavit of Good Faith of Aloney
Rivera, a co-driver; (3) pictures of the petitioners wearing company
shirts; and (4) Tenazas Certification/Record of Social Security System
(SSS) contributions.
LA: There was no illegal dismissal.
NLRC: the additional pieces of evidence belatedly submitted by the
petitioners sufficed to establish the existence of employer-employee
relationship and their illegal dismissal.
CA: agreed with the NLRCs finding that Tenazas and Endraca were
employees of the company, but ruled otherwise in the case of Francisco
for failing to establish his relationship with the company.

ISSUES: Whether or not there exists employer-employee relationship


HELD: Affirmed CA Decision

In determining the presence or absence of an employer-employee


relationship, the Court has consistently looked for the following
incidents, to wit: (a) the selection and engagement of the employee;
(b) the payment of wages; (c) the power of dismissal; and (d) the
employers power to control the employee on the means and
methods by which the work is accomplished. The last element, the
so-called control test, is the most important element."
There is no hard and fast rule designed to establish the aforesaid
elements. Any competent and relevant evidence to prove the
relationship may be admitted. Identification cards, cash vouchers, social
security registration, appointment letters or employment contracts,
payrolls, organization charts, and personnel lists, serve as evidence of
employee status.
In this case, however, Francisco failed to present any proof
substantial enough to establish his relationship with the
respondents. Anent his claim that he was not issued with employment
records, he could have, at least, produced his social security records
which state his contributions, name and address of his employer, as his
co-petitioner Tenazas did. He could have also presented testimonial
evidence showing the respondents exercise of control over the
means and methods by which he undertakes his work.
Here, Francisco simply relied on his allegation that he was an
employee of the company without any other evidence supporting
his claim. Bereft of any evidence, the CA correctly ruled that
Francisco could not be considered an employee of the respondents.
The CAs order of reinstatement of Tenazas and Endraca, instead of the
payment of separation pay, is also well in accordance with prevailing
jurisprudence.
An illegally dismissed employee is entitled to two reliefs: back wages
and reinstatement.1wphi1 The two reliefs provided are separate and
distinct. In instances where reinstatement is no longer feasible because
of strained relations between the employee and the employer,
separation pay is granted. In effect, an illegally dismissed employee is
entitled to either reinstatement, if viable, or separation pay if
reinstatement is no longer viable, and back wages.

3E Andaya Ching Espiritu Hefti Galvez Gammad Lainez Lui Madamba Nagera Narvasa
Ong Palangdao Rosales Sanchez Santos Satrain Tabo (2014-2015)

Labor Relations Case Digest - Atty. Joyrich Golangco

After a perusal of the NLRC decision, this Court failed to find the factual
basis of the award of separation pay to the petitioners. The NLRC
decision did not state the facts which demonstrate that reinstatement is
no longer a feasible option that could have justified the alternative relief
of granting separation pay instead. Thus, it was a prudent call for the CA
to delete the award of separation pay and order for reinstatement
instead, in accordance with the general rule stated in Article 279 of the
Labor Code.
11. CITIBANK V CA

FACTS: In 1983, Citibank and El Toro Security Agency, Inc. (hereafter El Toro)
entered into a contract to protect the banks premises situated at Paseo de Roxas,
Makati, Metro Manila. Under the contract, El Toro obligated itself to provide the
services of security guards to safeguard and protect the premises and property
of Citibank against theft, robbery or any other unlawful acts. In
1990, this contract expired and was not renewed. Hence, private respondent
Citibank Integrated Guards Labor Alliance SEGATUPAS/ FSM (CIGLA) filed with
the National Conciliation and Mediation Board (NCMB) a request for preventive
mediation citing Citibank as respondent therein giving as issues for preventive
mediation the following:
a) Unfair labor practice
b) Dismissal of union officers/members and
c) Union busting.
Petitioner Citibank served on El Toro a written notice that the bank would not
renew anymore the service agreement with the latter. Simultaneously, Citibank
hired another security agency, the
Golden Pyramid Security Agency, to render security services at Citibank's
premises.

Private respondent CIGLA filed a manifestation with the NCMB that it was
converting its request for preventive mediation into a notice of strike for failure
of the parties to reach a mutually acceptable settlement of the issues, which it
followed with a supplemental notice of strike alleging as supplemental issue the
mass dismissal of all union officers and members. Security guards of El Toro who

were replaced by guards of the Golden Pyramid Security Agency considered the
nonrenewal
of El Toro's service agreement with Citibank as constituting a lockout and/or a
mass dismissal. They threatened to go on strike against Citibank and picket its
premises. In fact, security guards formerly assigned to Citibank under the
expired agreement loitered around and near the Citibank premises in large
groups of from twenty (20) and at times fifty (50) persons.

Faced with the prospect of disruption of its business operations, petitioner


Citibank filed with the RTC of Makati, a complaint for injunction and damages.
The complaint sought to enjoin CIGLA and any person claiming membership
therein from striking or otherwise disrupting the operations
of the bank. CIGLA filed a motion to dismiss on the ground that the RTC has no
jurisdiction, the subject matter being a labor dispute. The motion to dismiss was
denied.

CIGLA then filed with the CA a petition for certiorari assailing the validity of the
proceedings had before the regional trial court. The CA ruled in CIGLAs favor.
Hence, this petition by Citibank. The basic issue involved is whether it is the
labor tribunal or the regional trial court that has jurisdiction over the subject
matter of the complaint filed by Citibank with the trial court.
Petitioner Citibank contends that there is no employer-employee relationship
between Citibank and the security guards represented by respondent CIGLA and
that there is no "labor dispute" in the subject controversy. The security guards
were employees of El Toro security agency, not of Citibank. Its service contract
with Citibank had expired and not renewed.

We sustain the petitioner's contention. This Court has held in many cases that "in
determining the existence of an employer- employee relationship, the following
elements are generally considered: 1) the selection and engagement of the
employee 2) The payment of wages 3) The power of dismissal and 4) the
employer's power to control the employee with respect to the means and
methods by which the work is to be accomplished". It has been decided also that
the Labor Arbiter has no jurisdiction over a claim filed where no employeremployee relationship existed between a company and the security guards
assigned to it by a security service contractor. In this case, it was the security

3E Andaya Ching Espiritu Hefti Galvez Gammad Lainez Lui Madamba Nagera Narvasa
Ong Palangdao Rosales Sanchez Santos Satrain Tabo (2014-2015)

Labor Relations Case Digest - Atty. Joyrich Golangco

agency El Toro that recruited, hired and assigned the watchmen to their place of
work. It was the security agency that was answerable to Citibank for the conduct
of its guards.
ISSUES: Is there a labor dispute between Citibank and the security guards,
members of respondent CIGLA, regardless of whether they stand in the
relation of employer and employees?

HELD: NO, Article 212, paragraph l of the Labor Code provides the definition of a
"labor dispute". It "includes any controversy or matter concerning terms or
conditions of employment or the association or representation of persons in
negotiating, fixing, maintaining, changing or arranging the terms and conditions
of employment, regardless of whether the disputants stand in the proximate
relation of employer and employee."

If at all, the dispute between Citibank and El Toro security agency is one
regarding the termination or nonrenewal of the contract of services. This is a
civil dispute. El Toro was an independent contractor. Thus, no employeremployee relationship existed between Citibank and the security guard members
of the union in the security agency who were assigned to secure the bank's
premises and property. Hence, there was no labor dispute and no right to strike
against the bank.

On the basis of the allegations of the complaint, it is safe to conclude that the
dispute involved is a civil one, not a labor dispute. Consequently, we rule that
jurisdiction over the subject matter of the complaint lies with the regional trial
court.

12: PHILIPPINE AIRLINES vs. NATIONAL LABOR RELATIONS COMMISSION


(PAL vs. NLRC)
FACTS:

Private respondents are flight stewards of the petitioner. Both were


dismissed from the service for their alleged involvement in the April 3,
1993 currency smuggling in Hong Kong.
A confrontation between them and Mr. Abaca (the man who was

carrying the bag containing the smuggled money worth 2.5 Million pesos
when converted to Philippine currency) was compulsorily arranged by
PALs disciplinary board; Abaca was made to identify petitioners as coconspirators, which was anomalous because there was no one else in the
line-up but them.
Despite that, Abaca still had difficulty in identifying Pineda as his coconspirator, and as to Cabling, he was pointed by Abaca only after PALs
lawyer pressed the him to identify Cabling as co-conspirator;
During the next hearing Abaca finally gave statements to the board
clearing Pineda and Cabling from any participation or from being the
owners of the currencies.
Just as they thought that they were already fully cleared of the charges,
they were surprised to receive a Memorandum terminating their
services for alleged violation of PALs Code of Discipline.
Aggrieved by said dismissal, private respondents filed with the NLRC a
petition for injunction
NLRC: issued a temporary mandatory injunction enjoining petitioner to
cease and desist from enforcing its Memorandum of dismissal, adopting
the view that:
1. PALs Code of Discipline was declared illegal by the SC in the
case of PAL, Inc. vs. NLRC for the reason that it was formulated
by the petitioner without the participation of its employees as
required in R.A. 6715, amending Article 211 of the Labor Code
2. The whimsical, baseless and premature dismissals of private
respondents which "caused them grave and irreparable injury"
is enjoinable as private respondents are left "with no speedy
and adequate remedy at law
3. NLRC is empowered under Article 218 (e) of the Labor Code to
issue a temporary mandatory injunction
PAL moved for reconsideration (some of PALs contentions among
others)
1. NLRC has no jurisdiction to issue an injunction or
restraining order since this may be issued only under
Article 218 of the Labor Code if the case involves or arises
from labor disputes
2. NLRC divested the labor arbiter of its original and

3E Andaya Ching Espiritu Hefti Galvez Gammad Lainez Lui Madamba Nagera Narvasa
Ong Palangdao Rosales Sanchez Santos Satrain Tabo (2014-2015)

Labor Relations Case Digest - Atty. Joyrich Golangco

exclusive jurisdiction over illegal dismissal cases;


3. There is no irreparable or substantial injury.
4. Assuming that the acts of dismissing petitioners 'may be great,
still the same is capable of compensation', consequently,
'injunction need not be issued where adequate
compensation at law could be obtained'.
ISSUES: Whether or Not the NLRC acted in excess of its jurisdiction.

Labor Code, as amended, when it is established on the bases of the sworn


allegations in the petition that the acts complained of, involving or
arising from any labor dispute before the Commission, which, if not
restrained or performed forthwith, may cause grave or irreparable
damage to any party or render ineffectual any decision in favor of such
party.

HELD: YES

Generally, injunction is not a cause of action in itself but merely a


provisional remedy, an adjunct to a main suit. It is resorted
to only when there is a pressing necessity to avoid injurious
consequences, which cannot be remedied under any standard of
compensation. The essential conditions for granting such temporary
injunctive relief is that the complaint alleges facts which appear to be
sufficient to constitute a proper basis for injunction and that on the
entire showing from the contending parties, the injunction is
reasonably necessary to protect the legal rights of the plaintiff pending
the litigation. Injunction is also a special equitable relief granted only in
cases where there is no plain, adequate and complete remedy at
law.

In labor cases, Article 218 of the Labor Code empowers the NLRC-

"(e) To enjoin or restrain any actual or threatened commission of any or


all prohibited or unlawful acts or to require the performance of a
particular act in any labor dispute which, if not restrained or performed
forthwith, may cause grave or irreparable damage to any party or
render ineffectual any decision in favor of such party; x x x."

Complementing the above-quoted provision, Sec. 1, Rule XI of the New Rules of


Procedure of the NLRC, pertinently provides as follows:
"Section 1. Injunction in Ordinary Labor Dispute.-A preliminary injunction
or a restraining order may be granted by the Commission through its
divisions pursuant to the provisions of paragraph (e) of Article 218 of the

From the foregoing provisions of law, the power of the NLRC to issue an
injunctive writ originates from "any labor dispute" upon application by
a party thereof, which application if not granted "may cause grave or
irreparable damage to any party or render ineffectual any decision
in favor of such party."
The term "labor dispute" is defined as "any controversy or matter
concerning terms and conditions of employment or the association
or representation of persons in negotiating, fixing, maintaining,
changing, or arranging the terms and conditions of employment
regardless of whether or not the disputants stand in the proximate
relation of employers and employees."
It is an essential requirement that there must first be a labor
dispute between the contending parties before the labor arbiter. In
the present case, there is no labor dispute between the petitioner
and private respondents.
The petition for injunction directly filed before the NLRC is in
reality an action for illegal dismissal. This is clear from the
allegations in the petition which prays for: reinstatement of private
respondents; award of full back wages, moral and exemplary damages;
and attorney's fees. As such, the petition should have been filed with
the labor arbiter who has the original and exclusive jurisdiction to
hear and decide such cases. The only exceptions are where the Secretary
of Labor and Employment or the NLRC exercises the power of
compulsory arbitration, or the parties agree to submit the matter to
voluntary arbitration pursuant to Article 263 (g) of the Labor Code.
The jurisdiction of the NLRC in illegal dismissal cases is appellate in
nature and, therefore, it cannot entertain the private respondents'
petition for injunction which challenges the dismissal orders of
petitioner. Article 218(e) of the Labor Code does not provide

3E Andaya Ching Espiritu Hefti Galvez Gammad Lainez Lui Madamba Nagera Narvasa
Ong Palangdao Rosales Sanchez Santos Satrain Tabo (2014-2015)

Labor Relations Case Digest - Atty. Joyrich Golangco

blanket authority to the NLRC or any of its divisions to issue writs


of injunction, injunction is only an ancillary remedy in ordinary
labor disputes Thus, the NLRC exceeded its jurisdiction when it
issued the assailed Order
Under the Labor Code, the ordinary and proper recourse of an
illegally dismissed employee is to file a complaint for illegal
dismissal with the labor arbiter. If the remedy is specifically
provided by law, it is presumed to be adequate. Moreover, the
preliminary mandatory injunction prayed for by the private
respondents in their petition before the NLRC can also be
entertained by the labor arbiter who, as shown earlier, has the
ancillary power to issue preliminary injunctions or restraining orders as
an incident in the cases pending before him in order to preserve the
rights of the parties during the pendency of the case.
Furthermore, there is no showing of any urgency or irreparable
injury, which the private respondents might suffer. It is considered
irreparable injury when it cannot be adequately compensated in
damages due to the nature of the injury itself or the nature of the right or
property injured or when there exists no certain pecuniary standard for
the measurement of damages
In the case at bar, the alleged injury which private respondents stand
to suffer by reason of their alleged illegal dismissal can be
adequately compensated and therefore, there exists no
"irreparable injury"
Finally, an injunction, as an extraordinary remedy, is not favored in
labor law considering that it generally has not proved to be an
effective means of settling labor disputes. It has been the policy of the
State to encourage the parties to use the non-judicial process of
negotiation and compromise, mediation and arbitration. Thus,
injunctions may be issued only in cases of extreme necessity based
on legal grounds clearly established, after due consultations or
hearing and when all efforts at conciliation are exhausted which
factors, however, are clearly absent in the present case.

13: CHARLITO PEARANDA vs. BAGANGA PLYWOOD CORPORATION and


HUDSON CHUA

Facts: Sometime in June 1999, Petitioner Charlito Pearanda was hired as an


employee of Baganga Plywood Corporation (BPC) to take charge of the
operations and maintenance of its steam plant boiler. In May 2001, Pearanda
filed a Complaint for illegal dismissal with money claims against BPC and its
general manager, Hudson Chua, before the NLRC.

After the parties failed to settle amicably, the labor arbiter8 directed the parties
to file their position papers and submit supporting documents. Their respective
allegations are summarized by the labor arbiter as follows:

Pearanda in his position paper alleges that he was employed by Baganga as


Foreman/Boiler Head/Shift Engineer until he was illegally terminated. Further,
[he] alleges that his services [were] terminated without the benefit of due
process and valid grounds in accordance with law. Furthermore, he was not paid
his overtime pay, premium pay for working during holidays/rest days, night
shift differentials and finally claims for payment of damages and attorneys fees
having been forced to litigate the present complaint.
The respondent [BPC] alleges that it was on temporary closure due to repair and
general maintenance and it applied for clearance with the DOLE Regional Office
No. XI to shut down and to dismiss employees. And due to the insistence of
herein complainant he was paid his separation benefits. Consequently, when
respondent [BPC] partially reopened in January 2001, [Pearanda] failed to
reapply. Hence, he was not terminated from employment much less illegally. He
opted to severe employment when he insisted payment of his separation
benefits. Furthermore, being a managerial employee he is not entitled to
overtime pay and if ever he rendered services beyond the normal hours of work,
[there] was no office order/or authorization for him to do so.
The labor arbiter ruled that there was no illegal dismissal and that petitioners
Complaint was premature because he was still employed by BPC. The temporary
closure of BPCs plant did not terminate his employment; hence, he need not
reapply when the plant reopened.

According to the labor arbiter, petitioners money claims for illegal dismissal was
also weakened by his quitclaim and admission during the clarificatory

3E Andaya Ching Espiritu Hefti Galvez Gammad Lainez Lui Madamba Nagera Narvasa
Ong Palangdao Rosales Sanchez Santos Satrain Tabo (2014-2015)

Labor Relations Case Digest - Atty. Joyrich Golangco

conference that he accepted separation benefits, sick and vacation leave


conversions and thirteenth month pay.

Nevertheless, the labor arbiter found petitioner entitled to overtime pay,


premium pay for working on rest days, and attorneys fees.

On appeal, the NLRC deleted the award of overtime pay and premium pay for
working on rest days. According to the Commission, petitioner was not entitled
to these awards because he was a managerial employee.
CA dismissed Penarandas petition for certiorari as well as his motion for
reconsideration.

ISSUES: Is Penaranda a managerial employee, hence not worthy of the awards


claimed?

HELD: No, However, he is still not worthy of the awards claimed because he is a
managerial staff.

Article 82 of the Labor Code exempts managerial employees from the coverage of
labor standards. Labor standards provide the working conditions of employees,
including entitlement to overtime pay and premium pay for working on rest
days. Under this provision, managerial employees are "those whose primary duty
consists of the management of the establishment in which they are employed or
of a department or subdivision."
The Implementing Rules of the Labor Code state that managerial employees are
those who meet the following conditions:
"(1) Their primary duty consists of the management of the
establishment in which they are employed or of a department or
subdivision thereof;

"(2) They customarily and regularly direct the work of two or more
employees therein;

"(3) They have the authority to hire or fire other employees of lower
rank; or their suggestions and recommendations as to the hiring and
firing and as to the promotion or any other change of status of other
employees are given particular weight."

The Court disagrees with the NLRCs finding that petitioner was a managerial
employee. However, petitioner was a member of the managerial staff, which also
takes him out of the coverage of labor standards. Like managerial employees,
officers and members of the managerial staff are not entitled to the provisions of
law on labor standards. The Implementing Rules of the Labor Code define
members of a managerial staff as those with the following duties and
responsibilities:
"(1) the primary duty consists of the performance of work directly
related to management policies of the employer;

"(2) customarily and regularly exercise discretion and independent


judgment;

"(3) (i) Regularly and directly assist a proprietor or a managerial


employee whose primary duty consists of the management of the
establishment in which he is employed or subdivision thereof; or (ii)
execute under general supervision work along specialized or technical
lines requiring special training, experience, or knowledge; or (iii)
execute under general supervision special assignments and tasks; and

"(4) who do not devote more than 20 percent of their hours worked in a
workweek to activities which are not directly and closely related to the
performance of the work described in paragraphs (1), (2), and (3)
above."

As shift engineer, petitioners duties and responsibilities were as follows:


"1. To supply the required and continuous steam to all consuming units
at minimum cost.
"2. To supervise, check and monitor manpower workmanship as well as
operation of boiler and accessories.

3E Andaya Ching Espiritu Hefti Galvez Gammad Lainez Lui Madamba Nagera Narvasa
Ong Palangdao Rosales Sanchez Santos Satrain Tabo (2014-2015)

Labor Relations Case Digest - Atty. Joyrich Golangco

"3. To evaluate performance of machinery and manpower.


"5. To train new employees for effective and safety while working.
"7. To recommend personnel actions such as: promotion, or disciplinary
action.

The foregoing enumeration, illustrates that petitioner was a member of the


managerial staff. His duties and responsibilities conform to the definition of a
member of a managerial staff under the Implementing Rules.

Petitioner supervised the engineering section of the steam plant boiler. His work
involved overseeing the operation of the machines and the performance of the
workers in the engineering section. This work necessarily required the use of
discretion and independent judgment to ensure the proper functioning of the
steam plant boiler. As supervisor, petitioner is deemed a member of the
managerial staff.

Noteworthy, even petitioner admitted that he was a supervisor. In his Position


Paper, he stated that he was the foreman responsible for the operation of the
boiler. The term foreman implies that he was the representative of management
over the workers and the operation of the department. Petitioners evidence also
showed that he was the supervisor of the steam plant. His classification as
supervisor is further evident from the manner his salary was paid. He belonged
to the 10% of respondents 354 employees who were paid on a monthly basis;
the others were paid only on a daily basis.
On the basis of the foregoing, the Court finds no justification to award overtime
pay and premium pay for rest days to petitioner.

14: SAMAHANG MANGGAGAWA SA CHARTER CHEMICAL SOLIDARITY OF


UNIONS IN THE PHILIPPINES FOR EMPOWERMENT AND REFORMS (SMCCSUPER), ZACARRIAS JERRY VICTORIO-Union President vs CHARTER
CHEMICAL and COATING CORPORATION

FACTS: Samahang Manggagawa sa Charter Chemical Solidarity of Unions in the


Philippines for Empowerment and Reforms (petitioner union) filed a petition for
certification election among the regular rank-and-file employees of Charter

Chemical and Coating Corporation (respondent company) with the Mediation


Arbitration of the DOLE, NCR.

Respondent company filed an Answer with Motion to Dismiss on the ground that
petitioner union is not a legitimate labor organization because of the inclusion of
supervisory employees within petitioner union among others

Med-Arbiter Tomas F. Falconitin issued a Decision dismissing the petition for


certification election. The Med-Arbiter ruled that petitioner union is not a
legitimate labor organization because the list of membership of petitioner union
consisted of 12 batchman, mill operator and leadman who performed
supervisory functions. Under Article 245 of the Labor Code, said supervisory
employees are prohibited from joining petitioner union which seeks to represent
the rank-and-file employees of respondent company.
DOLE initially issued a Decision in favor of respondent company dismissing
petitioner unions appeal on the ground that the latters petition for certification
election was filed out of time. On motion for reconsideration, however, the DOLE
reversed its earlier ruling. (More on documentation requirements itong part na
to so di ko na inelaborate).

In nullifying the decision of the DOLE, the CA gave credence to the findings of the
Med-Arbiter that petitioner union failed to comply with the documentation
requirements under the Labor Code. It, likewise, upheld the Med-Arbiters
finding that petitioner union consisted of both rank-and-file and supervisory
employees.
ISSUES: Is the mixture of rank-and-file and supervisory employee[s] of
petitioner [unions] membership [a] ground for the cancellation of petitioner
[unions] legal personality and dismissal of [the] petition for certification
election?

HELD: No. The mixture of rank-and-file and supervisory employees in petitioner


union does not nullify its legal personality as a legitimate labor organization.

3E Andaya Ching Espiritu Hefti Galvez Gammad Lainez Lui Madamba Nagera Narvasa
Ong Palangdao Rosales Sanchez Santos Satrain Tabo (2014-2015)

Labor Relations Case Digest - Atty. Joyrich Golangco

The CA found that petitioner union has for its membership both rank-and-file
and supervisory employees. However, petitioner union sought to represent the
bargaining unit consisting of rank-and-file employees. Under Article 245 of the
Labor Code, supervisory employees are not eligible for membership in a labor
organization of rank-and-file employees. Thus, the appellate court ruled that
petitioner union cannot be considered a legitimate labor organization pursuant
to Toyota Motor Philippines v. Toyota Motor Philippines Corporation Labor
Union (hereinafter Toyota).

Nonetheless, the inclusion of the aforesaid supervisory employees in petitioner


union does not divest it of its status as a legitimate labor organization. The
appellate courts reliance on Toyota is misplaced in view of this Courts
subsequent ruling in Republic v. Kawashima Textile Mfg., Philippines, Inc.
(hereinafter Kawashima). In Kawashima, we explained at length how and why
the Toyota doctrine no longer holds sway under the altered state of the law and
rules applicable to this case, viz:

R.A. No. 6715 omitted specifying the exact effect any violation of the prohibition
[on the co-mingling of supervisory and rank-and-file employees] would bring
about on the legitimacy of a labor organization.
Then came Tagaytay Highlands Int'l. Golf Club, Inc. v. Tagaytay Highlands
Employees Union-PGTWO in which the core issue was whether mingling affects
the legitimacy of a labor organization and its right to file a petition for
certification election. This time, given the altered legal milieu, the Court
abandoned the view in Toyota and Dunlop and pronounced that while there is a
prohibition against the mingling of supervisory and rank-and-file employees in
one labor organization, the Labor Code does not provide for the effects thereof.
Thus, the Court held that after a labor organization has been registered, it may
exercise all the rights and privileges of a legitimate labor organization. Any
mingling between supervisory and rank-and-file employees in its membership
cannot affect its legitimacy for that is not among the grounds for cancellation of
its registration, unless such mingling was brought about by misrepresentation,
false statement or fraud under Article 239 of the Labor Code.

15: Pamela Florentina P. Jumuad vs Hi-flyer Food, Inc. and Jesus R.


Montemayor

Facts: Petitioner Pamela Florentina P. Jumuad (Jumuad) began her employment


with respondent Hi-Flyer Food, Inc. (Hi-Flyer), as management trainee. Hi-Flyer
is a corporation licensed to operate Kentucky Fried Chicken (KFC) restaurants in
the Philippines. Based on her performance through the years, Jumuad received
several promotions until she became the area manager for the entire VisayasMindanao 1 region, comprising the provinces of Cebu, Bacolod, Iloilo and Bohol.
Aside from being responsible in monitoring her subordinates, Jumuad was
tasked to: 1) be highly visible in the restaurants under her jurisdiction; 2)
monitor and support day-to-day operations; and 3) ensure that all the facilities
and equipment at the restaurant were properly maintained and serviced. Among
the branches under her supervision were the KFC branches in Gaisano
Mall, Cebu City (KFC-Gaisano); in Cocomall, Cebu City (KFC-Cocomall); and in
Island City Mall, Bohol (KFC-Bohol).
In just her first year as Area Manager, Jumuad gained distinction and was
awarded the 3rd top area manager nationwide. She was rewarded with a trip
to Singapore for her excellent performance.

Hi Flyer conducted a food safety, service and sanitation audit at KFC-Gaisano. The
audit, denominated as CHAMPS Excellence Review (CER), revealed several
sanitation violations, such as the presence of rodents and the use of a defective
chiller for the storage of food. When asked to explain, Jumuad first pointed out
that she had already taken steps to prevent the further infestation of the branch.
As to why the branch became infested with rodents, Jumuad faulted
managements decision to terminate the services of the branchs pest control
program and to rely solely on the pest control program of the mall. As for the
defective chiller, she explained that it was under repair at the time of the
CER. Soon thereafter, Hi-Flyer ordered the KFC-Gaisano branch closed.
Hi-Flyer audited the accounts of KFC-Bohol amid reports that certain employees
were covering up cash shortages. As a result, the following irregularities were
discovered: 1) cash shortage amounting to 62,290.85; 2) delay in the deposits of

3E Andaya Ching Espiritu Hefti Galvez Gammad Lainez Lui Madamba Nagera Narvasa
Ong Palangdao Rosales Sanchez Santos Satrain Tabo (2014-2015)

Labor Relations Case Digest - Atty. Joyrich Golangco

cash sales by an average of three days; 3) the presence of two sealed cash-fordeposit envelopes containing paper cut-outs instead of cash; 4) falsified entries
in the deposit logbook; 5) lapses in inventory control; and 6) material product
spoilage. In her report regarding the incident, Jumuad disclaimed any fault in the
incident by pointing out that she was the one responsible for the discovery of this
irregularity.

Hi-Flyer conducted another CER, this time at its KFC-Cocomall branch. Grout and
leaks at the branchs kitchen wall, dried up spills from the marinator, as well as a
live rat under postmix, and signs of rodent gnawing/infestation were found. This
time, Jumuad explained to management that she had been busy conducting
management team meetings at the other KFC branches and that, at the date the
CER was conducted, she had no scheduled visit at the KFC-Cocomall branch.
Seeking to hold Jumuad accountable for the irregularities uncovered in the
branches under her supervision, Hi-Flyer sent Jumuad an Irregularities Report
and Notice of Charges which she received. Jumuad submitted her written
explanation. Hi-Flyer held an administrative hearing where Jumuad appeared
with counsel. Apparently not satisfied with her explanations, Hi-Flyer served her
a Notice of Dismissal effecting her termination.
This prompted Jumuad to file a complaint against Hi-Flyer and/or Jesus R.
Montemayor (Montemayor) for illegal dismissal before the NLRC on October 17,
2005, praying for reinstatement and payment of separation pay, 13th month pay,
service incentive leave, moral and exemplary damages, and attorneys fees, etc.

Labor Arbiter Ruling: Jumuad was not completely blameless for the anomalies
discovered, the dismissal was too harsh considering the circumstances. After
finding that no serious cause for termination existed, the LA ruled that Jumuad
was illegally dismissed.
NLRC affirmed the decision in toto.

CA reversed the ruling. On the issue of loss of trust and confidence, the CA
considered the deplorable sanitary conditions and the cash shortages uncovered

at three of the seven KFC branches supervised by Jumuad as enough bases for HiFlyer to lose its trust and confidence in her.
ISSUES: Was Jumuad Illegally dismissed?

HELD: No. On whether Jumuad was illegally dismissed, Article 282 of the Labor
Code provides that an employer may terminate an employment for Gross and
habitual neglect by the employee of his duties, and Fraud or willful breach by the
employee of the trust reposed in him by his employer or duly authorized
representative among others.
Jumuad was terminated for neglect of duty and breach of trust and confidence.
To be a ground for removal, the neglect of duty must be both gross and habitual.
On the other hand, breach of trust and confidence, as a just cause for termination
of employment, is premised on the fact that the employee concerned holds a
position of trust and confidence, where greater trust is placed by management
and from whom greater fidelity to duty is correspondingly expected. The betrayal
of this trust is the essence of the offense for which an employee is penalized.

After an assiduous review of the facts as contained in the records, the Court is
convinced that Jumuad cannot be dismissed on the ground of gross and habitual
neglect of duty. The Court notes the apparent neglect of Jumuad of her duty in
ensuring that her subordinates were properly monitored and that she had
dutifully done all that was expected of her to ensure the safety of the consuming
public who continue to patronize the KFC branches under her jurisdiction. Had
Jumuad discharged her duties to be highly visible in the restaurants under her
jurisdiction, monitor and support the day to day operations of the branches and
ensure that all the facilities and equipment at the restaurant were properly
maintained and serviced, the deplorable conditions and irregularities at the
various KFC branches under her jurisdiction would have been prevented.
Considering, however, that over a year had lapsed between the incidences at
KFC-Gaisano and KFC-Bohol, and that the nature of the anomalies uncovered
were each of a different nature, the Court finds that her acts or lack of action in
the performance of her duties is not born of habit.

3E Andaya Ching Espiritu Hefti Galvez Gammad Lainez Lui Madamba Nagera Narvasa
Ong Palangdao Rosales Sanchez Santos Satrain Tabo (2014-2015)

Labor Relations Case Digest - Atty. Joyrich Golangco

Despite saying this, it cannot be denied that Jumuad willfully breached her duties
as to be unworthy of the trust and confidence of Hi-Flyer. First, there is no
denying that Jumuad was a managerial employee. As correctly noted by
the appellate court, Jumuad executed management policies and had the
power to discipline the employees of KFC branches in her
area. She recommended actions on employees to the head office. Pertinent is
Article 212 (m) of the Labor Code defining a managerial employee as one who is
vested with powers or prerogatives to lay down and execute management
policies and/or hire, transfer, suspend, lay off, recall, discharge, assign or
discipline employees.
Based on established facts, the mere existence of the grounds for the loss of trust
and confidence justifies petitioners dismissal. Pursuant to the Courts ruling
in Lima Land, Inc. v. Cuevas, as long as there is some basis for such loss of
confidence, and the nature of his participation therein renders him unworthy of
the trust and confidence demanded of his position, a managerial employee may
be dismissed.

In the present case, the CERs reports of Hi-Flyer show that there were anomalies
committed in the branches managed by Jumuad. On the principle
of respondeat superioror command responsibility alone, Jumuad may be held
liable for negligence in the performance of her managerial duties. She may not
have been directly involved in causing the cash shortages in KFC-Bohol, but her
involvement in not performing her duty monitoring and supporting the day to
day operations of the branches and ensure that all the facilities and equipment at
the restaurant were properly maintained and serviced, could have truly
prevented the whole debacle from ever occurring.
Thus, there is reasonable basis for Hi-Flyer to withdraw its trust in her and
dismissing her from its service.
16. Ex-Bataan Veterans Security Agency V. Secretary of Labor Bienvenido E.
Laguesma

FACTS: Ex- Bataan Veterans Security Agency, Inc. (EBVSAI) is in the business of
providing security services while Private Respondents are employees assigned to
the National Power Corporation at Ambuklao Hydro Electric Plant, Bokod,
Benguet (Ambuklao Plant). On February 20, 1996, Private Respondents led by

Alexander Pocding instituted a complaint for underpayment of wages against


EBVSAI before the Regional Office of the Department of Labor and Employment
(DOLE). On March 7, 1996, the Regional Office Conducted a complaint inspection
and violations were noted: (1) non-presentation of records; (2) non-payment of
holiday pay; (3) non-payment of rest day premium; (4) underpayment of night
shift differential pay; (5) non-payment of service incentive leave; (6)
underpayment of 13th month pay; (7) no registration; (8) no annual medical
report; (9) no annual work accidental report; (10) no safety committee; and (11)
no trained first aider. On the same day the Regional Office issued a notice of
hearing requiring EBVSAI and private respondents to attend the hearing on 22
March 1996. On August 19,1996, the Regional Director issued an order for
EBVSAI to pay computed deficiencies owing to the affected employees in
the total amount of P763,997.85. EBVSAI filed a Motion for Reconsideration, it
alleged that the Regional Director does not have any jurisdiction over the subject
matter of the case because the money claim exceeded P5,000 and that the
Regional Director should have endorsed the case to the Labor Arbiter. Also in a
supplemental motion for reconsideration, EBVSAI questioned the Regional
Directors basis for the computation of the deficiencies due to each Private
Respondent.
The Regional Director (RD) denied the motion for
reconsideration and supplemental motion for reconsideration. The RD
stated that pursuant to RA#7730, the limitations under Articles 129 and
217(6) of the Labor Code no longer apply to the Secretary of Labors
visitorial and enforcement powers under Article 128(b). The Secretary of
Labor or his duly authorized representatives are now empowered to hear and
decide, in a summary proceeding, any matter involving the recovery of any
amount of wages and other monetary claims arising out of the EmployerEmployee relations at the time of the inspection. EBVSAI appealed to the
Secretary of Labor. The Secretary of Labor affirmed with modification the
RDs order. The Secretary of Labor ruled that, pursuant to RA 7730, the
Court's decision in the Servando case is no longer controlling insofar as the
restrictive effect of Article 129 on the visitorial and enforcement power of
the Secretary of Labor is concerned. There was also no denial of due
process because EBVSAI was accorded several opportunities to present its
side but it failed to present any evidence to controvert the findings of the
RD. The EBVSAI filed motion for reconsideration which was denied by the
Secretary of Labor, and then it filed a petition for certiorari before the court of

3E Andaya Ching Espiritu Hefti Galvez Gammad Lainez Lui Madamba Nagera Narvasa
Ong Palangdao Rosales Sanchez Santos Satrain Tabo (2014-2015)

Labor Relations Case Digest - Atty. Joyrich Golangco

appeals. The CA dismissed the petition and affirmed the Secretary of


Labors decision. CA denied motion for reconsideration.
ISSUES: Whether the Secretary of Labor or his duly authorized representatives
have jurisdiction over the money claims of private respondents which
exceed P5,000.

HELD: EBVSAI maintains that under Articles 129 and 217(6) of the Labor Code,
the Labor Arbiter, not the Regional Director, has exclusive and original
jurisdiction over the case because the individual monetary claim of private
respondents exceeds P5,000. EBVSAI also argues that the case falls under the
exception clause in Article 128(b) of the Labor Code. EBVSAI asserts that the
Regional Director should have certified the case to the Arbitration Branch of the
National Labor Relations Commission (NLRC) for a full-blown hearing on the
merits. Said provisions of law do not contemplate nor cover the visitorial and
enforcement powers of the Secretary of Labor or his duly authorized
representatives. The visitorial and enforcement powers of the DOLE Regional
Director to order and enforce compliance with labor standard laws can be
exercised even where the individual claim exceeds P5,000 (Cirineo Bowling
Plaza, Inc. v. Sensing). In order to divest the Regional Director or his
representatives of Jurisdiction, the following elements must be present; (a) that
the employer contests the findings of the labor regulations officer and raises
issues thereon; (b) that in order to resolve such issues, there is a need to examine
evidentiary matters; and (c) that such matters are not verifiable in the normal
course of inspection. The rules also provide that the employer shall raise such
objections during the hearing of the case or at any time after receipt of the notice
of inspection results. In this case, the Regional Director validly assumed
jurisdiction over the money claims of private respondents even if the claims
exceeded P5,000 because such jurisdiction was exercised in accordance with
Article 128(b) of the Labor Code and the case does not fall under the exception
clause. It was only in the supplemental motion for reconsideration before the
Regional Director that EBVSAI questioned the findings of labor regulations
officer and presented documentary evidence to controvert the claims of private
respondents. The RD and the Secretary of Labor still looked into and considered
EBVSAIs documentary evidence and found that such did not warrant the
reversal of the RDs order. The Secretary of Labor also doubted the veracity and
authenticity of EBVSAI's documentary evidence. Moreover, the pieces of

evidence presented by EBVSAI were verifiable in the normal course of inspection


because all employment records of the employees should be kept and
maintained in or about the premises of the workplace, which in this case is in
Ambuklao Plant, the establishment where private respondents were regularly
assigned.
17. Arsenio Z. Locsin v. Nissan Car Lease Phils., Inc. and Luis Banson

FACTS: On January 1, 1992, Locsin was elected Executive Vice President and
Treasurer (EVP/Treasurer) of NCLPI. As EVP/Treasurer, his duties and
responsibilities included: (1) the management of the finances of the company;
(2) carrying out the directions of the President and/or the Board of Directors
regarding financial management; and (3) the preparation of financial reports to
advise the officers and directors of the financial condition of NCLPI. Locsin held
this position for 13 years, having been re-elected every year since 1992, until
January 21, 2005, when he was nominated and elected Chairman of NCLPIs
Board of Directors. On August 25, 2005, the NCLPI Board held a special meeting.
One of items of the agenda was the election of the new set of
officers. Unfortunately, Locsin was neither re-elected Chairman nor reinstated to
his previous position as EVP/Treasurer. On June 19, 2007, Locsin filed a
complaint for illegal dismissal with prayer for reinstatement, payment of back
wages, damages and attorneys fees before the Labor Arbiter against NCLPI and
Banson, who was then the President of NCLPI. Instead of filing their position
paper NCLPI and Banson filed a motion to dismiss on the ground that the Labor
Arbiter has no jurisdiction over the case since the removal of Locsin as
EVP/Treasurer involves an intra-corporate dispute and Jurisdiction is with the
RTC.
Locsin submitted his opposition to the motion maintaining his position as an
employee of NCLPI. On March 10, 2008, Labor Arbiter Concepcion issued an
Order denying the Motion to Dismiss, holding that her office acquired
jurisdiction to arbitrate and/or decide the instant complaint finding
extant in the case an employer-employee relationship. NCLPI elevated the
case to the CA through a Petition for Certiorari under Rule 65 of the Rules of
Court. NCLPI raised the issue on whether the Labor Arbiter committed grave
abuse of discretion by denying the Motion to Dismiss and holding that her office
had jurisdiction over the dispute. The CA reversed and set aside the Labor

3E Andaya Ching Espiritu Hefti Galvez Gammad Lainez Lui Madamba Nagera Narvasa
Ong Palangdao Rosales Sanchez Santos Satrain Tabo (2014-2015)

Labor Relations Case Digest - Atty. Joyrich Golangco

Arbiters Order denying the Motion to Dismiss and ruled that Locsin was a
corporate officer. Citing PD 902-A, the CA defined corporate officers as those
officers of a corporation who are given that character either by the Corporation
Code or by the corporations by-laws. The CA held that petitioners successfully
discharged their onus of establishing that private respondent was a corporate
officer who held the position EVP/Treasurer as provided in the by-laws of
Petitioner Corporation and that he held such position by virtue of election by the
Board of Directors. Article 280 of the Labor Code, the receipt of salaries by
Locsin, SSS deductions on that salary, and the element of control in the
performance of work duties indicia used by the Labor Arbiter to conclude that
Locsin was a regular employee were held inapplicable by the CA. Further, the
CA pointed out Locsins failure to state any circumstance by which NCLPI
engaged his services as a corporate officer that would make him an employee.
Locsin filed the present petition. He essentially submits that the NCLPI
wrongfully filed a petition for certiorari before the CA, as the latters remedy is to
proceed with the arbitration, and to appeal to the NLRC after the Labor Arbiter
shall have ruled on the merits of the case, as cited on Rule V, Section 6 of the
Revised Rules of the NLRC. And even if the Labor Arbiter committed grave abuse
of discretion in denying the NCLPI motion, a special civil action for certiorari,
filed with CA was not the appropriate remedy, since this was a breach of the
doctrine of exhaustion of administrative remedies. Locsin submits that he is a
regular employee of NCLPI. First, Locsin contends that NCLPI had the power to
engage his services as EVP/Treasurer. Second, he received regular wages from
NCLPI, from which his SSS and Philhealth contributions, as well as his
withholding taxes were deducted. Third, NCLPI had the power to terminate his
employment. Lastly, Nissan had control over the manner of the performance of
his functions as EVP/Treasurer, as shown by the 13 years of faithful execution of
his job, which he carried out in accordance with the standards and expectations
set by NCLPI. NCLPI submits that the CA correctly ruled that the Labor Arbiter
does not have jurisdiction over Locsins complaint for illegal dismissal. Rule VI,
Section 2(1) of the NLRC does not apply since only appealable decisions,
resolutions and orders are covered under the rule.
ISSUES: Whether the CA has original jurisdiction to review decision of the Labor
Arbiter under Rule 65.

HELD: Prefatorily, we agree with Locsins submission that the NCLPI incorrectly
elevated the Labor Arbiters denial of the Motion to Dismiss to the CA. Locsin is
correct in positing that the denial of a motion to dismiss is unappeasable. As a
general rule, an aggrieved partys proper recourse to the denial is to file his
position paper, interpose the grounds relied upon in the motion to dismiss before
the labor arbiter, and actively participate in the proceedings. Thereafter, the labor
arbiters decision can be appealed to the NLRC, not to the CA. As a rule, we
strictly adhere to the rules of procedure and do everything we can, to the point of
penalizing violators, to encourage respect for these rules. We take exception to
this general rule, however, when a strict implementation of these rules would
cause substantial injustice to the parties. In the context of this case, we see
sufficient justification to rule on the employer-employee relationship issue raised
by NCLPI, even though the Labor Arbiters interlocutory order
was incorrectly brought to the CA under Rule 65. Art 223. Of the Labor Code:
Decisions, awards, or orders of the Labor Arbiter are final and executory
unless appealed to the Commission by any or both parties within ten (10)
calendar days from receipt of such decisions, awards, or orders. Such appeal may
be entertained only on any of the following grounds: (a) If there is prima
facie evidence of abuse of discretion on the part of the Labor Arbiter; x x x.
In Air Services Cooperative, et al. v.The Court of Appeals, et al., a case where the
jurisdiction of the labor arbiter was put in issue and was assailed through a
petition for certiorari, prohibition and annulment of judgment before a regional
trial court, this Court had the opportunity to expound on the nature of appeal as
embodied in Article 223 of the Labor Code. Abuse of discretion is admittedly
within the ambit of certiorari and its grant of review thereof to the NLRC
indicates the lawmakers intention to broaden the meaning of appeal as that term
is used in the Code. For this reason, petitioners cannot argue now that the NLRC
is devoid of any corrective power to rectify a supposed erroneous assumption of
jurisdiction by the Labor Arbiter x x x. the CA clearly erred in the application
of the procedural rules by disregarding the relevant provisions of the NLRC
Rules, as well as the requirements for a petition for certiorari under the
Rules of Court. To reiterate, the proper action of an aggrieved party faced with
the labor arbiters denial of his motion to dismiss is to submit his position paper
and raise therein the supposed lack of jurisdiction. The aggrieved party cannot
immediately appeal the denial since it is an interlocutory order; the appropriate
remedial recourse is the procedure outlined in Article 223 of the Labor Code, not

3E Andaya Ching Espiritu Hefti Galvez Gammad Lainez Lui Madamba Nagera Narvasa
Ong Palangdao Rosales Sanchez Santos Satrain Tabo (2014-2015)

Labor Relations Case Digest - Atty. Joyrich Golangco

a petition for certiorari under Rule 65. But a strict implementation of the
NLRC Rules and the Rules of Court would cause injustice to the parties
because the Labor Arbiter clearly has no jurisdiction over the present intracorporate dispute. Due to existing exceptional circumstances, the ruling on
the merits that Locsin is an officer and not an employee of Nissan must take
precedence over procedural considerations. We have to give precedence to
the merits of the case, and primacy to the element of jurisdiction. Jurisdiction is
the power to hear and rule on a case and is the threshold element that must exist
before any quasi-judicial officer can act. In the context of the present case, the
Labor Arbiter does not have jurisdiction over the termination dispute Locsin
brought, and should not be allowed to continue to act on the case after the
absence of jurisdiction has become obvious, based on the records and the law. In
more practical terms, a contrary ruling will only cause substantial delay and
inconvenience as well as unnecessary expenses, to the point of injustice, to the
parties. This conclusion, of course, does not go into the merits of termination of
relationship and is without prejudice to the filing of an intra-corporate dispute on
this point before the appropriate RTC.
18; Oscar C. Reyes v. Hon. RTC of Makati

FACTS: Oscar and private respondent Rodrigo C. Reyes (Rodrigo) are two of the
four children of the spouses Pedro and Anastacia Reyes. Pedro, Anastacia, Oscar,
and Rodrigo each owned shares of stock of Zenith Insurance Corporation
(Zenith), a domestic corporation established by their family. Pedro died in 1964,
while Anastacia died in 1993. Although Pedros estate was judicially partitioned
among his heirs sometime in the 1970s, no similar settlement and partition
appear to have been made with Anastacias estate, which included her
shareholdings in Zenith. As of June 30, 1990, Anastacia owned 136,598 shares of
Zenith; Oscar and Rodrigo owned 8,715,637 and 4,250 shares, respectively.
Zenith and Rodrigo filed a complaint with the Securities and Exchange
Commission (SEC) against Oscar. It is a derivative suit initiated and filed by
Rodrigo C. Reyes to obtain an accounting of the funds and assets of Zenith
Insurance Corporation to determine the shares of stocks of deceased spouses
Pedro and Anastacia Reyes that were arbitrarily and fraudulently appropriated
by Oscar for himself which were not collated and taken into account in the
partition, distribution, and/or settlement of the estate of the deceased spouses,
for which he should be ordered to account for all the income from the time he

took these shares of stock, and should now deliver to his brothers and sisters
their just and respective shares. Oscar denied the charge that he illegally
acquired the shares of Anastacia Reyes. He asserted, as a defense, that he
purchased the subject shares with his own funds from the unissued stocks of
Zenith, and that the suit is not a bona fide derivative suit because the requisites
therefor have not been complied with. He thus questioned the SECs jurisdiction
to entertain the complaint because it pertains to the settlement of the estate of
Anastacia Reyes. When Republic Act (R.A.) No. 8799 took effect, the SECs
exclusive and original jurisdiction over cases enumerated in Section 5 of
Presidential Decree (P.D.) No. 902-A was transferred to the RTC designated as a
special commercial court.8 The records of Rodrigos SEC case were thus turned
over to the RTC, Branch 142, Makati, and docketed as Civil Case No. 00-1553.
Oscar filed a Motion to Declare Complaint as Nuisance or Harassment Suit. He
claimed that the complaint is a mere nuisance or harassment suit and should,
according to the Interim Rules of Procedure for Intra-Corporate Controversies,
be dismissed; and that it is not a bona fide derivative suit as it partakes of the
nature of a petition for the settlement of estate of the deceased Anastacia that is
outside the jurisdiction of a special commercial court. RTC denied the motion
in part and declared: A close reading of the Complaint disclosed the presence
of two (2) causes of action, namely: a) a derivative suit for accounting of the
funds and assets of the corporation which are in the control, custody, and/or
possession of the respondent [herein petitioner Oscar] with prayer to appoint a
management committee; and b) an action for determination of the shares of
stock of deceased spouses Pedro and Anastacia Reyes allegedly taken by
respondent, its accounting and the corresponding delivery of these shares to the
parties brothers and sisters. The latter is not a derivative suit and should
properly be threshed out in a petition for settlement of estate. Oscar thereupon
went to the CA on a petition for certiorari, prohibition, and mandamus and
prayed that the RTC Order be annulled and set aside and that the trial court
be prohibited from continuing with the proceedings. The appellate court
affirmed the RTC Order and denied the petition in its Decision dated May
26, 2004. It likewise denied Oscars motion for reconsideration in a
Resolution dated October 21, 2004.

3E Andaya Ching Espiritu Hefti Galvez Gammad Lainez Lui Madamba Nagera Narvasa
Ong Palangdao Rosales Sanchez Santos Satrain Tabo (2014-2015)

Labor Relations Case Digest - Atty. Joyrich Golangco

ISSUES: Whether the complaint of Rodrigo involves an Intra-Corporate


Controversies and is within the jurisdiction of the RTC acting as a special
commercial court.

HELD: To resolve it, we rely on the judicial principle that "jurisdiction over the
subject matter of a case is conferred by law and is determined by the allegations
of the complaint, irrespective of whether the plaintiff is entitled to all or some of
the claims asserted therein." P.D. No. 902-A enumerates the cases over which the
SEC (now the RTC acting as a special commercial court) exercises exclusive
jurisdiction. The allegations set forth in Rodrigos complaint principally invoke
Section 5, paragraphs (a) and (b) as basis for the exercise of the RTCs special
court jurisdiction. a) Devices or schemes employed by or any acts of the board of
directors, business associates, its officers or partners, amounting to fraud and
misrepresentation which may be detrimental to the interest of the public and/or
of the stockholders, partners, members of associations or organizations
registered with the Commission. b) Controversies arising out of intra-corporate
or partnership relations, between and among stockholders, members, or
associates; between any or all of them and the corporation, partnership or
association of which they are stockholders, members, or associates, respectively;
and between such corporation, partnership or association and the State insofar
as it concerns their individual franchise or right to exist as such entity. In an
attempt to hold Oscar responsible for corporate fraud, Rodrigo alleged in the
complaint the following: to determine the shares of stock of the deceased
spouses Pedro and Anastacia Reyes that were arbitrarily and fraudulently
appropriated for himself [herein petitioner Oscar]; Respondent Oscar C.
Reyes, through other schemes of fraud including misrepresentation,
unilaterally, and for his own benefit, capriciously transferred and took
possession and control of the management of Zenith Insurance
Corporation. Allegations of deceit, machination, false pretenses,
misrepresentation, and threats are largely conclusions of law that, without
supporting statements of the facts to which the allegations of fraud refer, do not
sufficiently state an effective cause of action. The late Justice Jose Feria, a noted
authority in Remedial Law, declared that fraud and mistake are required to be
averred with particularity in order to enable the opposing party to controvert
the particular facts allegedly constituting such fraud or mistake. Tested against
these standards, we find that the charges of fraud against Oscar were not

properly supported by the required factual allegations. While the complaint


contained allegations of fraud purportedly committed by him, these allegations
are not particular enough to bring the controversy within the special commercial
courts jurisdiction; they are not statements of ultimate facts, but are mere
conclusions of law: how and why the alleged appropriation of shares can be
characterized as "illegal and fraudulent" were not explained nor elaborated on.
Not every allegation of fraud done in a corporate setting or perpetrated by
corporate officers will bring the case within the special commercial courts
jurisdiction. No corporate power or office was alleged to have facilitated the
transfer of the shares; rather, Oscar, as an individual and without reference to his
corporate personality, was alleged to have transferred the shares of Anastacia to
his name, allowing him to become the majority and controlling stockholder of
Zenith, and eventually, the corporations President. Regretfully, we cannot read
into the complaint any specifically alleged corporate fraud that will call for the
exercise of the courts special commercial jurisdiction. Thus, we cannot affirm
the RTCs assumption of jurisdiction over Rodrigos complaint on the basis of
Section 5(a) of P.D. No. 902-A. To determine whether a case involves an intracorporate controversy, and is to be heard and decided by the branches of the RTC
specifically designated by the Court to try and decide such cases, two elements
must concur: (a) the status or relationship of the parties; and (2) the nature of
the question that is the subject of their controversy. What is material in
resolving the issues of this case under the allegations of the complaint is
Rodrigos interest as an heir since the subject matter of the present controversy
centers on the shares of stocks belonging to Anastacia, not on Rodrigos
personally-owned shares or on his personality as shareholder owning these
shares. In this light, all reference to shares of stocks in this case shall pertain to
the shareholdings of the deceased Anastacia and the parties interest therein as
her heirs. We find that insofar as the subject shares of stock (i.e., Anastacias
shares) are concerned Rodrigo cannot be considered a stockholder of Zenith.
Consequently, we cannot declare that an intra-corporate relationship exists that
would serve as basis to bring this case within the special commercial courts
jurisdiction under Section 5(b) of PD 902-A, as amended. Rodrigos complaint,
therefore, fails the relationship test. In the application of the Controversy
Test: The body rather than the title of the complaint determines the nature of an
action. Our examination of the complaint yields the conclusion that, more than
anything else, the complaint is about the protection and enforcement of

3E Andaya Ching Espiritu Hefti Galvez Gammad Lainez Lui Madamba Nagera Narvasa
Ong Palangdao Rosales Sanchez Santos Satrain Tabo (2014-2015)

Labor Relations Case Digest - Atty. Joyrich Golangco

successional rights. The controversy it presents is purely civil rather than


corporate, although it is denominated as a "complaint for accounting of all
corporate funds and assets." we hold that the nature of the present controversy
is not one which may be classified as an intra-corporate dispute and is beyond
the jurisdiction of the special commercial court to resolve. In short, Rodrigos
complaint also fails the nature of the controversy test. The RTC sitting as
special commercial court has no jurisdiction to hear Rodrigos complaint since
what is involved is the determination and distribution of successional rights to
the shareholdings of Anastacia Reyes. Rodrigos proper remedy, under the
circumstances, is to institute a special proceeding for the settlement of the estate
of the deceased Anastacia Reyes, a move that is not foreclosed by the dismissal of
his present complaint.
19. Okoy vs. Slimmers World Intl., Behavior Modification Inc. and Ronald
Joseph Moy

FACTS: Respondent Slimmers World International operating under the name


Behavior Modifications, Inc. (Slimmers World) employed petitioner Leslie Okol
(Okol) as a management trainee on 15 June 1992. She rose up the ranks to
become Head Office Manager and then Director and Vice President from 1996
until her dismissal on 22 September 1999. Prior to Okols dismissal, Slimmers
World preventively suspended Okol. The suspension arose from the seizure by
the Bureau of Customs of seven Precor elliptical machines and seven Precor
treadmills belonging to or consigned to Slimmers World. The shipment of the
equipment was placed under the names of Okol and two customs brokers for a
value less than US$500. For being undervalued, the equipment were seized.
Okol received a memorandum that her suspension had been extended
from 2 September until 1 October 1999 pending the outcome of the investigation
on the Precor equipment importation. On 17 September 1999, Okol received
another memorandum from Slimmers World requiring her to explain why no
disciplinary action should be taken against her in connection with the equipment
seized by the Bureau of Customs. Okol filed her written explanation. However,
Slimmers World found Okols explanation to be unsatisfactory so Slimmers
World terminated Okols employment.
Okol filed a complaint with the Arbitration branch of the NLRC against
Slimmers World, Behavior Modifications, Inc. and Moy (collectively called

respondents) for illegal suspension, illegal dismissal, unpaid commissions,


damages and attorneys fees, with prayer for reinstatement and payment of
Back wages.
On 22 February 2000, respondents filed a Motion to Dismiss the case
with a reservation of their right to file a Position Paper at the proper time.
Respondents asserted that the NLRC had no jurisdiction over the subject matter
of the complaint. The labor arbiter granted the motion to dismiss. The labor
arbiter ruled that Okol was the vice-president of Slimmers World at the time of
her dismissal. Since it involved a corporate officer, the dispute was an intracorporate controversy falling outside the jurisdiction of the Arbitration branch.
Okol filed an appeal with the NLRC. The NLRC reversed and set aside the labor
arbiters order. It then ordered respondent Behavior Modification, Inc./Slimmers
World International to reinstate complainant Leslie F. Okol to her former
position with full back wages plus indemnity pay and attorneys fee and should
reinstatement be not feasible separation pay equivalent to one month pay per
year of service is awarded, a fraction of at least six months considered one whole
year. Respondents filed a Motion for Reconsideration with the NLRC but the
latter denied the motion for lack of merit. They then filed an appeal with the
Court of Appeals, which set aside the NLRCs decision ruling that the case, being
an intra-corporate dispute, falls within the jurisdiction of regular courts, as
provided under R.A. 8799. The NLRC had acted without jurisdiction in giving due
course to the complaint and deprived respondents of their right to due process in
deciding the case on the merits.
ISSUES: W/N the petitioner was an employee or corporate officer of Slimmers
World; Consequently, W/N NLRC has jurisdiction over the illegal dismissal case
filed by the petitioner

HELD: Section 25 of the Corporation Code enumerates corporate officers as the


president, secretary, treasurer and such other officers as may be provided for in
the by-laws. An office is created by the charter of the corporation and the
officer is elected by the directors or stockholders. On the other hand, an
employee usually occupies no office and generally is employed not by action of
the directors or stockholders but by the managing officer of the corporation who
also determines the compensation to be paid to such employee.

3E Andaya Ching Espiritu Hefti Galvez Gammad Lainez Lui Madamba Nagera Narvasa
Ong Palangdao Rosales Sanchez Santos Satrain Tabo (2014-2015)

Labor Relations Case Digest - Atty. Joyrich Golangco

In the present case, the respondents, in their motion to dismiss filed


before the labor arbiter, questioned the jurisdiction of the NLRC in taking
cognizance of petitioners complaint. In the motion, respondents attached the
General Information Sheet (GIS) dated 14 April 1998, Minutes of the meeting of
the Board of Directors dated 14 April 1997 and Secretarys Certificate, and the
Amended By-Laws dated 1 August 1994 of Slimmers World as submitted to the
SEC to show that petitioner was a corporate officer whose rights do not fall
within the NLRCs jurisdiction. The GIS and minutes of the meeting of the board
of directors indicated that petitioner was a member of the board of directors,
holding one subscribed share of the capital stock, and an elected corporate
officer. From the documents submitted by respondents, petitioner was a director
and officer of Slimmers World. The charges of illegal suspension, illegal
dismissal, unpaid commissions, reinstatement and back wages imputed by
petitioner against respondents fall squarely within the ambit of intra-corporate
disputes. A corporate officers dismissal is always a corporate act, or an intracorporate controversy which arises between a stockholder and a corporation.
The question of remuneration involving a stockholder and officer, not a mere
employee, is not a simple labor problem but a matter that comes within the area
of corporate affairs and management and is a corporate controversy in
contemplation of the Corporation Code. It is a settled rule that jurisdiction over
the subject matter is conferred by law. The determination of the rights of a
director and corporate officer dismissed from his employment as well as the
corresponding liability of a corporation, if any, is an intra-corporate dispute
subject to the jurisdiction of the regular courts. Thus, the appellate court
correctly ruled that it is not the NLRC but the regular courts which have
jurisdiction over the present case.
20. RURAL BANK OF CORON (PALAWAN), INC., et al. vs. ANNALISA CORTES

FACTS: Virgilio Garcia, founder of petitioner corporations (the corporations),


hired the then still single Annalisa Cortes (respondent) as clerk of the Rural Bank
of Coron (Manila Office). After Virgilio died, his son Victor took over the
management of the corporations. Anita Cortes (Anita), the wife of Victor Garcia,
was also involved in the management of the corporations. Respondent later
married Anitas brother Eduardo Cortes.

Anita soon assumed the position of Vice President of petitioner Citizens


Development Incorporated (CDI) and practically controlled the financial
operations of almost all of the other corporations in the course of which she
allowed some of her relatives and in-laws, including respondent, to hold several
key sensitive positions thereat. Respondent later became the Financial Assistant,
Personnel Officer and Corporate Secretary of The Rural Bank of Coron, Personnel
Officer of CDI, and also Personnel Officer and Disbursing Officer of The Empire
Cold Storage Development Corporation (ECSDC). She simultaneously received
salaries from these corporations. On examination of the financial books of the
corporations by petitioner Sandra Garcia Escat, a daughter of Virgilio Garcia who
was previously residing in Spain, she found out
that respondent was involved
in several anomalies, drawing petitioners to terminate respondents services on
November 23, 1998 in petitioner corporations. Respondents counsel conveyed
respondents willingness to abide by the decision to terminate her but reminded
them that she was entitled to separation pay equivalent to 11 months salary as
well as to the other benefits provided by law in her favor. Respondents counsel
thus demanded the payment of respondents unpaid salary for the months of
October and November 1998, separation pay equivalent to 12 months salary,
13th month pay and other benefits. As the demand remained unheeded,
respondent filed a complaint for illegal dismissal and non-payment of salaries
and other benefits.
Petitioners moved for the dismissal of the complaint on the ground of lack
of jurisdiction, contending that the case was an intra-corporate controversy
involving the removal of a corporate officer, respondent being the Corporate
Secretary of the Rural Bank of Coron, Inc., hence, cognizable by the Securities and
Exchange Commission (SEC).
In resolving the issue of jurisdiction, the Labor Arbiter noted that aside
from her being Corporate Secretary of Rural Bank of Coron, complainant was
likewise appointed as Financial Assistant & Personnel Officer of all respondents
herein. Verily, a Financial Assistant & Personnel Officer is not a Corporate Officer
of the [petitioners] corporation, thus, pursuant to Article 217 of the Labor Code,
as amended, the instant case falls within the ambit of original and exclusive
jurisdiction of this Office. Eventually, the Labor Arbiter found for
respondent, computing the monetary award due her and ordered respondents
jointly and severally pay complainant.

3E Andaya Ching Espiritu Hefti Galvez Gammad Lainez Lui Madamba Nagera Narvasa
Ong Palangdao Rosales Sanchez Santos Satrain Tabo (2014-2015)

Labor Relations Case Digest - Atty. Joyrich Golangco

On the tenth or last day of the period of appeal, petitioners filed their
appeal. By resolution, the NLRC, while noting that petitioners timely filed the
appeal, held that the same was not accompanied by an appeal bond, a mandatory
requirement under Article 223 of the Labor Code and Section 6, Rule VI of the
NLRC New Rules of Procedure. It also noted that the Motion for Reduction of
Bond was premised on self-serving allegations. It accordingly dismissed the
appeal. Petitioners Motion for Reconsideration was denied by the NLRC hence,
they filed a Petition for Certiorari before the Court of Appeals. The CA dismissed
the petition for lack of merit. Petitioners motion for reconsideration was also
denied.
ISSUES: W/N the labor arbiter had jurisdiction to hear the case

HELD: While, indeed, respondent was the Corporate Secretary of the Rural
Bank of Coron, she was also its Financial Assistant and the Personnel
Officer of the two other petitioner corporations. Mainland Construction Co.,
Inc. v. Movilla instructs that a corporation can engage its corporate officers
to perform services under a circumstance which would make them
employees. The Labor Arbiter has thus jurisdiction over respondents
complaint.
To reiterate, the appellate court did not err in dismissing the petition
before it. And contrary to petitioners assertion, the appellate court dismissed its
petition not on a mere technicality. For the non-posting of an appeal bond
within the reglementary period divests the NLRC of its jurisdiction to entertain
the appeal. Article 223, which prescribes the appeal bond requirement, is
a rule of jurisdiction and not of procedure. There is a little leeway for
condoning a liberal interpretation thereof, and certainly none premised on the
ground that its requirements are mere technicalities. It must be emphasized that
there is no inherent right to an appeal in a labor case, as it arises solely from
grant of statute, namely the Labor Code. The requirement for posting the
surety bond is not merely procedural but jurisdictional and cannot be trifled
with. Non-compliance with such legal requirements is fatal and has the effect of
rendering the judgment final and executory. The petitioners cannot be allowed to
seek refuge in a liberal application of rules for their act of negligence.
It bears emphasis that all that is required to perfect the appeal is the
posting of a bond to ensure that the award is eventually paid should the appeal

be dismissed. Petitioners should thus have posted a bond, even if it were only
partial, but they did not. No relaxation of the Rule may thus be considered.
21: Halaguena vs. PAL

FACTS: Petitioners were employed as female flight attendants of respondent


Philippine Airlines (PAL) on different dates prior to November 22, 1996. They
are members of the Flight Attendants and Stewards Association of the
Philippines (FASAP), a labor organization certified as the sole and exclusive
certified as the sole and exclusive bargaining representative of the flight
attendants, flight stewards and pursers of respondent. Respondent and FASAP
entered into a Collective Bargaining Agreement incorporating the terms and
conditions of their agreement for the years 2000 to 2005, hereinafter referred to
as PAL-FASAP CBA. Section 144, Part A of the PAL-FASAP CBA, provides for the
compulsory retirement of cabin attendants hired before Nov. 22, 1996 and that it
shall be fifty-five (55) for females and sixty (60) for males. Petitioners and
several female cabin crews manifested that the aforementioned CBA provision
on compulsory retirement is discriminatory, and demanded for an equal
treatment with their male counterparts. This demand was reiterated in a
letter by petitioners' counsel addressed to respondent demanding the removal of
gender discrimination provisions in the coming re-negotiations of the PALFASAP CBA. Robert D. Anduiza, President of FASAP submitted their 2004-2005
CBA proposals and manifested their willingness to commence the collective
bargaining negotiations between the management and the association, at the
soonest possible time. Petitioners then filed a Special Civil Action for Declaratory
Relief with Prayer for the Issuance of Temporary Restraining Order and Writ of
Preliminary Injunction with the Regional Trial Court (RTC) of Makati City. The
RTC then issued an Order upholding its jurisdiction over the present case. It
reasoned that the thrust of the Petition is Sec. 144 of the subject CBA which is
allegedly discriminatory as it discriminates against female flight attendants, in
violation of the Constitution, the Labor Code, and the CEDAW. The allegations in
the Petition do not make out a labor dispute arising from employer-employee
relationship as none is shown to exist. This case is not directed specifically
against respondent arising from any act of the latter, nor does it involve a claim
against the respondent. Rather, this case seeks a declaration of the nullity of the
questioned provision of the CBA, which is within the Court's competence, with

3E Andaya Ching Espiritu Hefti Galvez Gammad Lainez Lui Madamba Nagera Narvasa
Ong Palangdao Rosales Sanchez Santos Satrain Tabo (2014-2015)

Labor Relations Case Digest - Atty. Joyrich Golangco

the allegations in the Petition constituting the bases for such relief sought. The
RTC found for the petitioner. Aggrieved, respondent, filed a petition with the CA
praying that the order of the RTC, which denied its objection to its jurisdiction,
be annuled and set aside for having been issued without and/or with grave abuse
of discretion amounting to lack of jurisdiction.
The CA declared the RTC to have NO JURISDICTION OVER THE CASE.
ISSUES: W/N the RTC has jurisdiction over the petitioners' action
challenging the legality or constitutionality of the provisions on the
compulsory retirement age contained in the CBA between respondent
PAL and FASAP.

HELD: Petitioners submit that the RTC has jurisdiction in all civil actions in
which the subject of the litigation is incapable of pecuniary estimation and in all
cases not within the exclusive jurisdiction of any court, tribunal, person or body
exercising judicial or quasi-judicial functions. The RTC has the power to
adjudicate all controversies except those expressly withheld from the plenary
powers of the court. Accordingly, it has the power to decide issues of
constitutionality or legality of the provisions of Section 144, Part A of the PALFASAP CBA. As the issue involved is constitutional in character, the labor arbiter
or the National Labor Relations Commission (NLRC) has no jurisdiction over the
case and, thus, the petitioners pray that judgment be rendered on the merits
declaring Section 144, Part A of the PAL-FASAP CBA null and void.
Respondent, on the other hand, alleges that the labor tribunals have
jurisdiction over the present case, as the controversy partakes of a labor dispute.
The dispute concerns the terms and conditions of petitioners' employment in
PAL, specifically their retirement age. The RTC has no jurisdiction over the
subject matter of petitioners' petition for declaratory relief because the
Voluntary Arbitrator or panel of Voluntary Arbitrators have original and
exclusive jurisdiction to hear and decide all unresolved grievances arising from
the interpretation or implementation of the CBA. Regular courts have no power
to set and fix the terms and conditions of employment. Finally, respondent
alleged that petitioners' prayer before this Court to resolve their petition for
declaratory relief on the merits is procedurally improper and baseless.
Petitioner is correct. Jurisdiction of the court is determined on the basis of
the material allegations of the complaint and the character of the relief prayed

for irrespective of whether plaintiff is entitled to such relief. The allegations in


the petition for declaratory relief plainly show that petitioners' cause of action is
the annulment of Section 144, Part A of the PAL-FASAP CBA. From the
petitioners' allegations and relief prayed for in its petition, it is clear that the
issue raised is whether Section 144, Part A of the PAL-FASAP CBA is unlawful and
unconstitutional. The petitioners' primary relief is the annulment of Section 144,
Part A of the PAL-FASAP CBA, which allegedly discriminates against them for
being female flight attendants. The subject of litigation is incapable of pecuniary
estimation, exclusively cognizable by the RTC. Being an ordinary civil action, the
same is beyond the jurisdiction of labor tribunals.
The said issue cannot be resolved solely by applying the Labor Code.
Rather, it requires the application of the Constitution, labor statutes, law on
contracts and the Convention on the Elimination of All Forms of Discrimination
Against Women, and the power to apply and interpret the constitution and
CEDAW is within the jurisdiction of trial courts, a court of general jurisdiction.
Not every dispute between an employer and employee involves matters that only
labor arbiters and the NLRC can resolve in the exercise of their adjudicatory or
quasi-judicial powers. The jurisdiction of labor arbiters and the NLRC under
Article 217 of the Labor Code is limited to disputes arising from an employeremployee relationship which can only be resolved by reference to the Labor Code,
other labor statutes, or their collective bargaining agreement.
Not every controversy or money claim by an employee against the
employer or vice-versa is within the exclusive jurisdiction of the labor arbiter.
Actions between employees and employer where the employer-employee
relationship is merely incidental and the cause of action precedes from a
different source of obligation is within the exclusive jurisdiction of the regular
court. Here, the employer-employee relationship between the parties is merely
incidental and the cause of action ultimately arose from different sources of
obligation, i.e., the Constitution and CEDAW.
22: Santiago vs. CF SHARP CREW MANAGEMENT, INC.

FACTS: Petitioner Santiago had been working as a seafarer for Smith Bell
Management, Inc. (respondent) for about 5 years. He had signed a new contract
of employment with respondent, with the duration of nine (9) months. Sais
contract was approved by the POEA. Petitioner was to be deployed on board the

3E Andaya Ching Espiritu Hefti Galvez Gammad Lainez Lui Madamba Nagera Narvasa
Ong Palangdao Rosales Sanchez Santos Satrain Tabo (2014-2015)

Labor Relations Case Digest - Atty. Joyrich Golangco

"MSV Seaspread" which was scheduled to leave the port of Manila for Canada on
13 February 1998. However, a week before the scheduled date of departure,
Capt. Pacifico Fernandez, respondents Vice President, sent a facsimile message
to the captain of "MSV Seaspread," from the wife of Santiago asking him not to
send her husband to MSV Seaspread anymore. He was also informed
anonymously that Santiago, if allowed to depart will jump ship in Canada like his
brother Christopher Santiago, O/S who jumped ship from the C.S. Thus, the
captain cancelled plans for Santiagos departure. Petitioner was thus told that he
would not be leaving for Canada anymore, but he was reassured that he might be
considered for deployment at some future date. Petitioner filed a complaint for
illegal dismissal, damages, and attorney's fees against respondent and its foreign
principal, Cable and Wireless (Marine) Ltd. The case was raffled to Labor Arbiter
Teresita Castillon-Lora, who ruled that the employment contract remained valid
but had not commenced since petitioner was not deployed. According to her,
respondent violated the rules and regulations governing overseas employment
when it did not deploy petitioner, causing petitioner to suffer actual damages
representing lost salary income for nine (9) months and fixed overtime fee.
On appeal by respondent, the NLRC ruled that there is no employeremployee relationship between petitioner and respondent because under the
Standard Terms and Conditions Governing the Employment of Filipino Seafarers
on Board Ocean Going Vessels (POEA Standard Contract), the employment
contract shall commence upon actual departure of the seafarer from the airport
or seaport at the point of hire and with a POEA-approved contract. In the absence
of an employer-employee relationship between the parties, the claims for illegal
dismissal, actual damages, and attorneys fees should be dismissed. On the other
hand, the NLRC found respondents decision not to deploy petitioner to be a valid
exercise of its management prerogative.
Petitioner moved for the reconsideration of the NLRCs Decision but his
motion was denied for lack of merit. He elevated the case to the Court of Appeals
through a petition for certiorari. The CA noted that there is an ambiguity in the
NLRCs Decision when it affirmed with modification the labor arbiters Decision,
because by the very modification introduced by the Commission (vacating the
award of actual damages and attorneys fees), there is nothing more left in the
labor arbiters Decision to affirm. According to the appellate court, petitioner is
not entitled to actual damages because damages are not recoverable by a worker
who was not deployed by his agency within the period prescribed in the POEA

Rules. It agreed with the NLRCs finding that petitioners non-deployment was a
valid exercise of respondents management prerogative. It added that since
petitioner had not departed from the Port of Manila, no employer-employee
relationship between the parties arose and any claim for damages against the socalled employer could have no leg to stand on.
ISSUES: W/N the seafarer, who was prevented from leaving the port of Manila
and refused deployment without valid reason but whose POEA-approved
employment contract provides that the employer-employee relationship shall
commence only upon the seafarers actual departure from the port in the point of
hire, is entitled to relief?

HELD: Petitioner maintains that respondent violated the Migrant Workers Act
and the POEA Rules when it failed to deploy him within thirty (30) calendar days
without a valid reason. In doing so, it had unilaterally and arbitrarily prevented
the consummation of the POEA- approved contract. Since it prevented his
deployment without valid basis, said deployment being a condition to the
consummation of the POEA contract, the contract is deemed consummated, and
therefore he should be awarded actual damages, consisting of the stipulated
salary and fixed overtime pay. Petitioner adds that since the contract is deemed
consummated, he should be considered an employee for all intents and purposes,
and thus the labor arbiter and/or the NLRC has jurisdiction to take cognizance of
his claims. Petitioner additionally claims that he should be considered a regular
employee, having worked for five (5) years on board the same vessel owned by
the same principal and manned by the same local agent. He argues that
respondents act of not deploying him was a scheme designed to prevent him
from attaining the status of a regular employee. Petitioner submits that
respondent had no valid and sufficient cause to abandon the employment
contract, as it merely relied upon alleged phone calls from his wife and other
unnamed callers in arriving at the conclusion that he would jump ship like his
brother.
On the other hand, respondent argues that the Labor Arbiter has no
jurisdiction to award petitioners monetary claims. His employment with
respondent did not commence because his deployment was withheld for a valid
reason. Consequently, the labor arbiter and/or the NLRC cannot entertain
adjudication of petitioners case much less award damages to him. The

3E Andaya Ching Espiritu Hefti Galvez Gammad Lainez Lui Madamba Nagera Narvasa
Ong Palangdao Rosales Sanchez Santos Satrain Tabo (2014-2015)

Labor Relations Case Digest - Atty. Joyrich Golangco

controversy involves a breach of contractual obligations and as such is


cognizable by civil courts.
There is no question that the parties entered into an employment
contract on 3 February 1998, whereby petitioner was contracted by respondent
to render services on board "MSV Seaspread" for the consideration of US$515.00
per month for nine (9) months, plus overtime pay. However, respondent failed to
deploy petitioner from the port of Manila to Canada. Considering that petitioner
was not able to depart from the airport or seaport in the point of hire, the
employment contract did not commence, and no employer-employee
relationship was created between the parties. However, a distinction must be
made between the perfection of the employment contract and the
commencement of the employer-employee relationship.
The perfection of the contract, which in this case coincided with the date
of execution thereof, occurred when petitioner and respondent agreed on the
object and the cause, as well as the rest of the terms and conditions therein. The
commencement of the employer-employee relationship, as earlier discussed,
would have taken place had petitioner been actually deployed from the point of
hire. Thus, even before the start of any employer-employee relationship,
contemporaneous with the perfection of the employment contract was the birth
of certain rights and obligations, the breach of which may give rise to a cause of
action against the erring party. Thus, if the reverse had happened, that is the
seafarer failed or refused to be deployed as agreed upon, he would be liable for
damages.
Moreover, while the POEA Standard Contract must be recognized and
respected, neither the manning agent nor the employer can simply prevent a
seafarer from being deployed without a valid reason. Respondents act of
preventing petitioner from departing the port of Manila and boarding "MSV
Seaspread" constitutes a breach of contract, giving rise to petitioners cause of
action. Respondent unilaterally and unreasonably reneged on its obligation to
deploy petitioner and must therefore answer for the actual damages he suffered.
Despite the absence of an employer-employee relationship between
petitioner and respondent, the Court rules that the NLRC has jurisdiction over
petitioners complaint. The jurisdiction of labor arbiters is not limited to claims
arising from employer-employee relationships as provided under Section 10 of
R.A. No. 8042 (Migrant Workers Act). Since the present petition involves the

employment contract entered into by petitioner for overseas employment, his


claims are cognizable by the labor arbiters of the NLRC.
23. Atlas Farms Inc. v. NLRC Nov. 18, 2002

FACTS: Private respondent Jaime O. dela Pea was employed as a veterinary aide
by petitioner Atlas Farms, Inc., in December 1975. On March 3, 1993, Pea was
allegedly caught urinating and defecating on company premises not intended for
the purpose. The farm manager of petitioner issued a formal notice directing him
to explain within 24 hours why disciplinary action should not be taken against
him for violating company rules and regulations. Pea refused, however, to
receive the formal notice. On March 20, 1993, a notice of termination with
payment of his monetary benefits was sent to him. He duly acknowledged receipt
of his separation pay of P13, 918.67. Correspondent Martial I. Abion was a
carpenter/mason and a maintenance man whose employment by petitioner
commenced on October 8, 1990. He allegedly caused the clogging of the fishpond
drainage resulting in damages worth several hundred thousand pesos when he
improperly disposed of the cut grass and other waste materials into the pond's
drainage system. Petitioner sent a written notice to Abion, requiring him to
explain what happened, otherwise, disciplinary action would be taken against
him. He refused to receive the notice and give an explanation. The company
terminated his services on October 27, 1992. He acknowledged receipt of a
written notice of dismissal, with his separation pay.
Thereafter, Pea and Abion filed separate complaints for illegal dismissal that
were later consolidated. Both claimed that their termination from the service
was due to petitioner's suspicion that they were the leaders in a plan to form a
union to compete and replace the existing
Management dominated union. The labor arbiter dismissed their complaints on
the ground that the grievance machinery in the collective bargaining agreement
(CBA) had not yet been exhausted. Private respondents availed of the grievance
process, but later on refilled the case before the NLRC in Region IV. They alleged
"lack of sympathy" on petitioner's part to engage in conciliation proceedings.
Their cases were consolidated in the NLRC. At the initial mandatory conference,
petitioner filed

3E Andaya Ching Espiritu Hefti Galvez Gammad Lainez Lui Madamba Nagera Narvasa
Ong Palangdao Rosales Sanchez Santos Satrain Tabo (2014-2015)

Labor Relations Case Digest - Atty. Joyrich Golangco

a motion to dismiss on the ground of lack of jurisdiction, alleging private


respondents themselves admitted that they were members of the employees'
union with which petitioner had an existing CBA. According to petitioner,
jurisdiction over the case belonged to the grievance machinery and thereafter
the voluntary arbitrator, as provided in the CBA. The labor arbiter dismissed the
complaint for lack of merit, finding that the case was one of illegal dismissal and
did not involve the interpretation or implementation of any CBA provision.
Private respondents appealed to the NLRC, which reversed the labor arbiter's
decision. Dissatisfied with the NLRC ruling, petitioner went to the Court of
Appeals by way of a petition for certiorari under Rule 65, seeking reinstatement
of the labor arbiter's decision. The appellate court denied the petition and
affirmed the NLRC resolution. Hence, the present petition
.
ISSUES:
1. Whether private respondents were legally and validly dismissed
2. Whether the labor arbiter and the NLRC had jurisdiction to decide complaints
for illegal dismissal

Held:
1. No. Private respondents were illegally dismissed. Coming to the merits of the
petition, the NLRC found that petitioner did not comply with the requirements of
a valid dismissal. For a dismissal to be valid, the employer must show that: (1)
the employee was accorded due process, and (2) the dismissal must be for any of
the valid causes provided for by law.22 No evidence was shown that private
respondents refused, as alleged, to receive the notices requiring them to show
cause why no disciplinary action should be taken against them. Without proof of
notice, private respondents were subsequently dismissed without hearing.
2. Yes. The labor arbiter and then the NLRC had jurisdiction over the cases
involving private respondents dismissal, and no error was committed by the
appellate court in upholding their assumption of jurisdiction.
The General rule is that, under Article 217 of the Labor Code, labor arbiters have
original and exclusive jurisdiction over termination disputes. An exception to
that rule is found in Article 261 of the Labor Code, which provides that the
Voluntary Arbitrator or panel of voluntary arbitrators shall have original and
exclusive jurisdiction to hear and decide all unresolved grievances arising from

the interpretation or implementation of the CBA and those arising from the
interpretation or enforcement of company personnel policies referred to in the
immediately preceding article. Accordingly, violations of a CBA, except those
which are gross in character, shall no longer be treated as unfair labor practice
and shall be resolved as grievances under the Collective Bargaining Agreement.
For purposes of this article, gross violations of Collective Bargaining Agreement
shall mean flagrant and or malicious refusal to comply with the economic
provisions of such agreement. The Commission, its Regional Offices and the
Regional Directors of the Department of Labor and Employment shall not
entertain disputes, grievances or matters under the exclusive and original
jurisdiction of the Voluntary Arbitrator or panel of Voluntary Arbitrators and
shall immediately dispose and refer the same to the grievance Machinery or
Arbitration provided in the Collective Bargaining Agreement.

However, the instant case is a termination dispute falling under the original and
exclusive jurisdiction of the Labor Arbiter, and does not specifically involve the
application, implementation or enforcement of company personnel policies/CBA
since the private respondents were already dismissed from work without
hearing, also depriving them of a chance to air their side at the level of the
grievance machinery. Given the fact of dismissal, it can be said that the cases
were effectively removed from the jurisdiction of the voluntary arbitrator, thus
placing them within the jurisdiction of the labor arbiter.
Where the dispute is just in the interpretation, implementation or enforcement
stage, it may be referred to the grievance machinery set up in the CBA, or
brought to voluntary arbitration. But, where there was already actual
termination, with alleged violation of the employees rights, it is already
cognizable by the labor arbiter. Also, records show, however, that private
respondents sought without success to avail of the grievance procedure in their
CBA. On this point, petitioner maintains that by so doing, private respondents
recognized that their cases still fell under the grievance machinery. According to
petitioner, without having exhausted said machinery, the private respondents
filed their action before the NLRC, in a clear act of forum shopping.
However, it is worth pointing out that private respondents went to the NLRC
only after the labor arbiter dismissed their original complaint for illegal
dismissal. Given the circumstances, private respondents acted within their legal

3E Andaya Ching Espiritu Hefti Galvez Gammad Lainez Lui Madamba Nagera Narvasa
Ong Palangdao Rosales Sanchez Santos Satrain Tabo (2014-2015)

Labor Relations Case Digest - Atty. Joyrich Golangco

rights in finding another avenue for the redress of their grievances. The Court
also upheld the NLRC in concluding that private respondents had already
exhausted the remedies under the grievance procedure and in ruling that it was
petitioner who failed to show proof that it took steps to convene the grievance
machinery after the labor arbiter first dismissed the complaint for illegal
dismissal and directed the parties to avail of the grievance procedure under
Article VII of the existing CBA. Private respondents could not be faulted for
attempting to find an impartial forum, after petitioner failed to listen to them and
after the intercession of the labor arbiter proved futile.
24 Perpetual Help Credit Cooperative Inc., vs Faburada

FACTS: Private Respondents, Benedicto Faburada, Sisinita Vilar, Imelda Tamayo


and Harold Catipay, filed a complaint against the Perpetual Help Credit
Cooperative, Inc. (PHCCI), petitioner, with the Arbitration Branch, Department of
Labor and Employment (DOLE), Dumaguete City, for illegal dismissal, premium
pay on holidays and rest days, separation pay, wage differential, moral damages,
and attorney's fees.

Forthwith, petitioner PHCCI filed a motion to dismiss the complaint on the


ground that there is no employer-employee relationship between them as
private respondents are all members and co-owners of the cooperative. On
September 3, 1990, petitioner filed a supplemental motion to dismiss alleging
that Article 121 of R.A. No. 6939, otherwise known as the Cooperative
Development Authority Law which took effect on March 26, 1990, requires
conciliation or mediation within the cooperative before a resort to judicial
proceeding.

The Labor Arbiter denied the motion to dismiss and rendered a decision in favor
of the petitioners declaring them illegally dismissed which was affirmed by the
NLRC, Hence this petition by the PHCCI, they contended that:
1. There is no Employer-Employee relationship,
2. That the labor arbiter has no jurisdiction to take cognizance of the
complaint of private respondents considering that they failed to
submit their dispute to the grievance machinery as required by P.D.
175 (strengthening the Cooperative Movement) and its
implementing rules and regulations under LOI 23. Likewise, the

Cooperative Development Authority did not issue a Certificate of


Non-Resolution pursuant to Section 8 of R.A. 6939 or the
Cooperative Development Authority Law.

ISSUES: Whether or not, the LA has jurisdiction to take cognizance of the petition

HELD: Yes, As aptly stated by the Solicitor General in his comment, P.D. 175 does
not provide for a grievance machinery where a dispute or claim may first be
submitted. LOI 23 refers to instructions to the Secretary of Public Works and
Communications to implement immediately the recommendation of the
Postmaster General for the dismissal of some employees of the Bureau of Post.
Obviously, this LOI has no relevance to the instant case. Art 121 of Republic Act
No. 6938 (Cooperative Code of the Philippines) provides the procedure how
cooperative disputes are to be resolved, thus:
"ART. 121. Settlement of Disputes. Disputes among members, officers,
directors, and committee members, and intra-cooperative disputes shall, as far as
practicable, be settled amicably in accordance with the conciliation or mediation
mechanisms embodied in the bylaws of the cooperative, and in applicable laws.
Should such a conciliation/mediation proceeding fail, the matter shall be settled
in a court of competent jurisdiction."

Complementing this Article is Section 8 of R.A. No. 6939 (Cooperative


Development Authority Law) which reads:
SEC. 8 Mediation and Conciliation. Upon request of either or both
parties, the Authority shall mediate and conciliate disputes within a cooperative
or between cooperatives: Provided that if no mediation or conciliation succeeds
within three (3) months from request thereof, a certificate of non-resolution
shall be issued by the Commission prior to the filing of appropriate action before
the proper courts.
The above provisions apply to members, officers and directors of the cooperative
involved in disputes within a cooperative or between cooperatives. There is no
evidence that private respondents are members of petitioner PHCCI and even if
they are, the dispute is about payment of wages, overtime pay, rest day and
termination of employment. Under Art. 217 of the Labor Code, these disputes are
within the original and exclusive jurisdiction of the Labor Arbiter.

3E Andaya Ching Espiritu Hefti Galvez Gammad Lainez Lui Madamba Nagera Narvasa
Ong Palangdao Rosales Sanchez Santos Satrain Tabo (2014-2015)

Labor Relations Case Digest - Atty. Joyrich Golangco


25. Austria v. NLRC, August 16, 1999
FACTS: Pastor Dionisio Austria was dismissed from service by the Seventh Day
Adventist (SDA) for misappropriation of funds, willful breach of trust, serious
misconduct; gross and habitual neglect of duties and commission of an offense
against the person of employers duly authorized representative as grounds.
Pastor Austria began serving the religious corporation on 1963 until October
1991 when his services was terminated.

From August up to October 1991, petitioner received several communications


from Mr. Ibesate the Treasurer of the Negros Mission asking him to admit
accountability and responsibility for the church tithes and offerings collected by
his wife. He explained that it was Ibesate and Pastor Buhat, the president of the
said Mission who authorized his wife to collect since he was sick at that time. On
the other hand, Pastor Buhat and petitioner had a heated argument when Pastor
Rodrigo harbored ill feelings against the latter for helping out one Danny
Diamada for collecting the unpaid balance for the repair of Rodrigos motor
vehicle. Upon discovery that Rodrigo was about to file a complaint against him
with the Mission, Austria went to Buhats office to convene the Executive
Committee but he failed since there was no quorum. Austria banged the attach
case of Buhat, scattered the books and tried to overturn the table. Subsequently,
on October 29, 1991, the Executive Committee was convened and rendered the
dismissal of Austria.

The LA ruled in favor of the petitioner. NLRC vacated the findings of the LA for
want of merit, but it reinstated the decision of LA upon filing of the MR by
petitioner. Hence, on the ground that LA has no jurisdiction due to the
constitutional provision of the separation of church and state since the case
involved an ecclesiastical affair to which the state cannot interfere. The SDA filed
a motion for reconsideration contending, for the first time on appeal, that the
Labor Arbiter has no jurisdiction over the complaint filed by petitioner due to the
constitutional provision on the separation of church and state since the case
allegedly involved an ecclesiastical affair to which the State cannot interfere. The
NLRC without ruling on the merits of the case, reversed itself once again,

sustained the argument posed by private respondents and dismissed the


complaint of petitioner.

ISSUES:
1. Whether or not the Labor Arbiter/NLRC has jurisdiction to try and decide the
complaint filed by petitioner against the SDAYES!
2. Whether or not the termination of the services of petitioner is an ecclesiastical
affair, and, as such, involves the separation of church and state andNO
3. Whether or not such termination is valid.NO

HELD:
1. Yes. The grounds for the petitioners dismissal are based on Art 282 of the
Labor Code which enumerates the just causes for termination of employment. By
this alone, it is palpable that the reason for his dismissal from the service is not a
religious nature. Coupled with this is the act of the SDA in furnishing NLRC with a
copy of petitioners letter of termination recognizing his 28 years of service. As
such, the State, through the LA and the NLRC, has the right to take cognizance of
the case and to determine whether SDA, as employer, rightfully exercised its
management prerogative to dismiss an employee. This is in consonance with the
mandate of the Constitution to afford full protection to labor. Furthermore, the
Labor Code, under Art 278 on post-employment states that the provisions of
this Title shall apply to all establishments or undertakings, whether for profit or
not. The provision does not make any exception in favor of a religious
corporation. This is made more evident by the fact that the Rules Implementing
the LC, particularly, Section 1, Rule 1, Book VI on the termination of Employment
and Retirement, categorically includes religious institutions in the coverage of
the law. Private respondents are also estopped from raising the issue of lack of
jurisdiction for the first time on appeal since the SDA had fully participated in the
trials and hearings of the case from start to finish.
2. The principle of separation of church and state finds no application in this
case. The case does not concern an ecclesiastical or purely religious affair as to
bar the State from taking cognizance of the same. An ecclesiastical affair involves
the relationship between the church and its members and relate to matters of
faith, religious doctrines, worship and governance of the congregation. Examples
of this are proceedings for excommunication, ordinations of religious ministers,

3E Andaya Ching Espiritu Hefti Galvez Gammad Lainez Lui Madamba Nagera Narvasa
Ong Palangdao Rosales Sanchez Santos Satrain Tabo (2014-2015)

Labor Relations Case Digest - Atty. Joyrich Golangco

administration of sacraments and other activities with attached religious


significance. What is involved here is the relationship of the church as an
employer and the minister as an employee. It is purely secular and has no
relation whatsoever with the practice of faith, worship or doctrines of the
church. Aside from these, SDA admitted in a certification23 issued by its officer,
Mr. Ibesate, that petitioner has been its employee for twenty-eight (28) years.
SDA even registered petitioner with the Social Security System (SSS) as its
employee. As a matter of fact, the worker's records of petitioner have been
submitted by private respondents as part of their exhibits.

3. In termination cases, the burden of proving that the termination was for a
valid or authorized cause rests on the employer. The requisites for a valid
dismissal are: (1) the employee must be afforded due process, and (2) the
dismissal must be for a valid cause as provided in Article 282 of the Labor Code.
Without these twin requirements, termination would be illegal. The rules further
require the employer to furnish the employee with two written notices, (a) a
written notice served on the employee specifying the ground or grounds for
termination and giving the said employee reasonable opportunity within which
to explain his side And (b) a written notice of termination served on the
employee indicating that upon due consideration of all circumstances, grounds
have been established to justify his termination
.
Noncompliance is fatal because the requirements are conditions sine qua non
before dismissal may validly be effected. Private respondents substantially failed
to comply with the abovementioned requirements as regards the first notice. The
notice to attend the meeting cannot be construed as a written charge. It never
stated the particular acts or omissions on which petitioner's impending
termination was grounded. The alleged grounds for Austrias dismissal from the
service were only revealed to him when the actual letter of dismissal was finally
issued. Furthermore, the Court did not sustain the validity of the dismissal based
on the grounds enumerated by the private respondents Private respondent
failed to substantially comply with the above requirements.
26 DFA vs NLRC

FACTS: On 27 January 1993, private respondent Jose C. Magnayi initiated NLRCNCR Case No. 00-01-0690-93 for his alleged illegal dismissal by ADB and the
latter's violation of the "labor-only" contracting law. Two summonses were
served, one sent directly to the ADB and the other through the DFA, both with a
copy of the complainant. Forthwith, the ADB and the DFA notified respondent LA
Nieves de Castro that the ADB, as well as its President and Officers, were covered
by an immunity from legal process except for borrowings, guaranties or the sale
of securities pursuant to Article 50(1) and Article 55 of the Agreement
Establishing the Asian Development Bank (the "Charter") in relation to Section 5
and Section 44 of the Agreement Between The Bank And The Government Of The
Philippines Regarding The Bank's Headquarters (the "Headquarters
Agreement").
The LA took cognizance of the complaint on the impression that the ADB had
waived its diplomatic immunity from suit by entering into service contracts with
different private companies, ADB has descended to the level of an ordinary party
to a commercial transaction giving rise to a waiver of its immunity from suit, In
time, the LA rendered his decision declaring ADB liable for illegal dismissal.

The ADB did not appeal the decision. Instead, on 03 November 1993, the DFA
referred the matter to the NLRC; in its referral, the DFA sought a "formal vacation
of the void judgment." The NLRC chairman replied saying that while they
exercised administrative supervision over the Commission and its regional
branches and all its personnel, including the Executive LA and Labor Arbiters'
(penultimate paragraph, Art. 213, Labor Code), he does not have the competence
to investigate or review any decision of a Labor Arbiter. However, on the purely
administrative aspect of the decision-making process, he may cause that an
investigation be made of any misconduct, malfeasance or misfeasance, upon
complaint properly made and if DFA feels that the action of LA Nieves de Castro
constitutes misconduct, malfeasance or misfeasance, it is suggested that an
appropriate complaint be lodged with the Office of the Ombudsman.
Dissatisfied, the DFA lodged the instant petition for certiorari.
ISSUES: Whether or Not, ADB is immune from labor suits

3E Andaya Ching Espiritu Hefti Galvez Gammad Lainez Lui Madamba Nagera Narvasa
Ong Palangdao Rosales Sanchez Santos Satrain Tabo (2014-2015)

Labor Relations Case Digest - Atty. Joyrich Golangco

HELD: Yes, Article 50(1) of the Charter provides: "The Bank shall enjoy
immunity from every form of legal process, except in cases arising out of or in
connection with the exercise of its powers to borrow money, to guarantee
obligations, or to buy and sell or underwrite the sale of securities."
Under Article 55 thereof "All Governors, Directors, alternates, officers and
employees of the Bank, including experts performing missions for the Bank:
"(1) shall be immune from legal process with respect of acts performed by them
in their official capacity, except when the Bank waives the immunity.

Like provisions are found in the Headquarters Agreement. Thus, its Section 5
reads: "The Bank shall enjoy immunity from every form of legal process, except
in cases arising out of, or in connection with, the exercise of its powers to borrow
money, to guarantee obligations, or to buy and sell or underwrite the sale of
securities."
And, with respect to certain officials of the bank, Section 44 of the agreement
states: "Governors, other representatives of Members, Directors, the President,
Vice-President and executive officers as may be agreed upon between the
Government and the Bank shall enjoy, during their stay in the Republic of the
Philippines in connection with their official duties with the Bank:
"xxx xxx xxx
"(b) Immunity from legal process of every kind in respect of words spoken or
written and all acts done by them in their official capacity."
The above stipulations of both the Charter and Headquarters Agreement should
be able, nay well enough, to establish that, except in the specified cases of
borrowing and guarantee operations, as well as the purchase, sale and
underwriting of securities, the ADB enjoys immunity from legal process of every
form. The Bank's officers, on their part, enjoy immunity in respect of all acts
performed by them in their official capacity. The Charter and the Headquarters
Agreement granting these immunities and privileges are treaty covenants and
commitments voluntarily assumed by the Philippine government which must be
respected.

In World Health Organization vs. Aquino, we have declared:


"It is a recognized principle of international law and under our system of
separation of powers that diplomatic immunity is essentially a political question

and courts should refuse to look beyond a determination by the executive branch
of the government, and where the plea of diplomatic immunity is recognized and
affirmed by the executive branch of the government. . . it is then the duty of the
courts to accept the claim of immunity upon appropriate suggestion by the
principal law officer of the government, . . . or other officer acting under his
direction. Hence, in adherence to the settled principle that courts may not so
exercise their jurisdiction . . . as to embarrass the executive arm of the
government in conducting foreign relations, it is accepted doctrine that 'in such
cases the judicial department of government follows the action of the political
branch and will not embarrass the latter by assuming an antagonistic
jurisdiction."
Neither is the contention of respondent that by entering into service
contracts with different private companies, ADB has descended to the level
of an ordinary party to a commercial transaction giving rise to a waiver of
its immunity from suit.
"Certainly, the mere entering into a contract by a foreign state with a private
party cannot be the ultimate test. Such an act can only be the start of the inquiry.
The logical question is whether the foreign state is engaged in the activity in the
regular course of business. If the foreign state is not engaged regularly in a
business or trade, the particular act or transaction must then be tested by its
nature. If the act is in pursuit of a sovereign activity, or an incident thereof, then
it is an act jure imperii, especially when it is not undertaken for gain or profit."
The service contracts referred to by private respondent have not been intended
by the ADB for profit or gain but are official acts over which a waiver of
immunity would not attach.
27. PNB v. Cabansag, June 21, 2005

FACTS: Respondent Florence Cabansag arrived in Singapore as a tourist. She


applied for employment, with the Singapore Branch of the PNB, a private
banking corporation organized and existing under the laws of the Philippines. At
the time, the Branch Office had two (2) types of employees: (a) expatriates or the
regular employees, hired in Manila and assigned abroad including Singapore, and
(b) locally (direct) hired. She applied for employment as Branch Credit Officer.

3E Andaya Ching Espiritu Hefti Galvez Gammad Lainez Lui Madamba Nagera Narvasa
Ong Palangdao Rosales Sanchez Santos Satrain Tabo (2014-2015)

Labor Relations Case Digest - Atty. Joyrich Golangco


She was hired (appointed) upon the recommendation of Ruben C. Tobias a
General Manager of the PNB Singapore branch. As required, she obtained an
Employee Pass for two years. Her appointment as a Credit Officer stated that
she would undergo a probation for a period of three months from the date of
assumption of duty that can be terminated upon 1 day notice and after probation
upon one month notice. The Philippine Embassy in Singapore processed the
employment contract of respondent and then she was issued by the POEA, an
Overseas Employment Certificate, certifying that she was a bona fide contract
worker for Singapore.
Barely three (3) months in office respondent Cabansag, she submitted to Tobias,
her initial performance report that the latter was so impressed with the it that he
made a notation on said Report: GOOD WORK. However, later she was told that
she had to resign because of cost cutting measure.

Perplexed, she asked for a formal advice but she received none. She refused to
resign. Tobias demanded that she submit her letter of resignation, with the
pretext that he needed a Chinese speaking Credit Officer to penetrate the local
market, with the information that a Chinese speaking Credit Officer had already
been hired and will be reporting for work soon. She was warned that, unless she
submitted her letter of resignation, her employment record will be blemished
with the notation DISMISSED spread thereon. Still respondent Cabansag
refused to resign; Tobias dismissed her just about four months from hiring.
Cabansag filed a complaint of illegal dismissal with the labor arbiter which
rendered a decision finding respondents guilty of Illegal dismissal and devoid of
due process. The respondents appealed to the NLRC, contending that the Labor
Arbiter has no jurisdiction because respondent was "locally hired" and totally
"governed by and subject to the laws, common practices and customs" of
Singapore, not of the Philippines. The NLRC denied the respondents motion.
The CA also held that petitioner had failed to establish a just cause for the
dismissal of respondent. The bank had also failed to give her sufficient notice and
an opportunity to be heard and to defend herself.
ISSUES: Whether or not the arbitration branch of the NLRC in the National
Capital Region has jurisdiction over the instant controversy

HELD: Yes. Based on Art. 127 and more specifically, Section 10 of RA 80421 and
Labor arbiters clearly have original and exclusive jurisdiction over claims arising
from Employer Employee relations, including termination disputes involving all
workers, among whom are overseas Filipino workers (OFW).

Prior to employing respondent, petitioner had to obtain an employment pass for


her from the Singapore Ministry of Manpower. Securing the pass was a
regulatory requirement pursuant to the immigration regulations of Singapore.
Similarly, the Philippine government requires non-Filipinos working in the
country to first obtain a local work permit in order to be legally employed here.
That permit, however, does not automatically mean that the noncitizen is
thereby bound by local laws only, as averred by petitioner. It does not at all imply
a waiver of ones national laws on labor. Absent any clear and convincing
evidence to the contrary, such permit simply means that its holder has a legal
status as a worker in the issuing country.
Noteworthy is the fact that respondent likewise applied for and secured an
Overseas Employment Certificate from the POEA through the Philippine
Embassy in Singapore. The Certificate issued declared her a bona fide contract
worker for Singapore. Under Philippine law, this document authorized her
working status in a foreign country and entitled her to all benefits and processes
under our statutes. Thus, even assuming arguendo that she was considered at the
start of her employment as a direct hire governed by and subject to the laws,
common practices and customs prevailing in Singapore[17] she subsequently
became a contract worker or an OFW who was covered by Philippine labor laws
and policies upon certification by the POEA. At the time her employment was
illegally terminated, she already possessed the POEA employment Certificate.
Whether employed locally or overseas, all Filipino workers enjoy the protective
mantle of Philippine labor and social legislation, contract stipulations to the
contrary notwithstanding.
Moreover, petitioner admits that it is a Philippine corporation doing business
through a branch office in Singapore. Significantly, respondents employment by
the Singapore branch office had to be approved by the president of the bank
whose principal offices were in Manila. This circumstance militates against
petitioner's contention that respondent was "locally hired, and totally "governed
by and subject to the laws, common practices and customs" of Singapore, not of

3E Andaya Ching Espiritu Hefti Galvez Gammad Lainez Lui Madamba Nagera Narvasa
Ong Palangdao Rosales Sanchez Santos Satrain Tabo (2014-2015)

Labor Relations Case Digest - Atty. Joyrich Golangco

the Philippines. Instead, this only reinforces the fact that respondent falls under
the legal definition of migrant worker, in this case one deployed in Singapore.
Hence, petitioner cannot escape the application of Philippine laws or the
jurisdiction of the NLRC and the labor arbiter
28. Banez vs Valdevilla and Oro Marketing Inc

FACTS: Petitioner Bebiano Baez was the sales operations manager of private
respondent Oro Marketing in its branch in Iligan City. In 1993, private
respondent "indefinitely suspended" petitioner and the latter filed a complaint
for illegal dismissal with NLRC-iligan City.

On July 7, 1994, LA Nicodemus G. Palangan rendered a judgment in favor of the


petitioner and ordered the payment of separation pay in lieu of reinstatement,
and of back wages and attorney's fees. The decision was appealed to the NLRC,
which dismissed the same for having been filed out of time. Elevated by petition
for certiorari before this Court, the case was dismissed on technical grounds;
however, the Court also pointed out that even if all the procedural requirements
for the filing of the petition were met, it would still be dismissed for failure to
show grave abuse of discretion on the part of the NLRC.

On November 13, 1995, private respondent filed a complaint for damages before
the RTC - Misamis Oriental, docketed as Civil Case No. 95-554, which prayed for
the payment of the following:
a. P709,217.97 plus 12% interest as loss of profit and/or unearned income of
three years;
b. P119,700.00 plus 12% interest as estimated cost of supplies, facilities,
properties, space, etc. for three years;
c. P5,000.00 as initial expenses of litigation; and
d. P25,000.00 as attorney's fees. 4
On January 30, 1996, petitioner filed a motion to dismiss the above complaint.
He contends that the action for damages having arisen from an employeremployee relationship was squarely under the exclusive original jurisdiction of
the NLRC under Article 217(a), paragraph 4 of the Labor Code and is barred by
reason of the final judgment in the labor case and that He accused private

respondent of splitting causes of action, stating that the latter could very well
have included the instant claim for damages in its counterclaim before the LA. He
also pointed out that the civil action of private respondent is an act of forumshopping and was merely resorted to after a failure to obtain a favorable decision
with the NLRC.
Respondent Judge Downey Valdevilla ruled that the complaint which is for
damages does not ask for any relief under the Labor Code of the Philippines. It
seeks to recover damages as redress for defendant's breach of his contractual
obligation to plaintiff who was damaged and prejudiced. The Court believes such
cause of action is within the realm of civil law, and jurisdiction over the
controversy belongs to the regular courts.

ISSUES: whether or not the petition is within the jurisdiction of the Labor Arbiter

HELD: Yes, Article 217(a), paragraph 4 of the Labor Code, which was already in
effect at the time of the filing of this case, reads:
ARTICLE 217. Jurisdiction of Labor Arbiters and the Commission. (a)
Except as otherwise provided under this Code, the Labor Arbiters shall have
original and exclusive jurisdiction to hear and decide, within thirty (30) calendar
days after the submission of the case by the parties for decision without
extension, even in the absence of stenographic notes, the following cases
involving all workers, whether agricultural or non-agricultural:
xxx xxx xxx
4. Claims for actual, moral, exemplary and other forms of damages arising from
the employer-employee relations;
xxx xxx xxx

The above provisions are a result of the amendment by Sec 9 of R.A. No. 6715,
which took effect on March 21, 1989, and which put to rest the earlier confusion
as to who between Labor Arbiters and regular courts had jurisdiction over claims
for damages as between employers and employees.
History of Art 271

3E Andaya Ching Espiritu Hefti Galvez Gammad Lainez Lui Madamba Nagera Narvasa
Ong Palangdao Rosales Sanchez Santos Satrain Tabo (2014-2015)

Labor Relations Case Digest - Atty. Joyrich Golangco

Prior to R.A. 6715, jurisdiction over all money claims of workers, including
claims for damages, was originally lodged with the Labor Arbiters and the NLRC
by Article 217 of the Labor Code. On May 1, 1979, however, P.D. 1367 amended
said Article 217 to the effect that "Regional Directors shall not indorse and Labor
Arbiters shall not entertain claims for moral or other forms of damages." This
limitation in jurisdiction, however, lasted only briefly since on May 1, 1980, P.D.
No. 1691 nullified P.D. No. 1367 and restored Article 217 of the Labor Code
almost to its original form. Presently, and as amended by R.A. 6715, the
jurisdiction of Labor Arbiters and the NLRC in Article 217 is comprehensive
enough to include claims for all forms of damages "arising from the employeremployee relations".
There is no mistaking the fact that in the case before us, private respondent's
claim against petitioner for actual damages arose from a prior employeremployee relationship.

In Ebon vs. de Guzman, 113 SCRA 52, 13 this Court discussed:


The lawmakers in divesting the Labor Arbiters and the NLRC of jurisdiction to
award moral and other forms of damages in labor cases could have assumed that
the Labor Arbiters' position-paper procedure of ascertaining the facts in dispute
might not be an adequate tool for arriving at a just and accurate assessment of
damages, as distinguished from back wages and separation pay, and that the trial
procedure in the Court of First Instance would be a more effective means of
determining such damages. . .
Evidently, the lawmaking authority had second thoughts about depriving the
Labor Arbiters and the NLRC of the jurisdiction to award damages in labor cases
because that setup would mean duplicity of suits, splitting the cause of action
and possible conflicting findings and conclusions by two tribunals on one and the
same claim.
So, on May 1, 1980, PD No. 1691 (which substantially reenacted Article 217 in its
original form) nullified PD No. 1367 and restored to the Labor Arbiter and the
NLRC their jurisdiction to award all kinds of damages in cases arising from
employer-employee relations. . . .
Clearly, respondent court's taking jurisdiction over the instant case would bring
about precisely the harm that the lawmakers sought to avoid in amending the

Labor Code to restore jurisdiction over claims for damages of this nature to the
NLRC.
29. SANTOS V SERVIER PHILS

FACTS: Petitioner Isabela Santos was the Human Resource Manager of


respondent Servier Philippines, Inc. since 1991 until her termination from
service in 1999. (8years) In 1998, Isabela Santos attended a meeting of all human
resource managers of respondent, held in Paris, France. Since the last day of the
meeting coincided with the graduation of her only child, she arranged for a
European vacation with her family right after the meeting. She, thus, filed a
vacation leave. Petitioner Isabela, together with her husband Antonio P. Santos,
her son, and some friends, had dinner at Leon des Bruxelles, a Paris restaurant
known for mussels as their specialty. While having dinner, petitioner complained
of stomach pain, then vomited. Eventually, she was brought to the hospital
known as Centre Chirurgical de LQuest where she fell into coma for 21 days and
later stayed at the Intensive Care Unit (ICU) for 52 days. The hospital found that
the probable cause of her sudden attack was "alimentary allergy," as she had
recently ingested a meal of mussels which resulted in a concomitant urticarial
eruption.
During the time that petitioner was confined at the hospital, her husband and son
stayed with her in Paris. Petitioners hospitalization expenses, as well as those of
her husband and son, were paid by respondent Servier Phils. Respondent Servier
Phils had requested the petitioner Isabelas physician to conduct a thorough
physical and psychological evaluation of her condition, to determine her fitness
to resume her work at the company. Petitioners physician concluded that the
petitioner had not fully recovered mentally and physically. Hence, respondent
was constrained to terminate petitioners services. As a consequence of
petitioners termination from employment, respondent offered a retirement
package which consists of:
Retirement Plan Benefits: P 1,063,841.76
Insurance Pension at 20,000.00/month for 60 months from company sponsored
group life policy: P 1,200,000.00
Educational assistance: P 465,000.00

3E Andaya Ching Espiritu Hefti Galvez Gammad Lainez Lui Madamba Nagera Narvasa
Ong Palangdao Rosales Sanchez Santos Satrain Tabo (2014-2015)

Medical and Health Care: P 200,000.00

Labor Relations Case Digest - Atty. Joyrich Golangco

Of the promised retirement benefits amounting to P1, 063,841.76, only


P701,454.89 was released to petitioners husband, the balance thereof was
withheld allegedly for taxation purposes. Respondent also failed to give the other
benefits listed above. Petitioner, represented by her husband, instituted the
instant case for unpaid salaries Unpaid separation pay Unpaid balance of
retirement package plus interest Insurance pension for permanent disability
Educational assistance for her son Medical assistance Reimbursement of
medical and rehabilitation expenses Moral, exemplary, and actual damages, plus
attorneys fees The Labor Arbiter dismissed the petition. The arbiter refused to
rule on the legality of the deductions made by respondent from petitioners total
retirement benefits for taxation purposes, as the issue was beyond the
jurisdiction of the NLRC.

On appeal, the NLRC emphasized that petitioner was dismissed from


employment due to a disease/disability under Article 284 of the Labor Code. In
view of her non-entitlement to retirement benefits, the amounts received by
petitioner should then be treated as her separation pay.

Unsatisfied, petitioner elevated the matter to the Court of Appeals which


affirmed the NLRC decision. The SC, before deciding on whether or not the
LA/NLRC may decide on the legality of the deduction due to tax, first settled the
issue on whether or not the benefit received by petitioner is a retirement benefit
or a separation pay. Respondent dismissed the petitioner from her employment
based on Article 284 of the Labor Code, as amended, which reads:
Art. 284. DISEASE AS GROUND FOR TERMINATION
An employer may terminate the services of an employee who
has been found to be suffering from any disease and whose continued
employment is prohibited by law or is prejudicial to his health as well as
to the health of his co-employees: Provided, That he is paid separation
pay equivalent to at least one (1) month salary or to one-half (1/2)
month salary for every year of service, whichever is greater, a fraction of
at least six (6) months being considered as one (1) whole year.

As she was dismissed on the abovementioned ground, the law gives the
petitioner the right to demand separation pay. However, respondent established
a retirement plan in favor of all its employees which specifically provides for
"disability retirement," to wit:
Sec. 4. DISABILITY RETIREMENT
In the event that a Member is retired by the Company due to
permanent total incapacity or disability, as determined by a competent
physician appointed by the Company, his disability retirement benefit
shall be the Full Members Account Balance determined as of the last
valuation date. x x

The receipt of retirement benefits does not bar the retiree from receiving
separation pay. Separation pay is a statutory right designed to provide the
employee with the wherewithal during the period that he/she is looking for
another employment. On the other hand, retirement benefits are intended to
help the employee enjoy the remaining years of his life, lessening the burden of
worrying about his financial support, and are a form of reward for his loyalty and
service to the employer. Hence, they are not mutually exclusive. However, this is
only true if there is no specific prohibition against the payment of both benefits
in the retirement plan and/or in the Collective Bargaining Agreement (CBA).In
the instant case, the Retirement Plan bars the petitioner from claiming additional
benefits on top of that provided for in the Plan.

There being such a provision, petitioner is entitled only to either the separation
pay under the law or retirement benefits under the Plan, and not both. Clearly,
the benefits received by petitioner from the respondent represent her retirement
benefits under the Plan. The question that now confronts us is whether these
benefits are taxable. If so, respondent correctly made the deduction for tax
purposes. Otherwise, the deduction was illegal and respondent is still liable for
the completion of petitioners retirement benefits. Respondent argues that the
legality of the deduction from petitioners total benefits cannot be taken
cognizance of by this Court since the issue was not raised during the early stage
of the proceedings.
ISSUES: W/N Isabel Santoss claim for illegal deduction falls within the
LA/NLRCs jurisdiction

3E Andaya Ching Espiritu Hefti Galvez Gammad Lainez Lui Madamba Nagera Narvasa
Ong Palangdao Rosales Sanchez Santos Satrain Tabo (2014-2015)

Labor Relations Case Digest - Atty. Joyrich Golangco


HELD: YES. The issue of deduction for tax purposes is intertwined with the main
issue of whether or not petitioners benefits have been fully given her. It is,
therefore, a money claim arising from the employer-employee relationship,
which clearly falls within the jurisdiction41 of the Labor Arbiter and the NLRC.
Retirement benefits are EXEMPT from tax, provided that:
(1) a reasonable private benefit plan is maintained by the employer
(2) the retiring official or employee has been in the service of the same
employer for at least ten (10) years
(3) the retiring official or employee is not less than fifty (50) years of age
at the time of his retirement and
(4) the benefit had been availed of only once.
Here, Isabela was disqualified for disability retirement. At the time of such
retirement, she was only 41 years of age and had been in the service for more or
less eight (8) years. As such, such retirement benefits are NOT EXEMPT from
taxation for failure to comply with the age and length of service requirements.
Therefore, SERRVIER PHILIPPINES cannot be faulted for deducting from
Isabela's total retirement benefits the amount of P362,386.87, for taxation
purposes.
30. Pepsi Cola v hon. Lolita o. Gal-lang et al

FACTS: The private respondents were employees of the petitioner who were
suspected of complicity in the irregular disposition of empty Pepsi Cola bottles.
On July 16, 1987, the petitioners filed a criminal complaint for theft against them
but this was later withdrawn and substituted with a criminal complaint for
falsification of private documents. On November 26, 1987, after a preliminary
investigation conducted by the Municipal Trial Court of Tanauan, Leyte, the
complaint was dismissed. The dismissal was affirmed on April 8, 1988, by the
Office of the Provincial Prosecutor.
Meantime, allegedly after an administrative investigation, the private
respondents were dismissed by the petitioner company on November 23, 1987.
As a result, they lodged a complaint for illegal dismissal with the Regional
Arbitration Branch of the NLRC in Tacloban City on December 1, 1987, and
decisions demanded reinstatement with damages. In addition, they instituted in

the Regional Trial Court of Leyte, on April 4, 1988, a separate civil complaint
against the petitioners for damages arising from what they claimed to be their
malicious prosecution.

The petitioners moved to dismiss the civil complaint on the ground that the trial
court had no jurisdiction over the case because it involved employee-employer
relations that were exclusively cognizable by the labor arbiter.

ISSUES: Whether the private respondents civil complaint for damages falls
under the jurisdiction of the labor arbiter.

HELD: NO, It must be stressed that not every controversy involving workers and
their employers can be resolved only by the labor arbiters. This will be so only if
there is a "reasonable causal connection" between the claim asserted and
employee-employer relations to put the case under the provisions of Article 217.
Absent such a link, the complaint will be cognizable by the regular courts of
justice in the exercise of their civil and criminal jurisdiction.

The case now before the Court involves a complaint for damages for malicious
prosecution which was filed with the Regional Trial Court of Leyte by the
employees of the defendant company. It does not appear that there is a
"reasonable causal connection" between the complaint and the relations of the
parties as employer and employees. The complaint did not arise from such
relations and in fact could have arisen independently of an employment
relationship between the parties. No such relationship or any unfair labor
practice is asserted. What the employees are alleging is that the petitioners acted
with bad faith when they filed the criminal complaint which the Municipal Trial
Court said was intended "to harass the poor employees" and the dismissal of
which was affirmed by the Provincial Prosecutor "for lack of evidence to
establish even a slightest probability that all the respondents herein have
committed the crime imputed against them." This is a matter which the labor
arbiter has no competence to resolve as the applicable law is not the Labor Code
but the Revised Penal Code.
31. 7K CORPORATION, v. EDDIE ALBARICO

FACTS: When he was dismissed on 5 April 1993, respondent Eddie Albarico


(Albarico) was a regular employee of petitioner 7K Corporation, a company

3E Andaya Ching Espiritu Hefti Galvez Gammad Lainez Lui Madamba Nagera Narvasa
Ong Palangdao Rosales Sanchez Santos Satrain Tabo (2014-2015)

Labor Relations Case Digest - Atty. Joyrich Golangco

selling water purifiers. In April of 1993, the chief operating officer of petitioner
7K Corporation terminated Albaricos employment allegedly for his poor sales
performance. Respondent had to stop reporting for work, and he subsequently
submitted his money claims against petitioner for arbitration before the National
Conciliation and Mediation Board (NCMB). The issue for voluntary arbitration
before the NCMB, according to the parties Submission Agreement dated 19 April
1993, was whether respondent Albarico was entitled to the payment of
separation pay and the sales commission reserved for him by the corporation.
On 17 September 1997, petitioner corporation filed its Position Paper in the
NCMB arbitration case. It denied that respondent was terminated from work,
much less illegally dismissed. The corporation claimed that he had voluntarily
stopped reporting for work after receiving a verbal reprimand for his sales
performance; hence, it was he who was guilty of abandonment of employment.
Respondent made an oral manifestation that he was adopting the position paper
he submitted to the labor arbiter, a position paper in which the former claimed
that he had been illegally dismissed.

On 18 November 2005, the NCMB voluntary arbitrator rendered a Decision


finding petitioner corporation liable for illegal dismissal. The termination of
respondent Albarico, by reason of alleged poor performance, was found invalid.
The arbitrator explained that the promotions, increases in salary, and awards
received by respondent belied the claim that the latter was performing poorly. It
was also found that Albarico could not have abandoned his job, as the
abandonment should have been clearly shown. Mere absence was not sufficient,
according to the arbitrator, but must have been accompanied by overt acts
pointing to the fact that the employee did not want to work anymore. It was
noted that, in the present case, the immediate filing of a complaint for illegal
dismissal against the employer, with a prayer for reinstatement, showed that the
employee was not abandoning his work. The voluntary arbitrator also found that
Albarico was dismissed from his work without due process.
However, it was found that reinstatement was no longer possible because of the
strained relationship of the parties. Thus, in lieu of reinstatement, the voluntary
arbitrator ordered the corporation to pay separation pay for two years at P4,456
for each year, or a total amount of P8,912.

Additionally, in view of the finding that Albarico had been illegally dismissed, the
voluntary arbitrator also ruled that the former was entitled to back wages in the
amount of P90, 804. Finally, the arbitrator awarded attorneys fees in
respondents favor, because he had been compelled to file an action for illegal
dismissal.

Petitioner corporation subsequently appealed to the CA, imputing to the


voluntary arbitrator grave abuse of discretion amounting to lack or excess of
jurisdiction for awarding back wages and attorneys fees to respondent Albarico
based on the formers finding of illegal dismissal. The arbitrator contended that
the issue of the legality of dismissal was not explicitly included in the Submission
Agreement dated 19 April 1993 filed for voluntary arbitration and resolution. It
prayed that the said awards be set aside, and that only separation pay of
P8,912.00 and sales commission of P4,787.60 be awarded.
The CA affirmed the Decision of the voluntary arbitrator, but eliminated the
award of attorneys fees for having been made without factual, legal or equitable
justification. Petitioners Motion for Partial Reconsideration was denied as well.
Hence, this Petition.

ISSUES: Whether the voluntary arbitrator properly assumed jurisdiction to


decide the issue of the legality of the dismissal of respondent as well as the
latters entitlement to back wages, (even if neither the legality nor the
entitlement was expressly claimed in the Submission Agreement of the parties).
HELD: YES. Voluntary arbitrators may, by agreement of the parties, assume
jurisdiction over a termination dispute such as the present case, contrary to the
assertion of petitioner that they may not.

We rule that although petitioner correctly contends that separation pay may in
fact be awarded for reasons other than illegal dismissal, the circumstances of the
instant case lead to no other conclusion than that the claim of respondent
Albarico for separation pay was premised on his allegation of illegal dismissal.
Thus, the voluntary arbitrator properly assumed jurisdiction over the issue of
the legality of his dismissal.

3E Andaya Ching Espiritu Hefti Galvez Gammad Lainez Lui Madamba Nagera Narvasa
Ong Palangdao Rosales Sanchez Santos Satrain Tabo (2014-2015)

Labor Relations Case Digest - Atty. Joyrich Golangco

True, under the Labor Code, separation pay may be given not only when there is
illegal dismissal. In fact, it is also given to employees who are terminated for
authorized causes, such as redundancy, retrenchment or installation of laborsaving devices under Article 28329 of the Labor Code. Additionally,
jurisprudence holds that separation pay may also be awarded for considerations
of social justice, even if an employee has been terminated for a just cause other
than serious misconduct or an act reflecting on moral character.30 The Court has
also ruled that separation pay may be awarded if it has become an established
practice of the company to pay the said benefit to voluntarily resigning
employees31 or to those validly dismissed for non-membership in a union as
required in a closed-shop agreement.32
The above circumstances, however, do not obtain in the present case. There is no
claim that the issue of entitlement to separation pay is being resolved in the
context of any authorized cause of termination undertaken by petitioner
corporation. Neither is there any allegation that a consideration of social justice
is being resolved here. In fact, even in instances in which separation pay is
awarded in consideration of social justice, the issue of the validity of the
dismissal still needs to be resolved first. Only when there is already a finding of a
valid dismissal for a just cause does the court then award separation pay for
reason of social justice. The other circumstances when separation pay may be
awarded are not present in this case.
The foregoing findings indisputably prove that the issue of separation pay
emanates solely from respondents allegation of illegal dismissal. In fact,
petitioner itself acknowledged the issue of illegal dismissal in its position paper
submitted to the NCMB.

There has to be a reason for deciding the issue of respondents entitlement to


separation pay. To think otherwise would lead to absurdity, because the
voluntary arbitrator would then be deciding that issue in a vacuum. The
arbitrator would have no basis whatsoever for saying that Albarico was entitled
to separation pay or not if the issue of the legality of respondents dismissal was
not resolve first.

Hence, the voluntary arbitrator correctly assumed that the core issue behind the
issue of separation pay is the legality of the dismissal of respondent. Moreover,

we have ruled in Sime Darby Pilipinas, Inc. v. Deputy Administrator Magsalin33


that a voluntary arbitrator has plenary jurisdiction and authority to interpret an
agreement to arbitrate and to determine the scope of his own authority when the
said agreement is vague subject only, in a proper case, to the certiorari
jurisdiction of this Court.
Having established that the issue of the legality of dismissal of Albarico was in
fact necessarily albeit not explicitly included in the Submission Agreement
signed by the parties, this Court rules that the voluntary arbitrator rightly
assumed jurisdiction to decide the said issue.

Consequently, we also rule that the voluntary arbitrator may award back wages
upon a finding of illegal dismissal, even though the issue of entitlement thereto is
not explicitly claimed in the Submission Agreement. Backwages, in general, are
awarded on the ground of equity as a form of relief that restores the income lost
by the terminated employee by reason of his illegal dismissal.
32. LAND BANK OF THE PHILIPPINES, petitioner, vs. SEVERINO LISTANA,
SR., respondent.

FACTS: Respondent Severino Listana is the owner of a parcel of land containing


an area of 246.0561 hectares, located in Inlagadian, Casiguran, Sorsogon, covered
by Transfer Certificate of Title No. T-20193. He voluntarily offered to sell the said
land to the government, through the Department of Agrarian Reform (DAR),
under Section 20 of R.A. 6657, also known as the Comprehensive Agrarian
Reform Law of 1988 (CARL). The DAR valued the property at P5,871,689.03,
which was however rejected by the respondent. Hence, the Department of
Agrarian Reform Adjudication Board (DARAB) of Sorsogon commenced
summary administrative proceedings to determine the just compensation of the
land.
On October 14, 1998, the DARAB rendered a Decision, the dispositive portion of
which reads as follows:
WHEREFORE, taking into consideration the foregoing
computation, the prior valuation made by the Land Bank of the
Philippines is hereby set aside and a new valuation in the
amount of TEN MILLION NINE HUNDRED FIFTY SIX THOUSAND

3E Andaya Ching Espiritu Hefti Galvez Gammad Lainez Lui Madamba Nagera Narvasa
Ong Palangdao Rosales Sanchez Santos Satrain Tabo (2014-2015)

Labor Relations Case Digest - Atty. Joyrich Golangco

NINE HUNDRED SIXTY THREE PESOS AND 25 CENTAVOS


(P10,956,963.25) for the acquired area of 240.9066 hectares.
The Land Bank of the Philippines is hereby ordered to pay the
same to the landowner in the manner provided for by law.
SO ORDERED.

Thereafter, a Writ of Execution was issued by the PARAD directing the manager
of Land Bank (Mr. Alex Lorayes) to pay the respondent the aforesaid amount as
just compensation in the manner provided by law.

On September 2, 1999, respondent filed a Motion for Contempt with the PARAD,
alleging that petitioner Land Bank failed to comply with the Writ of Execution
issued on June 18, 1999. He argued that such failure of the petitioner to comply
with the writ of execution constitutes contempt of the DARAB.
On August 20, 2000, the PARAD issued an Order granting the Motion for
Contempt, as follows:
WHEREFORE, premises considered, the motion for contempt is
hereby GRANTED, thus ALEX A. LORAYES, as Manager of
respondent LAND BANK, is cited for indirect contempt and
hereby ordered to be imprisoned until he complies with the
Decision of the case dated October 14, 1998.

SO ORDERED.

Petitioner Land Bank filed a petition for injunction before the Regional Trial
Court of Sorsogon, Sorsogon, with application for the issuance of a writ of
preliminary injunction to restrain PARAD Capellan from issuing the order of
arrest.13 The case was raffled to Branch 51 of said court. On January 29, 2001,
the trial court issued an Order, the dispositive portion of which reads:
WHEREFORE, premises considered, the respondent Provincial
Adjudicator of the DARAB or anyone acting in its stead is
enjoined as it is hereby enjoined from enforcing its order of
arrest against Mr. Alex A. Lorayes pending the final termination

of the case before RTC Branch 52, Sorsogon upon the posting of
a cash bond by the Land Bank.
SO ORDERED

Respondent filed a Motion for Reconsideration of the trial courts order, which
was denied in an Order dated April 2, 2001.

Thus, respondent filed a special civil action for certiorari with the Court of
Appeals, docketed as CA-G.R. SP No. 65276. On December 11, 2001, the Court of
Appeals rendered the assailed decision which nullified the Orders of the Regional
Trial Court of Sorsogon, Sorsogon, Branch 51.
Hence, the instant petition for review

ISSUES: Whether the order for the arrest of petitioners manager, Mr. Alex
Lorayes by the PARAD, was valid

HELD: No. There are only two ways a person can be charged with indirect
contempt, namely, (1) through a verified petition; and (2) by order or formal
charge initiated by the court motu proprio.In the case at bar, neither of these
modes was adopted in charging Mr. Lorayes with indirect contempt.
More specifically, Rule 71, Section 12 of the 1997 Rules of Civil Procedure,
referring to indirect contempt against quasi-judicial entities, provides:
Sec. 12. Contempt against quasi-judicial entities. Unless
otherwise provided by law, this Rule shall apply to contempt
committed against persons, entities, bodies or agencies
exercising quasi-judicial functions, or shall have suppletory
effect to such rules as they may have adopted pursuant to
authority granted to them by law to punish for contempt. The
Regional Trial Court of the place wherein the contempt has
been committed shall have jurisdiction over such charges as
may be filed therefore. (emphasis supplied)

3E Andaya Ching Espiritu Hefti Galvez Gammad Lainez Lui Madamba Nagera Narvasa
Ong Palangdao Rosales Sanchez Santos Satrain Tabo (2014-2015)

Labor Relations Case Digest - Atty. Joyrich Golangco

The foregoing amended provision puts to rest once and for all the questions
regarding the applicability of these rules to quasi-judicial bodies, to wit:
1. This new section was necessitated by the holdings that the
former Rule 71 applied only to superior and inferior courts and
did not comprehend contempt committed against
administrative or quasi-judicial officials or bodies, unless said
contempt is clearly considered and expressly defined as
contempt of court, as is done in the second paragraph of Sec.
580, Revised Administrative Code. The provision referred to
contemplates the situation where a person, without lawful
excuse, fails to appear, make oath, give testimony or produce
documents when required to do so by the official or body
exercising such powers. For such violation, said person shall be
subject to discipline, as in the case of contempt of court, upon
application of the official or body with the Regional Trial Court
for the corresponding sanctions.

Evidently, quasi-judicial agencies that have the power to cite persons for indirect
contempt pursuant to Rule 71 of the Rules of Court can only do so by initiating
them in the proper Regional Trial Court. It is not within their jurisdiction and
competence to decide the indirect contempt cases. These matters are still within
the province of the Regional Trial Courts. In the present case, the indirect
contempt charge was filed, not with the Regional Trial Court, but with the
PARAD, and it was the PARAD that cited Mr. Lorayes with indirect contempt.

Hence, the contempt proceedings initiated through an unverified Motion for


Contempt filed by the respondent with the PARAD were invalid for the following
reasons: First, the Rules of Court clearly require the filing of a verified petition
with the Regional Trial Court, which was not complied with in this case. The
charge was not initiated by the PARAD motu proprio; rather, it was by a motion
filed by respondent. Second, neither the PARAD nor the DARAB have jurisdiction
to decide the contempt charge filed by the respondent. The issuance of a warrant
of arrest was beyond the power of the PARAD and the DARAB. Consequently, all
the proceedings that stemmed from respondents Motion for Contempt,
specifically the Orders of the PARAD dated August 20, 2000 and January 3, 2001
for the arrest of Alex A. Lorayes, are null and void.

33. MANILA ELECTRIC COMPANY, v .J AN CARLO GALA

FACTS: On March 2, 2006, respondent Jan Carlo Gala commenced employment


with the petitioner Meralco Electric Company (Meralco) as a probationary
lineman. He was assigned at Meralcos Valenzuela Sector. Barely four months on
the job, Gala was dismissed for the alleged involvement in stealing Meralcos
electrical supplies sometime in May 25, 2006.

Apparently, prior to the incident, there had been "reported pilferage" or


"rampant theft" by the crew of Gala. At the time of the incident itself, unused
supplies and materials had not been returned to the prejudice of the company. So
on May 25, unknown to the workers, a surveillance task force was formed by
Meralco to monitor the activities of Gala and his colleagues on the worksite and
to record everything using a handy cam. While the Meralco crew was at work,
one Noberto "Bing" Llanes, a non Meralco employee, arrived. He appeared to be
known to the Meralco foremen as the supervisors and other crew members were
seen conversing with him. Llanes boarded the trucks, without being stopped,
and took out what were later found as electrical supplies. Gala did not call the
attention of his supervisors because, as he contends, he was just a mere
linesman. After proper investigation of Meralco, Gala was dismissed. Gala filed a
case for illegal dismissal contending that his mere presence at the scene of the
incident was not sufficient to hold him liable as a conspirator.
Facts directly related to Article 221, LCP:
By way of Galas Comment to the Petition for Cetiorari filed by Meralco, Gala
asks for a denial of the petition because of, among others, serious and fatal
infirmities in the petition. Gala contends, in regard to the alleged procedural
defects of the petition, that the "Verification and Certification," "Secretarys
Certificate" and "Affidavit of Service" do not contain the details of the Community
or Residence Tax Certificates of the affiants, in violation of Section 6 of
Commonwealth Act No. 465 (an Act to Impose a Residence Tax). Additionally, the
lawyers who signed the petition failed to indicate their updated Mandatory
Continuing Legal Education (MCLE) certificate numbers, in violation of the rules.

3E Andaya Ching Espiritu Hefti Galvez Gammad Lainez Lui Madamba Nagera Narvasa
Ong Palangdao Rosales Sanchez Santos Satrain Tabo (2014-2015)

Labor Relations Case Digest - Atty. Joyrich Golangco

LA RULING: DISMISSED FOR LACK OF MERIT. Galas participation in the


pilferage of Meralcos property rendered him unqualified to become a regular
employee.

NLRC RULING: REVERSED LA. It found that Gala had been illegally dismissed,
since there was "no concrete showing of complicity with the alleged
misconduct/dishonesty." The NLRC,
However, ruled out Galas reinstatement, stating that his tenure lasted only up to
the end of his probationary period. It awarded him back wages and attorneys
fees.

CA RULING: AFFIRMED NLRC. Gala had been illegally dismissed, a ruling that
was supported by the evidence. It opined that nothing in the records show Galas
knowledge of or complicity in the pilferage. Ordered Galas reinstatement with
full back wages and other benefits.
ISSUES: Whether or not the petition shall be dismissed outright based on
procedural grounds, to wit:
(a) lack of the Community Tax Certificate details of the affiants and (b) failure of
the lawyers who signed the petition to indicate their updated MCLE certificate
numbers.

HELD: NO. The Court stresses at this point that it is the spirit and intention of
labor legislation that the NLRC and the labor arbiters shall use every reasonable
means to ascertain the facts in each case speedily and objectively, without regard
to technicalities of law or procedure, provided due process is duly observed. In
keeping with this policy and in the interest of substantial justice, we deem it
proper to give due course to the petition, especially in view of the conflict
between the findings of the labor arbiter, on the one hand, and the NLRC and the
CA, on the other. As we said in S.S. Ventures International, Inc. v. S.S. Ventures
Labor Union, "the application of technical rules of procedure in labor cases may
be relaxed to serve the demands of substantial justice. The Court found the
dismissal to be valid. Gala violated his probationary employment agreement,
especially the requirement for him "to observe at all times the highest degree of
transparency, selflessness and integrity in the performance of their duties and
responsibilities". He failed to qualify as a regular employee. For ignoring the

evidence in this case, the NLRC committed grave abuse of discretion and, in
sustaining the NLRC, the CA committed a reversible error.

34. NATIONWIDE SECURITY and ALLIED SERVICES, INC., Petitioner, v .THE


COURT OF APPEALS, NATIONAL LABOR RELATIONS COMMISSION and
JOSEPH DIMPAZ, HIPOLITO LOPEZ, EDWARD ODATO, FELICISIMO PABON
and JOHNNY AGBAY

FACTS: Labor Arbiter Manuel M. Manansala found petitioner Nationwide


Security and Allied Services, Inc., a security agency, not liable for illegal
dismissal in NLRC NCR 00010083396 and 00020112996 involving eight
security guards who were employees of the petitioner. However, the Labor
Arbiter directed the petitioner to pay the aforementioned security guards
P81,750.00 in separation pay, P8,700.00 in unpaid salaries, P93,795.68 for
underpayment and 10% attorneys fees based on the total monetary award. As
supported by sufficient evidence, Nationwide received the decision of the Labor
Arbiter on July 16, 1999. Nationwide then simultaneously filed its "Appeal
Memorandum", "Notice of Appeal" and "Motion to Reduce Bond", by registered
mail on July 29, 1999, under Registry Receipt No. 003098. These were received
by the NLRC on July 30, 1999. The appeal to the NLRC should have been
perfected, as provided by its Rules, within a period of 10 days from receipt by
petitioner of the decision on July 16, 1999.
On the petition for certiorari filed to the Supreme Court, Nationwide contends
that the Court of Appeals erred when it dismissed its case based on
technicalities while the private respondents contend that the appeal to the
NLRC had not been perfected, since the appeal was filed outside the
reglementary period, and the bond was insufficient.

NLRC RULING: DISMISSED PETITION. First, for having been filed beyond the
reglementary period within which to perfect the appeal and second, for filing an
insufficient appeal bond.
CA RULING: DISMISSED PETITION TO RESOLVE ON MERITS. Nationwide
failed to prove not merely reversible error, but grave abuse of discretion

3E Andaya Ching Espiritu Hefti Galvez Gammad Lainez Lui Madamba Nagera Narvasa
Ong Palangdao Rosales Sanchez Santos Satrain Tabo (2014-2015)

Labor Relations Case Digest - Atty. Joyrich Golangco

amounting to lack of or excess of jurisdiction on the part of public respondent


NLRC.
ISSUES: Whether or not the CA erred in dismissing Nationwides appeal based
on technicalities.

HELD: NO, Failure to perfect an appeal renders the decision final and executory.
The right to appeal is a statutory right and one who seeks to avail of the right
must comply with the statute or the rules. The rules, particularly the
requirements for perfecting an appeal within the reglementary period specified
in the law, must be strictly followed as they are considered indispensable
interdictions against needless delays and for the orderly discharge of judicial
business. It is only in highly meritorious cases that this Court will opt not to
strictly apply the rules and thus prevent a grave injustice from being done. The
exception does not obtain here. Thus, we are in agreement that the decision of
the Labor Arbiter already became final and executory because petitioner failed
to file the appeal within 10 calendar days from receipt of the decision.

Clearly, the NLRC committed no grave abuse of discretion in dismissing the


appeal before it. It follows that the Court of Appeals, too, did not err, nor gravely
abuse its discretion, in sustaining the NLRC Order, by dismissing the petition for
certiorari before it.
35. DIAMOND TAXI and/or BRIAN ONG v. FELIPE LLAMAS, JR
FACTS: Llamas worked as a taxi driver for petitioner Diamond Taxi, owned
and operated by petitioner Bryan Ong. On July 18, 2005, Llamas filed before
the Labor Arbiter (LA) a complaint for illegal dismissal against the Diamond
Taxi. In their position papers, Diamond Taxi claims that Llamas has been
dismissed because of his absence without leave for two weeks, several traffic
violations in the year 20002005 and refusal to heed to management instruction.
Diamond Taxi argues that these acts constitute grounds for termination of
Llamas employment.

Llamas failed to seasonably file his position paper because his previous counsel,
despite repeated demands, deferred compliance with the LAs orders for its

submission. Llamas further claims that Bryan Ongs brother and head of
operations, Aljuver, refused to give him the car keys unless he signs a
resignation letter. The LA dismissed his petition for lack of merit

LA RULING: DISMISSED FOR LACK OF MERIT. The LA declared that Llamas


was not dismissed, legally or illegally, hence, he left his job and had been absent
for several days without leave.

NLRC RULING: DISMISSED PETITION. Non-perfection of appeal due to the


failure of Llamas to attach the required certification for non-forum shopping as
required under Section 4, Rule VI of the 2005 NLRC Rules. Llamas moved for
reconsideration, this time attaching the required certification, but was again
denied.

CA RULING: REVERSED NLRC. CA found equitable grounds to brush aside the


mandatory requirement. It ruled that noncompliance with the requirement of
filing a certification for non- forum shopping, while mandatory, may be excused
upon showing of manifest equitable grounds proving compliance. Additionally, in
order to determine if cogent reasons exist to suspend the rules of procedure, the
court must first examine the substantive aspect of the case. Diamond Taxi failed
to clearly prove that Llamas did intend to abandon his work. They even failed to
charge Llamas of his alleged infractions to give the latter the opportunity to
explain. CA awarded Llamas full back wages and other benefits. Separation pay
is also awarded instead of reinstatement because of the strained working
relationship between Llamas and Bryan Ong.
ISSUE Whether or not the CA correctly found that the NLRC committed grave
abuse of discretion in dismissing Llamas appeal on purely technical grounds.

HELD: YES. Under Article 221 (now Article 227) of the Labor Code, the
Commission and its members and the Labor Arbiters shall use every and all
reasonable means to ascertain the facts in each case speedily and objectively and
without regard to technicalities of law or procedure, all in the interest of due
process. Consistently, we have emphasized that rules of procedure are mere
tools designed to facilitate the attainment of justice. A strict and rigid application
which would result in technicalities that tend to frustrate rather than promote

3E Andaya Ching Espiritu Hefti Galvez Gammad Lainez Lui Madamba Nagera Narvasa
Ong Palangdao Rosales Sanchez Santos Satrain Tabo (2014-2015)

Labor Relations Case Digest - Atty. Joyrich Golangco

substantial justice should not be allowed x x x. No procedural rule is sacrosanct if


such shall result in subverting justice. Ultimately, what should guide judicial
action is that a party is given the fullest opportunity to establish the merits of his
action or defense rather than for him to lose life, honor, or property on mere
technicalities. Faced with these circumstances, i.e., Llamas subsequent
compliance with the certification against forum shopping requirement the utter
negligence and inattention of
Llamas former counsel to his pleas and cause, and his vigilance in immediately
securing the services of a new counsel Llamas filing of his position paper before
he learned and received a copy of the LAs decision the absence of a meaningful
opportunity for Llamas to present his case before the LA and the clear merits of
his case, the NLRC should have relaxed the application of procedural rules in the
broader interests of substantial justice.
The decision of CA is AFFIRMED. Court found that Llamas has been
constructively dismissed.
36. ISLRIZ TRADING vs CAPADA et. Al

FACTS: Respondents were helpers of Islriz Trading, a gravel and sand business owned
and operated by petitioner Victor Hugo Lu. Claiming that they were illegally dismissed,
respondents filed a Complaint for illegal dismissal and non-payment of overtime pay,
holiday pay, rest day pay, allowances and separation pay against petitioner before the
Labor Arbiter. On his part, petitioner imputed abandonment of work against
respondents.

LA Gan rendered decision against petitioner. Petitioner appealed to the


NLRC which granted the appeal and ordered respondents reinstatement but without
backwages. Respondents filed an MR but was denied. Respondents filed with the
Labor Arbiter an Ex-Parte Motion to Set Case for Conference with Motion, they averred
that despite the issuance and subsequent finality of the NLRC Resolution which likewise
ordered respondents reinstatement, petitioner still refused to reinstate them. They
prayed that in view of the orders of reinstatement, a computation of the award of
backwages be made and that an Alias Writ of Execution for its enforcement be issued.
The case was then set for pre-execution conference. Since the parties failed to come to
terms of the issue of the monetary award, LA through Fiscal Examiner Trinchera issued

an undated computation. LA Castillon then issued a writ of execution to enforce the


monetary award in accordance with the computation. By virtue of such writ, petitioners
properties were levied and set for auction sale where the respondents were the only
bidders. Later, petitioners claimed that they could not take possession of the properties
because they were padlocked by the petitioner. They asked LA Castillon to issue a
break/open order. Petitioner then filed a motion to quash the writ of execution, notice of
sale/levy. It stated that NLRCs decision did not include payment of backwages but only
reinstatement therefore the writ of execution was null and void. CA dismissed the
petition and agreed with LA Castillons ratiocination that the subject of the writ were
accrued salaries owing to respondents by virtue of the reinstatement order of LA
as provided in Article 223.

ISSUES: 1. Whether the provision of Article 223 of the Labor Code is applicable to this
case?
2. Whether respondents may collect their wages during the period between the
Labor Arbiters order of reinstatement pending appeal and the NLRC Resolution
overturning that of the Labor Arbiter?

HELD: 1. Yes. The Court held that even if the order of reinstatement of the Labor
Arbiter is reversed on appeal, it is obligatory on the part of the employer to
reinstate and pay the wages of the dismissed employee during the period of
appeal until reversal by the higher court or tribunal. It likewise settled the view
that the Labor Arbiters order of reinstatement is immediately executory and the
employer has to either re-admit them to work under the same terms and
conditions prevailing prior to their dismissal, or to reinstate them in the payroll,
and that failing to exercise the options in the alternative, employer must pay the
employees salaries.
2. Yes. The court went on to declare that after the Labor Arbiters decision is
reversed by a higher tribunal, the employee may be barred from collecting the
accrued wages, if it is shown that the delay in enforcing the reinstatement pending
appeal was without fault on the part of the employer. It then provided for the twofold test in determining whether an employee is barred from recovering his accrued
wages, to wit: (1) there must be actual delay or that the order of reinstatement pending
appeal was not executed prior to its reversal; and (2) the delay must not be due to the

3E Andaya Ching Espiritu Hefti Galvez Gammad Lainez Lui Madamba Nagera Narvasa
Ong Palangdao Rosales Sanchez Santos Satrain Tabo (2014-2015)

Labor Relations Case Digest - Atty. Joyrich Golangco

employers unjustified act or omission. If the delay is due to the employers unjustified
refusal, the employer may still be required to pay the salaries notwithstanding the
reversal of the Labor Arbiters Decision.
Applying the two-fold test, Respondents have the right to collect their accrued
salaries during the period between the Labor Arbiters Decision ordering their
reinstatement pending appeal and the NLRC Resolution overturning the same because
petitioners failure to reinstate them either actually or through payroll was due to
petitioners unjustified refusal to effect reinstatement.
1.

A Writ of Execution was issued by LA Gan on April 22, 2002. However, until the
issuance of the September 5, 2002 NLRC Resolution overturning Labor Arbiter
Gans Decision, petitioner still failed to reinstate respondents or effect payroll
reinstatement. This was what actually prompted respondents to file an ExParte Motion to Set Case for Conference with Motion wherein they also prayed
for the issuance of a computation of the award of backwages and Alias Writ of
Execution for its enforcement. It cannot therefore be denied that there was an
actual delay in the execution of the reinstatement aspect of the Decision of
Labor Arbiter Gan prior to the issuance of the NLRC Resolution overturning the
same.

2.

After petitioner was served with the Writ of Execution dated April 22, 2002 he
promised that he would first refer the matter to his counsel as he could not
effectively act on the order of execution without the latters advice. He gave his
word that upon conferment with his lawyer; he will inform the Office of the
Labor Arbiter of his action on the writ. Petitioner, however, without any
satisfactory reason, failed to fulfill this promise and respondents remained to be
not reinstated until the NLRC resolved petitioners appeal. Evidently, the delay
in the execution of respondents reinstatement was due to petitioners
unjustified refusal to affect the same.
37. GARCIA v KJ COMMERCIAL & REYNALDO QUE

FACTS: Respondent owns trucks and engages in the business of distributing


cement products. It employed as truck drivers and truck helpers petitioners.
petitioners demanded for a P40 daily salary increase. To pressure KJ Commercial

to grant their demand, they stopped working and abandoned their trucks at the
Northern Cement Plant Station in Sison, Pangasinan. They also blocked other
workers from reporting to work. Petitioners then filed with the LA a complaint
for illegal dismissal, underpayment of salary and non-payment of service
incentive leave and thirteenth month pay.

In his 30 October 2008 Decision, LA decided in favor of respondents


stating that complainants were illegally dismissed from their work and they are
entitled to their separation in lieu of reinstatement equivalent to their salary for
one (1) month for every year of service and backwages from the time that they
were terminated. KJ Commercial appealed to the NLRC. It filed before the NLRC a
motion to reduce bond and posted a P50,000 cash bond. In its 9 March 2009
Decision, the NLRC dismissed the appeal for non-perfection of the appeal and
held that the P50,000.00 cash bond posted by respondents-appellants which
represents less than two (2) percent of the monetary award is dismally
disproportionate to the monetary award of P2,612,930.00 and that the amount
of bond posted by respondents-appellants is not reasonable in relation to the
monetary award. A motion to reduce bond that does not satisfy the conditions
required under NLRC Rules shall not stop the running of the period to perfect an
appeal.

KJ filed a motion for reconsideration and posted a P2,562,930 surety


bond and . In its 8 February 2010 Resolution, NLRC granted the motion stating
that there has been an honest effort by the appellants to comply with putting up
the full amount of the required appeal bond. Moreover, considering the merit of
the appeal, by granting the motion for reconsideration, the paramount interest of
justice is better served in the resolution of this case. Petitioners filed an MR
which was denied by NLRC on June 25, 2010. They then filed a petition for
certiorari with the CA. CA affirmed the decision of NLRC on 29 April 2011.
ISSUES: Whether or not LAs 30 October 2008 Decision became final
and executory for non perfection of the appeal; thus, the NLRCs 8 February and
25 June 2010 Resolutions and the Court of Appeals 29 April 2011 Decision are
void for lack of jurisdiction?

3E Andaya Ching Espiritu Hefti Galvez Gammad Lainez Lui Madamba Nagera Narvasa
Ong Palangdao Rosales Sanchez Santos Satrain Tabo (2014-2015)

Labor Relations Case Digest - Atty. Joyrich Golangco

HELD: NO. Petitioner contends that the labor arbiters monetary award has
already reached finality, since private respondents were not able to file a timely
appeal before the NLRC, since they have failed to post the required surety bond.
KJ Commercials filing of a motion to reduce bond and delayed posting of
the P2,562,930 surety bond did not render the Labor Arbiters 30 October 2008
Decision final and executory. The Rules of Procedure of the NLRC allows the
filing of a motion to reduce bond subject to two conditions: (1) there is
meritorious ground, and (2) a bond in a reasonable amount is posted. The filing
of a motion to reduce bond and compliance with the two conditions stop the
running of the period to perfect an appeal.
The NLRC has full discretion to grant or deny the motion to reduce
bond, and it may rule on the motion beyond the 10-day period within which to
perfect an appeal. Obviously, at the time of the filing of the motion to reduce
bond and posting of a bond in a reasonable amount, there is no assurance
whether the appellants motion is indeed based on meritorious ground and
whether the bond he or she posted is of a reasonable amount. Thus, the
appellant always runs the risk of failing to perfect an appeal.

While the bond requirement on appeals involving monetary awards has


been relaxed in certain cases, this can only be done where there was substantial
compliance of the Rules or where the appellants, at the very least, exhibited
willingness to pay by posting a partial bond.

In the present case, KJ Commercial showed willingness to post a partial


bond. In fact, it posted a P50,000 cash bond. When the NLRC denied its motion,
KJ Commercial filed a motion for reconsideration and posted the full P2,
562,930 surety bond. The NLRC then granted the motion for reconsideration.
38. ONG vs CA

FACTS: Ong, owner of Milestone Metal Mfg. was charged before the NLRC for
illegal dismissal, underpayment of wages and non-payment of monetary benefits
by his employees, herein private respondents. Ong allegedly had to adopt a
rotation scheme reducing his employees workdays to 3 days a week or less for

indefinite period due to economic crisis affecting his business. The Labor Arbiter
ruled in favor of the respondents awarding P1.1M representing wage differential,
payment of benefits and attorneys fees and ordered Ong to pay separation pay
equivalent to month salary for every year of service of respondents. Ong filed a
notice and memorandum of appeal with the NLRC but instead of posting the
required cash or surety bond, he filed a motion to reduce the appeal bond. The
NLRC denied the motion and the subsequent motion for reconsideration due to
Ongs failure to post the required cash or surety bond. On appeal for certiorari,
the CA dismissed Ongs appeal for non-perfection of appeal and his subsequent
appeal for reconsideration was likewise dismissed for lack of merit.

ISSUES: Whether or not the CA erred in dismissing petitioners appeal which


was seasonably filed?

HELD: NO. The right to appeal is not a natural right but a statutory privilege i.e.
it must comply with the requirements of the rule or law. Art. 223 of the Labor
Code provides: Appeal. Decisions, awards, or orders of the Labor Arbiter are
final and executory xxx xxx In case of a judgment involving a monetary award, an
appeal by the employer may be perfected only upon the posting of a cash or
surety bond xxx xxx in the amount equivalent to the monetary award in the
judgment appealed from. Section 3, Rule VI of the New Rules of Procedure of the
NLRC provides: Requisites for Perfection of Appeal. (a) The appeal shall be
filed within the reglementary period as provided in Section 1 of this Rule; shall
be under oath with proof of payment of the required appeal fee and the posting
of a cash or surety bond as provided in Section 5 of this Rule; shall be
accompanied by a memorandum of appeal which shall state the grounds relied
upon and the arguments in support thereof; the relief prayed for; and a
statement of the date when the appellant received the appealed decision, order
or award and proof of service on the other party of such appeal. A mere notice of
appeal without complying with the other requisite aforestated shall not stop the
running of the period for perfecting an appeal. The posting of cash or surety
bond is not only mandatory but jurisdictional as well, and non-compliance
therewith is fatal and has the effect of rendering the judgment final and
executory (Phil.Transmarine Carriers, Inc. v. Cortina, G.R. No. 146094, 12
November 2003). For petitioners failure to post a full or partial appeal bond
within the prescribed period, his appeal was not perfected and the NLRC has no

3E Andaya Ching Espiritu Hefti Galvez Gammad Lainez Lui Madamba Nagera Narvasa
Ong Palangdao Rosales Sanchez Santos Satrain Tabo (2014-2015)

Labor Relations Case Digest - Atty. Joyrich Golangco

authority to entertain the appeal nor reverse the decision of the Labor Arbiter
which has become final and executory.

The Court denied and dismissed the petition for lack of merit, affirming the
assailed decision of the Court of Appeals. No costs.
39. ROSEWOOD PROCESSING INC. vs NLRC

FACTS: Six (6) security guards, private respondents herein, filed a complaint
against their employer, Veterans Phil. Scout Security Agency (Veterans for
brevity), for illegal dismissal, unpaid wages, underpayment of wages and nonpayment of other monetary benefits. Petitioner Rosewood was impleaded as
third-party respondent by Veterans being an indirect employer of the guards
during some points in time. The Labor Arbiter rendered a decision ordering
Veterans and Rosewood to pay jointly and severally the complainants a total of
P789,154.39 plus attorneys fees. Rosewood appealed to the NLRC but was
dismissed for failure to file the required appeal bond within the reglementary
period. Rosewood filed a motion for reconsideration contending that it timely
filed a Notice of Appeal with Memorandum on Appeal, a Motion to Reduce Appeal
Bond and a surety bond of P50,000 issued by Prudential Guarantee and
Assurance, Inc. The NLRC denied the motion for recon for lack of merit.
ISSUES:

1). Whether or not petitioners appeal to the NLRC was perfected on time?

2). Whether or not petitioner is solidarily liable with the security agency for
payment of salary differentials to the complainants?
HELD:

1). YES. While it is indisputable that the appeal to the Commission of the
decisions, awards or orders of the Labor Arbiter should be made within ten (10)
calendar days from receipt of such decisions, awards, or orders and may be
perfected only upon posting of a cash or surety bond in an amount equivalent to
the monetary award in the judgment appealed from (Art. 223 of the Labor Code),

the Court on some cases (Quiambao vs NLRC, 254 SCRA 211, 216-217, March 4,
1996; Globe Gen. Services & Security Agency vs NLRC, 249 SCRA 408, 414-415,
October 23 1995) had relaxed the appeal bond requirement if justified by
substantial compliance with the rule. The Court noted that the NLRC dismissed
the appeal due to late filing of the appeal bond while petitioner filed its
memorandum of appeal 10 days after its receipt of the arbiters decision together
with a motion to reduce the appeal bond accompanied by a P50,000 surety bond.
The Court held that with petitioners motion to reduce the bond together with a
surety bond, it has substantially complied with the Labor Code. The Court also
finds petitioners motion to reduce the appeal bond meritorious while the NLRC
acted with grave abuse of discretion in ignoring the merits of petitioners
motions which caused its dismissal.

2). YES. The liability of petitioner as an indirect employer was provided under
Articles 106, 107 and 109 of the Labor Code. However, petitioners liability to the
guards as contractor of the security agency extends only to the period during
which they are working for the petitioner and ends when the guards were
reassigned to another contractor. Further, petitioners liability is only limited to
the wage difference if any, between the contract price with the security agency
and the statutory minimum wage. Petitioner cannot be held liable for back wages
and separation pay as there is no showing that it committed or conspired in the
illegal dismissal of the security guards.
The Court partially granted the petition. The assailed decision of the NLRC was
modified such that petitioner, with the Security Agency, is solidarily liable to pay
complainants wage differentials during the period that the complainants were
actually under its employ. Petitioner was exonerated from payment of back
wages and separation pay. The NLRC was required to recompute the wage
differentials liability of petitioner within 15 days from finality of its decision. No
pronouncement as to costs.
40. FILIPINAS SYSTEMS, INC. vs. NLRC

FACTS: A complaint for illegal dismissal and monetary claims were filed by
private respondents against their employer, Filipinas Systems, Inc. (Filsystems
for brevity). Filsystems failed to file its position paper in spite of the order of the

3E Andaya Ching Espiritu Hefti Galvez Gammad Lainez Lui Madamba Nagera Narvasa
Ong Palangdao Rosales Sanchez Santos Satrain Tabo (2014-2015)

Labor Relations Case Digest - Atty. Joyrich Golangco

Labor Arbiter prompting the Labor Arbiter to decide in favor of respondents in


the illegal dismissal complaints and awarded their monetary claims. Filsystems
appealed to the NLRC submitting for the first time evidence showing that
respondents were project employees whose dismissal was due to the
discontinuation of the project they were assigned. Respondents questioned the
jurisdiction of the NLRC over the appeal as petitioner belatedly file the appeal
bond however, the NLRC assumed jurisdiction and remanded the case to the
Labor Arbiter for further proceedings. Respondents motion for reconsideration
was denied so they appeal to the CA via a Petition for Certiorari. The CA ruled
that the NLRC lacks jurisdiction over the appeal for late filing of the appeal bond
and reinstated the Labor Arbiters decision. Peitioners motion for
reconsideration was denied.

ISSUES: Whether or not the NLRC acquired jurisdiction over petitioners appeal?
HELD: NO. Art. 223 of the Labor Code and Section 1 of the NLRC Rules of
Procedure provide a ten (10)-day period from receipt of the decision of the
Arbiter to file an appeal together with an appeal bond if the decision involves a
monetary award. Records showed that petitioners received a copy of the
Arbiters decision on October 31. Their memorandum of appeal was dated
November 9, but their appeal bond was executed only on November 17; no
partial payment of the bond was made within the reglementary period nor did
they submit an explanation for its late filing. Thereof, the late filing of the bond
divested the NLRC of its jurisdiction to entertain petitioners appeal. Further,
petitioners failed to submit their evidence to the Labor Arbiter in spite of the
opportunities given them and submit the evidence instead to the NLRC when the
decision became adverse to them.
The Court dismissed the petition; the decision of the Labor Arbiter was
reinstated with the modification that if reinstatement of respondents is not
feasible, petitioner should pay their separation pay in accordance with law.
41. Buenaobra, et. al. vs Lim King Guan

FACTS: Petitioners were employees of private respondent Unix International


Export Corporation (UNIX), a corporation engaged in the business of
manufacturing bags, wallets and the like. Sometime in 1991 and 1992,
petitioners filed several cases against UNIX and its incorporators and officers for

unfair labor practice, illegal lockout/dismissal, underpayment of wages, holiday


pay, proportionate 13th month pay, unpaid wages, interest, moral and exemplary
damages and attorneys fees.

On February 23, 1993 the Labor Arbiter (LA) de Vera decided in favor of
the petitioners ordering UNIX to pay the former the following (more than
8Million):
P 5,821,838.40 as backwages;

P 1,484,912.00 as separation pay;

P 527,748.00 as wage differentials;

P 33,830.00 as regular holiday pay differentials; and

P 365,551.95 as proportionate 13th month pay for 1990.

The decision of the LA became final and executory as neither of the


parties appealed.

However, petitioners complained that the decision could not be


executed because UNIX allegedly diverted, invested and transferred all its
money, assets and properties to respondent Fuji Zipper Manufacturing
Corporation (FUJI) whose stockholders and officers were also those of UNIX.

Thus, on March 25, 1997, petitioners filed another complaint against


respondents UNIX, its corporate officers and stockholders of record, and FUJI.
Petitioners mainly prayed that respondents UNIX and FUJI be held jointly and
severally held liable for the payment of the monetary awards ordered LA de
Vera. A judgment was rendered by LA Pati in favor again of the petitioners
piercing the veil of corporate fiction of the two respondent sister companies
ordering them to pay the 8M plus 3M and 1M for moral and exemplary damages.

On July 30, 1998, FUJI, its officers and stockholders filed a memorandum
on appeal and a motion to dispense with the posting of a cash or surety appeal
bond on the ground that they were not the employers of petitioners before the
NLRC. They alleged that they could not be held responsible for petitioners claims
and to require them to post the bond would be unjust and unfair, and not

3E Andaya Ching Espiritu Hefti Galvez Gammad Lainez Lui Madamba Nagera Narvasa
Ong Palangdao Rosales Sanchez Santos Satrain Tabo (2014-2015)

Labor Relations Case Digest - Atty. Joyrich Golangco

sanctioned by law. The 3rd Division of the NLRC denied the motion to exempt to
post appeal bond and instead ordered them to post the appeal bond of 8M within
unextendable period of 10 days upon receipt.

The petitioners argued that timely posting of the appeal bond is


mandatory for the perfection of the appeal and should be complied with hence
they moved for the reconsideration of the decision rendered by the 3rd division
of the NLRC. The NLRC dismissed the petitioners MR for lack of merit and
allowed respondents Supplemental Memorandum of Appeal. Hence, petitioners
elevated it to the CA imputing grave abuse of discretion to the NLRC, Third
Division when it allowed private respondents to post the mandated cash or
surety bond four months after the filing of their memorandum on appeal. CA
dismissed it.
ISSUES:
Whether or not respondents be allowed to appeal and post an
appeal bond even beyond the reglementary period provided in Article 223 of the
Labor Code.
HELD: Affirmative. Respondent should be allowed to appeal and post appeal
bond. The petition has no merit.

The provision of Article 223 of the Labor Code requiring the posting of
bond on appeals involving monetary awards must be given liberal interpretation
in line with the desired objective of resolving controversies on the merits. If only
to achieve substantial justice, strict observance of the reglementary periods may
be relaxed if warranted. The NLRC, Third Division could not be said to have
abused its discretion in requiring the posting of bond after it denied private
respondents motion to be exempted therefrom.

It is true that the perfection of an appeal in the manner and within the
period prescribed by law is not only mandatory but jurisdictional, and failure to
perfect an appeal has the effect of making the judgment final and
executory. However, technicality should not be allowed to stand in the way of
equitably and completely resolving the rights and obligations of the parties. We
have allowed appeals from the decisions of the labor arbiter to the NLRC, even if
filed beyond the reglementary period, in the interest of justice. The facts and
circumstances of the instant case warrant liberality considering the amount

involved and the fact that petitioner already obtained a favorable judgment on
February 23, 1993 against their employer UNIX.

In the same decision which has already become final and executory,
labor arbiter de Vera HELD:

This Branch upholds and maintains in the absence of substantial


evidence to the contrary that both respondent corporations have legitimate
distinct and separate juridical personalities. Thus, respondent Fuji Zipper
Manufacturing, Inc. has been erroneously impleaded in this case.

It is only fair and just that respondent FUJI be afforded the opportunity
to be heard on appeal before the NLRC, especially in the light of labor arbiter
Patis later decision holding FUJI jointly and severally liable with UNIX in the
payment of the monetary awards adjudged by labor arbiter de Vera against
UNIX.

In the absence of any showing that the NLRC committed grave abuse of
discretion, or otherwise acted without or in excess of jurisdiction, this Court is
bound by its findings. Furthermore, the Court of Appeals upheld the assailed
orders of the said Commission.
42. Lepanto vs Belio Icao

FACTS: Respondent Icao filed a complaint for illegal dismissal and damages
against herein petitioner Lepanto Consolidated Mining Company (LCMC) and its
CEO Felipe Yap before the Arbitration Branch of the NLRC. Icao was a former
employee of the company and worked as the Lead Miner in its underground
mine in Paco, Mankayan, Benguet. On February 4, 2008, the company ordered
the dismissal from employment of Icao due to breach of trust and confidence and
act of highgrading (or act of concealing, possessing or unauthorized extraction of
highgrade material/ore without proper authority). This order stemmed from
what transpired on January 4, 2008, when Icao allegedly had in his possession a
wrapped object containing gold bearing highgrade ores found in his skullguard
upon being apprehended by the security guards of the company. Icao denied the
charge against him and claimed that his dismissal from work was without just or
authorized cause.

3E Andaya Ching Espiritu Hefti Galvez Gammad Lainez Lui Madamba Nagera Narvasa
Ong Palangdao Rosales Sanchez Santos Satrain Tabo (2014-2015)

Labor Relations Case Digest - Atty. Joyrich Golangco

The Labor Arbiter ruled in favor of Icao on September 30, 2008. It found
out that the charge of highgrading was fabricated and there was no just cause for
the dismissal of respondent. It further concluded that the claim of the security
guards that Icao had inserted ores in his boots while in a standing position was
not in accord with normal human physiological functioning and that it was
inconsistent with normal human behavior for a man, who knew that he was
being chased for allegedly placing wrapped ore inside his boots, to then transfer
the ore to his skullguard, where it could be found once he was apprehended.
LCMC was ordered to pay Icao his full backwages amounting to P345, 879.45.

LCMC appealed to the NLRC. On February 29, 2009, the NLRC 1st
Division ruled for the dismissal on the ground that there was non-perfection of
the appeal provided for under Article 223 of the Labor Code and consequently
declaring LAs decision to be final and executory. It noted that instead of posting
an appeal bond required under the Labor Code for the perfection of an appeal,
LCMC and its CEO filed a Consolidated Motion For Release Of Cash Bond And To
Apply Bond Subject For Release As Payment For Appeal Bond (Consolidated
Motion). They requested therein that the NLRC release the cash bond of
P401,610.84, which they had posted in the separate case Dangiw Siggaao v.
LCMC, and apply the same cash bond to their present appeal bond liability. They
reasoned that since this Court had already decided Dangiw Siggaao in their favor,
and that the ruling therein had become final and executory, the cash bond posted
therein could now be released. They also cited financial difficulty as a reason for
resorting to this course of action and prayed that, in the interest of justice, the
motion be granted.

An appeal before the CA was made but to no avail, said court affirmed
the NLRCs decision of dismissal due to non-perfection of appeal, that the posting
of the appeal bond is indispensable jurisdictional requirement. However, the CA
dropped the CEO as a party to this case as it found that no specific act was
alleged in private respondents pleadings to show that he had a hand in Icaos
illegal dismissal; much less, that he acted in bad faith.
ISSUES: Whether or not petitioner complied with the appeal bond requirement
under the Labor Code and the NLRC Rules by filing a Consolidated Motion to
release the cash bond it posted in another case, which had been decided with

finality in its favor, with a view to applying the same cash bond to the present
case.

HELD: Affirmative, We reiterate our pronouncement in Araneta v.


Rodas,22where the Court said that when the law does not clearly provide a rule
or norm for the tribunal to follow in deciding a question submitted, but leaves to
the tribunal the discretion to determine the case in one way or another, the judge
must decide the question in conformity with justice, reason and equity, in view of
the circumstances of the case. Applying this doctrine, we rule that petitioner
substantially complied with the mandatory requirement of posting an appeal
bond for the reasons explained below.
First, there is no question that the appeal was filed within the 10-day
reglementary period.23Except for the alleged failure to post an appeal bond, the
appeal to the NLRC was therefore in order.
Second, it is also undisputed that petitioner has an unencumbered
amount of money in the form of cash in the custody of the NLRC in the separate
case Dangiw Siggaao, which was earlier decided in its favor.

Under the Rule VI, Section 6 of the 2005 NLRC Rules, "[a] cash or surety
bond shall be valid and effective from the date of deposit or posting, until the
case is finally decided, resolved or terminated, or the award satisfied." Hence, it
is clear that a bond is encumbered and bound to a case only for as long as 1) the
case has not been finally decided, resolved or terminated; or 2) the award has
not been satisfied. Therefore, once the appeal is finally decided and no award
needs to be satisfied, the bond is automatically released. Since the money is now
unencumbered, the employer who posted it should now have unrestricted access
to the cash which he may now use as he pleases as appeal bond in another case,
for instance. This is what petitioner simply did.
Third, the cash bond in the amount ofP401,610.84 posted in Dangiw
Siggaao is more than enough to cover the appeal bond in the amount
ofP345,879.45 required in the present case.

Fourth, this ruling remains faithful to the spirit behind the appeal bond
requirement which is to ensure that workers will receive the money awarded in
their favor when the employers appeal eventually fails.There was no showing at

3E Andaya Ching Espiritu Hefti Galvez Gammad Lainez Lui Madamba Nagera Narvasa
Ong Palangdao Rosales Sanchez Santos Satrain Tabo (2014-2015)

Labor Relations Case Digest - Atty. Joyrich Golangco

all of any attempt on the part of petitioner to evade the posting of the appeal
bond

Court found exceptional circumstances that warranted an extraordinary


exercise of its power to exempt a party from the rules on appeal bond, there is all
the more reason in the present case to find that petitioner substantially complied
with the requirement. We emphasize that in this case we are not even exempting
petitioner from the rule, as in fact we are enforcing compliance with the posting
of an appeal bond. We are simply liberally applying the rules on what constitutes
compliance with the requirement, given the special circumstances surrounding
the case as explained above.
43. Bergonio vs SEAIR

FACTS: In 2004, petitioners filed before the LA a complaint for illegal dismissal
and illegal suspension with prayer of reinstatement against respondents. On May
31, 2005, the LA ruled in favor of the petitioners that they were illegally
dismissed and suspended and ordered respondent among others, to immediately
reinstate the petitioners with full backwages. On August 20, 2005, petitioners
filed before LA a Motion for issuance of Writ of Execution and for their
immediate reinstatement. A pre-execution conference was made and respondent
manifested their intention to reinstate petitioners in the payroll but it did not
materialize and instead the respondents subsequently filed an Opposition to the
Motion for execution on October 3, 2005 arguing its strained relationship with
the petitioners because of threatening text messages.
On October 7, 2005, LA issued a Writ of Execution. However, it was
returned unsatisfied prompting the petitioners to file for a recomputation of
accrued wages that was granted by LA.

On February 21, 2006, FOUR MONTHS after the Oct. 7, 2005 issuance of
the Writ of Execution, repondents issued a Memorandum directing petitioner to
report for work on February 24, 2006 at Clark-Field, Pampanga but petitioners
failed to report prompting the respondent to move for the suspension of the
reinstatement.

Meanwhile, the May 31, 2005 decision was appealed by the respondent
to the NLRC but the latter dismissed the same on the ground of non-perfection of

appeal on Nov. 29, 06. And on February 6, 2007, the NLRC issued an Entry of
Judgment declaring the Nov 29, 06 decision final and executory. In this
regard, another Writ of Execution was issued by the LA together with A Notice of
Garnishment sent to the respondents depositary bank in the amount of 1.9M.
On Dec. 18, 07 respondents got a partly favorable decision when it
elevated the case to the CA, ruling that the dismissal of the petitioners was in fact
valid.

But the petitioners filed with LA an Urgent Ex-Parte Motion for the
Immediate Release of the Garnished amount and LA noted that due to
respondents refusal to reinstate petitioners despite the final and executory
nature of the Reinstatement Order on May 31, 05 - the accrued wages should be
computed until the the Dec. 18, 07 CA decision reversing LAs earlier ruling of
illegal dismissal, - amounting to P 3, 078, 366.33. The LA granted the Urgent ExParte Motion and affirmed in toto by the NLRC on July 16, 08.

Again, the respondent assailed the July 16, 08 NLRC decision before the
CA. The appellate court ruled in favor of respondents and remanded the case to
the Computation and Examination Unit of the NLRC for proper computation only
up to Feb. 24, 06. Furthermore, CA agrees that the Reinstatement is immediate
in nature and should be executed even pending appeal until the decision is
reversed by a higher court --- applying this principle, the computation should be
up to Dec. 18 CA decision BUT the CA further pointed out that the petitioners
cannot do this computation because the delay of the reinstatement was
WITHOUT the fault of the employer as it was petitioners who did not show up on
Feb. 24 return-to-work order of the respondent, in effect barring them to
compute backwages up to Dec. 18, 07. Accordingly, CA reversed, for grave
abuse of discretion, the NLRC July 16, 08 decision that affirmed LAs order to
release the garnished amount and recomputation of backwages.
ISSUES: Whether or not the CA correctly found the NLRC in grave abuse of
discretion in affirming the release of the garnished amount despite the
respondents issuance of and the petitioners failure to comply with the February
21, 2006 return-to-work Memorandum.
HELD: Negative. Under Article 223 (now 229) paragraph 3 of our Labor Code:

3E Andaya Ching Espiritu Hefti Galvez Gammad Lainez Lui Madamba Nagera Narvasa
Ong Palangdao Rosales Sanchez Santos Satrain Tabo (2014-2015)

Labor Relations Case Digest - Atty. Joyrich Golangco


Xxxx

In any event, the decision of the Labor Arbiter reinstating a


dismissed or separated employee, insofar as the
reinstatement aspect is concerned, shall immediately be
executory, pending appeal. The employee shall either be
admitted back to work under the same terms and
conditions prevailing prior to his dismissal or separation
or, at the option of the employer, merely reinstated in the
payroll. The posting of a bond by the employer shall not
stay the execution for reinstatement provided herein.

Otherwise stated, a dismissed employee whose case was favorably


decided by the LA is entitled to receive wages pending appeal upon
reinstatement, which reinstatement is immediately executory. Unless the
appellate tribunal issues a restraining order, the LA is duty bound to implement
the order of reinstatement and the employer has no option but to comply with it.
Moreover, and equally worth emphasizing, is that an order of reinstatement
issued by the LA is self- executory, i.e., the dismissed employee need not even
apply for and the LA need not even issue awrit of execution to trigger the
employers duty to reinstate the dismissed employee.

The reversal by a higher tribunal of the LAs finding (of illegal dismissal),
notwithstanding, an employer, who, despite the LAs order of reinstatement, did
not reinstate the employee during the pendency of the appeal up to the reversal
by a higher tribunal may still be held liable for the accrued wages of the
employee, i.e., the unpaid salary accruing up to the time the higher tribunal
reverses the decision. The rule, therefore, is that an employee may still recover
the accrued wages up to and despite the reversal by the higher tribunal. This
entitlement of the employee to the accrued wages proceeds from the immediate
and self-executory nature of the reinstatement aspect of the LAs decision.
There are only 2 exceptions in the above rule:

1) actual delay or the fact that the order of reinstatement


pending appeal was not executed prior to its reversal; and
2) the delay must not be due to the employers unjustified act or
omission

In reversing the CAs decision for its legal error we apply this exception
as two-fold test:

FIRST, the existence of delay, whether there was actual delay or whether
the order of reinstatement pending appeal was not executed prior to its reversal?
We answer this test in the affirmative.

To recall, on May 31, 2005, the LA rendered the decision finding the
petitioners illegally dismissed and ordering their immediate reinstatement. Per
the records, the respondents received copy of this decision on July 8, 2005. From
the time the respondents received copy of the LAs decision, and the issuance of
the writ of execution, until the CA reversed this decision on December 17, 2008,
the respondents had not reinstated the petitioners, either by actual
reinstatement or in the payroll.
From these facts and without doubt, there was actual delay in the
execution of the reinstatement aspect of the LAs May 31, 2005 decision before it
was reversed in the CAs decision.

SECOND, the cause of the delay whether the delay was not due to the
employers unjustified act or omission. We answer this test in the negative; we
find that the delay in the execution of the reinstatement pending appeal was due
to the respondents unjustified acts.

In reversing, for grave abuse of discretion, the NLRCs order affirming


the release of the garnished amount, the CA relied on the fact of the issuance of
the February 21, 2006 Memorandum and of the petitioners failure to comply
with its return-to-work directive. In other words, with the issuance of this
Memorandum, the CA considered the respondents as having sufficiently
complied with their obligation to reinstate the petitioners. And, the subsequent
delay in or the non-execution of the reinstatement order was no longer the
respondents fault, but rather of the petitioners who refused to report back to
work despite the directive.
The Court is convinced that the delay in the reinstatement pending
appeal was due to respondents fault. For one, respondent filed several pleading
to suspend the execution of reinstatement. These pleadings to our mind show a
determined effort on the respondents part to prevent or suspend the execution

3E Andaya Ching Espiritu Hefti Galvez Gammad Lainez Lui Madamba Nagera Narvasa
Ong Palangdao Rosales Sanchez Santos Satrain Tabo (2014-2015)

Labor Relations Case Digest - Atty. Joyrich Golangco

of reinstatement pending appeal. Another is that NO actual intention to reinstate


despite return-to-work directive being issued as it was only sent to one paryt
(Pelaez) who did not act in representation of the others hence there was really
no sufficient notice.

All told, under the facts and the surrounding circumstances, the delay
was due to the acts of the respondents that we find were unjustified. We reiterate
and emphasize, Article 223, paragraph 3, of the Labor Code mandates the
employer to immediately reinstate the dismissed employee, either by actually
reinstating him/her under the conditions prevailing prior to the dismissal or, at
the option of the employer, in the payroll. The respondents' failure in this case to
exercise either option rendered them liable for the petitioners' accrued salary
until the LA decision was reversed by the CA on December 17, 2008. We,
therefore, find that the NLRC, in affirming the release of the garnished amount,
merely implemented the mandate of Article 223; it simply recognized as
immediate and self-executory the reinstatement aspect of the LA's decision.
44: LOON vs POWER MASTER

FACTS: Respondents Power Master, Inc. and Tri-C General Services employed
and assigned the petitioners as janitors and leadsmen in various Philippine Long
Distance Telephone Company (PLDT) offices in Metro Manila area. Subsequently,
the petitioners filed a complaint for money claims against Power Master, Inc.,
Tri-C General Services and their officers, the spouses Homer and Carina Alumisin
(collectively, the respondents). The petitioners alleged in their complaint that
they were not paid minimum wages, overtime, holiday, premium, service
incentive leave, and thirteenth month pays. They further averred that the
respondents made them sign blank payroll sheets. On June 11, 2001, the
petitioners amended their complaint and included illegal dismissal as their cause
of action. They claimed that the respondents relieved them from service in
retaliation for the filing of their original complaint. Notably, the respondents did
not participate in the proceedings before the Labor Arbiter except on April 19,
2001 and May 21, 2001 when Mr. Romulo Pacia, Jr. appeared on the respondents
behalf. The respondents counsel also appeared in a preliminary mandatory
conference on July 5, 2001.

LAs Ruling: The LA awarded the petitioners salary differential, service incentive

leaves and 13th month pays. In awarding these claims the LA stated that the
burden in proving the payment of these money claims rests with the employer.
However, they were not awarded backwages, overtime, holiday and premium
pays for failure to show that they rendered overtime work and worked on
holidays. Moreover, it was not decided that they were illegally dismissed for
failure to show notice of termination of employment.
NLRC: Both arties appealed to the ruling of the LA. NLRC affirmed LAs ruling
with regard the payment of holiday pay and attorneys fees but vacated the
awards of salary differential, 13th month pays and service incentive leaves.
Moreover, NLRC allowed the respondents to present pieces of evidence for the
first time on appeal on the ground that they have been deprived of due process. It
also ruled that petitioners were legally dismissed due to gross misconduct.
CA: Ruling of the NLRC was affirmed.

ISSUES: Whether the respondents perfected their appeal before the NLRC

HELD: Pursuant to Article 223 of the Labor Code, an appeal by the employer
may be perfected only upon the posting of a cash or surety bond issued by a
reputable bonding company duly accredited by the Commission in the amount
equivalent to the monetary award in the judgment appealed from. In the
present case, the respondents filed a surety bond issued by Security Pacific
Assurance Corporation (Security Pacific) on June 28, 2002. At that time, Security
Pacific was still an accredited bonding company. However, the NLRC revoked its
accreditation on February 16, 2003. Nonetheless, this subsequent revocation
should not prejudice the respondents who relied on its then subsisting
accreditation in good faith. In Del Rosario v. Philippine Journalists, Inc., we ruled
that a bonding companys revocation of authority is prospective in application.
However, the respondents should post a new bond issued by an accredited
bonding company in compliance with paragraph 4, Section 6, Rule 6 of the NLRC
Rules of Procedure. This provision states that [a] cash or surety bond shall be
valid and effective from the date of deposit or posting, until the case is finally
decided, resolved or terminated or the award satisfied.
45. Mcburnie vs Ganzon

3E Andaya Ching Espiritu Hefti Galvez Gammad Lainez Lui Madamba Nagera Narvasa
Ong Palangdao Rosales Sanchez Santos Satrain Tabo (2014-2015)

Labor Relations Case Digest - Atty. Joyrich Golangco

FACTS: On October 2002, McBurnie, an Australian national, instituted a


complaint for illegal dismissal and other monetary claims against the
respondents. McBurnie claimed that on May 11, 1999, he signed a five-year
employment agreement with the company EGI as an
Executive Vice-president who shall oversee the management of the companys
hotels and resorts within the Philippines. He performed work for the company
until sometime in November 1999, when he figured in an accident that
compelled him to go back to Australia while recuperating from his injuries. While
in Australia, he was informed by respondent Ganzon that his services were no
longer needed because their intended project would no longer push through. The
respondents opposed the complaint, contending that their agreement with
McBurnie was to jointly invest in and establish a company for the management of
hotels. They did not intend to create an employer employee relationship, and the
execution of the employment contract that was being invoked by McBurnie was
solely for the purpose of allowing McBurnie to obtain an alien work permit in the
Philippines. At the time McBurnie left for Australia for his medical treatment, he
had not yet obtained a work permit.
LAs Ruling: The LA declared McBurnie as having been illegally dismissed from
employment, and thus entitled to receive from the respondents the following
amounts:
(a) US$985,162.00 as salary and benefits for the unexpired term of their
employment contract,
(b) P2, 000,000.00 as moral and exemplary damages, and (c) attorneys fees
equivalent to 10% of the total monetary award.

The respondents appealed the LAs Decision to the NLRC. On November 2004,
they filed their Memorandum of Appeal and Motion to Reduce Bond, and posted
an appeal bond in the amount of P100,000.00. The respondents contended in
their Motion to Reduce Bond, inter alia, that the monetary awards of the LA were
null and excessive, allegedly with the intention of rendering them incapable of
posting the necessary appeal bond. They claimed that an award of "more than
P60 Million Pesos to a single foreigner who had no work permit and who left the
country for good one month after the purported commencement of his
employment" was a patent nullity.

NLRC: NLRC denied the motion to reduce bond, explaining that "in cases
involving monetary award, an employer seeking to appeal the LAs decision to
the Commission is unconditionally required by Art. 223, Labor Code to post bond
in the amount equivalent to the monetary award.
CA: Petitioners Motion to Reduce Appeal Bond was granted. Petitioners were
directed to post appeal bond in the amount of P10,000,000.00. The NLRC was
also directed to give due course to petitioners appeal which was ordered to be
remanded to the NLRC for further proceedings. The CA explained that "while Art.
223 of the Labor Code requiring bond equivalent to the monetary award is
explicit, Section 6, Rule VI of the NLRC Rules of Procedure, as amended,
recognized as exception a motion to reduce bond upon meritorious grounds and
upon posting of a bond in a reasonable amount in relation to the monetary
award." Moreover, the appellate court ruled that such bond was unreasonable
and excessive.
ISSUES: Whether or not the appeal bond should be reduced

HELD: In accordance with the foregoing, although the general rule provides that
an appeal in labor cases from a decision involving a monetary award may be
perfected only upon the posting of a cash or surety bond, the Court has relaxed
this requirement under certain exceptional circumstances in order to resolve
controversies on their merits. These circumstances include:
(1) the fundamental consideration of substantial justice
(2) the prevention of miscarriage of justice or of unjust enrichment and
(3) special circumstances of the case combined with its legal merits, and the
amount and the issue involved.

The bond requirement in appeals involving monetary awards has been and may
be relaxed in meritorious cases, including instances in which (1) there was
substantial compliance with
the Rules, (2) surrounding facts and circumstances constitute meritorious
grounds to reduce the bond, (3) a liberal interpretation of the requirement of an
appeal bond would serve the desired objective of resolving controversies on the
merits, or (4) the appellants, at the very least, exhibited their willingness and/or
good faith by posting a partial bond during the reglementary period. The Court

3E Andaya Ching Espiritu Hefti Galvez Gammad Lainez Lui Madamba Nagera Narvasa
Ong Palangdao Rosales Sanchez Santos Satrain Tabo (2014-2015)

Labor Relations Case Digest - Atty. Joyrich Golangco

held that the reduction decided upon by the CA was the reasonable amount to be
posted as bond.
46. PIONEER TEXTURIZING CORP vs. NLRC

FACTS:

Private respondent Lourdes A. de Jesus is petitioners reviser/trimmer


since 1980. As reviser/trimmer, de Jesus based her assigned work on a
paper note, identified by its P.O. Number. On August 15, 1992, de Jesus
worked on P.O. No. 3853 by trimming the cloths ribs. She thereafter
submitted tickets corresponding to the work done to her
supervisor. Three days later, de Jesus received from petitioners
personnel manager a memorandum requiring her to explain why no
disciplinary action should be taken against her for dishonesty and
tampering of official records and documents with the intention of
cheating as P.O. No. 3853 allegedly required no trimming. The
memorandum also placed her under preventive suspension for thirty
days starting from August 19, 1992. In her handwritten explanation, de
Jesus maintained that she merely committed a mistake in trimming P.O.
No. 3853 as it has the same style and design as P.O. No. 3824 which has
an attached price list for trimming the ribs and admitted that she may
have been negligent in presuming that the same work was to be done
with P.O. No. 3853, but not for dishonesty or tampering Petitioners
personnel department, nonetheless, she was terminated.

NLRC: In its July 21, 1994 decision, the NLRC ruled that de Jesus was negligent in
presuming that the ribs of P.O. No. 3853 should likewise be trimmed for having
the same style and design as P.O. No. 3824, thus petitioners cannot be entirely
faulted for dismissing de Jesus. The NLRC declared that the status quo between
them should be maintained and affirmed the Labor Arbiter's order of
reinstatement, but without back wages. The NLRC further "directed petitioner to
pay de Jesus her back salaries from the date she filed her motion for execution on
September 21, 1993 up to the date of the promulgation of the decision.

On September 22, 1992, de Jesus filed a complaint for illegal dismissal


against petitioners with the Labor Arbiter.

Labor Arbiter: The Labor Arbiter who heard the case noted that de Jesus was
amply accorded procedural due process in her termination from service.
Nevertheless, after observing that de Jesus made some further trimming on P.O.
No. 3853 and that her dismissal was not justified, the Labor Arbiter held
petitioners guilty of illegal dismissal. Petitioners were accordingly ordered to
reinstate de Jesus to her previous position without loss of seniority rights and
with full back wages from the time of her suspension on August 19, 1992

Petitioners appealed to the public respondent National Labor Relations


Commission (NLRC)

Petitioners insist that the NLRC gravely abused its discretion in holding
that de Jesus is entitled to reinstatement to her previous position for she
was not illegally dismissed in the first place. Petitioners further add that
de Jesus breached the trust reposed in her, hence her dismissal from
service is proper on the basis of loss of confidence, citing as authority
the cases of Ocean Terminal Services, Inc. v. NLRC, 197 SCRA 491; CocaCola Bottlers Phil., Inc. v. NLRC, 172 SCRA 751, and Piedad v. Lanao del
Norte Electric Cooperative,154 SCRA 500.

Petitioners' also argued the theory that an order for reinstatement is not
self-executory. They stress that there must be a writ of execution which
may be issued by the NLRC or by the Labor Arbiter motu proprio or on
motion of an interested party. They further maintain that even if a writ
of execution was issued, a timely appeal coupled by the posting of
appropriate supersedes as bond, which they did in this case, effectively
forestalled and stayed execution of the reinstatement order of the Labor
Arbiter.
Private respondent de Jesus, for her part, maintains that petitioners
should have reinstated her immediately after the decision of the Labor
Arbiter ordering her reinstatement was promulgated since the law
mandates that an order for reinstatement is immediately executory. An
appeal, she says, could not stay the execution of a reinstatement order
for she could either be admitted back to work or merely reinstated in the

3E Andaya Ching Espiritu Hefti Galvez Gammad Lainez Lui Madamba Nagera Narvasa
Ong Palangdao Rosales Sanchez Santos Satrain Tabo (2014-2015)

Labor Relations Case Digest - Atty. Joyrich Golangco

payroll without need of a writ of execution. De Jesus argues that a writ


of execution is necessary only for the enforcement of decisions, orders,
or awards which have acquired finality.
Issues:
(1) Whether or not de Jesus was illegally dismissed.
(2) Whether or not an order for reinstatement needs a writ of execution.
HELD:

(1) Yes, de Jesus was illegally dismissed. Based on the Labor Arbiter's
observations or from the NLRC's assessment, it distinctly appears that
petitioners' accusation of dishonesty and tampering of official records and
documents with intention of cheating against de Jesus was not substantiated by
clear and convincing evidence. Petitioners simply failed, both before the Labor
Arbiter and the NLRC, to discharge the burden of proof and to validly justify de
Jesus' dismissal from service. The law, in this light, directs the employers, such as
herein petitioners, not to terminate the services of an employee except for a just
or authorized cause under the Labor Code. Lack of a just cause in the dismissal
from service of an employee, as in this case, renders the dismissal illegal, despite
the employer's observance of procedural due process.And while the NLRC stated
that "there was no illegal dismissal to speak of in the case at bar" and that
petitioners cannot be entirely faulted therefor, said statements are inordinate
pronouncements which did not remove the assailed dismissal from the realm of
illegality. Neither can these pronouncements preclude us from holding
otherwise.
Equally unmeritorious is petitioners assertion that the dismissal is justified on
the basis of loss of confidence. While loss of confidence, as correctly argued by
petitioners, is one of the valid grounds for termination of employment, the same,
however, cannot be used as a pretext to vindicate each and every instance of
unwarranted dismissal. To be a valid ground, it must shown that the employee
concerned is responsible for the misconduct or infraction and that the nature of
his participation therein rendered him absolutely unworthy of the trust and
confidence demanded by his position. In this case, petitioners were unsuccessful

in establishing their accusations of dishonesty and tampering of records with


intention of cheating. Indeed, even if petitioners allegations against de Jesus
were true, they just the same failed to prove that her position needs the
continued and unceasing trust of her employees functions. Surely, de Jesus who
occupies the position of a reviser/trimmer does not require the petitioners
perpetual and full confidence. In this regard, petitioners reliance on the cases
of Ocean Terminal Services, Inc. v. NLRC; Coca-Cola Bottlers Phil., Inc. v. NLRC;
and Piedad v. Lanao del Norte Electric Cooperative, which when perused involve
positions that require the employers full trust and confidence, is wholly
misplaced. Undoubtedly, the position of a reviser/trimmer could not be equated
with that of a canvasser, sales agent, or a bill collector. Besides, the involved
employees in the three aforementioned cases were clearly proven guilty of
infractions unlike private respondent in the case at bar. Thus, petitioners
dependence on these cited cases is inaccurate, to say the least. More, whether or
not de Jesus meets the days quota of work she, just the same, is paid the daily
minimum wage

(2) No, writ of execution is not necessary for order of reinstatement. Under
Article 223 of the Labor Code, as amended, an employer has two options in order
for him to comply with an order of reinstatement, which is immediately
executory, even pending appeal. Firstly, he can admit the dismissed employee
back to work under the same terms and conditions prevailing prior to his
dismissal or separation or to a substantially equivalent position if the former
position is already filled up. Secondly, he can reinstate the employee merely in
the payroll. Failing to exercise any of the above options, the employer can be
compelled under pain of contempt, to pay instead the salary of the
employee. This interpretation is more in consonance with the constitutional
protection to labor (Section 3, Art. XIII, 1987 Constitution). The right of a person
to his labor is deemed to be property within the meaning of the constitutional
guaranty that no one shall be deprived of life, liberty, and property without due
process of law. Therefore, he should be protected against any arbitrary and
unjust deprivation of his job (Bondoc vs. Peoples Bank and Trust Co., Inc., 103
SCRA 599 [1981]). The employee should not be left without any remedy in case
the employer unreasonably delays reinstatement.
Article 224 states that the need for a writ of execution applies only within five
(5) years from the date a decision, an order or awards becomes final and

3E Andaya Ching Espiritu Hefti Galvez Gammad Lainez Lui Madamba Nagera Narvasa
Ong Palangdao Rosales Sanchez Santos Satrain Tabo (2014-2015)

Labor Relations Case Digest - Atty. Joyrich Golangco

executory. It cannot relate to an award or order of reinstatement still to be


appealed or pending appeal which Article 223 contemplates. The provision of
Article 223 is clear that an award for reinstatement shall be immediately
executory even pending appeal and the posting of a bond by the employer
shall not stay the execution for reinstatement. The legislative content is quite
obvious, to make an award of reinstatement immediately enforceable, even
pending appeal. To require the application for and issuance of a writ of execution
as prerequisites for the execution of a reinstatement award would certainly
betray and run counter to the very object and intent of Article 223, the
immediate execution of a reinstatement order. The reason is simple. An
application for a writ of execution and its issuance could be delayed for
numerous reasons. A mere continuance or postponement of a scheduled
hearing, for instance, or an inaction on the part of the Labor Arbiter or the NLRC
could easily delay the issuance of the writ thereby setting at naught the strict
mandate and noble purpose envisioned by Article 223. Statutes, as a rule, are to
be construed in the light of the purpose to be achieved and the evil sought to be
remedied and where statues are fairly susceptible of two or more construction,
that construction should be adopted which will most tend to give effect to the
manifest intent of the law maker and promote the object for which the statute
was enacted, and a construction should be rejected which would tend to render
abortive other provisions of the statute and to defeat the object which the
legislator sought to attain by its enactment.
47: ALEJANDRO ROQUERO vs. PHILIPPINE AIRLINES, INC

FACTS: Alejandro Roquero, along with Rene Pabayo, were ground equipment
mechanics of respondent Philippine Airlines, Inc. (PAL for brevity). From the
evidence on record, it appears that Roquero and Pabayo were caught red-handed
possessing and using shabu in a raid conducted by PAL security officers and
NARCOM personnel. The two alleged that they did not voluntarily indulge in the
said act but were instigated by a certain Jojie Alipato who was introduced to
them by Joseph Ocul, Manager of the Airport Maintenance Division of PAL. When
they started the procedure of taking the drugs, armed men entered the room,
arrested Roquero and Pabayo and seized the drugs and the paraphernalia used.
They assailed their arrest and asserted that they were instigated by PAL to take

the drugs. They argued that Alipato was not really a trainee of PAL but was
placed in the premises to instigate the commission of the crime. They based their
argument on the fact that Alipato was not arrested. Moreover, Alipato has no
record of employment with PAL.
In a Memorandum dated July 14, 1994, Roquero and Pabayo were dismissed by
PAL. Thus, they filed a case for illegal dismissal.

Labor Arbiter: Dismissal of Roquero and Pabayo was upheld. Both parties are
found at fault, PAL for applying means to entice the complainants into
committing the infraction and the complainants for giving in to the temptation
and eventually indulging in the prohibited activity. Nonetheless, the Labor
Arbiter awarded separation pay and attorneys fees to the complainants.

While the case was on appeal with the NLRC the complainants were acquitted by
the Regional Trial Court (RTC) Branch 114, Pasay City, in the criminal case which
charged them with conspiracy for possession and use of a regulated drug in
violation of Section 16, Article III of Republic Act 6425, on the ground of
instigation.
NLRC: The NLRC found PAL guilty of instigation and ordered reinstatement to
their former positions but without backwages.

Complainants did not appeal from the decision but filed a motion for a writ of
execution of the order of reinstatement. The Labor Arbiter granted the motion
but PAL refused to execute the said order on the ground that they have filed a
Petition for Review before this Court. PALs petition was referred to the Court of
Appeals.
During the pendency of the case with the Court of Appeals, PAL and Pabayo filed
a Motion to Withdraw/Dismiss the case with respect to Pabayo, after they
voluntarily entered into a compromise agreement. The motion was granted in a
Resolution promulgated by the Former Thirteenth Division of the Court of
Appeals on January 29, 2002.
Court of Appeals: Reversed the decision of the NLRC and reinstated the decision
of the Labor Arbiter insofar as it upheld the dismissal of Roquero. However, it
denied the award of separation pay and attorneys fees to Roquero on the ground
that one who has been validly dismissed is not entitled to those benefits.

3E Andaya Ching Espiritu Hefti Galvez Gammad Lainez Lui Madamba Nagera Narvasa
Ong Palangdao Rosales Sanchez Santos Satrain Tabo (2014-2015)

ISSUES:

Labor Relations Case Digest - Atty. Joyrich Golangco

(1) Whether or not the instigated employee shall be solely responsible for an
action arising from the instigation perpetrated by the employer.

(2) whether or not the executory nature of the decision, more so the
reinstatement aspect of a labor tribunals order be halted by a petition having
been filed in higher courts without any restraining order or preliminary
injunction.
(3) Whether or not the employer who refused to reinstate the employee despite
a writ duly issued be held to pay the salary of the subject employee from the time
he was ordered reinstated up to the time of the reversal of the decision.
HELD:

(1) Instigation is only a defense against criminal liability. It cannot be used as a


shield against dismissal from employment especially when the position involves
the safety of human lives. Even if he was instigated to take drugs he has no right
to be reinstated to his position. He took the drugs fully knowing that he was on
duty and more so that it is prohibited by company rules.
Roquero is guilty of serious misconduct for possessing and using shabu. For
serious misconduct to warrant the dismissal of an employee, it (1) must be
serious; (2) must relate to the performance of the employees duty; and (3) must
show that the employee has become unit to continue working for the employer.
It is of public knowledge that drugs can damage the mental faculties of the user.
Roquero was tasked with the repair and maintenance of PALs airplanes. He
cannot discharge that duty if he is a drug user. His failure to do his job can mean
great loss of lives and properties.

There was procedural due process. PAL complied with the twin-notice
requirement before dismissing the petitioner. The twin-notice rule requires (1)
the notice which apprises the employee of the particular acts or omissions for
which his dismissal is being sought along with the opportunity for the employee
to air his side, and (2) the subsequent notice of the employers decision to
dismiss him.

(2) The order of reinstatement is immediately executory. The unjustified refusal


of the employer to reinstate a dismissed employee entitles him to payment of his
salaries effective from the time the employer failed to reinstate him despite the
issuance of a writ of execution. Unless there is a restraining order issued, it is
ministerial upon the Labor Arbiter to implement the order of reinstatement. In
the case at bar, no restraining order was granted. Thus, it was mandatory on PAL
to actually reinstate Roquero or reinstate him in the payroll.
(3)Having failed to do so, PAL must pay Roquero the salary he is entitled to, as if
he was reinstated, from the time of the decision of the NLRC until the finality of
the decision of this Court.

Technicalities have no room in labor cases where the Rules of Court are applied
only in a suppletory manner and only to effectuate the objectives of the Labor
Code and not to defeat them. Hence, even if the order of reinstatement of the
Labor Arbiter is reversed on appeal, it is obligatory on the part of the employer
to reinstate and pay the wages of the dismissed employee during the period of
appeal until reversal by the higher court. On the other hand, if the employee has
been reinstated during the appeal period and such reinstatement order is
reversed with finality, the employee is not required to reimburse whatever
salary he received for he is entitled to such, more so if he actually rendered
services during the period.
48: Air Philippines Corporation vs. Enrico Zamora

FACTS: Enrico Zamora (Zamora) was employed with Air Philippines Corporation
(APC) as Flight Deck Crew when applied for promotion to the position of airplane
captain and underwent the requisite training program. After completing training,
he inquired about his promotion but APC did not act on it. APC continued to give
him assignments as flight deck crew. Thus, Zamora filed a Complaint with the
Labor Arbiter. He argued that the act of APC of withholding his promotion
rendered his continued employment with it oppressive and unjust. He therefore
asked that APC be held liable for constructive dismissal.
APC denied that it dismissed complainant. It pointed out that, when the
complaint was filed on May 14, 1997, complainant was still employed with it. It
was only on May 22, 1997 that complainant stopped reporting for work, not

3E Andaya Ching Espiritu Hefti Galvez Gammad Lainez Lui Madamba Nagera Narvasa
Ong Palangdao Rosales Sanchez Santos Satrain Tabo (2014-2015)

Labor Relations Case Digest - Atty. Joyrich Golangco

because he was forced to resign, but because he had joined a rival airline,
GrandAir.

Labor Arbiter: Declared APC liable for constructive dismissal and ordered the
reinstate complainant to his position as B-737 Captain without loss of seniority
right immediately upon receipt thereof, Pay complainant his full backwages from
May 15, 1997 up to the promulgation of this decision, TWO MILLION PESOS (P2,
000,000.00) in the concept of moral damages, ONE MILLION PESOS
(P1,000,000.00) as exemplary damages and attorneys fees equivalent to TEN
PERCENT (10%) of the total award.

Zamora immediately filed a Motion for Execution of the order of reinstatement.


Meanwhile, APC filed with the NLRC an appeal assailing the finding of the Labor
Arbiter that it was liable for constructive dismissal.

NLRC: The NLRC granted the appeal in a Resolution dated and held that no
dismissal, constructive or otherwise, took place for it was Zamora himself who
voluntarily terminated his employment by not reporting for work and by joining
a competitor Grand Air. However, upon Motion for Reconsideration filed by
Zamora, the NLRC, in a Resolution dated December 17, 1999, modified its earlier
Resolution.
Court of Appeals: APC hereafter filed a Petition for Certiorari with the Court of
Appeals to have the December 17, 1999 Resolution of the NLRC partially
annulled and its October 11, 2000 Resolution set aside on the ground that these
were issued with grave abuse of discretion. Court of Appeals dismissed the
petition for failure of petitioner to attach copies of all pleadings (such complaint,
answer, position paper) and other material portions of the record as would
support the allegations therein. APC filed a Motion for Reconsideration and
attached the documents required by the Court but it was denied.
ISSUES:

(1) Whether or not the dismissal issued by the Court of Appeals was valid on the
ground that petitioner failed to attach required documents.
(2) Whether or not the employer is obligated to reinstate and pay the wages of
the dismissed employee during the period of appeal.

HELD:

(1) It is readily apparent in this case that the Court of Appeals was overzealous in
its enforcement of the rules. The pleadings and other documents it required of
petitioner were not at all relevant to the petition. It is noted that the only issue
raised by petitioner was whether the NLRC committed grave abuse of discretion
in granting respondent unpaid salaries while declaring him guilty of
abandonment of employment. Certainly, copies of the Resolutions of the NLRC
dated February 10, 1999, December 17, 1999 and October 11, 2000 would have
sufficed as basis for the Court of Appeals to resolve this issue. After all, it is in
these Resolutions that the NLRC purportedly made contrary findings. In sum, we
annul and set aside the January 11, 2000 and May 23, 2001 Resolutions of the
Court of Appeals.
(2) Rather than remand it to the Court of Appeals for resolution, the main issue
was resolved in an expedite matters. The Supreme Court ruled that NLRC did not
commit grave abuse of discretion in holding petitioner liable to respondent for
P198, 502.30. The premise of the award of unpaid salary to respondent is that
prior to the reversal by the NLRC of the decision of the Labor Arbiter, the order
of reinstatement embodied therein was already the subject of an alias writ of
execution even pending appeal. Although petitioner did not comply with this writ
of execution, its intransigence made it liable nonetheless to the salaries of
respondent pending appeal. There is logic in this reasoning of the NLRC.
In Roquero v. Philippine Airlines, Inc. it was that technicalities have no room in
labor cases where the Rules of Court are applied only in a suppletory manner and
only to effectuate the objectives of the Labor Code and not to defeat them.Hence,
even if the order of reinstatement of the Labor Arbiter is reversed on appeal, it is
obligatory on the part of the employer to reinstate and pay the wages of the
dismissed employee during the period of appeal until reversal by the higher
court. On the other hand, if the employee has been reinstated during the appeal
period and such reinstatement order is reversed with finality, the employee is
not required to reimburse whatever salary he received for he is entitled to such,
more so if he actually rendered services during the period.
There is a policy elevated in this ruling. In Aris (Phil.) Inc. v. National Labor
Relations Commission, it was held that with respect to decisions reinstating

3E Andaya Ching Espiritu Hefti Galvez Gammad Lainez Lui Madamba Nagera Narvasa
Ong Palangdao Rosales Sanchez Santos Satrain Tabo (2014-2015)

Labor Relations Case Digest - Atty. Joyrich Golangco

employees, the law itself has determined a sufficiently overwhelming reason for
its execution pending appeal. It is pursuant to the same power (police power),
the State may authorize an immediate implementation, pending appeal, of a
decision reinstating a dismissed or separated employee since that saving act is
designed to stop, although temporarily since the appeal may be decided in favor
of the appellant, a continuing threat or danger to the survival or even the life of
the dismissed or separated employee and his family.
49: LANSANGAN vs. AMKOR TECHNOLOGY

FACTS: An anonymous e-mail was sent to the General Manager of Amkor


Technology Philippines (respondent) detailing allegations of malfeasance on the
part of its supervisory employees Lunesa Lansangan and Rosita Cendaa
(petitioners) for "stealing company time." Respondent thus investigated the
matter, requiring petitioners to submit their written explanation. In handwritten
letters, petitioners admitted their wrongdoing. Respondent thereupon
terminated petitioners for "extremely serious offenses" as defined in its Code of
Discipline, prompting petitioners to file a complaint for illegal dismissal against
it.
LA: Labor Arbiter Amansec, dismissed petitioners complaint, he having found
them guilty of "Swiping another employees I.D. card or requesting another
employee to swipe ones I.D. card to gain personal advantage and/or in the
interest of cheating", an offense of dishonesty punishable as a serious form of
misconduct and fraud or breach of trust under Article 282 of the Labor Code. The
Arbiter, however, ordered the reinstatement of petitioners to their former
positions without backwages "as a measure of equitable and compassionate
relief" owing mainly to petitioners prior unblemished employment records,
show of remorse, harshness of the penalty and defective attendance monitoring
system of respondent.
NLRC: The LAs decision was modified and the portion regarding the
reinstatement of the petitioners was deleted.

CA: The CA affirmed the finding that petitioners were guilty of misconduct and
the like, and further ordered respondent to "pay petitioners their corresponding
backwages without qualification and deduction for the period covering October

20, 2004 (date of the Arbiters decision) up to June 30, 2005 (date of the NLRC
Decision)," citing Article 223 of the Labor Code and Roquero v. Philippine
Airlines.
ISSUES: Whether or not petitioners are entitled to receive backwages pursuant
to Article 223

HELD: Roquero, as well as Article 22318 of the Labor Code on which the appellate
court also relied, finds no application in the present case. Article 223 concerns
itself with an interim relief, granted to a dismissed or separated employee while
the case for illegal dismissal is pending appeal, as what happened in Roquero. It
does not apply where there is no finding of illegal dismissal, as in the present
case.
50. Genuino v NLRC

FACTS: Citibank is an American banking corporation duly licensed to do


business in the Philippines. Genuino was employed by Citibank as Treasury Sales
Division Head with the rank of Assistant Vice-president. On August 23, 1993,
Citibank sent Genuino a letter charging her with "knowledge and/or
involvement" in transactions "which were irregular or even fraudulent and was
informed she was under preventive suspension. She was further directed to
explain in writing why she should not be terminated. Petitioners counsel replied
and demanded a bill of particular regarding the charges against her. The bank
claimed that the petitioner and Mr. Dante Santos, using the facilities of their
family corporations (Torrance and Global) appear to have participated in the
diversion of bank clients' funds from Citibank to other companies and that they
made money in the process. Genuino did not appear in the administrative
investigation and thereafter Citibank informed Genuino of the result of their
investigation and was further informed that her employment was terminated by
Citibank on grounds of (1) serious misconduct, (2) willful breach of the trust
reposed upon her by the bank, and (3) commission of a crime against the bank.
Genuino filed before the Labor Arbiter a Complaint for illegal suspension and
illegal dismissal with damages

3E Andaya Ching Espiritu Hefti Galvez Gammad Lainez Lui Madamba Nagera Narvasa
Ong Palangdao Rosales Sanchez Santos Satrain Tabo (2014-2015)

Labor Relations Case Digest - Atty. Joyrich Golangco

LA: that the dismissal was without just cause and in violation of her right to due
process, and the bank is ordered to reinstate complainant immediately and pay
other benefits, with back wages. Both parties appealed to the NLRC.
NLRC: reversed the Labor Arbiter's decision and declared that the dismissal of
the complainant valid and legal but ORDERING bank to pay the salaries due to
the complainant from the date it reinstated complainant in the payroll. The
parties filed a petition for certiorari before the Court of Appeals.

CA only modified the amount of indemnity (P5000). Citibank contends that the
Labor Arbiters decision in upholding the right of Genuino to reinstatement is not
supported by evidence thus there can be no right to payroll reinstatement.
ISSUES: WON bank shall pay the salaries due the complainant from the date or
reinstatement up to the date of final decision.

HELD: The court held that the dismissal was for just cause but lacked due
process due to failure of the bank to meet the requirement of twin notices. The
first notice informing the employee of the charges should neither be pro forma
nor vague. It should set out clearly what the employee is being held liable for.
Since the notice of charges given to Genuino is inadequate for not specifying the
specific acts and surrounding circumstances of the transactions, the dismissal
could not be in accordance with due process. However the Court nevertheless
fined Genuino's dismissal justified. Loss of confidence is a valid ground for
dismissing an employee. It is sufficient if there is some basis for such loss of
confidence. In this case, Genuino was tasked to solicit investments and keep
them in Citibank.

Curiously, Genuino did not even dissuade the depositors from withdrawing their
monies from Citibank. The Court was thus compelled to conclude that Genuino
did not have her employer's interest. Furthermore, Court cancels the directive of
NLRC directing the bank to pay salaries due to the complainant from the date it
reinstated complainant in the payroll up to and until the date of this decision,
view of the Courts finding that the dismissal is valid. In any event, the decision of
the Labor Arbiter reinstating a dismissed or separated employee, insofar as the
reinstatement aspect is concerned, shall immediately be executory, even pending

appeal. If the decision of the labor arbiter is later reversed on appeal upon the
finding that the ground for dismissal is valid, then the employer has the right to
require the dismissed employee on payroll reinstatement to refund the salaries
s/he received while the case was pending appeal, or it can be deducted from the
accrued benefits that the dismissed employee was entitled to receive from
his/her employer under existing laws, collective bargaining agreement
provisions, and company practices. However, if the employee was reinstated to
work during the pendency of the appeal, then the employee is entitled to the
compensation received for actual services rendered without need of refund.
51. GARCIA VS PAL

FACTS: an administrative charge was filed by PAL against its employees herein
petitioners after they were allegedly caught in the act of sniffing shabu when a
team of company security personnel and law enforcers raided the PAL Technical
Centers Toolroom. After due notice, PAL dismissed petitioners prompting them
to file a complaint for illegal dismissal and damages.
Labor Arbiter resolved in their favor, ordering PAL to immediately comply with
the reinstatement aspect of the decision.
Prior to the promulgation of the Labor Arbiters decision, PAL was under
Permanent Rehabilitation Receiver due to severe financial loss. Respondent
appealed to the NLRC

NLRC: reversed said decision and dismissed petitioners complaint for lack of
merit. Subsequently the Labor Arbiter issued a Writ of Execution (Writ)
respecting the reinstatement aspect and issued a Notice of Garnishment (Notice).
Respondent filed an Urgent Petition for Injunction with the NLRC which affirmed
the validity of the Writ and the Notice issued by the Labor Arbiter but suspended
and referred the action to the Rehabilitation Receiver for appropriate action. The
case was elevated to appellate court

CA nullified the NLRC Resolutions on two grounds, essentially espousing that: (1)
a subsequent finding of a valid dismissal removes the basis for implementing the
reinstatement aspect of a labor arbiters decision (the first ground), and (2) the
impossibility to comply with the reinstatement order due to corporate

3E Andaya Ching Espiritu Hefti Galvez Gammad Lainez Lui Madamba Nagera Narvasa
Ong Palangdao Rosales Sanchez Santos Satrain Tabo (2014-2015)

Labor Relations Case Digest - Atty. Joyrich Golangco

rehabilitation provides a reasonable justification for the failure to exercise the


options under Article 223 of the Labor Code
ISSUES: whether or not petitioners may collect their wages during the period
between the Labor Arbiters order of reinstatement pending appeal and the
NLRC decision overturning that of the Labor Arbiter

HELD: Even if the order of reinstatement of the Labor Arbiter is reversed on


appeal, it is obligatory on the part of the employer to reinstate and pay the wages
of the dismissed employee during the period of appeal until reversal by the
higher court. On the other hand, if the employee has been reinstated during the
appeal period and such reinstatement order is reversed with finality, the
employee is not required to reimburse whatever salary he received for he is
entitled to such, more so if he actually rendered services during the period. the
Genuino ruling not only disregards the social justice principles behind the rule,
but also institutes a scheme unduly favorable to management. The provision of
Article 223 is clear that an award [by the Labor Arbiter] for reinstatement shall
be immediately executory even pending appeal and the posting of a bond by the
employer shall not stay the execution for reinstatement. To require the
application for and issuance of a writ of execution as prerequisites for the
execution of a reinstatement award would certainly betray and run counter to
the very object and intent of Article 223, i.e., the immediate execution of a
reinstatement order. It was ruled that the inaction of the Labor Arbiter who
failed to act upon the employees motion for the issuance of a writ of execution
may no longer adversely affect the cause of the dismissed employee in view of
the self-executory nature of the order of reinstatement. After the labor arbiters
decision is reversed by a higher tribunal, the employee may be barred from
collecting the accrued wages, if it is shown that the delay in enforcing the
reinstatement pending appeal was without fault on the part of the employer.
The test is twofold:
(1) there must be actual delay or the fact that the order of reinstatement pending
appeal was not executed prior to its reversal and
(2) the delay must not be due to the employers unjustified act or omission
It is settled that upon appointment by the SEC of a rehabilitation receiver, all
actions for claims before any court, tribunal or board against the corporation

shall ipso jure be suspended. Case law recognizes that unless there is a
restraining order, the implementation of the order of reinstatement is ministerial
and mandatory. This injunction or suspension of claims by legislative fiat
partakes of the nature of a restraining order that constitutes a legal justification
for respondents noncompliance with the reinstatement order. Such being the
case, respondents obligation to pay the salaries pending appeal, as the normal
effect of the non-exercise of the options, did not attach.
52. MT. CARMEL COLLEGEvs. JOCELYN RESUENA

FACTS: Petitioner Mt. Carmel College is a private educational institution and


administered by the Carmelite Fathers . Respondents, together with several
faculty members, nonacademic
personnel, and other students, participated in a protest action against petitioner.
Thereafter, petitioner's Director, Rev. Fr. Modesto E. Malandac, issued a
Memorandum to explain in writing why they should not be dismissed for loss of
trust and confidence for joining the protest action. After a hearing dismissal or
suspension of respondents were recommended. On May 15 1998 respondents
were terminated by petitioner. Respondents filed a complaint for illegal
dismissal.

Labor Arbiter Drilon issued a decision affirming validity of termination dated 25


May 1999. On 9 September 1999, Labor Arbiter Drilon issued to the parties a
Notice of Judgment/Decision of his decision. The notice indicated that the
reinstatement of the respondents. Petitioner appealed to the NLRC.
NLRC declared the termination of respondents to be illegal. It ordered the
reinstatement of respondents, with payment of backwages or payment of
separation pay as computed in the appealed decision. The case was elevated to
the Court of Appeals

CA: decided that [NLRC] correctly held that even if participation in the protest
picket is rather improper or disappointing to the School Administrator,
termination it is definitely too harsh and having been illegally dismissed,
respondents are entitled to back wages from the time of their termination until
reinstatement, and if reinstatement is no longer possible, the grant of separation

3E Andaya Ching Espiritu Hefti Galvez Gammad Lainez Lui Madamba Nagera Narvasa
Ong Palangdao Rosales Sanchez Santos Satrain Tabo (2014-2015)

Labor Relations Case Digest - Atty. Joyrich Golangco

pay equivalent to one (1) month for every year of service. However, in this case
since the Labor Arbiter did not order reinstatement, the NLRC correctly excluded
the period of the appeal in the computation of back wages due to respondents.
Hence, appeal to Supreme
Court.

Petitioner claims that Labor Arbiter exceeded his jurisdiction in issuing the writ
of execution despite the fact that his decision did not order reinstatement. The
petitioner further avers that the Court of Appeals erred in upholding the Labor
Arbiter and the NLRC that the award of backwages goes beyond the period 15
May 1998 to 25 May 1999 on the supposition that reinstatement is selfexecutory
and does not need a writ of execution for its enforcement. Petitioner vehemently
raises the argument that the award of backwages subject to execution is limited
to the period prior to the appeal and does not include the period during the
pendency of the appeal
ISSUES: (1) whether reinstatement in the instant case is self executory and does
not need a writ of execution for its enforcement
(2) whether the continuing award of backwages is proper.

HELD: 1. An order for reinstatement must be specifically declared and cannot be


presumed like back wages, it is a separate and distinct relief given to an illegally
dismissed employee. Art. 223 of the Labor Code provide that reinstatement is
immediately executory even pending appeal only when the Labor Arbiter himself
ordered the reinstatement. In this case, the original Decision of Labor Arbiter
Drilon did not order reinstatement. Reinstatement in this case was actually
ordered by the NLRC, affirmed by the Court of Appeals. Thus, Art. 223 find no
application in the instant case. Considering that the order for reinstatement was
first decided upon appeal to the NLRC and affirmed with finality by the Court of
Appeals.
2. Petitioner seems to have missed that the NLRC Decision also directed it to
reinstate respondents, or in lieu thereof, pay separation pay. This, petitioner
failed to do. Petitioner did not exercise the option of either reinstatement or
paying the separation pay of respondents. Back wages are to be computed from
the time of illegal dismissal until reinstatement or upon petitioner's payment of

separation pay to respondents if reinstatement is no longer possible. The


backwages due respondents must be computed from the time they were unjustly
dismissed until their actual reinstatement to their former position or upon
petitioner's payment of separation pay to them if reinstatement is no longer
feasible.
53. Buenviaje vs CA

FACTS: Petitioners were former employees of Cottonway Marketing Corp.


(Cottonway), hired as promo girls for their garment products. In October, 1994,
after their services were terminated as the company was allegedly suffering
business losses, petitioners filed with the National Labor Relations Commission
(NLRC) a complaint for illegal dismissal, underpayment of salary, and nonpayment of premium pay for rest day, service incentive leave pay and thirteenth
month pay against Cottonway Marketing Corp. and Network Fashion Inc./JCT
International Trading.[1]

On December 19, 1995, Labor Arbiter Romulus S. Protasio issued a Decision


finding petitioners' retrenchment valid and ordering Cottonway to pay
petitioners' separation pay and their proportionate thirteenth month pay. NLRC
reversed the decision and ordered the reinstatement of petitioners with payment
of full wages and proportionate 13th month pay. On August 30, 1996 Cottonway
filed a manifestation with NLRC claiming that the petitioners have lost their
employment for failing to comply with the return to work order. On November 6,
1997, petitioners filed with the NLRC a motion for execution of its Decision on
the ground that it had become final and executory. Cottonway filed another
manifestation on march 4, 1997 informing NLRC that petitioners have found
work elsewhere.
LA - Nonetheless, on April 8, 1998, Labor Arbiter Romulus S. Protasio issued an
Order declaring that the award of backwages and proportionate thirteenth
month pay to petitioners should be limited from the time of their illegal dismissal
up to the time they received the notice of termination sent by the company upon
their refusal to report for work despite the order of reinstatement. He cited the
fact that petitioners failed to report to their posts without justifiable reason
despite respondent's order requiring them to return to work immediately. The

3E Andaya Ching Espiritu Hefti Galvez Gammad Lainez Lui Madamba Nagera Narvasa
Ong Palangdao Rosales Sanchez Santos Satrain Tabo (2014-2015)

Labor Relations Case Digest - Atty. Joyrich Golangco

Labor Arbiter ordered the Research and Investigation Unit to recompute the
monetary award in accordance with its ruling.

The April 8 Order of the Labor Arbiter, however, was set aside by the
Commission in its Resolution dated September 21, 1998. The Commission ruled
that its Decision dated March 26, 1996 has become final and executory and it is
the ministerial duty of the Labor Arbiter to issue the corresponding writ of
execution to effect full and unqualified implementation of said decision. The
Commission thus ordered that the records of the case be remanded to the Labor
Arbiter for execution. Cottonway moved for reconsideration of said resolution,
to no avail.

CA granted Cottonways petition for certiorari ruling that petitioners'


reinstatement was no longer possible as they deliberately refused to return to
work despite the notice given by Cottonway. The Court of Appeals thus held that
the amount of backwages due them should be computed only up to the time they
received their notice of termination.
ISSUES: WON the Honorable Court of Appeals gravely abused its discretion
amounting to lack of and/or in excess of jurisdiction in reinstating the irregular
and illegal April 8, 1998 Order of Honorable Arbiter Romulus Protasio

HELD: Petitioners' alleged failure to return to work cannot be made the basis for
their termination. Such failure does not amount to abandonment which would
justify the severance of their employment. To warrant a valid dismissal on the
ground of abandonment, the employer must prove the concurrence of two
elements: (1) the failure to report for work or absence without valid or
justifiable reason, and (2) a clear intention to sever the employer-employee
relationshipIf Cottonway were really sincere in its offer to immediately reinstate
petitioners to their former positions, it should have given them reasonable time
to wind up their current preoccupation or at least to explain why they could not
return to work at Cottonway at once. Cottonway did not do either. Instead, it
gave them only five days to report to their posts and when the petitioners failed
to do so, it lost no time in serving them their individual notices of
termination. We are, therefore, not impressed with the claim of respondent
company that petitioners have been validly dismissed on August 1, 1996 and
hence their backwages should only be computed up to that time. We hold that

petitioners are entitled to receive full backwages computed from the time their
compensation was actually withheld until their actual reinstatement, or if
reinstatement is no longer possible, until the finality of the decision, in
accordance with the Decision of the NLRC dated March 26, 1996 which has
attained finality.
54. Pfizer vs Velasco

FACTS: Respondent was on leave when she received a show- cause notice from
petitioner, mentioning about an investigation on her possible violations of
company work rules regarding "unauthorized deals and/or discounts in money
or samples and unauthorized withdrawal and/or pull-out of stocks" and
instructing her to submit her explanation on the matter within 48 hours from
receipt of the same, the notice also advised her that she was being placed under
"preventive suspension" for 30 days and ordered the surrender of
accountabilities. A second show-cause notice was sent to her informing her of
the developments in the investigation. That same day, Velasco filed a complaint
for illegal suspension with money claims before the Regional Arbitration Branch.
The following day, 17 July 2003, PFIZER sent her a letter inviting her to a
disciplinary hearing to be held on 22 July 2003. Velasco received it under protest
and informed PFIZER via the receiving copy of the said letter that she had lodged
a complaint against the latter and that the issues that may be raised in the July 22
hearing "can be tackled during the hearing of her case" or at the preliminary
conference set for 5 and 8 of August 2003. She likewise opted to withhold
answering the Second Show-cause Notice. On 25 July 2003, Velasco received a
"Third Show-cause Notice," together with copies of the affidavits of two Branch
Managers of Mercury Drug, asking her for her comment within 48 hours. Finally,
on 29 July 2003, PFIZER informed Velasco of its "Management Decision"
terminating her employment.
LA rendered Velascos dismissal illegal and ordered her reinstatement as well as
payment of backwages, moral and exemplary damages and attorneys fees. NLRC
modified the ruling and deleted the award of damages.
CA annulled the previous rulings and declared respondents dismissal with just
cause and dismissed her complaint for illegal dismissal. CA later modified its
ruling by ordering the payment of respondents back wages from the date of the

3E Andaya Ching Espiritu Hefti Galvez Gammad Lainez Lui Madamba Nagera Narvasa
Ong Palangdao Rosales Sanchez Santos Satrain Tabo (2014-2015)

Labor Relations Case Digest - Atty. Joyrich Golangco

Labor Arbiters Decision dated December 5, 2003 up to the Court of Appeals


Decision dated November 23, 2005.

ISSUES: Whether or not the Court of Appeals committed a serious but reversible
error when it ordered Pfizer to pay Velasco wages from the date of the Labor
Arbiters decision ordering her reinstatement until November 23, 2005, when
the Court of Appeals rendered its decision declaring Velascos dismissal valid.

HELD: In the case at bar, PFIZER did not immediately admit respondent back to
work which, according to the law, should have been done as soon as an order or
award of reinstatement is handed down by the Labor Arbiter without need for
the issuance of a writ of execution. Thus, respondent was entitled to the wages
paid to her under the aforementioned writ of execution. At most, PFIZERs
payment of the same can only be deemed partial compliance/execution of the
Court of Appeals Resolution dated October 23, 2006 and would not bar
respondent from being paid her wages from May 6, 2005 to November 23,
2005.It would also seem that PFIZER waited for the resolution of its appeal to the
NLRC and, only after it was ordered by the Labor Arbiter to pay the amount
of P1,963,855.00 representing respondents full backwages from December 5,
2003 up to May 5, 2005, did PFIZER decide to require respondent to report back
to work via the Letter dated June 27, 2005.PFIZER makes much of respondents
non-compliance with its return- to-work directive by downplaying the reasons
forwarded by respondent as less than sufficient to justify her purported refusal
to be reinstated. In PFIZERs view, the return-to-work order it sent to respondent
was adequate to satisfy the jurisprudential requisites concerning the
reinstatement of an illegally dismissed employee.
In the case at bar, respondents decision to claim separation pay over
reinstatement had no legal effect, not only because there was no genuine
compliance by the employer to the reinstatement order but also because the
employer chose not to act on said claim. If it was PFIZERs position that
respondents act amounted to a "resignation" it should have informed
respondent that it was accepting her resignation and that in view thereof she
was not entitled to separation pay. PFIZER did not respond to respondents
demand at all. As it was, PFIZERs failure to effect reinstatement and accept
respondents offer to terminate her employment relationship with the company
meant that, prior to the Court of Appeals reversal in the November 23, 2005

Decision, PFIZERs liability for backwages continued to accrue for the period not
covered by the writ of execution dated May 24, 2005 until November 23, 2005.
. It cannot be denied that, under our statutory and jurisprudential framework,
respondent is entitled to payment of her wages for the period after December 5,
2003 until the Court of Appeals Decision dated November 23, 2005,
notwithstanding the finding therein that her dismissal was legal and for just
cause. Thus, the payment of such wages cannot be deemed as unjust enrichment
on respondents part.
55. Wenphil Corp. vs. Abing

FACTS: This case stemmed from a complaint for illegal dismissal filed by the
respondents, Almer Abing and Anabelle Tuazon, against Wenphil.

On December 8, 2000, LA Geobel A. Bartolabac ruled that the


respondents had been illegally dismissed by Wenphil. According to the LA, the
allegation of serious misconduct against the respondents had no factual and legal
basis. LA Bartolabac ordered Wenphil to immediately reinstate the
respondents to their respective positions or to equivalent ones, whether
actuall or in the payroll and to pay the respondents their backwages.
Wenphil appealed to the NLRC on April 16, 2001. In the meantime, the
respondents moved for the immediate execution of the LAs December 8, 2000
decision.

On October 29, 2001, Wenphil and the respondents entered into a


compromise agreement before LA Bartolabac. They agreed to the
respondents payroll reinstatement while Wenphils appeal with the NLRC
was ongoing. Wenphil also agreed to pay the accumulated salaries of the
respondents for the payroll until such time that the questioned decision of
LA Bartolabac is either modified, amended or reversed by the Honorable
National Labor Relations Commission.
On January 30, 2002, the NLRC issued a resolution affirming LA
Bartolabacs decision with modifications. Instead of ordering the respondents

3E Andaya Ching Espiritu Hefti Galvez Gammad Lainez Lui Madamba Nagera Narvasa
Ong Palangdao Rosales Sanchez Santos Satrain Tabo (2014-2015)

Labor Relations Case Digest - Atty. Joyrich Golangco

reinstatement, the NLRC directed Wenphil to pay the respondents their


respective separation pay.

Wenphil filed a petition for certiorari with the CA to question the NLRCs
resolutions. On August 27, 2003, the CA rendered its decision reversing the
NLRCs finding that the respondents had been illegally dismissed. On appeal
to the Supreme Court (docketed as G.R. No. 162447 and dated December 27,
2006), the SC denied the respondents petition for review and affirmed the CAs
resolution. The respondents did not file any motion for reconsideration to
question the SCs decision; thus, the decision became final and executory on
February 15, 2007.
Sometime after the SCs decision in G.R. No. 162447 became final and
executory, the respondents filed with LA Bartolabac a motion for computation
and issuance of writ of execution. The respondents asserted that although the
CAs ruling on the absence of illegal dismissal was adverse to them, under the
law and settled jurisprudence, they were still entitled to backwages from
the time of their dismissal until the NLRCs decision finding them to be
illegally dismissed was reversed with finality.
LA Bartolabac granted the respondents motion and directed Wenphil to pay
each complainant their salaries on reinstatement.

Both parties appealed to the NLRC. Wenphil argued that the


respondents were no longer entitled to payment of backwages in view of the
compromise agreement they executed on October 29, 2001. Since the NLRC
modified the LAs ruling by ordering the payment of separation pay in lieu of
reinstatement, then the respondents, were entitled to backwages only up to the
finality of the NLRC decision.
The respondents questioned in their appeal the determined period for
the computation of their backwages; they posited that the period for payment
should end, not on November 8, 2002, but on February 14, 2007, the date SCs
decision became final and executor.

NLRC denied the parties respective appeals in its decision dated March 26,
201034 and affirmed in toto the LAs order.

CA reversed the NLRC rulings and prescribed a different computation period.

The CA cited the case of Pfizer v. Velasco37 where this Court ruled that
even if the order of reinstatement of the Labor Arbiter is reversed on appeal, it is
obligatory on the part of the employer to reinstate and pay the dismissed
employees wages during the period of appeal until reversal by the higher
court.38 The CA construed this "higher court" to be the CA, not the SC. The CA
reasoned out that it was a "higher court" than the NLRC when it reversed the
NLRCs rulings; thus, the period for computation should end when it
promulgated its decision reversing that of the NLRC, and not on the date
when the SC affirmed its decision.
ISSUES: Wenphil maintained that the respondents were no longer entitled to
payment of backwages in view of the modification of the LAs ruling by the NLRC
pursuant with their October 29, 2001 compromise agreement.
Wenphil claimed that the reliefs of reinstatement and backwages are
only available to illegally dismissed employees. A ruling that the respondents
were still entitled to reinstatement pay notwithstanding the validity of their
dismissal, would amount to the courts tolerance of an unjust and equitable
situation.

HELD: We resolve to DENY the petition. An order of reinstatement is


immediately executory even pending appeal. (Art. 223 of the Labor Code).The
employer has the obligation to reinstate and pay the wages of the dismissed
employee during the period of appeal until reversal by the higher court.
The Court discussed reason behind this legal policy in Aris v. NLRC,where it
explained:

In authorizing execution pending appeal of the reinstatement aspect of a


decision of the Labor Arbiter reinstating a dismissed or separated employee, the
law itself has laid down a compassionate policy which, once more, vivifies and

3E Andaya Ching Espiritu Hefti Galvez Gammad Lainez Lui Madamba Nagera Narvasa
Ong Palangdao Rosales Sanchez Santos Satrain Tabo (2014-2015)

Labor Relations Case Digest - Atty. Joyrich Golangco

enhances the provisions of the 1987 Constitution on labor and the working-man.
These provisions are the quintessence of the aspirations of the workingman for
recognition of his role in the social and economic life of the nation, for the
protection of his rights, and the promotion of his welfare
Since the decision is immediately executory, it is the duty of the
employer to comply with the order of reinstatement, which can be done
either actually or through payroll reinstatement. As provided under Article
223 of the Labor Code, this immediately executory nature of an order of
reinstatement is not affected by the existence of an ongoing appeal. The
employer has the duty to reinstate the employee in the interim period until a
reversal is decreed by a higher court or tribunal.

In the case of payroll reinstatement, even if the employers appeal turns


the tide in its favor, the reinstated employee has no duty to return or reimburse
the salary he received during the period that the lower court or tribunals
governing decision was for the employees illegal dismissal.

The reinstatement salaries due to the respondents were, by their nature,


payment of unworked backwages. These were salaries due to the respondents
because they had been prevented from working despite the LA and the NLRC
findings that they had been illegally dismissed.

Reinstatement and backwages are two separate reliefs available to


an illegally dismissed employee. The normal consequences of a finding that an
employee has been illegally dismissed are: first, that the employee becomes
entitled to reinstatement to his former position without loss of seniority rights;
and second, the payment of backwages covers the period running from his illegal
dismissal up to his actual reinstatement. These two reliefs are not inconsistent
with one another and the labor arbiter can award both simultaneously.
The relief of separation pay may be granted in lieu of reinstatement but
it cannot be a substitute for the payment of backwages. In instances where
reinstatement is no longer feasible because of strained relations between the
employee and the employer, separation pay should be granted.

Had Wenphil really wanted to put a stop to the running of the period for
the payment of the respondents backwages, then it should have immediately
complied with the NLRCs order to award the employees their separation pay in
lieu of reinstatement. This action would have immediately severed the employeremployee relationship. However, the records are bereft of any evidence that
Wenphil actually paid the respondents separation pay. Thus, the employeremployee relationship between Wenphil and the respondents never ceased and
the employment status remained pending and uncertain until the CA actually
rendered its decision that the respondents had not been illegally dismissed. In
the context of the parties agreement, it was only at this point that the payment of
backwages should have stopped.
The period for computing the backwages due to the respondents during
the period of appeal should end on the date that a higher court reversed the
labor arbitration ruling of illegal dismissal. In this case, the higher court which
first reversed the NLRCs ruling was not the SC but rather the CA. In this light, the
CA was correct when it found that that the period of computation should end on
August 27, 2003. The date when the SCs decision became final and executory
need not matter as the rule in Roquero, Garcia and Pfizer merely referred to the
date of reversal, not the date of the ultimate finality of such reversal.
56: Sy, et. al., vs Fairland Knitcraft Co.

FACTS: Fairland is a domestic corporation engaged in garments business, while


Susan de Leon (Susan) is the owner/proprietors of Weesan Garments (Weesan).
The complaining workers are sewers, trimmers, helpers, a guard and a secretary
who were hired by Weesan.
On December 23, 2002 and on January 2003, workers filed with the Arbitration
Branch of the NLRC a Complaint for underpayment and/or non-payment of
wages, and other monetary benefits against Susan/Weesan.

On February 5, 2003, Weesan filed before the DOLE-NCR a report on its


temporary closure for a period of not less than six months. The workers filed an
Amended Complaint, as they are not allowed to work anymore, to include the
charge of illegal dismissal and impleaded Fairland and its manager, Debbie
Manduabas (Debbie), as additional respondents.

3E Andaya Ching Espiritu Hefti Galvez Gammad Lainez Lui Madamba Nagera Narvasa
Ong Palangdao Rosales Sanchez Santos Satrain Tabo (2014-2015)

Labor Relations Case Digest - Atty. Joyrich Golangco

A Notice of Hearing was thereafter sent to Weesan requesting it to appear before


Labor Arbiter Reyes on April 3, 2003, at 10:00 a.m. On said date and time, Atty.
Geronimo appeared as counsel for Weesan and requested for an extension
of time to file his clients position paper.
On May 16, 2003, Atty. Geronimo filed two separate position papers one
for Fairland and another for Susan/Weesan. Atty. Geronimo then filed a
Consolidated Reply verified both by Susan and Debbie.
LA: On November 26, 2003, Labor Arbiter Reyes rendered his Decision,
dismissing the complaint for lack of merit; and ordering the respondents to pay
each complainant P5,000.00 by way of financial assistance.

NLRC: The workers filed their appeal which was granted by the NLRC, ruling that
the dismissal of complainants is declared illegal. Respondents are ordered to
reinstate complainants with full backwages with legal interests.
Atty. Geronimo filed a Motion for Reconsideration. However, Fairland filed
another Motion for Reconsideration through Atty. Tecson assailing the
jurisdiction of the Labor Arbiter and the NLRC over it, claiming that it was never
summoned to appear, attend or participate in all the proceedings conducted
therein. It also denied that it engaged the services of Atty. Geronimo.
The NLRC however, denied both motions for lack of merit.

In-affirm ng CA ung NLRC na may illegal dismissal na naganap. Sabi ng CA,


solidarily liable si Weeson with Fairland as labor-only contractor and principal
respectively. Nag-file ang Fairland ng MoR claiming na independent contractor
ang Weesan at nag-file din sila ng Motion for Voluntary Inhibition of two CA
Assoc. Justices. Na-grant ung Inhibition, hence, the case was transferred to CAs
Special Ninth Division for resolution ng MoR ng Fairland.

On May 9, 2008, the CAs Special Ninth Division reversed the First Divisions
ruling. It held that the labor tribunals did not acquire jurisdiction over the person
of Fairland, and even assuming they did, Fairland is not liable to the workers
since Weesan is not a mere labor-only contractor but a bona fide independent

contractor. The Special Ninth Division thus annulled and set aside the assailed
NLRC Decision and Resolution insofar as Fairland is concerned and excluded the
latter therefrom.

Syempre, nag-appeal mga workers. Sabi nila, among others, may jurisdiction over
the person of Fairland.
The Workers Arguments

The workers contend that the Labor Arbiter and the NLRC properly acquired
jurisdiction over the person of Fairland because the latter voluntarily appeared
and actively participated in the proceedings below when Atty. Geronimo
submitted on its behalf a Position Paper verified by its manager, Debbie. As
manager, Debbie knew of all the material and significant events which transpired
in Fairland since she had constant contact with the people in the day-to-day
operations of the company.
Fairlands Arguments

In gist, Fairland contests the labor tribunals acquisition of jurisdiction over its
person either through service of summons or voluntary appearance. It denies
that it engaged the services of Atty. Geronimo and asserts that it has its own legal
counsel, Atty. Tecson, who would have represented it had it known of the
pendency of the complaints against Fairland. In the absence, therefore, of a valid
service of summons or voluntary appearance, the proceedings conducted and the
judgment rendered by the labor tribunals are null and void as against it. Hence,
Fairland cannot be held solidarily liable with Susan/Weesan.
Supreme Court: Grant ang Petition ng Workers

"It is basic that the Labor Arbiter cannot acquire jurisdiction over the person of
the respondent without the latter being served with summons. However, "if
there is no valid service of summons, the court can still acquire jurisdiction over
the person of the defendant by virtue of the latters voluntary appearance."
Fairland argued before the CA that it did not engage Atty. Geronimo as its
counsel.

3E Andaya Ching Espiritu Hefti Galvez Gammad Lainez Lui Madamba Nagera Narvasa
Ong Palangdao Rosales Sanchez Santos Satrain Tabo (2014-2015)

Labor Relations Case Digest - Atty. Joyrich Golangco

The fact that Atty. Geronimo entered his appearance for Fairland and Debbie and
that he actively defended them before the Labor Arbiter raised the presumption
that he is authorized to appear for them. As held in Santos, it is unlikely that Atty.
Geronimo would have been so irresponsible as to represent Fairland and Debbie
if he were not in fact authorized. As an officer of the Court, Atty. Geronimo is
presumed to have acted with due propriety. Moreover, "[i]t strains credulity that
a counsel who has no personal interest in the case would fight for and defend a
case with persistence and vigor if he has not been authorized or employed by the
party concerned.
The presumption of authority of counsel to appear on behalf of a client is found
both in the Rules of Court and in the New Rules of Procedure of the NLRC.

Between the two provisions providing for such authority of counsel to appear,
the Labor Arbiter is primarily bound by the latter one, the NLRC Rules of
Procedure being specifically applicable to labor cases. As Atty. Geronimo
consistently indicated his PTR and IBP numbers in the pleadings he filed, there is
no reason for the Labor Arbiter not to extend to Atty. Geronimo the presumption
that he is authorized to represent Fairland.
The CA likewise emphasized that in labor cases, both the party and his counsel
must be duly served their separate copies of the order, decision or resolution
unlike in ordinary proceedings where notice to counsel is deemed notice to the
party, quoting Article 224 of the Labor Code.
The Court disagrees.

Article 224 of the Labor Code does not govern the procedure for filing a petition
for certiorari with the Court of Appeals from the decision of the NLRC but rather,
it refers to the execution of final decisions, orders or awards and requires the
sheriff or a duly deputized officer to furnish both the parties and their counsel
with copies of the decision or award for that purpose. There is no reference,
express or implied, to the period to appeal or to file a petition for certiorari as
indeed the caption is execution of decisions, orders or awards. Taken in proper
context, Article 224 contemplates the furnishing of copies of final decisions,
orders or awards and could not have been intended to refer to the period for
computing the period for appeal to the Court of Appeals from a non-final
judgment or order. The period or manner of appeal from the NLRC to the Court

of Appeals is governed by Rule 65 pursuant to the ruling of the Court in the case
of St. Martin Funeral Homes vs. NLRC. Section 4 of Rule 65, as amended, states
that the petition may be filed not later than sixty (60) days from notice of the
judgment, or resolution sought to be assailed.

Corollary, Section 4, Rule III of the New Rules of Procedure of the NLRC expressly
mandates that (F)or the purposes of computing the period of appeal, the same
shall be counted from receipt of such decisions, awards or orders by the counsel
of record. Although this rule explicitly contemplates an appeal before the Labor
Arbiter and the NLRC, we do not see any cogent reason why the same rule should
not apply to petitions for certiorari filed with the Court of Appeals from decisions
of the NLRC. This procedure is in line with the established rule that notice to
counsel is notice to party and when a party is represented by counsel,
notices should be made upon the counsel of record at his given address to
which notices of all kinds emanating from the court should be sent. It is to
be noted also that Section 7 of the NLRC Rules of Procedure provides that
(A)ttorneys and other representatives of parties shall have authority to
bind their clients in all matters of procedure.
To stress, Article 224 contemplates the furnishing of copies of final decisions,
orders or awards both to the parties and their counsel in connection with the
execution of such final decisions, orders or awards. However, for the purpose of
computing the period for filing an appeal from the NLRC to the CA, same shall be
counted from receipt of the decision, order or award by the counsel of record
pursuant to the established rule that notice to counsel is notice to party. And
since the period for filing of an appeal is reckoned from the counsels receipt of
the decision, order or award, it necessarily follows that the reckoning period for
their finality is likewise the counsels date of receipt thereof, if a party is
represented by counsel. Hence, the date of receipt referred to in Sec. 14, Rule VII
of the then in force New Rules of Procedure of the NLRC which provides that
decisions, resolutions or orders of the NLRC shall become executory after 10
calendar days from receipt of the same, refers to the date of receipt by counsel.
Thus contrary to the CAs conclusion, the said NLRC Decision became final, as to
Fairland, 10 calendar days after Atty. Tecsons receipt thereof. In sum, we hold
that the Labor Arbiter had validly acquired jurisdiction over Fairland and its
manager, Debbie, through the appearance of Atty. Geronimo as their counsel and
likewise, through the latters filing of pleadings on their behalf.

3E Andaya Ching Espiritu Hefti Galvez Gammad Lainez Lui Madamba Nagera Narvasa
Ong Palangdao Rosales Sanchez Santos Satrain Tabo (2014-2015)

Labor Relations Case Digest - Atty. Joyrich Golangco

57: Yupangco Cotton Mills vs CA

FACTS: Petitioner claims that a sheriff of the National Labor Relations


Commission "erroneously and unlawfully levied" upon certain properties which
it claims as its own.
"1. It filed a notice of third-party claim with the Labor Arbiter on May 4, 1995.

"2. It filed an Affidavit of Adverse Claim with the NLRC on July 4, 1995, which was
dismissed on August 30, 1995, by the labor Arbiter.
"3. It filed a petition for certiorari and prohibition with the Regional Trial Court
of Manilaon October 6, 1995. The Regional Trial Court dismissed the case on
October 11, 1995 for lack of merit.

"4. It appealed to the NLRC the order of the Labor Arbiter dated August 13, 1995
which dismissed the appeal for lack of merit on December 8, 1995.
"5. It filed an original petition for mandatory injunction with the NLRC on
November 16, 1995. This case is still pending with that Commission.

"6. It filed a complaint in the Regional Trial Court in Manila. The dismissal of
this case by public respondent triggered the filing of the instant petition.
In all of the foregoing actions, petitioner raised a common issue, which is that it
is the owner of the properties located in the compound and buildings of Artex
Development Corporation, which were erroneously levied upon by the sheriff
of the NLRC as a consequence of the decision rendered by the said
Commission in a labor case.
CA dismissed the petition on the ground of Forum Shopping. The Court of
Appeals sustained the trial court's ruling that the remedies granted under
Section 17, Rule 39 of the Rules of Court are not available to the petitioner
because the Manual of Instructions for Sheriffs of the NLRC does not include the
remedy of an independent action by the owner to establish his right to his
property.

In a MoR, petitioner argued that the filing of a complaint for accion


reinvindicatoria with the Regional Trial Court was proper because it is a remedy
specifically granted to an owner (whose properties were subjected to a writ of
execution to enforce a decision rendered in a labor dispute in which it was not a
party) by Section 17 (now 16), Rule 39, Revised Rules of Court.

Petitioner argued that the reliefs sought and the issues involved in the complaint
for recovery of property and damages filed with the Regional Trial Court of
Manila, were entirely distinct and separate from the reliefs sought and the issues
involved in the proceedings before the Labor Arbiter and the NLRC.
CA denied the MoR.

ISSUES: whether the Court of Appeals erred in dismissing the petitioner's accion
reinvindicatoria on the ground of lack of jurisdiction of the trial court.

HELD: A third party whose property has been levied upon by a sheriff to enforce
a decision against a judgment debtor is afforded with several alternative
remedies to protect its interests. The third party may avail himself of alternative
remedies cumulatively, and one will not preclude the third party from availing
himself of the other alternative remedies in the event he failed in the remedy
first availed of.
Thus, a third party may avail himself of the following alternative remedies:

a) File a third party claim with the sheriff of the Labor Arbiter, and
b) If the third party claim is denied, the third party may appeal the
denial to the NLRC.

Even if a third party claim was denied, a third party may still file a proper action
with a competent court to recover ownership of the property illegally seized by
the sheriff. This finds support in Section 17 (now 16), Rule 39, Revised Rules of
Court.

3E Andaya Ching Espiritu Hefti Galvez Gammad Lainez Lui Madamba Nagera Narvasa
Ong Palangdao Rosales Sanchez Santos Satrain Tabo (2014-2015)

Labor Relations Case Digest - Atty. Joyrich Golangco

As held in the case of Ong v. Tating, et. al., construing the aforecited rule, a third
person whose property was seized by a sheriff to answer for the obligation of a
judgment debtor may invoke the supervisory power of the court which
authorized such execution. Upon due application by the third person and after
summary hearing, the court may command that the property be released from
the mistaken levy and restored to the rightful owner or possession. What said
court do in these instances, however, is limited to a determination of whether the
sheriff has acted rightful or wrongly in the performance of his duties in the
execution of judgment, more specifically, if he has indeed take hold of property
not belonging to the judgment debtor. The court does not and cannot pass
upon the question of title to the property, with any character of finality.
The well-settled doctrine is that a 'proper levy' is indispensable to a valid sale on
execution. A sale unless preceded by a valid levy is void. Therefore, since there
was no sufficient levy on the execution in question, the private respondent did
not take any title to the properties sold.
"A person other than the judgment debtor who claims ownership or right over
the levied properties is not precluded, however, from taking other legal
remedies."

Jurisprudence is likewise replete with rulings that since the third-party claimant
is not one of the parties to the action, he could not, strictly speaking, appeal from
the order denying his claim, but should file a separate reinvindicatory action
against the execution creditor or the purchaser of the property after the sale at
public auction, or a complaint for damages against the bond filed by the
judgment creditor in favor of the sheriff.

A separate civil action for recovery of ownership of the property would not
constitute interference with the powers or processes of the Arbiter and the NLRC
which rendered the judgment to enforce and execute upon the levied properties.
The property levied upon being that of a stranger is not subject to levy. Thus, a
separate action for recovery, upon a claim and prima-facie showing of ownership
by the petitioner, cannot be considered as interference.

The Court renders judgment ANNULLING the sale on execution of the subject
property conducted by NLRC Sheriff Anam Timbayan in favor of respondent
SAMAR-ANGLO and the subsequent sale of the same to Rodrigo Sy Mendoza. The
Court declares the petitioner to be the rightful owner of the property involved
and remands the case to the trial court to determine the liability of respondents
SAMAR-ANGLO, Rodrigo Sy Mendoza, and WESTERN GUARANTY CORPORATION
to pay actual damages that petitioner claimed.
58. Ando vs Campo

FACTS: Petitioner was the president of Premier Allied and Contracting Services,
Inc. (PACSI), an independent labor contractor. Respondents were hired by PACSI
as pilers or haulers tasked to manually carry bags of sugar from the warehouse
of Victorias Milling Company and load them on trucks. In June 1998, respondents
were dismissed from employment. They filed a case for illegal dismissal and
some money claims with the National Labor Relations Commission (NLRC),
Regional Arbitration Branch No. VI, Bacolod City. LA Pura ruled in respondents
favour, directing PACSI and petitioner to pay P 422, 702.28.
Petitioner and PACSI appealed to the NLRC. In a decision, the NLRC ruled that
petitioner failed to perfect his appeal because he did not pay the supersedeas
bond. Upon finality of the decision, respondents moved for its execution.

To answer for the monetary award, NLRC Acting Sheriff Romeo Pasustento
issued a Notice of Sale on Execution of Personal Property over the property in
the name of "Paquito V. Ando x x x married to Erlinda S. Ando."

This prompted petitioner to file an action for prohibition and damages with
prayer for the issuance of a temporary restraining order (TRO) before the
Regional Trial Court (RTC). Petitioner claimed that the property belonged to
him and his wife, not to the corporation, and, hence, could not be subject of
the execution sale. Since it is the corporation that was the judgment debtor,
execution should be made on the latters properties.
On December 27, 2006, the RTC issued an Order denying the prayer for a TRO,
holding that the trial court had no jurisdiction to try and decide the case. The

3E Andaya Ching Espiritu Hefti Galvez Gammad Lainez Lui Madamba Nagera Narvasa
Ong Palangdao Rosales Sanchez Santos Satrain Tabo (2014-2015)

Labor Relations Case Digest - Atty. Joyrich Golangco

RTC ruled that, pursuant to the NLRC Manual on the Execution of Judgment,
petitioners remedy was to file a third-party claim with the NLRC Sheriff. Despite
lack of jurisdiction, however, the RTC went on to decide the merits of the case.

to the conjugal partnership, not to petitioner alone. Thus, the property belongs to
a third party, i.e., the conjugal partnership. At the very least, the Court can
consider that petitioners wife is a third party within contemplation of the law.

Petitioner then filed the present petition seeking the nullification of the CA
Decision. He argues that he was never sued in his personal capacity, but in his
representative capacity as president of PACSI.

Whatever irregularities attended the issuance an execution of the alias writ of


execution should be referred to the same administrative tribunal which rendered
the decision. This is because any court which issued a writ of execution has the
inherent power, for the advancement of justice, to correct errors of its ministerial
officers and to control its own processes.

Petitioner filed a petition for certiorari under Rule 65 before the CA. The CA
affirmed the RTC Order in so far as it dismissed the complaint on the ground that
it had no jurisdiction over the case, and nullified all other pronouncements in the
same Order. Petitioner moved for reconsideration, but the motion was denied.

Petitioner also raises anew his argument that he can choose between filing a
third-party claim with the sheriff of the NLRC and filing a separate action. He
maintains that this special civil action is purely civil in nature since it "involves
the manner in which the writ of execution in a labor case will be implemented
against the property of petitioner which is not a corporate property of
PACSI." What he is seeking to be restrained, petitioner maintains, is not the
Decision itself but the manner of its execution.

HELD: The CA did not, in fact, err in upholding the RTCs lack of jurisdiction to
restrain the implementation of the writ of execution issued by the Labor Arbiter.

The Court has long recognized that regular courts have no jurisdiction to hear
and decide questions which arise from and are incidental to the enforcement of
decisions, orders, or awards rendered in labor cases by appropriate officers and
tribunals of the Department of Labor and Employment. To hold otherwise is to
sanction splitting of jurisdiction which is obnoxious to the orderly administration
of justice.
There is no doubt in our mind that petitioners complaint is a third- party claim
within the cognizance of the NLRC. Petitioner may indeed be considered a "third
party" in relation to the property subject of the execution vis--vis the Labor
Arbiters decision. There is no question that the property belongs to petitioner
and his wife, and not to the corporation. It can be said that the property belongs

The subject matter of the third party claim is but an incident of the labor case, a
matter beyond the jurisdiction of regional trial courts.

The TCT of the property bears out that, indeed, it belongs to petitioner and his
wife. Thus, even if we consider petitioner as an agent of the corporation and,
therefore, not a stranger to the case such that the provision on third-party
claims will not apply to him, the property was registered not only in the name of
petitioner but also of his wife. She stands to lose the property subject of
execution without ever being a party to the case. This will be tantamount to
deprivation of property without due process.
Moreover, the power of the NLRC, or the courts, to execute its judgment extends
only to properties unquestionably belonging to the judgment debtor alone. A
sheriff, therefore, has no authority to attach the property of any person except
that of the judgment debtor. Likewise, there is no showing that the sheriff ever
tried to execute on the properties of the corporation.

59. EMPLOYEES UNION OF BAYER PHILS., FFW and JUANITO S. FACUNDO, in


his capacity as President,Petitioners,
vs.
BAYER PHILIPPINES, INC., DIETER J. LONISHEN (President), ASUNCION
AMISTOSO (HRD Manager), AVELINA REMIGIO AND ANASTACIA
VILLAREAL, Respondents.

FACTS:

3E Andaya Ching Espiritu Hefti Galvez Gammad Lainez Lui Madamba Nagera Narvasa
Ong Palangdao Rosales Sanchez Santos Satrain Tabo (2014-2015)

Labor Relations Case Digest - Atty. Joyrich Golangco

Petitioner Employees Union of Bayer Philippines (EUBP) is the exclusive


bargaining agent of all rank-and-file employees of Bayer Philippines
(Bayer); also an affiliate of the Federation of Free Workers (FFW).
1997: EUBP, headed by its president Juanito S. Facundo (Facundo),
negotiated with Bayer for the signing of a collective bargaining
agreement (CBA); it resulted in a bargaining deadlock (9.9% wage
increase proposal was rejected by Facundo)
Thus, strike by EUBP was staged; DOLE assumed jurisdiction over the
dispute
November 1997: pending resolution, respondent Avelina Remigio
(Remigio) and 27 other union members, without any authority from
their union leaders, accepted Bayer's wage-increase proposal.
EUBP's grievance committee questioned Remigio's action and
reprimanded Remigio and her allies.
January 1998: the DOLE Secretary issued an arbitral award ordering EUBP
and Bayer to execute a CBA retroactive to January 1, 1997 til December
31, 2001.
Meanwhile, the rift between Facundo's leadership and Remigio's group
broadened; Remigio solicited signatures from union members in support
of a resolution to: (1) disaffiliate from FFW, (2) rename the union as
Reformed Employees Union of Bayer Philippines (REUBP), (3) adopt a
new constitution and by-laws for the union, (4) abolish all existing
officer positions in the union and elect a new set of interim officers, and
(5) authorize REUBP to administer the CBA between EUBP and Bayer.
A tug-of-war then ensued between the two rival groups (Facundos group
vs. Remigios group), with both seeking recognition from Bayer and
demanding remittance of the union dues collected from its rank-and-file
members.
September 1998: Remigio's group wrote Facundo, etc. informing them
that majority wished to disaffiliate from FFW. Facundo, meanwhile, sent
similar requests to Bayer requesting for the remittance of union dues in
favor of EUBP and accusing the company of interfering with purely
union matters. Bayer responded by deciding not to deal with either, and
by placing the union dues collected in a trust account until the conflict
between the two groups is resolved.
September 1998: EUBP filed a complaint for unfair labor practice against

Bayer for non-remittance of union dues


November 1998: EUBP later sent a letter to Bayer asking for a grievance
conference. Such was unheeded.
February 1999: while the first ULP case was still pending and despite
EUBP's repeated request for a grievance conference, Bayer decided to
turn over the collected union dues amounting to P254,857.15 to
respondent Anastacia Villareal, Treasurer of REUBP.
LABOR ARBITER DISMISSED: On June 18, 1999, Labor Arbiter Jovencio Ll.
Mayor, Jr. dismissed the first ULP complaint for lack of jurisdiction. The
Arbiter explained that the root cause for Bayer's failure to remit the
collected union dues can be traced to the intra-union conflict between
EUBP and Remigio's group and that the charges imputed against Bayer
should have been submitted instead to voluntary arbitration. EUBP did
not appeal the said decision.
December 1999: petitioners filed a second ULP complaint against,
complaining that Bayer refused to remit the collected union dues to
EUBP despite several demands sent to the management and also, alleged
that notwithstanding the requests sent to Bayer for a renegotiation of
the last two years of the 1997-2001 CBA between EUBP and Bayer, the
latter opted to negotiate instead with Remigio's group. On even date,
REUBP and Bayer agreed to sign a new CBA.
NLRC and LA: Petitioners filed for issuance of RO or Injunction, asserting
that their authority as the exclusive bargaining representative of all
employees of Bayer and
Regional Director of the Industrial Relations Division of DOLE: DISMISSED
for failure to exhaust reliefs within the union and ordering the conduct
of a referendum to determine which of the two groups should be
recognized as union officers.
Bureau of Labor Relations (BLR): REVERSED the Regional Director's
ruling and ordered the management of Bayer to respect the authority of
the duly-elected officers of EUBP in the administration of the prevailing
CBA.
Unfortunately, the said BLR ruling came late since Bayer had already
signed a new CBA with REUBP on February 21, 2000. The said CBA was
eventually ratified by majority of the bargaining unit.
Labor Arbiter: DISMISSED EUBP's second ULP complaint for lack of

3E Andaya Ching Espiritu Hefti Galvez Gammad Lainez Lui Madamba Nagera Narvasa
Ong Palangdao Rosales Sanchez Santos Satrain Tabo (2014-2015)

Labor Relations Case Digest - Atty. Joyrich Golangco

jurisdiction.
NLRC: resolved to DISMISS petitioners' motion for a restraining order
and/or injunction stating that the subject matter involved an intra-union
dispute, over which the said Commission has no jurisdiction.
CA: affirmed LA and NLRC

ISSUES:
WHETHER OR NOT THE LABOR ARBITER AND THE NLRC HAVE
JURISDICTION OVER THE INSTANT CASE;
WHETHER OR NOT THE INSTANT CASE INVOLVES AN INTRA-UNION
DISPUTE;
HELD:
Petitioners contention: Pertain to the unfair labor practice to
Respondents contention: Contend that there can be no unfair labor
practice on their part since the requisites for unfair labor practice
i.e., that the violation of the CBA should be gross, and that it should
involve violation in the economic provisions of the CBA were not
satisfied.
The petition is partly meritorious.
An intra-union dispute refers to any conflict between and among union
members, including grievances arising from any violation of the rights
and conditions of membership, violation of or disagreement over any
provision of the union's constitution and by-laws, or disputes arising
from chartering or disaffiliation of the union.
It is clear that the issues raised by petitioners do not fall under any of the
circumstances in RULE XI of Dept. Order 40-03 constituting an intraunion dispute. More importantly, the petitioners do not seek a
determination of whether it is the Facundo group (EUBP) or the Remigio
group (REUBP) which is the true set of union officers. Instead, the issue
raised pertained only to the validity of the acts of management in light of
the fact that it still has an existing CBA with EUBP. Thus as to Bayer,
Lonishen and Amistoso the question was whether they were liable for

unfair labor practice, which issue was within the jurisdiction of the
NLRC. The dismissal of the second ULP complaint was therefore
erroneous.
However, as to respondents Remigio and Villareal, we find that
petitioners' complaint was validly dismissed. Petitioners' ULP complaint
cannot prosper as against respondents Remigio and Villareal because
the issue, as against them, essentially involves an intra-union dispute
based on Section 1 (n) of DOLE Department Order No. 40-03. To rule on
the validity or illegality of their acts, the Labor Arbiter and the NLRC will
necessarily touch on the issues respecting the propriety of their
disaffiliation and the legality of the establishment of REUBP issues
that are outside the scope of their jurisdiction. Accordingly, the dismissal
of the complaint was validly made, but only with respect to these two
respondents.
But Bayer, Lonishen and Amistoso are liable for unfair labor practice. It
must be remembered that a CBA is entered into in order to foster
stability and mutual cooperation between labor and capital. An
employer should not be allowed to rescind unilaterally its CBA with the
duly certified bargaining agent it had previously contracted with, and
decide to bargain anew with a different group if there is no legitimate
reason for doing so and without first following the proper procedure. If
such behavior would be tolerated, bargaining and negotiations between
the employer and the union will never be truthful and meaningful, and
no CBA forged after arduous negotiations will ever be honored or be
relied upon.
Respondents Bayer, Lonishen and Amistoso, contend that their acts
cannot constitute ULP as the same did not involve gross violations in the
economic provisions of the CBA, citing the provisions of Articles 248 (1)
and 261 of the Labor Code, as amended. Their argument is, however,
misplaced.
In Silva v. National Labor Relations Commission, we explained the
correlations of Art. 248 (1) and Art. 261 to mean that for a ULP case to
be cognizable by the Labor Arbiter, and for the NLRC to exercise
appellate jurisdiction thereon, the allegations in the complaint must
show prima facie the concurrence of two things, namely: (1) gross

3E Andaya Ching Espiritu Hefti Galvez Gammad Lainez Lui Madamba Nagera Narvasa
Ong Palangdao Rosales Sanchez Santos Satrain Tabo (2014-2015)

Labor Relations Case Digest - Atty. Joyrich Golangco

violation of the CBA; and (2) the violation pertains to the economic
provisions of the CBA.
This pronouncement in Silva, however, should not be construed to apply
to violations of the CBA which can be considered as gross violations per
se, such as utter disregard of the very existence of the CBA itself, similar
to what happened in this case. When an employer proceeds to negotiate
with a splinter union despite the existence of its valid CBA with the duly
certified and exclusive bargaining agent, the former indubitably
abandons its recognition of the latter and terminates the entire CBA.
Respondents cannot claim good faith to justify their acts. They knew that
Facundos group represented the duly-elected officers of EUBP. They
were cognizant of the fact that even the DOLE Secretary himself had
recognized the legitimacy of EUBPs mandate by rendering an arbitral
award ordering the signing of the 1997-2001 CBA between Bayer and
EUBP. They were well-aware of the pendency of the intra-union dispute
case, yet they still proceeded to turn over the collected union dues to
REUBP and to effusively deal with Remigio. The totality of respondents
conduct, therefore, reeks with anti-EUBP animus.

WHEREFORE, the
petition
for
GRANTED. MODIFIED as follows:

review

on certiorari is PARTLY

1)Respondents Bayer Phils., Dieter J. Lonishen and Asuncion Amistoso are


found LIABLE for Unfair Labor Practice, and are hereby ORDERED to remit to
petitioners P254,857.15 representing the collected union dues previously turned
over to Avelina Remigio and Anastacia Villareal; and
2)The complaint, as against respondents Remigio and Villareal
is DISMISSED due to the lack of jurisdiction of the Labor Arbiter and the NLRC,
the complaint being in the nature of an intra-union dispute.
60. Montano vs Verceles

FACTS: Atty. Montao worked as legal assistant of Federation of Free Workers


(FFW) Legal Center. Subsequently, he joined the union of rank-and-file employees, the
FFW Staff Association, and eventually became the employees union president later on he

was likewise designated officer-in-charge of FFW Legal Center. During the 21st National
Convention and Election of National Officers of FFW, Atty. Montao was nominated for
the position of National Vice-President. However, the Commission on Election (FFW
COMELEC), informed him that he is not qualified for the position as his candidacy violates
the 1998 FFW Constitution and By-Laws, on the ground that Atty. Monatno is not an
officer or member of a legitimate labor organization because he was merely a confidential
employee of FFW. Atty. Montao thus filed an Urgent Motion for Reconsideration
praying that his name be included in the list for nominees.

The election pursued and despite a pending motion for reconsideration with the
FFW COMELEC and the opposition of Atty. Verceles, the president of University of the
East Employees Association (UEEA-FFW), an affiliate of FFW, Atty. Montano emerged
victorious. This now lead to the filing of a nullification of the election of Atty. Montano by
Atty. Verceles with the BLR, as well as a petition for injuction against the premature
assumption of office by the Atty. Montano. The latter then filed a motion to dismiss based
on the ground that it was the Regional Director of DOLE who has jurisdiction over the
intra-union dispute and not the BLR, and that Atty. Verceles was not a real party of
interest in the case. Meanwhile, a letter was sent by FFW COMELEC reiterating its former
decision that Atty. Monatnos candidacy contravenes the FFWs Constitution and that the
National Convention as a co-equal constitutional body of the Comelec was not given the
license nor the authority to violate the Constitution.
BLR issued its decision dismissing the case for lack of merit while upholding its
authority to try the case and affirming that Atty. Verceles was a real party of interest and
declared Atty. Montano as qualified by virtue of Sec. 26 of the FFW Constitution and bylaws. Thus the case was elevated to the CA which partially reversed the decision of the
BLR and declaring Atty. Mont;ano as unqualified for office, hence this petition.
ISSUES: WON it is the BLR who has jurisdiction over the intra-union dispute? If in the
affirmative, was its decision over the qualification of Atty. Montano correct?

HELD: Section 226 of the Labor Code clearly provides that the BLR and the Regional
Directors of DOLE have concurrent jurisdiction over inter-union and intra-union
disputes. Such disputes include the conduct or nullification of election of union and
workers association officers. Rule XVI lays down the decentralized intra-union dispute
settlement mechanism. Section 1 states that any complaint in this regard shall be filed in
the Regional Office where the union is domiciled. The concept of domicile in labor

3E Andaya Ching Espiritu Hefti Galvez Gammad Lainez Lui Madamba Nagera Narvasa
Ong Palangdao Rosales Sanchez Santos Satrain Tabo (2014-2015)

Labor Relations Case Digest - Atty. Joyrich Golangco

relations regulation is equivalent to the place where the union seeks to operate or has
established a geographical presence for purposes of collective bargaining or for dealing
with employers concerning terms and conditions of employment.
The matter of venue becomes problematic when the intra-union
dispute involves a federation, because the geographical presence of a federation
may encompass more than one administrative region. Pursuant to its authority
under Article 226, this Bureau exercises original jurisdiction over intra-union
disputes involving federations. It is well-settled that FFW, having local unions all
over the country, operates in more than one administrative region. Therefore,
this Bureau maintains original and exclusive jurisdiction over disputes arising
from any violation of or disagreement over any provision of its constitution and
by-laws.

In the issue of Montanos qualification, FFW COMELEC is vested with authority


and power, under the FFW Constitution and By-Laws, to screen candidates and
determine their qualifications and eligibility to run in the election and to adopt and
promulgate rules concerning the conduct of elections. The Committee is also regarded
as the final arbiter of all election protests. The FFW Constitution and By-laws are clear
that no member of the Governing Board shall at the same time perform functions of the
rank-and-file staff. Thus we note the CAs declaration of the illegitimate status of FFW
Staff Association is proscribed by law, owing to the preclusion of collateral attack and of
such accordingly, the election of Atty. Montao as FFW Vice-President is null and void.
61. DIOKNO et al. vs. CACDAC

FACTS: Petitioners Diokno et al. and private respondents Ong et al. and Daya et
al., members of First Line Association of Meralco Supervisory Employees
(FLAMES) union, filed their certificates of candidacy for the union election. Ong
et al.s candidacy were rejected by the FLAMES Comelec on the ground that Ong
is not a member of FLAMES while the others belong to a department excluded
from the scope of the Collective Bargaining Agreement (CBA). Ong et al. filed a
Petition before the Med-Arbitration Unit of the DOLE praying for the nullification
of the order of the FLAMES Comelec disallowing their candidacy and that a
representative from the Bureau of Labor Relations be designated to act as
chairman of the Comelec. The DOLE-NCR directed DOLE personnel to observe
the conduct of election on 7 May 2003. On 2 May, Diokno et al. filed with the

Comelec the disqualification of Dava et al. for committing acts inimical to the
interest of FLAMES and on 6 May, the Comelec disqualified and excluded the
names of Daya et al. in the election allegedly for violating the FLAMES
Constitution and By-Laws. On 7 May, the COMELEC proclaimed Diokno et al. as
winners in the election. On 8 May, Daya et al. together with Ong et al. filed with
the Med-Arbitration Unit a Petition to nullify the order of disqualification,
election proceedings & counting of votes and to declare a failure of election and
hold a new election to be controlled & supervised by the DOLE. The Med-Arbiter
granted the petition declaring a failure of election and ordering a new election
under the supervision of the DOLE; ruling that the Comelec erred in relying to
Art. IV, Sec. 4(a) (6) of the CBL as basis for disqualification of Daya et al. because
the provision refers to dismissal and/or expulsion of a member from FLAMES
which power is vested on the Executive Board and not to the disqualification of a
member as a candidate in a union election and that the Comelec also denied Daya
et al. due process when it failed to receive their motion for reconsideration and
refused to receive their written protest in violation of the unions CBL. The
Director of the BLR (Hon. Cacdac) affirmed in toto the assailed decision of the
Med-Arbiter. A new election was held on 30 June 2004 where Daya et al. won but
was challenged by Diokno et al. before the DOLE and the CA questioning the
jurisdiction of the BLR. The CA upheld the validity of the 30 June elections
declaring Daya et al. as the duly elected winners therein and affirmed the
jurisdiction of the BLR.
ISSUES: Whether or not the BLR acquired jurisdiction over the case at bar?

HELD: YES. Under Article 226 of the Labor Code, the Bureau of Labor Relations
(BLR) has the original and exclusive jurisdiction on all inter-union and intraunion conflicts. The case at bar falls under intra-union conflicts which was
defined under Section (z), Rule I of the Rules Implementing Book V of the Labor
Code, viz: (z) "Intra-Union Dispute refers to any conflict between and among
union members, and includes all disputes or grievances arising from any
violation of or disagreement over any provision of the constitution and by-laws
of a union, including cases arising from chartering or affiliation of labor
organizations or from any violation of the rights and conditions of union
membership provided for in the Code.

3E Andaya Ching Espiritu Hefti Galvez Gammad Lainez Lui Madamba Nagera Narvasa
Ong Palangdao Rosales Sanchez Santos Satrain Tabo (2014-2015)

Labor Relations Case Digest - Atty. Joyrich Golangco

The Court denied the petition and affirmed the decision of the CA with costs
against petitioners.
62. MAGBANUA vs. UY

DOCTRINE: Rights may be waived through a compromise agreement,


notwithstanding a final judgment that has already settled the rights of the
contracting parties. To be binding, the compromise must be shown to have been
voluntarily, freely and intelligently executed by the parties, who had full
knowledge of the judgment. Furthermore, it must not be contrary to law, morals,
good customs and public policy.

FACTS: As a final consequence of the final and executory decision of the


Supreme Court in Rizalino P. Uy v. National Labor Relations Commission, et. al.
(GR No. 117983, September 6, 1996), hearings were conducted to determine the
amount of wage differentials due the eight (8) complainants therein, now
[petitioners]. As computed, the award amounted to P1,487,312.69 x x x.

On February 3, 1997, [petitioners] filed a Motion for Issuance of Writ of


Execution.
On May 19, 1997, [respondent] Rizalino Uy filed a Manifestation requesting that
the cases be terminated and closed, stating that the judgment award as
computed had been complied with to the satisfaction of [petitioners]. Said
Manifestation was also signed by the eight (8) [petitioners]. Together with the
Manifestation is a Joint Affidavit dated May 5, 1997 of [petitioners], attesting to
the receipt of payment from [respondent] and waiving all other benefits due
them in connection with their complaint.
On June 3, 1997, [petitioners] filed an Urgent Motion for Issuance of Writ of
Execution wherein they confirmed that each of them received P40,000 from
[respondent] on May 2, 1997.
On June 9, 1997, [respondent] opposed the motion on the ground that the
judgment award had been fully satisfied. In their Reply, [petitioners] claimed
that they received only partial payments of the judgment award.
LA: issued an order denying the motion for issuance of writ of execution.

NLRC: reversed,holding that a final and executory judgment can no longer be


altered and that quitclaims and releases are normally frowned upon as contrary
to public policy.
CA held that compromise agreements may be entered into even after a final
judgment. Thus, petitioners validly released respondent from any claims, upon
the voluntary execution of a waiver pursuant to the compromise agreement.
Issues: Whether or not the final and executory judgment of the Supreme Court
could be subject to compromise settlement;
Whether or not the petitioners affidavit waiving their awards in the labor case
executed without the assistance of their counsel and labor arbiter is valid;

HELD:
1. Yes, A compromise agreement is a contract whereby the parties make
reciprocal concessions in order to resolve their differences and thus
avoid or put an end to a lawsuit.
The issue involving the validity of a compromise agreement notwithstanding a
final judgment is not novel. Jesalva v. Bautista upheld a compromise agreement
that covered cases pending trial, on appeal, and with final judgment. The Court
noted that Article 2040 impliedly allowed such agreements; there was no
limitation as to when these should be entered into.
There is no justification to disallow a compromise agreement, solely because it
was entered into after final judgment. The validity of the agreement is
determined by compliance with the requisites and principles of contracts, not by
when it was entered into. As provided by the law on contracts, a valid
compromise must have the following elements: (1) the consent of the
parties to the compromise, (2) an object certain that is the subject matter
of the compromise, and (3) the cause of the obligation that is established.
In the present factual milieu, compliance with the elements of a valid contract is
not in issue. Petitioners do not challenge the factual finding that they entered
into a compromise agreement with respondent. There are no allegations of
vitiated consent. Instead, petitioners base their argument on the sole fact that
the agreement was executed despite a final judgment, which the Court had
previously ruled to be allowed by law.

3E Andaya Ching Espiritu Hefti Galvez Gammad Lainez Lui Madamba Nagera Narvasa
Ong Palangdao Rosales Sanchez Santos Satrain Tabo (2014-2015)

Labor Relations Case Digest - Atty. Joyrich Golangco

The principle of novation supports the validity of a compromise after final


judgment. Novation, a mode of extinguishing an obligation, is done by changing
the object or principal condition of an obligation, substituting the person of the
debtor, or surrogating a third person in the exercise of the rights of the creditor.
For an obligation to be extinguished by another, the law requires either of these
two conditions: (1) the substitution is unequivocally declared, or (2) the old and
the new obligations are incompatible on every point.[45] A compromise of a final
judgment operates as a novation of the judgment obligation, upon compliance
with either requisite.[46] In the present case, the incompatibility of the final
judgment with the compromise agreement is evident, because the latter was
precisely entered into to supersede the former.

confronting the parties, particularly the complainants, he is satisfied that they


understand the terms and conditions of the settlement and that it was entered
into freely and voluntarily by them and the agreement is not contrary to law,
morals, and public policy.
This provision refers to proceedings in a mandatory/conciliation conference
during the initial stage of the litigation. Such provision should be made
applicable to the proceedings in the pre-execution conference, for which the
procedure for approving a waiver after final judgment is not stated. There is no
reason to make a distinction between the proceedings in mandatory/conciliation
and those in pre-execution conferences.
63. Solomon et. Al vs Powertech Corp.

Yes, The presence or the absence of counsel when a waiver is executed


does not determine its validity. There is no law requiring the presence
of a counsel to validate a waiver. The test is whether it was executed
voluntarily, freely and intelligently; and whether the consideration for it
was credible and reasonable. Where there is clear proof that a waiver
was wangled from an unsuspecting or a gullible person, the law must
step in to annul such transaction. In the present case, petitioners failed
to present any evidence to show that their consent had been vitiated.

FACTS: The case stems from a complaint for illegal dismissal and other money
claims filed by the Nagkakaisang Manggagawa Ng Powertech Corporation in
behalf of its 52 individual members and non-union members against their
employer, Powertech. The case was dismissed as to twenty-seven (27)
employees by virtue of duly executed affidavits of repudiation and quitclaim.
The case proceeded with respect to the remaining twenty-five (25) employees,
petitioners in this case.

Should the parties arrive at any agreement as to the whole or any part of the
dispute, the same shall be reduced to writing and signed by the parties and their
respective counsel, or authorized representative, if any, before the Labor Arbiter.

Powertech appealed to the NLRC. During its pendency, Carlos Gestiada, for
himself and on behalf of other petitioners, executed a quitclaim, release and
waiver in favor of Powertech in consideration of the amount of P150,000.00.

2.

The law is silent with regard to the procedure for approving a waiver after a case
has been terminated. Relevant, however, is this reference to the NLRCs New
Rules of Procedure:

The settlement shall be approved by the Labor Arbiter after being satisfied that
it was voluntarily entered into by the parties and after having explained to them
the terms and consequences thereof.

A compromise agreement entered into by the parties not in the presence of the
Labor Arbiter before whom the case is pending shall be approved by him, if after

On June 25, 1999, Labor Arbiter Dela Cruz rendered a Decision declaring illegal
the termination of twenty (20) of petitioners and granting their monetary claims
in the total amount of P2,538,728.84.

Relying on the quitclaim and release, Powertech filed a motion for the
withdrawal of the appeal and cash bond. The NLRC granted the motion,
dismissed the appeal and ordered the release of the cash bond.

The P150,000.00 check, however, bounced due to a stop payment order of


Powertech.

3E Andaya Ching Espiritu Hefti Galvez Gammad Lainez Lui Madamba Nagera Narvasa
Ong Palangdao Rosales Sanchez Santos Satrain Tabo (2014-2015)

Labor Relations Case Digest - Atty. Joyrich Golangco


Aggrieved, petitioners moved to nullify the release and quitclaim for lack of
consideration. In a Resolution dated February 29, 2000, the NLRC declared the
quitclaim, release and waiver void for lack of consideration, reinstated the appeal
and ordered Powertech to post a cash or surety bond for the monetary judgment
less the amount it had previously posted

A day later, Powertech paid P150,000.00 to Gestiada purportedly as compromise


amount for all of petitioners. That same day, Gestiada, through Atty. Felipe, and
Powertech filed a joint motion to dismiss with the NLRC based on the
compromise agreement. Atty. Evangelista opposed the motion, alleging that the
compromise agreement is unconscionable, that he was illegally terminated as
counsel for the other petitioners without their consent, and that the P150,000.00
was received by Gestiada as payment solely for his backwages and other
monetary claims.
On July 31, 2000, the NLRC issued a resolution denying the joint motion to
dismiss. The NLRC held that the P150,000.00 received by Gestiada did not cover
the monetary claim of petitioners against Powertech.

Undaunted, Powertech elevated the matter to the CA via petition for certiorari
under Rule 65 of the 1997 Rules of Civil Procedure. The CA upheld the validity of
the compromise agreement between petitioners and Powertech. Petitioners
moved to reconsider the CA decision but their motion was denied. Hence, the
present recourse.
ISSUES: Whether the Compromise Agreement is VALID?

HELD: NO. The P150,000 was paid to Gestiada solely as payment for his
backwages, not those of petitioners; there is evident collusion between
Powertech and Gestiada, hence, the compromise agreement is void.

First, the P150,000 compromise is rather measly when taken in light of the more
than P2.5 million judgment on appeal to the NLRC. Petitioners already won on
the arbiter level P2.5 million pesos. It is highly improbable that they would
suddenly agree to accept P150,000 as compromise for the P2.5 million. That

translates to a paltry sum of P6,000.00 each for petitioners. From this amount
will still be deducted attorneys fees and other litigation expenses. In effect,
petitioners agreed to waive more than 94% of what they expect to receive from
Powertech. We note that the compromise is a mere 6% of the contingent sum
that may be received by petitioners. This minuscule amount is certainly
questionable because, to Our mind, it does not represent a true and fair amount
which a reasonable agent may bargain for his principal.
Second, even granting for the mere sake of argument that the P150,000 was a fair
and reasonable compromise for all, petitioners failed to receive a single centavo
from the compromise. This conclusively indicates that Gestiada received the
P150,000 in payment of his backwages and no other.

Third, We give credence to the admission of Gestiada that he received the


P150,000.00 as payment for his own backwages. In his letter to Atty.
Evangelista, Gestiada said that he was pressured by Powertech to sign the waiver
and quitclaim for petitioners in order to receive his share in the P2.5 million
judgment. Having no stable job after his dismissal, Gestiada had no other choice
but to breach his fiduciary obligation to petitioners. He succumbed to the
pressure of Powertech in signing the waiver, release and quitclaim in exchange
for the P150,000.00. In short, he colluded with Powertech to the detriment of
petitioners.

Fourth, the events that led to the execution of the compromise agreement show
that Powertech was negotiating in bad faith. More importantly, they show that
Powertech colluded with Gestiada to defraud petitioners of their share of the
P2.5 million Labor Arbiter judgment.
Collusion is a species of fraud. Article 227 of the Labor Code empowers the NLRC
to void a compromise agreement for fraud, thus:
Any compromise settlement, including those involving labor
standard laws, voluntarily agreed upon by the parties with the
assistance of the Bureau or the regional office of the Department
of Labor, shall be final and binding upon the parties. The
National Labor Relations Commission or any court shall not

3E Andaya Ching Espiritu Hefti Galvez Gammad Lainez Lui Madamba Nagera Narvasa
Ong Palangdao Rosales Sanchez Santos Satrain Tabo (2014-2015)

Labor Relations Case Digest - Atty. Joyrich Golangco

assume jurisdiction over issues involved therein except in case


of non-compliance thereof or if there is prima facie evidence
that the settlement was obtained through fraud,
misrepresentation, or coercion.

In fine, We find that the CA erred in upholding the compromise agreement


between Powertech and Gestiada.
The NLRC justifiably declared the
compromise agreement as void.
64. Philippine Journalist, Inc. (PJI) vs NLRC

FACTS: -Petitioner PJI is a local corporation engaged in the business of


publication and sale of newspapers and magazines. The respondent Journal
Employees Union (or Union) is the bargaining agent of all the rank-and-file
employees.

-PJI implemented a retrenchment program due to over-staffing and continuing


actual losses for the past years where 31 employees were dismissed from
employment. That is the reason as to why the Union filed a notice to strike as
they claimed that PJI was guilty of unfair labor practice. Subsequently thereafter
the Labor Secretary certified the labor dispute between the parties to the NLRC
for mandatory arbitration.

-Ruling of NLRC (1st): the Commission issued its Resolution on May 31 2001
declaring that 31 employees were illegally dismissed, PJI failed to prove that the
retrenchment was valid as PJI was, among others, able to hold its Christmas
party in a luxurious hotel, made renovations and various purchases for the office,
and able to grant merit increases to employees.
-Subsequently the parties entered into a Compromise Agreement (July 9,
2001) that PJI would reinstate the 31 employees without loss of seniority rights
and benefits and to pay the latter their full and complete monetary claims.
-comes 2002 where another notice to strike was filed by the Union. They claimed
that 29 employees were illegally dismissed, salaries of 50 others were reduced,
and 200 Union members were made to sign a 5-month contract immediately
after the retrenchment program was implemented. The Union prayed that
reinstatement be made and that the benefits under the CBA be restored.

-Ruling of the NLRC (2nd): they were not illegally dismissed. The Commission
noted that the May 31, 2001 Reso couldnt be used by the Union as it DID NOT
attain its finality due to the fact that a Compromise Agreement was made it has
already been mooted and rendered academic. Neither can they claim stare
decisis based on the May 31 Reso. as it involves different set facts (first one
during the retrenchment program and the 2nd one involves facts long after the
retrenchment). Also, the Union cannot invoke CBA benefits as the 200 employees
are now considered contractual employees and necessarily excluded in the
collective bargaining unit. ---this ruling was elevated to the CA
-Ruling of the CA: NLRC gravely abused its discretion in ruling in favor of PJI.
Making 200 employees sign a 5-month contract is an act which shows
circumvention of law and the CBA as it was made after the retrenchment
program and only goes to show the intent of PJI to reduce the rights and benefits
of its employees. The May 31 Reso stands as it reached a verdict that the
retrenchment was illegally made despite the Compromise Agreement being
made.
ISSUES: Whether an NLRC Resolution, which includes a pronouncement that the
members of a union had been illegally dismissed, is abandoned or rendered
"moot and academic" by a compromise agreement subsequently entered into
between the dismissed employees and the employer; this, in turn, raises the
question of whether such a compromise agreement constitutes res judicata to a
new complaint later filed by other union members-employees, not parties to the
agreement, who likewise claim to have been illegally dismissed.

HELD: Negative. Petition DENIED. We affirm the CA. The nature of a compromise
is spelled out in Article 2028 of the New Civil Code: it is "a contract whereby the
parties, by making reciprocal concessions, avoid litigation or put an end to one
already commenced." Parties to a compromise are motivated by "the hope of
gaining, balanced by the dangers of losing." It contemplates mutual concessions
and mutual gains to avoid the expenses of litigation, or, when litigation has
already begun, to end it because of the uncertainty of the result. Article 227 of
the Labor Code of the Philippines authorizes compromise agreements voluntarily
agreed upon by the parties, in conformity with the basic policy of the State "to

3E Andaya Ching Espiritu Hefti Galvez Gammad Lainez Lui Madamba Nagera Narvasa
Ong Palangdao Rosales Sanchez Santos Satrain Tabo (2014-2015)

Labor Relations Case Digest - Atty. Joyrich Golangco

promote and emphasize the primacy of free collective bargaining and


negotiations, including voluntary arbitration, mediation and conciliation, as
modes of settling labor or industrial disputes."
ART. 227 Compromise Agreements. Any compromise settlement, including
those involving labor standard laws, voluntarily agreed upon by the parties with
the assistance of the Bureau or the regional office of the Department of Labor,
shall be final and binding upon the parties. The National Labor Relations
Commission or any court shall not assume jurisdiction over issues involved
therein except in case of noncompliance thereof or if there is prima facie
evidence that the settlement was obtained through fraud, misrepresentation, or
coercion.

A judgment rendered in accordance with a compromise agreement is not


appealable, and is immediately executory unless a motion is filed to set aside the
agreement on the ground of fraud, mistake, or duress, in which case an appeal
may be taken against the order denying the motion. Under Article 2037 of the
Civil Code, "a compromise has upon the parties the effect and authority of res
judicata," even when effected without judicial approval; and under the principle
of res judicata, an issue which had already been laid to rest by the parties
themselves can no longer be relitigated.

Adjective law governing judicial compromises annunciate that once


approved by the court, a judicial compromise is not appealable and it thereby
becomes immediately executory but this rule must be understood to refer and
apply only to those who are bound by the compromise and, on the assumption
that they are the only parties to the case, the litigation comes to an end except
only as regards to its compliance and the fulfillment by the parties of their
respective obligations thereunder. The reason for the rule, said the Court in
Domingo v. Court of Appeals [325 Phil. 469], is that when both parties so enter
into the agreement to put a close to a pending litigation between them and ask
that a decision be rendered in conformity therewith, it would only be "natural to
presume that such action constitutes an implicit waiver of the right to appeal"
against that decision. The order approving the compromise agreement thus
becomes a final act, and it forms part and parcel of the judgment that can be
enforced by a writ of execution unless otherwise enjoined by a restraining order.

Thus, contrary to the allegation of petitioners, the execution and


subsequent approval by the NLRC of the agreement forged between it and the
respondent Union did not render the NLRC resolution ineffectual, nor rendered it
"moot and academic." The agreement becomes part of the judgment of the court
or tribunal, and as a logical consequence, there is an implicit waiver of the right
to appeal.

In any event, the compromise agreement cannot bind a party who did
not voluntarily take part in the settlement itself and gave specific individual
consent. It must be remembered that a compromise agreement is also a contract;
it requires the consent of the parties, and it is only then that the agreement may
be considered as voluntarily entered into.
A careful perusal of the wordings of the compromise agreement will
show that the parties agreed that the only issue to be resolved was the question
of the monetary claim of several employees. The prayer of the parties in the
compromise agreement which was submitted to the NLRC reads:

WHEREFORE, premises considered, it is respectfully prayed that the


Compromise Settlement be noted and considered; that the instant case [be]
deemed close[d] and terminated and that the Decision dated May 31, 2001
rendered herein by this Honorable Commission be deemed to be fully
implemented insofar as concerns the thirty-one (31) employees mentioned in
paragraphs 2c and 2d hereof; and, that the only issue remaining to be resolved
be limited to the question of the monetary claim raised in the motion for
clarification by the seven employees mentioned in paragraph 2e hereof.
The agreement was later approved by the NLRC. The case was
considered closed and terminated and the Resolution dated May 31, 2001 fully
implemented insofar as the employees "mentioned in paragraphs 2c and 2d of
the compromise agreement" were concerned. Hence, the CA was correct in
holding that the compromise agreement pertained only to the "monetary
obligation" of the employer to the dismissed employees, and in no way affected
May 31, 2001 Reso where the NLRC made the pronouncement that there was no
basis for the implementation of petitioners' retrenchment program.

3E Andaya Ching Espiritu Hefti Galvez Gammad Lainez Lui Madamba Nagera Narvasa
Ong Palangdao Rosales Sanchez Santos Satrain Tabo (2014-2015)

Labor Relations Case Digest - Atty. Joyrich Golangco

To reiterate, the rule is that when judgment is rendered based on a


compromise agreement, the judgment becomes immediately executory, there
being an implied waiver of the parties' right to appeal from the decision.43 The
judgment having become final, the Court can no longer reverse, much less modify
it.

Lastly, it could not be said that the employees in this case are barred from
pursuing their claims because of their acceptance of separation pay and their
signing of quitclaims. It is settled that "quitclaims, waivers and/or complete
releases executed by employees do not stop them from pursuing their claims if
there is a showing of undue pressure or duress. The basic reason for this is that
such quitclaims, waivers and/or complete releases being figuratively exacted
through the barrel of a gun, are against public policy and therefore null and void
ab initio. In the case at bar, the employees were faced with impending
termination. As such, it was but natural for them to accept whatever monetary
benefits that they could get.
65. COLEGIO DE SAN JUAN DE LETRAN,-versus-ASSOCIATION OF
EMPLOYEES AND FACULTY OF LETRAN and ELEONOR AMBAS

FACTS: On December 1992, Salvador Abtria, then President of respondent union,


Association of Employees and Faculty of Letran, initiated the renegotiation of its
Collective Bargaining Agreement with petitioner for the last two (2) years of the
CBAs five (5) year lifetime from 1989-1994. On the same year, the union elected
a new set of officers wherein private respondent Eleanor Ambas emerged as the
newly elected President.
Ambas wanted to continue the renegotiation of the CBA but petitioner, through
Fr. Edwin Lao, claimed that the CBA was already prepared for signing by the
parties. The parties submitted the disputed CBA to a referendum by the union
members, who eventually rejected the said CBA. Petitioner accused the union
officers of bargaining in bad faith before the NLRC.

LA:Labor Arbiter Edgardo M. Madriaga decided in favor of petitioner. However,


the Labor Arbiters decision was reversed on appeal before the NLRC.

The union notified the National Conciliation and Mediation Board (NCMB) of its
intention to strike on the grounds (sic) of petitioners: non-compliance with the

NLRC (1) order to delete the name of Atty. Federico Leynes as the unions legal
counsel; and (2) refusal to bargain.

The parties agreed to disregard the unsigned CBA and to start negotiation on a
new five-year CBA starting 1994-1999. On February 7, 1996, the union
submitted its proposals to petitioner, which notified the union six days later or
on February 13, 1996 that the same had been submitted to its Board of Trustees.

In the meantime, Ambas was informed through a letter dated February 15, 1996
from her superior that her work schedule was being changed from Monday to
Friday to Tuesday to Saturday. Ambas protested and requested management to
submit the issue to a grievance machinery under the old CBA (Ibid, p. 2-3). Due to
petitioners inaction, the union filed a notice of strike on March 13, 1996. The
parties met on March 27, 1996 before the NCMB to discuss the ground rules for
the negotiation. On March 29, 1996, the union received petitioners letter
dismissing Ambas for alleged insubordination. Hence, the union amended its
notice of strike to include Ambas dismissal.
Petitioner stopped the negotiations after it purportedly received information
that a new group of employees had filed a petition for certification election.

On June 18, 1996, the union finally struck. The Secretary of Labor and
Employment assumed jurisdiction and ordered all striking employees including
the union president to return to work and for petitioner to accept them back
under the same terms and conditions before the actual strike. Petitioner
readmitted the striking members except Ambas.
DOLE: On December 2, 1996, public respondent issued an order declaring
petitioner guilty of unfair labor practice on two counts and directing the
reinstatement of private respondent Ambas with backwages.

CA: Having been denied its motion for reconsideration, petitioner sought a
review of the order of the Secretary of Labor and Employment before the Court
of Appeals. The appellate court dismissed the petition and affirmed the findings
of the Secretary of Labor and Employment.
Petitioner ascribes the following errors to the Court of Appeals:

3E Andaya Ching Espiritu Hefti Galvez Gammad Lainez Lui Madamba Nagera Narvasa
Ong Palangdao Rosales Sanchez Santos Satrain Tabo (2014-2015)

Labor Relations Case Digest - Atty. Joyrich Golangco

ISSUES: (1) Whether petitioner is guilty of unfair labor practice by refusing to


bargain with the union when it unilaterally suspended the ongoing negotiations
for a new Collective Bargaining Agreement (CBA) upon mere information that a
petition for certification has been filed by another legitimate labor organization?
(2)Whether the termination of the union president amounts to an interference of
the employees right to self-organization.
HELD: The petition is without merit. As regards the first issue, Article 252 of the
Labor Code defines the meaning of the phrase duty to bargain collectively,

Noteworthy in the above definition is the requirement on both parties of the


performance of the mutual obligation to meet and convene promptly and
expeditiously in good faith for the purpose of negotiating an agreement.
Undoubtedly, respondent Association of Employees and Faculty of Letran (AEFL)
(hereinafter, union) lived up to this requisite when it presented its proposals
for the CBA to petitioner on February 7, 1996. On the other hand, petitioner
devised ways and means in order to prevent the negotiation.
Petitioners utter lack of interest in bargaining with the union is obvious in its
failure to make a timely reply to the proposals presented by the latter. More than
a month after the proposals were submitted by the union, petitioner still had not
made any counterproposals. This inaction on the part of petitioner prompted the
union to file its second notice of strike. Petitioner could only offer a feeble
explanation that the Board of Trustees had not yet convened to discuss the
matter as its excuse for failing to file its reply. This is a clear violation of
Article 250 of the Labor Code governing the procedure in collective bargaining,
to wit:

Art. 250. Procedure in collective bargaining. The following procedures shall be


observed in collective bargaining: (a) When a party desires to negotiate an
agreement, it shall serve a written notice upon the other party with a statement
of its proposals. The other party shall make a reply thereto not later than ten
(10) calendar days from receipt of such notice.[4]
xxxxxxxxx

As we have held in the case of Kiok Loy vs. NLRC,[5] the companys refusal to
make counter-proposal to the unions proposed CBA is an indication of its bad
faith.

In the case at bar, petitioners actuation show a lack of sincere desire to negotiate
rendering it guilty of unfair labor practice.

Moreover, the series of events that transpired after the filing of the first notice of
strike in January 1996 show petitioners resort to delaying tactics to ensure that
negotiation would not push through. Thus, on February 15, 1996, or barely a few
days after the union proposals for the new CBA were submitted, the union
president was informed by her superior that her work schedule was being
changed from Mondays to Fridays to Tuesdays to Saturdays. A request from the
union president that the issue be submitted to a grievance machinery was
subsequently denied. Thereafter, the petitioner and the union met on March 27,
1996 to discuss the ground rules for negotiation. However, just two days later, or
on March 29, 1996, petitionerdismissed the union president for alleged
insubordination. In its final attempt to thwart the bargaining process, petitioner
suspended the negotiation on the ground that it allegedly received information
that a new group of employees called the Association of Concerned Employees of
Colegio (ACEC) had filed a petition for certification election. Clearly, petitioner
tried to evade its duty to bargain collectively.
PETiTIONERS CONTENTION:

Petitioner, however, argues that since it has already submitted the unions
proposals to the Board of Trustees and that a series of conferences had already
been undertaken to discuss the ground rules for negotiation such should already
be considered as acts indicative of its intention to bargain.

Petitioner, likewise, claims that the suspension of negotiation was proper since
by the filing of the petition for certification election the issue on majority
representation of the employees has arose. According to petitioner, the authority
of the union to negotiate on behalf of the employees was challenged when a rival
union filed a petition for certification election..
We disagree.

3E Andaya Ching Espiritu Hefti Galvez Gammad Lainez Lui Madamba Nagera Narvasa
Ong Palangdao Rosales Sanchez Santos Satrain Tabo (2014-2015)

Labor Relations Case Digest - Atty. Joyrich Golangco

In order to allow the employer to validly suspend the bargaining process there
must be a valid petition for certification election raising a legitimate
representation issue. Hence, the mere filing of a petition for certification election
does not ipso facto justify the suspension of negotiation by the employer.
The Contract Bar Rule under Section 3, Rule XI, Book V, of the Omnibus Rules
Implementing the Labor Code, provides that:

If a collective bargaining agreement has been duly registered in accordance


with Article 231 of the Code, a petition for certification election or a motion for
intervention can only be entertained within sixty (60) days prior to the expiry
date of such agreement.

The rule is based on Article 232, in relation to Articles 253, 253-A and 256 of the
Labor Code. No petition for certification election for any representation issue
may be filed after the lapse of the sixty-day freedom period. The old CBA is
extended until a new one is signed. The petition for certification election by
ACEC, allegedly a legitimate labor organization, was filed with the Department of
Labor and Employment (DOLE) only on May 26, 1996.

Clearly, the petition was filed outside the sixty-day freedom period. Hence, the
filing thereof was barred by the existence of a valid and existing collective
bargaining agreement. Consequently, there is no legitimate representation issue
and, as such, the filing of the petition for certification election did not constitute a
bar to the ongoing negotiation.

In view of the above, there is no doubt that petitioner is guilty of unfair labor
practice by its stern refusal to bargain in good faith with respondent union.
Concerning the issue on the validity of the termination of the union president, we
hold that the dismissal was effected in violation of the employees right to self
organization.
Admittedly, management has the prerogative to discipline its employees for
insubordination. But when the exercise of such management right tends to
interfere with the employees right to self-organization, it amounts to unionbusting and is therefore a prohibited act.
WHEREFORE, premises considered, the petition is DENIED for lack of merit.

66. Mariwasa Siam Ceramics vs. Secretary of Labor and Employment, et. al.

FACTS: On May 2005, private respondent Samahan Ng Mga Manggagawa Sa


Mariwasa Siam
Ceramics, Inc. (SMMSC-Independent) was issued a Certificate of Registration as a
legitimate labor organization by the Department of Labor and Employment
(DOLE), Region IV-A.On June 2005, petitioner Mariwasa Siam Ceramics, Inc. filed
a Petition for Cancellation of Union Registration against private respondent,
claiming that the latter violated Article 234 of the Labor Code for not complying
with the 20% requirement and that it committed massive fraud and
misrepresentation in violation of Article 239 of the same code. The Regional
Director of DOLE IV-A issued an Order granting the petition, revoking the
registration of respondent, and delisting it from the roster of active labor unions.
SMMSC Independent appealed to the Bureau of Labor Relations. BLR ruled in
favor of the respondent, thus, they remain in the roster of legitimate labor
organizations. The petitioner appealed and insisted that private respondent
failed to comply with the 20% union membership requirement for its
registration as a legitimate labor organization because of the disaffiliation from
the total number of union members of 102 employees who executed affidavits
recanting their union membership.
Hence, this petition for review on certiorari under Rule 45 of the Rules of Court.
Issues: 1) Was there failure to comply with the 20% union membership
requirement?
2) Did the withdrawal of 31 union members affect the petition for certification
election insofar as the 30% requirement is concerned?

HELD: No.While it is true that the withdrawal of support may be considered as a


resignation from the union, the fact remains that at the time of the unions
application for registration, the affiants were members of respondent and they
comprised more than the required 20% membership for purposes of registration
as a labor union. Article 234 of the Labor Code merely requires a 20% minimum
membership during the application for union registration. It does not mandate
that a union must maintain the 20% minimum membership requirement all
throughout its existence. On the second issue, it appears undisputedly that the 31

3E Andaya Ching Espiritu Hefti Galvez Gammad Lainez Lui Madamba Nagera Narvasa
Ong Palangdao Rosales Sanchez Santos Satrain Tabo (2014-2015)

Labor Relations Case Digest - Atty. Joyrich Golangco

union members had withdrawn their support to the petition before the filing of
said petition. The distinction must be that withdrawals made before the filing of
the petition are presumed voluntary unless there is convincing proof to the
contrary, whereas withdrawals made after the filing of the petition are deemed
involuntary. Therefore, following jurisprudence, the employees were not totally
free from the employers pressure and so the voluntariness of the employees
execution of the affidavits becomes suspect.

The cancellation of a unions registration doubtless has an impairing dimension


on the right of labor to self-organization. For fraud and misrepresentation to be
grounds for cancellation of union registration under the Labor Code, the nature
of the fraud and misrepresentation must be grave and compelling enough to
vitiate the consent of a majority of union members.
67. Electromat Manufacturing and Recording Corporation vs. Lagunzad

FACTS: the Private respondent Nagkakaisang Samahan ng Manggawa ng


Electomat-Wasto (union) a chartered affiliate of the Workers Advocate for
Struggle, transformation and organization (Wasto) applied for a registration with
the BLR and passed the following documents
a.
b.
c.
d.
e.
f.
g.
h.
i.
j.
k.

Copies of the ratified constitution and by-laws (CBL)


Minutes of the CBLs adoption and ratification
Minutes of the organizational meetings
Names and addresses of the union officers
List of union members
List of rank and file employees of the company
Certification of non-existence of a CBA in the company
Resolution of affiliation with WASTO a labor federation
WASTOs resolution of acceptance
Charter certificate
Verification under oath

BLR thereafter issued a certification of creation of Local Chapter (equivalent to


certificate of recognition of an independent union) pursuant to D.O. 40-03,
Petitioner Electromat filled for the cancellation of the certificate on the ground of
failure to comply with Art 234 of LC and that DO 40-03 if unconstitutional for

being an invalid amendment of the LC since it reduced the requirements of Art


234 of LC and that it is an invalid exercise of the rule making power of DOLE

DOLE acting director Ciriaco Langunzad dismissed the petition which was
affirmed by BLR Director Hans Leo Cacdac and also affirmed by the CA, hence
this appeal to SC, stating that the CA committed reversible error in upholding the
registration certificate issued by BLR
Sec 2(E), Rule III of DO 40-03 the report of creation of chartered local shall be
accompanied by a charter certificate issued by the federal or national union
indicating the creation or establishment of the chartered local.

Art 234 - requirements for registration. Any applicant labor organization,


association or group of unions or workers shall acquire legal personality and
shall be entitled to the rights and privileges granted by law to legitimate labor
organizations upon issuance of the certificate of registration based on the
following requirements:
(a) Fifty pesos (P50.00) registration fee;
(b) The names of its officers, their addresses, the principal address of the labor
organization, the minutes of the organizational meetings and the list of the
workers who participated in such meetings;
(c) The names of all its members comprising at least twenty percent (20%) of
all the employees in the bargaining unit where it seeks to operate;
(d) If the applicant union has been in existence for one or more years, copies of
its annual financial reports; and
(e) Four (4) copies of the constitution and by-laws of the applicant union,
minutes of its adoption or ratification, and the list of the members who
participated in it.
ISSUES: whether or not the union satisfied the requirements of Art 234

HELD: Yes, the union in this case has more than satisfied the requirements the
petitioner complains about specifically the union has submitted
a. Copies of the ratified constitution and by-laws (CBL)
b. Minutes of the CBLs adoption and ratification
c. Minutes of the organizational meetings
d. Names and addresses of the union officers

3E Andaya Ching Espiritu Hefti Galvez Gammad Lainez Lui Madamba Nagera Narvasa
Ong Palangdao Rosales Sanchez Santos Satrain Tabo (2014-2015)

e.
f.
g.
h.
i.

Labor Relations Case Digest - Atty. Joyrich Golangco

List of union members


List of rank and file employees of the company
Certification of non-existence of a CBA in the company
Resolution of affiliation with WASTO and the latters acceptance
Their Charter certificate

and these submissions are property verified as required by the rules. In sum, the
petitioner has no factual basis for questioning the unions registration as event
the requirements for registration as an independent local have been complied
with.

(In case na tanungin) WON DO 40-03 is a valid exercise of the rule-making power
of the DOLE
Yes, in an Earlier case in Progressive Development Corporation v. Secretary, of
DOLE the Courte encountered a similar question on the validity of the old Section
3, Rule II, Book V of the Rules Implementing the Labor Code which stated:
Union affiliation; direct membership with a national union. - The affiliate of a
labor federation or national union may be a local or chapter thereof or an
independently registered union.

a) The labor federation or national union concerned shall issue a charter


certificate indicating the creation or establishment of a local or chapter,
copy of which shall be submitted to the Bureau of Labor Relations within
thirty (30) days from issuance of such charter certificate.
xxxx
e) The local or chapter of a labor federation or national union shall have
and maintain a constitution and by-laws, set of officers and books of
accounts. For reporting purposes, the procedure governing the reporting of
independently registered unions, federations or national unions shall be
observed.
Interpreting these provisions of the old rules, the Court said that by force of law
the local or chapter of a labor federation or national union becomes a legitimate
labor organization upon compliance with Section 3, Rule II, Book V of the Rules
Implementing the Labor Code, the only requirement being the submission of the
charter certificate to the BLR, Undoubtedly, the intent of the law in imposing

lesser requirements in the case of a branch or local of a registered federation or


national union is to encourage the affiliation of a local union with a federation or
national union in order to increase the local unions bargaining powers
respecting terms and conditions of labor.

D.O. 40-03 represents an expression of the governments implementing policy on


trade unionism. It builds upon the old rules by further simplifying the
requirements for the establishment of locals or chapters. We see nothing
contrary to the law or the Constitution in the adoption by the Secretary of Labor
and Employment of D.O. 40-03 as this department order is consistent with the
intent of the government to encourage the affiliation of a local union with a
federation or national union to enhance the locals bargaining power.
68. Eagle Ridge Golf and Country Club v. CA March 18, 2010

Nature: Petitioner Eagle Ridge is a corporation engaged in the business


of maintaining golf courses. It had, at the endof CY 2005, around 112 rank-andfile employees. The instant case is an off-shot of the desire of a number of these
employees to organize themselves as a legitimate labor union and their
employers opposition to their aspiration.

FACTS: On December 6, 2005, at least 20% of Eagle Ridges rank-and-file


employeesthe percentage threshold required under Article 234(c) of the Labor
Code for union registrationhad a meeting where they organized themselves
into an independent labor union, named "Eagle Ridge Employees Union" (EREU
or Union), elected a set of officers, and ratified their constitution and by-laws.
EREU formally applied for registration before DOLE Regional Office IV which
granted the application and issued EREU Registration Certificate. EREU then filed
a petition for certification election in Eagle Ridge Golf & Country Club. Eagle
Ridge opposed this petition and filed a petition for the cancellation of EREUs Reg
Certificate. It ascribed misrepresentation, false statement, or fraud to EREU in
connection with the adoption of its constitution and by-laws, the numerical
composition of the Union, and the election of its officers.
EAGLE RIDGEs SPECIFIC ALLEGATIONS:

3E Andaya Ching Espiritu Hefti Galvez Gammad Lainez Lui Madamba Nagera Narvasa
Ong Palangdao Rosales Sanchez Santos Satrain Tabo (2014-2015)

1.
2.
3.
4.

Labor Relations Case Digest - Atty. Joyrich Golangco

EREU declared in its application for registration having 30 members,


when the minutes of its December 6, 2005 organizational meeting
showed it only had 26 members.
Discrepancy between the certification issued by the Union secretary and
president that 25 members actually ratified the constitution and by-laws
on December 6, 2005 and the fact that 26 members affixed
their signatures on the documents, making one signature a forgery.
It also contends that 5 employees who attended the organizational
meeting had manifested the desire to withdraw from the union.
The five executed individual affidavits or Sinumpaang Salaysay on
February 15, 2006, attesting that they arrived late at said meeting which
they claimed to be drinking spree; that they did not know that the
documents they signed on that occasion pertained to the organization of
a union; and that they now wanted to be excluded from the Union. The
withdrawal of the five, Eagle Ridge maintained, effectively reduced the
union membership to 20 or 21, either of which is below the mandatory
minimum 20% membership requirement under Art. 234(c) of the Labor
Code. Reckoned from 112rank-and-file employees of Eagle Ridge, the
required number would be 22 or 23 employees.

EREU/UNIONs COUNTERARGUMENTS:
1. Alleged discrepancies are not real for before filing of its application on
December 19, 2005, 4 additional employees joined the union on
December 8, 2005, thus raising the union membership to 30 members as
of December 19, 2005;
2. The understatement by one member who ratified the constitution and
by-laws was a typographical error, which does not make it either grave
or malicious warranting the cancellation of the unions registration;
3. The retraction of 5 union members should not be given any credence for
the reasons that: a) the sworn statements of the five retracting union
members sans other affirmative evidence presented hardly qualify as
clear and credible evidence considering the joint affidavits of the other
members attesting to the orderly conduct of the organizational meeting;
b) the retracting members did not deny signing the union documents;
c)it can be presumed that "duress, coercion or valuable consideration"
was brought to bear on the retracting members; and d) once the

required percentage requirement has been reached, the employees


withdrawal from union membership taking place after the filing of the
petition for certification election will not affect the petition. It asserted
the applicability of said ruling as the petition for certification election
was filed on January 10, 2006 or long before February 15, 2006 when
the affidavits of retraction were executed by the five union members,
thus contending that the retractions do not affect nor be deemed
compelling enough to cancel its certificate of registration.

Ruling of DOLE regional director: ruled in favor of Eagle Ridge and granted the
cancellation of the Reg Certificate of EREU.
Ruling of Bureau of Labor Relations (BLR) headed by Officer in charge
(OIC): affirmed DOLE regional directors ruling. On appeal, BLR headed by
Director Chato set aside the BLR OICs decision, declaring that the Eagle Ridge
Employees Union (EREU) shall remain in the roster of legitimate organizations.
she found as without basis allegations of misrepresentation or fraud as ground
for cancellation of EREUs registration.
CA: dismissed Eagle Ridges petition for being deficient.
ISSUES: WON fraud is procured in the registration of the labor union

HELD: No fraud in the application. Eagle Ridge cites the grounds provided under
Art. 239(a) and (c) of the Labor Code for its petition for cancellation of the
EREUs registration. On the other hand, the Union asserts bona fide compliance
with the registration requirements under Art. 234 of the Code, explaining the
seeming discrepancies between the number of employees who participated in
the organizational meeting and the total number of union members at the time it
filed its registration, as well as the typographical error in its certification which
understated by one the number of union members who ratified the unions
constitution and by-laws.
Before their amendment by Republic Act No. 9481, the then governing
Art. 234 (on the requirements of registration of a labor union) and Art. 239 (on
the grounds for cancellation of union registration) of the Labor Code respectively
provided as follows:

3E Andaya Ching Espiritu Hefti Galvez Gammad Lainez Lui Madamba Nagera Narvasa
Ong Palangdao Rosales Sanchez Santos Satrain Tabo (2014-2015)

Labor Relations Case Digest - Atty. Joyrich Golangco

ART. 234. REQUIREMENTS OF REGISTRATION. Any applicant labor


organization, association or group of unions or workers shall acquire legal
personality and shall be entitled to the rights and privileges granted by law to
legitimate labor organizations upon issuance of the certificate of registration
based on the following requirements:
(a) Fifty pesos (P50.00) registration fee;
(b) The names of its officers, their addresses, the principal address of the
labor organization, the minutes of the organizational meetings and
the list of workers who participated in such meetings;
(c) The names of all its members comprising at least twenty percent
(20%) of all the employees in the bargaining unit where it seeks to
operate;
xxxx
(e) Four copies (4) of the constitution and by-laws of the applicant
union, minutes of its adoption or ratification and the list of the
members who participated in it.41
xxxx
ART. 239. GROUNDS FOR CANCELLATION OF UNION REGISTRATION. The
following shall constitute grounds for cancellation of union registration:
(a) Misrepresentation, false statements or fraud in connection with
the adoption or ratification of the constitution and by-laws or
amendments thereto, the minutes of ratification, and the list of
members who took part in the ratification;
xxxx
(c) Misrepresentation, false statements or fraud in connection with
the election of officers, minutes of the election of officers, the list of
voters, or failure to submit these documents together with the list of the
newly elected/appointed officers and their postal addresses within
thirty (30) days from election. (Emphasis supplied.)
A scrutiny of the records fails to show any misrepresentation, false statement, or
fraud committed by EREU to merit cancellation of its registration based on the
following arguments:
1. First. The Union submitted the required documents attesting to the facts of
the organizational meeting, the election of its officers, and the adoption of the

Unions constitution and by-laws. It submitted before the DOLE Regional Office
with its Application for Registration the: (a) the minutes of its organizational
meeting held on December 6, 2005 showing 26 founding members who elected
its union officers by secret ballot; (b) the list of rank-and-file employees of Eagle
Ridge who attended the organizational meeting and the election of officers with
their individual signatures; (c) the list of rank-and-file employees45 who ratified
the unions constitution and by-laws showing the very same list as those who
attended the organizational meeting and the election of officers with their
individual signatures except the addition of four employees without their
signatures, i.e., Cherry Labajo, Grace Pollo, Annalyn Poniente and Rowel Dolendo;
(d) the unions constitution and by-laws as approved on December 6, 2005; (e)
the list of officers and their addresses; (f) the list of union members48 showing a
total of 30 members; and (g) the Sworn Statement49 of the unions elected
president and secretary. All the foregoing documents except the sworn
statement of the president and the secretary were accompanied by
Certifications by the union secretary duly attested to by the union president.
2. Second. The members of the EREU totaled 30 employees when it applied for
registration, thereby complying with the mandatory minimum 20% membership
requirement under Art. 234(c).
3. Third. The Union has sufficiently explained the discrepancy between the
number of those who attended the organizational meeting showing 26
employees and the list of union members showing 30. The difference is due to
the additional four members admitted two days after the organizational meeting
as attested to by their duly accomplished Union Membership forms.
Consequently, the total number of union members, as of December 8, 2005, was
30, which was truthfully indicated in its application for registration on December
19, 2005. As aptly found by the BLR Director, the Union already had 30 members
when it applied for registration, for the admission of new members is neither
prohibited by law nor was it concealed in its application for registration.
4. Fourth. In its futile attempt to clutch at straws, Eagle Ridge assails the
inclusion of the additional four members allegedly for not complying with what it
termed as "the sine qua non requirements" for union member applications under
the Unions constitution and by-laws, specifically Sec. 2 of Art. IV. We are not
persuaded. Any seeming infirmity in the application and admission of union
membership, most especially in cases of independent labor unions, must be
viewed in favor of valid membership.

3E Andaya Ching Espiritu Hefti Galvez Gammad Lainez Lui Madamba Nagera Narvasa
Ong Palangdao Rosales Sanchez Santos Satrain Tabo (2014-2015)

Labor Relations Case Digest - Atty. Joyrich Golangco

The right of employees to self-organization and membership in a union


must not be trammeled by undue difficulties. In this case, when the Union said
that the four employee-applicants had been admitted as union members, it is
enough to establish the fact of admission of the four that they had duly signified
such desire by accomplishing the membership form.
5. Fifth. The difference between the number of 26 members, who ratified the
Unions constitution and by-laws, and the 25 members shown in the certification
of the Union secretary as having ratified it, is, as shown by the factual
antecedents, a typographical error. It was an insignificant mistake committed
without malice or prevarication. The list of those who attended the
organizational meeting shows 26 members, as evidenced by the signatures
beside their handwritten names. Thus, the certifications understatement by one
member, while not factual, was clearly an error, but neither a misleading one nor
a misrepresentation of what had actually happened.
6. Sixth. In the more meaty issue of the affidavits of retraction executed by six
union members, we hold that the probative value of these affidavits cannot
overcome those of the supporting affidavits of 12 union members and their
counsel as to the proceedings and the conduct of the organizational meeting on
December 6, 2005. The DOLE Regional Director and the BLR OIC Director
obviously erred in giving credence to the affidavits of retraction, but not
according the same treatment to the supporting affidavits.
It is settled that affidavits partake the nature of hearsay evidence, since
they are not generally prepared by the affiant but by another who uses his own
language in writing the affiants statement, which may thus be either omitted or
misunderstood by the one writing them.
For their non-presentation and consonant to the above-quoted rule, the six
affidavits of retraction are inadmissible as evidence against the Union in the
instant case.
7. Seventh. The fact that six union members, indeed, expressed the desire to
withdraw their membership through their affidavits of retraction will not cause
the cancellation of registration on the ground of violation of Art. 234(c) of the
Labor Code requiring the mandatory minimum 20% membership of rank-andfile employees in the employees union. The six retracting union members clearly
severed and withdrew their union membership. The query is whether such
separation from the Union can detrimentally affect the registration of the Union.
We answer in the negative.

Twenty percent (20%) of 112 rank-and-file employees in Eagle Ridge


would require a union membership of at least 22 employees (112 x 205 = 22.4).
When the EREU filed its application for registration on December 19, 2005, there
were clearly 30 union members. Thus, when the certificate of registration was
granted, there is no dispute that the Union complied with the mandatory 20%
membership requirement.
With the withdrawal of six union members, there is still compliance with
the mandatory membership requirement under Art. 234(c), for the remaining 24
union members constitute more than the 20% membership requirement of 22
employees.
Evidently, as the Union persuasively argues, the withdrawal of six
member-employees from the Union will affect neither the Unions registration
nor its petition for certification election, as their affidavits of retraction were
executed after the Unions petition for certification election had been filed. The
initial five affidavits of retraction were executed on February 15, 2006; the sixth,
on March 15, 2006. Indisputably, all six were executed way after the filing of the
petition for certification election on January 10, 2006.
The Court ends this disposition by reproducing the following apt excepts
from its holding in S.S. Ventures International, Inc. v. S.S. Ventures Labor Union
(SSVLU) on the effect of the withdrawal from union membership right before or
after the filing of a petition for certification election: We are not persuaded. As
aptly noted by both the BLR and CA, these mostly undated written statements
submitted by Ventures on March 20, 2001, or seven months after it filed its
petition for cancellation of registration, partake of the nature of withdrawal of
union membership executed after the Unions filing of a petition for certification
election on March 21, 2000. We have in precedent cases said that the
employees withdrawal from a labor union made before the filing of the
petition for certification election is presumed voluntary, while withdrawal
after the filing of such petition is considered to be involuntary and does not
affect the same. Now then, if awithdrawal from union membership done
after a petition for certification election has been filed does not vitiate such
petition, is it not but logical to assume that such withdrawal cannot work to
nullify the registration of the union? Upon this light, the Court is inclined to
agree with the CA that the BLR did not abuse its discretion nor gravely err when
it concluded that the affidavits of retraction of the 82 members had no
evidentiary weight.(Emphasis supplied.)

3E Andaya Ching Espiritu Hefti Galvez Gammad Lainez Lui Madamba Nagera Narvasa
Ong Palangdao Rosales Sanchez Santos Satrain Tabo (2014-2015)

Labor Relations Case Digest - Atty. Joyrich Golangco


69. TAGAYTAY HIGHLANDS INTERNATIONAL GOLF CLUB INC v. TAGAYTAY
HIGHLANDS EMPLOYEES UNIONPGTWO
DOCTRINE: The Supreme Court ruled that the effect of issuance of certificate of
registration to a union is that it becomes legitimate and its legal personality can
only be attacked through a petition for cancellation of registration and not thru
intervention in a certification election petition.

FACTS: On October 16, 1997, the Tagaytay Highlands Employees Union (THEU)
Philippine Transport and General Workers Organization (PTGWO), Local Chapter
No. 776, a legitimate labor organization said to represent majority of the rankand-file employees of Tagaytay Highlands International Golf Club Incorporated
(THIGCI), filed a petition for certification election before the DOLE MediationArbitration Unit, Regional Branch No. IV. THIGCI, in its Comment, opposed
THEUs petition for certification election on the ground that the list of union
members submitted by it was defective and fatally flawed as it included the
names and signatures of supervisors, resigned, terminated and absent without
leave (AWOL) employees, as well as employees of The Country Club, Inc., a
corporation distinct and separate from THIGCI; and that out of the 192
signatories to the petition, only 71 were actual rank-and-file employees of
THIGCI. THIGCI thus submitted a list of the names of its 71 actual rank-and-file
employees to the petition for certification election. And it therein incorporated a
tabulation showing the number of signatories to said petition whose
membership in the union was being questioned as disqualified and the reasons
for disqualification.
THEU asserted that it complied with all the requirements for valid affiliation and
inclusion in the roster of legitimate labor organizations pursuant to DOLE
Department Order No. 9, series of 1997, on account of which it was duly granted
a Certification of Affiliation by DOLE on October 10, 1997; and that Section 5,
Rule V of said Department Order provides that the legitimacy of its registration
cannot be subject to collateral attack, and for as long as there is no final order of
cancellation, it continues to enjoy the rights accorded to a legitimate
organization. Therefore, the Med-Arbiter should, pursuant to Article 257 of the
Labor Code and Section 11, Rule XI of DOLE Department Order No. 09,

automatically order the conduct of a certification election. On January 28, 1998,


DOLE Med-Arbiter Anastacio Bactin ordered the holding of a certification
election.

THIGCI appealed to the Office of the DOLE Secretary which, by Resolution of June
4, 1998, set aside the said Med-Arbiters Order and accordingly dismissed the
petition for certification election on the ground that there is a "clear absence of
community or mutuality of interests," it finding that THEU sought to represent
two separate bargaining units (supervisory employees and rank-and-file
employees) as well as employees of two separate and distinct corporate entities.
Upon Motion for Reconsideration by THEU, DOLE Undersecretary Rosalinda
Dimalipis-Baldoz, by authority of the DOLE Secretary, issued DOLE Resolution of
November 12, 1998 setting aside the June 4, 1998 Resolution dismissing the
petition for certification election. She held that since THEU is a local chapter, the
twenty percent (20%) membership requirement is not necessary for it to acquire
legitimate status; hence, "the alleged retraction and withdrawal of support by 45
of the 70 remaining rank-and-file members . . . cannot negate the legitimacy it
has already acquired before the petition". THIGCIs Motion for Reconsideration
was denied by the DOLE Undersecretary hence it filed a petition for certiorari
with the CA.
The CA denied THIGCIs Petition for Certiorari and affirmed the DOLE Resolution
dated November 12, 1998. It held that while a petition for certification election is
an exception to the innocent bystander rule, hence, the employer may pray for
the dismissal of such petition on the basis of lack of mutuality of interests of the
members of the union as well as lack of employer-employee relationship and
petitioner failed to adduce substantial evidence to support its allegations.
ISSUES: Whether the unions legal personality can be subject to collateral attack
after a certificate of registration is issued

HELD: NO, Petition is DENIED, and the records of the case are remanded to the
office of origin. While Article 245 expressly prohibits supervisory employees
from joining a rank-and-file union, it does not provide what would be the effect if
a rank-and-file union counts supervisory employees among its members, or viceversa. Citing Toyota which held that "a labor organization composed of both

3E Andaya Ching Espiritu Hefti Galvez Gammad Lainez Lui Madamba Nagera Narvasa
Ong Palangdao Rosales Sanchez Santos Satrain Tabo (2014-2015)

Labor Relations Case Digest - Atty. Joyrich Golangco

rank-and-file and supervisory employees is no labor organization at all," and the


subsequent case of Progressive Development Corp. Pizza Hut v. Ledesma which
held that:
"The Labor Code requires that in organized and unorganized
establishments, a petition for certification election must be filed by a legitimate
labor organization. The acquisition of rights by any union or labor organization,
particularly the right to file a petition for certification election, first and
foremost, depends on whether or not the labor organization has attained the
status of a legitimate labor organization.
In the case before us, the Med-Arbiter summarily disregarded the petitioners
prayer that the former look into the legitimacy of the respondent Union by a
sweeping declaration that the union was in the possession of a charter certificate
so that for all intents and purposes, Sumasaklaw sa Manggagawa sa Pizza Hut
(was) a legitimate organization," (Underscoring and emphasis supplied),
We also do not agree with the ruling of the respondent Secretary of Labor that
the infirmity in the membership of the respondent union can be remedied in
"the pre-election conference thru the exclusion-inclusion proceedings wherein
those employees who are occupying rank-and-file positions will be excluded
from the list of eligible voters."

After a certificate of registration is issued to a union, its legal personality cannot


be subject to collateral attack. It may be questioned only in an independent
petition for cancellation in accordance with Section 5 of Rule V, Book IV of the
"Rules to Implement the Labor Code" (Implementing Rules) which section reads:

Sec. 5. Effect of registration. The labor organization or workers


association shall be deemed registered and vested with legal personality on the
date of issuance of its certificate of registration. Such legal personality cannot
thereafter be subject to collateral attack, but may be questioned only in an
independent petition for cancellation in accordance with these Rules.
(Emphasis supplied)

The inclusion in a union of disqualified employees is not among the grounds for
cancellation, unless such inclusion is due to misrepresentation, false statement

or fraud under the circumstances enumerated in Sections (a) and (c) of


Article 239 of above quoted Article 239 of the Labor Code. THEU, having
been validly issued a certificate of registration, should be considered to have
already acquired juridical personality which may not be assailed collaterally. As
for petitioners allegation that some of the signatures in the petition for
certification election were obtained through fraud, false statement and
misrepresentation, the proper procedure is, as reflected above, for it to file a
petition for cancellation of the certificate of registration, and not to intervene in a
petition for certification election.
Regarding the alleged withdrawal of union members from participating in the
certification election, this Courts following ruling is instructive:
"[T]he best forum for determining whether there were indeed
retractions from some of the laborers is in the certification election itself wherein
the workers can freely express their choice in a secret ballot. Suffice it to say that
the will of the rank-and file employees should in every possible instance be
determined by secret ballot rather than by administrative or quasi-judicial
inquiry. Such representation and certification election cases are not to be taken
as contentious litigations for suits but as mere investigations of a non-adversary,
fact-finding character as to which of the competing unions represents the
genuine choice of the workers to be their sole and exclusive collective bargaining
representative with their employer."
As for the lack of mutuality of interest argument of petitioner, it, at all events,
does not lie given, as found by the court a quo, its failure to present substantial
evidence that the assailed employees are actually occupying supervisory
positions.
While petitioner submitted a list of its employees with their corresponding job
titles and ranks, there is nothing mentioned about the supervisors respective
duties, powers and prerogatives that would show that they can effectively
recommend managerial actions which require the use of independent judgment.
As this Court put it in Pepsi-Cola Products Philippines, Inc. v. Secretary of Labor,
Designation should be reconciled with the actual job description of subject
employees x x x The mere fact that an employee is designated manager does not
necessarily make him one.

3E Andaya Ching Espiritu Hefti Galvez Gammad Lainez Lui Madamba Nagera Narvasa
Ong Palangdao Rosales Sanchez Santos Satrain Tabo (2014-2015)

Labor Relations Case Digest - Atty. Joyrich Golangco

Otherwise, there would be an absurd situation where one can be given the title
just to be deprived of the right to be a member of a union. In the case of National
Steel Corporation vs. Laguesma (G. R. No. 103743, January 29, 1996), it was
stressed that What is essential is the nature of the employees function and
not the nomenclature or title given to the job which determines whether the
employee has rank-and-file or managerial status or whether he is a supervisory
employee.
70. SS Ventures International vs SS Ventures Labor Union

FACTS: Petitioner Ventures is in the business of manufacturing sports shoes.


Respondent on the is a labor organization registered with the DOLE. Union filed
with DOLE-Region III a petition for certification election in behalf of the rankand-file employees of Ventures. 542 signatures, 82 of which belong to terminated
Ventures employees, appeared on the basic documents supporting the petition.
Ventures filed a Petition to cancel the Unions certificate of registration
arguing that the
1.

2.

The Union deliberately and maliciously included the names of more or


less 82 former employees no longer connected with Ventures in its list of
members who attended the organizational meeting and in the
adoption/ratification of its constitution and by-laws held on January 9,
2000; and the Union forged the signatures of these 82 former employees
to make it appear they took part in the organizational meeting and
adoption and ratification of the constitution;
Unions application for registration was not supported by at least 20% of
the rank-and-file employees of Ventures, or 418 of the total 2,197employee complement. Since more or less 82 of the 500 signatures were
forged or invalid, then the remaining valid signatures would only be 418,
which is very much short of the 439 minimum (2197 total employees x
20% = 439.4) required by the Labor Code.

Regional Director rendered judgment cancelling the certificate of


registration of the Union. Union appealed to the BLR which granted such appeal.
Ventures filed a petition for certiorari with the CA which was denied.

ISSUES: Whether or not respondent union committed fraud and


misrepresentation in connection with the adoption or ratification of the unions
constitution as to warrant cancellation of its registration?

HELD: NO. Once registered with the DOLE, a union is considered a legitimate
labor organization endowed with the right and privileges granted by law to such
organization. While a certificate of registration confers a union with legitimacy
with the concomitant right to participate in or ask for certification election in a
bargaining unit, the registration may be cancelled or the union may be
decertified as the bargaining unit, in which case the union is divested of the
status of a legitimate labor organization. Among the grounds for cancellation is
the commission of any of the acts enumerated in Art. 239(a) of the Labor Code,
such as fraud and misrepresentation in connection with the adoption or
ratification of the unions constitution and like documents. The Court, has in
previous cases, said that to decertify a union, it is not enough to show that the
union includes ineligible employees in its membership. It must also be shown
that there was misrepresentation, false statement, or fraud in connection with
the application for registration and the supporting documents, such as the
adoption or ratification of the constitution and by-laws or amendments thereto
and the minutes of ratification of the constitution or by-laws, among other
documents.
The Court need not delve into the question of whether these 82
dismissed individuals were still Union members qualified to vote and affix their
signature on its application for registration and supporting documents. The
relevancy of the 82 individuals active participation in the Unions organizational
meeting and the signing ceremonies thereafter comes in only for purposes of
determining whether or not the Union, even without the 82, would still meet
what Art. 234(c) of the Labor Code requires to be submitted, to wit:
Art. 234. Requirements of Registration.Any applicant labor organization x x x
shall acquire legal personality and shall be entitled to the rights and privileges
granted by law to legitimate labor organizations upon issuance of the certificate of
registration based on the following requirements:
xxxx

3E Andaya Ching Espiritu Hefti Galvez Gammad Lainez Lui Madamba Nagera Narvasa
Ong Palangdao Rosales Sanchez Santos Satrain Tabo (2014-2015)

Labor Relations Case Digest - Atty. Joyrich Golangco

(c) The names of all its members comprising at least twenty percent (20%) of all
the employees in the bargaining unit where it seeks to operate.

In its union records on file with this Bureau, respondent union submitted the
names of [542] members. This number easily complied with the 20%
requirement, be it 1,928 or 2,202 employees in the establishment. Even
subtracting the 82 employees from 542 leaves 460 union members, still within
440 or 20% of the maximum total of 2,202 rank-and-file employees.

Whatever misgivings the petitioner may have with regard to the 82 dismissed
employees is better addressed in the inclusion-exclusion proceedings during a
pre-election conference. The issue surrounding the involvement of the 82
employees is a matter of membership or voter eligibility. It is not a ground
to cancel union registration.
71. THE HERITAGE HOTEL MANILA vs. NATIONAL UNION OF WORKERS IN
THE HOTEL, RESTAURANT AND ALLIED INDUSTRIES-HERITAGE HOTEL
MANILA SUPERVISORS CHAPTER (NUWHRAIN-HHMSC)
Digesters note: The case has 2 issues: the first is on the Jurisdiction of the labor
secretary to review upon inhibition of the BLR director who has jurisdiction
conferred to him by law. The second one is about cancellation of union and is the
more relevant one to LabRel. The specific provisions covered by the case in
relation to the case list are Articles 238-239 (now 244-245) on Cancellation of
Registration of Unions both before and after the amendment by RA 9481; and
then there is also Article 248-A after the amendment which basically made the
alleged ground for cancellation no longer such.
FACTS: Respondent filed with the Department of Labor and EmploymentNational Capital Region (DOLE-NCR) a petition for certification election. The
Med-Arbiter granted the petition and ordered the holding of a certification
election. On appeal, the DOLE Secretary affirmed the Med-Arbiters order and
remanded the case to the Med-Arbiter for the holding of a preelection conferenc.
Petitioner filed a motion for reconsideration, but it was denied.

The preelection conference was not held as initially scheduled; it was held a
year later, or on February 20, 1998. Petitioner moved to archive or to dismiss the

petition due to alleged repeated non-appearance of respondent. The latter


agreed to suspend proceedings until further notice. The preelection conference
resumed on January 29, 2000.

Subsequently, petitioner discovered that respondent had failed to


submit to the Bureau of Labor Relations (BLR) its annual financial report
for several years and the list of its members since it filed its registration
papers in 1995. Consequently, petitioner filed a Petition for Cancellation of
Registration of respondent, on the ground of the non-submission of the said
documents. Petitioner prayed that respondents Certificate of Creation of
Local/Chapter be cancelled and its name be deleted from the list of legitimate
labor organizations. It further requested the suspension of the certification
election proceedings.
On June 1, 2000, petitioner reiterated its request by filing a Motion to
Dismiss or Suspend the [Certification Election] Proceedings. Nevertheless,
the certification election pushed through on June 23, 2000. Respondent emerged
as the winner.

On June 28, 2000, petitioner filed a Protest with Motion to Defer


Certification of Election Results and Winner, stating that the certification
election held on June 23, 2000 was an exercise in futility because, once
respondents registration is cancelled, it would no longer be entitled to be
certified as the exclusive bargaining agent of the supervisory employees.

Meanwhile, respondent filed its Answer to the petition for the


cancellation of its registration. It averred that the petition was filed primarily to
delay the conduct of the certification election, the respondents certification as
the exclusive bargaining representative of the supervisory employees, and the
commencement of bargaining negotiations.

Respondent prayed for the dismissal of the petition for cancellation for
the following reasons: (a) petitioner is estopped from questioning respondents
status as a legitimate labor organization as it had already recognized respondent
as such during the preelection conferences; (b) petitioner is not the party-ininterest, as the union members are the ones who would be disadvantaged by the
non-submission of financial reports; (c) it has already complied with the
reportorial requirements, having submitted its financial statements for

3E Andaya Ching Espiritu Hefti Galvez Gammad Lainez Lui Madamba Nagera Narvasa
Ong Palangdao Rosales Sanchez Santos Satrain Tabo (2014-2015)

Labor Relations Case Digest - Atty. Joyrich Golangco

1996, 1997, 1998, and 1999, its updated list of officers, and its list of
members for the years 1995, 1996, 1997, 1998, and 1999; (d) the petition is
already moot and academic, considering that the certification election had
already been held, and the members had manifested their will to be represented
by respondent.

association and right of workers to self-organization outweighed respondents


noncompliance with the statutory requirements to maintain its status as a
legitimate labor organization.

Petitioner subsequently appealed the said Order to the DOLE Secretary.


The appeal was later dismissed by DOLE Secretary Patricia A. Sto. Tomas (DOLE
Secretary Sto. Tomas) in the Resolution of August 21, 2002. Petitioner moved for
reconsideration, but the motion was also denied.

Petitioner filed a petition for certiorari with the CA, raising the issue of
whether the DOLE Secretary acted with grave abuse of discretion in taking
cognizance of the appeal and affirming the dismissal of its petition for
cancellation of respondents registration.

The Med-Arbiter held that the pendency of a petition for cancellation of


registration is not a bar to the holding of a certification election. Thus, in an
Order dated January 26, 2001, the Med-Arbiter dismissed petitioners
protest, and certified respondent as the sole and exclusive bargaining
agent of all supervisory employees.

In the meantime, Regional Director Alex E. Maraan (Regional


Director Maraan) of DOLE-NCR finally resolved the petition for cancellation
of registration. While finding that respondent had indeed failed to file financial
reports and the list of its members for several years, he, nonetheless, denied the
petition, ratiocinating that freedom of association and the employees right to
self-organization are more substantive considerations. He took into account the
fact that respondent won the certification election and that it had already been
certified as the exclusive bargaining agent of the supervisory employees. In view
of the foregoing, Regional Director Maraanwhile emphasizing that the
non-compliance with the law is not viewed with favorconsidered the
belated submission of the annual financial reports and the list of members
as sufficient compliance thereof and considered them as having been
submitted on time.
Aggrieved, petitioner appealed the decision to the BLR. BLR Director
Hans Leo Cacdac inhibited himself from the case because he had been a
former counsel of respondent.
In view of Director Cacdacs inhibition, DOLE Secretary Sto. Tomas
took cognizance of the appeal. In a resolution dated February 21, 2003, she
dismissed the appeal, holding that the constitutionally guaranteed freedom of

Petitioner filed a motion for reconsideration, but the motion was


likewise denied in a resolution dated May 30, 2003. DOLE Secretary Sto. Tomas
admitted that it was the BLR which had jurisdiction over the appeal, but she
pointed out that the BLR Director had voluntarily inhibited himself from the case
because he used to appear as counsel for respondent. In order to maintain the
integrity of the decision and of the BLR, she therefore accepted the motion to
inhibit and took cognizance of the appeal.

The CA denied the petition. The CA opined that the DOLE Secretary may
legally assume jurisdiction over an appeal from the decision of the Regional
Director in the event that the Director of the BLR inhibits himself from the case.
According to the CA, in the absence of the BLR Director, there is no person
more competent to resolve the appeal than the DOLE Secretary. The CA
brushed aside the allegation of bias and partiality on the part of the DOLE
Secretary, considering that such allegation was not supported by any evidence.

The CA also found that the DOLE Secretary did not commit grave
abuse of discretion when she affirmed the dismissal of the petition for
cancellation of respondents registration as a labor organization. Echoing
the DOLE Secretary, the CA held that the requirements of registration of
labor organizations are an exercise of the overriding police power of the
State, designed for the protection of workers against potential abuse by the
union that recruits them. These requirements, the CA opined, should not be
exploited to work against the workers constitutionally protected right to selforganization.
ISSUES:

3E Andaya Ching Espiritu Hefti Galvez Gammad Lainez Lui Madamba Nagera Narvasa
Ong Palangdao Rosales Sanchez Santos Satrain Tabo (2014-2015)

Labor Relations Case Digest - Atty. Joyrich Golangco

1.) Whether or not the Labor secretary had jurisdiction to review the
decision of the Regional Director in a petition for cancellation when such
jurisdiction is conferred by law to the BLR (Bureau of Labor Relations).
2.) Whether the CA gravely erred in affirming the dismissal of the
Cancellation petition despite the mandatory and unequivocal provisions
of the Labor Code (Articles 238-239) and its Implementing rules.

HELD:

1.) Yes, it does. Jurisdiction to review the decision of the Regional Director
lies with the BLR. But as pointed out by the CA, the present case
involves a peculiar circumstance that was not present or covered by the
ruling in Abbott. In this case, the BLR Director inhibited himself from
the case because he was a former counsel of respondent. Who, then,
shall resolve the case in his place? Jurisdiction remained with the BLR
despite the BLR Directors inhibition.
When the DOLE Secretary resolved the appeal, she merely
stepped into the shoes of the BLR Director and performed a
function that the latter could not himself perform. She did so
pursuant to her power of supervision and control over the BLR. The
DOLE Secretary, as the person exercising the power of supervision and
control over the BLR, has the authority to directly exercise the quasijudicial function entrusted by law to the BLR Director.
It is true that the power of control and supervision does not give
the Department Secretary unbridled authority to take over the functions
of his or her subordinate. Such authority is subject to certain guidelines
which are stated in Book IV, Chapter 8, Section 39(1)(a) of the
Administrative Code of 1987. However, in the present case, the DOLE
Secretarys act of taking over the function of the BLR Director was
warranted and necessitated by the latters inhibition from the case
and the objective to maintain the integrity of the decision, as well
as the Bureau itself.
Petitioner insists that the BLR Directors subordinates should
have resolved the appeal, citing the provision under the Administrative
Code of 1987 which states, in case of the absence or disability of the
head of a bureau or office, his duties shall be performed by the assistant
head. The provision clearly does not apply considering that the BLR

Director was neither absent nor suffering from any disability; he


remained as head of the BLR. Thus, to dispel any suspicion of bias, the
DOLE Secretary opted to resolve the appeal herself.

2.) Articles 238 and 239 of the Labor Code read (these provisions are
before the amendment):
ART. 238. CANCELLATION OF REGISTRATION; APPEAL
The certificate of registration of any
legitimate labor organization, whether
national or local, shall be canceled by the
Bureau if it has reason to believe, after due
hearing, that the said labor organization no
longer meets one or more of the
requirements herein prescribed.
ART. 239. GROUNDS FOR CANCELLATION OF
UNION REGISTRATION.
The
following shall
constitute
grounds for cancellation of union registration:
xxxx

(d) Failure to submit the annual


financial report to the Bureau within thirty
(30) days after the closing of every fiscal year
and misrepresentation, false entries or fraud
in the preparation of the financial report itself;
xxxx

(i) Failure to submit list of individual


members to the Bureau once a year or
whenever required by the Bureau.

3E Andaya Ching Espiritu Hefti Galvez Gammad Lainez Lui Madamba Nagera Narvasa
Ong Palangdao Rosales Sanchez Santos Satrain Tabo (2014-2015)

Labor Relations Case Digest - Atty. Joyrich Golangco

Articles 238-239 give the Regional Director ample


discretion in dealing with a petition for cancellation of a unions
registration, particularly, determining whether the union still
meets the requirements prescribed by law. It is sufficient to give the
Regional Director license to treat the late filing of required documents as
sufficient compliance with the requirements of the law. After all, the law
requires the labor organization to submit the annual financial report and
list of members in order to verify if it is still viable and financially
sustainable as an organization so as to protect the employer and
employees from fraudulent or fly-by-night unions. With the submission
of the required documents by respondent, the purpose of the law has
been achieved, though belatedly.

We cannot ascribe abuse of discretion to the Regional Director


and the DOLE Secretary in denying the petition for cancellation of
respondents registration. The union members and, in fact, all the
employees belonging to the appropriate bargaining unit should not
be deprived of a bargaining agent, merely because of the negligence
of the union officers who were responsible for the submission of
the documents to the BLR.
It is worth mentioning that the Labor Codes provisions on
cancellation of union registration and on reportorial requirements have
been recently amended by Republic Act (R.A.) No. 9481, An Act
Strengthening the Workers Constitutional Right to Self-Organization,
Amending for the Purpose Presidential Decree No. 442, As Amended,
Otherwise Known as the Labor Code of the Philippines, which lapsed into
law on May 25, 2007 and became effective on June 14, 2007.
Thus, R.A. No. 9481 amended Article 239 (now 245) to read:
[refer to current codal provisions]
R.A. No. 9481 also inserted in the Labor Code Article 242-A
(now 248-A), which provides:
ART. 242-A. Reportorial Requirements.The
following are documents required to be submitted to
the Bureau by the legitimate labor organization
concerned:
xxxxxxx

Failure to comply with the above


requirements shall not be a ground for cancellation
of union registration but shall subject the erring
officers or members to suspension, expulsion from
membership, or any appropriate penalty.

ILO Convention No. 87, which we have ratified in 1953,


provides that workers and employers organizations shall not be liable
to be dissolved or suspended by administrative authority. The ILO has
expressed the opinion that the cancellation of union registration by the
registrar of labor unions, which in our case is the BLR, is tantamount to
dissolution of the organization by administrative authority when such
measure would give rise to the loss of legal personality of the union or
loss of advantages necessary for it to carry out its activities, which is
true in our jurisdiction. Although the ILO has allowed such measure to
be taken, provided that judicial safeguards are in place, i.e., the right to
appeal to a judicial body, it has nonetheless reminded its members that
dissolution of a union, and cancellation of registration for that matter,
involve serious consequences for occupational representation. It has,
therefore, deemed it preferable if such actions were to be taken only as a
last resort and after exhausting other possibilities with less serious
effects on the organization.

72. Republic Of The Philippines vs. Kawashima Textile Manufacturing,


Phils., Inc.

FACTS: On January 24, 2000, KFWU filed with DOLE Regional Office No. IV, a Petition for
Certification Election to be conducted in the bargaining unit composed of 145 rank-and-file
employees of respondent. Attached to its petition are a Certificate of Creation of
Local/Chapter issued on January 19, 2000 by DOLE Regional Office No. IV, stating that it
[KFWU] submitted to said office a Charter Certificate issued to it by the national federation
Phil. Transport & General Workers Organization (PTGWO), and a Report of Creation of
Local/Chapter.
Respondent filed a Motion to Dismiss the petition on the ground that KFWU did not
acquire any legal personality because its membership of mixed rank-and-file and
supervisory employees violated Article 245 of the Labor Code, and its failure to submit its

3E Andaya Ching Espiritu Hefti Galvez Gammad Lainez Lui Madamba Nagera Narvasa
Ong Palangdao Rosales Sanchez Santos Satrain Tabo (2014-2015)

Labor Relations Case Digest - Atty. Joyrich Golangco

books of account contravened the ruling of the Court in Progressive Development


Corporation v. Secretary, Department of Labor and Employment.
Med-Arbiter: In an Order dated May 17, 2000, Med-Arbiter Bactin found KFWUs legal
personality defective and dismissed its petition for certification election.

We scrutinize the facts and evidences presented by the parties and arrived at a decision
that at least two (2) members of [KFWU], namely: Dany I. Fernandez and Jesus R. Quinto, Jr.
are supervisory employees, having a number of personnel under them. Being supervisory
employees, they are prohibited under Article 245 of the Labor Code, as amended, to join the
union of the rank and file employees. Dany I. Fernandez and Jesus R. Quinto, Jr., Chief
Engineers of the Maintenance and Manufacturing Department, respectively, act as foremen
to the line engineers, mechanics and other non-skilled workers and responsible [for] the
preparation and organization of maintenance shop fabrication and schedules, inventory
and control of materials and supplies and tasked to implement training plans on line
engineers and evaluate the performance of their subordinates. The above-stated actual
functions of Dany I. Fernandez and Jesus R. Quinto, Jr. are clear manifestation that they are
supervisory employees.
Since petitioners members are mixture of rank and file and supervisory employees,
petitioner union, at this point [in] time, has not attained the status of a legitimate
labor organization. Petitioner should first exclude the supervisory employees from
it membership before it can attain the status of a legitimate labor organization.
Judgment is supported by the decision of the Supreme Court in the Toyota Case.

Furthermore, the commingling of rank and file and supervisory employees in one (1)
bargaining unit cannot be cured in the exclusion-inclusion proceedings [at] the pre-election
conference. The above ruling is supported by the Decision of the Supreme Court in Dunlop
Slazenger (Phils.), Inc. vs. Honorable Secretary of Labor and Employment, et al., G.R. No.
131248 dated December 11, 1998
On the basis of the aforecited decision, respondent filed with DOLE Regional Office
No. IV a Petition for Cancellation of Charter/Union Registration of KFWU, the final outcome
of which, unfortunately, cannot be ascertained from the records.
Meanwhile, KFWU appealed] to the DOLE which issued a Decision on August 18, 2000.

DOLE Regional Office No. IV:


The DOLE held that Med-Arbiter Bactin's reliance on
the decisions of the Court in Toyota Motor Philippines Corporation v. Toyota Motor
Philippines Corporation Labor Union and Dunlop Slazenger, Inc. v. Secretary of Labor and
Employment was misplaced, for while Article 245 declares supervisory employees ineligible
for membership in a labor organization for rank-and-file employees, the provision did not
state the effect of such prohibited membership on the legitimacy of the labor organization
and its right to file for certification election. Neither was such mixed membership a ground
for cancellation of its registration. Section 11, Paragraph II, Rule XI of Department Order No.
9 provides for the dismissal of a petition for certification election based on lack of legal
personality of a labor organization only on the following grounds: (1) [KFWU] is not listed
by the Regional Office or the Bureau of Labor Relations in its registry of legitimate labor
organizations; or (2) [KFWU's] legal personality has been revoked or canceled with
finality. The DOLE noted that neither ground existed; on the contrary, KFWU's legal
personality was well-established, for it held a certificate of creation and had been listed in
the registry of legitimate labor organizations.
As to the failure of KFWU to file its books of account, the DOLE held that such omission
was not a ground for revocation of union registration or dismissal of petition for
certification election, for under Section 1, Rule VI of Department Order No. 9, a local or
chapter like KFWU was no longer required to file its books of account.
Court of Appeals: Respondent filed a Motion for Reconsideration but the DOLE denied
the same in its September 28, 2000 Resolution.

However, on appeal by respondent, the CA rendered the December 13, 2002 Decision
assailed herein, reversing the August 18, 2000 DOLE Decision, thus:
Since respondent union clearly consists of both rank and file and supervisory
employees, it cannot qualify as a legitimate labor organization imbued with the
requisite personality to file a petition for certification election. This infirmity in
union membership cannot be corrected in the inclusion-exclusion proceedings
during the pre-election conference.
Finally, contrary to the pronouncement of public respondent, the application of the
doctrine enunciated in Toyota Motor Philippines Corporation vs. Toyota Motor Philippines
Corporation Labor Union was not construed in a way that effectively denies the

3E Andaya Ching Espiritu Hefti Galvez Gammad Lainez Lui Madamba Nagera Narvasa
Ong Palangdao Rosales Sanchez Santos Satrain Tabo (2014-2015)

Labor Relations Case Digest - Atty. Joyrich Golangco

fundamental right of respondent union to organize and seek bargaining representation


x x x.
For ignoring jurisprudential precepts on the matter, the Court finds that
the Undersecretary of Labor, acting under the authority of the Secretary
of Labor, acted with grave abuse of discretion amounting to lack or
excess of jurisdiction.

KFWU filed a Motion for Reconsideration but the CA denied it. The Republic of the
Philippines filed the present petition to seek closure on two issues.

ISSUES: 1. Whether a mixed membership of rank and file and supervisory employees in a
union is a ground for the dismissal of a petition for certification election in view of the
amendment brought by D.O. 9 series of 1997, which deleted the phraseology in the old rule
that the appropriate bargaining unit of the rank and file employee shall not include the
supervisory employees and/or security guards
2. Whether the legitimacy of a duly registered labor organization can be
collaterally attacked in a petition for a certification election through a motion to dismiss filed
by an employer such as Kawashima Textile Manufacturing Phils, Inc.

HELD: No. Under Section 12 of RA. No. 9481 employers have no personality to interfere
with or thwart a petition for certification election filed by a legitimate labor organization, to
wit: Sec 12, A new provision, Article 258-A is hereby inserted into the Labor Code to read as
follows: Art 258-A. Employer as Bystander. In all cases, whether the petition for
certification election is filed by an employer or a legitimate labor organization, the
employer shall not be considered a party thereto with a concomitant right to oppose
a petition for certification election. The employers participation in such proceedings
shall be limited to (1) being notified or informed of petitions of such nature; and (2)
submitting the list of employees during the pre-election conference should the Med
Arbiter act favorably on the petition However, RA No. 9481 took effect only on June 14,
2007; hence, it applies only to labor representation cases filed on or after said date. As the
petition for certification election subject matter of the present petition was filed by KFWU
on January 24, 2000, RA no. 9481 cannot apply to it. There may have been curative labor
legislations that were given retrospective effect, but not the aforecited provisions of RA no.
9481, for otherwise, substantive rights and interests already vested would be impaired in
the process. Instead, the law rules in force at the time of the filing by KFWU of the petition

for certification election on January 24, 2000 are RA 6715 amending Book V of Presidential
Decree 442 as amended, and the Rules and regulations Implementing RA no. 6715 as
amended by Department Order no. 9, series of 1997. If there is one constant precept in our
labor laws- be it Commonwealth Act No. 213 (1936), RA no. 875 (1953) P.D. No. 442
(1974), Executive Order (E.O) no. 111 (1986) or RA. No. 6715 (1989) it is that only a
legitimate labor organization may exercise the right to be certified as the exclusive
representative of all the employees in an appropriate collective bargaining unit for
purposes of collective bargaining. What has varied over the years has been the degree of
enforcement of this precept, as reflected in the shifting scope of administrative and judicial
scrutiny of the composition of a labor organization before it is allowed to exercise the right
of representation. One area of contention has been the composition of the membership of a
labor organization, specifically whether there is a mingling of supervisory and rank and file
employees and how such questioned mingling affects its legitimacy. More to the point is Air
Philippines Corporation v. Bureau of Labor Relations, which involved a petition for
cancellation of union registration filed by the employer in 1999 against a rank-and-file labor
organization on the ground of mixed membership: the Court therein reiterated its ruling
in Tagaytay Highlands that the inclusion in a union of disqualified employees is not among
the grounds for cancellation, unless such inclusion is due to misrepresentation, false
statement or fraud under the circumstances enumerated in Sections (a) and (c) of Article
239 of the Labor Code.

All said, while the latest issuance is R.A. No. 9481, the 1997 Amended Omnibus Rules,
as
interpreted
by
the
Court
in TagaytayHighlands, San
Miguel and Air
Philippines, had already set the tone for it. Toyota and Dunlop no longer hold sway in the
present altered state of the law and the rules.
Consequently, the Court reverses the ruling of the CA and reinstates that of the DOLE
granting the petition for certification election of KFWU.

2. No. Now to the second issue of whether an employer like respondent may collaterally
attack the legitimacy of a labor organization by filing a motion to dismiss the latters petition
for certification election. Except when it is requested to bargain collectively, an employer is
a mere bystander to any petition for certification election; such proceeding is nonadversarial and merely investigative, for the purpose thereof is to determine which
organization will represent the employees in their collective bargaining with the
employer. The choice of their representative is the exclusive concern of the employees; the

3E Andaya Ching Espiritu Hefti Galvez Gammad Lainez Lui Madamba Nagera Narvasa
Ong Palangdao Rosales Sanchez Santos Satrain Tabo (2014-2015)

Labor Relations Case Digest - Atty. Joyrich Golangco

employer cannot have any partisan interest therein; it cannot interfere with, much less
oppose, the process by filing a motion to dismiss or an appeal from it not even a mere
allegation that some employees participating in a petition for certification election are
actually managerial employees will lend an employer legal personality to block the
certification election.[ The employer's only right in the proceeding is to be notified or
informed thereof.
73. DEL PILAR ACADEMY VS DEL PILAR EMPLOYEES UNION

FACTS: Respondent Del Pilar Academy Employees Union (the UNION) is the
certified collective bargaining representative of teaching and non-teaching
personnel of petitioner Del Pilar Academy (DEL PILAR), an educational
institution
operating
in
Imus, Cavite. On September
15,
1994,
the UNION and DEL PILAR entered into a Collective Bargaining Agreement
(CBA), where it was agreed that:

1) The ACADEMY and the UNION agreed to maintain the wage


increase in absolute amount as programmed in the computation
prepared by the ACADEMY and dated30 June 1994 initialed by
the members of the bargaining panel of both parties, taking into
account increases in tuition fees, if any.
2) The teaching staff shall have a maximum load of 23 hours per week in
teaching and any excess thereon shall be considered as overload with
pay;
3) Any overload shall be paid extra;
4) There shall be an increase in the longevity pay;
5) Every faculty member who has rendered at least six (6) consecutive
academic semester of service shall be entitled to the 11th month and
12th month pay as summer vacation leave with pay;
6) Non-teaching employees who shall have rendered at least one (1) year
of service shall be entitled to fifteen days leave with pay.

The UNION then assessed agency fees from non-union employees, and
requested DEL PILAR to deduct said assessment from the employees salaries
and wages. DELPILAR, however, refused to effect deductions claiming that the
non-union employees were not amenable to it.

In September 1997, the UNION negotiated for the renewal of the


CBA. DEL PILAR, however, refused to renew the same unless the provision
regarding entitlement to two (2) months summer vacation leave with pay will be
amended by limiting the same to teachers, who have rendered at least three (3)
consecutive academic years of satisfactory service. The UNION objected to the
proposal claiming diminution of benefits. DEL PILAR refused to sign the CBA,
resulting in a deadlock. The UNION requested DEL PILAR to submit the case for
voluntary arbitration, but the latter allegedly refused, prompting the UNION to
file a case for unfair labor practice with the Labor Arbiter.

DEL PILAR
denied
committing
unfair
labor
practices
against
the UNION. DEL PILAR admitted its failure to deduct the agency fees from the
salaries of non-union employees, but justifies the non-deduction by the absence
of individual written authorization. It posits that Article 248(e) is inapplicable
considering that its employees derived no benefits from the CBA. The annual
salary increase of its employee is a benefit mandated by law, and not derived
from the CBA. Besides, the non-union employees objected to the deduction;
hence, a written authorization is indispensable to effect a valid check off.
As regards the proposal to amend the provision on summer vacation leave with
pay, DEL PILAR alleged that the proposal cannot be considered unfair for it was
done to make the provision of the CBA conformable to the DECS Manual of
Regulations for Private Schools.

Labor Arbiter: There was an error on the part of DEL PILAR not to have
collected agency fee due other workers who are non-union members but are
included in the bargaining unit being represented by the UNIO. As stated in Art.
248, to wit:
Employees of an appropriate collective bargaining unit
who are not members of the recognized collective
bargaining agency may be assessed a reasonable fee
equivalent to the dues and other fees paid by members of
the
recognized
collective
bargaining
agreement: Provided, that the individual authorization
required under Article [241], paragraph (o) of this Code

3E Andaya Ching Espiritu Hefti Galvez Gammad Lainez Lui Madamba Nagera Narvasa
Ong Palangdao Rosales Sanchez Santos Satrain Tabo (2014-2015)

Labor Relations Case Digest - Atty. Joyrich Golangco

shall not apply to the non-members of the recognized


collective bargaining agent.

For receipt of CBA benefits brought about by the CBA negotiated with
petitioners, non-union members are duty bound to pay agency fees
which may lawfully be deducted sans individual check-off
authorization. Being recipients of said benefits; they should share and
be made to pay the same considerations imposed upon the union
members.

The proposal to decrease the coverage of the 11th and 12th month
vacation with pay was not done in bad faith but rather in an honest
attempt to make perfect procession following the DECS Manuals. It is of
judicial notice that in the course of negotiation, almost all provisions are
up for grabs, amendments or change. This is something normal in the
course of a negotiation and does not necessarily connote bad faith as
each everyone has the right to negotiate reward or totally amend the
provisions of the contract/agreement. It must be noted that a CBA is a
contract between labor and management and is not simply a litany of
benefits for labor. For unfair labor practice to prosper there must be a
clear showing of acts aimed at stifling the workers right to selforganization. Mere allegations and mistake notions would not suffice.
NLRC: Affirmed the Arbiters ruling, upheld the UNIONs right to agency
fee, but did not consider DELPILARs failure to deduct the same an unfair
labor practice

CA: Private respondent Del Pilar Academy is ordered to deduct the agency fees
from non-union members. The agency fees shall be equivalent to the dues and
other fees paid by the union members.
ISSUES: Whether or not the UNION is entitled to collect agency fees from nonunion members, and if so, whether an individual written authorization is
necessary for a valid check off.

HELD: Yes, the grant of annual salary increase is not the only provision in the
CBA that benefited the non-union employees. The UNION negotiated for other

benefits, namely, limitations on teaching assignments to 23 hours per week,


additional compensation for overload units or teaching assignments in excess of
the 23 hour per week limit, and payment of longevity pay. It also negotiated for
entitlement to summer vacation leave with pay for two (2) months for teaching
staff who have rendered six (6) consecutive semesters of service. For the nonteaching personnel, the UNION worked for their entitlement to fifteen (15) days
leave with pay. These provisions in the CBA surely benefited the non-union
employees, justifying the collection of, and the UNIONs entitlement to, agency
fees.
Accordingly, no requirement of written authorization from the non-union
employees is needed to effect a valid check off. Article 248(e) makes it explicit
that Article 241, paragraph (o), requiring written authorization is inapplicable to
non-union members, especially in this case where the non-union employees
receive several benefits under the CBA.
As explained by this Court in Holy Cross of Davao College, Inc. v. Hon. Joaquin
The employee's acceptance of benefits resulting from a collective
bargaining agreement justifies the deduction of agency fees from
his pay and the union's entitlement thereto. In this aspect, the
legal basis of the union's right to agency fees is neither
contractual nor statutory, but quasi-contractual, deriving from
the established principle that non-union employees may not
unjustly enrich themselves by benefiting from employment
conditions negotiated by the bargaining union.
74. Edgardo Marino vs Gamilla
FACTS: Petitioners were among the executive officers and directors (collectively
called the Mario Group) of the University of Sto. Tomas Faculty Union (USTFU),
a labor union duly organized and registered under the laws of the Republic of the
Philippines and the bargaining representative of the faculty members of the
University of Santo Tomas (UST).
Respondents were UST professors and USTFU members.

3E Andaya Ching Espiritu Hefti Galvez Gammad Lainez Lui Madamba Nagera Narvasa
Ong Palangdao Rosales Sanchez Santos Satrain Tabo (2014-2015)

Labor Relations Case Digest - Atty. Joyrich Golangco

The 1986 Collective Bargaining Agreement (CBA) between UST and


USTFU expired on 31 May 1988. Thereafter, bargaining negotiations ensued
between UST and the Mario Group, which represented USTFU. They were not
able to reach an agreement and a bargaining deadlock was declared. USTFU filed
a notice of strike. Secretary of (DOLE) assumed jurisdiction over the dispute and
issued an Order laying the terms and conditions for a new CBA between the UST
and USTFU. In accordance with said Order, UST and USTFU entered into a CBA
which was to be effective for the period of 1988 to 1993 (hereinafter 1988-1993
CBA). Economic provisions of the 1988-1993 CBA were subject to renegotiation
for the fourth and fifth years.
On 1992, UST and USTFU executed a (MOA), whereby UST faculty
members belonging to the collective bargaining unit were granted additional
economic benefits for the fourth and fifth years of the 1988-1993 CBA,
specifically, the period from 1 June 1992 up to 31 May 1993.
MEMORANDUM OF AGREEMENT

1.0. The University hereby grants additional benefits


to Faculty Members belonging to the collective bargaining unit
xxx, which additional benefits shall amount in the aggregate
to P42,000,000.00[.]
7.0. It is clearly understood and agreed upon that the
aggregate sum of P42 million is chargeable against the
share of the faculty members in the incremental proceeds
of tuition fees collected and still to be collected; xxx
and incremental proceeds are, by law and pertinent (DECS)
regulations, required to be allotted for the payment of
salaries, wages, allowances and other benefits of teaching
and non-teaching personnel for the UNIVERSITY.
On 12 September 1992, the majority of USTFU members signed
individual instruments of ratification, which purportedly signified their consent
to the economic benefits granted under the MOA.

RATIFICATION OF THE UST-USTFU MEMORANDUM OF


AGREEMENT DATED SEPTEMBER 10, 1992 GRANTING A
PACKAGE OF THE P42 MILLION FACULTY BENEFITS WITH
PROVISION FOR CHECK-OFF.

September 12, 1992


Date

TO WHOM IT MAY CONCERN:

I, the undersigned UST faculty member xxx

In consideration of the efforts of the UST Faculty Union


as the faculty members sole and exclusive collective bargaining
representative in obtaining the said P42 million package of
economic benefits, a check-off of ten percent thereof
covering union dues, and special assessment for Labor
Education Fund and attorneys fees from USTFU members
and agency fee from non-members for the period of the
Agreement is hereby authorized xxx

USTFU, through its President, petitioner Atty. Mario, wrote a letter to


the UST Treasurer requesting the release to the union of the sum ofP4.2 million,
which was 10% of the P42 million economic benefits package granted by the
MOA to faculty members belonging to the collective bargaining unit. UST
remitted the sum of P4.2 million to USTFU.

After deducting from the P42 million economic benefits package several
expenses, a net amount of P6,389,145.04 remained and was distributed to the
faculty members.
On 15 December 1994, respondents filed with the Med-Arbiter, DOLENational Capital Region (NCR), a Complaint for the expulsion of the Mario
Group as USTFU officers and directors, which was docketed as Case No. NCROD-M-9412-022. Alleged in their Complaint that the Mario Group violated the
rights and conditions of membership in USTFU, particularly by: 1) investing the

3E Andaya Ching Espiritu Hefti Galvez Gammad Lainez Lui Madamba Nagera Narvasa
Ong Palangdao Rosales Sanchez Santos Satrain Tabo (2014-2015)

Labor Relations Case Digest - Atty. Joyrich Golangco

unspent balance of the P42 million economic benefits package given by UST
without prior approval of the general membership; 2) simultaneously holding
elections viva voce; 3) ratifying the CBA involving the P42 million economic
benefits package; and 4) approving the attorneys/agency fees worth P4.2
million in the form of check-off. Prayed that the Mario Group be declared
jointly and severally liable for refunding all collected attorneys/agency fees from
individual members of USTFU and the collective bargaining unit; and that, after
due hearing, the Mario group be expelled as USTFU officers and directors.

(2)

Case No. NCR-OD-M-9510-028

On 16 December 1994, UST and USTFU, represented by the Mario


Group, entered into a new CBA, effective 1 June 1993 to 31 May 1998 (19931998 CBA).
Respondents filed with the Med-Arbiter, DOLE-NCR another Complaint
against the Mario Group for violation of the rights and conditions of union
membership, which was docketed as Case No. NCR-OD-M-9510-028. The
Complaint sought to invalidate certain provisions of the1993-1998 CBA.
(3)

Case No. NCR-OD-M-9610-001

24 September 1996, petitioner Collantes, as USTFU SecretaryGeneral, posted notices in some faculty rooms at UST, informing the
union members of a general assembly. The agenda was the election of
new USTFU officers. Respondents wrote a letter[14] to the USTFU
Committee on Elections, urging the latter to re-schedule the elections.
But the Committee failed to act positively on respondents letter.

Thus, Respondents filed with the Med-Arbiter, DOLE-NCR, an Urgent ExParte Petition/Complaint, which was docketed as Case No. NCR-OD-M-9610001. Alleged that the general membership meeting was in violation of the
provisions of the Constitution and By-Laws of USTFU. Prayed that the DOLE
supervise the conduct of the USTFU elections, and that they be awarded
attorneys fees.

4 October 1996, Med-Arbiter DOLE-NCR, issued a (TRO) enjoining the


holding of the USTFU elections scheduled the next day.
(4)

Case No. NCR-OD-M-9610-016

4 October 1996, the UST Secretary General headed a general faculty


assembly. Respondents were among the elected officers of USTFU (collectively
referred to as the Gamilla Group). Petitioners filed with the Med-Arbiter, DOLENCR, a Petition seeking injunctive reliefs and the nullification of the results of
the 4 October 1994 election. The Petition was docketed as Case No. NCR-OD-M9610-016.
In Case No. NCR-OD-M-9610-016, Med-Arbiter DOLE-NCR, nullified the
election of the Gamilla Group as USTFU officers. Affirmed on appeal by the (BLR).
Respondents were prompted to file a Petition for Certiorari before this Court,
docketed as G.R. No. 131235.
While G.R. No. 131235 was pending, the term of office of the Gamilla
Group as USTFU officers expired on 4 October 1999. The Gamilla Group then
scheduled the next election of USTFU officers on 14 January 2000.

16 November 1999, the Court promulgated its Decision in G.R. No. 131235,
affirming the BLR Resolution.
(5)

Case No. NCR-OD-M-9611-009

15 November 1996, respondents[ filed before the Med-Arbiter, DOLE-NCR, a


fourth Complaint/Petition against the Mario Group, as well as the Philippine
Foundation for the Advancement of the Teaching Profession, Inc., Security Bank
Corporation, and Bank of the Philippine Islands, which was docketed as Case No.
NCR-OD-M-9611-009. Claimed that they were the legitimate USTFU officers,
having been elected on 4 October 1996. They prayed for an order directing the
Mario Group to cease and desist from using the name of USTFU and from
performing acts for and on behalf of the USTFU and the rest of the members of
the collective bargaining unit.

3E Andaya Ching Espiritu Hefti Galvez Gammad Lainez Lui Madamba Nagera Narvasa
Ong Palangdao Rosales Sanchez Santos Satrain Tabo (2014-2015)

Labor Relations Case Digest - Atty. Joyrich Golangco

DOLE Department Order No. 9 took effect on 21 June 1997, amending


the Rules Implementing Book V of the Labor Code. Thereunder, jurisdiction over
the complaints for any violation of the union constitution and by-laws and the
conditions of union membership was vested in the Regional Director of the
DOLE. All four Petitions/Complaints filed by respondents against the Mario
Group were consolidated and indorsed to the Office of the Regional Director of
the DOLE-NCR.

Director, for the expulsion of the Mario Group from their positions as USTFU
officers, practically extinguished Case No. NCR-OD-M-9611-009.
Petitioners interposed an appeal before the BLR, which was docketed
as BLR-A-TR-52-25-10-99.
The election of USTFU officers was held as scheduled on 14 January
2000, in which the Gamilla Group claimed victory. They entered into a
MOA with the UST, which provided for the economic benefits to be
granted to the faculty members of the UST for the years 19992001. Said Agreement was ratified by the USTFU members on 9 March
2000.

DOLE-NCR Regional Director rendered a Decision in the consolidated


cases in respondents favor.

In Case No. NCR-OD-M-9412-022 and Case No. NCR-OD-M-9510-028, the


DOLE-NCR Regional Director adjudged the Mario Group, as the executive
officers of USTFU, guilty of violating the provisions of the USTFU Constitution
and By-laws. Also, they violated Article 241(c) and (l) of the Labor Code when
among others, they invested in a bank, without prior consent of USTFU members,
the sum of P9,766,570.01, which formed part of the P42 million economic
benefits package. Additionally, the check-off of P4.2 million collected by the
Mario Group, as negotiation fees, was invalid. According to the MOA executed
on 10 September 1992 by UST and USTFU, the P42 million economic benefits
package was chargeable against the share of the faculty members in the
incremental proceeds of tuition fees collected and still to be collected. Under
Republic Act No. 6728, 70% of the tuition fee increases should be allotted to
academic and non-academic personnel. Given that the records were silent as to
how much of the P42 million economic benefits package was obtained through
negotiations and how much was from the statutory allotment of 70% of the
tuition fee increases, the DOLE-NCR Regional Director held that the entire
amount was within the statutory allotment, which could not be the subject of
negotiation and, thus, could not be burdened by negotiation fees. Lastly, the
principal subject of Case No. NCR-OD-M-9610-001 (i.e., violation by the Mario
Group of the provisions on election of officers in the Labor Code and the USTFU
Constitution and By-Laws) had been superseded by the central event in Case No.
NCR-OD-M-9611-009 (i.e., the subsequent election of another set of USTFU
officers consisting of the Gamilla Group). While there were two sets of USTFU
officers vying for legitimacy, the eventual ruling of the DOLE-NCR Regional

99:

9 March 2000, the BLR promulgated its Decision in BLR-A-TR-52-25-10Appeal is GRANTED IN PART. xxx appellant USTFU
officers are hereby ordered to return to the general
membership the amount of P4.2 million they have collected by
way of attorneys fees.

The election shall be held under the control and


supervision of the Regional Office, in accordance with Section 1
(b), Rule XV of Department Order No. 9, unless the parties
mutually agree to a different procedure consistent with
ensuring integrity and fairness in the electoral exercise.

BLR agreed in the finding of the DOLE-NCR Regional Director that the P42
million economic benefits package was sourced from the faculty members share
in the tuition fee increases under Republic Act No. 6728. This allotment is
mandatory and cannot be diminished, although it may be increased by collective
bargaining. Only the amount beyond that mandated by law shall be subject to
negotiation fees and attorney's fees for the simple reason that it was only this
amount that the school employees had to bargain for.
The BLR further reasoned that the P4.2 million collected by the Mario
Group was in the nature of attorneys fees or negotiation fees and fell under the

3E Andaya Ching Espiritu Hefti Galvez Gammad Lainez Lui Madamba Nagera Narvasa
Ong Palangdao Rosales Sanchez Santos Satrain Tabo (2014-2015)

Labor Relations Case Digest - Atty. Joyrich Golangco

general prohibition against such fees in Article 222(b) of the Labor Code. The
exception to charging against union funds was not applicable because the P42
million economic benefits package under the 10 September 1992 MOA was not
union fund, as the same was intended not for the union coffers, but for the
members of the entire bargaining unit. The fact that the P4.2 million check-off
was approved by the majority of USTFU members was immaterial in view of the
clear command of Article 222(b) that any contract, agreement, or arrangement of
any sort, contrary to the prohibition contained therein, shall be null and void.
Lastly, as to the alleged failure of the Mario Group to perform some of its
duties, the BLR held that the change of USTFU officers can best be decided, not by
outright expulsion, but by the general membership through the actual conduct of
elections.

Petitioners Motion for Partial Reconsideration of the foregoing Decision


was denied by the BLR.
petitioners filed with the Court of Appeals a Petition for Certiorari under
Rule 65 of the Rules of Court.
The Court of Appeals rendered its Decision favoring respondents.

It held that BLR did not commit grave abuse of discretion, amounting to
lack or excess of jurisdiction, in ruling that the P42 million economic benefits
package was merely the share of the faculty members in the tuition fee increases
pursuant to Republic Act No. 6728.
Xxx We thus agree with the [BLR] that the aforesaid
amount of P42,000,000.00 should not answer for any attorneys
fees claimed by the Petitioners. x x x.
xxxx

Moreover, [Section 5 of Rule X of] the CBL of


the Union provides that:

Section 5. Special assessments or other


extraordinary fees such as for payment of
attorneys fees shall be made only upon such a
resolution duly ratified by the general
membership by secret balloting. x x x.

Also, Article 241(n) of the Labor Code, as amended,


provides that no special assessment shall be levied upon the
members of the union unless authorized by a written resolution
of a majority of all the members at a general membership
meeting duly called for the purpose[.]
xxxx

Article 241(n) of the Labor Code, as amended, speaks of


three (3) requisites, to wit: (1) authorization by a written
resolution of the majority of all members at the general
membership meeting called for the purpose; (2) secretarys
record of the minutes of the meeting; and (3) individual written
authorization for check-off duly signed by the employee
concerned.

Contrary to the provisions of Articles 222(b) and 241(n)


of the Labor Code, as amended, and Section 5, Rule X of [the]
CBL of the Union, no resolution ratified by the general
membership of [the] USTFU through secret balloting which
embodied the award of attorneys fees was submitted. Instead,
the Petitioners submitted copies of the form for the ratification
of the MOA and the check-off for attorneys fees.
Worse, the check-off for union dues and attorneys fees
were included in the ratification of the MOA. The members
were thus placed in a situation where, upon ratification of the
MOA, not only the check-off of union dues and special
assessment for labor education fund but also the payment of
attorneys fees were (sic) authorized.

3E Andaya Ching Espiritu Hefti Galvez Gammad Lainez Lui Madamba Nagera Narvasa
Ong Palangdao Rosales Sanchez Santos Satrain Tabo (2014-2015)

Labor Relations Case Digest - Atty. Joyrich Golangco


CA found no grave abuse of discretion, amounting to lack or excess of
jurisdiction, on the part of the BLR in ordering the conduct of elections under the
control and supervision of the DOLE-NCR.
xxx
It appears that the term of office of the Petitioners had
already expired in September of 1996. On October 4, 1996,
[respondents] and the members of the faculty of UST elected
[respondents] as the new officers of the USTFU. The same was,
however nullified by the Supreme Court, on November 16,
1999. However, as the term of office of the [respondents] had
expired, on October 4, 1999, there is nothing to nullify
anymore. By virtue of an election, held on January 14, 2000, the
[respondents] were elected as the new officers of the Union,
which election was not contested by the Petitioners or any other
group in the union.
xxxx
We are thus faced with a situation where one set of
officers claim to be the legitimate and incumbent officers of the
Union, pursuant to the CBL of the Union, and another set of
officers who claim to have been elected by the members of the
faculty of the Union thru an election alleged to have been
supervised by the DOLE which situation partakes of and is akin
to the nature of an intra-union dispute[.] x x x.
CBL gives the Board of Officers the right to create and
appoint members of the Comelec. However, the CBL has no
application to a situation where there are two (2) sets of
officers. The BLR, which has jurisdiction over the intra-union
dispute, can validly order the immediate conduct of election of
officers. Department Order No. 09, Series of 1997, provides that,
in the absence of any agreement among the members or any
provision in the constitution and by-laws of the labor
organization, in an election ordered by the Regional Director,
the chairman of the committee shall be a representative of the
Labor Relations Division of the Regional Office[.]

Petitioners moved for reconsideration. It was denied.

ISSUES: Essentially, in order to arrive at a final disposition of the instant


case, this Court is tasked to determine the following: (1) the nature of
the P42 million economic benefits package granted by UST to USTFU; (2)
the legality of the 10% check-off collected by the Mario Group from
the P42 million economic benefits package; and (3) the validity of the
BLR order for USTFU to conduct election of union officers under the
control and supervision of the DOLE-NCR Regional Director.
HELD:
(1) The P42 million economic benefits package

Section 7 of the MOA they signed on 10 September 1992 that:

7.0. It is clearly understood and agreed upon that the


aggregate sum of P42 million is chargeable against the
share of the faculty members in the incremental proceeds
of tuition fees collected and still to be collected xxx which
incremental proceeds are, by law and pertinent
Department of Education Culture and Sports (DECS)
regulations, required to be allotted for the payment of
salaries, wages, allowances and other benefits of teaching
and non-teaching personnel for the UNIVERSITY.[44]
The law in the aforequoted Section 7 of the MOA can only
refer to Republic Act No. 6728, Government Assistance to
Students and Teachers in Private Education Act." Said statute
primarily grants various forms of financial aid to private
educational institutions such as tuition fee supplements,
assistance funds, and scholarship grants.[46]

Although Section 5 of Republic Act No. 6728 does speak of government


assistance to students in private high schools, it is not limited to the
same. Contrary to petitioners puerile claim, Section 5 likewise grants an

3E Andaya Ching Espiritu Hefti Galvez Gammad Lainez Lui Madamba Nagera Narvasa
Ong Palangdao Rosales Sanchez Santos Satrain Tabo (2014-2015)

Labor Relations Case Digest - Atty. Joyrich Golangco

unmistakable authority to private high schools to increase their tuition


fees, subject to the condition that seventy (70%) percent of the tuition fee
increases shall go to the payment of the salaries, wages, allowances, and other
benefits of their teaching and non-teaching personnel. The said allocation may
also be used to cover increases in the salaries, wages, allowances, and other
benefits of school employees as provided for in the CBAs existing or in force at
the time when Republic Act No. 6728 was approved and made effective.

Contrary to petitioners argument, the right of private schools to


increase their tuition fee -- with their corresponding obligation to allocate 70%
of said increase to the payment of the salaries, wages, allowances, and other
benefits of their employees -- is not limited to private high schools.
Indeed, a private educational institution under Republic Act No. 6728
still has the discretion on the disposition of 70% of the tuition fee increase. The
only precondition is that 70% percent of the incremental tuition fee increase
goes to the payment of salaries, wages, allowances and other benefits of teaching
and non-teaching personnel.

In this case, UST and USTFU stipulated in their 10 September 1992 MOA
that the P42 million economic benefits package granted by UST to the members
of the collective bargaining unit represented by USTFU, was chargeable against
the 70% allotment from the proceeds of the tuition fee increases collected and
still to be collected by UST. As observed by the DOLE-NCR Regional Director, and
affirmed by both the BLR and the Court of Appeals, there is no showing that any
portion of the P42 million economic benefits package was derived from sources
other than the 70% allotment from tuition fee increases of UST.

Given the lack of evidence to the contrary, it can be conclusively


presumed that the entire P42 million economic benefits package extended to
USTFU came from the 70% allotment from tuition fee increases of
UST. Preceding from this presumption, any deduction from the P42 million
economic benefits package, such as the P4.2 million claimed by the Mario Group
as attorneys/agency fees, should not be allowed, because it would ultimately
result in the reduction of the statutorily mandated 70% allotment from the
tuition fee increases of UST.

(2)

The P4.2 Million Check-off

The pertinent legal provisions on a check-off are found in Articles 222(b)


and 241(n) and (o) of the Labor Code, as amended.
Article 222(b) states:

(b) No attorney's fees, negotiation fees or similar charges


of any kind arising from any collective bargaining negotiations
or conclusion of the collective agreement shall be imposed on
any
individual
member
of
the
contracting
union: Provided, however, that attorney's fees may be charged
against unions funds in an amount to be agreed upon by the
parties. Any contract, agreement or arrangement of any sort to
the contrary shall be null and void.

Article 241(n) reads:

(n) No special assessment or other extraordinary fees


may be levied upon the members of a labor organization unless
authorized by a written resolution of a majority of all the
members at a general membership meeting duly called for the
purpose. The secretary of the organization shall record the
minutes of the meeting including the list of all members present,
the votes cast, the purpose of the special assessment or fees and
the recipient of such assessment or fees. The record shall be
attested to by the president.

And Article 241(o) provides:

(o) Other than for mandatory activities under the Code,


no special assessments, attorney's fees, negotiation fees or any
other extraordinary fees may be checked off from any amount
due to an employee without an individual written authorization
duly signed by the employee. The authorization should

3E Andaya Ching Espiritu Hefti Galvez Gammad Lainez Lui Madamba Nagera Narvasa
Ong Palangdao Rosales Sanchez Santos Satrain Tabo (2014-2015)

Labor Relations Case Digest - Atty. Joyrich Golangco

specifically state the amount, purpose and beneficiary of the


deduction.

General rule is that attorneys fees, negotiation fees, and other similar
charges may only be collected from union funds, not from the amounts that
pertain to individual union members. As an exception, special assessments or
other extraordinary fees may be levied upon or checked off from any amount due
an employee for as long as there is proper authorization by the employee.

A check-off is a process or device whereby the employer, on agreement


with the Union, recognized as the proper bargaining representative, or on prior
authorization from the employees, deducts union dues or agency fees from the
latter's wages and remits them directly to the Union.

The Court finds that, in the instant case, the P42 million economic
benefits package granted by UST did not constitute union funds from whence
the P4.2 million could have been validly deducted as attorneys fees. The P42
million economic benefits package was not intended for the USTFU coffers, but
for all the members of the bargaining unit USTFU represented, whether members
or non-members of the union. A close reading of the terms of the MOA reveals
that after the satisfaction of the outstanding obligations of UST under the 1986
CBA, the balance of the P42 million was to be distributed to the covered faculty
members of the collective bargaining unit in the form of salary increases, returns
on paycheck deductions; and increases in hospitalization, educational, and
retirement benefits, and other economic benefits. The deduction of the P4.2
million, as alleged attorneys/agency fees, from the P42 million economic
benefits package effectively decreased the share from said package accruing to
each member of the collective bargaining unit.
Petitioners line of argument that the amount of P4.2 million became
union funds after its deduction from the P42 million economic benefits package
and, thus, could already be used to pay attorneys fees, negotiation fees, or
similar charges from the CBA is absurd. Petitioners reasoning is evidently
flawed since the attorneys fees may only be paid from union funds; yet the
amount to be used in paying for the same does not become union funds until it is
actually deducted as attorneys fees from the benefits awarded to the

employees. What the law requires is that the funds be already deemed union
funds even before the attorneys fees are deducted or paid therefrom; it does not
become union funds after the deduction or payment.

The Court further determines that the requisites for a valid levy and
check-off of special assessments, laid down by Article 241(n) and (o),
respectively, of the Labor Code, as amended, have not been complied with in the
case at bar. To recall, these requisites are: (1) an authorization by a written
resolution of the majority of all the union members at the general membership
meeting duly called for the purpose; (2) secretary's record of the minutes of the
meeting; and (3) individual written authorization for check-off duly signed by
the employee concerned.

Additionally, Section 5, Rule X of the USTFU Constitution and By-Laws


mandates that:
Section 5. Special assessments or other extraordinary
fees such as for payment of attorneys fees shall be made only
upon a resolution duly ratified by the general membership by
secret balloting.

In an attempt to comply with the foregoing requirements, the Mario


Group caused the majority of the general membership of USTFU to individually
sign a document, which embodied the ratification of the MOA between UST and
USTFU, dated 10 September 1992, as well as the authorization for the check-off
of P4.2 million, from the P42 million economic benefits package, as payment for
attorneys fees. As held by the Court of Appeals, however, the said documents
constitute unsatisfactory compliance with the requisites set forth in the Labor
Code, as amended, and in the USTFU Constitution and By-Laws, even though
individually signed by a majority of USTFU members.
The inclusion of the authorization for a check-off of union dues and
special assessments for the Labor Education Fund and attorneys fees, in the
same document for the ratification of the 10 September 1992 MOA granting

3E Andaya Ching Espiritu Hefti Galvez Gammad Lainez Lui Madamba Nagera Narvasa
Ong Palangdao Rosales Sanchez Santos Satrain Tabo (2014-2015)

Labor Relations Case Digest - Atty. Joyrich Golangco

the P42 million economic benefits package, necessarily vitiated the consent of
USTFU members.

The failure of the Mario Group to strictly comply with the requirements
set forth by the Labor Code, as amended, and the USTFU Constitution and ByLaws, invalidates the questioned special assessment. Substantial compliance is
not enough in view of the fact that the special assessment will diminish the
compensation of the union members. Their express consent is required, and this
consent must be obtained in accordance with the steps outlined by law, which
must be followed to the letter. No shortcuts are allowed.

Court does not hesitate to declare as illegal the check-off of P4.2 million,
from the P42 million economic benefits package. Said amount rightfully belongs
to and should be returned by petitioners to the intended beneficiaries
thereof, i.e., members of the collective bargaining unit, whether or not members
of USTFU.
(3)

Election of new officers

Having been overtaken by subsequent events, the Court need no longer


pass upon the issue of the validity of the order of BLR for USTFU to conduct its
long overdue election of union officers, under the control and supervision of the
DOLE-NCR Regional Director.
The Court points out, however, that neither the Decision of the BLR nor
of the Court of Appeals took into account the fact that an election of USTFU
officers was already conducted on 14 January 2000, which was won by the
Gamilla Group. There is nothing in the records to show that the said election was
contested or made the subject of litigation. The Gamilla Group had exercised
their powers as USTFU officers during their elected term. Since the term of
union officers under the USTFU Constitution and By-Laws was only for three
years, then the term of the Gamilla Group already expired in 2003. It is already
beyond the jurisdiction of this Court, in the present Petition, to still look into the
subsequent elections of union officers held after 2003.
The election of the Gamilla Group as union officers in 2000 should have
already been recognized by the BLR and the Court of Appeals. The order for

USTFU to conduct another election was only a superfluity. The issue of who
between the officers of the Mario Group and of the Gamilla Group are the
legitimate USTFU officers has been rendered moot by the succeeding events in
the case.
WHEREFORE, premises considered, the Petition for Review under Rule
45 of the Rules of Court is hereby DENIED.
75 ABARIA vs. NLRC

FACTS:

Metro Cebu Community Hospital, Inc. (MCCHI), presently known as the


Visayas Community Medical Center (VCMC) operates the Metro Cebu
Community Hospital (MCCH) owned by the United Church of Christ in
the Philippines (UCCP) and Rev. Gregorio P. Iyoy is the Hospital
Administrator.
The National Federation of Labor (NFL) is the exclusive bargaining
representative of the rank-and-file employees of MCCHI.
On December 6, 1995, Nava through NAMA-MCCH-NFL wrote Rev. Iyoy
expressing the unions desire to renew the CBA, attaching to her letter a
statement of proposals signed/endorsed by 153 union members. Nava
subsequently requested that some employees to be allowed to avail of
one-day union leave with pay
Meanwhile, Atty. Alforque informed MCCHI that NFL has not authorized
any person for collective bargaining negotiations. By January 1996, the
collection of union fees (check-off) was temporarily suspended by
MCCHI in view of the existing conflict between the federation and its
local affiliate.
On February 26, 1996, upon the request of Atty. Alforque, MCCHI
granted one-day union leave with pay for 12 union members
The next day, several union members led by Nava and her group
launched a series of mass actions such as wearing black and red
armbands/headbands, marching around the hospital premises and
putting up placards, posters and streamers.
MCCHI directed the union officers led by Nava to submit within 48 hours
a written explanation why they should not be terminated for having

3E Andaya Ching Espiritu Hefti Galvez Gammad Lainez Lui Madamba Nagera Narvasa
Ong Palangdao Rosales Sanchez Santos Satrain Tabo (2014-2015)

Labor Relations Case Digest - Atty. Joyrich Golangco

engaged in illegal concerted activities amounting to strike, and placed


them under immediate preventive suspension. Responding to this
directive, Nava and her group denied there was a temporary stoppage of
work, explaining that employees wore their armbands only as a sign of
protest and reiterating their demand for MCCHI to comply with its duty
to bargain collectively.
Rev. Iyoy, having been informed that Nava and her group have also been
suspended by NFL, directed said officers to appear before his office for
investigation in connection with the illegal strike. Said union officers,
however, invoked the grievance procedure provided in the CBA to settle
the dispute between management and the union.
On March 13 and 19, 1996, DOLE issued certifications stating that there
is nothing in their records which shows that NAMA-MCCH-NFL is a
registered labor organization, and that said union submitted only a copy
of its Charter Certificate on January 31, 1995.
The union members contended that MCCHI cannot question the legal
personality of the union which had actively assisted in CBA negotiations
and implementation
NAMA-MCCH-NFL filed a Notice of Strike but the same was deemed
not filed for want of legal personality on the part of the filer. Despite
such rebuff, Nava and her group still conducted a strike vote, which an
overwhelming majority of union members approved of.
MCCHI again sent notices informing them that their refusal to submit to
investigation is deemed a waiver of their right to explain their side and
management shall proceed to impose proper disciplinary action under
the circumstances. MCCHI then sent termination letters to union leaders
and other members who participated in the strike and picketing
activities and also issued a cease-and-desist order to the rest of the
striking employees stressing that the wildcat concerted activities
spearheaded by the Nava group is illegal without a valid Notice of Strike
and warning them that non-compliance will compel management to
impose disciplinary actions against them. For their continued picketing
activities despite the said warning, more than 100 striking employees
were dismissed
Unfazed, the striking union members held more mass actions. The
means of ingress to and egress from the hospital were blocked.

Employees and patients reported acts of intimidation and harassment


perpetrated by union leaders and members. With the intensified
atmosphere of violence and animosity within the hospital premises as a
result of continued protest activities by union members, MCCHI suffered
heavy losses due to low patient admission rates.
MCCHI filed a petition for injunction in the NLRC and a TRO was
issued.
MCCHI presented 12 witnesses (hospital employees and patients),
including a security guard who was stabbed by an identified
sympathizer while in the company of Navas group. MCCHIs petition
was granted and a permanent injunction was issued enjoining the
Nava group from committing illegal acts mentioned in Art. 264 of
the Labor Code
Thereafter, several complaints for illegal dismissal and unfair labor
practice were filed by the terminated employees against MCCHI, Rev.
Iyoy, UCCP and members of the Board of Trustees of MCCHI.
Executive Labor Arbiter Reynoso A. Belarmino rendered his decision
dismissing the complaints for unfair labor practice in NLRC for finding
no basis for the charge of unfair labor practice and declared the
strike and picketing activities illegal having been conducted by
NAMA-MCCH-NFL which is not a legitimate labor organization; the
NLRC denied complainants motion for reconsideration
Complainants elevated the case to the CA via a petition for certiorari,
which was because only 47 out of 88 petitioners signed the
certification against forum shopping.

ISSUES (1) whether the CA erred in dismissing the petition for certiorari; (2)
whether MCCHI is guilty of unfair labor practice; (3) whether petitioning
employees were illegally dismissed; and (4) if their termination was illegal,
whether petitioning employees are entitled to separation pay, backwages,
damages and attorneys fees.
1. Yes, CA erred in dropping those who did not sign the certification

The Court has laid down the rule in Altres v. Empleo as culled

3E Andaya Ching Espiritu Hefti Galvez Gammad Lainez Lui Madamba Nagera Narvasa
Ong Palangdao Rosales Sanchez Santos Satrain Tabo (2014-2015)

Labor Relations Case Digest - Atty. Joyrich Golangco

from jurisprudential pronouncements, that the certification


against forum shopping must be signed by all the plaintiffs or
petitioners in a case; otherwise, those who did not sign will be
dropped as parties to the case. Under reasonable or justifiable
circumstances, however, as when all the plaintiffs or petitioners
share a common interest and invoke a common cause of action
or defense, the signature of only one of them in the certification
against forum shopping substantially complies with the Rule.
In the case at bar, the signatures of 47 out of 88 petitioning
employees in the certification against forum shopping
constitute substantial compliance with the rule. When they
appealed their case to the CA, they pursued the same as a
collective body, raising only one argument in support of their
cause of action, i.e., the illegal dismissal allegedly committed by
MCCHI when union members resorted to strike and mass
actions due to MCCHIs refusal to bargain with officers of the
local chapter. Clearly, the CA erred in dropping as partiespetitioners those who did not sign the certification against
forum shopping.
2. No. MCCHI is not guilty of ULP

Art. 248 (g) of the Labor Code, as amended, makes it an unfair labor
practice for an employer [t]o violate the duty to bargain collectively as
prescribed by the Code. The applicable provision in this case is Art. 253
which provides:
ART. 253. Duty to bargain collectively when
there exists a collective bargaining agreement. When
there is a collective bargaining agreement, the duty to
bargain collectively shall also mean that neither party shall
terminate nor modify such agreement during its
lifetime. However, either party can serve a written notice to
terminate or modify the agreement at least sixty (60) days
prior to its expiration date. It shall be the duty of both
parties to keep the status quo and to continue in full force
and effect the terms and conditions of the existing
agreement during the 60-day period and/or until a new

agreement is reached by the parties.


Records of the NCMB and DOLE confirmed that NAMAMCCH-NFL had not registered as a labor organization,
having submitted only its charter certificate as an affiliate or
local chapter of NFL. Not being a legitimate labor
organization, NAMA-MCCH-NFL is not entitled to those
rights granted to a legitimate labor organization under
Art. 242, specifically:
(a) To act as the representative of its members for the
purpose of collective bargaining;
(b) To be certified as the exclusive representative of
all the employees in an appropriate collective
bargaining unit for purposes of collective bargaining;
NAMA-MCCH-NFL is not the labor organization certified or
designated by the majority of the rank-and-file hospital employees
to represent them in the CBA negotiations but the NFL, as evidenced
by CBAs concluded in 1987, 1991 and 1994. While it is true that a local
union has the right to disaffiliate from the national federation,
NAMA-MCCH-NFL has not done so as there was no any effort on its
part to comply with the legal requisites for a valid disaffiliation,
Nava and her group simply demanded that MCCHI directly negotiate
with the local union which has not even registered as one.
In any case, NAMA-MCCH-NFL at the time of submission of said
proposals was not a duly registered labor organization, hence it
cannot legally represent MCCHIs rank-and-file employees for
purposes of collective bargaining. Hence, even assuming that
NAMA-MCCH-NFL had validly disaffiliated from its mother union,
NFL, it still did not possess the legal personality to enter into CBA
negotiations
The NFL as the mother union has the right to investigate members of its
local chapter under the federations Constitution and By-Laws, and if
found guilty to expel such members. MCCHI therefore cannot be faulted
for deferring action on the CBA proposal submitted by NAMA-MCCHNFL in view of the union leaderships conflict with the national
federation.
NAMA-MCCH-NFL cannot demand from MCCHI the right to bargain

3E Andaya Ching Espiritu Hefti Galvez Gammad Lainez Lui Madamba Nagera Narvasa
Ong Palangdao Rosales Sanchez Santos Satrain Tabo (2014-2015)

Labor Relations Case Digest - Atty. Joyrich Golangco

collectively in their behalf. Hence, MCCHIs refusal to bargain then


with NAMA-MCCH-NFL cannot be considered an unfair labor
practice to justify the staging of the strike
Strike and picketing activities conducted by union officers and members
were illegal
Art. 263 (b) of the Labor Code, as amended, provides:
ART. 263. Strikes, picketing and lockouts. x x x
(b) Workers shall have the right to engage in
concerted activities for purposes of collective bargaining or
for their mutual benefit and protection. The right of
legitimate labor organizations to strike and picket and
of employers to lockout, consistent with the national
interest, shall continue to be recognized and
respected. However, no labor union may strike and no
employer may declare a lockout on grounds involving interunion and intra-union disputes.
x x x x (Emphasis supplied.)
Since NAMA-MCCH-NFL was not a duly registered or an
independently registered union at the time it filed the notice
of strike and when it conducted the strike vote.
Consequently, the mandatory notice of strike and the
conduct of the strike vote report were ineffective for
having been filed and conducted by NAMA-MCCH-NFL
which has no legal personality as a legitimate labor
organization, in violation of Art. 263 (c), (d) and (f) of the
Labor Code and Rule XXII, Book V of the Omnibus Rules
Implementing the Labor Code.
Art. 263 of the Labor Code provides:
ART. 263. Strikes, picketing and lockouts.
(a) x x x
xxxx
(c) In cases of bargaining deadlocks, the duly
certified or recognized bargaining agent may file a
notice of strike or the employer may file a notice of lockout
with the Department at least 30 days before the intended

date thereof. In cases of unfair labor practice, the period of


notice shall be 15 days and in the absence of a duly
certified or recognized bargaining agent, the notice of
strike may be filed by any legitimate labor organization
in behalf of its members. However, in case of dismissal
from employment of union officers duly elected in
accordance with the union constitution and by-laws, which
may constitute union busting, where the existence of the
union is threatened, the 15-day cooling-off period shall not
apply and the union may take action immediately. (As
amended by Executive Order No. 111, December 24, 1986.)
Rule XXII, Book V of the Omnibus Rules Implementing the
Labor Code reads:
RULE XXII
CONCILIATION, STRIKES AND LOCKOUTS
xxxx
SEC. 6. Who may declare a strike or lockout.
Any certified or duly recognized bargaining representative
may declare a strike in cases of bargaining deadlocks and
unfair labor practices. The employer may declare a lockout
in the same cases. In the absence of a certified or duly
recognized bargaining representative, any legitimate
labor organization in the establishment may declare a
strike but only on grounds of unfair labor
practice. (Emphasis supplied.)
Furthermore, the strike was illegal due to the commission of the
following prohibited activities: (1) violence, coercion, intimidation
and harassment against non-participating employees; and (2)
blocking of free ingress to and egress from the hospital, including
preventing patients and their vehicles from entering the hospital and
other employees from reporting to work, the putting up of placards
with a statement advising incoming patients to proceed to another
hospital because MCCHI employees are on strike/protest. The
prolonged work stoppage and picketing activities of the striking
employees severely disrupted hospital operations that MCCHI
suffered heavy financial losses.

3E Andaya Ching Espiritu Hefti Galvez Gammad Lainez Lui Madamba Nagera Narvasa
Ong Palangdao Rosales Sanchez Santos Satrain Tabo (2014-2015)

Labor Relations Case Digest - Atty. Joyrich Golangco


3. Consequences of illegal strike to union officers and members
Art. 264 (a) of the Labor Code, as amended, provides for the consequences
of an illegal strike to the participating workers:
x x x Any union officer who knowingly participates in
illegal strike and any worker or union officer who knowingly
participates in the commission of illegal acts during a strike
may be declared to have lost his employment status:
Provided, That mere participation of a worker in a lawful
strike shall not constitute sufficient ground for termination
of his employment, even if a replacement had been hired by
the employer during such lawful strike.
The above provision makes a distinction between workers and union
officers who participate in an illegal strike: an ordinary striking
worker cannot be terminated for mere participation in an illegal
strike. There must be proof that he or she committed illegal acts during
a strike. A union officer, on the other hand, may be terminated from
work when he knowingly participates in an illegal strike, and like
other workers, when he commits an illegal act during a strike.
Considering their persistence in holding picketing activities despite
the declaration by the NCMB that their union was not duly
registered as a legitimate labor organization and the letter from
NFLs legal counsel informing that their acts constitute disloyalty to
the national federation, and their filing of the notice of strike and
conducting a strike vote notwithstanding that their union has no
legal personality to negotiate with MCCHI for collective bargaining
purposes, there is no question that NAMA-MCCH-NFL officers
knowingly participated in the illegal strike.
With respect to the dismissed union members, although MCCHI
submitted photographs taken at the picket line, it did not individually
name those striking employees and specify the illegal act committed by
each of them. Consequently, we find no error committed by the CA in
CA-G.R. SP No. 66540 when it modified the decision of the NLRC and
ruled that the dismissal of union members who merely participated
in the illegal strike was illegal.

4. Separation Pay Only


Since there is no clear proof that union members actually
participated in the commission of illegal acts during the strike, they
are not deemed to have lost their employment status as a
consequence of a declaration of illegality of the strike.
The CA decision ordering the payment of separation pay in lieu of
reinstatement without back wages is thus in order, to conform to the
policy of a fair days wage for a fair days labor.
Separation pay is made an alternative relief in lieu of reinstatement in
certain circumstances, like: (a) when reinstatement can no longer be
effected in view of the passage of a long period of time or because of the
realities of the situation; (b) reinstatement is inimical to the employers
interest; (c) reinstatement is no longer feasible; (d) reinstatement does
not serve the best interests of the parties involved; (e) the employer is
prejudiced by the workers continued employment; (f) facts that make
execution unjust or inequitable have supervened; or (g) strained
relations between the employer and employee.

Considering that 15 years had lapsed from the onset of this labor
dispute, and in view of strained relations that ensued, in addition to
the reality of replacements already hired by the hospital which had
apparently recovered from its huge losses, and with many of the
petitioners either employed elsewhere, already old and sickly, or
otherwise incapacitated, separation pay without back wages is the
appropriate relief. We note that during the pendency of the cases in
this Court, some of the petitioners have entered into compromise
agreements with MCCHI, all of which were duly approved by this
Court. Thus, there are some employees who are excluded from the
herein monetary awards are the following petitioners whose
compromise agreements have been approved by this Court.
76. Wesleyan University-Philippines v. Wesleyan University-Philippines
Faculty and Staff

FACTS: Petitioner Wesleyan University-Philippines is a non-stock, non-profit


educational institution duly organized and existing under the laws of the
Philippines. Respondent Wesleyan University-Philippines Faculty and Staff

3E Andaya Ching Espiritu Hefti Galvez Gammad Lainez Lui Madamba Nagera Narvasa
Ong Palangdao Rosales Sanchez Santos Satrain Tabo (2014-2015)

Labor Relations Case Digest - Atty. Joyrich Golangco

Association, on the other hand, is a duly registered labor organization acting as


the sole and exclusive bargaining agent of all rank-and-file faculty and staff
employees of petitioner. Parties entered into a 5-year CBA effective June 1, 2003
until May 31, 2008. On August 2005 petitioner issued a memorandum regarding
the implementation of sick leave and vacation leave benefits which modified the
agreement stated in the CBA. Under the memorandum, vacation and sick leave
benefits will no longer be automatic and will have to be earned by each
employee. Aside from this, only vacation leaves will be commuted to cash.
Respondents president De Lara questioned the new changes made by the
memorandum for being violative of the existing CBA practices. During the Labor
Management Committee meeting, petitioner also announced that it will be
adopting a one-retirement policy. The dispute was submitted to a voluntary
arbitrator after the parties failed to settle at the grievance level. Respondent
submitted evidence supporting its claim that there has been an established
practice of giving two retirement benefits, one from the Private Education
Retirement Annuity Association(PERAA) Plan and another from the CBA
Retirement Plan. The arbitrator ruled in favor of respondent and held that the
new memorandum and the one-retirement policy are contrary to law. CA
affirmed the decision.
ISSUES: WON the granting of two retirement policies has already been
established as practice

HELD: Yes. In this case, respondent was able to present substantial evidence in
the form of affidavits to support its claim that there are two retirement plans.
Based on the affidavits, petitioner has been giving two retirement benefits as
early as 1997. Petitioner, on the other hand, failed to present any evidence to
refute the veracity of these affidavits. Petitioners contention that these
affidavits are self-serving holds no water. The retired employees of petitioner
have nothing to lose or gain in this case as they have already received their
retirement benefits. Thus, they have no reason to perjure themselves.
Obviously, the only reason they executed those affidavits is to bring out the truth.
As we see it then, their affidavits, corroborated by the affidavits of incumbent
employees, are more than sufficient to show that the granting of two retirement
benefits to retiring employees had already ripened into a consistent and
deliberate practice.

77. CATHAY PACIFIC STEEL CORPORATION, BENJAMIN CHUA JR., VIRGILIO


AGERO, and LEONARDO VISORRO, JR., Petitioners, vs. HON. COURT OF
APPEALS, CAPASCO UNION OF SUPERVISORY EMPLOYEES (CUSE) and
ENRIQUE TAMONDONG III, Respondents.
FACTS: Petitioner CAPASCO hired private respondent Enrique Tamondong III as
Assistant to the Personnel Manager for its Cainta Plant. Thereafter, he was
promoted to the position of Personnel/Administrative Officer, and later to that of
Personnel Superintendent. Sometime in June 1996, the supervisory personnel of
CAPASCO launched a move to organize a union among their ranks, later known
as private respondent CUSE. Tamondong actively involved himself in the
formation of the union and was even elected as one of its officers after its
creation. Consequently, petitioner CAPASCO sent a memo to Tamondong
requiring him to explain and to discontinue from his union activities, with a
warning that a continuance thereof shall adversely affect his employment in the
company. Tamondong ignored said warning and made a reply letter invoking his
right as a supervisory employee to join and organize a labor union. In view of
that, CAPASCO through a memo terminated his employment on the ground of
loss of trust and confidence, citing his union activities as acts constituting serious
disloyalty to the company.

Tamondong challenged his dismissal for being illegal and as an act


involving unfair labor practice by filing a Complaint for Illegal Dismissal and
Unfair Labor Practice before the NLRC. The Acting Executive Labor Arbiter Pedro
C. Ramos rendered a Decision in favor of Tamondong.

Aggrieved, petitioners appealed said Decision to the NLRC, which


dismissed the complaint for illegal dismissal and unfair labor practice, but
ordered the payment of backwages and reinstatement. Petitioners filed a Motion
for Clarification and Partial Reconsideration while Tamondong filed a Motion for
Reconsideration of the said NLRC Decision, but the NLRC affirmed its original
Decision.
Dissatisfied, Tamondong and CUSE filed a Petition for Certiorari under
Rule 65 of the Rules of Court before the Court of Appeals, alleging grave abuse of

3E Andaya Ching Espiritu Hefti Galvez Gammad Lainez Lui Madamba Nagera Narvasa
Ong Palangdao Rosales Sanchez Santos Satrain Tabo (2014-2015)

Labor Relations Case Digest - Atty. Joyrich Golangco

discretion on the part of the NLRC. The CA granted the said Petition and
reinstated the decision of the Labor Arbiter.

Consequently, petitioners filed a Motion for Reconsideration of the said CA


Decision but this was denied.

Hence, this present Petition for Certiorari under Rule 65. Petitioners
aver that Tamondong as Personnel Superintendent of CAPASCO was performing
functions of a managerial employee because he was the one laying down major
management policies on personnel relations such as: issuing memos on company
rules and regulations, imposing disciplinary sanctions such as warnings and
suspensions, and executing the same with full power and discretion. They claim
that no further approval or review is necessary to execute these functions, and
the notations "NOTED BY" of petitioner Agerro, the Vice-President of petitioner
CAPASCO, on the aforesaid memos are nothing but mere notice that Agerro was
aware of such company actions performed by Tamondong. Additionally,
Tamondong was not only a managerial employee but also a confidential
employee having knowledge of confidential information involving company
policies on personnel relations.
Petitioners further argue that they are not guilty of illegal dismissal and
unfair labor practice because Tamondong was validly dismissed and the reason
for preventing him to join a labor union was the nature of his position and
functions as Personnel Superintendent, which position was incompatible and in
conflict with his union activities. Consequently, the CA acted with grave abuse of
discretion amounting to lack or excess of jurisdiction when it held that
Tamondong was not a managerial employee but a mere supervisory employee,
making him eligible to participate in the union activities of private respondent
CUSE, and when it held that petitioner CAPASCO was guilty of illegal dismissal
and unfair labor practice.

ISSUES: WON Tamondong is a managerial employee and is thus ineligible to join


CUSE.
HELD: NO, (In the case before this Court, petitioners fail to meet the third
requisite for the proper invocation of Petition for Certiorari under Rule 65, to

wit: that there is no appeal or any plain, speedy, and adequate remedy in the
ordinary course of law.)
In any event, granting arguendo, that the present petition is proper, still it is
dismissible. The CA cannot be said to have acted with grave abuse of discretion
amounting to lack or excess of jurisdiction in annulling the Decision of the NLRC
because the findings of the CA that Tamondong was indeed a supervisory
employee and not a managerial employee, thus, eligible to join or participate in
the union activities of private respondent CUSE, were supported by evidence on
record. In the Decision of the CA, it made reference to the Memorandum which
required private respondent Tamondong to observe fixed daily working hours
from 8:00 am to 12:00 noon and from 1:00 pm to 5:00 pm. This imposition upon
private respondent Tamondong, according to the Court of Appeals, is very
uncharacteristic of a managerial employee. To support such a conclusion, the
Court of Appeals cited the case of Engineering Equipment, Inc. v. NLRC where
this Court held that one of the essential characteristics of an employee holding a
managerial rank is that he is not subjected to the rigid observance of regular
office hours or maximum hours of work.
CA also held that upon careful examination of the documents submitted before it,
it found out that:

Tamondong may have possessed enormous powers and was performing


important functions that goes with the position of Personnel Superintendent,
nevertheless, there was no clear showing that he is at liberty, by using his own
discretion and disposition, to lay down and execute major business and
operational policies for and in behalf of CAPASCO. CAPASCO miserably failed to
establish that Tamondong was authorized to act in the interest of the company
using his independent judgment. x x x. Withal, Tamondong may have been
exercising certain important powers, such as control and supervision over erring
rank-and-file employees, however, x x x he does not possess the power to hire,
transfer, terminate, or discipline erring employees of the company. At the most,
the record merely showed that Tamondong informed and warned rank-and-file
employees with respect to their violations of CAPASCOs rules and regulations. x
x x. [Also, the functions performed by private respondent such as] issuance of
warning to employees with irregular attendance and unauthorized leave of

3E Andaya Ching Espiritu Hefti Galvez Gammad Lainez Lui Madamba Nagera Narvasa
Ong Palangdao Rosales Sanchez Santos Satrain Tabo (2014-2015)

Labor Relations Case Digest - Atty. Joyrich Golangco

absences and requiring employees to explain regarding charges of abandonment


of work, are normally performed by a mere supervisor, and not by a manager.

Article 212(m) of the Labor Code, as amended, differentiates supervisory


employees from managerial employees, to wit: supervisory employees are those
who, in the interest of the employer, effectively recommend such managerial
actions, if the exercise of such authority is not merely routinely or clerical in
nature but requires the use of independent judgment; whereas, managerial
employees are those who are vested with powers or prerogatives to lay down
and execute management policies and/or hire, transfer, suspend, lay off, recall,
discharge, assign or discipline employees. Thus, it can be clearly inferred that
Tamondong was just a supervisory employee. He did not perform any of the
functions of a managerial employee as stated in the definition given to it by the
Code. Hence, the Labor Code provisions regarding disqualification of a
managerial employee from joining, assisting or forming any labor organization
does not apply to Tamondong. Being a supervisory employee of CAPASCO, he
cannot be prohibited from joining or participating in the union activities of
private respondent CUSE, and in making such a conclusion, the CA did not act
whimsically, capriciously or in a despotic manner, rather, it was guided by the
evidence submitted before it. Thus, given the foregoing findings of the Court of
Appeals that private respondent is a supervisory employee, it is indeed an unfair
labor practice on the part of petitioner CAPASCO to dismiss him on account of his
union activities, thereby curtailing his constitutionally guaranteed right to selforganization.
78. Chris Garments Corporation vs HON. Patricia A. Sto. Tomas

FACTS: Respondent Chris Garments Workers UnionPTGWO, filed a petition for


certification election with the Med-Arbiter. The union sought to represent
petitioners rank-and-file employees not covered by its CBA with the Samahan
Ng Mga Manggagawa sa Chris Garments CorporationSolidarity of Union in
the Philippines for Empowerment and Reforms (SMCGC-SUPER), the certified
bargaining agent of the rank-and-file employees. Petitioner Chris Garments
moved to dismiss the petition. It argued that it has an existing CBA from July 1,
1999 to June 30, 2004 with SMCGC-SUPER which bars any petition for certification
election prior to the 60-day freedom period. It also contended that the union
members are not its regular employees since they are direct employees of qualified

and independent contractors. The union countered that its members are regular
employees of petitioner. The Med-Arbiter ruled that there was no employeremployee relationship between the parties since the union itself admitted that its
members are agency employees and that even if the union members are
considered direct employees of petitioner, the petition for certification election
will still fail due to the contract bar rule under Article 232 of the Labor Code.
Nevertheless, the Med-Arbiter ruled that the union may avail of the CBA benefits
by paying agency fees to SMCGC-SUPER. The Secretary of Labor and Employment
affirmed the decision of the Med-Arbiter. She ruled that petitioner failed to
prove that the union members are employees of qualified and independent
contractors. Thus, the union members may be considered part of the bargaining
unit of petitioners rank-and-file employees. However, she held that the petition
could not be entertained except during the 60-day freedom period.

On 2003, the union filed a second petition for certification election. The
Med-Arbiter dismissed the petition on the ground that it was barred by a prior
judgment. On appeal, the Secretary of Labor and Employment affirmed the
decision of the Med-Arbiter. On 2004, the union filed a third petition for
certification election. The Med-Arbiter dismissed the petition on the grounds
that no employer-employee relationship exists between the parties and that the
case was barred by a prior judgment. On appeal, the Secretary of Labor and
Employment granted the petition. Petitioner filed a petition for certiorari with
the Court of Appeals which was dismissed due to its failure to file a motion for
reconsideration of the decision before filing the petition.
ISSUES: WON the Secretary of Labor erred in granting the certificate of election
to herein private respondents despite the ground of res judicata

HELD: Res judicata has a dual aspect: first, bar by prior judgment which is
provided in Rule 39, Section 47(b) of the 1997 Rules of Civil Procedure and
second, conclusiveness of judgment which is provided in Section 47(c) of the
same Rule. In this instance, the judgment in the first case constitutes an absolute
bar to the second action. Otherwise put, the judgment or decree of the court of
competent jurisdiction on the merits concludes the litigation between the
parties, as well as their privies, and constitutes a bar to a new action or suit
involving the same cause of action before the same or any other tribunal. On the
other hand, the doctrine of conclusiveness of judgment provides that issues

3E Andaya Ching Espiritu Hefti Galvez Gammad Lainez Lui Madamba Nagera Narvasa
Ong Palangdao Rosales Sanchez Santos Satrain Tabo (2014-2015)

Labor Relations Case Digest - Atty. Joyrich Golangco

actually and directly resolved in a former suit cannot again be raised in any
future case between the same parties involving a different cause of action. Under
this doctrine, identity of causes of action is not required but merely identity of
issues.
The Secretary of Labor and Employment dismissed the first petition as it
was filed outside the 60-day freedom period. At that time therefore, the union has
no cause of action since they are not yet legally allowed to challenge openly and
formally the status of SMCGC-SUPER as the exclusive bargaining representative of
the bargaining unit. Such dismissal, however, has no bearing in the instant case
since the third petition for certification election was filed well within the 60-day
freedom period. Otherwise stated, there is no identity of causes of action to speak
of since in the first petition, the union has no cause of action while in the third, a
cause of action already exists for the union as they are now legally allowed to
challenge the status of SMCGC-SUPER as exclusive bargaining representative.
79. Gen Santos Coca-cola vs Coca Cola Bottlers

FACTS: Sometime in the late 1990s, CCBPI experienced a significant decline in


profitability due to the Asian economic crisis, decrease in sales, and tougher
competition. To curb the negative effects on the company, it implemented three
(3) waves of an Early Retirement Program.

Meanwhile, there was an inter-office memorandum sent to all of CCBPIs Plant


Human Resources Managers/Personnel Officers, including CCBPI Gen San
mandating them to put on hold "all requests for hiring to fill in vacancies in both
regular and temporary positions in [the] Head Office and in the Plants." Because
several employees availed of the early retirement program, vacancies were
created in some departments, prompting petitioner to negotiate with the Labor
Management Committee for filling up the vacancies with permanent employees.
No resolution was reached on the matter.
Faced with the "freeze hiring" directive, CCBPI Gen San engaged the services of
JLBP Services Corporation (JLBP), a company in the business of providing labor
and manpower services, including janitorial services, messengers, and office
workers to various private and government offices.

On January 21, 2002, petitioner filed with the National Conciliation and
Mediation Board (NCMB) a Notice of Strike on the ground of alleged unfair labor
practice committed by CCBPI Gen San for contracting-out services regularly
performed by union members ("union busting"). The parties failed to come to an
amicable settlement. On July 3, 2002, CCBPI filed a Petition for Assumption of
Jurisdiction with the Sec. of Labor and Employment which issued an Order
enjoining the threatened strike and certifying the dispute to the NLRC for
compulsory arbitration.

In a Resolution, the NLRC ruled that CCBPI was not guilty of unfair labor practice
for contracting out jobs to JLBP. The NLRC anchored its ruling on the validity of
the "Going-to-the-Market" (GTM) system implemented by the company, which
called for restructuring its selling and distribution system, leading to the closure
of certain sales offices and the elimination of conventional sales routes. The
NLRC held that petitioner failed to prove by substantial evidence that the system
was meant to curtail the right to self-organization of petitioners members.
Petitioner filed a motion for reconsideration, but was denied, hence, petitioner
filed a Petition for Certiorari before the CA.

The CA upheld the NLRCs finding that CCBPI was not guilty of unfair labor
practice. It held that the contract between CCBPI and JLBP did not amount to
labor-only contracting. It found that JLBP was an independent contractor and
that the decision to contract out jobs was a valid exercise of management
prerogative to meet exigent circumstances. On the other hand, petitioner failed
to adduce evidence to prove that contracting out of jobs by the company resulted
in the dismissal of petitioners members, prevented them from exercising their
right to self-organization, led to the Unions demise or that their group was
singled out by the company.
Its motion for reconsideration having been denied, petitioner now comes to this
Court seeking the reversal of the CA Decision.
ISSUES: WON there is ULP committed by respondent (contracting out
services/functions)

3E Andaya Ching Espiritu Hefti Galvez Gammad Lainez Lui Madamba Nagera Narvasa
Ong Palangdao Rosales Sanchez Santos Satrain Tabo (2014-2015)

Labor Relations Case Digest - Atty. Joyrich Golangco

HELD: The petition is bereft of merit. Hence, we deny the Petition. Under Rule 45
of Rules of Court, only questions of law may be raised in a Petition for Review on
Certiorari. The issues raised by petitioner reveals that they are questions of fact.
The issues raised, i.e., whether JLBP is an independent contractor, whether
CCBPIs contracting-out of jobs to JLBP amounted to unfair labor practice, and
whether such action was a valid exercise of management prerogative, call for a
re-examination of evidence, which is not within the ambit of this Courts
jurisdiction.
It is true that the NLRC erroneously concluded that the contracting- out of jobs in
CCBPI Gen San was due to the GTM system, which actually affected CCBPIs sales
and marketing departments, and had nothing to do with petitioners complaint.
However, this does not diminish the NLRCs finding that JLBP was a legitimate,
independent contractor and that CCBPI Gen San engaged the services of JLBP to
meet business exigencies created by the freeze-hiring directive of the CCBPI
Head Office.
On the other hand, the CA squarely addressed the issue of job contracting in its
assailed Decision and Resolution. The CA itself examined the facts and evidence
of the parties and found that, based on the evidence; CCBPI did not engage in
labor-only contracting and, therefore, was not guilty of unfair labor practice.

The NLRC found and the same was sustained by the CA that the companys
action to contract-out the services and functions performed by Union members
did not constitute unfair labor practice as this was not directed at the members
right to self-organization.
Unfair labor practice refers to "acts that violate the workers right to organize."
The prohibited acts are related to the workers right to self-organization and to
the observance of a CBA. Without that element, the acts, even if unfair, are not
unfair labor practices.

Both the NLRC and the CA found that petitioner was unable to prove its charge of
unfair labor practice. It was the Union that had the burden of adducing
substantial evidence to support its allegations of unfair labor practice, which
burden it failed to discharge.

80. UST Faculty Union vs UST

FACTS: On September 21, 1996, the University of Santo Tomas Faculty Union
(USTFU) wrote a letter to all its members informing them of a General Assembly
(GA) that was to be held on October 5, 1996. The letter contained an agenda for
the GA which included an election of officers. The then incumbent president of
the USTFU was Atty. Eduardo J. Marin o, Jr. On October 2, 1996, Fr. Rodel Aligan,
O.P., Secretary General of the UST, issued a Memorandum allowing the request of
the Faculty Clubs of the university to hold a convocation on October 4, 1996.
Members of the faculties of the university attended the convocation, including
members of the USTFU, without the participation of the members of the UST
administration. Also during the convocation, an election for the officers of the
USTFU was conducted by a group called the Reformist Alliance. Upon learning
that the convocation was intended to be an election, members of the USTFU
walked out. Meanwhile, an election was conducted among those present, and Gil
Gamilla and other faculty members (Gamilla Group) were elected as the
president and officers, respectively, of the union. Such election was
communicated to the UST administration in a letter. Thus, there were two (2)
groups claiming to be the USTFU: the Gamilla Group and the group led by Atty.
Marin o, Jr. (Marin o Group). The Marin o Group filed a complaint for ULP against
the UST with the Arbitration Branch of the NLRC, it also filed a complaint with
the Office of the Med-Arbiter of the DOLE, praying for the nullification of the
election of the Gamilla Group as officers of the USTFU.

On December 3, 1996, a CBA was entered into by the Gamilla Group and the UST.
The CBA superseded an existing CBA entered into by the UST and USTFU which
was intended for the period of June 1, 1993 to May 31, 1998.

Meanwhile med-arbiter issued a Resolution, declaring the election of the Gamilla


group as null and void and ordering that this group cease and desist from
performing the duties and responsibilities of USTFU officers. This Resolution was
appealed to the Director of the BLR the director issued a Resolution affirming the
Resolution of the med-arbiter. His Resolution was then appealed to this Court
which upheld the ruling of the BLR. USTFU filed a Manifestation with the
Arbitration Branch of the NLRC informing it of the Decision of the Court.
Thereafter, the Arbitration Branch of the NLRC issued a Decision dismissing the

3E Andaya Ching Espiritu Hefti Galvez Gammad Lainez Lui Madamba Nagera Narvasa
Ong Palangdao Rosales Sanchez Santos Satrain Tabo (2014-2015)

Labor Relations Case Digest - Atty. Joyrich Golangco

complaint for lack of merit for reason that the USTFU failed to prove UST
committed ULP.

The USTFU appealed the labor arbiters Decision to the NLRC which rendered a
Resolution affirming the Decision of the labor arbiter. USTFUs Motion for
Reconsideration to the NLRC was denied. The case was then elevated to the CA
which rendered the assailed Decision affirming the Resolutions of the NLRC.

ISSUES: Whether or not UST has committed unfair labor practices against USTFU

HELD: No.The general principle is that one who makes an allegation has the
burden of proving it. While there are exceptions to this general rule, in the case
of ULP, the alleging party has the burden of proving such ULP. In order to show
that the employer committed ULP under the Labor Code, substantial evidence is
required to support the claim. Substantial evidence has been defined as such
relevant evidence as a reasonable mind might accept as adequate to support a
conclusion. Such principle finds justification in the fact that ULP is punishable
with both civil and/or criminal sanctions.

In any event, it bears stressing that at the time of these events, the legitimacy of
the Gamilla Group as the valid officers and directors of the USTFU was already
submitted to the med-arbiter and no decision had yet been reached on the
matter. Having been shown evidence to support the legitimacy of the Gamilla
Group with no counter-evidence from the Marin o Group, UST had to recognize
the Gamilla Group and negotiate with it. Thus, the acts of UST in support of the
USTFU as the legitimate representative of the bargaining unit, albeit through the
Gamilla Group, cannot be considered as ULP.

Finally, petitioner claims that despite the ruling of this Honorable Court, the
University of Santo Tomas still entertains the interlopers whose claim to the
leadership of the USTFU has been rejected by the [DOLE] and the Highest
Tribunal. Petitioner, however, fails to enumerate such objectionable actions of
the UST. Again, petitioner fails to present substantial evidence in support of its
claim.

In sum, petitioner makes several allegations that UST committed ULP. The onus
probandi falls on the shoulders of petitioner to establish or substantiate such

claims by the requisite quantum of evidence. In labor cases as in other


administrative proceedings, substantial evidence or such relevant evidence as a
reasonable mind might accept as sufficient to support a conclusion is required. In
the petition at bar, petitioner miserably failed to adduce substantial evidence as
basis for the grant of relief.
81. PHIL. SKYLANDERS, INC. vs NLRC

FACTS: Philippine Skylanders Employees Association (PSEA), a local labor union


affiliated with the Philippine Association of Free Labor Unions (PAFLU) won in
the certification election conducted among the rank and file employees of
Philippine Skylanders, Inc. (PSI). Its rival union, PSEA-WATU protested the result
of the election before the Secretary of Labor. Pending settlement of the
controversy, PSEA disaffiliated with PAFLU which was adopted and signed by
about 92.5% of its total membership and affiliated itself with the National
Congress of Workers (NCW) then changed its name to PSEA-NCW. Thereafter,
PSEA-NCW entered into a collective bargaining agreement (CBA) with PSI which
was registered with the DOLE. PAFLU filed a complaint against PSI, Romulo (PSI
President) and Dakila (Personnel Manager) for interfering with its employees
union activities and unfair labor practice and later, against the elected officers of
the PSEA-PAFLU who remained officers of the PSEA-NCW as additional
respondents for allowing themselves to be manipulated and influenced by PSI.
PSI moved for the dismissal of the complaint on the ground that the issue of
disaffiliation was an inter-union conflict which is beyond the jurisdiction of the
Labor Arbiter while the PSEA-NCW asserted that since PSEA was no longer
affiliated with PAFLU, the latter had no personality to file the complaint. The
Labor Arbiter rendered a decision declaring PSEAs disaffiliation from PAFLU
invalid and holding PSI and PSEA officers guilty of unfair labor practice on the
ground that the company knowingly sanctioned and confederated with Dakila in
actively assisting a rival union which is a classic case of interference and since
PSEA-NCWs personality was not recognized, its collective bargaining agreement
with PSI was invalid and ordered PSI, PSEA and their officers to pay PAFLU
P150,000 in damages. On appeal, the NLRC upheld the decision of the Labor
Arbiter holding that since the election protest is still pending, PSEA could not
validly separate from PAFLU, federate with another and enter into a CBA with
PSI. Petitioners motions for reconsideration were denied.

3E Andaya Ching Espiritu Hefti Galvez Gammad Lainez Lui Madamba Nagera Narvasa
Ong Palangdao Rosales Sanchez Santos Satrain Tabo (2014-2015)

Labor Relations Case Digest - Atty. Joyrich Golangco


ISSUES:
1.
2.

HELD:
1.

2.

Whether or not PSEA may validly disaffiliate from PAFLU pending the
settlement of an election protest questioning its status as the sole and
exclusive bargaining agent of PSIs rank and file employees?
Whether or not petitioners are guilty of unfair labor practice?

YES. First, the Labor Arbiter acquired no jurisdiction to try and hear the
issue of disaffiliation as it is an inter-union conflict which jurisdiction
lies with the Bureau of Labor Relations and should have been dismissed
right away. Second, in jurisprudence, the Court upheld the right of local
unions to separate from their mother federation on the ground that as
separate and voluntary associations, local unions do not owe their
creation and existence to the national federation to which they are
affiliated but, instead, to the will of their members (Liberty Cotton Mills
Workers Union vs Liberty Cotton Mills, Inc., 66 SCRA 512). In fact, the
disaffiliation was supported by 92.5% of total membership of the local
union. As the local unions remain the basic units of association, they are
free to serve their own interests subject to the restraints imposed by the
constitution and by-laws of the national federation, and free also to
renounce the affiliation upon the terms laid down in the agreement
which brought such affiliation into existence. Third, there was no
showing of a claim by PAFLU that the local union was forbidden to
disaffiliate from the federation nor there were any conditions imposed
for a valid breakaway thus, the pendency of an election protest did not
constitute a bar to a valid disaffiliation.
NO. PAFLU instituted the complaint for unfair labor practice against the
wishes of the workers (111 out of 120 total membership) whose
interests it was supposed to protect. The mere act of disaffiliation did
not divest PSEA of its own personality; neither did it give PAFLU the
license to act independently of the local union. As an agent, PAFLU can
only act in representation of and in accordance with the interests of the

local union. PAFLUs complaint, having been filed by a party with no


legal personality failed to state a cause of action.

The Court reversed and set aside the Decision of the NLRC affirming the Decision
of the Labor Arbiter holding PSEA-NCW and PSI and its officers guilty of unfair
labor practice and ordering them to pay damages to PAFLU. No pronouncement
as to costs.
82. TROPICAL HUT EMPLOYEES UNION-CGW vs. TROPICAL HUT FOOD
MARKET, INC.

FACTS: January 2, 1968, the rank and file workers of the Tropical Hut Food
Market Incorporated, referred to herein as respondent company, organized a
local union called the Tropical Hut Employees Union, known for short as the
THEU, elected their officers, adopted their constitution and by-laws and
immediately sought affiliation with the National Association of Trade Unions
(NATU). On January 3, 1968, the NATU accepted the THEU application for
affiliation. Following such affiliation with NATU, Registration Certificate No.
5544-IP was issued by the Department of Labor in the name of the Tropical Hut
Employees Union NATU. It appears, however, that NATU itself as a labor
federation, was not registered with the Department of Labor. Collective
Bargaining Agreement was concluded between the parties on April 1, 1968, the
term of which expired on March 31, 1971.
Sec. 1. The COMPANY recognizes the UNION as the sole and exclusive collective
bargaining agent for all its workers and employees in all matters concerning
wages, hours of work, and other terms and conditions of employment.
Sec. 1 . . . Employees who are already members of the UNION at the time of the
signing of this Agreement or who become so thereafter shall be required to
maintain their membership therein as a conditionof continued employment. Xxx
Sec. 3Any employee who is expelled from the UNION for joining another
federation or forming another union, or who fails or refuses to maintain his
membership therein as required, . . . shall, upon written request of the UNION be
discharged by the COMPANY.

3E Andaya Ching Espiritu Hefti Galvez Gammad Lainez Lui Madamba Nagera Narvasa
Ong Palangdao Rosales Sanchez Santos Satrain Tabo (2014-2015)

Labor Relations Case Digest - Atty. Joyrich Golangco


May 21, 1971, respondent company and THEU-NATU entered into a new
Collective Bargaining Agreement which ended on March 31, 1974. This new CBA
incorporated the previous union-shop security clause and the attached check off
authorization form.

NATU received a letter dated December 15, 1973, jointly signed by the
incumbent officers of the local union informing the NATU that THEU was
disaffiliating from the NATU federation. Secretary of the THEU, Nemesio Barro,
made an announcement in an open letter to the general membership of the
THEU, concerning the latters disaffiliation from the NATU and its affiliation with
the Confederation of General Workers (CGW). The letter was passed around
among the members of the THEU-NATU, to which around one hundred and
thirtyseven (137) signatures appeared as having given their consent to and
acknowledgment of the decision to disaffiliate the THEU from the NATU. THEUCGW held its annual election of officers, with Jose Encinas elected as President.
On January 3, 1974, Encinas, in his capacity as THEU-CGW President, informed
the respondent company of the result of the elections. On January 9, 1974,
Pacifico Rosal, President of the Confederation of General Workers (CGW), wrote
a letter in behalf of complainant THEU-CGW to the respondent company
demanding the remittance of the union dues collected by the Tropical Hut Food
Mart, Incorporated to the THEU-CGW, but this was refused by the respondent
company.

Request was made by the NATU federation to the respondent company to


dismiss him (Encinas) in view of his violation of Section 3 of Article III of the
Collective Bargaining Agreement. Respondent company applied for clearance
with the Secretary of Labor to dismiss the other officers and members of THEUCGW. The company also suspended them effective that day. NLRC Case No. LR2521 was filed by THEU-CGW and individual complainants against private
respondents for unfair labor practices. THEU-CGW asked the employees to affirm
their membership. Some did not abide so they were informed that they will be
dismissed under the CBA.
The President/General Manager of respondent company, upon Dilags request,
suspended twenty four (24) workers on March 5, 1974,another thirty seven (37)

on March 8, 1974 and two (2) more on March 11, 1974, pending approval by the
Secretary of Labor of the application for their dismissal.
LA: Arbitrator Daniel Lucas issued an order dated March 21, 1974, holding that
the issues raised by the parties became moot and academic with the issuance of
NLRC Order dated February 25, 1974 in NLRC Case No. LR-2670, which directed
the holding of a certification election among the rank and file workers of the
respondent company between the THEU-NATU and THEUCGW. He also ordered:
a) the reinstatement of all complainants; b) for the respondent company to cease
and desist from committing further acts of dismissals without previous order
from the NLRC and for the complainant Tropical Hut Employees UNION-CGW to
file representation cases on a case to case basis during the freedom period
provided for by the existing CBA between the parties.

NLRC: Reversed the decision. Secretary of Labor rendered a decision affirming


the findings of the Commission.

ISSUES: 1) whether or not the petitioners failed to exhaust administrative


remedies when they immediately elevated the case to this Court without an
appeal having been made to the Office of the President; 2) whether or not the
disaffiliation of the local union from the national federation was valid; and 3)
whether or not the dismissal of petitioner employees resulting from their unions
disaffiliation for the mother federation was illegal and constituted unfair labor
practice on the part of respondent company and federation

HELD: 1) remedy of appeal from the Secretary of Labor to the Office of the
President is not a mandatory requirement before resort to courts can be had, but
an optional relief provided by law to parties seeking expeditious disposition of
their labor disputes. Failure to avail of such relief shall not in any way served as
an impediment to judicial intervention. And where the issue is lack of power or
arbitrary or improvident exercise thereof, decisions of the Secretary of Labor
may be questioned in a certiorari proceeding without prior appeal to the
President.
2) A local union, being a separate and voluntary association, is free to serve the
interest of all its members including the freedom to disaffiliate when

3E Andaya Ching Espiritu Hefti Galvez Gammad Lainez Lui Madamba Nagera Narvasa
Ong Palangdao Rosales Sanchez Santos Satrain Tabo (2014-2015)

Labor Relations Case Digest - Atty. Joyrich Golangco

circumstances warrant. This right is consistent with the constitutional guarantee


of freedom of association. All employees enjoy the right to self-organization and
to form and join labor organizations of their own choosing for the purpose of
collective bargaining and to engage in concerted activities for their mutual aid or
protection. This is a fundamental right of labor that derives its existence from the
Constitution.
The inclusion of the word NATU after the name of the local union THEU in the
registration with the Department of Labor is merely to stress that the THEU is
NATUs affiliate at the time of the registration. It does not mean that the said local
union cannot stand on its own. Neither can it be interpreted to mean that it
cannot pursue its own interests independently of the federation. A local union
owes its creation and continued existence to the will of its members and not to
the federation to which it belongs. When the local union withdrew from the old
federation to join a new federation, it was merely exercising its primary right to
labor organization for the effective enhancement and protection of common
interests. In the absence of enforceable provisions in the federations
constitution preventing disaffiliation of a local union a local may sever its
relationship with its parent. Nothing in the constitution and bylaws of THEU
NATU, prohibits the disaffiliation from NATU. Besides NATU is not even
recognized as a national federation.
3) When the THEU disaffiliated from its mother federation, the former did not
lose its legal personality as the bargaining union under the CBA. Moreover, the
union security clause embodied in the agreements cannot be used to justify the
dismissals meted to petitioners since it is not applicable to the circumstances
obtaining in this case. The CBA imposes dismissal only in case an employee is
expelled from the union for joining another federation or for forming another
union or who fails or refuses to maintain membership therein. The case at bar
does not involve the withdrawal of merely some employees from the union but of
the whole THEU itself from its federation. Clearly, since there is no violation of
the union security provision in the CBA, there was no sufficient ground to
terminate the employment of petitioners.
83 PUREFOODS CORPORATION Vs NAGKAKAISANG SAMAHANG
MANGGAGAWA NG PUREFOODS RANK-AND-FILE, ST. THOMAS FREE

WORKERS UNION, PUREFOODS GRANDPARENT FARM WORKERS UNION


and PUREFOODS UNIFIED LABOR ORGANIZATION

FACTS: On February 8, 1995, NAGSAMA-Purefoods manifested to petitioner


corporation its desire to re-negotiate the collective bargaining agreement (CBA)
then due to expire on the 28th of the said month. Together with its demands and
proposal, the organization submitted to the company its January 28, 1995
General Membership Resolution approving and supporting the unions affiliation
with PULO, adopting the draft CBA proposals of the federation, and authorizing a
negotiating panel which included among others a PULO representative. While
Purefoods formally acknowledged receipt of the unions proposals, it refused to
recognize PULO and its participation, even as a mere observer, in the negotiation.
Consequently, notwithstanding the PULO representatives non-involvement, the
negotiation of the terms of the CBA still resulted in a deadlock. A notice of strike
was then filed by NAGSAMA-Purefoods on May 15, 1995. In the subsequent
conciliation conference, the deadlock issues were settled except the matter of the
companys recognition of the unions affiliation with PULO.
In the meantime, STFWU and PGFWU also submitted their respective proposals
for CBA renewal, and their general membership resolutions which, among
others, affirmed the two organizations affiliation with PULO. Consistent with its
stance, Purefoods refused to negotiate with the unions should a PULO
representative be in the panel. The parties then agreed to postpone the
negotiations indefinitely.

On July 24, 1995, however, the petitioner company concluded a new CBA with
another union in its farm in Malvar, Batangas. Five days thereafter, or on July 29,
1995, at around 8:00 in the evening, four company employees facilitated the
transfer of around 23,000 chickens from the poultry farm in Sto. Tomas,
Batangas (where STFWU was the exclusive bargaining agent) to that in Malvar.
The following day, the regular rank-and-file workers in the Sto. Tomas farm were
refused entry in the company premises; and on July 31, 1995, 22 STFWU
members were terminated from employment. The farm manager, supervisors
and electrical workers of the Sto. Tomas farm, who were members of another
union, were nevertheless retained by the company in its employ.

3E Andaya Ching Espiritu Hefti Galvez Gammad Lainez Lui Madamba Nagera Narvasa
Ong Palangdao Rosales Sanchez Santos Satrain Tabo (2014-2015)

Labor Relations Case Digest - Atty. Joyrich Golangco

On August 17, 1999, the LA rendered a Decision dismissing the complaint, and
declaring that the company neither committed ULP nor illegally dismissed the
employees.

On appeal, the NLRC reversed the ruling of the LA, ordered the payment
of P500,000.00 as moral and exemplary damages and the reinstatement with full
backwages of the STFWU members. In its March 16, 2001 Decision (CA No.
022059-00), the labor commission ruled that the petitioner companys refusal to
recognize the labor organizations affiliation with PULO was unjustified
considering that the latter had been granted the status of a federation by the
Bureau of Labor Relations; and that this refusal constituted undue interference
in, and restraint on the exercise of the employees right to self-organization and
free collective bargaining.

In the assailed October 25, 2001 Resolution, the appellate court dismissed
outright the companys petition for certiorari on the ground that the verification
and certification of non-forum shopping was defective since no proof of authority
to act for and on behalf of the corporation was submitted by the corporations
senior vice-president who signed the same; thus, the petition could not be
deemed filed for and on behalf of the real party-in-interest.
ISSUES: Whether Purefoods committed unfair labor practice

HELD: YES, It is crystal clear that the closure of the Sto. Tomas farm was
made in bad faith. Badges of bad faith are evident from the following acts of
the petitioner: it unjustifiably refused to recognize the STFWUs and the
other unions affiliation with PULO; it concluded a new CBA with another
union in another farm during the agreed indefinite suspension of the
collective bargaining negotiations; it surreptitiously transferred and
continued its business in a less hostile environment; and it suddenly
terminated the STFWU members, but retained and brought the nonmembers to the Malvar farm. Petitioner presented no evidence to support
the contention that it was incurring losses or that the subject farms lease
agreement was pre-terminated. Ineluctably, the closure of the Sto. Tomas
farm circumvented the labor organizations right to collective bargaining
and violated the members right to security of tenure.

We deem as proper the award of moral and exemplary damages. We hold that
the sudden termination of the STFWU members is tainted with ULP because it
was done to interfere with, restrain or coerce employees in the exercise of their
right to self-organization.
As to the order of reinstatement, the Court modifies the same in that if it is no
longer feasible considering the length of time that the employees have been out
of petitioners employ, the company is ordered to pay the illegally dismissed
STFWU members separation pay equivalent to one (1) month pay, or one-half
(1/2) month pay for every year of service, whichever is higher.

WHEREFORE, premises considered, the petition for review on certiorari is


DENIED. The October 25, 2001 and the November 22, 2001 Resolutions of the
Court of Appeals in CA-G.R. SP No. 66871 are AFFIRMED. The March 16, 2001
Decision of the National Labor Relations Commission in NLRC-NCR-00-0705159-95 (CA No. 022059-00) is AFFIRMED with the MODIFICATION that
petitioner company is ordered to: (1) reinstate the illegally dismissed STFWU
members and pay them full backwages from the time of illegal termination up to
actual reinstatement; (2) if reinstatement is no longer feasible, pay the illegally
dismissed STFWU members their separation pay equivalent to one month pay, or
one-half month pay for every year of service, whichever is higher; and (3) pay
moral and exemplary damages in the aggregate amount of P500,000.00 to the
said illegally dismissed STFWU members.
84. De la sale University v. DLSUEA-NAFTEU

FACTS:
In 2001, Aliaza and other s (Aliaza
s gr oup) w ho w er e m em ber s of the
Respondent Union and obviously had conflict with the latter, filed a petition with
DOLE-NCR to conduct elections as the group alleged that there was a failure to
call for such election in the Union since 1985. This was opposed by the Union
declaring that an election transpired in 1987 BUT because of RA 6715 (an act
that extends protection to labor kasama na yung term of office) the officers
terms office were extended until 1992 by virtue of which, the General Assembly
(of the Union) affirmed the hold-over of tenure of the officers UNTIL a CBA is
executed. Such CBA was only executed in 2000. (kaya humihingi na ng election
yung Aliazas group).

3E Andaya Ching Espiritu Hefti Galvez Gammad Lainez Lui Madamba Nagera Narvasa
Ong Palangdao Rosales Sanchez Santos Satrain Tabo (2014-2015)

Labor Relations Case Digest - Atty. Joyrich Golangco

DOLE-NCR ruling: the hold-over of office by the incumbent officers had been
extinguished due to the CBA and ordered election with pre-election conference
under the supervision of its Labor Relations Division.

On June 26, 2003 pending appeal befor e the NLRC 2nd of the July 12, 2002
ruling was denied and it affirmed LAs ruling dismissing the ULP Complaint.
this order was appealed by the Union before the CA 1st Division.

In this r egar d, Aliazas gr oup r equested her ein Petitioner DLSU to put on escrow
all the Union dues, fees and other money considerations of concerned academic
personnel until election is held. Hence, DLSU advised the Union that it would
implement two actions to maintain its neutrality in the intra-union dispute:

CA 1st Division ruling: ruled that NLRC 2nd committed grave abuse of discretion
in ignoring the NLRC 3rds order and in effect SET ASIDE NLRC 2nds order of
dismissing ULP complaint --- this is the assailed order in this entire case before
the SC.

Respondent Union disr egar dedthe order and conducted election without prior
notice to DOLE. This prompted Aliazas group to file an Urgent motion for
Intervention that was granted 3 days before the supposed election.

1) Established Savings Account where dues and fees will be


deposited
and held in trust.

2) It discontinued normal relations with any group within the


Union including incumbent set of officers.

This m ove by DSLU pr om pted Union to file a com plaint for ULP claim ing that
DLSU unduly interfered with its internal affairs and discriminated against Union
members -- this was however dismissed by LA Pati of NLRC 2nd Div. on July 12,
2002 it was of course appealed before the same division (pending appeal)

Meanw hile, w hile such complaint was still pending before the 2nd Division of
NLRC, a Notice to Strike (March 2002) was filed before the Office of the Secretary
of Labor that was later on referred to NLRC 3rd Div. for Compulsory Arbitration
as a certified case. To this effect, 4 other complaints with ULP were filed by the
Union on behalf of its members against DLSU before the NLRC 3rd division and
subsequently filed a motion to subsume (or consolidate in other words) the
ULP complaint in NLRC 2nd (pending appeal) and the 4 other ULP complaints
before the NLRC 3rd -- this motion was granted and ordered the 2nd Division to
transmit the entire record to the 3rd Division -- hence DLSU filed an appeal before
CA 10th Division.

CA 10th Division ruling: reversed NLRC 3rds ruling in consolidating/subsuming


the ULP complaints because at that time the ULP complaint was already disposed
of or dismissed by the LA and such can no longer be the subject of arbitration.

ISSUES: WOR DLSU is guilty of ULP


HELD: YES

The acts of DLSU constitute INTERFERENCE as it w as done pending the


resolution of the intra-union dispute and to that effect the respondents officers
remained duly authorized to conduct union affairs. This is evidenced by a
Clarificatory Letter issued by the BLR Director that no void leadership in the
Union exists and the DOLE-NCR order should not be construed as an automatic
termination of the incumbent officers tenure that ends only until their successors
shall have been elected and qualified.
Fur therm or e, it is im por tant to note that the tw o acts by DLSU w er e adopted
despite of an existing CBA between the Union and the former and that the
University should have observed the terms and conditions thereof bearing on
union dues and representation.

Ruling on the conflicting or der s of the CA


s tw o divisions: CA 10ths ruling
became final and executory on July 11, 2004 while CA 1sts ruling was only
promulgated on Septmeber 16, 2005. Nauna kasi yung 10th Division so yung
order ng 1st cannot be effected. To do otherwise and transmit the records to the
3rd would create absurd consquences.
DLSU is liable for ULP.

85. MSMG-UWP VS RAMOS

3E Andaya Ching Espiritu Hefti Galvez Gammad Lainez Lui Madamba Nagera Narvasa
Ong Palangdao Rosales Sanchez Santos Satrain Tabo (2014-2015)

Labor Relations Case Digest - Atty. Joyrich Golangco

FACTS: The petitioner, Malayang Samahan ng mga Manggagawa sa M. Greenfield,


Inc., (B) (MSMG), hereinafter referred to as the "local union", is an affiliate of the
private respondent, United Lumber and General Workers of the Philippines
(ULGWP), referred to as the "federation".
On September 12, 1986, a local union election was held under the auspices of the
ULGWP wherein the herein petitioner, Beda Magdalena Villanueva, and the other
union officers were proclaimed as winners. Minutes of the said election were
duly filed with the Bureau of Labor Relations on September 29, 1986.
On March 21, 1987, a Petition for Impeachment was filed with the national
federation ULGWP by the defeated candidates in the aforementioned election.

On June 16, 1987, the federation conducted an audit of the local union funds. The
investigation did not yield any unfavorable result and the local union officers
were cleared of the charges of anomaly in the custody, handling and disposition
of the union funds.
The 14 defeated candidates filed a Petition for Impeachment/Expulsion of the
local union officers with the DOLE NCR on November 5, 1987, docketed as NCROD-M-11-780-87. However, the same was dismissed on March 2, 1988, by MedArbiter Renato Parungo for failure to substantiate the charges and to present
evidence in support of the allegations.

On April 17, 1988, the local union held a general membership meeting at the
Caruncho Complex in Pasig. Several union members failed to attend the meeting,
prompting the Executive Board to create a committee tasked to investigate the
non-attendance of several union members in the said assembly, pursuant to
Sections 4 and 5, Article V of the Constitution and By-Laws of the union, which
read: "Seksyon 4. Ang mga kinukusang hindi pagdalo o hindi paglahok sa lahat ng
hakbangin ng unyon ng sinumang kasapi o pinuno ay maaaring maging sanhi ng
pagtitiwalag o pagpapataw ng multa ng hindi hihigit sa P50.00 sa bawat araw na
nagkulang.
Seksyon 5. Ang sinumang dadalo na aalis ng hindi pa natatapos ang pulong ay
ituturing na pagliban at maparusahan ito ng alinsunod sa Article V, Seksyong 4

ng Saligang Batas na ito. Sino mang kasapi o pisyales na mahuli and dating sa
takdang oras ng di lalampas sa isang oras ay magmumulta ng P25.00 at
babawasin sa sahod sa pamamagitan ng salary deduction at higit sa isang oras ng
pagdating ng huli ay ituturing na pagliban.
On June 27, 1988, the local union wrote respondent company a letter requesting
it to deduct the union fines from the wages/salaries of those union members who
failed to attend the general membership meeting.

On July 11, 1988, the Federation wrote respondent company a letter advising the
latter not to deduct the fifty- peso fine. The imposition of P50.00 fine became the
subject of bitter disagreement between the Federation and the local union
culminating in the latters declaration of general autonomy from the former
through Resolution No. 10 passed by the local executive board and ratified by the
general membership on July 16, 1988.
In retaliation, the national federation asked respondent company to stop the
remittance of the local unions share in the education funds effective August
1988. This was objected to by the local union which demanded that the
education fund be remitted to it in full.

As early as November 21, 1988, the officers were expelled from the ULGWP. On
the same day, the federation advised respondent company of the expulsion of the
30 union officers and demanded their separation from employment pursuant to
the Union Security Clause in their collective bargaining agreement. This demand
was reiterated twice, through letters dated February 21 and March 4, 1989,
respectively, to respondent company. Thereafter, the Federation filed a Notice of
Strike with the National Conciliation and Mediation Board to compel the
company to effect the immediate termination of the expelled union officers.
On March 7, 1989, under the pressure of a threatened strike, respondent
company terminated the 30 union officers from employment, serving them
identical copies of the termination letter.

On that same day, the expelled union officers assigned in the first shift were
physically or bodily brought out of the company premises by the companys

3E Andaya Ching Espiritu Hefti Galvez Gammad Lainez Lui Madamba Nagera Narvasa
Ong Palangdao Rosales Sanchez Santos Satrain Tabo (2014-2015)

Labor Relations Case Digest - Atty. Joyrich Golangco

security guards. Likewise, those assigned to the second shift were not allowed to
report for work. This provoked some of the members of the local union to
demonstrate their protest for the dismissal of the said union officers. Some union
members left their work posts and walked out of the company premises. On the
other hand, the Federation, having achieved its objective, withdrew the Notice of
Strike filed with the NCMB.
On August 7, 1989, the petitioners filed a verified complaint with the Arbitration
Branch, National Capital Region, DOLE, Manila, charging private respondents of
unfair labor practice which consists of union busting, illegal dismissal, illegal
suspension, interference in union activities, discrimination, threats, intimidation,
coercion, violence, and oppression.
LA - termination to be valid in compliance with the union security clause of the
collective bargaining agreement. NLRC affirmed Las decision.
ISSUES: Whether the union officers were validly and legally terminated.

HELD: No. Although this Court has ruled that union security clauses embodied in
the collective bargaining agreement may be validly enforced and that dismissals
pursuant thereto may likewise be valid, this does not erode the fundamental
requirement of due process.
The reason behind the enforcement of union security clauses which is the
sanctity and inviolability of contracts cannot override ones right to due process.

In the case under scrutiny, petitioner union officers were expelled by the
federation for allegedly committing acts of disloyalty and/or inimical to the
interest of ULGWP and in violation of its Constitution and By-laws. Upon demand
of the federation, the company terminated the petitioners without conducting a
separate and independent investigation. Respondent company did not inquire
into the cause of the expulsion and whether or not the federation had sufficient
grounds to effect the same. Relying merely upon the federations allegations,
respondent company terminated petitioners from employment when a separate
inquiry could have revealed if the federation had acted arbitrarily and
capriciously in expelling the union officers. Respondent companys allegation
that petitioners were accorded due process is belied by the termination letters

received by the petitioners which state that the dismissal shall be immediately
effective.

As held in the aforecited case of Cario, "the right of an employee to be informed


of the charges against him and to reasonable opportunity to present his side in a
controversy with either the company or his own union is not wiped away by a
union security clause or a union shop clause in a collective bargaining agreement.
An employee is entitled to be protected not only from a company which
disregards his rights but also from his own union the leadership of which could
yield to the temptation of swift and arbitrary expulsion from membership and
mere dismissal from his job."

While respondent company may validly dismiss the employees expelled by the
union for disloyalty under the union security clause of the collective bargaining
agreement upon the recommendation by the union, this dismissal should not be
done hastily and summarily thereby eroding the employees right to due process,
self- organization and security of tenure. The enforcement of union security
clauses is authorized by law provided such enforcement is not characterized by
arbitrariness, and always with due process. Even on the assumption that the
federation had valid grounds to expel the union officers, due process requires
that these union officers be accorded a separate hearing by respondent company.
86. Alabang Country Club vs NLRC

FACTS: Petitioner Alabang Country Club, Inc. (Club) is a domestic non-profit


corporation with principal office at Country Club Drive, Ayala Alabang,
Muntinlupa City. Respondent Alabang Country Club Independent Employees
Union (Union) is the exclusive bargaining agent of the Clubs rank-and-file
employees. In April 1996, respondents Christopher Pizarro, Michael Braza, and
Nolasco Castueras were elected Union President, Vice-President, and Treasurer,
respectively.
On June 21, 1999, the Club and the Union entered into a Collective Bargaining
Agreement (CBA), which provided for a Union shop and maintenance of
membership shop.

The pertinent parts of the CBA included in Article II on Union Security read, as
follows:

3E Andaya Ching Espiritu Hefti Galvez Gammad Lainez Lui Madamba Nagera Narvasa
Ong Palangdao Rosales Sanchez Santos Satrain Tabo (2014-2015)

ARTICLE II

Labor Relations Case Digest - Atty. Joyrich Golangco

UNION SECURITY

SECTION 1. CONDITION OF EMPLOYMENT. All regular rank-and-file employees,


who are members or subsequently become members of the UNION shall
maintain their membership in good standing as a condition for their continued
employment by the CLUB during the lifetime of this Agreement or any extension
thereof.
SECTION 2. [COMPULSORY] UNION MEMBERSHIP FOR NEW REGULAR RANKAND-FILE EMPLOYEES

a)New regular rank-and-file employees of the Club shall join the UNION within
five (5) days from the date of their appointment as regular employees as a
condition for their continued employment during the lifetime of this Agreement,
otherwise, their failure to do so shall be a ground for dismissal from the CLUB
upon demand by the UNION.

b)The Club agrees to furnish the UNION the names of all new probationary and
regular employees covered by this Agreement not later than three (3) days from
the date of regular appointment showing the positions and dates of hiring.
xxxx

SECTION 4. TERMINATION UPON UNION DEMAND. Upon written demand of the


UNION and after observing due process, the Club shall dismiss a regular rankand-file employee on any of the following grounds:... (f)Malversation of union
funds;

Subsequently, in July 2001, an election was held and a new set of officers was
elected. Soon thereafter, the new officers conducted an audit of the Union funds.
They discovered some irregularly recorded entries, unaccounted expenses and
disbursements, and uncollected loans from the Union funds. The Union notified
respondents Pizarro, Braza, and Castueras of the audit results and asked them to
explain the discrepancies in writing.
Despite their explanations, respondents Pizarro, Braza, and Castueras were
expelled from the Union, and, on October 16, 2001, were furnished individual
letters of expulsion for malversation of Union funds.[6] Attached to the letters

were copies of the Panawagan ng mga Opisyales ng Unyon signed by 37 out of 63


Union members and officers, and a Board of Directors Resolution[7] expelling
them from the Union.

In a letter dated October 18, 2001, the Union, invoking the Security Clause of the
CBA, demanded that the Club dismiss respondents Pizarro, Braza, and Castueras
in view of their expulsion from the Union.The Club required the three
respondents to show cause in writing within 48 hours from notice why they
should not be dismissed. Pizarro and Castueras submitted their respective
written explanations on October 20, 2001, while Braza submitted his explanation
the following day.
During the last week of October 2001, the Clubs general manager called
respondents Pizarro, Braza, and Castueras for an informal conference inquiring
about the charges against them. Said respondents gave their explanation and
asserted that the Union funds allegedly malversed by them were even over the
total amount collected during their tenure as Union officersPhP 120,000 for
Braza, PhP 57,000 for Castueras, and PhP 10,840 for Pizarro, as against the total
collection from April 1996 to December 2001 of only PhP 102,000. They claimed
the charges are baseless. The general manager announced he would conduct a
formal investigation.
Nonetheless, after weighing the verbal and written explanations of the three
respondents, the Club concluded that said respondents failed to refute the
validity of their expulsion from the Union. Thus, it was constrained to terminate
the employment of said respondents. On December 26, 2001, said respondents
received their notices of termination from the Club.

Respondents Pizarro, Braza, and Castueras challenged their dismissal from the
Club in an illegal dismissal complaint. the Labor Arbiter ruled in favor of the
Club, and found that there was justifiable cause in terminating said respondents.
He dismissed the complaint for lack of merit.

On February 21, 2003, respondents Pizarro, Braza, and Castueras filed an Appeal
with the NLRC.
On February 26, 2004, the NLRC rendered a Decision granting the appeal.

3E Andaya Ching Espiritu Hefti Galvez Gammad Lainez Lui Madamba Nagera Narvasa
Ong Palangdao Rosales Sanchez Santos Satrain Tabo (2014-2015)

Labor Relations Case Digest - Atty. Joyrich Golangco

Aggrieved by the Decision and Resolution of the NLRC, the Club filed a Petition
for Certiorari with the Court of Appeals (CA).
The CA Upheld the NLRC Ruling.

ISSUE WON termination or dismissal from employment due to the enforcement


of the union security clause in the CBA is valid.

HELD: YES. In terminating the employment of an employee by enforcing the


union security clause, the employer needs only to determine and prove that: (1)
the union security clause is applicable; (2) the union is requesting for the
enforcement of the union security provision in the CBA; and (3) there is
sufficient evidence to support the unions decision to expel the employee from
the union. These requisites constitute just cause for terminating an employee
based on the CBAs union security provision.
The three respondents were expelled from and by the Union after due
investigation for acts of dishonesty and malversation of Union funds. In
accordance with the CBA, the Union properly requested the Club to enforce the
Union security provision in their CBA and terminate said respondents. Then, in
compliance with the Unions request, the Club reviewed the documents
submitted by the Union, requested said respondents to submit written
explanations, and thereafter afforded them reasonable opportunity to present
their side. After it had determined that there was sufficient evidence that said
respondents malversed Union funds, the Club dismissed them from their
employment conformably with Sec. 4 of the CBA.

In the above case, we pronounced that while the company, under a maintenance
of membership provision of the CBA, is bound to dismiss any employee expelled
by the union for disloyalty upon its written request, this undertaking should not
be done hastily and summarily. The company acts in bad faith in dismissing a
worker without giving him the benefit of a hearing. We cautioned in the same
case that the power to dismiss is a normal prerogative of the employer; however,
this power has a limitation. The employer is bound to exercise caution in
terminating the services of the employees especially so when it is made upon the
request of a labor union pursuant to the CBA. Dismissals must not be arbitrary
and capricious. Due process must be observed in dismissing employees because
the dismissal affects not only their positions but also their means of livelihood.

Employers should respect and protect the rights of their employees, which
include the right to labor.

87. Standard Charter Bank Employees union (UNION) vs Confessor as Sec of


labor (SOLE) and Standard Charter Bank (BANK)

FACTS: In 1990 the Bank and union signed a CBA. Prior to its expiration but
within the 60-days freedom period, the union initiated the negotiations. On Feb
18, 1993 the Union through its president Eddie Divinagracia sent a proposal
covering political and economic provision and included the name of the members
negotiating panel. On Feb 24 the bank responded with its counter proposal on
the non-economic provisions and said that it is in better position to propose it
counter-proposal for the economic provisions after the union presents its
justification and list of negotiating panel. They agreed to set the meeting to settle
the differences.
Before the commencement of the negotiation, Divinagracia suggested to the bank
that the bank lawyers should be excluded from the negotiating, the bank
acceded, meanwhile the bank suggested that Jose P. Umali the president of
National union of Bank employees (NUBE), the federation to which the UNION
was affiliated be excluded in the negotiating panel. However Umali was retained
as a member thereof.

March 12 1993 the day of the negotiations Diokno suggested that the
negotiations be kept a family affair the proposed non-economic benefits were
discussed first but there are still some provisions that the parties cant agree so
the parties agreed to place the notation Deferred/Deadlock

On May 18, 1993 the negotiation for economic union commenced, both parties
presented their proposal, Umali requested the bank to validate the Union
guestimates especially the figures for the rank and file. In succeeding meetings
Umali expressed his mild disappointment for the insufficiency of the banks
counter- proposal. He reminded the bank, how the UNION got what it wanted in
1987 and stated that if need be, the UNION would go through the same route to
get what it wanted.

3E Andaya Ching Espiritu Hefti Galvez Gammad Lainez Lui Madamba Nagera Narvasa
Ong Palangdao Rosales Sanchez Santos Satrain Tabo (2014-2015)

Labor Relations Case Digest - Atty. Joyrich Golangco

Upon the bank insistence, the parties agreed to tackle the economic package item
by item, the bank indicated which provisions it would accept, reject, retain and
agree to discuss but both parties still failed to reach an agreement on some of the
provisions of the CBA. The union declared a deadlock and filled a notice of strike
before the NCMB on june 21 1993. On the other hand the Bank filed a complaint
for ULP and damages before the NLRC alleging that the union violated its duty to
bargain, and demanded sky high economic demands indicative of blue sky
bargaining and that violated its no-strike no lock out clause by filing a notice of
strike before the NCMB and with the act of the union the bank suffered nominal
and actual damages and was forced to litigate and hire the services of a lawyer.
On July 21 1993, the SOLE assumed jurisdiction of the case pursuant to Art
263(g) LC and ordered the bank and Union to execute a CBA incorporating the
economic awards proposed by SOLE. And the ULP case filed by the bank and
union be dismissed on the ground that both parties failed to substantiate their
claim and stated the ULP charges would prosper only if shown to have directly
prejudiced the public interest. The union filed 2 motion for reconsideration but
both was denied

On March 22, 1994 the union and bank signed the CBA the wage increase was
effected and the signing bonus was released, but on April 28, 1994 the union filed
a petition for certiorari under rule 65 contending the following:
1.

2.
3.

The SOLE committed grave abuse of discretion amounting to lack of


jurisdiction when it found out that the BANK did not commit ULP when
it interfered with the UNION choice of negotiator and suggested that that
negotiation be limited as family affair and that damage to public
interest need not be present in order for ULP to prosper
That the SOLE failed to rule on the ULP charges arising from the banks
surface bargaining and contended that the bank merely went through
the motions of collective bargaining without the intent to reach an
agreement and the made bad faith proposals
The union also accused the bank of refusing to disclose material and
necessary date, even after the request to validate the guestimates were
made

In its comment the bank moved for dismissal on the ground that the union is
estopped considering that it signed the CBA and that it was the union who
committed ULP by engaging in blue-sky bargaining and isolated the no-strike- no
lock out clause of the existing CBA

ISSUES: whether or not the contention of the union is with merit


1. Issue on Interference
2. Duty to bargain
3. Failure to furnish information
4. Grave abuse of Discretion
5. Estopped
6. Blue-sky bargaining
HELD: NO.
Interference-The union cited
1.

2.

U.S postal service and Harley Davidson Motor Co. where NLRB held that
upon the employers refusal to engage in negotiations with the union for
CBA when the union include a person not an employee, or one who is a
member or an official of other labor org, such employer is engaged in
ULP
And that in the case of Insular life the court said that the test to
determine whether an employer interfered with and coerced employees
in the exercise of their right to self-organization is whether the employer
has engaged in conduct which it may reasonably said, tends to interfere
with the free exercise of employees right, it is not necessary that there
be direct evidence that any employee was in fact intimidated or coerced
by statements of treats of the employer if there is reasonable inference
that anti-union conduct of the employer does have an adverse effect on
self-organization and collective bargaining.

Court: Art 2 of International Labor Organization Convention NO.87 FREEDOM


OF ASSOCIATION AND PROTECTION OF THE RIGHT TO ORGANIZE which the
Philippines is a signatory provides:
1.

Workers and employers organization shall enjoy adequate protection


against any acts or interference by each other or each others agent or
members in their establishment, functioning or administration

3E Andaya Ching Espiritu Hefti Galvez Gammad Lainez Lui Madamba Nagera Narvasa
Ong Palangdao Rosales Sanchez Santos Satrain Tabo (2014-2015)

2.

Labor Relations Case Digest - Atty. Joyrich Golangco

In particular, acts which are designed to promote the establishment of


workers organization under the domination of employer or employers
organization or to support workers organization by financial or other
means, with the object of placing such organization under the control of
employers or employers organization within the meaning of this article

Such ILO convention is incorporated in our labor code in art 243, 248, 249 of
LC

248(a) the labor code considers it ULP when an employer interferes with,
restrain or coerce employees in the exercise of their right to self-organization or
the right to form association. The right to self-organization necessarily includes
the right to collective bargaining. Parenthetically, if an employer interferes with
the selection of negotiators or coerces a union to exclude from its panel a
representative of union it can be inferred that the employer adopted acts that
yield to adverse effects of the free exercise to the right of self-organization and is
considered ULP in LC but, substantial evidence is required to support such claim.
Substantial evidence- is such relevant evidence as a reasonable mind might
accept as adequate to support a conclusion. Such was not present in this case the
circumstances that occurred do not show that the suggestion made by DIOKNO
to exclude Umali is an anti-union conduct, especially considering that such was
undertaken previous to the commencement of the negotiation and
simultaneously with DIVINAGRACIAs suggestion that the bank lawyers be
excluded and also the records show that the negotiations pushed through despite
the inclusion of UMALI
Duty to Bargain- Surface bargaining

Surface Bargaining is defined as going through the motions of negotiating


without any legal intent to reach an agreement. It involves the question of
whether an employers conduct demonstrate an unwillingness to bargain in good
faith or is merely hard bargaining
The minutes of the meetings do not show that the Bank has intention of violating
its duty to bargain. Records shows that after the union sent its proposal to the
bank on Feb 17, the latter replied with its counter proposal on Feb 24 and
thereafter meeting were set for the settlement of their differences and that both

exchanged economic and non-economic proposals and counter proposals. Also it


was shown that the bank did not refuse to bargain collectively because they
accepted some of the demands while refused the others and even leaving open
other demands for future discussion and those demands where discussed in a
meeting called by the respondent themselves
It is to be emphasized that the duty to bargain does not compel either party to
agree to a proposal or require the making of a concession hence the parties
failure to agree did not amount to ULP.

Failure to furnish Information

While the refusal to furnish requested information is in itself ULP. The request
should be in writing, in which the union failed. Art 242 (c) Rights of legitimate
Labor Organization - To be furnished by the employer, upon written request,
with its annual audited financial statements, including the balance sheet and the
profit and loss statement, within thirty (30) calendar days from the date of
receipt of the request, after the union has been duly recognized by the employer
or certified as the sole and exclusive bargaining representative of the employees
in the bargaining unit, or within sixty (60) calendar days before the expiration of
the existing collective bargaining agreement, or during the collective bargaining
negotiation

The union, did not as the LC requires, send a written request for the issuance of a
copy of the data about the banks rank and file employees. Moreover, as alleged
by the union. The fact that the bank made use of the aforesaid guestimates,
amounts to a validation of the data it had used in its presentations.
Grave abuse of Discretion

No, certiorari may be availed of when a tribunal, board or officer exercising


judicial or QJ function has acted without or in excess of jurisdiction and there is
no appeal or any plain, speedy and adequate remedy in the ordinary course of
law. Grave abuse of discretion implies such capricious and whimsical exercise of
judgment as is equivalent to lack of jurisdiction, or where the power is exercised
in an arbitrary or despotic manner by reason of passion or personal hostility
which must be so patent and gross as to amount to an invasion of positive duty

3E Andaya Ching Espiritu Hefti Galvez Gammad Lainez Lui Madamba Nagera Narvasa
Ong Palangdao Rosales Sanchez Santos Satrain Tabo (2014-2015)

Labor Relations Case Digest - Atty. Joyrich Golangco

or to a virtual refusal to perform the duty enjoined or to act all in contemplation


of the law. Mere abuse of discretion is not enough

While it is true that a showing of prejudice to public interest is not a requisite for
ULP to prosper, it cannot be said that the SOLE acted in capricious and whimsical
exercise of judgment, equivalent to lack of jurisdiction or excess of.
Estoppel

The approval of the CBA and the release of the signing bonus do not necessarily
mean that the UNION waived its ULP claim against the bank. After all, the
conclusions of the CBA was included in the order of the SOLE, while the signing
bonus was included in the CBA itself, Moreover the Union twice filed an MR
Blue-Sky Bargainin

The Union is not guilty of Blue-sky bargaining or making exaggerated or


unreasonable proposals. The minutes of the meeting shows that the union based
its economic proposals on data of rank and file employees and the prevailing
economic benefits received by bank employees from other foreign banks doing
business in the Philippines (same as Standard charter) and other branches of the
bank in the asian region.
88. GMC v. CA

FACTS: In its two plants located at Cebu City and Lapu-Lapu City, petitioner GMC
employed 190 workers. They were all members of private respondent General
Milling Corporation Independent Labor Union. On April 28, 1989, GMC and the
union concluded a CBA which included the issue of representation effective for a
term of three years. The day before the expiration of the CBA, the union sent GMC
a proposed CBA, with a request that a counter-proposal be submitted within 10
days. However, GMC had received collective and individual letters from workers
who stated that they had withdrawn from their union membership, on grounds
of religious affiliation and personal differences. Believing that the union no
longer had standing to negotiate a CBA, GMC did not send any counter-proposal.
GMC wrote a letter to the unions officers, stating that it felt there was no
basis to negotiate with a union which no longer existed. In answer, the union
officers wrote a letter disclaiming any massive disaffiliation or resignation from

the union and submitted a manifesto, signed by its members, stating that they
had not withdrawn from the union.

Then GMC dismissed Marcia Tumbiga, a union member, on the ground of


incompetence. The union protested and requested GMC to submit the matter to
the grievance procedure provided in the CBA. But GMC refused. Thus, the union
filed a complaint against GMC with the NLRC, Arbitration Division, Cebu City. The
complaint alleged ULP on the part of GMC for: (1) refusal to bargain collectively;
(2) interference with the right to self-organization; and (3) discrimination.

LA dismissed the case with the recommendation that a petition for


certification election be held to determine if the union still enjoyed the support of
the workers. However, NLRC ruled in favor of the Union but with respect to the
unions claim of discrimination, the NLRC found the claim unsupported by
substantial evidence.
ISSUES: WON GMC committed ULP. YES!

HELD: 1. Refusal to Bargain collectively: It was obvious that GMC had no valid
reason to refuse to negotiate in good faith with the union. For refusing to send a
counter-proposal to the union and to bargain anew on the economic terms of the
CBA, the company committed an unfair labor practice under Article 248 of the
Labor Code by violating the duty to bargain collectively as prescribed by this
Code; Under the LC, the duty to bargain collectively means the performance of a
mutual obligation to meet and convene promptly and expeditiously in good faith
for the purpose of negotiating an agreement
GMCs failure to make a timely reply to the proposals presented by the
union is indicative of its utter lack of interest in bargaining with the union. Its
excuse that it felt the union no longer represented the workers, was mainly
dilatory as it turned out to be utterly baseless.
Its refusal to make a counter-proposal to the unions proposal for CBA
negotiation is an indication of its bad faith. Where the employer did not even
bother to submit an answer to the bargaining proposals of the union, there is a
clear evasion of the duty to bargain collectively, making it liable for ULP.
2. Interference: The letters by 13 union members signifying their resignation
from the union clearly indicated that GMC exerted pressure on its
employees. The records show that GMC presented these letters to prove that the

3E Andaya Ching Espiritu Hefti Galvez Gammad Lainez Lui Madamba Nagera Narvasa
Ong Palangdao Rosales Sanchez Santos Satrain Tabo (2014-2015)

Labor Relations Case Digest - Atty. Joyrich Golangco

union no longer enjoyed the support of the workers. The fact that the
resignations of the union members occurred during the pendency of the case
before the labor arbiter shows GMCs desperate attempts to cast doubt on the
legitimate status of the union. The ill-timed letters of resignation from the union
members indicate that GMC had interfered with the right of its employees to selforganization.
89. Hacienda Fatima vs NFSW (Natl Federation of Sugar Cane WorkersFood and General Trade)

FACTS: Petitioner did not look with favor those workers having organized
themselves into a union. Thus, when NSFW was certified as the collective
bargaining representative in the certification elections, petitioner under the
pretext that the result was on appeal, refused to sit down with the union for the
purpose of entering into a collective bargaining agreement. Moreover, the
workers including respondents herein were not given work for more than one
month. In protest, NSFW staged a strike which was however settled upon the
signing of a Memorandum of Agreement.

However, alleging that NSFW failed to load the fifteen wagons as stated
in the Memorandum of Agreement petitioner reneged on its commitment to sit
down and bargain collectively. Instead, petitioner employed all means including
the use of private armed guards to prevent the organizers from entering the
premises. Petitioner then did not give respondents any more work assignments,
which prompted the latter to stage a strike which was also grounded on the
dismissal of union officers and members. But due to the conciliation efforts by
the DOLE, another Memorandum of Agreement was signed by the parties.
Pursuant thereto, the parties subsequently met. When petitioners again reneged
on its commitment, respondents filed the present complaint. Petitioner on its
part alleged that the respondents refused to work and they are being choosy in
the kind of work they will perform. Labor Arbiter held that petitioner was not
guilty of unfair labor practice. NLRC reversed the decision, finding the petitioner
guilty of ULP which was affirmed by CA.
ISSUES: Whether or not petitioner is guilty of Unfair labor Practice?

HELD:YES. Indeed, from respondents refusal to bargain, to their acts of


economic inducements resulting in the promotion of those who withdrew from
the union, the use of armed guards to prevent the organizers to come in, and the
dismissal of union officials and members, one cannot but conclude that
respondents did not want a union in their haciendaa clear interference in the
right of the workers to self-organization.
90. St. John Colleges, Inc., vs. St John Academy Faculty Employees and
Employees Union

FACTS: The Collective Bargaining Agreement (CBA) between SJCI (petitioner)


and the Union was set to expire on May 31, 1997. During the ensuing collective
bargaining negotiations, SJCI rejected all the proposals of the Union for an
increase in workers benefits. This resulted to a bargaining deadlock which led
to the holding of a valid strike by the Union on November 10, 1997. In order to
end the strike, on November 27, 1997, SJCI and the Union, through the efforts of
the National Conciliation and Mediation Board (NCMB), agreed to refer the labor
dispute to the Secretary of Labor and Employment (SOLE) for assumption of
jurisdiction. Both parties came to some agreements and after which, the strike
ended and classes resumed. Subsequently, the SOLE issued an Order dated
January 19, 1998 assuming jurisdiction over the labor dispute pursuant to Article
263 of the Labor Code. The parties were required to submit their respective
position papers within ten (10) days from receipt of said Order. Pending
resolution of the labor dispute before the SOLE, the Board of Directors of SJCI
approved on February 22, 1998 a resolution recommending the closure of the
high school which was approved by the stockholders on even date. Thereafter,
SJCI informed the Department of Labor and Employment (DOLE), Department of
Education, Culture and Sports (DECS), parents, students and the Union of the
impending closure of the high school which took effect on March 31, 1998.
Subsequently, some teaching and non-teaching personnel of the high school
agreed to the closure. On April 2, 1998, SJCI informed the DOLE that as of March
31, 1998, 51 employees had received their separation compensation package
while 25 employees refused to accept the same. On May 4, 1998, the
aforementioned 25 employees conducted a protest action within the perimeter
of the high school. The Union filed a notice of strike with the NCMB only on May
7, 1998. SJCI filed a petition to declare the strike illegal before the NLRC. It

3E Andaya Ching Espiritu Hefti Galvez Gammad Lainez Lui Madamba Nagera Narvasa
Ong Palangdao Rosales Sanchez Santos Satrain Tabo (2014-2015)

Labor Relations Case Digest - Atty. Joyrich Golangco

claimed that the strike was conducted in violation of the procedural


requirements for holding a valid strike under the strike under the Labor Code.
The 25 employees filed a complaint for unfair labor practice (ULP), illegal
dismissal and non-payment of monetary benefits against SJCI before the NLRC.
The Union members alleged that the closure of the high school was done in bad
faith in order to get rid of the Union and render useless any decision of the SOLE
on the CBA deadlocked issues. These two cases were then consolidated. On
January 8, 1999, Labor Arbiter Antonio R. Macam rendered a Decision
dismissing the Union's complaint for ULP and illegal dismissal while
granting SJCI's petition to declare the strike illegal coupled with a
declaration of loss of employment status of the 25 Union members involved
in the strike. Meanwhile, in the proceedings before the SOLE, the Union filed a
manifestation to maintain the status quo on March 30, 1998 praying that SJCI be
enjoined from closing the high school. It claimed that the decision of SJCI to close
the high school violated the SOLE's assumption order and the agreement of the
parties not to take any retaliatory action against the other. After the favorable
decision of the Labor Arbiter, SJCI resolved to reopen the high school for school
year 1999-2000. However, it did not restore the high school teaching and nonteaching employees it earlier terminated. That same school year SJCI opened an
elementary and college department. On July 23, 1999, the SOLE denied SJCI's
motions to dismiss and certified the CBA deadlock case to the NLRC. It ordered
the consolidation of the CBA deadlock case with the ULP, illegal dismissal, and
illegal strike cases which were then pending appeal before the NLRC. On June
28, 2002, the NLRC rendered judgment reversing the decision of the Labor
Arbiter. It found SJCI guilty of ULP and illegal dismissal and ordered it to
reinstate the 25 employees to their former positions without loss of
seniority rights and other benefits, and with full backwages. It also required
SJCI to pay moral and exemplary damages, attorney's fees, and two (2) months
summer/vacation pay. Moreover, it ruled that the mass actions conducted by the
25 employees on May 4, 1998 could not be considered as a strike since, by then,
the employer-employee relationship had already been terminated due to the
closure of the high school. Finally, it dismissed, without prejudice, the certified
case on the CBA deadlocked issues for failure of the parties to substantiate their
respective positions. The Court of Appeals affirmed the decision of the NLRC.
Under Article 283 of the Labor Code, the following requisites must concur for a
valid closure of the business: (1) serving a written notice on the workers at least

one (1) month before the intended date thereof; (2) serving a notice with the
DOLE one month before the taking effect of the closure; (3) payment of
separation pay equivalent to one (1) month or at least one half (1/2) month pay
for every year of service, whichever is higher, with a fraction of at least six (6)
months to be considered as a whole year; and (4) cessation of the operation must
be bona fide. 12 It is not disputed that the first two requisites were satisfied. The
third requisite would have been satisfied were it not for the refusal of the herein
private respondents to accept the separation compensation package. The
instant case, thus, revolves around the fourth requisite, i.e., whether SJCI
closed the high school in good faith.
ISSUES: W/N the closure of the high school was done in good faith and is not
liable for ULP and illegal dismissal.

HELD: NO. Whether or not the closure of the high school was done in good faith
is a question of fact and is not reviewable by this Court in a petition for review on
certiorari saves for exceptional circumstances. In fine, the finding of the NLRC,
which was affirmed by the Court of Appeals, that SJCI closed the high school in
bad faith is supported by substantial evidence and is, thus, binding on this Court.
Consequently, SJCI is liable for ULP and illegal dismissal. The determination of
whether SJCI acted in bad faith depends on the particular facts as established by
the evidence on record. Bad faith is, after all, an inference which must be drawn
from the peculiar circumstances of a case. The two decisive factors in
determining whether SJCI acted in bad faith are (1) the timing of, and reasons for
the closure of the high school, and (2) the timing of, and the reasons for the
subsequent opening of a college and elementary department, and, ultimately, the
reopening of the high school department by SJCI after only one year from its
closure. After the SOLE assumed jurisdiction, it required the parties to submit
their respective position papers. However, instead of filing its position paper,
SJCI closed its high school, allegedly because of the "irreconcilable differences
between the school management and the Academy's Union particularly the
safety of our students and the financial aspect of the ongoing CBA negotiations."
Thereafter, SJCI moved to dismiss the pending labor dispute with the SOLE
contending that it had become moot because of the closure. Nevertheless, a year
after said closure, SJCI reopened its high school and did not rehire the previously
terminated employees. Under these circumstances, it is not difficult to

3E Andaya Ching Espiritu Hefti Galvez Gammad Lainez Lui Madamba Nagera Narvasa
Ong Palangdao Rosales Sanchez Santos Satrain Tabo (2014-2015)

Labor Relations Case Digest - Atty. Joyrich Golangco

discern that the closure was done to defeat the parties' agreement to refer
the labor dispute to the SOLE; to unilaterally end the bargaining deadlock;
to render nugatory any decision of the SOLE; and to circumvent the Union's
right to collective bargaining and its members' right to security of tenure.
By admitting that the closure was due to irreconcilable differences
between the Union and school management, specifically, the financial
aspect of the ongoing CBA negotiations, SJCI in effect admitted that it
wanted to end the bargaining deadlock and eliminate the problem of
dealing with the demands of the Union. This is precisely what the Labor
Code abhors and punishes as unfair labor practice since the net effect is to
defeat the Union's right to collective bargaining. With respect to SJCI's claim
that during the 1997 CBA negotiations the Union made illegal demands because
they exceeded the 70% limitation set by R.A. No. 6728, it is important to note
that the alleged illegality or excessiveness of the Union's demands were the
issues to be resolved by the SOLE after the parties agreed to refer the said labor
dispute to the latter for assumption of jurisdiction. As previously mentioned, the
SOLE certified the case to the NLRC, which on June 28, 2002, rendered a decision
finding that there was insufficient evidence to determine the reasonableness of
the Union's proposals. The
NLRC found that SJCI failed to establish that the Union's demands were illegal or
excessive. A review of the records clearly shows that the Union submitted a
position paper detailing its demands in actual monetary terms. However, SJCI
failed to establish how and why these demands were in excess of the limitation
set by R.A. 6728. Up to this point in the proceedings, it has merely relied on its
self-serving statements that the Union's demands were illegal and excessive.
There is no basis, therefore, to hold that the Union ever made illegal or excessive
demands. At any rate, even assuming that the Union's demands were illegal or
excessive, the important and crucial point is that these alleged illegal or excessive
demands did not justify the closure of the high school and do not, in any way,
establish SJCI's good faith. The employer cannot unilaterally close its
establishment on the pretext that the demands of its employees are excessive.
Furthermore, if SJCI was after the interests of the students, then it should not
have closed the school because the parents and the students were vehemently
opposed to the same. To recapitulate, there is insufficient evidence to hold that
the safety and well-being of the students were endangered and/or compromised,
and that the Union was responsible therefor. Even assuming arguendo that the

students' safety and well-being were jeopardized by the said protest actions, the
alleged threat to the students' safety and well-being had long ceased by the time
the high school was closed. Moreover, the parents were vehemently opposed to
the closure of the school because there was no basis to claim that the students'
safety was at risk. Taken together, these circumstances lead to the inescapable
conclusion that SJCI merely used the alleged safety and well-being of the students
as a subterfuge to justify its actions. With respect to the mass action of the Union
members on May 4, 1998, I agree with the majority that, as ruled by the NLRC
and the CA, the Union members did not conduct an illegal strike, since by then,
the employee-employer relationship had ceased. Based on the records, petitioner
admitted that the school did not really operate or accept enrollees for the SY
1998-1999; 39 neither was it shown that it possessed the requisite government
permits when the alleged strike was held. Consequently, the 25 Union members
are entitled to the same separation package, i.e., separation pay plus two-month
summer/vacation pay and 13th month pay, given to other dismissed employees.
Although there is no specific law or rule as regards the payment of
summer/vacation pay, justice and equity demands that the Union members who
had honestly fought for their cause must be granted equal privileges given to
those who no longer questioned the validity of the school's closure.
91. Central Azucarera De Bias Employees Union NFL v. Central Azucarera
De Bias Inc

FACTS: Respondent Central Azucarera De Bais, Inc. (CAB) is a corporation duly


organized and existing under the laws of the Philippines while CABEU-NFL is a
duly registered labor union and a certified bargaining agent of the CAB rank-andfile employees, represented by its President, Pablito Saguran
On January 19, 2004, CABEU-NFL sent CAB a proposed Collective Bargaining
Agreement (CBA) seeking increases in the daily wage and vacation and sick leave
benefits of the monthly employees and the grant of leave benefits and 13th
month pay to seasonal workers. CAB did not agree to grant additional and
separate Christmas bonuses.
On May 21, 2004, CAB received an Amended Union Proposal sent by CABEU-NFL
r however, CAB maintained its position on the matter. Thus, the collective
bargaining negotiations resulted in a deadlock.

3E Andaya Ching Espiritu Hefti Galvez Gammad Lainez Lui Madamba Nagera Narvasa
Ong Palangdao Rosales Sanchez Santos Satrain Tabo (2014-2015)

Labor Relations Case Digest - Atty. Joyrich Golangco


CABEU-NFL filed a Notice of Strike with the National Conciliation and Mediation
Board (NCMB). The NCMB then assumed conciliatory-mediation jurisdiction and
summoned the parties to conciliation conferences.
In its June 2, 2005 Letter sent to CAB, CABEU-NFL requested copies of CABs
annual financial statements from 2001 to 2004 and asked for the resumption of
conciliation meetings.
CAB replied through its June 14, 2005 Letter to NCMB Regional Director of
Dumaguete City Isidro Cepeda, which reads:

At the outset, it observed that the letter signed by Mr. Pablito Saguran who is no
longer an employee of the Central for he was one of those lawfully terminated due
to an authorized cause x x
More importantly, the declared purpose of the requested conciliation meeting has
already been rendered moot and academic because:
(1) the Union which Mr. Saguran purportedly represents has already lost its
majority status by reason of the disauthorization and withdrawal of support; and
(2) the workers themselves, acting as principal, after disauthorizing the previous
agent CABEU-NFL have organized themselves into a new Union known as Central
Azucarera de Bais Employees Labor Association (CABELA)
Reacting from the letter-response of CAB, CABEU-NFL filed a Complaint for
Unfair Labor Practice for the formers refusal to bargain with it.

LA:In the case at bar, the record shows that respondent CAB replied to the
complainant Unions CBA proposals with its own set of counterproposals. Record
further shows that respondent CAB participated in a series of CBA negotiations
conducted by the parties at the plant level as well as in the
conciliation/mediation proceedings conducted by the NCMB. At this juncture it
cannot be said, therefore, that respondent CAB refused to negotiate or that it
violated its duty to bargain collectively. We do not agree that respondent CAB
committed an unfair labor practice act in questioning the capacity of Mr. Pablito
Saguran to represent complainant union in the CBA negotiations because he was

no longer an employee of respondent.

NLRC: On appeal, the NLRC in its July 18, 2007 Decision reversed the LAs
decision and found CAB guilty of unfair labor practice. Respondents failure to act
on the request of the complainant to resume negotiation for no valid reason
constitutes unfair labor practice. Consequently, the proposed CBA as amended
should be imposed to respondent.

CA: Reversed the NLRC's decision. In the case at bar, private respondent CABEUNFL failed in its burden of proof to present substantial evidence to support the
allegation of unfair labor practice. The assailed Decision and Resolution of public
respondent referred merely to two (2) circumstances which allegedly support
the conclusion that the presumption of good faith had been rebutted and that
bad faith was extant in petitioners actions. To recall, these circumstances are: (a)
the execution of a supposed collective bargaining agreement with another labor
union, CABELA; and (b) CABs sending of the letter dated June 14, 2005 to NCMB
seeking to call off the collective bargaining negotiations. These, however, are not
enough to ascribe the very serious offense of unfair labor practice upon
petitioner.
ISSUES: Whether or not respondent CAB was guilty of acts constituting unfair
labor practice by refusing to bargain collectively.

HELD: NO, The concept of unfair labor practice is provided in Article 247 of the
Labor Code which states:

Article 247. Concept of Unfair Labor Practice and Procedure for Prosecution
thereof. -- Unfair labor practices violate the constitutional right of workers and
employees to self-organization, are inimical to the legitimate interests of both labor
and management, including their right to bargain collectively and otherwise deal
with each other in an atmosphere of freedom and mutual respect, disrupt industrial
peace and hinder the promotion of healthy and stable labor-management relations.

The Labor Code, likewise, enumerates the acts constituting unfair labor practices of
the employer, thus:

3E Andaya Ching Espiritu Hefti Galvez Gammad Lainez Lui Madamba Nagera Narvasa
Ong Palangdao Rosales Sanchez Santos Satrain Tabo (2014-2015)

Labor Relations Case Digest - Atty. Joyrich Golangco


Article 248. Unfair Labor Practices of Employers.It shall be unlawful for an
employer to commit any of the following unfair labor practice:
xxx

xxx

xxx

UNION OF FILIPRO EMPLOYEES DRUG, FOOD AND ALLIED INDUSTRIES


UNIONS KILUSANG MAYO UNO (UFE-DFA-KMU), petitioner, vs. NESTL
PHILIPPINES, INCORPORATED, respondent.

[G.R. Nos. 158944-45. March 3, 2008.]

(g) To violate the duty to bargain collectively as prescribed by this Code.


For a charge of unfair labor practice to prosper, it must be shown that CAB was
motivated by ill will, bad faith, or fraud, or was oppressive to labor, or done in a
manner contrary to morals, good customs, or public policy, and, of course, that
social humiliation, wounded feelings or grave anxiety resulted, in suspending
negotiations with CABEU-NFL. Notably, CAB believed that CABEU-NFL was no
longer the representative of the workers. It just wanted to foster industrial peace
by bowing to the wishes of the overwhelming majority of its rank and file
workers and by negotiating and concluding in good faith a CBA with CABELA.
Such actions of CAB are nowhere tantamount to anti-unionism, the evil sought to
be punished in cases of unfair labor practices.
Furthermore, basic is the principle that good faith is presumed and he who
alleges bad faith has the duty to prove the same. By imputing bad faith to the
actuations of CAB, CABEU-NFL has the burden of proof to present substantial
evidence to support the allegation of unfair labor practice. Apparently, CABEUNFL refers only to the circumstances mentioned in the letter-response, namely,
the execution of the supposed CBA between CAB and CABELA and the request to
suspend the negotiations, to conclude that bad faith attended CABs actions. The
Court is of the view that CABEU-NFL, in simply relying on the said letterresponse, failed to substantiate its claim of unfair labor practice to rebut the
presumption of good faith.
Moreover, as correctly determined by the LA, the filing of the complaint for
unfair labor practice was premature inasmuch as the issue of collective
bargaining is still pending before the NCMB.
92. UFE-DFA-KMU VS NESTLE

[G.R. Nos. 158930-31. March 3, 2008.]

NESTL PHILIPPINES, INCORPORATED, petitioner, vs. UNION OF FILIPRO


EMPLOYEES DRUG, FOOD AND ALLIED INDUSTRIES UNIONS KILUSANG
MAYO UNO (UFE-DFA-KMU), respondent.

FACTS: UFE-DFA-KMU was the sole and exclusive bargaining agent of the rankand-file employees of Nestl. As the existing (CBA) between Nestl and UFE-DFAKMU was about to end, Presidents of UFE-DFA-KMU informed Nestl of their
intent to "open [our] new Collective Bargaining Negotiation. Nestl informed
them that it was also preparing its own counter-proposal and proposed ground
rules to govern the impending conduct of the CBA negotiations.
In another letter to the UFE-DFA-KMU, Nestl reiterated its stance that
"unilateral grants, one-time company grants, company-initiated policies and
programs, which include, but are not limited to the Retirement Plan, Incidental
Straight Duty Pay and Calling Pay Premium, are by their very nature not proper
subjects of CBA negotiations and therefore shall be excluded therefrom."

Nestl requested the (NCMB) to conduct preventive mediation proceedings


between it and UFE-DFA-KMU owing to an alleged impasse in said dialogue.

Conciliation proceedings proved ineffective. UFE-DFA-KMU filed a Notice of


Strike with the NCMB, complaining of a bargaining deadlock pertaining to
economic issues. Another Notice of Strike was filed by the union, this time
predicated on Nestl's alleged unfair labor practices, that is, bargaining in bad
faith by setting pre-conditions in the ground rules and/or refusing to include the
issue of the Retirement Plan in the CBA negotiations.

Prior to holding the strike, Nestl filed with the DOLE a Petition for Assumption
of Jurisdiction, praying for the Secretary of the DOLE to assume jurisdiction over
the current labor dispute in order to effectively enjoin any impending strike by
the members of the UFE-DFA-KMU.

3E Andaya Ching Espiritu Hefti Galvez Gammad Lainez Lui Madamba Nagera Narvasa
Ong Palangdao Rosales Sanchez Santos Satrain Tabo (2014-2015)

Labor Relations Case Digest - Atty. Joyrich Golangco

The Secretary issued an Order assuming jurisdiction over the subject labor
dispute. The fallo of said Order states that:

and the evidence on record. The union posited that the Sec. "could only assume
jurisdiction over the issues mentioned in the notice of strike subject of the
current dispute," and that the Amended Notice of Strike it filed did not cite, as
one of the grounds, the CBA deadlock.

The parties are further directed to meet and convene for the discussion of the
union proposals and company counter-proposals before the (NCMB) who is
hereby designated as the delegate/facilitator of this Office for this purpose. xxx If
no settlement of all the issues is reached, this Office shall thereafter define the
outstanding issues and order the filing of position papers for a ruling on the
merits.

On March 8, 2002, the Sec. denied the motion for reconsideration of UFE-DFAKMU.

Accordingly, any strike or lockout is hereby enjoined. The parties are directed to
cease and desist from committing any act that might lead to the further
deterioration of the current labor relations situation.

UFE-DFA-KMU sought reconsideration of the above but nonetheless moved for


additional time to file its position paper as directed by the Assumption of
Jurisdiction Order.
The Sec. denied said motion for reconsideration.

Despite the order, the employee members of UFE-DFA-KMU at Nestl's Cabuyao


Plant went on strike.

The Sec. directed: (1) the members of UFE-DFA-KMU to return-to-work; (2)


Nestl to accept back all returning workers; (3) both parties to cease and desist
from committing acts inimical to the on-going conciliation proceedings leading to
the further deterioration of the situation; and (4) the submission of their
respective position papers within ten (10) days from receipt thereof. The
members of UFE-DFA-KMU continued with their strike, thus, prompting the Sec.
to seek the assistance of the (PNP) for the enforcement of said order.
Nestl and UFE-DFA-KMU filed their respective position papers.

On 11 February 2002, the Sec. allowed UFE-DFA-KMU the chance to tender its
stand on the other issues raised by Nestl but not covered by its initial position
paper by way of a Supplemental Position Paper.

UFE-DFA-KMU filed several pleadings, one of which was a Manifestation with


Motion for Reconsideration of the Order dated February 11, 2002 assailing the
Order of February 11, 2002 for supposedly being contrary to law, jurisprudence

UFE-DFA-KMU filed a Petition for Certiorari before the CA, alleging that the Sec.
committed grave abuse of discretion amounting to lack or excess of jurisdiction
when she issued the Orders of 11 February 2002 and 8 March 2002.
Acting Secretary of the DOLE, Hon. Arturo D. Brion, came out with an Order
dated 02 April 2002, ruling that:
xxx

b. the Union's charge of unfair labor practice against the Company is hereby
dismissed for lack of merit;

c. the parties are directed to secure the best applicable terms of the recently
concluded CBSs between Nestl Phils. Inc. and it eight (8) other bargaining units,
and to adopt these as the terms and conditions of the Nestl Cabuyao Plant CBA;
xxx

f. the parties shall execute their CBA within thirty (30) days from receipt of this
Order, furnishing this Office a copy of the signed Agreement;
xxx

UFE-DFA-KMU moved to reconsider the aforequoted ruling, but such was


subsequently denied on May 6, 2002.

For the second time, UFE-DFA-KMU went to the CA via another Petition for
Certiorari seeking to annul the Orders of 02 April 2002 and 06 May 2002 of the
Secretary of the DOLE, having been issued in grave abuse of discretion
amounting to lack or excess of jurisdiction.

The appellate court promulgated its Decision on the twin petitions for certiorari,
ruling entirely in favor of UFE-DFA-KMU.

3E Andaya Ching Espiritu Hefti Galvez Gammad Lainez Lui Madamba Nagera Narvasa
Ong Palangdao Rosales Sanchez Santos Satrain Tabo (2014-2015)

Labor Relations Case Digest - Atty. Joyrich Golangco


HELD:

Both parties appealed. Nestl essentially assailed that part of the decision finding
the DOLE Secretary to have gravely abused her discretion amounting to lack or
excess of jurisdiction when she ruled that the Retirement Plan was not a valid
issue to be tackled during the CBA negotiations; UFE-DFA-KMU, in contrast,
questioned the appellate court's decision finding Nestl free and clear of any
unfair labor practice.
The motions for reconsideration of both parties were denied.UFE-DFA-KMU and
Nestl separately filed the instant Petitions for Review on Certiorari under Rule
45 of the Rules of Court.
This Court resolved to consolidate the two petitions.

This Court promulgated on 22 August 2006 its Decision, now subject of UFEDFA-KMU's Motion for Partial Reconsideration and Nestl's Motion for
Clarification.

In its Motion for Partial Reconsideration, UFE-DFA-KMU would have this Court
address and discuss anew points or arguments that have been passed upon.
Firstly, it questions this Court's finding that Nestl was not guilty of unfair labor
practice. Second, is the question of whether or not the DOLE Secretary can take
cognizance of matters beyond the amended Notice of Strike.

As to Nestl's prayer for clarification, the corporation seeks elucidation


respecting the dispositive part of this Court's Decision directing herein parties to
resume negotiations on the retirement compensation package of the concerned
employees. It posits that "[i]n directing the parties to negotiate the Retirement
Plan, the Honorable Court . . . might have overlooked the fact that here, the
Secretary of Labor had already assumed jurisdiction over the entire 2001-2004
CBA controversy . . . ."
ISSUES: 1. Whether or not Nestle was guilty of any unfair labor practice.

2. Whether or not the DOLE Secretary can take cognizance of matters beyond the
amended Notice of Strike.

1. No. The duty to bargain collectively is mandated by Articles 252 and 253 of the
Labor Code, as amended. The purpose of collective bargaining is the reaching of
an agreement resulting in a contract binding on the parties; but the failure to
reach an agreement after negotiations have continued for a reasonable period
does not establish a lack of good faith. The statutes invite and contemplate a
collective bargaining contract, but they do not compel one. There is no per se test
of good faith in bargaining. Good faith or bad faith is an inference to be drawn
from the facts.
For a charge of unfair labor practice to prosper, it must be shown that Nestl was
motivated by ill will, "bad faith, or fraud, or was oppressive to labor, or done in a
manner contrary to morals, good customs, or public policy, and, of course, that
social humiliation, wounded feelings, or grave anxiety resulted . . ." in disclaiming
unilateral grants as proper subjects in their collective bargaining negotiations.

Herein, the union merely bases its claim of refusal to bargain on a letter written
by Nestl where the latter laid down its position that "unilateral grants, one-time
company grants, company-initiated policies and programs, which include, but
are not limited to the Retirement Plan, Incidental Straight Duty Pay and Calling
Pay Premium, are by their very nature not proper subjects of CBA negotiations
and therefore shall be excluded therefrom." Said letter is not tantamount to
refusal to bargain. Nestl's desire to settle the dispute and proceed with the
negotiation being evident in its cry for compulsory arbitration is proof enough of
its exertion of reasonable effort at good-faith bargaining.

Nestle never refused to bargain collectively with UFE-DFA-KMU. An employer's


steadfast insistence to exclude a particular substantive provision is no different
from a bargaining representative's perseverance to include one that they deem
of absolute necessity.
The concept of "unfair labor practice" is defined by the Labor Code under Art.
247.
The same code likewise provides the acts constituting unfair labor practices
committed by employers under Art. 248.

3E Andaya Ching Espiritu Hefti Galvez Gammad Lainez Lui Madamba Nagera Narvasa
Ong Palangdao Rosales Sanchez Santos Satrain Tabo (2014-2015)

Labor Relations Case Digest - Atty. Joyrich Golangco

Nestl is accused of violating its duty to bargain collectively when it purportedly


imposed a pre-condition to its agreement to discuss and engage in collective
bargaining negotiations with UFE-DFA-KMU.
A meticulous review of the record and pleadings of the cases at bar shows that, of
the two notices of strike filed by UFE-DFA-KMU before the NCMB, it was only on
the second that the ground of unfair labor practice was alleged. The Notice of
Strike merely contained a general allegation that Nestl committed unfair labor
practice. In contrast, Nestl, in its Position Paper, did not confine itself to the
issue of the non-inclusion of the Retirement Plan but extensively discussed its
stance on other economic matters pertaining to the CBA. It is UFE-DFA-KMU,
therefore, who had the burden of proof to present substantial evidence to
support the allegation of unfair labor practice.
Under Rule XIII, Sec. 4, Book V of the Implementing Rules of the Labor Code:

. . . . In cases of unfair labor practices, the notice of strike shall as far as


practicable, state the acts complained of and the efforts to resolve the dispute
amicably."

In the case at bar, except for the assertion put forth by UFE-DFA-KMU, neither
the second Notice of Strike nor the records of these cases substantiate a finding
of unfair labor practice.

The rule is that good faith is always presumed. As long as the company's exercise
of the same is in good faith to advance its interest and not for purpose of
defeating or circumventing the rights of employees under the law or a valid
agreement, such exercise will be upheld.
2. Yes. This Court is not convinced by the argument raised by UFE-DFA-KMU that
the DOLE Secretary should not have gone beyond the disagreement on the
ground rules of the CBA negotiations. The union doggedly asserts that the entire
labor dispute between herein parties concerns only the ground rules.
It was UFE-DFA-KMU which first alleged a bargaining deadlock as the basis for
the filing of its Notice of Strike; and at the time of the filing of the first Notice of

Strike, several conciliation conferences had already been undertaken where both
parties had already exchanged with each other their respective CBA proposals.

The Secretary of the DOLE simply relied on the Notices of Strike that were filed
by UFE-DFA-KMU as stated in her Order of 08 March 2002.
The second Notice of Strike is dated November 7, 2001 and the cited ground is
like quoted verbatim below:
"B. Unfair Labor Practices (specify)
Bargaining in bad faith

Setting pre-condition in the ground rules (Retirement issue)"

Nowhere in the second Notice of Strike is it indicated that this Notice is an


amendment to and took the place of the first Notice of Strike.

Had the parties not been at the stage where the substantive provisions of the
proposed CBA had been put in issue, the union would not have based thereon its
initial notice to strike. This Court maintains its original position in the Decision
that, based on the Notices of Strike filed by UFE-DFA-KMU, the Secretary of the
DOLE rightly decided on matters of substance. That the union later on changed
its mind is of no moment because to give premium to such would make the
legally mandated discretionary power of the Dole Secretary subservient to the
whims of the parties.
WHEREFORE, premises considered, the basic issues of the case having been
passed upon and there being no new arguments availing, the Motion for Partial
Reconsideration is hereby DENIED WITH FINALITY for lack of merit.
93. MALAYANG MANGGAGAWA vs. NLRC (Aug. 28, 2013)

FACTS:

Petitioner and Nagkakaisang Lakas ng Manggagawa sa Stayfast (NLMSOlalia) sought to be the exclusive bargaining agent of the employees of
respondent company, Stayfast Philippines, Inc.

3E Andaya Ching Espiritu Hefti Galvez Gammad Lainez Lui Madamba Nagera Narvasa
Ong Palangdao Rosales Sanchez Santos Satrain Tabo (2014-2015)

Labor Relations Case Digest - Atty. Joyrich Golangco

A certification election was conductedPetitioner garnered 109 votes


while NLMS-Olalia received112 votes. The Med-Arbiter who supervised the cert
election issued an order certifying NLMS-Olalia as the sole and exclusive
bargaining agent of all rank and file employees of Stayfast.

Petitioner appealed the Order to the Secretary of Labor, who initially set
aside the Order and called for run-off election between petitioner and NLMSOlalia.

However, on motion of NLMS-Olalia, the Secretary of Labor restored the


Med-Arbiters Order.

NLMS-Olalia demanded to collectively bargain with Stayfast, which


rejected the same insisting that it would negotiate a collective bargaining
agreement only with whichever union is finally certified as the sole and exclusive
bargaining agent of the workers. Nevertheless, NLMS-Olalia went on strike until
it was temporarily restrained eight days later.

Subsequently, petitioner filed its own notice of strike in the NCMB.


Respondent company opposed petitioners move and filed a motion to dismiss on
the ground that petitioner was not the certified bargaining agent and therefore
lacked personality to file a notice of strike.

Thereafter, the parties were able to make concessions during the


conciliation-mediation stage in the NCMB which led petitioner to withdraw its
notice of strike.

However, petitioners members staged a "sit-down strike" to dramatize


their demand for a fair and equal treatment as Stayfast allegedly continued to
discriminate against them. Stayfast issued a memorandum requiring the alleged
participants in the "sit-down strike" to explain why they should not be
terminated or suspended from work. As no one complied with the memorandum,
respondent company promptly terminated the service of the participants in the
"sit-down strike"

Consequently, petitioner staged a strike and filed a complaint for unfair


labor practice, union busting and illegal lockout against respondent company and
its General Manager, Maria Almeida, in the NLRC
Petitioners contention: Respondents had repeatedly committed acts of
discrimination, such as the denial of the use of the company canteen for

purposes of conducting a strike vote, the constant denial of applications of


petitioners members for leave to attend hearings in relation to certain labor
cases and the suspension of petitioners president for being absent due to
attendance in hearings of labor cases involving petitioners members.
Respondents contention: Petitioner lacked legal authority to go on strike since it
is a minority union. As petitioner withdrew its notice of strike during the
proceedings in the NCMB, the strike conducted by petitioner was illegal as it
constituted a wildcat strike and later became a full-blown strike. Petitioner
committed illegal acts during the strike and obstructed the free ingress and
egress from respondent companys premise

LA: While petitioner may file a notice of strike on behalf of its members,
petitioner failed to cite any instance of discrimination or harassment when it
filed its notice of strike and the incidents mentioned as discriminatory occurred
after the filing of the said notice. Moreover, assuming the strike was legal at the
beginning, it became illegal when petitioner committed acts prohibited under
Article 264(e) of the Labor Code, such as acts of violence, coercion and
intimidation and obstruction of the free ingress to and egress from respondent
companys premises.
NLRC: The actuations of petitioner were patently illegal because the sit-down
strike was made barely a week after petitioner withdrew its notice of strike, with
prejudice, on account of the concessions agreed upon by the parties. Petitioner
filed no new notice of strike that could have supported its charges of
discriminatory acts and unfair labor practice. Hence it upheld the LAs decision.
CA: NLRC Resolution was supported by justifiable reason and cannot be faulted
with grave abuse of discretion. Petitioner failed to establish that the NLRC
committed grave abuse of discretion.
ISSUES: Whether or not the petition should prosper.
HELD: No. Petition fails for many reasons (both procedural and Substantive)
PROCEDURAL:

3E Andaya Ching Espiritu Hefti Galvez Gammad Lainez Lui Madamba Nagera Narvasa
Ong Palangdao Rosales Sanchez Santos Satrain Tabo (2014-2015)

1. Petition for certiorari is a wrong remedy.

Labor Relations Case Digest - Atty. Joyrich Golangco

A petition for certiorari under Rule 65 of the Rules of Court is a special


civil action that may be resorted to only in the absence of appeal or any plain,
speedy and adequate remedy in the ordinary course of law. The right recourse
was to appeal to this Court in the form of a petition for review on certiorari
under Rule 45 of the Rules of Court. For purposes of appeal, the Decision of the
Court of Appeals was a final judgment as it denied due course to, and dismissed,
the petition. Thus, the Decision disposed of the petition of petitioner in a manner
that left nothing more to be done by the Court of Appeals in respect to the said
case. Thus, petitioner should have filed an appeal by petition for review on
certiorari under Rule 45. Where the rules prescribe a particular remedy for the
vindication of rights, such remedy should be availed of. The existence and
availability of the right of appeal prohibits the resort to certiorari because one of
the requirements for the latter remedy is that there should be no appeal.

Moreover, certiorari is not and cannot be made a substitute for an


appeal where the latter remedy is available but was lost through fault or
negligence. In this case, petitioner received the Decision on August 2, 2002 and,
under the rules, had until August 19, 2002 to file an appeal by way of a petition
for review in this Court. Petitioner let this period lapse without filing an appeal
and, instead, filed this petition for certiorari on October 1, 2002.
2. Even assuming that a petition for certiorari is the correct remedy in this case,
petitioner failed to comply with the requirement of a prior motion for
reconsideration.

As a general rule, a motion for reconsideration is a prerequisite for the


availment of a petition for certiorari under Rule 65 to afford the public
respondent an opportunity to correct any actual or fancied error attributed to it
by way of re-examination of the legal and factual aspects of the case. Here, the
Court of Appeals was not given any opportunity either to rectify whatever error
it may have made or to address the ascription and aspersion of grave abuse of
discretion thrown at it by petitioner. Nor did petitioner offer any compelling
reason to warrant a deviation from the rule.

3. Petitioner was not able to establish its allegation of grave abuse of discretion
on the part of the Court of Appeals.

In this case, nowhere in the petition did petitioner show that the
issuance of the Decision of the Court of Appeals was patent and gross that would
warrant striking it down through a petition for certiorari. Petitioner failed to
substantiate its imputation of grave abuse of discretion on the part of the Court
of Appeals. Petitioner failed in its duty to demonstrate with definiteness the
grave abuse of discretion that would justify the proper availment of a petition for
certiorari under Rule 65 of the Rules of Court.
4. Petitioner essentially questioned the factual findings of the Labor Arbiter and
the NLRC. Petitioner cannot properly do that in a petition for certiorari.

For petitioner to question the identical findings of the Labor Arbiter and
the NLRC is to raise a question of fact. However, it is settled that questions of fact
cannot be raised in an original action for certiorari. Only established or admitted
facts can be considered.

The Supreme Court is not a trier of facts, more so in the consideration of


the extraordinary writ of
certiorari where neither questions of fact nor of law are entertained, but only
questions of lack or excess of jurisdiction or grave abuse of discretion.

5. Considering that petitioner basically presented an issue of fact, its petition for
certiorari crumbles in view of the identical findings of the Labor Arbiter and the
NLRC which were further upheld by the Court of Appeals.

The Court of Appeals correctly ruled that findings of fact made by Labor
Arbiters and affirmed by the NLRC are not only entitled to great respect, but even
finality, and are considered binding if the same are supported by substantial
evidence. In arriving at the said ruling, the Court of Appeals even reviewed the
rationale of the Labor Arbiters decision and was convinced that there was
justifiable reason for the NLRC to uphold the same. This Court finds no
compelling reason to rule otherwise.
SUBSTANTIVE:

3E Andaya Ching Espiritu Hefti Galvez Gammad Lainez Lui Madamba Nagera Narvasa
Ong Palangdao Rosales Sanchez Santos Satrain Tabo (2014-2015)

Labor Relations Case Digest - Atty. Joyrich Golangco

6. Even on the merits, the case of petitioner has no leg to stand on.

Petitioners case rests on the alleged discriminatory acts of respondent


company against petitioners officers and members. However, both the Labor
Arbiter and the NLRC held that there was no sufficient proof of respondent
companys alleged discriminatory acts. Thus, petitioners unfair labor practice,
union-busting and unlawful lockout claims do not hold water.

Moreover, the established facts as found by the NLRC are as follows: the
"sit-down strike" made by petitioners officers and members was in violation of
respondent companys rules, and petitioners officers and members ignored the
opportunity given by respondent company for them to explain their misconduct,
which resulted in the termination of their employment. The Court of Appeals
ruled that the said findings were supported by substantial evidence. This Court
finds that such ruling of the appellate court is not grave abuse of discretion, nor
could it be considered wrong.
In contrast, the instant petition for certiorari suffers from an acute scarcity of
legal and factual support.
94
. HOLY CHILD CATHOLIC SCHOOL, Petitioner, v. HON. PATRICIA STO.
TOMAS, IN HER OFFICIAL CAPACITY AS SECRETARY OF THE DEPARTMENT
OF LABOR AND EMPLOYMENT, AND PINAG-ISANG TINIG AT LAKAS NG
ANAKPAWIS HOLY CHILD CATHOLIC SCHOOL TEACHERS AND EMPLOYEES
LABOR UNION (HCCS-TELU-PIGLAS), Respondents.
FACTS: On May 31, 2002, a petition for certification election was filed by private
respondent Pinag-Isang Tinig at Lakas ng Anakpawis Holy Child Catholic School
Teachers and Employees Labor Union (HCCS-TELU-PIGLAS), alleging that: PIGLAS
is a legitimate labor organization duly registered with the Department of Labor
and Employment (DOLE) representing HCCS-TELU-PIGLAS; HCCS is a private
educational institution duly registered and operating under Philippine laws;
there are approximately one hundred twenty (120) teachers and employees
comprising the proposed appropriate bargaining unit; and HCCS is unorganized,
there is no collective bargaining agreement or a duly certified bargaining agent
or a labor organization certified as the sole and exclusive bargaining agent of the

proposed bargaining unit within one year prior to the filing of the
petition.6 Among the documents attached to the petition were the certificate of
affiliation with Pinag-Isang Tinig at Lakas ng Anakpawis Kristiyanong Alyansa ng
Makabayang Obrero (PIGLAS-KAMAO) issued by the Bureau of Labor Relations
(BLR), charter certificate issued by PIGLAS-KAMAO, and certificate of
registration of HCCS-TELU as a legitimate labor organization issued by the
DOLE.7
Herein petitioner opposed the petition for certification election on the ground
that members of private respondent do not belong to the same class; it is not
only a mixture of managerial, supervisory, and rank-and-file employees as
three (3) are vice-principals, one (1) is a department head/supervisor, and
eleven (11) are coordinators but also a combination of teaching and nonteaching personnel as twenty-seven (27) are non-teaching personnel.
Med-arbiter denied the petition for election and explained that mutuality of
interests is wanting between the teaching and non-teaching personnel.

SOLE agreed with the med-arbiter but instead directed the conduct of two
separate certification elections for the teaching and the non-teaching personnel.

Petitioner filed a petition for Certiorari with TRO with the CA which was denied.
As to the purported commingling of managerial, supervisory, and rank-and-file
employees in private respondents membership, it held that the Toyota ruling is
inapplicable because the vice-principals, department head, and coordinators are
neither supervisory nor managerial employees, because while they may
formulate policies or guidelines, nonetheless, such is merely recommendatory in
nature, and still subject to review and evaluation by the higher executives.
ISSUES: WHETHER OR NOT THE PETITION FOR CERTIFICATION ELECTION
SHOULD BE DISMISSED

HELD: NO, A.) Under the BYSTANDER RULE, it has been consistently held in a
number of cases that a certification election is the sole concern of the workers,
except when the employer itself has to file the petition pursuant to Article 259 of
the Labor Code, as amended, but even after such filing its role in the certification
process ceases and becomes merely a bystander.41The employer clearly lacks the
personality to dispute the election and has no right to interfere at all therein.
Following the doctrine laid down in Kawashima and SMCC-Super, it must be

3E Andaya Ching Espiritu Hefti Galvez Gammad Lainez Lui Madamba Nagera Narvasa
Ong Palangdao Rosales Sanchez Santos Satrain Tabo (2014-2015)

Labor Relations Case Digest - Atty. Joyrich Golangco

stressed that petitioner cannot collaterally attack the legitimacy of private


respondent by praying for the dismissal of the petition for certification election.

B.) In case of alleged inclusion of disqualified employees in a union, the proper


procedure for an employer like petitioner is to directly file a petition for
cancellation of the unions certificate of registration due to misrepresentation,
false statement or fraud under the circumstances enumerated in Article 239 of
the Labor Code, as amended.54 To reiterate, private respondent, having been
validly issued a certificate of registration, should be considered as having
acquired juridical personality which may not be attacked collaterally. A
bargaining unit is a group of employees sought to be represented by a petitioning
union. Such employees need not be members of a union seeking the conduct of a
certification election. A union certified as an exclusive bargaining agent
represents not only its members but also other employees who are not union
members. As pointed out in our assailed Decision, there were two contending
unions in the U.P. case, namely[,] the Organization of Non-Academic Personnel of
U.P. (ONAPUP) and the All U.P. Workers Union composed of both U.P. academic
and non-academic personnel. ONAPUP sought the conduct of a certification
election among the rank-and-file non-academic personnel only, while the All U.P.
Workers Union intended to cover all U.P. rank-and-file employees, involving both
academic and non-academic personnel.
The Supreme Court ordered the non-academic rank-and-file employees of U.P.
to constitute a bargaining unit to the exclusion of the academic employees of the
institution, but did not order them to organize a separate labor organization. In
the U.P. case, the Supreme Court did not dismiss the petition and affirmed the
order for the conduct of a certification election among the non-academic
personnel of U.P., without prejudice to the right of the academic personnel to
constitute a separate bargaining unit for themselves and for the All U.P. Workers
Union to institute a petition for certification election.

In the same manner, the teaching and non-teaching personnel of [petitioner]


school must form separate bargaining units. Thus, the order for the conduct of
two separate certification elections, one involving teaching personnel and the
other involving non-teaching personnel. It should be stressed that in the subject
petition, [private respondent] union sought the conduct of a certification election
among all the rank-and-file personnel of [petitioner] school. Since the decision of

the Supreme Court in the U.P. case prohibits us from commingling teaching and
non-teaching personnel in one bargaining unit, they have to be separated into
two separate bargaining units with two separate certification elections to
determine whether the employees in the respective bargaining units desired to
be represented by [private respondent]. In the U.P. case, only one certification
election among the non-academic personnel was ordered, because ONAPUP
sought to represent that bargaining unit only. No petition for certification
election among the academic personnel was instituted by All U.P. Workers Union
in the said case; thus, no certification election pertaining to its intended
bargaining unit was ordered by the Court.
95: Kiok Loy, doing business under the name and Style Sweden Ice Cream
Plant vs. NLRC and Pambansang Kilusan ng Paggawa

FACTS: The Pambansang Kilusang Paggawa (Union for short) is a legitimate late
labor federation which won the certification election and was subsequently
certified by the BLR as the sole and exclusive bargaining agent of the rank-andfile employees of Sweden Ice Cream Plant (Company, for short). The Company
filed a motion for reconsideration for the certification which was denied.

Thereafter, the Union furnished the Company with two copies of its
proposed collective bargaining agreement. At the same time, it requested the
Company for its counter proposals. Eliciting no response to the aforesaid
request, the Union again wrote the Company reiterating its request for collective
bargaining negotiations and for the Company to furnish them with its counter
proposals. Both requests were ignored and remained unacted upon by the
Company.
Left with no other alternative in its attempt to bring the Company to the
bargaining table, the Union, on February 14, 1979, filed a "Notice of Strike", with
the Bureau of Labor Relations (BLR) on ground of unresolved economic issues in
collective bargaining.
Conciliation proceedings then followed during the thirty-day statutory
cooling-off period. But all attempts towards an amicable settlement failed,
prompting the BLR to certify the case to the NLRC for compulsory

3E Andaya Ching Espiritu Hefti Galvez Gammad Lainez Lui Madamba Nagera Narvasa
Ong Palangdao Rosales Sanchez Santos Satrain Tabo (2014-2015)

Labor Relations Case Digest - Atty. Joyrich Golangco

arbitration. The labor arbiter, Andres Fidelino, to whom the case was assigned,
set the initial hearing for April 29, 1979. For failure however, of the parties to
submit their respective position papers as required, the said hearing was
cancelled and reset to another date. Meanwhile, the Union submitted its position
paper. The Company did not, and instead requested for a resetting which
was granted. The Company was directed anew to submit its financial
statements for the years 1976, 1977, and 1978.

While it is a mutual obligation of the parties to bargain, the employer,


however, is not under any legal duty to initiate contract negotiation. The
mechanics of collective bargaining is set in motion only when the following
jurisdictional preconditions are present, namely,
(1) possession of the status of majority representation
of the employees' representative in accordance with any of the
means of selection or designation provided for by the Labor
Code;

The case was further reset to May 11, 1979 due to the withdrawal of the
Company's counsel of record, Atty. Rodolfo dela Cruz. On May 24, 1978, Atty.
Fortunato Panganiban formally entered his appearance as counsel for the
Company only to request for another postponement allegedly for the purpose of
acquainting himself with the case. Meanwhile, the Company submitted its
position paper on May 28, 1979.

When the case was called for hearing on June 4, 1979 as scheduled, the
Company's representative, Mr. Ching, who was supposed to be examined,
failed to appear. Atty. Panganiban then requested for another
postponement which the labor arbiter denied. He also ruled that the
Company has waived its right to present further evidence and, therefore,
considered the case submitted for resolution.
The labor arbiter submitted its report to the NLRC which rendered the
decision declaring the Company guilty of unjustified refusal to bargain.
ISSUES: Whether Sweden Ice Cream Plant is guilty of the Unfair Labor Practice of
unjustified refusal to bargain.

HELD: Yes, it is. Collective bargaining which is defined as negotiations towards a


collective agreement is one of the democratic frameworks under the New Labor
Code, designed to stabilize the relation between labor and management and to
create a climate of sound and stable industrial peace. It is a mutual responsibility
of the employer and the Union and is characterized as a legal obligation. So much
so that Article 249, par. (g) of the Labor Code makes it an unfair labor practice.

(2) proof of majority representation; and

(3) a demand to bargain under Article 251, par.(a) of


the New Labor Code.
case.

... all of which preconditions are undisputedly present in the instant

From the over-all conduct of petitioner company in relation to the task


of negotiation, there can be no doubt that the Union has a valid cause to
complain against its (Company's) attitude, the totality of which is indicative
of the latter's disregard of, and failure to live up to, what is enjoined by the
Labor Code to bargain in good faith.
The Court is in total conformity with respondent NLRC's pronouncement
that petitioner Company is GUILTY of unfair labor practice. It has been
indubitably established that
agent;

(1) Respondent Union was a duly certified bargaining

(2) it made a definite request to bargain, accompanied


with a copy of the proposed Collective Bargaining Agreement,
to the Company not only once but twice which were left
unanswered and unacted upon; and

3E Andaya Ching Espiritu Hefti Galvez Gammad Lainez Lui Madamba Nagera Narvasa
Ong Palangdao Rosales Sanchez Santos Satrain Tabo (2014-2015)

Labor Relations Case Digest - Atty. Joyrich Golangco

(3) the Company made no counter proposal


whatsoever all of which conclusively indicate lack of a sincere
desire to negotiate.

A Company's refusal to make counter proposal if considered in relation


to the entire bargaining process, may indicate bad faith and this is especially true
where the Union's request for a counter proposal is left unanswered. Even
during the period of compulsory arbitration before the NLRC, petitioner
Company's approach and attitude-stalling the negotiation by a series of
postponements, non-appearance at the hearing conducted, and undue delay in
submitting its financial statements, lead to no other conclusion except that it is
unwilling to negotiate and reach an agreement with the Union. Petitioner has not
at any instance, evinced good faith or willingness to discuss freely and fully the
claims and demands set forth by the Union much less justify its opposition
thereto.
96. PHILIPPINE AIRLINES, INCORPORATED, Petitioner, vs. PHILIPPINE
AIRLINES EMPLOYEES ASSOCIATION (PALEA), Respondent.

FACTS: On February 6, 1987, PAL and PALEA entered into a CBA covering the
period of 1986-1989. Section 3 thereof provides that all the terms and
conditions of employment of employees within the bargaining unit are embodied
in this Agreement, and the same shall govern the relationship between the
Company and such employees. x x x Part of said agreement required petitioner
PAL to pay its rank and file employees the following: 13th Month Pay (Mid-year
Bonus) equivalent to one months current basic pay, to be paid in advance in
May; and Christmas Bonus which is the equivalent of one months current
basic pay as of November 30, to be paid in December.
On April 22, 1988, prior to the payment of the 13th month pay, PAL released a
guideline implementing said provisions, to wit:

1) Eligibility
a) Ground employees in the general payroll who are regular as of
April 30, 1988;
b) Other ground employees in the general payroll, not falling within

category a) above shall receive their 13th Month Pay on or before


December 24, 1988;

2) Amount
a) For category a) above, one month basic salary as of April 30,
1988;
b) Employees covered under 1 b) above shall be paid not less than
1/12 of their basic salary for every month of service within the
calendar year.
3) Payment Date: May 9, 1988 for category 1 a) above.

PALEA assailed the implementation of the guideline on the ground that all
employees of PAL, regular or non-regular, must be paid their 13th month pay.
Subsequently, in a letter, PALEA, through Herbert C. Baldovino, informed PAL
that some regular employees failed to receive their 13th Month Pay.

PAL answered that rank and file employees regularized after April 30, 1988 were
not entitled to the 13th month pay as they were already given their Christmas
bonuses on December 9, 1988 per the Implementing Rules of PD 851 (The 13th
Month Pay Law).
Disagreeing with PAL, PALEA filed a labor complaint for ULP against PAL before
the NLRC. The complaint interposed that "the cut-off period for regularization
should not be used as the parameter for granting [the] 13th month pay
considering that the law does not distinguish the status of employment but (sic)
the law covers all employees."

LA: dismissed PALEAs complaint, holding that the giving of the particular bonus
was said to be merely an additional practice made in the past.

NLRC: reversed the Decision of the Arbiter, convinced that the 13th month pay or
mid-year bonus is distinct from the Christmas Bonus.
PAL went directly to SC via Petition for Review on Certiorari but was referred to
the CA, which dismissed PALs petition and its subsequent MR.
Hence the instant Petition for Review on Certiorari under Rule 45.

3E Andaya Ching Espiritu Hefti Galvez Gammad Lainez Lui Madamba Nagera Narvasa
Ong Palangdao Rosales Sanchez Santos Satrain Tabo (2014-2015)

Labor Relations Case Digest - Atty. Joyrich Golangco

PAL argues that 1) the CBA does not apply to non-regular employees such that
any benefits arising from said agreement cannot be made to apply to them,
including the mid-year bonus; and 2) it has always been the company practice
not to extend the mid-year bonus to those employees who have not attained
regular status prior to the month of May, when payment of the particular bonus
accrues.

PALEA, however, disputes petitioner PALs allegations and maintains that "[a]ll
employees in PAL are entitled to the same benefit as they are within the same
collective bargaining unit and the entitlement to such benefit spills over to even
non-union members." Anent the supposed company practice of PAL not to
extend the payment of the 13th month pay or mid-year bonus to non-regular
employees, non-payment of said benefit is considered a diminution of privileges
or benefits proscribed by PD 851; that petitioner PAL misrepresented that the
13th month pay or mid-year bonus is the same as the Christmas bonus when, in
actuality, the latter is entirely different as it is a benefit paid under the provisions
of the CBA, while the former is one mandated by law, Presidential Decree No.
851, in particular.
ISSUES:
1.

WON the payment of the 13th month pay or mid-year bonus applies to
PAL employees regularized after April 30, 1988.

HELD:

YES. A cursory reading of the 1986-1989 CBA of the parties herein will instantly
reveal that Art. I, Sec. 3 of said agreement made its provision applicable to all
employees in the bargaining unit, without distinguishing between regular and
non-regular employees.

It is a well-settled doctrine that the benefits of a CBA extend to the laborers


and employees in the collective bargaining unit, including those who do not
belong to the chosen bargaining labor organization. Otherwise, it would be
a clear case of discrimination.

To be entitled to the benefits under the CBA, the employees must be members of
the bargaining unit, but not necessarily of the labor organization designated as
the bargaining agent. A "bargaining unit" has been defined as a group of
employees of a given employer, comprised of all or less than all of the entire
body of employees, which the collective interest of all the employees, consistent
with equity to the employer, indicates to be the best suited to serve the
reciprocal rights and duties of the parties under the collective bargaining
provisions of the law. PALs allegation that the non-regular employees do not
belong to the collective bargaining unit and are thus not covered by the CBA is
unjustified and unsubstantiated. PAL excludes certain employees from the
benefits of the CBA only because they have not yet achieved regular status by the
cut-off date, April 30, 1988. There is no showing that the non-regular status
of the concerned employees by said cut-off date sufficiently distinguishes
their interests from those of the regular employees so as to exclude them
from the collective bargaining unit and the benefits of the CBA.
2.

Whether the 13th month pay or mid-year bonus can be equated to the
Christmas bonus.

NO. While employers already paying their employees a 13th month pay or more
in a calendar year or its equivalent at the time of the issuance of PD 851 are
already exempted from the mandatory coverage of said law, PAL cannot escape
liability in this case by virtue thereof.
It must be stressed that in the 1986-1989 CBA, petitioner PAL agreed to pay its
employees 1) the 13th month pay or the mid-year bonus, and 2) the Christmas
bonus. The 13th month pay, guaranteed by PD 851, is explicitly covered or
provided for as the mid-year bonus in the CBA, while the Christmas bonus is
evidently and distinctly a separate benefit. PAL may not be allowed to brush
off said distinction, and unilaterally and arbitrarily declare that for nonregular employees, their Christmas bonus is the same as or equivalent to
the 13th month pay.

PD 851 mandates the payment of the 13th month pay to uniformly provide the
low-paid employees with additional income. It but sets a minimum requirement
that employers must comply with. It does not intend, however, to preclude the

3E Andaya Ching Espiritu Hefti Galvez Gammad Lainez Lui Madamba Nagera Narvasa
Ong Palangdao Rosales Sanchez Santos Satrain Tabo (2014-2015)

Labor Relations Case Digest - Atty. Joyrich Golangco

employers from voluntarily granting additional bonuses that will benefit their
employees. A bonus is an amount granted and paid to an employee for his
industry and loyalty which contributed to the success of the employer's business
and made possible the realization of profits. It is an act of generosity of the
employer for which the employee ought to be thankful and grateful. It is also
granted by an enlightened employer to spur the employee to greater efforts for
the success of the business and realization of bigger profits. We deem that the
Christmas bonus in this case is of this nature, although, by virtue of its
incorporation into the CBA, it has become more than just an act of generosity on
the part of PAL, but a contractual obligation it has undertaken.

97. FVC Labor Union Phil. Transport and General Workers Association vs.
SANAMA-FVC-SIGLO

FACTS: On December 22, 1997, the petitioner FVCLU-PTGWO the recognized


bargaining agent of the rank-and-file employees of the FVC Philippines,
Incorporated (company) signed a five-year collective bargaining agreement
(CBA) with the company. The five-year CBA period was from February 1, 1998 to
January 30, 2003. At the end of the 3rd year of the five-year term and pursuant to
the CBA, FVCLU-PTGWO and the company entered into the renegotiation of the
CBA and modified, among other provisions, the CBAs duration. Article XXV,
Section 2 of the renegotiated CBA provides that this re-negotiation agreement
shall take effect beginning February 1, 2001 and until May 31, 2003 thus
extending the original five-year period of the CBA by four (4) months.

On January 21, 2003, nine (9) days before the January 30, 2003 expiration
of the originally-agreed five-year CBA term (and four months and nine days away
from the expiration of the amended CBA period), the respondent Sama-Samang
Nagkakaisang Manggagawa sa FVC-Solidarity of Independent and General Labor
Organizations (SANAMA-SIGLO) filed before the Department of Labor and
Employment (DOLE) a petition for certification election for the same rank-andfile unit covered by the FVCLU-PTGWO CBA. FVCLU-PTGWO moved to dismiss
the petition on the ground that the certification election petition was filed
outside the freedom period or outside of the sixty (60) days before the
expiration of the CBA on May 31, 2003.
Med-arbiter: On June 17, 2003, Med-Arbiter Arturo V. Cosuco dismissed the

petition on the ground that it was filed outside the 60-day period counted from
the May 31, 2003 expiry date of the amended CBA.
Dole Sec: DOLE Secretary Patricia A. Sto. Tomas sustained SANAMA-SIGLOs
position, thereby setting aside the decision of the Med-Arbiter. She ordered the
conduct of a certification election in the company.

CA: Sustained the DOLE Secs decision.

ISSUES: W/N the extension of the life of the CBA extended the exclusive
bargaining status as well

HELD: We hold this FVCLU-PTGWO position to be correct, but only with respect
to the original five-year term of the CBA, which, by law, is also the effective period
of the unions exclusive bargaining representation status. While the parties may
agree to extend the CBAs original five-year term together with all other CBA
provisions, any such amendment or term in excess of five years will not carry
with it a change in the unions exclusive collective bargaining status. By express
provision of the above-quoted Article 253-A, the exclusive bargaining status
cannot go beyond five years and the representation status is a legal matter not
for the workplace parties to agree upon. In other words, despite an agreement
for a CBA with a life of more than five years, either as an original provision or by
amendment, the bargaining unions exclusive bargaining status is effective only
for five years and can be challenged within sixty (60) days prior to the expiration
of the CBAs first five years.

As discussed above, this negotiated extension of the CBA term has no


legal effect on the FVCLU-PTGWOs exclusive bargaining representation status
which remained effective only for five years ending on the original expiry date of
January 30, 2003. Thus, sixty days prior to this date, or starting December 2,
2002, SANAMA-SIGLO could properly file a petition for certification election. Its
petition, filed on January 21, 2003 or nine (9) days before the expiration of the
CBA and of FVCLU-PTGWOs exclusive bargaining status, was seasonably filed.
98. SMCEU vs Confesor

FACTS: On June 28, 1990, petitioner-union CBA with private respondent


San Miguel Corporation (SMC) to take effect upon the expiration of the previous

3E Andaya Ching Espiritu Hefti Galvez Gammad Lainez Lui Madamba Nagera Narvasa
Ong Palangdao Rosales Sanchez Santos Satrain Tabo (2014-2015)

Labor Relations Case Digest - Atty. Joyrich Golangco

CBA or on June 30, 1989. In keeping with their vision and long term strategy for
business expansion, SMC management informed its employees in a letter that the
company which was composed of four operating divisions namely: (1) Beer, (2)
Packaging, (3) Feeds and Livestocks, (4) Magnolia and Agri-business would
undergo a restructuring.
Magnolia and Feeds and Livestock Division were spun-off and became two
separate and distinct corporations: Magnolia Corporation (Magnolia) and San
Miguel Foods, Inc. (SMFI). Notwithstanding the spin-offs, the CBA remained in
force and effect.

After June 30, 1992, the CBA was renegotiated in accordance with the terms
of the CBA and Article 253-A of the Labor Code. Negotiations started sometime in
July, 1992 with the two parties submitting their respective proposals and
counterproposals.
During the negotiations, the petitioner-union insisted that the
bargaining unit of SMC should still include the employees of the spun-off
corporations: Magnolia and SMFI; and that the renegotiated terms of the
CBA shall be effective only for the remaining period of two years or until
June 30, 1994.

Petitioner-union contends that the duration for the non-representation


provisions of the CBA should be coterminous with the term of the bargaining
agency which in effect shall be for the remaining two years of the current CBA.
SMC, on the other hand, contended that the members/employees who
had moved to Magnolia and SMFI, automatically ceased to be part of the
bargaining unit at the SMC. Furthermore, the CBA should be effective for
three years in accordance with Art. 253-A of the Labor Code.
Unable to agree on these issues with respect to the bargaining unit and
duration of the CBA, petitioner-union declared a deadlock. A Notice of Strike was
filed against SMC. In order to avert a strike, SMC requested the NCMB to conduct
preventive mediation. No settlement was arrived at despite several meetings
held between the parties.

Private respondents SMC, Magnolia and SMFI filed a petition with the
Secretary of Labor praying that the latter assume jurisdiction over the labor
dispute in a vital industry.

The Secretary of Labor issued the assailed Order on February 15, 1993
directing, among others, that the renegotiated terms of the CBA shall be effective
for the period of three (3) years from June 30, 1992; and that such CBA shall
cover only the employees of SMC and not of Magnolia and SMFI.

Dissatisfied, petitioner-union now comes to this Court questioning this


Order of the Secretary of Labor.
ISSUES:
1. Whether or not the duration of the renegotiated terms of the CBA is to be
effective for three years or for only two years?
2. Whether or not the bargaining unit of SMC includes also the employees of
Magnolia and SMFI?
HELD:

1. The duration of the renegotiated term of the CBA is to be effective


for 3 years.
ART. 253-A. Terms of a Collective Bargaining Agreement. Any
Collective Bargaining Agreement that the parties may enter into shall,
insofar as the representation aspect is concerned, be for a term of five (5)
years. ..xxx. All other provisions of the Collective Bargaining Agreement
shall be renegotiated not later than three (3) years after its
execution xxx
Article 253-A is a new provision. This was incorporated by Section 21 of
Republic Act No. 6715. This new provision states that the CBA has a term of 5
years instead of 3 years, before the amendment of the law as far as the
representation aspect is concerned. All other provisions of the CBA shall be
negotiated not later than three (3) years after its execution. The representation
aspect refers to the identity and majority status of the union that negotiated the
CBA as the exclusive bargaining representative of the appropriate bargaining
unit concerned. All other provisions simply refers to the rest of the CBA,
economic as well as non-economic provisions, except representation

3E Andaya Ching Espiritu Hefti Galvez Gammad Lainez Lui Madamba Nagera Narvasa
Ong Palangdao Rosales Sanchez Santos Satrain Tabo (2014-2015)

Labor Relations Case Digest - Atty. Joyrich Golangco

As the Secretary of Labor herself observed in the instant case, the law is
clear and definite on the duration of the CBA insofar as the representation aspect
is concerned, but is quite ambiguous with the terms of the other provisions of the
CBA.
The framers of the law wanted to maintain industrial peace and stability
by having both management and labor work harmoniously together without any
disturbance. Thus, no outside union can enter the establishment within five (5)
years and challenge the status of the incumbent union as the exclusive
bargaining agent. Likewise, the terms and conditions of employment (economic
and non-economic) cannot be questioned by the employers or employees during
the period of effectivity of the CBA. The CBA is a contract between the parties
and the parties must respect the terms and conditions of the agreement. Notably,
the framers of the law did not give a fixed term as to the effectivity of the terms
and conditions of employment. It can be gleaned from their discussions that it
was left to the parties to fix the period.

In the instant case, it is not difficult to determine the period of effectivity


for the non-representation provisions of the CBA. Taking it from the history of
their CBAs, SMC intended to have the terms of the CBA effective for three (3)
years reckoned from the expiration of the old or previous CBA which was on June
30, 1989.

As a matter of policy the parties are encourages to enter into a


renegotiated CBA with a term which would coincide with the aforesaid five year
term of the bargaining representative.

In the event however, that the parties, by mutual agreement, enter


into a renegotiated contract with a term of three (3) years or one which
does not coincide with the said 5-year term, and said agreement is ratified
by majority of the members in the bargaining unit, the subject contract is
valid and legal and therefore, binds the contracting parties. The same will
however not adversely affect the right of another union to challenge the
majority status of the incumbent bargaining agent within sixty (60) days
before the lapse of the original five (5) year term of the CBA.

2. The SMC bargaining unit does not include the employees of


Magnolia and SMFI.

Magnolia and SMFI were spun-off to operate as distinct companies.


Management saw the need for these transformations in keeping with its vision
and long term strategy. Magnolia and SMFI became distinct entities with
separate juridical personalities. Thus, they can not belong to a single bargaining
unit
In determining an appropriate bargaining unit, the test of grouping is
mutuality or commonality of interests. The employees sought to be represented
by the collective bargaining agent must have substantial mutual interests in
terms of employment and working conditions as evinced by the type of work
they performed.

Considering the spin-offs, the companies would consequently have their


respective and distinctive concerns in terms of the nature of work, wages, hours
of work and other conditions of employment. Interests of employees in the
different companies perforce differ. SMC is engaged in the business of beer
manufacturing. Magnolia is involved in the manufacturing and processing of
dairy products while SMFI is involved in the production of feeds and the
processing of chicken. The nature of their products and scales of business may
require different skills which must necessarily be commensurate by different
compensation packages. The different companies may have different volumes of
work and different working conditions. For such reason, the employees of the
different companies see the need to group themselves together and organize
themselves into distinctive and different groups. It would then be best to have
separate bargaining units for the different companies where the employees can
bargain separately according to their needs and according to their own working
conditions.

Even assuming in gratia argumenti that at the time of the election they
were regular employees of San Miguel, nonetheless, these workers are no longer
connected with San Miguel Corporation in any manner because Magnolia has
ceased to be a division of San Miguel Corporation and has been formed into a
separate corporation with a personality of its own. This development, which was

3E Andaya Ching Espiritu Hefti Galvez Gammad Lainez Lui Madamba Nagera Narvasa
Ong Palangdao Rosales Sanchez Santos Satrain Tabo (2014-2015)

Labor Relations Case Digest - Atty. Joyrich Golangco

brought to our attention by private respondents, necessarily renders moot and


academic any further discourse on the propriety of the elections which
petitioners impugn via the present recourse
In view of all the foregoing, we do not find any grave abuse of discretion on
the part of the Secretary of Labor in rendering the assailed Order. Wherefore, the
petition is DISMISSED.
99. International School Alliance of Educators v. Quisumbing
FACTS: Private respondent International School, Inc. (School), pursuant to PD
732, is a domestic educational institution established primarily for dependents of
foreign diplomatic personnel and other temporary residents. The decree
authorizes the School to employ its own teaching and management personnel
selected by it either locally or abroad, from Philippine or other nationalities, such
personnel being exempt from otherwise applicable laws and regulations
attending their employment, except laws that have been or will be enacted for
the protection of employees. School hires both foreign and local teachers as
members of its faculty, classifying the same into two: (1) foreign-hires and (2)
local-hires.
The School grants foreign-hires certain benefits not accorded local-hires.
Foreign-hires are also paid a salary rate 25% more than local-hires.

When negotiations for a new CBA were held on June 1995, petitioner ISAE, a
legitimate labor union and the collective bargaining representative of all faculty
members of the School, contested the difference in salary rates between foreign
and local-hires. This issue, as well as the question of whether foreign-hires
should be included in the appropriate bargaining unit, eventually caused a
deadlock between the parties.
ISAE filed a notice of strike. Due to the failure to reach a compromise in the
NCMB, the matter reached the DOLE which favored the School. Hence this
petition.

ISSUES: Whether the foreign-hires should be included in bargaining unit of localhires.

HELD: NO. The Constitution, Article XIII, Section 3, specifically provides that
labor is entitled to humane conditions of work. These conditions are not
restricted to the physical workplace the factory, the office or the field but
include as well the manner by which employers treat their employees.

Discrimination, particularly in terms of wages, is frowned upon by the Labor


Code. Article 248 declares it an unfair labor practice for an employer to
discriminate in regard to wages in order to encourage or discourage membership
in any labor organization.
The Constitution enjoins the State to protect the rights of workers and promote
their welfare, In Section 18, Article II of the constitution mandates to afford
labor full protection. The State has the right and duty to regulate the relations
between labor and capital. These relations are not merely contractual but are so
impressed with public interest that labor contracts, collective bargaining
agreements included, must yield to the common good.
However, foreign-hires do not belong to the same bargaining unit as the localhires.

A bargaining unit is a group of employees of a given employer, comprised of all


or less than all of the entire body of employees, consistent with equity to the
employer indicate to be the best suited to serve the reciprocal rights and duties
of the parties under the collective bargaining provisions of the law.

The factors in determining the appropriate collective bargaining unit are (1) the
will of the employees (Globe Doctrine); (2) affinity and unity of the employees
interest, such as substantial similarity of work and duties, or similarity of
compensation and working conditions (Substantial Mutual Interests Rule); (3)
prior collective bargaining history; and (4) similarity of employment status. The
basic test of an asserted bargaining units acceptability is whether or not it is
fundamentally the combination which will best assure to all employees the
exercise of their collective bargaining rights.

In the case at bar, it does not appear that foreign-hires have indicated their
intention to be grouped together with local-hires for purposes of collective
bargaining. The collective bargaining history in the School also shows that these
groups were always treated separately. Foreign-hires have limited tenure; localhires enjoy security of tenure. Although foreign-hires perform similar functions

3E Andaya Ching Espiritu Hefti Galvez Gammad Lainez Lui Madamba Nagera Narvasa
Ong Palangdao Rosales Sanchez Santos Satrain Tabo (2014-2015)

Labor Relations Case Digest - Atty. Joyrich Golangco

under the same working conditions as the local-hires, foreign-hires are accorded
certain benefits not granted to local-hires such as housing, transportation,
shipping costs, taxes and home leave travel allowances. These benefits are
reasonably related to their status as foreign-hires, and justify the exclusion of the
former from the latter. To include foreign-hires in a bargaining unit with localhires would not assure either group the exercise of their respective collective
bargaining rights.
100. National Association of Free Trade Unions (NAFTU) vs. Mainit Lumber
Development Co. Workers Union-United Lumber and General Workers of
the Philippines (MALDECOWU-ULGWP)
FACTS: Mainit Lumber Devt. Co. Workers Union-United Lumber and Gen.
Workers of the Phils. (MALDECOWU-ULGWP for brevity) filed a petition for
certification election with the DOLE to determine the CBA representative of the
rank and file employees of Mainit Lumber Devt. Co., Inc. (MALDECO). The
petition alleged, among others that there was no certification election conducted
within 12 months prior to the filing of the petition; the CBA has expired, and is
supported by 101 out of a total of 201 rank & file employees or more than 30%
required by law. The Med-Arbiter granted the petition however, the National
Assn. Of Free Trade Unions (NAFTU) appealed on the ground that while
MALDECO was composed of two (2) separate and distinct bargaining units i.e.
Sawmill and Logging Divisions, both the petition and decision treated them as
one. The Bureau of Labor Relations (BLR) affirmed the Arbiters decision so a
certification election was held on separate dates for the Sawmill and Logging
Divisions where MALDECOWU-ULGWP won at a ratio of 146:2. NAFTU protested
on allegations of massive vote buying and with grave & serious threat, force &
intimidation which was corroborated by MALDECO. The Med-Arbiter dismissed
the election protest and the appeal of NAFTU to the BLR and its subsequent
motion for reconsideration were denied.
ISSUES: Whether or not the certification election is valid?

HELD: YES. NAFTUs election protest was grounded on only one (1) bargaining
representative in the concluded election instead of two separate units to be
represented, i.e. one each for Logging and Sawmill Divisions. The Court held that

in determining the appropriate bargaining unit, the test of grouping is


community or mutuality of interests since the basic test of an asserted
bargaining unit's acceptability is whether or not it is fundamentally the
combination which will best assure to all employees the exercise of their
collective bargaining rights." The petition for certification election was consented
and supported by 175 out of 201 employees of MALDECO confirming that they
desire only one bargaining representative. The functions of the Sawmill
intertwined with that of the Logging in the same way that the company needs
them both and while there may be difference in the individual assignments, the
distinctions are not that extensive as to warrant the formation of a separate
bargaining unit.
The Court affirmed the Resolution of the Bureau of Labor Relations.

101. Picop Resources Incorporated vs Ricardo Dequila et al.

Wilfredo Fruentes- Senior VP of PRI


Respondents, Ricardo Dequilla, Elmo Pabilando, Cesar Atienza, Aniceto
Orbeta Jr, and NAMAPRI-SPFI
FACTS: Pprivate respondents were regular rank-and-file employees of PICOP and
members of the NAMAPRI-SPFL, a duly registered labor organization and
existing bargaining agent of the PICOP rank-and-file employees. PICOP and
NAMAPRI-SPFL had a CBA which would expire on May 22, 2000.

The late Atty. Fuentes, then National President of the Southern Philippines
Federation of Labor (SPFL), advised the PICOP management to terminate about
800 employees due to acts of disloyalty, specifically, for allegedly campaigning,
supporting and signing a petition for the certification of a rival union, the
Federation of Free Workers Union (FFW) before the 60-day freedom period
and during the effectivity of the CBA. Such acts of disloyalty were construed to be
a valid cause for termination under the terms and conditions of the CBA. Based
on the CBA, the freedom period would start on March 22, 2000.
Atty. Boniel, Manager of the PICOP Legal and Labor Relations Department,
issued a memorandum directing the employees concerned to explain within
seventy-two (72) hours why their employment should not be terminated due to

3E Andaya Ching Espiritu Hefti Galvez Gammad Lainez Lui Madamba Nagera Narvasa
Ong Palangdao Rosales Sanchez Santos Satrain Tabo (2014-2015)

Labor Relations Case Digest - Atty. Joyrich Golangco

alleged acts of disloyalty. Upon receiving their explanation letters, Atty. Boniel
endorsed the same to Atty. Fuentes who then requested the termination of 46
employees found guilty of acts of disloyalty.

PICOP served a notice of termination due to acts of disloyalty to 31 of the


46 employees. Private respondents were among the 31 employees dismissed
from employment by PICOP
Private respondents filed a complaint before the NLRC for Unfair Labor
Practice and Illegal Dismissal with money claims, damages and attorneys fees.
LA- June 9, 2001, declaring as illegal the termination of the private respondents.

NLRC - PICOP elevated the LA decision to the NLRC but its appeal was dismissed
in the November 19, 2002 NLRC Resolution. On motion for reconsideration,
however, the NLRC issued another resolution dated December 27, 2002,
reversing and setting aside its November 19, 2002 Resolution
CA (I included the explanation ng CA in detailed manner kase eto nadin
explanation ng SC sa ruling nila, para hindi na doblihin) - CA rendered the subject
decision reversing and setting aside the December 27, 2002 NLRC resolution and
reinstating the June 9, 2001 Decision of the LA.

The CA ruled, among others, that although private respondents signed


an authorization for the filing of the petition for certification election of a rival
union,
PICOP
Democratic
Trade
Unionist-Federation
of
Free
Workers (FFW), such act was not a sufficient ground to terminate the
employment of private respondents. It explained:
Imputations of an alleged violation of the CBA should
not arise from a vague and all embracing definition of alleged
acts of disloyalty. Neither should it arise from speculative
inferences where no evidence appears from the record that
Respondent NAMAPRI-SPFL expressly defined acts of
disloyalty. Signing an authorization for the filing of the petition
for certification election does not constitute an act of

disloyalty per se. There must be proof of contemporaneous acts


of resignation or withdrawal of their membership from the
Respondent NAMAPRI-SPFL to which they are members.
Petitioners neither joined nor affiliated with FFW and
continuously paid their union dues with Respondent NAMAPRISPFL.
Likewise, the advise of the Respondent NAMAPRI-SPFL
to the Respondent PRI to effect the termination of employees,
including herein Petitioners, finds no basis in fact and in law
considering that at the time the Respondent PRI dismissed the
Petitioners, among others, on 16 November 2000, there was no
more CBA to speak of after it had already expired on 22 May
2000.[

CA further agreed with private respondents that Article 256 and not
Article 253, of the Labor Code applied in this case. The CA discussed this point as
follows:
The issue of acts of disloyalty relates more to a direct
connection on the alleged violation or breach of loyalty to the
majority status of the incumbent union than on violation of the
terms and conditions of the agreement under Article 253.
Article 256 provides that at the expiration of the 60-day period
reckoned from the expiration date of the CBA, the employer
shall continue to recognize the majority status of the incumbent
bargaining agent only where no petition for certification
election is filed. However, a petition was already filed by the
Petitioners, among others, during the 60-day freedom period.
Clearly, from the imports of said provision, it will render
nugatory the purpose of the law providing for a freedom period
for the filing of a petition for certification election should the act
of signing/filing the said petition be interpreted as an act of
disloyalty and will render farce the need for a certification
election as an instrument of ascertaining the true expression of

3E Andaya Ching Espiritu Hefti Galvez Gammad Lainez Lui Madamba Nagera Narvasa
Ong Palangdao Rosales Sanchez Santos Satrain Tabo (2014-2015)

Labor Relations Case Digest - Atty. Joyrich Golangco

the will of the workers as to which labor organization would


represent them.
To construe the provision of law in Article 253 as
imposing a restriction against the signing and filing a petition
for certification election during the freedom period, is to violate
the constitutional right of the employees to organize freely.

PICOP filed this petition for review.

ISSUES:
1. WON the act of signing an authorization for certification election before the
freedom period is an act of disloyalty
2. WON Article 256 of the Labor Code applies in the case.
HELD: The petition merits a denial.

1. No. There is no question that in the CBA entered into by the parties, there is a
union security clause. The clause imposes upon the workers the obligation to join
and maintain membership in the companys recognized union as a condition for
employment.
"Union security" is applied to and comprehends "closed
shop," "union shop," "maintenance of membership," or any
other form of agreement which imposes upon employees the
obligation to acquire or retain union membership as a condition
affecting employment. There is union shop when all new regular
employees are required to join the union within a certain period
as a condition for their continued employment. There is
maintenance of membership shop when employees, who are
union members as of the effective date of the agreement, or who
thereafter become members, must maintain union membership
as a condition for continued employment until they are
promoted or transferred out of the bargaining unit, or the
agreement is terminated. A closed shop, on the other hand, may

be defined as an enterprise in which, by agreement between the


employer and his employees or their representatives, no person
may be employed in any or certain agreed departments of the
enterprise unless he or she is, becomes, and, for the duration of
the agreement, remains a member in good standing of a union
entirely comprised of or of which the employees in interest are
a part.

The burden of proof rests upon management to show that the dismissal
of its worker was based on a just cause. When an employer exercises its power to
terminate an employee by enforcing the union security clause, it needs to
determine and prove the following: (1) the union security clause is applicable;
(2) the union is requesting for the enforcement of the union security provision in
the CBA; and (3) there is sufficient evidence to support the decision of the union
to expel the employee from the union.
Acts of private respondents are not enough proof of a violation of the
Union Security Clause which would warrant their dismissal. (Lahat ng sinabe ng
CA as to this issue was upheld by the SC so paki basa nlng ung nasa FACTS under
CA RULING as the explanation for this ruling by the SC.)

We are constrained to believe that an "authorization letter to file


a petition for certification election" is different from an actual "Petition
for Certification Election." It is clear that the actual Petition for Certification
Election of FFW was filed only on May 18, 2000. Thus, it was within the ambit of
the freedom period which commenced from March 21, 2000 until May 21, 2000.
Strictly speaking, what is prohibited is the filing of a petition for certification
election outside the 60-dayfreedom period. This is not the situation in this case.
If at all, the signing of the authorization to file a certification election was merely
preparatory to the filing of the petition for certification election, or an exercise of
respondents right to self-organization

3E Andaya Ching Espiritu Hefti Galvez Gammad Lainez Lui Madamba Nagera Narvasa
Ong Palangdao Rosales Sanchez Santos Satrain Tabo (2014-2015)

Labor Relations Case Digest - Atty. Joyrich Golangco

2. Yes. The Court agrees with the CA that its (PICOPs) argument is
misplaced. (Under the CA RULING ulit ung explanation dito, paki basa nlng ulit
ung nasa FACTS.)

While it is incumbent for the employer to continue to recognize the majority


status of the incumbent bargaining agent even after the expiration of the
freedom period, they could only do so when no petition for certification election
was filed. The reason is, with a pending petition for certification, any such agreement
entered into by management with a labor organization is fraught with the risk that such
a labor union may not be chosen thereafter as the collective bargaining
representative.
Moreover, the last sentence of Article 253 which provides for automatic renewal
pertains only to the economic provisions of the CBA, and does not
include representational aspect of the CBA. An existing CBA cannot constitute a
bar to a filing of a petition for certification election. When there is a
representational issue, the status quo provision in so far as the need to await the
creation of a new agreement will not apply. Otherwise, it will create an absurd
situation where the union members will be forced to maintain membership by virtue of the
union security clause existing under the CBA and, thereafter, support another
union when filing a petition for certification election. If we apply it, there will
always be an issue of disloyalty whenever the employees exercise their right to
self-organization. The holding of a certification election is a statutory policy that
should not be circumvented, or compromised.
Private respondents are also entitled to an award of attorneys fees
equivalent to 10% of the total monetary award as they were compelled to litigate
in order to seek redress for their illegal dismissal.
102. National Union of Workers in Hotel, Restaurants and Allied IndustriesManila Pavillon Chapter v. Sec of Labor

FACTS: National Union of Workers in Hotels, Restaurants and Allied Industries


Manila Pavilion Hotel Chapter (NUWHRAIN-MPHC), herein petitioner, seeks the
reversal of the Court of Appeals November 8, 2007 Decision and of the Secretary
of Labor and Employments January 25, 2008 Resolution in OS-A-9-52-05 which

affirmed the Med-Arbiters Resolutions dated January 22, 2007 and March 22, 20
07.
A certification election was conducted on June 16, 2006 among the rank-an
d-file employees of respondent Holiday Inn Manila Pavilion Hotel (the Hotel) wit
h the following results:
EMPLOYEES IN VOTERS LIST = 353
TOTAL VOTES CAST
= 346
NUWHRAIN-MPHC = 151
HIMPHLU
= 169
NO UNION
= 1
SPOILED
= 3
SEGREGATED
= 22

SOLE concluded that the certification of HIMPHLU as the exclusive bargaining ag


ent was proper.
Petitioner, which garnered 151 votes, appealed to the Secretary of Labor and Em
ployment (SOLE), arguing that the votes of the probationary employees should h
ave been opened considering that probationary employee Gatbontons vote was t
allied. And petitioner averred that respondent HIMPHLU, which garnered 169 vo
tes, should not be immediately certified as the bargaining agent, as the opening o
f the 17 segregated ballots would push the number of valid votes cast to 338 (15
1 + 169 + 1 + 17), hence, the 169 votes which HIMPHLU garnered would be one v
ote short of the majority which would then become 169.
ISSUES: WON HIMPHLU is the winner of the certification election

HELD: No.As to whether HIMPHLU should be certified as the exclusive bargainin


g agent, the Court rules in the negative. It is well-settled that under the so-called "
double majority rule," for there to be a valid certification election, majority of the
bargaining unit must have voted AND the winning union must have garnered maj
ority of the valid votes cast.
Prescinding from the Courts ruling that all the probationary employees
votes should be deemed valid votes while that of the supervisory employees sho
uld be excluded, it follows that the number of valid votes cast would increase fr

3E Andaya Ching Espiritu Hefti Galvez Gammad Lainez Lui Madamba Nagera Narvasa
Ong Palangdao Rosales Sanchez Santos Satrain Tabo (2014-2015)

Labor Relations Case Digest - Atty. Joyrich Golangco

om 321 to 337. Under Art. 256 of the Labor Code, the union obtaining the majorit
y of the valid votes cast by the eligible voters shall be certified as the sole and exc
lusive bargaining agent of all the workers in the appropriate bargaining unit. This
majority is 50% + 1. Hence, 50% of 337 is 168.5 + 1 or at least 170.
HIMPHLU obtained 169 while petitioner received 151 votes. Clearly, HI
MPHLU was not able to obtain a majority vote. The position of both the SOLE and
the appellate court that the opening of the 17 segregated ballots will not material
ly affect the outcome of the certification election as for, so they contend, even if s
uch member were all in favor of petitioner, still, HIMPHLU would win, is thus unt
enable.
It bears reiteration that the true importance of ascertaining the number
of valid votes cast is for it to serve as basis for computing the required majority, a
nd not just to determine which union won the elections. The opening of the segre
gated but valid votes has thus become material. To be sure, the conduct of a certif
ication election has a two-fold objective: to determine the appropriate bargaining
unit and to ascertain the majority representation of the bargaining representativ
e, if the employees desire to be represented at all by anyone. It is not simply the d
etermination of who between two or more contending unions won, but whether i
t effectively ascertains the will of the members of the bargaining unit as to wheth
er they want to be represented and which union they want to represent them.
Having declared that no choice in the certification election conducted ob
tained the required majority, it follows that a run-off election must be held to det
ermine which between HIMPHLU and petitioner should represent the rank-and-f
ile employees.
103. MIGUELA SANTUYO, CORAZON ZACARIAS, EUGENIA CINCO, ELIZABETH
PERALES, SUSANA BELEDIANO, RUFINA TABINAS, LETICIA L. DELA ROSA,
NENITA LINESES, EDITHA DELA RAMA, MARIBEL M. OLIVAR, LOEVEL
MALAPAD, FLORENDA M. GONZALO, ELEANOR O. BUEN, EULALIA ABAGAO,
LORECA MOCORRO, DIANA MAGDUA, LUZ RAGAY, LYDIA MONTE, CORNELIA
BALTAZAR and DAISY MANGANTE, Petitioners,
vs.REMERCO GARMENTS MANUFACTURING, INC. and/or VICTORIA REYES.
Respondents.

FACTS: From 1992 to 1994, due to a serious industrial dispute, the Kaisahan ng
Manggagawa sa Remerco Garments Manufacturing Inc.- KMM Kilusan (union)

staged a strike against respondent Remerco Garments Manufacturing, Inc.


(RGMI). Because the strike was subsequently declared illegal, all union officers
were dismissed. Employees who wanted to sever their employment were paid
separation pay while those who wanted to resume work were recalled on the
condition that they would no longer be paid a daily rate but on a piece-rate basis.

Without allowing RGMI to normalize its operations, the union filed a notice of
strike in the National Conciliation and Mediation Board (NCMB). According to the
union, RGMI conducted a time and motion study and changed the salary scheme
from a daily rate to piece-rate basis without consulting it. RGMI therefore not
only violated the existing collective bargaining agreement (CBA) but also
diminished the salaries agreed upon. On November 11, 1995, while the union
and RGMI were undergoing conciliation in the NCMB, RGMI transferred its
factory site. Two days after, the union went on strike and blocked the entry to
RGMIs (new) premises. The Secretary of Labor assumed jurisdiction pursuant to
Article 263(g) of the Labor Code and ordered RGMIs striking workers to return
to work immediately. He likewise ordered the union and RGMI to submit their
respective position papers. In its position paper, the union denied going on strike
and blocking entries (and exits) at RGMIs premises. Furthermore, the union
enumerated RGMIs alleged unfair labor practices. RGMI, on the other hand,
insisted that its employees refused to obey the November 21, 1995 order. Thus,
it prayed that the strike be declared illegal and that all union officers and those
employees who refused to return to work be declared to have abandoned their
employment. The Secretary of Labor held that RGMI did not lock out its
employees inasmuch as it informed them of the transfer of the worksite.
However, he did not rule on the legality of the strike. Furthermore, based on the
time and motion study, the Secretary of Labor found that the employees would
receive higher wages if they were paid on a piece-rate rather than on a daily rate
basis. Hence, the new salary scheme would be more advantageous to the
employees. the Secretary of Labor ordered all employees to return to work and
RGMI to pay its employees their unpaid salaries and neither parties appealed.
While the conciliation proceedings between the union and respondent were
pending, petitioners filed a complaint for illegal dismissal against RGMI and
respondent Victoria Reyes, accusing the latter of harassment. Respondents, on
the other hand, moved to dismiss the complaint in view of the pending

3E Andaya Ching Espiritu Hefti Galvez Gammad Lainez Lui Madamba Nagera Narvasa
Ong Palangdao Rosales Sanchez Santos Satrain Tabo (2014-2015)

Labor Relations Case Digest - Atty. Joyrich Golangco

conciliation proceedings (which involved the same issue) in the NCMB.


Moreover, alleged violations of the CBA should be resolved according to the
grievance procedure laid out therein.Thus, the labor arbiter had no jurisdiction
over the complaint. The labor arbiter found that respondents did not pay
petitioners their salaries and deprived them of the benefits they were entitled to
under the CBA and ordered that petitioners be paid of their unpaid salaries.
Petitioners moved for reconsideration but it was denied. Hence this recourse

ISSUES: WON the dispute falls under the grievance procedure laid out in the CBA
and if in the affirmative does the labor arbiter assume jurisdiction
HELD: Article 217. Jurisdiction of Labor Arbiters and the Commission. x x
x
xxx
xxx

(c) Cases arising from the interpretation or implementation of collective


bargaining agreements and those arising from the interpretation or
enforcement of company personnel policies shall be disposed of by the Labor
Arbiter by referring the same to the grievance machinery and voluntary
arbitration as may be provided in said agreements. (emphasis supplied)
This provision requires labor arbiters to refer cases involving the
implementation of CBAs to the grievance machinery provided therein and to
voluntary arbitration.

Moreover, Article 260 of the Labor Code clarifies that such disputes must be
referred first to the grievance machinery and, if unresolved within seven days,
they shall automatically be referred to voluntary arbitration. In this regard,
Article 261 thereof states:
Article 261. Jurisdiction of voluntary arbitrators and panel of voluntary
arbitrators. The Voluntary Arbitrator or panel of Voluntary Arbitrators
shall have original and exclusive jurisdiction to hear and decide all
unresolved grievances arising from the interpretation or implementation of
the Collective Bargaining Agreement and those arising from the interpretation
or enforcement of company personnel policies referred to in the immediately
preceding Article. Accordingly, violations of a Collective Bargaining
Agreement, except those which are gross in character, shall no longer be

treated as unfair labor practice and shall be resolved as grievances under


the Collective Bargaining Agreement. For purposes of this Article, gross
violations of a Collective Bargaining Agreement shall mean flagrant and/or
malicious refusal to comply with the economic provisions of such agreement.

Pursuant to Articles 217 in relation to Articles 260 and 261 of the Labor Code,
the labor arbiter should have referred the matter to the grievance machinery
provided in the CBA. Because the labor arbiter clearly did not have jurisdiction
over the subject matter, his decision was void.1avvphiNonetheless, the Secretary
of the Labor assumed jurisdiction over the labor dispute between the union and
RGMI and resolved the same in his September 18, 1996 order. Article 263(g) of
the Labor Code gives the Secretary of Labor discretion to assume jurisdiction
over a labor dispute likely to cause a strike or a lockout in an industry
indispensable to the national interest and to decide the controversy or to refer
the same to the NLRC for compulsory arbitration. In doing so, the Secretary of
Labor shall resolve all questions and controversies in order to settle the dispute.
His power is therefore plenary and discretionary in nature to enable him to
effectively and efficiently dispose of the issue.
104. Teng vs Pahagac

FACTS: Albert Teng Fish Trading is engaged in deep sea fishing and, for this
purpose, owns boats (basnig), equipment, and other fishing paraphernalia. As
owner of the business, Teng claims that he customarily enters into joint venture
agreements with master fishermen (maestros) who are skilled and are experts in
deep sea fishing; they take charge of the management of each fishing venture,
including the hiring of the members of its complement. He avers that the
maestros hired the respondent workers as checkers to determine the volume of
the fish caught in every fishing voyage.

On February 20, 2003, the respondent workers filed a complaint for


illegal dismissal against Albert Teng Fish Trading, Teng, and Chua before
the NCMB, Region Branch No. IX, Zamboanga City.
The respondent workers alleged that Teng hired them, without any
written employment contract, to serve as his "eyes and ears" aboard the fishing
boats; to classify the fish caught by baera; to report to Teng via radio

3E Andaya Ching Espiritu Hefti Galvez Gammad Lainez Lui Madamba Nagera Narvasa
Ong Palangdao Rosales Sanchez Santos Satrain Tabo (2014-2015)

Labor Relations Case Digest - Atty. Joyrich Golangco

communication the classes and volume of each catch; to receive instructions


from him as to where and when to unload the catch; to prepare the list of the
provisions requested by the maestro and the mechanic for his approval; and, to
procure the items as approved by him. They also claimed that they received
regular monthly salaries, 13th month pay, Christmas bonus, and incentives in the
form of shares in the total volume of fish caught.
They asserted that sometime in September 2002, Teng expressed his
doubts on the correct volume of fish caught in every fishing voyage. In December
2002, Teng informed them that their services had been terminated. In his
defense, Teng maintained that he did not have any hand in hiring the respondent
workers; the maestros, rather than he, invited them to join the venture.
According to him, his role was clearly limited to the provision of the necessary
capital, tools and equipment, consisting of basnig, gears, fuel, food, and other
supplies.

The VA rendered a decision in Tengs favor and declared that no


employer-employee relationship existed between Teng and the respondent
workers.
The respondent workers received the VAs decision on June 12,
2003. They filed a motion for reconsideration, which was denied in an
order dated June 27, 2003 and which they received on July 8, 2003. The VA
reasoned out that Section 6, Rule VII of the 1989 Procedural Guidelines in the
Conduct of Voluntary Arbitration Proceedings (1989 Procedural Guidelines)
does not provide the remedy of a motion for reconsideration to the party
adversely affected by the VAs order or decision.
On July 21, 2003, the respondent-workers elevated the case to the CA. In
its decision of September 21, 2004, the CA reversed the VAs decision after
finding sufficient evidence showing the existence of employer-employee
relationship.

ISSUES: 1) Whether the VAs decision is subject to a motion for reconsideration;


and
2) Whether an employer-employee relationship existed between Teng and the
respondent workers.

HELD:

1) Yes, it is. Article 262-A of the Labor Code does not prohibit the filing of a
motion for reconsideration.

On March 21, 1989, Republic Act No. 6715 took effect, amending, among others,
Article 263 of the Labor Code which was originally worded as:
Art. 263 x x x Voluntary arbitration awards or decisions shall be
final, unappealable, and executory.

As amended, Article 263 is now Article 262-A, which states:

Art. 262-A. x x x [T]he award or decision x x x shall contain the


facts and the law on which it is based. It shall be final and
executory after ten (10) calendar days from receipt of the
copy of the award or decision by the parties.

Notably, Article 262-A deleted the word "unappealable" from Article


263. Previous rulings fully establish that the absence of a categorical language in
Article 262-A does not preclude the filing of a motion for reconsideration of the
VAs decision within the 10-day period. Tengs allegation that the VAs decision
had become final and executory by the time the respondent workers filed an
appeal with the CA thus fails. The Court rules that the respondent workers
seasonably filed a motion for reconsideration of the VAs judgment, and the
VA erred in denying the motion because no motion for reconsideration is
allowed.
2) Yes, an employer-employee relationship existed.

While Teng alleged that it was the maestros who hired the respondent
workers, it was his company that issued to the respondent workers identification
cards (IDs) bearing their names as employees and Tengs signature as the
employer.

For the 13 years that the respondent workers worked for Teng, they
received wages on a regular basis, in addition to their shares in the fish
caught. The worksheet showed that the respondent workers received uniform

3E Andaya Ching Espiritu Hefti Galvez Gammad Lainez Lui Madamba Nagera Narvasa
Ong Palangdao Rosales Sanchez Santos Satrain Tabo (2014-2015)

Labor Relations Case Digest - Atty. Joyrich Golangco

amounts within a given year, which amounts annually increased until the
termination of their employment in 2002.

More importantly, the element of control which we have ruled in a


number of cases to be a strong indicator of the existence of an employeremployee relationship is present in this case. Teng not only owned the tools
and equipment, he directed how the respondent workers were to perform their
job as checkers; they, in fact, acted as Tengs eyes and ears in every fishing
expedition.

Teng cannot hide behind his argument that the respondent workers were
hired by the maestros. To consider the respondent workers as employees of the
maestros would mean that Teng committed impermissible labor-only
contracting. Accordingly, the Court holds that employer-employee ties exist
between Teng and the respondent workers. A finding that the maestros are
labor-only contractors is equivalent to a finding that an employer-employee
relationship exists between Teng and the respondent workers. As regular
employees, the respondent workers are entitled to all the benefits and rights
appurtenant to regular employment. And for his failure to comply with the Labor
Codes substantive requirement on termination of employment, the Court
declares that Teng illegally dismissed the respondent workers.
105. SAMASAH-NUWHRAIN (Hyatt) vs MAGSALIN

FACTS: Petitioner is a duly registered union and the certified bargaining


representative of the rank-and-file employees of Hyatt Regency Manila, a five-star
hotel owned and operated by respondent Hotel Enterprises of the Philippines, Inc.
Hyatts General Manager issued a Memorandum informing all hotel
employees that hotel security have been instructed to conduct a thorough bag
inspection and body frisking in every entrance and exit of the hotel. He enjoined
employees to comply therewith.
Angelito Caragdag, a waiter at the hotels Cafe Al Fresco restaurant and a
director of the union, refused to be frisked by the security personnel. The incident
was reported to the hotels Human Resources Department (HRD), which he was
imposed a penalty of reprimand. Caragdag again refused to be frisked by the security
personnel, HRD suspended him for three days which was considered as a second
offense. Both penalties were in accordance with the hotels Code of Discipline.

Subsequently, Moral, manager of Hyatts Cafe Al Fresco and Caragdags


immediate superior, was about to counsel two staff members, at the training room,
Caragdag suddenly opened the door and yelled at the two with an enraged look. In a
disturbing voice he said, Ang titigas talaga ng ulo nyo. Sinabi ko na sa inyo na huwag
kayong makikipagusap sa management habang ongoing pa ang kaso! Moral held
Caragdag liable for Offenses Subject to Disciplinary Action (OSDA) 3.01 of the hotels
Code of Discipline, i.e., threatening, intimidating, coercing, and provoking to a fight
your superior for reasons directly connected with his discharge of official duty.
Thus, Caragdag was imposed the penalty of seven days suspension in accordance
with the hotels Code of Discipline.
Caragdag committed another infraction. Caragdag left his work assignment
during official hours without prior permission from his Department Head. Moral
found Caragdag liable for violating OSDA 3.07, i.e., leaving work assignment during
official working hours without prior permission from the department head or
immediate superior, and suspended him for three days.
Because of the succession of infractions he committed, the HRD also
required Caragdag to explain why the hotels OSDA 4.32 (Committing offenses which
are penalized with 3 suspensions during a 12-month period) should not be enforced
against him. Despite notice of the scheduled hearing, both Caragdag and the Union
President failed to attend. Thereafter, the investigating board resolved on the said
date to dismiss Caragdag for violation of OSDA 4.32. Caragdag appealed but the
investigating board affirmed its resolution after hearing.
Hotel then sent Caragdag a notice of dismissal. Caragdags dismissal was
questioned by petitioner, and the dispute was referred to voluntary arbitration upon
agreement of the parties. The Voluntary Arbitrator ruled that the dismissal was valid.
Petitioner assailed the decision of the Voluntary Arbitrator before the CA in a
petition for certiorari . As mentioned at the outset, the CA dismissed the
petition outright for being the wrong remedy.
CA explained that Rule 43, Section 5 of the 1997 Rules of Civil Procedure
explicitly provides that the proper mode of appeal from judgments, final orders or
resolution of voluntary arbitrators is through a Petition for Review which should be
filed within fifteen (15) days from the receipt of notice of judgment, order or
resolution of the voluntary arbitrator. Considering that petitioner intends this
petition to be a Petition for Certiorari, the Court hereby resolves to dismiss the petition
outright for being an improper mode of appeal.

3E Andaya Ching Espiritu Hefti Galvez Gammad Lainez Lui Madamba Nagera Narvasa
Ong Palangdao Rosales Sanchez Santos Satrain Tabo (2014-2015)

Labor Relations Case Digest - Atty. Joyrich Golangco

Even if this Court treats the instant petition as a Petition for Review, still the
Court has no alternative but to dismiss the same for having been filed out of time. As
admitted by the petitioner it received the Order dated 26 May 2003 denying their
motion for reconsideration on 02 June 2003. The fifteen (15) day period within
which to appeal through a Petition for Review is until June 17, 2003. The
petitioner filed the present petition on August 1, 2003, way beyond the
reglementary period provided for by the Rules.

Petitioner: It argues that because decisions rendered by voluntary arbitrators are


issued under Title VII-A of the Labor Code, they are not covered by Rule 43 of
the 1997 Rules of Civil Procedure, as amended, by express provision of Section 2
thereof. Section 2, petitioner points out, expressly provides that Rule 43 shall not
apply to judgments or final orders issued under the Labor Code of
the Philippines. Hence, a petition for certiorari under Rule 65 is the proper remedy
for questioning the decision of the Voluntary Arbitrator, and petitioner having availed
of such remedy, the CA erred in declaring that the petition was filed out of time since
the petition was filed within the sixty (60)-day reglementary period.
Respondent: Maintains that the CA acted correctly in dismissing the petition
for certiorari for being the wrong mode of appeal. It stresses that Section 1 of Rule 43
clearly states that it is the governing rule with regard to appeals from awards,
judgments, final orders or resolutions of voluntary arbitrators. Respondent contends
that the voluntary arbitrators authorized by law include the voluntary arbitrators
appointed and accredited under the Labor Code, as they are considered as included in
the term quasi-judicial instrumentalities.
ISSUES: Whether or not petition for certiorari is an improper mode of appeal?

HELD: Yes. The proper remedy is a petition for review under Rule 43. The
question on the proper recourse to assail a decision of a voluntary arbitrator has
already been settled in Luzon Development Bank v. Association of Luzon
Development Bank Employees, where the Court held that the decision or award of
the voluntary arbitrator or panel of arbitrators should likewise be appealable to
the Court of Appeals, in line with the procedure outlined in Rule 43 of the 1997
Rules of Civil Procedure, just like those of the quasi-judicial agencies, boards and
commissions enumerated therein, and consistent with the original purpose to

provide a uniform procedure for the appellate review of adjudications of all


quasi-judicial entities.
Decisions of voluntary arbitrators issued pursuant to the
Labor Code do not come within the exception provided in Sec.2 of
Rule 43.

Hence, upon receipt on May 26, 2003 of the Voluntary Arbitrators


Resolution denying petitioners motion for reconsideration, petitioner should
have filed with the CA, within the fifteen (15)-day reglementary period, a petition
for review, not a petition for certiorari.
106. YSS EMPLOYEES UNION [SEBA] v. YSS LABORATORIES

FACTS: To arrest escalating business losses, the company implemented a


retrenchment program, which affected 11 employees purportedly chosen in
accordance with reasonable standards. Of the 11, 9 were YSSEU officers and
members. Initially, they were given the option to avail of the early retirement
program, but when no one chose to avail of it, the company exercised its option
to terminate the services of its employees as allegedly authorized under LC 283.
YSSEU decided to hold a strike, claiming that the company was guilty of
discrimination and union-busting in carrying out the said retrenchment
program. They staged a strike after the necessary strike vote was taken under
NCMB-NCRs supervision. A number of conciliation proceedings were conducted,
but these efforts proved futile.

In an Order, the SOLE certified the labor dispute to the NLRC for compulsory
arbitration. All striking workers were thereby directed to return to work within
24 hours from receipt of the Order, and YSS Laboratories to accept them under
the terms and conditions prevailing before the strike. Underscoring the
governments policy of preserving economic gains and employment levels, the
SOLE deemed that the continuation of the labor dispute was inimical to national
interest.
YSS Laboratories, however, refused to fully comply. In its Urgent Motion for
Reconsideration, it argued that the 9 union officers and members [previously

3E Andaya Ching Espiritu Hefti Galvez Gammad Lainez Lui Madamba Nagera Narvasa
Ong Palangdao Rosales Sanchez Santos Satrain Tabo (2014-2015)

Labor Relations Case Digest - Atty. Joyrich Golangco

terminated from service pursuant to a valid retrenchment] should be excluded


from the operation of the return-to-work order, and that the union officers who
participated in the purported illegal strike should likewise not be allowed to be
back to their employment deemed to have already lost their employment
status.
YSSEU moved that YSS Laboratories be cited for contempt for refusing to admit
the 18 workers back to work. YSSEU prayed for the award of backwages in favor
of these employees SOLE GRANTED THE UNIONS
MOTION: Accept 18 workers pending determination of the validity of the
retrenchment and illegal strike cases.

The company won at CA level, since the CA REVERSED THE TWO SOLE
ORDERSmade with grave abuse of discretion. CA found that the company
validly carried out the retrenchment program, which severed the concerned
employees.Strike employment was considered as illegal for lack of factual and
legal basis.
UNIONS CONTENTIONS

Once a labor dispute is certified to the NLRC for compulsory arbitration,


the employer should readily admit all striking employees under the status quo
ante.

The primary reason why the strike was conducted was to protest the
implementation of the retrenchment program, which clearly discriminated
against union officers and members.
COMPANYS ARGUMENTS

Those employees who were separated from service due to a valid


retrenchment should not be readmitted.

The retrenched employees were chosen after a thorough evaluation of


their work performance, including their frequencies of absence and tardiness,
and their respective lengths of service.

ISSUE :WON the retrenched employees should be excluded from the coverage of
the return-to-work-order. NO
HELD: The SOLE Orders certifying the labor dispute to the NLRC for compulsory
arbitration, and enjoining YSSEU to return to work and YSS Laboratories to
admit them, were issued pursuant to LC 263(g).
Art. 263. Strikes, picketing, and lockouts.

(g) When, in his opinion, there exists a labor dispute causing or likely to cause a
strike or lockout in an industry indispensable to the national interest, the
Secretary of Labor and Employment may assume jurisdiction over the dispute
and decide it or certify the same to the Commission for compulsory arbitration.
Such assumption or certification shall have the effect of automatically enjoining
the intended or impending strike or lockout as specified in the assumption or
certification order. If one has already taken place at the time of assumption or
certification, all striking or locked out employees shall immediately return to
work and the employer shall immediately resume operations and readmit all
workers under the same terms and conditions prevailing before the strike or
lockout. The Secretary of Labor and Employment or the Commission may seek
the assistance of law enforcement agencies to ensure compliance with this
provision as well as with such orders as he may issue to enforce the same.

After martial law was lifted and democracy was restored, the assumption of
jurisdiction in Art. 263(g) has now been viewed as an exercise of the police
power of the State with the aim of promoting the common good. The grant of
these plenary powers to the SOLE makes it incumbent upon him to bring about
soonest, a fair and just solution to the differences between the employer and the
employees, so that the damage such labor dispute might cause upon the national
interest may be minimized as much as possible, if not totally averted, by avoiding
stoppage of work or any lag in the activities of the industry or the possibility of
those contingencies that might cause detriment to the national interest.

3E Andaya Ching Espiritu Hefti Galvez Gammad Lainez Lui Madamba Nagera Narvasa
Ong Palangdao Rosales Sanchez Santos Satrain Tabo (2014-2015)

Labor Relations Case Digest - Atty. Joyrich Golangco

To effectively achieve such end, the assumption or certification order shall have
the effect of automatically enjoining the intended or impending strike or lockout.
If one has already taken place, all striking workers shall immediately return to
work, and the employer shall immediately resume operations and readmit all
workers under the same terms and conditions prevailing before the strike or
lockout. YSS

Laboratories vigorous insistence SOLEs seriously authority to impairs forestall


labor dispute that he deems inimical to the national economy. The SOLE is
afforded plenary and broad powers, and is granted great breadth of discretion to
adopt the most reasonable and expeditious way of writing finis to the labor
dispute.

When the SOLE directed YSS Laboratories to accept all the striking workers back
to work, the Secretary did not exceed his jurisdiction, or gravely abuse the same.
There is no showing that the orders were issued in an arbitrary or despotic
manner. The Orders were issued with the end in view of preserving the status
quo ante while the main issues of the validity of the retrenchment and legality of
the strike were being threshed out in the proper forum. This was done for the
promotion of the common good and to maintain industrial peace, considering
that a lingering strike could be inimical to the interest of both employer and
employee. His certification for compulsory arbitration is not intended to
interfere with the managements rights but to obtain a speedy settlement of the
dispute.
YSS Laboratories undermines the underlying principle embodied in LC 263(g) on
the settlement of labor disputes -- that assumption and certification orders are
executory in character and are to be strictly complied with by the parties, even
during the pendency of any petition questioning their validity. Regardless
therefore of its motives, or of the validity of its claims, YSS Laboratories must
readmit all striking employees and give them back their respective jobs.
Accepting back the workers in this case is not a matter of option, but of
obligation mandated by law. Its compulsory character is mandated, not to cater
to a narrow segment of society, or to favor labor at the expense of management,
but to serve the greater interest of society by maintaining economic equilibrium.
PALEA v. PAL

The nature of a return-to-work order issued in a certified case lends itself to no


other construction. The certification attests to the urgency of the matter,
affecting as it does an industry indispensable to the national interest. The order
is issued in the exercise of the courts compulsory power beyed of until
arbitration, set aside. and

The determination of who among the strikers could be admitted back to work
cannot be made to depend upon the discretion of employer, lest we strip the
certification or assumption-of-jurisdiction orders of the coercive power that is
necessary for attaining their laudable objective. The return-to-work order does
not interfere with the managements prerogative, but merely regulates it when,
in the exercise of such right, national interests will be affected. Constitutionallygranted rights are not absolute. They are still subject to control and limitation to
ensure that they are not exercised arbitrarily. The interests of both the
employers and employees are intended to be protected and not one of them is
given undue preference.
107. NATIONAL UNION OF WORKERS IN THE HOTEL RESTAURANT AND
ALLIED INDUSTRIES (NUWHRAIN-APL-IUF) DUSIT HOTEL NIKKO CHAPTER,
Vs. THE HONORABLE COURT OF APPEALS (Former Eighth Division), THE
NATIONAL LABOR RELATIONS COMMISSION (NLRC), PHILIPPINE
HOTELIERS INC., owner and operator of DUSIT HOTEL NIKKO and/or
CHIYUKI FUJIMOTO, and ESPERANZA V. ALVEZ,
FACTS: On October 24, 2000, the Union submitted its Collective Bargaining
Agreement (CBA) negotiation proposals to the Hotel. As negotiations ensued,
the parties failed to arrive at mutually acceptable terms and conditions. Due
to the bargaining deadlock, the Union, on December 20, 2001, filed a Notice
of Strike on the ground of the bargaining deadlock with the National
Conciliation and Mediation Board (NCMB), which was docketed as NCMBNCR-NS-12-369-01. Thereafter, conciliation hearings were conducted which
proved unsuccessful. Consequently, a Strike Vote was conducted by the
Union on January 14, 2002 on which it was decided that the Union would
wage a strike. Soon thereafter, in the afternoon of January 17, 2002, the
Union held a general assembly at its office located in the Hotels basement,
where some members sported closely cropped hair or cleanly shaven heads.

3E Andaya Ching Espiritu Hefti Galvez Gammad Lainez Lui Madamba Nagera Narvasa
Ong Palangdao Rosales Sanchez Santos Satrain Tabo (2014-2015)

Labor Relations Case Digest - Atty. Joyrich Golangco

The next day, or on January 18, 2002, more male Union members came to
work sporting the same hair style. The Hotel prevented these workers from
entering the premises claiming that they violated the Hotels Grooming
Standards.

In view of the Hotels action, the Union staged a picket outside the Hotel
premises. Later, other workers were also prevented from entering the Hotel
causing them to join the picket. For this reason the Hotel experienced a severe
lack of manpower which forced them to temporarily cease operations in three
restaurants.

Subsequently, on January 20, 2002, the Hotel issued notices to Union


members, preventively suspending them and charging them with the following
offenses: (1) violation of the duty to bargain in good faith; (2) illegal picket; (3)
unfair labor practice; (4) violation of the Hotels Grooming Standards; (5) illegal
strike; and (6) commission of illegal acts during the illegal strike. The next day,
the Union filed with the NCMB a second Notice of Strike on the ground of unfair
labor practice and violation of Article 248(a) of the Labor Code on illegal lockout,
which was docketed as NCMB-NCR-NS-01-019-02. In the meantime, the Union
officers and members submitted their explanations to the charges alleged by the
Hotel, while they continued to stage a picket just inside the Hotels compound.

On January 26, 2002, the Hotel terminated the services of twenty-nine


(29) Union officers and sixty-one (61) members; and suspended eighty-one (81)
employees for 30 days, forty-eight (48) employees for 15 days, four (4)
employees for 10 days, and three (3) employees for five days. On the same day,
the Union declared a strike. Starting that day, the Union engaged in picketing the
premises of the Hotel. During the picket, the Union officials and members
unlawfully blocked the ingress and egress of the Hotel premises.

NLRC: The NLRC held that the January 18, 2002 concerted action was an illegal
strike in which illegal acts were committed by the Union; and that the strike
violated the No Strike, No Lockout provision of the CBA. The NLRC explained
that the strike which occurred on January 18, 2002 was illegal because it failed to
comply with the mandatory 30-day cooling-off period and the seven-day
strike ban. The NLRC also ruled that even if the Union had complied with the

temporal requirements mandated by law, the strike would nonetheless be


declared illegal because it was attended by illegal acts committed by the Union
officers and members.
CA: Dismissed the Unions petition and affirmed the rulings of the NLRC.
ISSUES: Whether the strike was illegal.

HELD: YES, First, the Unions violation of the Hotels Grooming Standards was
clearly a deliberate and concerted action to undermine the authority of and to
embarrass the Hotel and was, therefore, not a protected action. The appearances
of the Hotel employees directly reflect the character and well-being of the Hotel,
being a five-star hotel that provides service to top-notch clients. This Court is of
the opinion, therefore, that the act of the Union was not merely an expression of
their grievance or displeasure but, indeed, a calibrated and calculated act
designed to inflict serious damage to the Hotels finances or its reputation.

Second, the Unions concerted action which disrupted the Hotels


operations clearly violated the CBAs No Strike, No Lockout provision. The facts
are clear that the strike arose out of a bargaining deadlock in the CBA
negotiations with the Hotel.
Third, the Union officers and members concerted action to shave their
heads and crop their hair not only violated the Hotels Grooming Standards but
also violated the Unions duty and responsibility to bargain in good faith. By
shaving their heads and cropping their hair, the Union officers and members
violated then Section 6, Rule XIII of the Implementing Rules of Book V of the
Labor Code. This rule prohibits the commission of any act which will disrupt or
impede the early settlement of the labor disputes that are under conciliation.
Since the bargaining deadlock is being conciliated by the NCMB, the Unions
action to have their officers and members heads shaved was manifestly
calculated to antagonize and embarrass the Hotel management and in doing so
effectively disrupted the operations of the Hotel and violated their duty to
bargain collectively in good faith.

Fourth, the Union failed to observe the mandatory 30-day cooling-off


period and the seven-day strike ban before it conducted the strike on January 18,
2002. The NLRC correctly held that the Union failed to observe the mandatory

3E Andaya Ching Espiritu Hefti Galvez Gammad Lainez Lui Madamba Nagera Narvasa
Ong Palangdao Rosales Sanchez Santos Satrain Tabo (2014-2015)

Labor Relations Case Digest - Atty. Joyrich Golangco

periods before conducting or holding a strike. Records reveal that the Union
filed its Notice of Strike on the ground of bargaining deadlock on December 20,
2001. The 30-day cooling-off period should have been until January 19, 2002. On
top of that, the strike vote was held on January 14, 2002 and was submitted to
the NCMB only on January 18, 2002; therefore, the 7-day strike ban should have
prevented them from holding a strike until January 25, 2002.

Last, the Union committed illegal acts in the conduct of its strike. The
NLRC ruled that the strike was illegal since, as shown by the pictures presented
by the Hotel, the Union officers and members formed human barricades and
obstructed the driveway of the Hotel. There is no merit in the Unions argument
that it was not its members but the Hotels security guards and the police officers
who blocked the driveway, as it can be seen that the guards and/or police
officers were just trying to secure the entrance to the Hotel. The pictures clearly
demonstrate the tense and highly explosive situation brought about by the
strikers presence in the Hotels driveway.
108. JACKBILT INDUSTRIES, INC., Petitioner, vs. JACKBILT EMPLOYEES
WORKERS UNION-NAFLU-KMU, Respondent.

FACTS: Due to the adverse effects of the Asian economic crisis on the
construction industry beginning 1997, petitioner Jackbilt Industries, Inc. decided
to temporarily stop its business of producing concrete hollow blocks, compelling
most of its employees to go on leave for six months.

Respondent Jackbilt Employees Workers Union-NAFLU-KMU immediately


protested the temporary shutdown. Because its collective bargaining agreement
with petitioner was expiring during the period of the shutdown, respondent
claimed that petitioner halted production to avoid its duty to bargain collectively.
The shutdown was allegedly motivated by anti-union sentiments.
Accordingly, respondent went on strike. Its officers and members picketed
petitioners main gates and deliberately prevented persons and vehicles from
going into and out of the compound.

Petitioner filed a petition for injunction with a prayer for the issuance of a TRO in
the NLRC to enjoin respondent from obstructing free entry to and exit from its
production facility. NLRC issued a TRO.
However, respondent union violated the order. NLRC ordered the issuance of a
writ of preliminary injunction.

Petitioner ordered [61] officers and members of respondent who participated in


the strike to explain why they should not be dismissed. Respondent repeatedly
ignored petitioners memoranda. Thus, they were dismissed.
Aggrieved, respondent filed complaints for illegal lockout, runaway shop and
damages, unfair labor practice, illegal dismissal and attorneys fees, and refusal
to bargain on behalf of its officers and members against petitioner and its
corporate officers. It argued that there was no basis for the temporary partial
shutdown as it was undertaken by petitioner to avoid its duty to bargain
collectively.

LA: dismissed the complaints for illegal lockout and unfair labor practice for lack
of merit. However, because petitioner did not file a petition to declare the strike
illegal before terminating respondents officers and employees, it was found
guilty of illegal dismissal. Petitioner and its officers were held solidarily liable for
backwages from June 1, 1998 (date of dismissal).
NLRC: (Dec 28, 2000): held that only petitioner should be liable. Both petitioner
and respondent moved for reconsideration but they were denied for lack of
merit.
Petitioner filed a petition for certiorari in the CA, asserting that in view of the
NLRC decision finding respondents officers and employees to have committed
illegal acts in the course of the strike, and pursuant to Article 264(a)(3) of the
Labor Code, petitioner validly terminated respondents officers and employees.

CA: dismissed the petition. It further held that, because most of affected
employees were union members, the temporary shutdown was moved by antiunion sentiments. Thus, petitioner was guilty of unfair labor practice and,

3E Andaya Ching Espiritu Hefti Galvez Gammad Lainez Lui Madamba Nagera Narvasa
Ong Palangdao Rosales Sanchez Santos Satrain Tabo (2014-2015)

Labor Relations Case Digest - Atty. Joyrich Golangco

consequently, was ordered to pay respondents officers and employees


backwages from March 9, 1998 (the date respondent went on strike) (instead of
June 1, 1998) to October 4, 1998 and separation pay.
Petitioner moved for reconsideration but it was denied. Thus, this recourse.

ISSUES: Whether or not the filing of a petition with the labor arbiter to declare a
strike illegal is a condition sine qua non for the valid termination of employees
who commit an illegal act in the course of such strike.
HELD : No. The principle of conclusiveness of judgment, embodied in Section
47(c), Rule 39 of the Rules of Court,24 holds that the parties to a case are bound
by the findings in a previous judgment with respect to matters actually raised
and adjudged therein.25

Article 264(e) of the Labor Code prohibits any person engaged in picketing from
obstructing the free ingress to and egress from the employers premises. Since
respondent was found in the July 17, 1998 decision of the NLRC to have
prevented the free entry into and exit of vehicles from petitioners compound,
respondents officers and employees clearly committed illegal acts in the course
of the March 9, 1998 strike.1awphi1

The use of unlawful means in the course of a strike renders such strike illegal.26
Therefore, pursuant to the principle of conclusiveness of judgment, the March 9,
1998 strike was ipso facto illegal. The filing of a petition to declare the strike
illegal was thus unnecessary.

Consequently, we uphold the legality of the dismissal of respondents officers and


employees. Article 264 of the Labor Code27 further provides that an employer
may terminate employees found to have committed illegal acts in the course of a
strike.28 Petitioner clearly had the legal right to terminate respondents officers
and employees
109. AIRLINE PILOTS ASSOCIATION OF THE PHILIPPINES (ALPAP) vs. PAL

FACTS: petitioner Airline Pilots Association of the Philippines (ALPAP) is the


legitimate labor organization and exclusive bargaining agent of all commercial

pilots of PAL. Claiming that PAL committed ULP, ALPAP filed on December 9,
1997, a notice of strike against respondent PAL with the DOLE. Upon PALs
petition and considering that its continued operation is impressed with public
interest, the DOLE Secretary assumed jurisdiction over the labor dispute. The
DOLE ordered that all strikes and lockouts at the PAL, whether actual or
impending, are hereby strictly prohibited. However, ALPAP went on strike.
Hence, DOLE issued a return-to-work order on June 7, 1998. But it was only on
June 26, 1998 when ALPAP officers and members reported back to work as
shown in a logbook signed by each of them. As a consequence, PAL refused to
accept the returning pilots for their failure to comply immediately with the
return-to-work order.
ALPAP filed with the LA a complaint for illegal lockout against PAL. ALPAP
contended that its counsel received a copy of the return-to-work order only on
June 25, 1998, which justified their non-compliance therewith until June 26,
1998.

The DOLE Sec ruled that there was illegal strike but no illegal lockout and that
the dismissal of those who failed to obey the return-to-work order be upheld. CA
affirmed. SC also affirmed.

ALPAP contends that it was erroneous for Sto. Tomas and Imson (DOLE Sec) to
merely take note of the motions when the issues raised therein sprang from the
DOLE Secretarys exercise of authority to assume jurisdiction over a labor
dispute which have nevertheless remained unresolved. ALPAP prays that the
assailed letters dated July 4, 2003 and July 30, 2003 be declared null and void. It
likewise seeks for a conduct of a proceeding to determine who actually
participated in the illegal strike of June 1998 and consequently who, from its vast
membership, should be deemed to have lost employment status.
ISSUES: WON there is a need for a proceeding to determine who among the
ALPAP members and officers actually participated in the illegal strike

HELD: No. Such proceeding would entail a reopening of a final judgment which
could not be permitted by this Court. Settled in law is that once a decision has
acquired finality, it becomes immutable and unalterable, thus can no longer be
modified in any respect. Subject to certain recognized exceptions, the principle of

3E Andaya Ching Espiritu Hefti Galvez Gammad Lainez Lui Madamba Nagera Narvasa
Ong Palangdao Rosales Sanchez Santos Satrain Tabo (2014-2015)

Labor Relations Case Digest - Atty. Joyrich Golangco

immutability leaves the judgment undisturbed as "nothing further can be done


except to execute it."
Although the dispositive portion of the DOLE Resolution does not specifically
enumerate the names of those who actually participated in the strike but only
mentions that those strikers who failed to heed the return-to-work order are
deemed to have lost their employment, this omission cant prevent an effective
execution of the decision.

There is no necessity to conduct a proceeding to determine the participants in


the illegal strike or those who refused to heed the return to work order because
the ambiguity can be cured by reference to the body of the decision and the
pleadings filed.

A review of the records reveals that in NCMB and the DOLE Secretary declared
the ALPAP officers and members to have lost their employment status based on
either of two grounds, viz: their participation in the illegal strike or their defiance
of the return-to-work order of the DOLE Secretary. The records of the case unveil
the names of each of these returning pilots. The logbook with the heading
"Return To Work Compliance/ Returnees" bears their individual signature
signifying their conformity that they were among those workers who returned to
work only on June 26, 1998 or after the deadline imposed by DOLE. From this
crucial and vital piece of evidence, it is apparent that each of these pilots is bound
by the judgment. Besides, the complaint for illegal lockout was filed on behalf of
all these returnees. Thus, a finding that there was no illegal lockout would be
enforceable against them. In fine, only those returning pilots, irrespective of
whether they comprise the entire membership of ALPAP, are bound by the June
1, 1999 DOLE Resolution.
ALPAP harps on the inequity of PALs termination of its officers and members
considering that some of them were on leave or were abroad at the time of the
strike. Some were even merely barred from returning to their work which
excused them for not complying immediately with the return-to-work order.
Again, a scrutiny of the records of the case discloses that these allegations
were raised at a very late stage, that is, after the judgment has finally
decreed that the returning pilots termination was legal. These defenses
were raised in ALPAPs twin motions only after the Resolution in G.R. No. 152306
reached finality in its last ditch effort to obtain a favorable ruling. It has been

held that a proceeding may not be reopened upon grounds already available to
the parties during the pendency of such proceedings; otherwise, it may give way
to vicious and vexatious proceedings. ALPAP was given all the opportunities to
present its evidence and arguments. It cannot now complain that it was denied
due process.
110. OLISA ET. AL. VS ESCARIO ET. AL.

FACTS: The petitioners were among the regular employees of respondent


Pinakamasarap Corporation (PINA) and were members of Malayang Samahan ng
mga Manggagawa sa Balanced Foods (Union).

At 8:30 in the morning of March 13, 1993, all the officers and some 200 members
of the Union walked out of PINAs premises and proceeded to the barangay office
to show support for Juanito Caete, an officer of the Union charged with oral
defamation by Aurora Manor, PINAs personnel manager. Subsequently all
officers of the Union were suspended and were terminated after a month. PINA
filed a complaint for unfair labor practice (ULP) and damages and was ruled that
the incident was an illegal walkout constituting ULP and that all the Unions
officers, except Caete, had thereby lost their employment.

On April 28, 1993, the Union filed a notice of strike, claiming that PINA was guilty
of union busting through the constructive dismissal of its officers. A strike vote
was at which a majority of 190 members of the Union voted to strike.

PINA retaliated by charging the petitioners with ULP and abandonment of work,
stating that they had violated provisions on strike of the collective bargaining
agreement (CBA), such as:
(a) sabotage by the insertion of foreign matter in the bottling of company
products;
(b) decreased production output by slowdown;
(c) serious misconduct, and willful disobedience and insubordination to the
orders of the Management and its representatives;
(d) disruption of the work place by invading the premises and perpetrating
commotion and disorder, and by causing fear and apprehension;
(e) abandonment of work since June 28, 1993 despite notices to return to work
individually sent to them; and

3E Andaya Ching Espiritu Hefti Galvez Gammad Lainez Lui Madamba Nagera Narvasa
Ong Palangdao Rosales Sanchez Santos Satrain Tabo (2014-2015)

Labor Relations Case Digest - Atty. Joyrich Golangco

(f) picketing within the company premises on June 15, 1993 that effectively
barred with the use of threat and intimidation the ingress and egress of PINAs
officials, employees, suppliers, and customers.

On September 30, 1994, the Third Division of the National Labor Relations
Commission (NLRC) issued a temporary restraining order (TRO), enjoining the
Unions officers and members to cease and desist from barricading and
obstructing the entrance to and exit from PINAs premises, to refrain from
committing any and all forms of violence, and to remove all forms of obstructions
such as streamers, placards, or human barricade. On November 29, 1994, the
NLRC granted the writ of preliminary injunction.
LA: The subject strike to is illegal.

NLRC: On appeal, sustained the finding that the strike was illegal, but reversed
the LAs ruling that there was abandonment. Union members should not be
considered to have abandoned their employment, as stated Under Article 264 of
the Labor Code, as amended, the union officers who knowingly participate in the
illegal strike may be declared to have lost their employment status. However,
mere participation of a union member in the illegal strike does not mean loss of
employment status unless he participates in the commission of illegal acts during
the strike. While it is true that complainant thru individual memorandum
directed the respondents to return to work there is no showing that respondents
deliberately refused to return to work. A worker who joins a strike does so
precisely to assert or improve the terms and conditions of his work. If his
purpose is to abandon his work, he would not go to the trouble of joining a strike.
PINA was directed to reinstate respondents named in the complaint to their
former positions but without backwages. In the event that reinstatement is not
feasible, to pay respondents separation pay at one (1/2) half month per year of
service.
Following the denial of their motion for reconsideration, the petitioners assailed
the NLRCs decision through a petition for certiorari in the Court of Appeals,
claiming that the NLRC gravely abused its discretion in not awarding backwages
pursuant to Article 279 of the Labor Code, and in not declaring their strike as a
good faith strike.

CA: Affirmed and denied the petitioners claim for full backwages, the CA applied
the third paragraph of Article 264(a) instead of Article 279 of the Labor Code,
explaining that the only instance under Article 264 when a dismissed employee
would be reinstated with full backwages was when he was dismissed by reason
of an illegal lockout; that Article 264 was silent on the award of backwages to
employees participating in a lawful strike; and that a reinstatement with full
backwages would be granted only when the dismissal of the petitioners was not
done in accordance with Article 282 (dismissals with just causes) and Article 283
(dismissals with authorized causes) of the Labor Code.
ISSUES: Whether or not the petitioners are entitled to full backwages from the
date of dismissal until the date of actual reinstatement due to their not being
found to have abandoned their jobs.
HELD: The Supreme Court sustained the ruling of the CA, but modified the
decision on the amount of the backwages in order to accord with equity and
jurisprudence.

I. The third Paragraph of Article 264 (a),Labor Code, is the applicable law
rather than Article 279 argued by the petitioners.
By its use of the phrase unjustly dismissed, Article 279 refers to a dismissal that
is unjustly done, that is, the employer dismisses the employee without observing
due process, either substantive or procedural. Contemplating two causes for the
dismissal of an employee, that is: (a) unlawful lockout; and (b) participation in an
illegal strike, the third paragraph of Article 264(a) authorizes the award of full
backwages only when the termination of employment is a consequence of an
unlawful lockout. On the consequences of an illegal strike, the provision
distinguishes between a union officer and a union member participating in an
illegal strike. A union officer who knowingly participates in an illegal strike is
deemed to have lost his employment status, but a union member who is merely
instigated or induced to participate in the illegal strike is more benignly treated.
Part of the explanation for the benign consideration for the union member is the
policy of reinstating rank-and-file workers who are misled into supporting illegal
strikes, absent any finding that such workers committed illegal acts during the

3E Andaya Ching Espiritu Hefti Galvez Gammad Lainez Lui Madamba Nagera Narvasa
Ong Palangdao Rosales Sanchez Santos Satrain Tabo (2014-2015)

period of the illegal strikes.

Labor Relations Case Digest - Atty. Joyrich Golangco

The petitioners were terminated for joining a strike that was later declared to be
illegal. The NLRC ordered their reinstatement or, in lieu of reinstatement, the
payment of their separation pay, because they were mere rank-and-file workers
whom the Unions officers had misled into joining the illegal strike. They were
not unjustly dismissed from work. Based on the text and intent of the two afore
quoted provisions of the Labor Code, therefore, it is plain that Article 264(a) is
the applicable one
II. Petitioners not entitled to backwages despite their reinstatement
because of the Doctrine of "A fair days wage for a fair days labor"

As a general rule, backwages are granted to indemnify a dismissed employee for


his loss of earnings during the whole period that he is out of his job. Considering
that an illegally dismissed employee is not deemed to have left his employment,
he is entitled to all the rights and privileges that accrue to him from the
employment. That backwages are not granted to employees participating in an
illegal strike simply accords with the reality that they do not render work for the
employer during the period of the illegal strike.

With respect to backwages, the principle of a fair days wage for a fair days
labor remains as the basic factor in determining the award thereof. If there is no
work performed by the employee there can be no wage or pay unless, of course,
the laborer was able, willing and ready to work but was illegally locked out,
suspended or dismissed or otherwise illegally prevented from working.

The petitioners herein do not deny their participation in the June 15, 1993 strike.
As such, they did not suffer any loss of earnings during their absence from work.
Their reinstatement sans backwages is in order, to conform to the policy of a fair
days wage for a fair days labor.
III. The appropriate amount for separation pay is one month per year of
service.

Separation pay is made an alternative relief in lieu of reinstatement in certain

circumstances, like:
(a) when reinstatement can no longer be effected in view of the passage of a long
period of time or because of the realities of the situation;
(b) reinstatement is inimical to the employers interest;
(c) reinstatement is no longer feasible;
(d) reinstatement does not serve the best interests of the parties involved;
(e) the employer is prejudiced by the workers continued employment;
(f) facts that make execution unjust or inequitable have supervened; or
(g) strained relations between the employer and employee.
Here, PINA manifested that the reinstatement of the petitioners would not be
feasible because: (a) it would inflict disruption and oppression upon the
employer; (b) petitioners [had] stayed away for more than 15 years; (c) its
machines had depreciated and had been replaced with newer, better ones; and
(d) it now sold goods through independent distributors, thereby abolishing the
positions related to sales and distribution.[29]

Under the circumstances, the grant of separation pay in lieu of reinstatement of


the petitioners was proper. It is not disputable that the grant of separation pay or
some other financial assistance to an employee is based on equity, which has
been defined as justice outside law, or as being ethical rather than jural and as
belonging to the sphere of morals than of law. This Court has granted separation
pay as a measure of social justice even when an employee has been validly
dismissed, as long as the dismissal has not been due to serious misconduct or
reflective of personal integrity or morality.
111. TABANGAO SHELL REFINERY EMPLOYEES ASSN. vs. PILIPINAS SHELL
PETROLEUM CORP.

FACTS: As the current collective bargaining agreement (CBA) between


petitioner and respondent is coming to an end, the parties started negotiations
for a new CBA. The union proposed a 20% annual across-the-board salary
increase for the next 3 years however, the company proposed a lump sum of
P80,000 yearly for the 3-year period to all covered employees. In reply to the
unions request to provide in full details the basis for its counter-proposal, the
company explained that it is based on the affordability of the corp. and the
current salary levels in the industry but the union rejected it. After another

3E Andaya Ching Espiritu Hefti Galvez Gammad Lainez Lui Madamba Nagera Narvasa
Ong Palangdao Rosales Sanchez Santos Satrain Tabo (2014-2015)

Labor Relations Case Digest - Atty. Joyrich Golangco

negotiation, the company increased its offer to P88,000 but again the union
requested for justification but the company refused to give in insisting that the
financial measures were available in the refinery scorecard in the website and
shared network drives. The union charged that the company was bargaining in
bad faith while the company expressed its disagreement with the unions
manifestation and proposed the declaration of a deadlock and recommended a
third partys assistance. The union filed a Notice of Strike in the Natl.
Conciliation & Mediation Board (NCMB) alleging bad faith bargaining on the part
of the company. In the mandatory conciliation-mediation proceedings, the
parties failed to reach an amicable settlement. When the company learned of the
unions unanimous vote to hold a strike, it filed a Petition for Assumption of
Jurisdiction with the Secretary of Labor & Employment under Art. 263(g) of the
Labor Code. The DOLE, finding that the strike would have a negative impact on
national interest assumed jurisdiction over the dispute and directed the parties
to submit their respective position paper and maintain the status quo existing at
the time of the Order or if the strike had commenced, the workers were directed
to return to work and the employer to readmit all workers under the same terms
and conditions prevailing before the strike. The union filed a petition before the
CA questioning the jurisdiction of the DOLE as the issue raised was unfair labor
practice of the company in the form of bad faith bargaining and not the CBA
deadlock citing item 8 of the CBA rules that deadlock can only be declared upon
mutual consent of both parties but where the union did not consent. The CA
dismissed the petition for lack of merit declaring that the DOLE acquired
jurisdiction over the dispute vested to it by Art. 263(g) of the Labor Code.
Meanwhile, the union filed a complaint against the company in the NLRC on
allegations of the companys refusal to bargain. The NLRC finding that the case
arose from the same CBA-related labor dispute, transmitted the case to the
DOLE. The DOLE, holding that there was already deadlock and there was no
showing that the company engaged in unfair labor practice by bargaining in bad
faith, decided the matter of the wage increase and other economic issues of the
new CBA, i.e. to give a lump sum package of P95,000 per year for three years per
covered employee & retention of benefits covered by the preceding CBA. Both
parties did not appeal the DOLE decision thus, it attained finality. The union then
went to the Court via a petition for review under Rule 45 of the Rules of Court.

ISSUES: Whether or not respondent company is guilty of bargaining in bad faith?

HELD: NO. The duty to bargain does not compel any party to accept a proposal
or to make any concession (Art. 252, Labor Code). While the purpose of collective
bargaining is the reaching of an agreement between the employer and the
employees union resulting in a binding contract between the parties, the failure
to reach an agreement after negotiations continued for a reasonable period does
not mean lack of good faith. A CBA, like any contract is a product of mutual
consent and not of compulsion and the duty to bargain does not include the
obligation to reach an agreement. Respondents unswerving position on the
matter of annual lump sum payment in lieu of wage increase did not, by itself,
constitute bad faith even if such position caused a stalemate in the negotiations.
As there was no bad faith on the part of the company in its bargaining with the
union, deadlock was possible and did occur. Fact is, that the negotiations
between the union and the company were stalled by the opposing offers of yearly
wage increase by the union, on the one hand, and annual lump sum payment by
the company, on the other hand. Each party found the others offer unacceptable
and neither party was willing to yield. The company suggested seeking the
assistance of a third party to settle the issue but the union preferred the remedy
of filing a notice of strike. The absence of the parties mutual declaration of
deadlock does not mean that there was no deadlock. This is the essence of Article
263(g) of the Labor Code which gives the Secretary of Labor & Employment
jurisdiction over a labor dispute causing or likely to cause a strike or lockout in
an industry indispensable to the national interest.
The Court denied the petition declaring that the Secretary of Labor and
Employment committed no abuse of discretion when she assumed jurisdiction
over the labor dispute of the union and the company.
112. ASIA BREWERY, INC (ABI) vs TUNAY NA PAGKAKAISA NG MGA
MANGGAGAWA SA ASIA (TPMA)

FACTS: TPMA is a legitimate labor organization, certified as the sole and


exclusive bargaining agent of all regular rank and file employees of petitioner
corporation Asia Brewery, Incorporated (ABI).
ABI and TPMA had been negotiating for a new CBA for the years 2003-2006
since the old CBA expired last July 2003. After about 18 sessions of failed

3E Andaya Ching Espiritu Hefti Galvez Gammad Lainez Lui Madamba Nagera Narvasa
Ong Palangdao Rosales Sanchez Santos Satrain Tabo (2014-2015)

Labor Relations Case Digest - Atty. Joyrich Golangco

negotiations TPMA declared a deadlock and filed a notice of strike before NCMB.
However, the parties did not come to terms even before the NCMB.

On November 18, 2003, TPMA conducted a strike vote. Out of the 840 union
members, 768 voted in favor of holding a strike. ABI now then petitioned the
SOLE to assume jurisdiction pursuant to Art 263(g) of LC. In the answer of TPMA
it opposed the the assumption of jurisdiction, reasoning therein that the business
of ABI is not in dispensable to the national interest.
On December 2, 2003, TPMA filed before the CA petition for injunction, which
sought to enjoin the respondent Secretary of Labor from assuming jurisdiction
over the labor dispute, or in the alternative, to issue a temporary restraining
order, likewise to enjoin the former from assuming jurisdiction.

On December 19, 2003, the SOLE issued an order assuming jurisdiction over the
dispute and ordered the submission of position papers and other documents
both from ABI and TPMA to wit:
"1. The Company shall be required to provide:
"a. Complete Audited Financial Statements for the past five
(5) years certified as to its completeness by the Chief Financial
Comptroller or Accountant;
"b. Projected Financial Statements of the Company for the next
three (3) years;
"xxx
"2. The Union is required to provide an itemized summary of their CBA
demands with financial costing and sample CBAs (if any) in similarly
situated or comparable bargaining units.

On January 19, 2004, TPMA filed another petition for certiorari with the CA,
imputing bad faith and grave abuse of discretion to the SOLE. In the meantime, in
a decision dated January 19, 2004, SOLE Patricia Sto. Tomas resolved the
deadlock between the parties and granted the following awards: wage increase
of the 1st year of 18.00, 2nd year 15.00, 3rd year 12.00; (2) HEALTH CARE
(HMO) P1,300 premium to be shouldered by ABI., for each covered employee
and P1,800 contribution for each Union member-dependent.

TPMA moved for a reconsideration of the decision on the ground that the ruling
lacks evidentiary proof to sufficiently justify the same. Similarly, ABI also filed a
motion for clarification/reconsideration. Thereafter, on February 9, 2004, the
parties executed and signed the Collective Bargaining Agreement with a term
from August 1, 2003 to July 31,2006.
Subsequently, on April 1, 2004, respondent union filed another petition for
certiorari before the Court of Appeals, which was docketed as SP-83168,
assailing the arbitral award and imputing grave abuse of discretion upon the
public respondent.

CA ruled affirming the order of SOLE, dismissing the injunction and TRO,
remanding the computation of wage increase to the SOLE because it based its
computation on the unaudited FS of ABI which have no probative value and
increasing the health benefit to 1,390.00.

ABI contended that the FS were duly signed and certified by its chief financial
officer, that it has also been allegedly submitted to various government agencies
and thus should be considered official and public documents. Moreover TPMA
did not objected to it during the proceedings before the SOLE and even used the
same in formulation its arguments in the said proceedings. Hence such FS
although not audited by an external and independent auditor should be
considered as substantial compliance with the order of SOLE and furthermore,
the decision of the SOLE was not solely based on these FS, as the CBA history,
costing and proposals and wages in other similarly situated bargaining units
were considered. And finally it claims that the demand of TPMA is unrealistic and
will cause the former to close shop.
ISSUES: Whether the CA erred when it remanded to the Secretary of Labor the
issue on wage increase.

HELD: Yes, In Restaurante Las Conchas v. Llego, it was held that such financial
statements are mere self-serving declarations and inadmissible in evidence even
if the employees did not object to their presentation before the Labor Arbiter.

3E Andaya Ching Espiritu Hefti Galvez Gammad Lainez Lui Madamba Nagera Narvasa
Ong Palangdao Rosales Sanchez Santos Satrain Tabo (2014-2015)

Labor Relations Case Digest - Atty. Joyrich Golangco

x x x It is true that administrative and quasi-judicial bodies like the NLRC are not
bound by the technical rules of procedure in the adjudication of cases. However,
this procedural rule should not be construed as a license to disregard certain
fundamental evidentiary rules. While the rules of evidence prevailing in the
courts of law or equity are not controlling in proceedings before the NLRC, the
evidence presented before it must at least have a modicum of admissibility for it
to be given some probative value.xxx FS without the accompanying signature of a
CPA or audited by an independent auditor are nothing but self-serving
documents which ought to be treated as a mere scrap of paper devoid of any
probative value.

Union Demands: 36.00 wage increase of every year for the next 3 years, while
the company offers an increase of 18.00 for the first 18 months and another
18.00 increase for the second 18 months while the SOLE granted and increase of
1st year of 18.00, 2nd year 15.00, 3rd year 12.00;

Thus, we rule that the Secretary of Labor gravely abused her discretion when she
relied on the unaudited financial statements of ABI in determining the wage
award because such evidence is self-serving and inadmissible. Not only did this
violate the December 19, 2003 order of SOLE herself to ABI to submit its
complete audited financial statements, but this may have resulted to a wage
award that is based on an inaccurate and biased picture of petitioner
corporation's capacity to pay one of the more significant factors in making a
wage award. Verily, we cannot countenance this procedure because this could
unduly deprive labor of its right to a just share in the fruits of production and
provide employers with a means to understate their profitability in order to
defeat the right of labor to a just wage.

NOTE: Middle Ground approach is discouraged because: it is a simplistic solution


that fails to recognize that the parties may already be at the limits of the wage
levels.

The extent of judicial review over the Secretary of Labor's arbitral award is not
limited to a determination of grave abuse in the manner of the secretary's
exercise of his statutory powers. This Court is entitled to, and must in the
exercise of its judicial power review the substance of the Secretary's award
when grave abuse of discretion is alleged to exist in the award, i.e., in the
appreciation of and the conclusions the Secretary drew from the evidence
presented.

We also note with disapproval the manner by which the Secretary of Labor
issued the wage award in this case, Based on such factors as BARGAINING
HISTORY, TRENDS OFARBITRATED AND AGREED AWARDS AND INDUSTRY
TRENDS, in general, we hold that vis--vis the Unions demands and the
Companys offers, as follows:

The SOLE failed to indicate the actual data upon which the award was based.
Even if it appears to have utilized the middle ground approach which is merely
finding the midway point between the demands of the company and the union,
and "splitting the difference". Factors such as the actual and projected net
operating income, impact of the wage increase on net operating income, the
company's previous CBAs, and industry trends were not discussed in detail so
that the precise bases of the wage award are not discernible on the face of the
Decision. In fine, there is no way of determining if the SOLE utilized the proper
evidence, figures or data in arriving at the subject wage award as well as the
reasonableness thereof. This falls short of the requirement of administrative due
process obligating the decision-maker to adjudicate the rights of the parties in
such a manner that they can know the various issues involved and the reasons
for the decision rendered.

113. DANILO ESCARIO, et. al. vs. NATIONAL LABOR RELATIONS


COMMISSION (THIRD DIVISION), PINAKAMASARAP CORPORATION, DR. SY
LIAN TIN, AND DOMINGO TAN

FACTS: The petitioners were among the regular employees of respondent


Pinakamasarap Corporation (PINA), a corporation engaged in manufacturing and
selling food seasoning. They were members of petitioner Malayang Samahan ng
mga Manggagawa sa Balanced Foods (Union).
ULP was filed by the Union because of the following incident: Sometime in March
1993, Union officers and 200 members walked out of PINA's premises and
proceeded to the barangay office to show support for Juanito Caete, an officer of

3E Andaya Ching Espiritu Hefti Galvez Gammad Lainez Lui Madamba Nagera Narvasa
Ong Palangdao Rosales Sanchez Santos Satrain Tabo (2014-2015)

Labor Relations Case Digest - Atty. Joyrich Golangco

the Union charged with oral defamation. As a result of the walkout, PINA
preventively suspended all officers and terminated them after a month.

In May 1993, the Union held a strike vote because of the constructive dismissal,
at which a majority of 190 members of the Union voted to strike. The strike
pushed through in June.

PINA retaliated by charging the petitioners with ULP and abandonment of work,
stating that they had violated provisions on strike of the collective bargaining
agreement.
In September 1994, the a TRO was issued by the NLRC, enjoining the Union's
officers and members to cease and desist from barricading and obstructing the
entrance to and exit from PINA's premises, to refrain from committing any and
all forms of violence, and to remove all forms of obstructions such as streamers,
placards, or human barricade.

The LA held that the strike held was illegal. The NLRC sustained, but reversed the
LA's ruling that there was abandonment. Under Art. 264 of the Labor Code, it was
stated that mere participation of a union member in the illegal strike does not
mean loss of employment status unless he participates in the commission of
illegal acts during the strike. CA affirmed NLRC. CA applied the third paragraph
of Article 264 (a) instead of Article 279 of the Labor Code, explaining that the
only instance under Article 264 when a dismissed employee would be reinstated
with full backwages was when he was dismissed by reason of an illegal lockout;
that Article 264 was silent on the award of backwages to employees participating
in a lawful strike; and that a reinstatement with full backwages would be granted
only when the dismissal of the petitioners was not done in accordance with
Article 282 (dismissals with just causes) and Article 283 (dismissals with
authorized causes) of the Labor Code.
ISSUES: Whether Art. 264 par. 3 is applicable or Art. 279 of the Labor Code.

WoN petitioners are entitled to full backwages from the date of dismissal until
the date of actual reinstatement due to their not being found to have abandoned
their jobs.

HELD: We sustain the CA, but modify the decision on the amount of the
backwages in order to accord with equity and jurisprudence.

By its use of the phrase unjustly dismissed, Article 279 refers to a dismissal that
is unjustly done, that is, the employer dismisses the employee without observing
due process, either substantive or procedural. In contrast, the third paragraph of
Article 264 (a) contemplates two causes for the dismissal of an employee, that is:
(a) unlawful lockout; and (b) participation in an illegal strike. The third
paragraph of Article 264 (a) authorizes the award of full backwages only when
the termination of employment is a consequence of an unlawful lockout.
On the consequences of an illegal strike, the provision distinguishes between a
union officer and a union member participating in an illegal strike. A union
officer who knowingly participates in an illegal strike is deemed to have lost his
employment status, but a union member who is merely instigated or induced to
participate in the illegal strike is more benignly treated. Part of the explanation
for the benign consideration for the union member is the policy of reinstating
rank-and-file workers who are misled into supporting illegal strikes, absent any
finding that such workers committed illegal acts during the period of the illegal
strikes.

Thus, the NLRC ordered their reinstatement or, in lieu of reinstatement, the
payment of their separation pay, because they were mere rank-and-file workers
whom the Union's officers had misled into joining the illegal strike. They were
not unjustly dismissed from work. Based on the text and intent of the two
aforequoted provisions of the Labor Code, therefore, it is plain that Article 264
(a) is the applicable one.

The petitioners argue that the finding of no abandonment equated to a finding of


illegal dismissal in their favor. Hence, they were entitled to full backwages.
The petitioners' argument cannot be sustained.

The petitioners' participation in the illegal strike was precisely what prompted
PINA to file a complaint to declare them, as striking employees, to have lost their
employment status. However, the NLRC ultimately ordered their reinstatement

3E Andaya Ching Espiritu Hefti Galvez Gammad Lainez Lui Madamba Nagera Narvasa
Ong Palangdao Rosales Sanchez Santos Satrain Tabo (2014-2015)

Labor Relations Case Digest - Atty. Joyrich Golangco

after finding that they had not abandoned their work by joining the illegal strike.
They were thus entitled only to reinstatement, regardless of whether or not the
strike was the consequence of the employer's ULP, considering that a strike was
not a renunciation of the employment relation. That backwages are not granted
to employees participating in an illegal strike simply accords with the reality that
they do not render work for the employer during the period of the illegal strike.

The petitioners herein do not deny their participation in the June 15, 1993 strike.
As such, they did not suffer any loss of earnings during their absence from work.
Their reinstatement sans backwages is in order, to conform to the policy of a fair
day's wage for a fair day's labor.

Under the principle of a fair day's wage for a fair day's labor, the petitioners were
not entitled to the wages during the period of the strike (even if the strike might
be legal), because they performed no work during the strike. Verily, it was
neither fair nor just that the dismissed employees should litigate against their
employer on the latter's time. Thus, the Court deleted the award of backwages
and held that the striking workers were entitled only to reinstatement in
Philippine Diamond Hotel and Resort, Inc. (Manila Diamond Hotel) v. Manila
Diamond Hotel Employees Union, considering that the striking employees did
not render work for the employer during the strike.
114. University of San Agustin Employees Union FFW (USAE-FFW) vs. CA
and University of San Agustin

FACTS: Respondent University of San Agustin (University) is a non-stock, nonprofit educational institution, which offers both basic and higher education
courses. Petitioner Union is the duly recognized collective bargaining unit for
teaching and non-teaching rank-and-file personnel of the University while the
other individual petitioners are its officers.
On July 27, 2000, the parties entered into a 5-year CBA which, among other
things, provided that the economic provisions thereof shall have a period of
three (3) years or up to 2003. The CBA contained a "no strike, no lockout" clause.
Pursuant to the CBA, the parties commenced negotiations for the economic
provisions for the remaining two years, i.e., SY2003-2004 and SY2004-2005.
During the negotiations, the parties could not agree on the manner of computing

the tuition incremental proceeds (TIP), thus the need to undergo preventive
mediation proceedings before the National Conciliation and Mediation Board
(NCMB), Iloilo City.

The impasse respecting the computation of TIP was not resolved. This
development prompted the Union to declare a bargaining deadlock grounded on
the parties failure to arrive at a mutually acceptable position on the manner of
computing the seventy percent (70%) of the net TIP to be allotted for salary and
other benefits.Thereafter, the Union filed a Notice of Strike before the NCMB
which was expectedly opposed by the University in a Motion to Strike Out Notice
of Strike and to Refer the Dispute to Voluntary Arbitration,3 invoking the "No
strike, no lockout" clause4 of the parties CBA. The NCMB, however, failed to
resolve the Universitys motion.
The parties then made a joint request for the SOLE to assume jurisdiction over
the dispute. The labor dispute was docketed as OS-AJ-0032-2003. On September
18, 2003, an Assumption of Jurisdiction Order5 (AJO) was issued by the SOLE,
thus:

WHEREFORE, this Office hereby ASSUMES JURISDICTION over the labor dispute
at the UNIVERSITY OF SAN AGUSTIN, pursuant to Article 263(g) of the Labor
Code, as amended. ACCORDINGLY, any strike or lockout whether actual or
intended, is hereby strictly enjoined and the parties are directed to cease and
desist from committing any act that might exacerbate the situation.
On September 19, 2003, the Union staged a strike. At 6:45 a.m. of the same day,
Sheriffs Francisco L. Reyes and Rocky M. Francisco had arrived at San Agustin
University to serve the AJO on the Union. At the main entrance of the University,
the sheriffs saw some elements of the Union at the early stages of the strike.
There they met Merlyn Jara, the Unions vice president, upon whom the sheriffs
tried to serve the AJO, but who, after reading it, refused to receive the same,
citing Union Board Resolution No. 3 naming the union president as the only
person authorized to do so. The sheriffs explained to Ms. Jara that even if she
refused to acknowledge receipt of the AJO, the same would be considered served.
Sheriff Reyes further informed the Union that once the sheriffs post the AJO, it
would be considered received by the Union.6

3E Andaya Ching Espiritu Hefti Galvez Gammad Lainez Lui Madamba Nagera Narvasa
Ong Palangdao Rosales Sanchez Santos Satrain Tabo (2014-2015)

Labor Relations Case Digest - Atty. Joyrich Golangco

At approximately 8:45 a.m., the sheriffs posted copies of the AJO at the main gate
of San Agustin University, at the main entrance of its buildings and at the Unions
office inside the campus. At 9:20 a.m., the sheriffs served the AJO on the
University. Notwithstanding the sheriffs advice as to the legal implication of the
Unions refusal to be served with the AJO, the Union went ahead with the strike.
At around 5:25 p.m., the Union president arrived at the respondent Universitys
premises and received the AJO from the sheriffs.
On September 24, 2003, the University filed a Petition to Declare Illegal Strike
and Loss of Employment Status7 at the National Labor Relations Commission
(NLRC) Sub-regional Arbitration Branch No. VI in Iloilo City. The case was
docketed as NLRC SRAB Case No. 06-09-50370-03, which the University later on
requested to be consolidated with OS-AJ-0032-2003 pending before the SOLE.
The motion for consolidation was granted by the Labor Arbiter in an Order dated
November 7, 2003.
SOLE: The petition to declare the strike illegal is hereby DISMISSED for want of
legal and factual basis. Consequently, there is no basis whatsoever to declare loss
of employment status on the part of any of the striking union members. Motion
for Reconsideration is denied.
CA: The assailed Decision of the public respondent SOLE is hereby MODIFIED to
the effect that the strike held by the petitioners on September 19, 2003 is illegal.
Hence, the union officers are deemed to have lost their employment status.
ISSUES: WON the strike is illegal

HELD: Art. 263. Strikes, picketing, and lockouts. - (g) When, in his opinion, there
exists a labor dispute causing or likely to cause a strike or lockout in an industry
indispensable to the national interest, the Secretary of Labor and Employment may
assume jurisdiction over the dispute and decide it or certify the same to the
Commission for compulsory arbitration. Such assumption or certification shall have
the effect of automatically enjoining the intended or impending strike or lockout as
specified in the assumption or certification order. If one has already taken place at
the time of assumption or certification, all striking or locked out employees shall
immediately return to work and the employer shall immediately resume operations
and readmit all workers under the same terms and conditions prevailing before the

strike or lockout.

Conclusively, when the SOLE assumes jurisdiction over a labor dispute in an


industry indispensable to national interest or certifies the same to the NLRC for
compulsory arbitration, such assumption or certification shall have the effect of
automatically enjoining the intended or impending strike or lockout. Moreover, if
one had already taken place, all striking workers shall immediately return to
work and the employer shall immediately resume operations and readmit all
workers under the same terms and conditions prevailing before the strike or
lockout. In this case, the AJO was served at 8:45 a.m. of September 19, 2003. The
strikers then should have returned to work immediately. However, they
persisted with their refusal to receive the AJO and waited for their union
president to receive the same at 5:25 p.m.
Thus, we see no reversible error in the CAs finding that the strike of September
19, 2003 was illegal. Consequently, the Union officers were deemed to have lost
their employment status for having knowingly participated in said illegal act.
115. Philippine Diamond Hotel and Resort Inc. vs Manila Diamond Hotel
Employees Union

FACTS: The petition for certification election filed by the Union was dismissed by
the DOLE for failure to comply with legal requirements. Through its president
Kimpo, it notified petitioner of its intention to negotiate a collective bargaining
agreement. Petitioner refused due to the fact that the union was not certified as
the exclusive bargaining agent by the DOLE. The union clarified that it sought to
bargain for its members only, and declared that [the Hotels] refusal to bargain
[would prompt] the union to engage in concerted activities to protect and assert
its rights under the Labor Code. The union went on to file a Notice of
Strike[16] on September 29, 1997 with the National Conciliation and Mediation
Board (NCMB) due to unfair labor practice (ULP) in that the Hotel refused to
bargain with it and the rank-and-file employees were being harassed and
prevented from joining it. Several conferences took place between the parties. In
the conference held on November 20, 1997, the union demanded the holding of a
consent election to which the Hotel interposed no objection, provided the union
followed the procedure under the law. Petitioner then requested that the election
be held in January 1998. The parties agreed to meet again on December 1, 1997.

3E Andaya Ching Espiritu Hefti Galvez Gammad Lainez Lui Madamba Nagera Narvasa
Ong Palangdao Rosales Sanchez Santos Satrain Tabo (2014-2015)

Labor Relations Case Digest - Atty. Joyrich Golangco

In the early morning of November 29, 1997, however, the union suddenly went
on strike. The following day, the National Union of Workers in the Hotel,
Restaurant and Allied Industries (NUWHRAIN) joined the strike and openly
extended its support to the union.[22] At about this time, Hotel supervisors
Vicente T. Agustin (Agustin) and Rowena Junio (Rowena) failed to report for
work and were, along with another supervisor, Mary Grace U. de Leon (Mary
Grace), seen participating in and supporting the strike. Mary Grace, who was
directed to explain her participation in the strike, alleged that she was merely
trying to pacify the group. Petitioner, finding her explanation arrogant and
unsatisfactory as her active participation in the strike was confirmed by an eye
witness, terminated her services, by communication sent on December 9, 1997,
drawing her to file a complaint for illegal dismissal against petitioner.[26]
Agustin, who was also terminated, filed a similar complaint against the HotelThe
strikers blocked the ingress and egress to and from the hotel. They also refused
to dismantle despite the issuance of a TRO commanding them to cease and desist
from obstructing the ingress and egress of the Hotel. Petitioner filed a petition
tom declare the strike illegal. By Resolution of November 19, 1999, the NLRC
declared that the strike was illegal and that the union officers and members who
were reinstated to the Hotels payroll were deemed to have lost their
employment status. And it dismissed the complaints filed by Mary Grace,
Agustin, and Rowena as well as the unions complaint for ULP. On appeal by the
union, the Court of Appeals affirmed the NLRC Resolution dismissing the
complaints of Mary Grace, Agustin and Rowena and of the union. It modified the
NLRC Resolution, however, by ordering the reinstatement with back wages of
union members.
ISSUES:
a. WON the strike was illegal
b. WON the employees were illegally dismissed
c. WON the dismissed employees are entitled to backwages
HELD:

a. YES it was illegal.

First, Union had no right to demand to collectively bargain with the Hotel.
Respondent insists, however, that it could validly bargain in behalf of its

members, relying on Article 242 of the Labor Code.[39] Respondents reliance


on said article, a general provision on the rights of legitimate labor organizations,
is misplaced, for not every legitimate labor organization possesses the rights
mentioned therein.[40] Article 242 (a) must be read in relation to above-quoted
Article 255. On respondents contention that it was bargaining in behalf only of
its members, the appellate court, affirming the NLRCs observation that the same
would only fragment the employees of petitioner,[41] held that what
[respondent] will be achieving is to divide the employees, more particularly, the
rank-and-file employees of [petitioner] . . . the other workers who are not
members are at a serious disadvantage, because if the same shall be allowed,
employees who are non-union members will be economically impaired and will
not be able to negotiate their terms and conditions of work, thus defeating the
very essence and reason of collective bargaining, which is an effective safeguard
against the evil schemes of employers in terms and conditions of work. This
Court finds the observation well-taken. Petitioners refusal to bargain then with
respondent cannot be considered a ULP to justify the staging of the strike.

Second, even if the purpose of a strike is valid, the strike may still be held illegal
where the means employed are illegal. Thus, the employment of violence,
intimidation, restraint or coercion in carrying out concerted activities which are
injurious to the rights to property renders a strike illegal. And so is picketing or
the obstruction to the free use of property or the comfortable enjoyment of life or
property, when accompanied by intimidation, threats, violence, and coercion as
to constitute nuisance.
b. Matter is remanded to LA. An ordinary striking worker cannot, thus be
dismissed for mere participation in an illegal strike. There must be proof that he
committed illegal acts during a strike, unlike a union officer who may be
dismissed by mere knowingly participating in an illegal strike and/or committing
an illegal act during a strike. The list failed to specifically identify the ones who
actually committed illegal acts, however. Such being the case, a remand of the
case to the Labor Arbiter, through the NLRC, is in order for the purpose only of
determining the respective liabilities of the strikers listed by petitioner. Those
proven to have committed illegal acts during the course of the strike are deemed
to have lost their employment, unless they have been readmitted by the Hotel,
whereas those not clearly shown to have committed illegal acts should be
reinstated.

3E Andaya Ching Espiritu Hefti Galvez Gammad Lainez Lui Madamba Nagera Narvasa
Ong Palangdao Rosales Sanchez Santos Satrain Tabo (2014-2015)

Labor Relations Case Digest - Atty. Joyrich Golangco

c. This Court must thus hearken to its policy that when employees voluntarily go
on strike, even if in protest against unfair labor practices, no backwages during
the strike is awarded. Reinstatement without backwages of striking members of
respondent who did not commit illegal acts would thus suffice under the
circumstances of the case. If reinstatement is no longer possible, given the lapse
of considerable time from the occurrence of the strike, the award of separation
pay of one (1) month salary for each year of service, in lieu of reinstatement, is in
order.
116. SUKHOTHAI CUISINE and RESTAURANT, petitioner,
vs.COURT OF APPEALS, NATIONAL LABOR PHILIPPINE LABOR ALLIANCE
COUNCIL (PLAC) Local 460 Sukhothai Restaurant Chapter, EMMANUEL
CAYNO, ALEX MARTINEZ, BILLY BACUS, HERMIE RAZ, JOSE LANORIAS, LITO
ARCE, LINO SALUBRE, CESAR SANGREO, ROLANDO FABREGAS, JIMMY
BALAN, JOVEN LUALHATI, ANTONIO ENEBRAD, JOSE NEIL ARCILLA, REY
ARSENAL, ROEL ESANCHA, EDGAR EUGENIO, ALBERT AGBUYA, ROLANDO
PUGONG, ARNEL SALVADOR, RICKY DEL PRADO, CLAUDIO PANALIGAN,
BERNIE DEL MUNDO, JOHN BATHAN, ROBERTO ECO, JOVEN TALIDONG,
LENY LUCENTE, ANALIZA CABLAY, RIGOBERTO TUBAON and MERLY
NAZ, respondents.

Majority of the employees of the petitioner organized themselves into a union


which affiliated with the Philippine Labor Alliance Council (PLAC), and was
designated as PLAC Local 460 Sukhothai Restaurant Chapter (Union).4 On
December 3, 1998 private respondent Union filed a Notice of Strike with the
NCMB on the ground of unfair labor practice, and particularly, acts of
harassment, fault-finding, and union busting through coercion and interference
with union affairs. In a conciliation conference, the representatives of the
petitioner agreed and guaranteed that there will be no termination of the
services of private respondents during the pendency of the case. Petitioner and
the Union entered into a Submission Agreement, thereby agreeing to submit the
issue of unfair labor practice for voluntary arbitration. During the pendency of
the voluntary arbitration proceedings, the petitioner dismissed Eugene Lucente,
a union member, due to an alleged petty quarrel with a co-employee. In view of
this termination, private respondent Union filed with the NLRC a complaint for
illegal dismissal. On June 24, 1999 Private respondent Jose Lanorias, a union

member, was also relieved from his post. Respondent Billy Bacus, the union vicepresident, protested Lanorias's dismissal. Shortly thereafter, respondents staged
a "wildcat strike." Notice of Strike was re-filed by the private respondents and
the protest, according to the respondents, was converted into a "sit-down strike."
On June 26, 1999, the same was transformed into an "actual strike."

Petitioner filed a complaint for illegal strike with the NLRC against private
respondents. Having arrived at no amicable settlement, the parties submitted
their position papers in compliance with the orders of the Labor Arbiter.

The principal question before the Labor Arbiter was whether the private
respondents staged an illegal strike. Ruling in the affirmative, the Labor Arbiter
held that the Notice of Strike dated December 3, 1998 referred to a prior dispute
submitted for voluntary arbitration and, hence, they cannot apply to the strike
staged about six months later, which commenced on June 24, 1999 and ended on
June 26, 1999; that, for these reasons, the Union failed to comply with the
mandatory requisites for a lawful strike; that, instead of resorting to a strike,
private respondents should have availed of the proper legal remedies; that, the
root causes of the controversy are the petition for certification election and
petition for cancellation of union registration which were then pending before
the DOLE as well as the issue on unfair labor practice then pending before the
voluntary arbitrator, and, hence, the parties should have awaited the resolution
of the cases in the proper fora; and that even if private respondents complied
with all the requisites of a valid strike, the strike is still illegal due to the
commission of prohibited acts. It HELD:
WHEREFORE, premises considered, respondents are hereby declared to have
staged an illegal strike, and the employment of union officers and all individual
respondents are deemed validly terminated in accordance with law.

Private respondents appealed to the NLRC. In overruling the Labor Arbiter, the
NLRC held that the petitioner is guilty of union busting; that the petitioner
violated the Submission Agreement in that no termination shall be effected
during the voluntary arbitration proceedings and, hence, the strike was justified;
that the Notice of Strike dated December 3, 1998 are applicable to the strike of
June 24, and 26, 1999 since the same issues of unfair labor practice were

3E Andaya Ching Espiritu Hefti Galvez Gammad Lainez Lui Madamba Nagera Narvasa
Ong Palangdao Rosales Sanchez Santos Satrain Tabo (2014-2015)

Labor Relations Case Digest - Atty. Joyrich Golangco

involved and that unfair labor practices are continuing offenses; that even if the
foregoing Notice of Strike were not applicable, the Union may take action
immediately since the petitioner is guilty of union busting; and that the re-filing
of a Notice of Strike cured the defect of non-compliance with the mandatory
requirements.
After the NLRC denied the Motion for Reconsideration, the petitioner appealed to
the CA. CA denied the petition and affirmed the NLRC.
ISSUES:

1. Whether the strike staged by the private respondents is illegal; and

2. Whether private respondents are deemed to have lost their employment


status by participating in the commission of illegal acts during the strike.
HELD:

The petition is meritorious.

1. YES. The undisputed fact is that at the time the strike was staged in June 1999,
voluntary arbitration between the parties was ongoing by virtue of the
Submission Agreement. The issue to be resolved under those proceedings
pertained to the very same issues stated in the Notice of Strike of December 3,
1998: the commission of unfair labor practices.
Article 264 of the Labor Code provides:

Art. 264. Prohibited activities.


xxxx

No strike or lockout shall be declared after assumption of jurisdiction by


the President or the Secretary or after certification or submission of the
dispute to compulsory or voluntary arbitration or during the pendency
of cases involving the same grounds for the strike or lockout.

x x x x (emphasis supplied)

This Court has held that strikes staged in violation of agreements providing for
arbitration are illegal, since these agreements must be strictly adhered to and
respected if their ends are to be achieved.10 The rationale of the prohibition
under Article 264 is that once jurisdiction over the labor dispute has been
properly acquired by competent authority, that jurisdiction should not be
interfered with by the application of the coercive processes of a strike.11
The alleged dismissals of Lucente and respondent Lanorias are not sufficient
grounds to justify the radical recourse of strike on the part of the private
respondents. The questions that surround their dismissal are connected to the
alleged breach of the "guarantee" by the petitioner not to dismiss its employees
during the pendency of the arbitration case. These matters should have been
raised and resolved in the voluntary arbitration proceedings that were
commenced precisely to address them. On the other hand, if private respondents
believed that the disciplinary measures had nothing to do with the issues under
arbitration, then they should have availed of the appropriate remedies under the
Labor Code, such as the institution of cases of illegal dismissal15 or, by agreement
of the parties, the submission of the cases to the grievance machinery of the CBA,
if one is available, so that they may be subjected to separate voluntary
arbitration proceedings,16 or simply seek to terminate the pending voluntary
arbitration case and complete the mandatory procedure for a lawful strike. And
because of the fact that the Union was fully aware that the arbitration
proceedings were pending, good faith cannot be invoked as a defense. Hence, this
Court declares that the strike staged by the private respondents is illegal.19
With respect to respondents' averment that assuming arguendo that the Notice
of Strike and Strike Vote in December 1998 cannot be made to apply to the strike
in June 1999, the requirements for a valid strike may nonetheless be dispensed
with in case of union busting. The language of the law leaves no room for doubt
that the cooling-off period and the seven-day strike ban after the strike-vote
report were intended to be mandatory,21 and in case of union busting where the
existence of the union is threatened, it is only the 15-day cooling-off period that
may be dispensed with.

3E Andaya Ching Espiritu Hefti Galvez Gammad Lainez Lui Madamba Nagera Narvasa
Ong Palangdao Rosales Sanchez Santos Satrain Tabo (2014-2015)

Labor Relations Case Digest - Atty. Joyrich Golangco

The implementing rules clarify Article 263(c) in that the union may strike
"immediately" provided that the strike vote is conducted, the results thereof
submitted "in every case" at least seven days before the intended strike or
lockout. In sum, in case of alleged union busting, the three remaining
requirements notice, strike vote, and seven-day report period cannot be
dispensed with.22
What is more, the strike had been attended by the widespread commission of
prohibited acts. Well-settled is the rule that even if the strike were to be declared
valid because its objective or purpose is lawful, the strike may still be declared
invalid where the means employed are illegal.23 Among such limits are the
prohibited activities under Article 264 of the Labor Code. The evidence in the
record clearly and extensively shows that the individual respondents engaged in
illegal acts during the strike, such as the intimidation and harassment of a
considerable number of customers to turn them away and discourage them from
patronizing the business of the petitioner; angry and unruly behavior calculated
to cause commotion38 which affected neighboring establishments within the
mall;39 openly cursing and shouting at the president in front of customers40 and
using loud and abusive language.
2. In the determination of the liabilities of the individual respondents, the
applicable provision is Article 264(a) of the Labor Code:
Art. 264. Prohibited Activities (a) x x x
xxxx

Thus, the Labor Arbiter is correct in ruling that the employment of all individual
private respondents are deemed validly terminated.

WHEREFORE, the petition is granted. Union officers who participated in the


illegal strike and in the commission of illegal acts, namely, Emmanuel Cayno,
Billy Bacus, Analiza Cablay, Jose Neil Arcilla, Roel Esancha, and Claudio
Panaligan, as well as the union members who participated in the commission of
illegal acts during the strike, namely, Rey Arsenal, Alex Martinez, Hermie Raz,
Jose Lanorias, Lito Arce, Cesar Sangreo, Rolando Fabregas, Jimmy Balan, Joven
Lualhati, Antonio Enebrad, Edgar Eugenio, Albert Agbuya, Arnel Salvador, Ricky
Del Prado, Bernie Del Mundo, Roberto Eco, Joven Talidong, Leny Lucente,
Rigoberto Tubaon, Merly Naz, Lino Salubre, Rolando Pugong, and John Bathan, all
private respondents, are hereby declared to have lost their employment status.
SO ORDERED.
FACTS:

x x x x Any union officer who knowingly participates in an illegal strike


and any worker or union officer who knowingly participates in the
commission of illegal acts during a strike may be declared to have lost
his employment status: Provided, That mere participation of a worker in
a lawful strike shall not constitute sufficient ground for termination of
his employment, even if a replacement had been hired by the employer
during such lawful strike.
xxxx

an ordinary striking worker cannot be terminated for mere participation in an


illegal strike. There must be proof that he or she committed illegal acts during a
strike. A union officer, on the other hand, may be terminated from work when he
knowingly participates in an illegal strike, and like other workers, when he
commits an illegal act during a strike.50 In all cases, the striker must be identified.
But proof beyond reasonable doubt is not required. Substantial evidence
available under the attendant circumstances, which may justify the imposition of
the penalty of dismissal, may suffice.51

117. BIFLEX UNION (and Filfex Union) vs. FILFLEX (Biflex Inc)

Petitioners were officers of Biflex (Phils.) Inc. Labor Union and Filflex
Industrial and Manufacturing Labor Union, which are the respective
collective bargaining agents of the employees of respondent
corporations.
Respondents are sister companies engaged in the garment business.
Situated in one big compound along with another sister company, they
have a common entrance.

3E Andaya Ching Espiritu Hefti Galvez Gammad Lainez Lui Madamba Nagera Narvasa
Ong Palangdao Rosales Sanchez Santos Satrain Tabo (2014-2015)

Labor Relations Case Digest - Atty. Joyrich Golangco

On October 24, 1990, the labor sector staged a welga ng bayan to


protest the accelerating prices of oil. On even date, petitioner-unions,
led by their officers (herein petitioners), staged a work stoppage
which lasted for several days, prompting respondents to file on October
31, 1990 a petition to declare the work stoppage illegal for failure to
comply with procedural requirements.
On November 13, 1990, respondents resumed their operations.
Petitioners, claiming that they were illegally locked out by
respondents, assert that aside from the fact that the welga ng bayan
rendered it difficult to get a ride and the apprehension that violence
would erupt between those participating in the welga and the
authorities, respondents workers were prevented from reporting for
work and were barred from entering the company premises.
Petitioners also alleged that the reason behind putting up of tents, tables
and chairs in front of the main gate of respondents premises were for
the convenience of union members who reported every morning to
check if the management would allow them to report for work. Further,
they claim that they filed a notice of strike on October 31, 1990.
Respondents, on the other hand, maintain that the work stoppage
was illegal since the following requirements for the staging of a valid
strike were not complied with: (1) filing of notice of strike; (2) securing
a strike vote, and (3) submission of a report of the strike vote to the
Department of Labor and Employment.
Labor Arbiter: strike was illegal and the officers are declared to
have lost their employment status.
Hence, Petitioners were terminated by the respondents
NLRC: reversed the ruling of LA, holding that there was no strike to
speak of as no labor or industrial dispute existed between the parties. It
ordered respondents to reinstate petitioners to their former
positions, without loss of seniority rights, and with full backwages from
the date of their termination.
Court of Appeals: reversed that of the NLRC and reinstated that of the
Labor Arbiter. CA discredited petitioners claim of having been illegally
locked out, given their failure to even file a letter of protest or complaint
with the management, and their failure to comply with the legal
requirements of a valid strike and while petitioners claimed that they

filed a notice of strike on October 31, 1990, no copy thereof was ever
produced before the Labor Arbiter

ISSUES: Illegal Stoppage of Work, Illegal Lockout, Illegal Strike, Dismissal of the
Officers
HELD: The petition fails.

The stoppage of work was illegal.

That petitioners staged a work stoppage on October 24, 1990 in


conjunction with the welga ng bayan organized by the labor sector to
protest the accelerating prices of oil, it is not disputed.
Stoppage of work due to welga ng bayan is in the nature of a
general strike, an extended sympathy strike. It affects numerous
employers including those who do not have a dispute with their
employees regarding their terms and conditions of employment.
Employees who have no labor dispute with their employer but who,
on a day they are scheduled to work, refuse to work and instead
join a welga ng bayan commit an illegal work stoppage.
Even if petitioners joining the welga ng bayan were considered merely
as an exercise of their freedom of expression, freedom of assembly or
freedom to petition the government for redress of grievances, the
exercise of such rights is not absolute. For the protection of other
significant state interests such as the "right of enterprises to reasonable
returns on investments, and to expansion and growth" enshrined in the
1987 Constitution must also be considered, otherwise, oppression or
self-destruction of capital in order to promote the interests of labor
would be sanctioned. And it would give imprimatur to workers
joining demonstrations/rallies even before affording the employer
an opportunity to make the necessary arrangements to counteract
the implications of the work stoppage on the business, and ignore
the novel "principle of shared responsibility between workers and
employers" aimed at fostering industrial peace.
There being no showing that petitioners notified respondents of
their intention, or that they were allowed by respondents, to join
the welga ng bayan on October 24, 1990, their work stoppage is

3E Andaya Ching Espiritu Hefti Galvez Gammad Lainez Lui Madamba Nagera Narvasa
Ong Palangdao Rosales Sanchez Santos Satrain Tabo (2014-2015)

beyond legal protection.

Labor Relations Case Digest - Atty. Joyrich Golangco

No illegal lockout

If there was illegal lockout, why did not petitioners file a protest
with the management or a complaint therefor against respondents?
As the Labor Arbiter observed, "[t]he inaction of [petitioners] betrays
the weakness of their contention for normally a locked-out union will
immediately bring management before the bar of justice."

Strike was illegal

Even assuming arguendo that in staging the strike, petitioners had


complied with legal formalities, the strike would just the same be
illegal, for by blocking the free ingress to and egress from the
company premises, they violated Article 264(e) of the Labor Code
which provides that "[n]o person engaged in picketing shall obstruct
the free ingress to or egress from the employers premises for lawful
purposes, or obstruct public thoroughfares."
In fine, the legality of a strike is determined not only by compliance
with its legal formalities but also by the means by which it is
carried out.

Officers who knowingly participates in an illegal strike may be declared


terminated by employer

Petitioners, being union officers, should thus bear the


consequences of their acts of knowingly participating in an illegal
strike, conformably with the third paragraph of Article 264 (a) of the
Labor Code which provides:
. . . Any union officer who knowingly participates in an illegal
strike and any worker or union officer who knowingly
participates in the commission of illegal acts during a strike may
be declared to have lost his employment status: Provided, That
mere participation of a worker in a lawful strike shall not

constitute sufficient ground for termination of his employment,


even if a replacement had been hired by the employer during such
lawful strike. (Emphasis and underscoring supplied)

In Gold City Integrated Port Service, Inc. v. National Labor Relations


Commission, this Court, passing on the use of the word "may" in the
immediately quoted provision, held that "[t]he law . . . grants the
employer the option of declaring a union officer who participated
in an illegal strike as having lost his employment." Reinstatement
of a striker or retention of his employment, despite his
participation in an illegal strike, is a management prerogative
which this Court may not supplant

118. SANTA ROSA COCA-COLA PLANT EMPLOYEES UNION, DONRICO V.


SEBASTIAN, EULOGIO G. BATINO, SAMUEL A. ATANQUE, MANOLO C.
ZABALJAUREGUI, DIONISIO TENORIO, EDWIN P. RELLORES, LUIS B.
NATIVIDAD, MYRNA PETINGCO, FELICIANO TOLENTINO, RODOLFO A.
AMANTE, JR., CIPRIANO C. BELLO, RONALDO T. ESPINO, EFREN GALAN, and
JUN CARMELITO SANTOS, Petitioners,
vs. COCA-COLA BOTTLERS PHILS., INC., Respondent.

FACTS: The petitioner Union is the sole and exclusive bargaining representative
of the regular daily paid workers and the monthly paid non-commission-earning
employees of the Coca-Cola Bottlers Philippines, Inc. (Company) in its Sta. Rosa,
Laguna plant.

The Union and the Company had entered into a three-year CBA effective July 1,
1996 to expire on June 30, 1999. Upon its expiration, the Union informed the
Company of its desire to renegotiate its terms. The CBA meetings commenced on
July 26, 1999 where they discussed the ground rules of the negotiations. The
Union insisted that representatives from the Alyansa ng mga Unyon sa Coca-Cola
be allowed to sit down as observers in the CBA meetings. The Union officers and
members also insisted that their wages be based on their work shift rates.

The Company was of the view that the members of the Alyansa were not
members of the bargaining unit. The Alyansa was a mere aggregate of employees

3E Andaya Ching Espiritu Hefti Galvez Gammad Lainez Lui Madamba Nagera Narvasa
Ong Palangdao Rosales Sanchez Santos Satrain Tabo (2014-2015)

Labor Relations Case Digest - Atty. Joyrich Golangco

of the Company in its various plants; and is not a registered labor organization.
Thus, an impasse ensued.

On August 30, 1999, the Union a "Notice of Strike" with the NCMB Regional Office
in Imus, Cavite claiming: (a) deadlock on CBA ground rules; and (b) unfair labor
practice arising from the companys refusal to bargain.

The Company filed a Motion to Dismiss alleging that the reasons cited by the
Union were not valid grounds for a strike. The Union then filed an Amended
Notice of Strike on September 17, 1999 on the following grounds: (a) unfair labor
practice for the companys refusal to bargain in good faith; and (b) interference
with the exercise of their right to self-organization.
Meanwhile, on September 15, 1999, the Union decided to participate in a mass
action organized by the Alyansa in front of the Companys premises set for
September 21, 1999. 106 Union members individually filed applications for leave
of absence for September 21, 1999.

Certain that its operations in the plant would come to a complete stop since there
were no sufficient trained contractual employees who would take over, the
Company disapproved all leave applications and notified the applicants
accordingly. A day before the mass action, some Union members wore gears, red
tag cloths stating "YES KAMI SA STRIKE" as headgears and on the different parts
of their uniform, shoulders and chests.
The Office of the Mayor issued a permit to the Union, allowing it "to conduct a
mass protest action within the perimeter of the Coca-Cola plant on September
21, 1999 from 9:00 a.m. to 12:00 noon." Union officers and members held a
picket along the front perimeter of the plant. As a result, only one of the three
bottling lines operated during the day shift. All the three lines were operated
during the night shift with cumulative downtime of five (5) hours due to lack of
manning, complement and skills requirement. The volume of production for the
day was short by 60,000 physical case[s] versus budget.
On October 13, 1999, the Company filed a "Petition to Declare Strike
Illegal" alleging, among others: on September 21, 1999, without observing the
requirements mandated by law, the Union picketed the premises of the

Company in clear violation of Article 262 of the Labor Code; the Company
suffered losses amounting to P2,733,366.29; the mass/protest action
conducted on September 21, 1999 was clearly a strike; since the Union did
not observe the requirements mandated by law, i.e., strike vote, cooling-off
period and reporting requirements, the strike was therefore illegal; the
Union also violated the provision of the CBA on the grievance machinery; and the
officers who knowingly participated in the commission of illegal acts during the
strike should be declared to have lost their employment status.

The Union filed an Answer with a Motion to Dismiss and/or to Suspend


Proceedings alleging therein that the mass action conducted by its officers and
members on September 21, 1999 was not a strike but just a valid exercise of
their right to picket, which is part of the right of free expression as
guaranteed by the Constitution; The Union insisted that officers and members
filed their applications for leave for September 21, 1999 knowing fully well that
there were no bottling operations scheduled on September 21 and 22, 1999; they
even secured a Mayors permit for the purpose. The workers, including the
petitioners, merely marched to and fro at the side of the highway near one of the
gates of the Sta. Rosa Plant, the loading bay for public vehicles. After 3 hours,
everyone returned to work according to their respective shifting schedules. The
Union averred that the petition filed by the Company was designed to harass and
its officers and members in order to weaken the Unions position in the on-going
collective bargaining negotiations.
In a letter to the Union President, NCMB stated that based on their allegations,
the real issue between the parties was not the proper subject of a strike, and
should be the subject of peaceful and reasonable dialogue. The NCMB
recommended that the Notice of Strike of the Union be converted into a
preventive mediation case. After conciliation proceedings failed, the parties were
required to submit their respective position papers.
On November 26, 1999, the Labor Arbiter rendered a Decision15 granting the
petition of the Company. He declared that the September 21, 1999 mass
leave was actually a strike under Article 212 of the Labor Code for the
following reasons: there was a temporary work stoppage/slowdown in the
company; out of the usual three (3) lines for production for the day shift, only

3E Andaya Ching Espiritu Hefti Galvez Gammad Lainez Lui Madamba Nagera Narvasa
Ong Palangdao Rosales Sanchez Santos Satrain Tabo (2014-2015)

Labor Relations Case Digest - Atty. Joyrich Golangco

one line operated by probationary employees was functional and there was a
cumulative downtime of five (5) hours attributed to the lack of manning
complement and skills requirement. The LA further held that the strike
conducted by the Union was illegal since there was no showing that the Union
conducted a strike vote, observed the prescribed cooling-off period, much less,
submitted a strike vote to the DOLE within the required time. Consequently, for
knowingly participating in the illegal strike, the individual petitioners were
considered to have lost their employment status.
The Union appealed the decision to the NLRC. NLRC affirmed the decision of the
Labor Arbiter. The Union filed a petition for certiorari in the CA. CA rendered
judgment dismissing the petition for lack of merit.
ISSUES: WON the activity conducted on Sept. 21 is a strike; WON the strike was
legal; WON the dismissal of the Union officers is valid.

HELD: Petitioners maintain that the September 21, 1999 mass protest action was
not a strike but a picket, a valid exercise of their constitutional right to free
expression and assembly. It was a peaceful mass protest action to dramatize
their legitimate grievances against respondent.
The petition is denied for lack of merit.

The ruling of the CA that petitioners staged a strike on September 21, 1999, and
not merely a picket is correct.

Article 212(o) of the Labor Code defines strike as a temporary stoppage of work
by the concerted action of employees as a result of an industrial or labor dispute.
In Bangalisan v. Court of Appeals, the Court ruled that "the fact that the
conventional term strike was not used by the striking employees to describe
their common course of action is inconsequential, since the substance of the
situation, and not its appearance, will be deemed to be controlling." The term
"strike" encompasses not only concerted work stoppages, but also slowdowns,
mass leaves, sit-downs, attempts to damage, destroy or sabotage plant
equipment and facilities, and similar activities.

Picketing involves merely the marching to and fro at the premises of the
employer, usually accompanied by the display of placards and other signs
making known the facts involved in a labor dispute.34 As applied to a labor
dispute, to picket means the stationing of one or more persons to observe and
attempt to observe. The purpose of pickets is said to be a means of peaceable
persuasion.35

The basic elements of a strike are present in this case: 106 members of petitioner
Union, whose respective applications for leave of absence on September 21, 1999
were disapproved, opted not to report for work on said date, and gathered in
front of the company premises to hold a mass protest action. Petitioners
deliberately absented themselves and instead wore red ribbons, carried placards
with slogans such as: "YES KAMI SA STRIKE," "PROTESTA KAMI," "SAHOD,
KARAPATAN NG MANGGAGAWA IPAGLABAN," "CBA-WAG BABOYIN," "STOP
UNION BUSTING." They marched to and fro in front of the companys premises
during working hours. Thus, petitioners engaged in a concerted activity which
already affected the companys operations. The mass concerted activity
constituted a strike.
The bare fact that petitioners were given a Mayors permit is not conclusive
evidence that their action/activity did not amount to a strike. The Mayors
description of what activities petitioners were allowed to conduct is
inconsequential. To repeat, what is definitive of whether the action staged by
petitioners is a strike and not merely a picket is the totality of the circumstances
surrounding the situation.

For a strike to be valid, the following procedural requisites provided by Art. 263
of the Labor Code must be observed: (a) a notice of strike filed with the DOLE 30
days before the intended date thereof, or 15 days in case of unfair labor practice;
(b) strike vote approved by a majority of the total union membership in the
bargaining unit concerned obtained by secret ballot in a meeting called for that
purpose, (c) notice given to the DOLE of the results of the voting at least seven
days before the intended strike. These requirements are mandatory and the
failure of a union to comply therewith renders the strike illegal. It is clear in
this case that petitioners totally ignored the statutory requirements and
embarked on their illegal strike.

3E Andaya Ching Espiritu Hefti Galvez Gammad Lainez Lui Madamba Nagera Narvasa
Ong Palangdao Rosales Sanchez Santos Satrain Tabo (2014-2015)

Labor Relations Case Digest - Atty. Joyrich Golangco

As observed by the LA, there is no showing that respondents had observed the
prescribed cooling-off period, conducted a strike vote, much less submitted a
strike vote report to the Department of Labor within the required time. This
being the case, respondents strike on September 21, 1999 is illegal.

Further, the strike staged by respondents was in violation of the CBA which
stipulated under Section 1, Article VI, thereof that, The UNION agrees that there
shall be no strike, walkout, stoppage or slowdown of work, boycott, secondary
boycott, refusal to handle any merchandise, picketing, sitdown strikes of any
kind, sympathetic or general strike, or any other interference with any of the
operations of the COMPANY during the term of this Agreement, so long as the
grievance procedure for which provision is made herein is followed by the
COMPANY.
Here, it is not disputed that respondents had not referred their issues to the
grievance machinery as a prior step. Instead, they chose to go on strike right
away, thereby bypassing the required grievance procedure dictated by the CBA.

As to the issue of dismissal, the law makes a distinction between union members
and union officers. A worker merely participating in an illegal strike may not be
terminated from employment. It is only when he commits illegal acts during a
strike that he may be declared to have lost employment status. For knowingly
participating in an illegal strike or participates in the commission of illegal acts
during a strike, the law provides that a union officer may be terminated from
employment The law grants the employer the option of declaring a union officer
who participated in an illegal strike as having lost his employment. It possesses
the right and prerogative to terminate the union officers from service. The
dismissal of the Union officers is proper.

119. Manila Hotel Employees Association and its members vs. Manila Hotel
Corporation

FACTS: On 11 November 1999, the MHEA filed a Notice of Strike with the
National Conciliation and Mediation Board (NCMB) in its National Capital Region
office against Manila Hotel on the grounds of unfair labor practices. Upon the
petition of Manila Hotel, the Secretary of Labor and Employment (SOLE) certified
the labor dispute to the NLRC for compulsory arbitration pursuant to Article

263(g) of the Labor Code on 24 November 1999. Specifically, the Order enjoined
any strike or lockout and the parties were ordered to cease and desist from
committing any acts that may exacerbate the situation. The parties and their
counsels were served copies of the said Order. MHEA filed a Motion for
Reconsideration dated 29 November 1999 assailing the validity of said Order.

The case was set for mandatory conference on 8 February 2000 before Presiding
Commissioner Rogelio I. Rayala. During the conference, the parties were advised
of the certification order, which prohibited them from taking any action that
would exacerbate the situation. At the instance of the MHEA officers, the hearing
of the case was reset to 29 February 2000 due to the absence of the counsel for
MHEA.

On 10 February 2000, the MHEA conducted a strike despite the clear terms of the
Order issued by the SOLE on 24 November 1999, and despite the repeated
reminders thereof. Several conferences were conducted by the NLRC, wherein
both parties were warned against aggravating the already volatile situation. The
NLRC sought to have both parties identify the issues and stipulate the facts,
despite their reluctance. It also allowed the parties sufficient time to file their
position papers, with which both parties failed to comply.

After the strike was conducted, both parties filed various motions and pleadings
before the NLRC. Manila Hotel filed a complaint with Prayer for Injunction
and/or Temporary Restraining Order on 11 February 2000, alleging that MHEA
conducted an illegal strike, blocked all ingress and egress of the hotel premises,
harassed and intimidated company officers, non-striking employees, customers
and suppliers. In addition, it sought a declaration that the strike was illegal and
that; consequently, the striking employees lost their employment.
The NLRC issued an Order dated 11 February 2000 directing the striking
workers to return to work immediately and the hotel to accept them back under
the same terms and conditions of employment. The NLRC received a copy of the
Compliance filed by Manila Hotel on 14 February 2000, manifesting that only six
striking employees complied with the return-to-work Order and were reinstated.
The other striking employees had openly defied the said Order. In response to
the NLRCs return-to-work order, dated 11 February 2000, the MHEA filed an

3E Andaya Ching Espiritu Hefti Galvez Gammad Lainez Lui Madamba Nagera Narvasa
Ong Palangdao Rosales Sanchez Santos Satrain Tabo (2014-2015)

Labor Relations Case Digest - Atty. Joyrich Golangco

Urgent Manifestation and Motion to Set Aside Order on 14 February 2000. It


alleged that the NLRC had not acquired jurisdiction over the labor dispute
pending the resolution of the Motion for Reconsideration filed before the SOLE.
The NLRC also issued another Order on 17 February 2000, ordering MHEA to
refrain from putting up a blockade or barricade or any mode of preventing the
free ingress to and egress from the hotel. Parenthetically, it also ordered Manila
Hotel to respect the right of the striking workers to peacefully picket in a
designated area outside the hotel. Manila Hotel moved for the Reconsideration of
the said Order on the ground that the picket, which they were ordered to respect,
was an unlawful activity.
NLRC ruled that the 10 February 2000 strike held by MHEA was illegal for
its defiance of the return-to-work order. However, it determined that only
the union officers were deemed to have lost their employment. It ruled that
there was no evidence showing who among the striking employees were
actually notified of the return-to-work order, and therefore, such
employees have not forfeited their employment. But in view of the
antagonism on both sides, the NLRC awarded a severance pay equivalent to onemonth salary to the returning union members for every year of service, instead
of ordering Manila Hotel to reinstate them. Manila Hotel filed a Petition for
Certiorari under Rule 65 before the Court of Appeals. In the decision of the CA,
the assailed Decision is MODIFIED in that both the incumbent officers and
members of the Union involved in the illegal strike are declared to have
lost their employment status. The award of severance compensation to the
striking members of the union is consequently DELETED.
ISSUES: W/N CA and the Respondent Commission had acted with grave abuse of
discretion and they have committed a reversible error, when by law and settled
jurisprudence, the individual petitioners who are mere ordinary members of the
Union, are entitled to be reinstated to work without loss of seniority or other
employees rights and benefits and with full back wages from date of dismissal
until actual reinstatement.
HELD: No. Aside from its procedural defects, the petition is also substantially
infirm. MHEA members seek their reinstatement after participating in an illegal
strike, that is, a strike that was conducted after receiving an Order of assumption

by the SOLE certifying the dispute to the NLRC for compulsory arbitration. Worse
still, the strikers failed to comply with the 11 February 2000 return-to-work
Order, issued by the NLRC, despite receipt thereof. The law explicitly prohibits
such acts.
ART. 263. STRIKES, PICKETING, AND LOCKOUTS
xxxx

(g) When, in his opinion there exists a labor dispute causing or likely to cause a
strike or lockout in an industry indispensable to the national interest, the
Secretary of Labor and Employment may assume jurisdiction over the dispute
and decide it or certify the same to the Commission for compulsory arbitration.
Such assumption or certification shall have the effect of automatically enjoining
the intended or impending strike or lockout as specified in the assumption or
certification order. If one has already taken place at the time of the assumption
or certification, all striking or locked out employees shall immediately return to
work and the employer shall immediately resume operations and readmit all
workers under the same terms and conditions prevailing before the strike or
lockout. The Secretary of Labor and Employment or the Commission may seek
the assistance of law enforcement agencies to ensure compliance with this
provision as well as with such orders as he may issue to enforce the same.
ART. 264. PROHIBITED ACTIVITIES
(a) x x x x

No strike or lockout shall be declared after assumption of jurisdiction by the


President or the Minister or after certification or submission of the dispute to
compulsory or voluntary arbitration or during the pendency of cases involving
the same grounds for the strike or lockout.

More to the point, the Court has consistently ruled in a long line of cases
spanning several decades that once the SOLE assumes jurisdiction over a labor
dispute, such jurisdiction should not be interfered with by the application of the
coercive processes of a strike or lockout. Defiance of the assumption order or a

3E Andaya Ching Espiritu Hefti Galvez Gammad Lainez Lui Madamba Nagera Narvasa
Ong Palangdao Rosales Sanchez Santos Satrain Tabo (2014-2015)

Labor Relations Case Digest - Atty. Joyrich Golangco

return-to work order by a striking employee, whether a union officer or a


member, is an illegal act and, therefore, a valid ground for loss of employment
status.

The assumption of jurisdiction by the SOLE over labor disputes causing or likely
to cause a strike or lockout in an industry indispensable to the national interest
is in the nature of a police power measure. In this case, the SOLE sufficiently
justified the assumption order.
The Hotel is engaged in the hotel and restaurant business and one of the de luxe
hotels operating in Metro Manila catering mostly to foreign tourist groups and
businessmen. It serves as venue for local and international conventions and
conferences. The Hotel provides employment to more than 700 employees as
well as conducts business with entities dependent on its continued operation. It
also provides substantial contribution to the government coffers in the form of
foreign exchange earnings and tax payments. Undoubtedly, a work stoppage
thereat will adversely affect the Hotel, its employees, the industry, and the
economy as a whole.

At this critical time when efforts of the present administration are seriously
focused on preserving the economic gains achieved and ensuring that existing
jobs are maintained, it is the utmost concern of this Office to avoid work
disruption that might result to the firms closure particularly so when an
alternative mechanism obtains to resolve the parties differences.

The allegation that the strikers relied on their honest belief that the filing of a
Motion for Reconsideration of the Order, issued by the SOLE on 24 November
1999, entitled them to participate in a strike, cannot be sustained. The very
nature of a return-to-work order issued in a certified case lends itself to no other
construction. The certification attests to the urgency of the matter, affecting as it
does an industry indispensable to the national interest. The order is issued in the
exercise of the courts compulsory power of arbitration, and therefore must be
obeyed until set aside. To say that its [return-to-work order] effectivity must
await affirmance on a motion for reconsideration is not only to emasculate it but
indeed to defeat its import, for by then the deadline fixed for the return to work

would, in the ordinary course, have already passed and hence can no longer be
affirmed insofar as the time element it concerned.
Returning to work in this situation is not a matter of option or voluntariness but
of obligation. The worker must return to his job together with his co-workers so
the operations of the company can be resumed and it can continue serving the
public and promoting its interest.

This extraordinary authority given to the Secretary of Labor is aimed at arriving


at a peaceful and speedy solution to labor disputes, without jeopardizing national
interests. Regardless therefore of their motives, or the validity of their claims, the
striking workers must cease and/or desist from any and all acts that tend to, or
undermine this authority of the Secretary of Labor, once an assumption and/or
certification order is issued. They cannot, for instance, ignore return-to-work
orders, citing unfair labor practices on the part of the company, to justify their
action.
This extraordinary authority given to the Secretary of Labor is aimed at arriving
at a peaceful and speedy solution to labor disputes, without jeopardizing national
interests. Regardless therefore of their motives, or the validity of their claims, the
striking workers must cease and/or desist from any and all acts that tend to, or
undermine this authority of the Secretary of Labor, once an assumption and/or
certification order is issued. They cannot, for instance, ignore return-to-work
orders, citing unfair labor practices on the part of the company, to justify their
action.
As a general rule, the sympathy of the Court is on the side of the laboring classes,
not only because the Constitution imposes sympathy but because of the onesided relation between labor and capital. The Court must take care, however, that
in the contest between labor and capital, the results achieved are fair and in
conformity with the rules.
120. G & S Transport Corporation vs Infante

3E Andaya Ching Espiritu Hefti Galvez Gammad Lainez Lui Madamba Nagera Narvasa
Ong Palangdao Rosales Sanchez Santos Satrain Tabo (2014-2015)

Labor Relations Case Digest - Atty. Joyrich Golangco

FACTS: - Petitioner was the exclusive coupon taxi concessionaire at the NAIA for
5 years (1989-1994 concession contract). Under such contract the taxi units
were given a garage located at the Duty Free compound just opposite NAIA.

- NAIA sent a letter to the NAIA Service Taxi Employees Union (Union)
demanding the dismissal from employment Ricardo Gonzales and Ephraim
Alzaga (both drivers of herein petitioner) on the grounds of disloyalty and
unbecoming of a union member inimical to the interests of the Union. Petitioner
terminated said drivers.
- This caused several drivers to stop driving their taxis apparently to sympathize
with their dismissed colleagues on May 16, 1990. Petitioner claims that this
brought stoppage to its business operation and is one that squarely falls under
the concept of an illegal strike at the work premises.

-Petitioner filed an action for illegal strike before the LA against 37 drivers
(among those drivers were herein private respondents Infante, Velasco, Borbo
and Castaeda). The 37 drivers filed an illegal dismissal case but most of them
filed their affidavits of desistance.

-Defense of Infante and Borbo: they reported to work but found no taxis in
their garage and the dispatcher and the sales clerk were also not around and
found out that on that same day the protest to sympathize with their colleagues
ensued. The day after the protest they did not report back to work, as it was their
day-off. However, when they returned to work they were refused entry by the
security guard because their names did not appear in the list of drivers allowed
to work by the petitioner.
-Defense of Castaeda: I was on a sick leave. But when I reported back to work
it was on the same day that the strike occurred. The following day, he reported
back to work but was refused entry by the guard.

-Decision of LA: concerted action by the drivers is illegal strike. Drivers


undertook those collective actions without first filing a notice to strike and taking
a strike vote, and in violation of the no strike no lock out clause in the CBA. The
LA further noted that when the strike occurred there was no labor dispute (as
Article 212 mandates that such stoppage of work by employees be a result of an
industrial of labor disputes). LA found respondents PARTICIPATED in the illegal

strike BUT did not affirm their dismissal. Instead, ordered that separation pay be
given in lieu of reinstatement but without backwages.
-NLRC affirmed in toto LAs ruling.

-CAs Decision: REVERSED. It remanded it to the LA for the computation of


backwages and other monetary benefits. It further noted that the dismissal of
respondents was illegal. CA relied on a certification from SEC that petitioner was
still operational and that the LA and NLRC ruling for separation pay instead of
reinstatement amounted to grave abuse of discretion.
-petitioner filed MR but was denied, CA should not try facts and order
reinstatement and backwages.
ISSUES:

1. Whether respondents participated in the illegal strike.


2. Whether the order for the payment of separation pay, in lieu of
reinstatement without backwages, was proper.
HELD:
1.
YES. The strike undertaken by respondents took the form of a sit-down
strike, or more aptly termed as a sympathetic strike, where the striking
employees have no demands or grievances of their own, but they strike for the
purpose of directly or indirectly aiding others, without direct relation to the
advancement of the interest of the strikers. It is indubitable that an illegal strike
in the form of a sit-down strike occurred in petitioners premises, as a show of
sympathy to the two employees who were dismissed by petitioner.
Respondents failed to satisfactorily explain their conspicuous absence
following the day of the purported illegal strike. No record whatsoever was
presented by Borbo and Infante to prove that 17 May 1990 was their day-off. It
was convenient to pass the buck on petitioner by alleging that proof of their alibi
is in petitioners file. Castaeda could not even present a sick leave form to attest
to his absence from 11-15 May 1990.

What is the effect of their participation? Article 264 of the Labor Code,
in providing for the consequences of an illegal strike, makes a distinction

3E Andaya Ching Espiritu Hefti Galvez Gammad Lainez Lui Madamba Nagera Narvasa
Ong Palangdao Rosales Sanchez Santos Satrain Tabo (2014-2015)

Labor Relations Case Digest - Atty. Joyrich Golangco

between union officers and members who participated therein. Thus, knowingly
participating in an illegal strike is a valid ground for termination of employment
of a union officer. The law, however, treats differently mere union members.
Mere participation in an illegal strike is not a sufficient ground for termination of
the services of the union members. The Labor Code protects an ordinary, rankand-file union member who participated in such a strike from losing his job,
provided that he did not commit an illegal act during the strike. It can be gleaned
from the aforecited provision of law in point, however, that an ordinary striking
employee cannot be terminated for mere participation in an illegal strike. There
must be proof that he committed illegal acts during the strike and the striker
who participated in the commission of illegal act must be identified. Proof
beyond reasonable doubt is not required. Substantial evidence available under
the attendant circumstances, which may justify the imposition of the penalty of
dismissal, may suffice.
In the case at bar, evidence presented by the petitioner DOES NOT in any
way constitute an affirmation that respondents committed illegal acts. Notably,
no questions during the hearing were asked relative to the supposed illegal acts.

As adverted to earlier, no matter by what term the respondents


complainants used in describing their concerted action, i.e. [,] protest, sympathy
or mere expression, their joint action have successfully paralyzed the operations
of G & S Transport, and this is considered a strike.

2.
YES. It can now therefore be concluded that the acts of respondents do
not merit their dismissal from employment because it has not been substantially
proven that they committed any illegal act while participating in the illegal strike.

With respect to backwages, the principle of a "fair days wage for a fair days
labor" remains as the basic factor in determining the award thereof. If there is no
work performed by the employee there can be no wage or pay unless, of course,
the laborer was able, willing and ready to work but was illegally locked out,
suspended or dismissed or otherwise illegally prevented from working. While it
was found that respondents expressed their intention to report back to work, the
latter exception cannot apply in this case. In Philippine Marine Officers Guild v.
Compaia Maritima, as affirmed in Philippine Diamond Hotel and Resort v.
Manila Diamond Hotel Employees Union, the Court stressed that for this

exception to apply, it is required that the strike be legal, a situation that does not
obtain in the case at bar. Under the circumstances, respondents reinstatement
without backwages suffices for the appropriate relief. If reinstatement is no
longer possible, given the lapse of considerable time from the occurrence of the
strike, the award of separation pay of one (1) month salary for each year of
service, in lieu of reinstatement, is in order.

The SEC has certified that G & S Transport Corporation was registered
on 5 January 1972 for a period of fifty (50) years and as of 6 August 1999; no
document showing its dissolution had been filed. Petitioner cannot seek refuge
behind the mere assertion that respondents employment is conditioned on the
five-year concession with NAIA. No employment contract was presented to
support such fact. Petitioner in fact even admitted that it obtained another
concession from NAIA in 2000.

The fact remains that petitioner still operates a taxi concession in NAIA
and that logically requires the service of taxi drivers, the same position held by
respondents back in 1990. Section 4, Rule I of the Rules Implementing Book VI of
the Labor Code provides:

SEC. 4. Reinstatement to former position.(a) An employee who is


separated from work without just cause shall be reinstated to his former
position, unless such position no longer exists at the time of his
reinstatement, in which case he shall be given a substantially equivalent
position in the same establishment without loss of seniority rights.
The above-quoted rule enunciates reinstatement as the standard relief. However,
in this case, seventeen (17) years have elapsed since respondents were illegally
dismissed. In Association of Independent Unions in the Philippines v. NLRC,
where more than eight (8) years have passed since the petitioners therein staged
an illegal strike and were found to have been unlawfully terminated, an award of
separation pay equivalent to one (1) month pay for every year of service, in lieu
of reinstatement, was deemed more practical and appropriate to all the parties
concerned. We adopt the same tack in this case.
In sum, the resolution and order of the NLRC, which adopted the findings of
the Labor Arbiter, are in accordance with law and jurisprudence.

3E Andaya Ching Espiritu Hefti Galvez Gammad Lainez Lui Madamba Nagera Narvasa
Ong Palangdao Rosales Sanchez Santos Satrain Tabo (2014-2015)

Labor Relations Case Digest - Atty. Joyrich Golangco

121. STEEL CORPORATION OF THE PHILIPPINES, petitioner, vs. SCP


EMPLOYEES UNION-NATIONAL FEDERATION OF LABOR UNIONS,
respondent.

FACTS: Petitioner Steel Corporation of the Philippines (SCP) is engaged in


manufacturing construction materials. SCP-Federated Union of the Energy
Leaders-General and Allied Services (FUEL-GAS) filed a petition for Certification
Election in its bid to represent the rank-and-file employees of the petitioner.
Respondent SCP Employees Union-National Federation of Labor Unions (SCPEUNAFLU) intervened, seeking to participate and be voted for in such election but
the same was denied for having been filed out of time.
A consent election was conducted, with "FUEL-GAS" and "NO UNION" as choices.
Said election was however declared a failure because less than a majority of the
rank-and-file employees cast their votes. FUEL-GAS filed an Election Protest
claiming that the certification election was characterized by and replete with
irregularities.
NAFLU, the mother federation of respondent, filed a petition for Certification
Election for and on behalf of its affiliate, seeking to represent the rank-and-file
employees of petitioner.

The Med-Arbiter denied the election protest of FUEL-GAS and granted the
petition for certification election filed by NAFLU and further ordered the conduct
of the election with "NAFLU" and "NO UNION" as choices.

Petitioner and FUEL-GAS appealed to the Secretary of Labor. The DOLE


Undersecretary ordered the conduct of a certification election with "FUEL-GAS,"
respondent SCPEU-NAFLU and "NO UNION" as choices. Subsequent motions for
reconsideration were denied. Unsatisfied, petitioner and FUEL-GAS appealed to
the CA by way of certiorari.

On April 14, 2000, the certification election proceeded. FUEL-GAS participated


without prejudice to the decision of the CA in its pending petition. In said
election, respondent SCPEU-NAFLU emerged as winner. FUEL-GAS filed a second
election protest.

The CA annulled and set aside the DOLE Usecs decision and resolution, and
directed the holding of a certification election with "FUEL-GAS" and "NO UNION"
as choices, to the exclusion of SCPEU-NAFLU.
The Med-Arbiter dismissed FUEL-GAS' election protest but deferred the
declaration of respondent as winner in the certification election until final
resolution of the pending petitions with the CA.

Respondent appealed to the Labor Secretary. It further filed a Manifestation


before the CA pointing out that in the certification election, it emerged as winner,
and thus, the election should be considered as an intervening event sufficient to
bar another certification election. The CA, however, dismissed said
manifestation.

Meanwhile, the Usec rendered a Decision certifying respondent as the exclusive


bargaining agent of petitioner's employees. Petitioner and FUEL-GAS filed MRs of
the aforesaid decision.
As a consequence of its certification as the exclusive bargaining agent,
respondent sent to petitioner CBA proposals. Petitioner, however, held in
abeyance any action on the proposals in view of its pending motion for
reconsideration.

Thus, respondent filed a Notice of Strike with NCMB raising the issue of unfair
labor practice (ULP) allegedly committed by petitioner for its refusal to bargain.
The labor dispute was certified to the NLRC for compulsory arbitration. Another
Notice of Strike was filed by respondent which was later consolidated with the
certified case.
NLRC held that petitioner has no obligation to recognize respondent as the
certified bargaining agent; dismissing the charge of ULP; declaring the strike as
illegal; and declaring the loss of employment of the officers of the union.

On May 20, 2002, respondent filed another Notice of Strike which was dismissed
and respondent was enjoined from holding a strike. On Jan. 7, 2003, it again filed
another Notice of Strike and thereafter went on strike on February 4, 2003. The

3E Andaya Ching Espiritu Hefti Galvez Gammad Lainez Lui Madamba Nagera Narvasa
Ong Palangdao Rosales Sanchez Santos Satrain Tabo (2014-2015)

Labor Relations Case Digest - Atty. Joyrich Golangco

Labor Secretary certified the dispute to the NLRC and directed the employees
to return to work. NLRC ordered petitioner to bargain collectively with
respondent as the duly certified bargaining agent. It also ordered the
reinstatement of the employees dismissed in connection with the strike.

Meantime, in the first certified case, the NLRC ordered the reinstatement of the
union officers whom it previously ordered terminated.

CA affirmed the order of reinstatement but nullified and set aside the second
certification election. It added that since petitioner did not recognize the second
certification election where NAFLU won, respondent has no basis for its claim
and no right to demand that petitioner collectively bargain with it.
Petitioner filed MR which was denied; hence, this petition.

(Whether or not respondent is the recognized collective bargaining agent had


been finally resolved in the negative)

ISSUES 1. WON the strike participated in by the officers of the respondent union
is valid
2. WON the respondent union officers termination from employment by reason
of such participation is valid
Held:

1, NO. The strike is a legitimate weapon in the human struggle for a decent
existence. It is considered as the most effective weapon in protecting the rights of
the employees to improve the terms and conditions of their employment. But to
be valid, a strike must be pursued within legal bounds. The right to strike as a
means for the attainment of social justice is never meant to oppress or destroy
the employer. The law provides limits for its exercise.59
In the instant case, the strike undertaken by the officers of respondent union is
patently illegal for the following reasons: (1) it is a union-recognition-strike
which is not sanctioned by labor laws; (2) it was undertaken after the dispute

had been certified for compulsory arbitration; and (3) it was in violation of the
Secretary's return-to-work order.

Respondent's notices of strike were founded on petitioner's continued refusal to


bargain with it. It thus staged the strike to compel petitioner to recognize it as
the collective bargaining agent, making it a union-recognition-strike. As its legal
designation implies, this kind of strike is calculated to compel the employer to
recognize one's union and not other contending groups, as the employees'
bargaining representative to work out a collective bargaining agreement despite
the striking union's doubtful majority status to merit voluntary recognition and
lack of formal certification as the exclusive representative in the bargaining unit.

The certification election that was conducted where respondent emerged as


winner, not having been recognized as valid, it has no authority to represent the
rank and file employees of petitioner. Thus, it could not ask petitioner to bargain
with it. As the issue of its identity had been the subject of a separate case which
had been settled by the court with finality, petitioner cannot, therefore, be
faulted in refusing to bargain. Neither could this Court sustain respondent's
imputation of unfair labor practice and union busting against petitioner. With
more reason, this Court cannot sustain the validity of the strike staged on such
basis.
Even if this Court were to uphold the validity of respondent's purpose or
objective in staging a strike, still, the strike would be declared illegal for having
been conducted in utter defiance of the Secretary's return-to-work order and
after the dispute had been certified for compulsory arbitration. Although
ostensibly there were several notices of strike successively filed by respondent,
these notices were founded on substantially the same grounds petitioner's
continued refusal to recognize it as the collective bargaining representative.

The powers granted to the Secretary under Article 263(g) of the Labor Code have
been characterized as an exercise of the police power of the State, aimed at
promoting the public good. When the Secretary exercises these powers, he is
granted "great breadth of discretion" to find a solution to a labor dispute. The
most obvious of these powers is the automatic enjoining of an impending strike
or lockout or its lifting if one has already taken place.

3E Andaya Ching Espiritu Hefti Galvez Gammad Lainez Lui Madamba Nagera Narvasa
Ong Palangdao Rosales Sanchez Santos Satrain Tabo (2014-2015)

Labor Relations Case Digest - Atty. Joyrich Golangco

The moment the Secretary of Labor assumes jurisdiction over a labor dispute in
an industry indispensable to national interest, such assumption shall have the
effect of automatically enjoining the intended or impending strike. It was not
even necessary for the Secretary of Labor to issue another order directing a
return to work. The mere issuance of an assumption order by the Secretary of
Labor automatically carries with it a return-to-work order, even if the directive
to return to work is not expressly stated in the assumption order.

A return-to-work order imposes a duty that must be discharged more than it


confers a right that may be waived. While the workers may choose not to obey,
they do so at the risk of severing their relationship with their employer.

Returning to work in this situation is not a matter of option or voluntariness but


of obligation. The worker must return to his job together with his co-workers so
that the operations of the company can be resumed and it can continue serving
the public and promoting its interest. This extraordinary authority given to the
Secretary of Labor is aimed at arriving at a peaceful and speedy solution to labor
disputes, without jeopardizing national interests. Regardless of their motives, or
the validity of their claims, the striking workers must cease and/or desist from
any and all acts that undermine or tend to undermine this authority of the
Secretary of Labor, once an assumption and/or certification order is issued. They
cannot, for instance, ignore return-to-work orders, citing unfair labor practices
on the part of the company, to justify their action.
xxx

The strike, having been staged after the dispute had been certified for arbitration
and contrary to the return-to-work order, became a prohibited activity, and was
thus illegal.
2. YES. The law (Art. 264) makes a distinction between union members and
union officers. A worker merely participating in an illegal strike may not be
terminated from employment. It is only when he commits illegal acts during a
strike that he may be declared to have lost employment status. For knowingly
participating in an illegal strike or participating in the commission of illegal acts
during a strike, the law provides that a union officer may be terminated from
employment. The law grants the employer the option of declaring a union officer

who participated in an illegal strike as having lost his employment. It possesses


the right and prerogative to terminate the union officers from service. Otherwise,
the workers will simply refuse to return to their work and cause a standstill in
the company operations while retaining the positions they refuse to discharge
and preventing management from filling up their positions.
122. ST. LUKE'S MEDICAL CENTER, INC. and ROBERT KUAN,
Chairman, Petitioners, vs. ESTRELITO NOTARIO, Respondent.

DOCTRINE: A single or isolated act of negligence that cannot be categorized as


habitual and, hence, not a just cause for his dismissal;

FACTS: On June 23, 1995, St. Lukes Medical Center, Inc. employed respondent as
In-House Security Guard. In August 1996, Nimaya Electro Corporation installed
CCTV system in the premises of petitioner hospital to enhance its security
measures6 and conducted an orientation seminar for the in-house security
personnel on the proper way of monitoring video cameras.

On December 30, 1996, respondent was on duty from 6:00 p.m. to 6:00 a.m. of
the following day. His work consisted mainly of monitoring the video
cameras. In the evening of December 30, 1996, Justin Tibon, a foreigner, then
attending to his 3-year-old daughter, Andanie De Brum, who was admitted at
room 257, cardiovascular unit of petitioner hospital, reported to the
management about the loss of his mint green traveling bag, which was placed
inside the cabinet, containing, Continental Airlines tickets, passports, and some
clothes. The Security Department of petitioner hospital conducted an
investigation. When the tapes of VCR no. 3 covering the subject period were
reviewed, it was shown that the VCR was focused on camera no. 2 (Old Maternity
Unit), from 9:03 p.m. to 10:15 p.m. of December 30, 1996, and camera no. 1 (New
Maternity Unit), from or 12:25 a.m. to 6:00 a.m. of December 31, 1996. The
cameras failed to record any incident of theft at room 257.
On January 6, 1997, petitioner hospital, through Abdul A. Karim, issued a
Memorandum8 to respondent directing him to explain in writing, within 24
hours upon its receipt, why no disciplinary action should be taken against him

3E Andaya Ching Espiritu Hefti Galvez Gammad Lainez Lui Madamba Nagera Narvasa
Ong Palangdao Rosales Sanchez Santos Satrain Tabo (2014-2015)

Labor Relations Case Digest - Atty. Joyrich Golangco

for violating the normal rotation/sequencing process of the VCR and failed
to capture the theft of Tibon's bag.

In his letter dated January 6, 1997, respondent explained that he was the only
personnel on duty as nobody wanted to assist him. Because of this, he decided to
focus the cameras on the Old and New Maternity Units, as these two units have
high incidence of crime.
Finding the written explanation of respondent to be unsatisfactory, respondent
was served a copy of the Notice of Termination, dated January 24, 1997,
dismissing him on the ground of gross negligence/inefficiency under
Section 1, Rule VII of its Code of Discipline.

Thus, on March 19, 1997, respondent filed a Complaint11 for illegal dismissal,
seeking reinstatement with payment of full backwages from the time of his
dismissal up to actual reinstatement, without of loss of seniority rights and other
benefits.
Petitioners countered that they validly dismissed respondent for gross
negligence and observed due process before terminating his employment.
Labor Arbiter dismissed respondents complaint for illegal dismissal on the
ground of negligence.

On appeal, the NLRC issued a Resolution reversing the Decision of the Labor
Arbiter. It stated that petitioners failed to submit proof that there was an existing
Standard Operating Procedure in the CCTV monitoring system, particularly on
the focusing procedure. MR was denied.

On September 21, 2001, the CA dismissed petitioners' petition for certiorari,


affirming the NLRCs finding that while respondent may appear to be negligent in
monitoring the cameras on the subject dates, the same would not constitute
sufficient ground to terminate his employment. Even assuming that
respondents act would constitute gross negligence, it ruled that the
ultimate penalty of dismissal was not proper as it was not habitual, and
that there was no proof of pecuniary injury upon petitioner hospital.

Moreover, it declared that petitioners failed to comply with the twin notice
rule and hearing as what they did was to require respondent to submit a
written explanation, within 24 hours and, thereafter, he was ordered
dismissed, without affording him an opportunity to be heard. MR was
denied.
ISSUES: WON respondent was guilty of gross negligence which warrants his
dismissal.

HELD: Respondent was illegally dismissed without just cause and compliance
with the notice requirement.

An employer can terminate the services of an employee only for valid and just
causes which must be supported by clear and convincing evidence. The employer
has the burden of proving that the dismissal was indeed for a valid and just
cause.
Under Article 282 (b) of the Labor Code, an employer may terminate an
employee for gross and habitual neglect of duties. Neglect of duty must be both
gross and habitual. Gross negligence connotes want of care in the performance of
ones duties. Habitual neglect implies repeated failure to perform ones duties
for a period of time, depending upon the circumstances. A single or isolated act
of negligence does not constitute a just cause for the dismissal of the
employee.

A perusal of the hospitals CCTV Monitoring Guidelines, disseminated to all inhouse security personnel, reveals that that there is no categorical provision
requiring an in-house security personnel to observe a rotation sequence
procedure in focusing the cameras so that the security monitoring would cover
as many areas as possible. Under the prevailing circumstances, respondent
exercised his best judgment in monitoring the CCTV cameras so as to ensure the
security within the hospital premises. Verily, assuming arguendo that respondent
was negligent, the lapse or inaction could only be regarded as a single or isolated
act of negligence that cannot be categorized as habitual and, hence, not a just
cause for his dismissal.

3E Andaya Ching Espiritu Hefti Galvez Gammad Lainez Lui Madamba Nagera Narvasa
Ong Palangdao Rosales Sanchez Santos Satrain Tabo (2014-2015)

Labor Relations Case Digest - Atty. Joyrich Golangco

As to the issue of the required twin notice rule and hearing, The CA found that
petitioner hospital failed to comply with the requirements as it merely required
respondent to give his written explanation within 24 hours and, thereafter,
ordered his dismissal.

To effectuate a valid dismissal from employment by the employer, the Labor


Code has set twin requirements, namely: (1) the dismissal must be for any of the
causes provided in Article 282 of the Labor Code; and (2) the employee must be
given an opportunity to be heard and defend himself. This first requisite is
referred to as the substantive aspect, while the second is deemed as the
procedural aspect.
The Implementing Rules provides that:

Section 2. Security of Tenure. (a) In cases of regular employment, the employer


shall not terminate the services of an employee except for just or authorized
causes as provided by law, and subject to the requirements of due process.
(d) In all cases of termination of employment, the following standards of due
process shall be substantially observed:

For termination of employment based on just causes as defined in Article 282 of


the Labor Code:
(i) A written notice served on the employee specifying the ground or
grounds for termination, and giving said employee reasonable
opportunity within which to explain his side.

(ii) A hearing or conference during which the employee concerned, with


the assistance of counsel if he so desires is given opportunity to respond
to the charge, present his evidence, or rebut the evidence presented
against him.

(iii) A written notice of termination served on the employee, indicating


that upon due consideration of all the circumstances, grounds have been
established to justify his termination.

The facts showed that on January 6, 1997, petitioner hospital issued a


Memorandum to respondent, with the directive to to explain in writing, within
24 hours upon receipt thereof, why no disciplinary action should be taken
against him. On the same day, respondent submitted a written explanation. On
January 24, 1997, petitioner hospital served a copy of the Notice of Termination
upon the respondent for gross negligence/inefficiency.

Petitioners lack of just cause and non-compliance with the procedural requisites
in terminating respondents employment renders them guilty of illegal dismissal.
123. Aliling vs World Express

DOCTRINES:
1.
2.
3.

Two-notice rule: one, of the intention to dismiss, indicating


therein his acts or omissions complained against, and two,
notice of the decision to dismiss;
Company did not make known the reasonable standards under
which he will qualify as a regular employee at the time of his
engagement. Hence, he was deemed to have been hired from
day one as a regular employee.
When reinstatement is no longer possible, this Court has
consistently ruled that backwages shall be computed from the
time of illegal dismissal until the date the decision becomes final

FACTS
Via a letter dated June 2, 2004, respondent Wide Wide World Express
Corporation (WWWEC) offered to employ petitioner Armando Aliling as
Account Executive (Seafreight Sales). The offer came with a six (6)month probation period condition with this express caveat:
Performance during probationary period shall be made as basis for
confirmation to Regular or Permanent Status.
On June 11, 2004, Aliling and WWWEC inked an Employment Contract
under the following terms, among others.
o Conversion to regular status shall be determined on the basis of
work performance; and
o Employment services may, at any time, be terminated for just
cause or in accordance with the standards defined
Training then started. Instead of a Seafreight Sale assignment, Aliling

3E Andaya Ching Espiritu Hefti Galvez Gammad Lainez Lui Madamba Nagera Narvasa
Ong Palangdao Rosales Sanchez Santos Satrain Tabo (2014-2015)

Labor Relations Case Digest - Atty. Joyrich Golangco

was asked to handle Ground Express (GX), a new company product


involving domestic cargo forwarding service for Luzon. Marketing this
product and finding daily contracts for it formed the core of Alilings new
assignment.
Barely a month after, Manuel San Mateo (WWWEC Sales and Marketing
Director) emailed Aliling to express dissatisfaction with the latters
performance at the time of engagement.
Thereafter, in a letter of September 25, 2004,Joseph Lariosa (Human
Resources Manager) asked Aliling to report to explain his absence taken
without leave from September 20, 2004.
Aliling responded two days later, denying being absent on the days in
question. Alilings explanation came with a query regarding the
withholding of his salary corresponding to September 11 to 25, 2004.
In a separate letter dated September 27, 2004, Aliling wrote San Mateo
stating: Pursuant to your instruction on September 20, 2004, I hereby
tender my resignation effective October 15, 2004.
While WWWEC took no action on his tender, Aliling nonetheless
demanded reinstatement and a written apology, claiming in a
subsequent letter dated October 1, 2004 to management that San Mateo
had forced him to resign.
Oct. 1, 2004 - Lariosas informed Aliling that his case was still in the
process of being evaluated.
Oct 4, 2.004 - Aliling filed a Complaint for illegal dismissal due to forced
resignation, nonpayment of salaries as well as damages with the NLRC
against WWWEC.
Alilings contention: At the time of his engagement, respondents did not make
known to him e the standards under which I will qualify as a regular employee.
WWWEC contention: The letter-offer and the employment contract contained
the terms of engagement with a note that failure to meet the job requirements
during the probation stage means his services may be terminated without prior
notice and without recourse to separation pay. And when San Meteo wrote him a
memo to explain his unsatisfactory performance, Aliling simply submitted a
resignation letter.

Oct. 6, 2004 Lariosa again wrote, to advise Aliling of the termination of

his services effective as of that date owing to his non-satisfactory


performance during his probationary period.

LA: Alilings termination is unjustified. Reason: The grounds upon which


complainants dismissal was based did not conform not only the standard but also
the compliance required under Article 281 of the Labor Code, The labor arbiter
gave credence to Alilings allegation about not receiving and, therefore, not bound
by, San Mateos purported September 20, 2004 memo. The memo, to reiterate,
supposedly apprised Aliling of the sales quota he was, but failed, to meet. Pushing
the point, the labor arbiter explained that Aliling cannot be validly terminated for
non-compliance with the quota threshold absent a prior advisory of the reasonable
standards upon which his performance would be evaluated.
NLRC: affirmed LA

CA: affirmed NLRCs decision with modifications. Reason: (a) respondents failed
to prove that Alilings dismal performance constituted gross and habitual neglect
necessary to justify his dismissal; (b) not having been informed at the time of his
engagement of the reasonable standards under which he will qualify as a
regular employee, Aliling was deemed to have been hired from day one as a
regular employee; and (c) the strained relationship existing between the parties
argues against the propriety of reinstatement.
ISSUE/S: W/N he is a regular employee. W/N he was illegally dismissed

HELD: Yes to both

PROBATIONARY OR REGULAR (ART. 281)

Petitioner was regularized from the time of the execution of the


employment contract on June 11, 2004, although respondent company
had arbitrarily shortened his tenure. As pointed out, respondent
company did not make known the reasonable standards under
which he will qualify as a regular employee at the time of his
engagement. Hence, he was deemed to have been hired from day
one as a regular employee.

3E Andaya Ching Espiritu Hefti Galvez Gammad Lainez Lui Madamba Nagera Narvasa
Ong Palangdao Rosales Sanchez Santos Satrain Tabo (2014-2015)

Labor Relations Case Digest - Atty. Joyrich Golangco

WWWEC, however, argues that the fact that it was made known to
Aliling that he would be evaluated on the 3rd and 5th months of his
probationary employment is sufficient compliance with the requirement
that a probationary worker be apprised of the reasonable standards for
his regularization.
o To note, the letter-offer itself states that the regularization
standards or the performance norms to be used are still to be
agreed upon by Aliling and his supervisor. WWWEC has
failed to prove that an agreement as regards thereto has
been reached. Clearly then, there were actually no
performance standards to speak of. And lest it be overlooked,
Aliling was assigned an activity entirely different to the job he
was originally hired and trained for. Thus, at the time of his
engagement, the standards relative to his assignment with GX
sales could not have plausibly been communicated to him as he
was under Seafreight Sales.
Petitioner Aliling, albeit hired from managements standpoint as a
probationary employee, was deemed a regular employee by force of the
following self-explanatory provisions: ART. 281 and Sec. 6(d) Rule
VIII-A of the IRR of Labor Code (paki tignan nalang, di ko na kinopya
para di humaba lalo.).
o Contrary to respondentscontention, San Mateos email cannot
support their allegation on Aliling being informed of the
standards for his continued employment, such as the sales
quota, at the time of his engagement. As it were, the email
message was sent to Aliling more than a month after he signed
his employment contract with WWWEC. The aforequoted
Section 6 of the Implementing Rules specifically requires the
employer to inform the probationary employee of such
reasonable standards at the time of his engagement, not at
any time later; else, the latter shall be considered a regular
employee.

Illegal Dismissal (ART. 282)

rule, prove that the dismissal was for a just cause and that the
employee was afforded due process prior to dismissal. termination
from employment on that ground would still be unjustified.

Article 282 of the Labor Code considers any of the following acts or omission on the
part of the employee
as just cause or ground for terminating employment:
xxxxxxx
(b) Gross and habitual neglect by the employee of his duties;
xxxxxxx
(e) Other causes analogous to the foregoing.

Due Process; 2-notice Rule (Art. 277(b))

To justify fully the dismissal of an employee, the employer must, as a

In fine, an employees failure to meet sales or work quotas falls under


the concept of gross inefficiency, which in turn is analogous to gross
neglect of duty that is a just cause for dismissal under Article 282 of
theCode. However, in order for the quota imposed to be considered a
valid
productivity standard and thereby validate a dismissal,
managements prerogative of fixing the quota must be exercised in
good faith for the advancement of its interest. WWWEC must show
that such quota was imposed in good faith. This WWWEC failed to do,
perceptibly because it could not. The fact of the matter is that the
alleged imposition of the quota was a desperate attempt to lend a
semblance of validity to Alilings illegal dismissal. Being an
experimental activity and having been launched for the first time, the
sales of GX services could not be reasonably quantified.
While probationary employees do not enjoy permanent status, they
enjoy the constitutional protection of security of tenure. They can
only be terminated for cause or when they otherwise fail to meet the
reasonable standards made known to them by the employer at the time
of their engagement. Here Alilings security of tenure was breached.
When the Labor Code speaks of procedural due process, the reference is
usually to the two (2)-written notice rule envisaged in Section 2 (III),

3E Andaya Ching Espiritu Hefti Galvez Gammad Lainez Lui Madamba Nagera Narvasa
Ong Palangdao Rosales Sanchez Santos Satrain Tabo (2014-2015)

Labor Relations Case Digest - Atty. Joyrich Golangco

Rule XXIII, Book V of the Omnibus Rules Implementing the Labor


Code, which provides:
Section 2. Standard of due process: requirements of notice. In
all cases of termination of employment, the following standards of
due process shall be substantially observed.
I. For termination of employment based on just causes as defined
in Article 282 of the Code:
(a) A written notice served on the employee specifying the
ground or grounds for termination, and giving to said
employee reasonable opportunity within which to explain his side;
(b) A hearing or conference during which the employee
concerned, with the assistance of counsel if the employee so
desires, is given opportunity to respond to the charge, present his
evidence or rebut the evidence presented against him; and (c) A
written notice [of] termination served on the employee
indicating that upon due consideration of all the
circumstance, grounds have been established to justify his
termination. In case of termination, the foregoing notices shall
be served on the employees last known address
two-notice rule, x x x one, of the intention to dismiss, indicating
therein his acts or omissions complained against, and two, notice of
the decision to dismiss;
Here, the first and second notice requirements have not been properly
observed, thus tainting petitioners dismissal with illegality.

Backwages; Reinstatement; Separation Pay (Art. 279)


Since he is deemed a regular employee from the time of his employment,
he is entitled to backwages reckoned from the time he was illegally
dismissed on October 6, 2004, until the finality of this Decision.
Art. 279. Security of Tenure. - In cases of regular employment, the employer shall
not terminate the services of an employee except for a just cause or when
authorized by this Title. An employee who is unjustly dismissed from work
shall be entitled to reinstatement without loss of seniority rights and other
privileges and to his full backwages, inclusive of allowances, and to his other
benefits or their monetary equivalent computed from the time his

compensation was withheld from him up to the time of his actual


reinstatement.

But if, as in this case, reinstatement is no longer possible, this Court has
consistently ruled that backwages shall be computed from the time
of illegal dismissal until the date the decision becomes final.
Additionally, Aliling is entitled to separation pay in lieu of reinstatement
on the ground of strained relationship.

124. FELIX B. PEREZ vs PHILIPPINE TELEGRAPH AND TELEPHONE


COMPANY

DOCTRINE:
The following are the guiding principles in connection with the hearing
requirement in dismissal cases:
(a) ample opportunity to be heard means any meaningful opportunity (verbal
or written) given to the employee to answer the charges against him and submit
evidence in support of his defense, whether in a hearing, conference or some
other fair, just and reasonable way.
(b) a formal hearing or conference becomes mandatory only when requested by
the employee in writing or substantial evidentiary disputes exist or a company
rule or practice requires it, or when similar circumstances justify it.
(c) the ample opportunity to be heard standard in the Labor Code prevails over
the hearing or conference requirement in the implementing rules and
regulations.
Therefore, while the phrase ample opportunity to be heard may in fact include
an actual hearing, it is not limited to a formal hearing only. In other words, the
existence of an actual, formal trial-type hearing, although preferred, is not
absolutely necessary to satisfy the employees right to be heard.

FACTS:Petitioners were employed by respondent as shipping clerk and


supervisor, respectively. Acting on an alleged unsigned letter regarding
anomalous transactions, respondents formed a special audit team. It discovered
that the Shipping Section jacked up the value of the freight costs for goods
shipped and that the duplicates of the shipping documents allegedly showed
traces of tampering, alteration and superimposition. Afterwards, petitioners

3E Andaya Ching Espiritu Hefti Galvez Gammad Lainez Lui Madamba Nagera Narvasa
Ong Palangdao Rosales Sanchez Santos Satrain Tabo (2014-2015)

Labor Relations Case Digest - Atty. Joyrich Golangco

were placed on preventive suspension for 30 days for their alleged involvement
in the anomaly. Said suspension was extended for 15 days twice. Then, a
memorandum was issued by respondent stating that petitioners were hereby
dismissed from the service for having falsified company documents.

Petitioners filed a complaint for illegal suspension and illegal dismissal. The LA
found that the 30-day extension of petitioners suspension and their subsequent
dismissal were both illegal. He ordered respondents to pay petitioners their
salaries during their 30-day illegal suspension, as well as to reinstate them with
backwages and 13th month pay.

The NLRC reversed the decision of the labor arbiter. It ruled that petitioners
were dismissed for just cause, that they were accorded due process and that they
were illegally suspended for only 15 days.

Petitioners appealed to the CA which affirmed the NLRC decision insofar as


petitioners illegal suspension for 15 days and dismissal for just cause were
concerned. However, it found that petitioners were dismissed without due
process.
Petitioners now seek a reversal of the CA decision. They contend that there was
no just cause for their dismissal, that they were not accorded due process and
that they were illegally suspended for 30 days.

ISSUES:
1. WON RESPONDENTS FAILED TO PROVE JUST CAUSE AND TO OBSERVE
DUE PROCESS
2. WON PETITIONERS WERE ILLEGALLY SUSPENDED
HELD: We rule in favor of petitioners.

RESPONDENTS FAILED TO PROVE JUST CAUSE AND TO OBSERVE DUE


PROCESS
alterations on the shipping documents could not reasonably be attributed to
petitioners because it was never proven that petitioners alone had control of or

access to these documents. The burden of proof rests on the employer to


establish that the dismissal is for cause. It must be adequately proven by
substantial evidence. Respondents failed to discharge this burden.

To meet the requirements of due process in the dismissal of an employee, an


employer must furnish the worker with two written notices: (1) a written notice
specifying the grounds for termination and giving to said employee a reasonable
opportunity to explain his side and (2) another written notice indicating that,
upon due consideration of all circumstances, grounds have been established to
justify the employer's decision to dismiss the employee. Petitioners were neither
apprised of the charges against them nor given a chance to defend themselves
Petitioners likewise contended that due process was not observed in the absence
of a hearing in which they could have explained their side and refuted the
evidence against them. There is no need for a hearing or conference.

We note a marked difference in the standards of due process to be followed as


prescribed in the Labor Code and its implementing rules. The Labor Code, on one
hand, provides that an employer must provide the employee ample opportunity
to be heard and to defend himself with the assistance of his representative if he so
desires as stated under Art. 277. The omnibus rules implementing the Labor
Code, on the other hand, require a hearing and conference during which the
employee concerned is given the opportunity to respond to the charge, present
his evidence or rebut the evidence presented against him as stated under Section
2. The time-honored doctrine states that in case of conflict, the law prevails over
the administrative regulations implementing it.
PETITIONERS WERE ILLEGALLY SUSPENDED FOR 30 DAYS

An employee may be validly suspended by the employer for just cause provided
by law. Such suspension shall only be for a period of 30 days, after which the
employee shall either be reinstated or paid his wages during the extended
period.[ Petitioners contended that they were not paid during the two 15-day
extensions of their preventive suspension. Respondents failed to adduce
evidence to the contrary. Thus, we uphold the ruling of the labor arbiter on this
point.

3E Andaya Ching Espiritu Hefti Galvez Gammad Lainez Lui Madamba Nagera Narvasa
Ong Palangdao Rosales Sanchez Santos Satrain Tabo (2014-2015)

Labor Relations Case Digest - Atty. Joyrich Golangco

Where the dismissal was without just or authorized cause and there was
no due process, Article 279 of the Labor Code, as amended, mandates that the
employee is entitled to reinstatement. In this case, however, reinstatement is no
longer possible because of the length of time that has passed. Fourteen years
have transpired from the time petitioners were wrongfully dismissed.
WHEREFORE, the petition is hereby GRANTED.

125. Agabon vs NLRC

FACTS: Private respondent Riviera Home Improvements, Inc. is engaged in the


business of selling and installing ornamental and construction materials. It
employed petitioners Virgilio Agabon and Jenny Agabon as gypsum board and
cornice installers on January 2, 1992 until February 23, 1999 when they were
dismissed for abandonment of work. Petitioners then filed a complaint for illegal
dismissal and payment of money claims and on December 28, 1999, the Labor
Arbiter rendered a decision declaring the dismissals illegal and ordered private
respondent to pay the monetary claims which included backwages amounting to
56,231.93 and service incentive leave pay, holiday pay and 13th month pay
differentials amounting to 2,150.00. NLRC reversed the judgment and found that
the petitioners were validly dismissed for a just cause hence were not entitled to
backwages. On appeal CA, affirmed the ruling of the NLRC but reinstated the
entitlement of the petitioners to the 2,150.00. The petitioners elevated the case
to the Supreme court and restated their argument that they were illegally
dismissed for they had not abandoned their work and no notice of termination
was sent to them by the private respondent.
ISSUES: WON the petitioner were illegally dismissed

HELD:No. The courts found that they had abandoned their work. Abandonment
is the deliberate and unjustified refusal of an employee to resume his
employment.[ It is a form of neglect of duty, hence, a just cause for termination
of employment by the employer.[ For a valid finding of abandonment, these two
factors should be present: (1) the failure to report for work or absence without
valid or justifiable reason; and (2) a clear intention to sever employer-employee
relationship, with the second as the more determinative factor which is
manifested by overt acts from which it may be deduced that the employees has

no more intention to work. The intent to discontinue the employment must be


shown by clear proof that it was deliberate and unjustified.

The dismissal should be upheld because it was established that the petitioners
abandoned their jobs to work for another company. Private respondent,
however, did not follow the notice requirements and instead argued that sending
notices to the last known addresses would have been useless because they did
not reside there anymore. Unfortunately for the private respondent, this is not a
valid excuse because the law mandates the twin notice requirements to the
employees last known address.[21] Thus, it should be held liable for noncompliance with the procedural requirements of due process.
The rule thus evolved: where the employer had a valid reason to
dismiss an employee but did not follow the due process requirement, the
dismissal may be upheld but the employer will be penalized to pay an indemnity
to the employee. This became known as the Wenphil or Belated Due Process
Rule.
Where the dismissal is for a just cause, as in the instant case, the lack of statutory
due process should not nullify the dismissal, or render it illegal, or
ineffectual. However, the employer should indemnify the employee for the
violation of his statutory rights, as ruled in Reta v. National Labor Relations
Commission.The indemnity to be imposed should be stiffer to discourage the
abhorrent practice of dismiss now, pay later, which we sought to deter in
the Serrano ruling. The sanction should be in the nature of indemnification or
penalty and should depend on the facts of each case, taking into special
consideration the gravity of the due process violation of the employer. The
violation of the petitioners right to statutory due process by the private
respondent warrants the payment of indemnity in the form of nominal
damages. The amount of such damages is addressed to the sound discretion of
the court, taking into account the relevant circumstances Considering the
prevailing circumstances in the case at bar, we deem it proper to fix it at
P30,000.00.
126. JAKA Food Processing Corporation, vs. Darwin Pacot, Robert
Parohinog, David Bisnar, Marlon Domingo, Rhoel Lescano and Jonathan
Cagabcab

3E Andaya Ching Espiritu Hefti Galvez Gammad Lainez Lui Madamba Nagera Narvasa
Ong Palangdao Rosales Sanchez Santos Satrain Tabo (2014-2015)

Labor Relations Case Digest - Atty. Joyrich Golangco

DOCTRINE: In all cases of business closure or cessation of operation or


undertaking of the employer, the affected employee is entitled to separation
pay. This is consistent with the state policy of treating labor as a primary social
economic force, affording full protection to its rights as well as its welfare. The
exception is when the closure of business or cessation of operations is due
to serious business losses or financial reverses; duly proved, in which case,
the right of affected employees to separation pay is lost for obvious
reasons.
FACTS: Respondents were earlier hired by petitioner JAKA Foods Processing
Corporation until the latter terminated their employment because the
corporation was in dire financial straits. It is not disputed, however, that the
termination was effected without JAKA complying with the requirement under
Article 283 of the Labor Code regarding the service of a written notice upon the
employees and the Department of Labor and Employment at least one (1) month
before the intended date of termination.

Respondents filed complaints for illegal dismissal, underpayment of


wages and nonpayment of service incentive leave and 13th month pay against
JAKA. The Labor Arbiter rendered a decision declaring the termination illegal
and ordering JAKA to reinstate respondents with full backwages, and separation
pay if reinstatement is not possible.

JAKA went on appeal to the NLRC, which, in a decision dated August 30,
1999, affirmed in toto that of the Labor Arbiter. Their motion for reconsideration
having been denied by the NLRC in its resolution of April 28, 2000, respondents
went to the Court of Appeals via a petition for certiorari.

The Court of Appeals reversed said decision and ordered respondent


JAKA to pay petitioners separation pay equivalent to one (1) month salary, the
proportionate 13th month pay and, in addition, full backwages from the time
their employment was terminated.
ISSUES: WON the CA erred in awarding full backwages and separation pay to
respondents

HELD: The Court upheld the legality of the dismissal but ordering petitioner to
pay each of the respondents the amount of P50,000.00, representing nominal
damages for non-compliance with statutory due process.

A dismissal for just cause under Article 282 implies that the employee
concerned has committed, or is guilty of, some violation against the employer, i.e.
the employee has committed some serious misconduct, is guilty of some fraud
against the employer, or, as in Agabon, he has neglected his duties. Thus, it can
be said that the employee himself initiated the dismissal process.

On another breath, a dismissal for an authorized cause under Article 283


does not necessarily imply delinquency or culpability on the part of the
employee. Instead, the dismissal process is initiated by the employers exercise
of his management prerogative, i.e. when the employer opts to install labor
saving devices, when he decides to cease business operations or when, as in this
case, he undertakes to implement a retrenchment program.

The clear-cut distinction between a dismissal for just cause under Article
282 and a dismissal for authorized cause under Article 283 is further reinforced
by the fact that in the first, payment of separation pay, as a rule, is not required,
while in the second, the law requires payment of separation pay.
For these reasons, there ought to be a difference in treatment when
the ground for dismissal is one of the just causes under Article 282, and
when based on one of the authorized causes under Article 283.

Accordingly, it is wise to hold that: (1) if the dismissal is based on a just


cause under Article 282 but the employer failed to comply with the notice
requirement, the sanction to be imposed upon him should be tempered because
the dismissal process was, in effect, initiated by an act imputable to the
employee; and (2) if the dismissal is based on an authorized cause under Article
283 but the employer failed to comply with the notice requirement, the sanction
should be stiffer because the dismissal process was initiated by the employers
exercise of his management prerogative.

The records before us reveal that, indeed, JAKA was suffering from serious
business losses at the time it terminated respondents employment. As aptly
found by the NLRC.

3E Andaya Ching Espiritu Hefti Galvez Gammad Lainez Lui Madamba Nagera Narvasa
Ong Palangdao Rosales Sanchez Santos Satrain Tabo (2014-2015)

Labor Relations Case Digest - Atty. Joyrich Golangco

It was established that there was ground for respondents dismissal, i.e.,
retrenchment, which is one of the authorized causes enumerated under Article
283 of the Labor Code. Likewise, it is established that JAKA failed to comply with
the notice requirement under the same Article. Considering the factual
circumstances in the instant case, the Court deem it proper to fix the indemnity
at P50, 000.00. The Court of Appeals have been in error when it ordered
JAKA to pay respondents separation pay equivalent to one (1) month salary
for every year of service.
127. CULILI VS. EASTERN TELECOM
DOCTRINE: 'Moral damages are awarded only where the dismissal was attended
by bad faith or fraud, or constituted an act oppressive to labor, or was done in a
manner contrary to morals, good customs or public policy. Exemplary damages
may avail if the dismissal was effected in a wanton, oppressive or malevolent ma
nner to warrant an award for exemplary damages.'

FACTS: Culili was a Senior Technician in the Customer Premises Equipment Man
agement Unit of the Service Quality Department of Eastern Telecom (ETPI). Due
to business troubles and losses, ETPI was compelled to implement a Right-Sizing
Program which consisted of two phases: the first phase involved the reduction of
ETPIs workforce to only those employees that were necessary and which ETPI c
ould sustain; the second phase entailed a company-wide reorganization which w
ould result in the transfer, merger, absorption or abolition of certain department
s of ETPI. ETPI offered the Special Retirement Program and the corresponding re
tirement package to the 102 employees who qualified for the program. Of all the
employees who qualified to avail of the program, only Culili rejected the offer. Af
ter the successful implementation of the first phase of the Right-Sizing Program,
ETPI proceeded with the second phase which necessitated the abolition, transfer
and merger of a number of ETPIs departments. the specialized functions of a Se
nior Technician unnecessary. As a result, Culilis position was abolished due to r
edundancy and his functions were absorbed by another employee. Later, he was
informed of his termination from employment effective April 8, 1999 through a l
etter which was similar to the memo shown to Culili by the union president week
s before Culili was dismissed. The memo was dated December 7, 1998, and was a
dvising him of his dismissal effective January 4, 1999 due to the Right-Sizing Prog
ram ETPI was going to implement to cut costs and avoid losses. Culili alleged tha
t neither he nor the Department of Labor and Employment (DOLE) were formally

notified of his termination. Culili claimed that he only found out about it someti
me in March 1999 when Vice President Virgilio Garcia handed him a copy of the
March 8, 1999 letter, after he was barred from entering ETPIs premises by its ar
med security personnel when he tried to report for work. Culili believed that ET
PI had already decided to dismiss him even prior to the March 8, 1999 letter as e
videnced by the December 7, 1998 version of that letter. Moreover, Culili asserte
d that ETPI had contracted out the services he used to perform to a labor-only co
ntractor which not only proved that his functions had not become unnecessary, b
ut which also violated their CBA and the Labor Code. Aside from these, Culili also
alleged that he was discriminated against when ETPI offered some of his co-empl
oyees an additional benefit in the form of motorcycles to induce them to avail of t
he Special Retirement Program, while he was not. ETPI denied singling Culili out
for termination. ETPI averred that since Culili did not avail of the Special Retire
ment Program and his position was subsequently declared redundant, it had no c
hoice but to terminate Culili. Culili, however, continued to report for work. ETPI
said that because there was no more work for Culili, it was constrained to serve a
final notice of termination to Culili, which Culili ignored. ETPI tendered to Culili
his final check of P859,033.99 consisting of his basic salary, leaves, 13th month p
ay and separation pay. Culili filed a complaint against ETPI and its officers for ille
gal dismissal, unfair labor practice, and money claims before the Labor Arbiter.
LABOR ARBITER: He rendered a decision finding, among others, ETPI guilty of il
legal dismissal and unfair labor practice, and ordering it to reinstate Culili with b
ackwages and such other benefits due him, and damages and attorneys fees.

NLRC: Affirmed the LAs decision but modified the amount of the moral and exe
mplary damages awarded to P200,000 for moral and P100,000 for exemplary da
mages. Cases remanded to LA for the computation of backwages and other mone
tary awards.

CA: partially granted ETPIs petition and deleted the award for damages. CA foun
d that Culilis position was validly abolished due to redundancy. Court of Appeals
said that ETPI had been very candid with its employees in implementing its Right
-Sizing Program, and that it was highly unlikely that ETPI would effect a company
-wide reorganization simply for the purpose of getting rid of Culili. But it found t
hat ETPI failed to observe the standards of due process as required by our laws

3E Andaya Ching Espiritu Hefti Galvez Gammad Lainez Lui Madamba Nagera Narvasa
Ong Palangdao Rosales Sanchez Santos Satrain Tabo (2014-2015)

Labor Relations Case Digest - Atty. Joyrich Golangco

when it failed to properly notify both Culili and the DOLE of Culilis termination.

ISSUES: Whether or not ETPI failed to properly comply with the procedural due
process requirement in terminating Culili.

HELD: Yes. ETPI, in effecting Culilis termination, simply asked one of its guards
to serve the required written notice on Culili. Culili, on one hand, claims in his pe
tition that this was handed to him by ETPIs vice president, but previously testifie
d before the Labor Arbiter that this was left on his table. Regardless of how this
notice was served on Culili, this Court believes that ETPI failed to properly notify
Culili about his termination. Aside from the manner the written notice was serve
d, a reading of that notice shows that ETPI failed to properly inform Culili of the g
rounds for his termination. Since it has been established that Culilis termination
was due to an authorized cause and cannot be considered unfair labor practice o
n the part of ETPI, his dismissal is valid. However, in view of ETPIs failure to co
mply with the notice requirements under the Labor Code, Culili is entitled to nom
inal damages in addition to his separation pay.
As for the award of moral and exemplary damages in illegal dismissal cases, mor
al damages are awarded only where the dismissal was attended by bad faith or fr
aud, or constituted an act oppressive to labor, or was done in a manner contrary
to morals, good customs or public policy. Exemplary damages may avail if the dis
missal was effected in a wanton, oppressive or malevolent manner to warrant an
award for exemplary damages. It is our considered view that Culili has failed to
prove that his dismissal was orchestrated by the individual respondents herein (
ETPI officers) for the mere purpose of getting rid of him. In fact, most of them ha
ve not even dealt with Culili personally. Moreover, it has been established that hi
s termination was for an authorized cause, and that there was no bad faith on the
part of ETPI in implementing its Right-Sizing Program, which involved abolishing
certain positions and departments for redundancy. It is not enough that ETPI fail
ed to comply with the due process requirements to warrant an award of damage
s, there being no showing that the companys and its officers acts were attended
with bad faith or were done oppressively.
128. ANTONIO M. SERRANO vs. Gallant MARITIME SERVICES, INC. and
MARLOW NAVIGATION CO., INC.

DOCTRINE: It must be stressed that Section 3, Article XIII does not directly
bestow on the working class any actual enforceable right, but merely clothes it
with the status of a sector for whom the Constitution urges protection through
executive or legislative action and judicial recognition. Its utility is best limited to
being an impetus not just for the executive and legislative departments, but for
the judiciary as well, to protect the welfare of the working class.
FACTS: Petitioner was hired by Gallant under a POEA-approved Contract of
Employment for duration of 12 months. On the date of his departure, petitioner
was constrained to accept a downgraded employment contract for the position of
Second Officer with lesser salary, upon the assurance and representation of
respondents that he would be made Chief Officer later on. Respondents did not
deliver on their promise to make petitioner Chief Officer. Hence, petitioner
refused to stay on as Second Officer and was repatriated to the Philippines.
Petitioner's employment contract was for a period of 12 months, but at the time
of his repatriation, he had served only two (2) months and seven (7) days of his
contract, leaving an unexpired portion of nine (9) months and twenty-three (23)
days.
ISSUES: Whether or not Sec. 10 of RA 8042 is constitutional.
HELD: NO. Unconstitutional.

A closer examination reveals that the subject clause has a discriminatory intent
against, and an invidious impact on, OFWs at two levels:
First, OFWs with employment contracts of less than one year vis--vis OFWs
with employment contracts of one year or more;

Second, among OFWs with employment contracts of more than one year; and
Third, OFWs vis--vis local workers with fixed-period employment;

3E Andaya Ching Espiritu Hefti Galvez Gammad Lainez Lui Madamba Nagera Narvasa
Ong Palangdao Rosales Sanchez Santos Satrain Tabo (2014-2015)

Labor Relations Case Digest - Atty. Joyrich Golangco

OFWs with employment contracts of less than one year vis--vis OFWs with
employment contracts of one year or more

The enactment of the subject clause in R.A. No. 8042 introduced a differentiated
rule of computation of the money claims of illegally dismissed OFWs based on
their employment periods, in the process singling out one category whose
contracts have an unexpired portion of one year or more and subjecting them to
the peculiar disadvantage of having their monetary awards limited to their
salaries for 3 months or for the unexpired portion thereof, whichever is less, but
all the while sparing the other category from such prejudice, simply because the
latter's unexpired contracts fall short of one year.
Among OFWs With Employment Contracts of More Than One Year

The subject clause creates a sub-layer of discrimination among OFWs whose


contract periods are for more than one year: those who are illegally dismissed
with less than one year left in their contracts shall be entitled to their salaries for
the entire unexpired portion thereof, while those who are illegally dismissed
with one year or more remaining in their contracts shall be covered by the
subject clause, and their monetary benefits limited to their salaries for three
months only.
OFWs vis--vis Local Workers With Fixed-Period Employment

The Court concludes that the subject clause contains a suspect classification in
that, in the computation of the monetary benefits of fixed-term employees who
are illegally discharged, it imposes a 3-month cap on the claim of OFWs with an
unexpired portion of one year or more in their contracts, but none on the claims
of other OFWs or local workers with fixed-term employment. The subject clause
singles out one classification of OFWs and burdens it with a peculiar
disadvantage.
Note: Section 3, Article XIII cannot be treated as a principal source of direct
enforceable rights, for the violation of which the questioned clause may be
declared unconstitutional. It may unwittingly risk opening the floodgates of

litigation to every worker or union over every conceivable violation of so broad a


concept as social justice for labor.

The word salaries in Section 10(5) does not include overtime and leave
pay. For seafarers like petitioner, DOLE Department Order No. 33, series 1996,
provides a Standard Employment Contract of Seafarers, in which salary is
understood as the basic wage, exclusive of overtime, leave pay and other
bonuses; whereas overtime pay is compensation for all work "performed" in
excess of the regular eight hours, and holiday pay is compensation for any work
"performed" on designated rest days and holidays.
By the foregoing definition alone, there is no basis for the automatic
inclusion of overtime and holiday pay in the computation of petitioner's
monetary award, unless there is evidence that he performed work during
those periods.
The Court granted the petition. The subject clause "or for three months for every
year of the unexpired term, whichever is less" in the 5th paragraph of Section 10
of Republic Act No. 8042 is DECLARED UNCONSTITUTIONAL; and the
December 8, 2004 Decision and April 1, 2005 Resolution of the Court of Appeals
are MODIFIED to the effect that petitioner is AWARDED his salaries for the
entire unexpired portion of his employment contract consisting of nine months
and 23 days computed at the rate of US$1,400.00 per month. No costs.
129. Yap vs Thenamaris Ships Management (the respondent owns the
vessel)

DOCRTINES:
(1)The clause or for three months for every year of the unexpired term,
whichever is less provided in the 5th paragraph of Section 10 of R.A. No. 8042 is
unconstitutional for being violative of the rights of Overseas Filipino Workers
(OFWs) to equal protection of the laws and substantive due process.

(2) The doctrine of operative fact, as an exception to the general rule,


only applies as a matter of equity and fair play. It nullifies the effects of an
unconstitutional law by recognizing that the existence of a statute prior to a

3E Andaya Ching Espiritu Hefti Galvez Gammad Lainez Lui Madamba Nagera Narvasa
Ong Palangdao Rosales Sanchez Santos Satrain Tabo (2014-2015)

Labor Relations Case Digest - Atty. Joyrich Golangco

determination of unconstitutionality is an operative fact and may have


consequences that cannot always be ignored. The past cannot always be erased
by a new judicial declaration.
FACTS: Claudio Yap was employed as an electrician of the vessel M/T SEASCOUT
by Intermare Maritime Agencies. His employment contract is only for 12 months.
The commencement of which started on August 23, 2001. However, it was
preterminated and he was only able to work until Nov. 10, 2001 because the
vessel was sold and will be scrapped. The captain informed them of such event
and gave them an option of an immediate transfer to another vessel otherwise
declare that they do not wish to transfer.

Respondent claimed that Yap did not opt for immediate transfer hence it
paid him bonuses for seniority, vacation and for the scrapping and paid him his
wages corresponding to the months of work he made.
Yap argued that he opted for immediate transfer but none was made. He
also insisted that he was entitled to the payment for the unexpired portion of his
contract since he was illegally dismissed. Hence, he filed an illegal dismissal
complaint before the LA.
LA: Yap was constructively and illegally dismissed and respondent acted in bad
faith in their failure to transfer Yap to another vessel owned by respondent.
Finally, since the unexpired portion (9 months) was less than one year, Yap is
entitled to his salaries for the 9mos equivalednt to $12,870 plus moral (P100K)
and exemplary (P50K) damages. ----respondent sought recourse from the NLRC
NLRC: Jan. 5, 2005 decision: affirmed with modification that Yap is only entitled
to 3 months provided for under Sec. 10 RA 8041[Migrant Workers and Overseas
Filipino Act]. ---- both parties filed their Motion for Partial Consideration. April
20, 2005 decision: modified January ruling by reinstating LAs award for his
salary equivalent to $12,870.
Respondens MR was denied hence availed of Rule 65 before the CA.

CA: upheld both LA and NLRC that Yap was constructively and illegally
dismissed. However, CA ruled that NLRC erred in sustaining LAs interpretation
of Sec. 10 of RA 8024. CA relied on the clause or for 3 months for every year of

the unexpired term, which ever is less and concluding that the 3 months option
for payment of salary is applicable in the case. CA ordered that Yaps basic salary
be only for 3 mos. Or $4290.
NOTE: while this case was pending before SC, the clause or for 3 months for
every year of the unexpired term, which ever is less was declared
UNCONSTITUTIONAL in the case of Serrano vs Gallant.
ISSUES: Whether or not Yap is entitled to his salary for the 9 months unexpired
period of his employment contract.

HELD: YES, It is important to note that there is unanimous finding that the
dismissal of Yap was illegal and that there is bad faith on the part of respondent
warranting the award of moral and exemplary damages.

We have already declared in Serrano that the clause or for three months
for every year of the unexpired term, whichever is less provided in the 5th
paragraph of Section 10 of R.A. No. 8042 is unconstitutional for being violative of
the rights of Overseas Filipino Workers (OFWs) to equal protection of the laws.

Moreover, this Court held therein that the subject clause does not state
or imply any definitive governmental purpose; hence, the same violates not just
therein petitioners right to equal protection, but also his right to substantive due
process under Section 1, Article III of the ConstitutionThe Court, in Serrano,
pertinently HELD:

The Court concludes that the subject clause contains a suspect classification
in that, in the computation of the monetary benefits of fixed-term employees
who are illegally discharged, it imposes a 3-month cap on the claim of OFWs
with an unexpired portion of one year or more in their contracts, but none on
the claims of other OFWs or local workers with fixed-term employment. The
subject clause singles out one classification of OFWs and burdens it with a
peculiar disadvantage.
As a general rule, an unconstitutional act is not a law; it confers no rights; it
imposes no duties; it affords no protection; it creates no office; it is inoperative as
if it has not been passed at all. The general rule is supported by Article 7 of the
Civil Code, which provides:

3E Andaya Ching Espiritu Hefti Galvez Gammad Lainez Lui Madamba Nagera Narvasa
Ong Palangdao Rosales Sanchez Santos Satrain Tabo (2014-2015)

Labor Relations Case Digest - Atty. Joyrich Golangco

Art. 7. Laws are repealed only by subsequent ones, and their violation or nonobservance shall not be excused by disuse or custom or practice to the contrary.

The doctrine of operative fact serves as an exception to the aforementioned


general rule. In Planters Products, Inc. v. Fertiphil Corporation, we HELD:

The doctrine of operative fact, as an exception to the general rule, only applies as
a matter of equity and fair play. It nullifies the effects of an unconstitutional law
by recognizing that the existence of a statute prior to a determination of
unconstitutionality is an operative fact and may have consequences that cannot
always be ignored. The past cannot always be erased by a new judicial
declaration.
The doctrine is applicable when a declaration of unconstitutionality will impose
an undue burden on those who have relied on the invalid law.

Following Serrano, we hold that this case should not be included in the
aforementioned exception. After all, it was not the fault of petitioner that he lost
his job due to an act of illegal dismissal committed by respondents. To rule
otherwise would be iniquitous to petitioner and other OFWs, and would, in
effect, send a wrong signal that principals/employers and recruitment/manning
agencies may violate an OFWs security of tenure which an employment contract
embodies and actually profit from such violation based on an unconstitutional
provision of law.
A final note.

We ought to be reminded of the plight and sacrifices of our OFWs. In Olarte v.


Nayona, this Court held that:

Our overseas workers belong to a disadvantaged class. Most of them come from
the poorest sector of our society. Their profile shows they live in suffocating
slums, trapped in an environment of crimes. Hardly literate and in ill health, their
only hope lies in jobs they find with difficulty in our country. Their unfortunate
circumstance makes them easy prey to avaricious employers. They will climb
mountains, cross the seas; endure slave treatment in foreign lands just to
survive. Out of despondence, they will work under sub-human conditions and

accept salaries below the minimum. The least we can do is to protect them with
our laws.
130. Bank of Lucbao vs Manabat

DOCTRINE: General Rule: an illegally dismissed employee is entitled to


reinstatement as a matter of right.
Exception: Doctrine of strained relations, the payment of separation pay is
considered an acceptable alternative to reinstatement when the latter option is
no longer desirable or viable.

FACTS: Sometime in 2001, Rommel J. Manabat (respondent) was hired by


petitioner Bank of Lubao, a rural bank, as a Market Collector. He was assigned as
an encoder at the Bank of Lubaos Sta. Cruz Extension Office, which he manned
together with two other employees, teller Susan P. Lingad (Lingad) and May O.
Manasan. As an encoder, the respondents primary duty is to encode the clients
deposits on the banks computer after the same are received by Lingad.
In November 2004, an initial audit on the Bank conducted by the
petitioner revealed that there was a misappropriation of funds in the amount
of P3,000,000.00, more or less. Apparently, there were transactions entered and
posted in the passbooks of the clients but were not entered in the banks book of
accounts. Further audit showed that there were various deposits which were
entered in the banks computer but were subsequently reversed and marked as
error in posting.
On November 17, 2004, the petitioner sent a memorandum asking the
respondent to explain in writing the discrepancies that were discovered during
the audit. The respondent in his letter-explanation asserted that there were
times when Lingad used the banks computer while he was out on errands.

On December 11, 2004, an administrative hearing was conducted by the


banks investigating committee where the respondent was further made to
explain his side. Subsequently, the investigating committee concluded that the
respondent conspired with Lingad in making fraudulent entries disguised as
error corrections in the banks computer.

3E Andaya Ching Espiritu Hefti Galvez Gammad Lainez Lui Madamba Nagera Narvasa
Ong Palangdao Rosales Sanchez Santos Satrain Tabo (2014-2015)

Labor Relations Case Digest - Atty. Joyrich Golangco

On August 9, 2005, the petitioner filed several criminal complaints for


qualified theft against Lingad and the respondent with MTC-Lubao Pampanga,
and thereafter terminated the respondent due to serious misconduct tantamount
to wilful breach of trust effective September 1, 2005. Which led to the filing of a
complaint by the respondent for illegal dismissal with the NLRC arguing that
there was no valid ground for his dismissal since the qualified theft charged
against him was dismissed for lack of sufficient basis that he consipired with
Lingad. The respondent sought an award for separation pay, full backwages,
13thmonth pay for 2004 and moral and exemplary damages.
Petitioner contends that the dismissal was justified based on the
February 14, 2006 Audit report and that the dismissal of the qualified theft
charge is immaterial to the validity of the ground of dismissal

LA: there is Illegal Dismissal since the Feb 14, 2006 Audit report was disregarded
for being unsigned thus ordering the petitioner to reinstate the respondent to his
former position and awarding the latter backwages,13th month pay

The petitioner to the NLRC, submitting a new audit report dated April
30, 2007. Pending appeal, the petitioner sent the respondent a letter dated April
30, 2007 requiring him to report for work on May 4, 2007 pursuant to the
reinstatement order of the LA. The said letter was served to the respondent on
May 3, 2007 but he refused to receive the same.
NLRC: affirmed the LA stating that Lingad was the one responsible for the said
misappropriations, and the February 14, 2006 and April 30, 2007 audit reports
could not be given evidentiary weight as the same were executed after the
respondent had already been dismissed. The respondent filed a petition for
Certiorari before the CA.
CA: denied the petition, However, the CA held that the respondent is entitled to
separation pay equivalent to one-month salary for every year of service in lieu of
reinstatement and back wages to be computed from the time of his illegal
dismissal until the finality of the said decision. since the latter did not pray for
reinstatement before the LA and that the same would be in the best interest of
the parties considering the animosity and antagonism that exist between them.

MR- denied, Hence this petition to the supreme court, Petitioner contends that
the CA erred in awarding separation pay in favor of the respondent in lieu of
reinstatement considering that the appeal before it only involved the issue of the
legality or illegality of the respondents dismissal; and the CA erred in awarding
backwages in favor of the respondent since it acted in good faith when it
terminated the respondents employment.

ISSUES: 1. whether the CA erred in ordering the petitioner to pay the respondent
separation pay in lieu of reinstatement;
(2) whether the respondent is entitled to payment of backwages.

HELD:
1. NO. the Labor Code provides:
Article 279. Security of Tenure. - In cases of regular employment, the
employer shall not terminate the services of an employee except for a just cause
or when authorized by this Title. An employee who is unjustly dismissed from
work shall be entitled to reinstatement without loss of seniority rights and
other privileges and to his full backwages, inclusive of allowances, and to his other
benefits or their monetary equivalent computed from the time his compensation
was withheld from him up to the time of his actual reinstatement. (emphasis
supplied)

Under the law and prevailing jurisprudence, an illegally dismissed


employee is entitled to reinstatement as a matter of right. However, if
reinstatement would only exacerbate the tension and strained relations between
the parties, or where the relationship between the employer and the employee
has
been unduly
strained by
reason
of
their
irreconcilable
differences, particularly where the illegally dismissed employee held a managerial
or key position in the company, it would be more prudent to order payment of
separation pay instead of reinstatement.
Under the doctrine of strained relations, the payment of separation pay is
considered an acceptable alternative to reinstatement when the latter option is
no longer desirable or viable.

3E Andaya Ching Espiritu Hefti Galvez Gammad Lainez Lui Madamba Nagera Narvasa
Ong Palangdao Rosales Sanchez Santos Satrain Tabo (2014-2015)

Labor Relations Case Digest - Atty. Joyrich Golangco

In such cases, it should be proved that the employee concerned occupies


a position where he enjoys the trust and confidence of his employer; and that it is
likely that if reinstated, an atmosphere of antipathy and antagonism may be
generated as to adversely affect the efficiency and productivity of the employee
concerned.
The court agrees that the relationship of the parties are already strained due to
the following:
1. The respondents reinstatement to his former position would only serve to
intensify the atmosphere of antipathy and antagonism between the
parties. Due to the filing of several criminal complaints and the subsequent
filing of Illegal Dismissal, taken together with the pendency of this case for
more than 6 years had caused strained relations between the parties
2. Considering that the respondents former position as bank encoder
involves the handling of accounts of the depositors of the Bank of Lubao, it
would not be equitable on the part of the petitioner to be ordered to
maintain the former in its employ since it may only inspire vindictiveness
on the part of the respondent.
3. The refusal of the respondent to be re-admitted to work is in itself
indicative of the existence of strained relations between him and the
petitioner.

to the time of their actual reinstatement. But if reinstatement is no longer


possible, the backwages shall be computed from the time of their illegal
termination up to the finality of the decision

Second ISSUES: Backwages

FACTS: On different dates in the late 1990s, petitioner hired respondents


Calibod, Laquio, Santander, Saile and Montederamos, as classroom teachers, and
respondent Palacio, as guidance counselor. In separate letters dated March 31,
2000, however, petitioner informed them that their re-application for school
year 2000-2001 could not be accepted because they failed to pass the Licensure
Examination for Teachers (LET). According to petitioner, as non-board passers,
respondents could not continue practicing their teaching profession pursuant to

Time and again, this Court has recognized that strained relations
between the employer and employee is an exception to the rule requiring actual
reinstatement for illegally dismissed employees for the practical reason that the
already existing antagonism will only fester and deteriorate, and will only
worsen with possible adverse effects on the parties, if we shall compel
reinstatement; thus, the use of a viable substitute that protects the interests of
both parties while ensuring that the law is respected.
The argument of Good faith has no merit , However, the backwages that should
be awarded to the respondent should be modified.
Employees who are illegally dismissed are entitled to full backwages,
inclusive of allowances and other benefits or their monetary equivalent,
computed from the time their actual compensation was withheld from them up

Thus, when there is an order of reinstatement, the computation of


backwages shall be reckoned from the time of illegal dismissal up to the time that
the employee is actually reinstated to his former position.

Pursuant to the order of reinstatement rendered by the LA, the


petitioner sent the respondent a letter requiring him to report back to work on
May 4, 2007. Notwithstanding the said letter, the respondent opted not to report
for work. Thus, it is but fair that the backwages that should be awarded to the
respondent be computed from the time that the respondent was illegally
dismissed until the time when he was required to report for work, i.e. from
September 1, 2005 until May 4, 2007. It is only during the said period that the
respondent is deemed to be entitled to the payment of backwages.
131. ST. MARYS ACADEMY VS PALACIO ET AL.

DOCTRINE: Backwages is a remedy affording the employee a way to recover


what he has lost by reason of the unlawful dismissal. In awarding backwages, the
primordial consideration is the income that should have accrued to the employee
from the time that he was dismissed up to his reinstatement and the length of
service prior to his dismissal is definitely inconsequential. However in the case at
bar the Supreme court ordered limited backwages instead of full backwages
because of the respondents only assailed the amount of backwages for the first time in
its motion for reconsideration of the Decision of the CA. Thus, the Court cannot entertain
the issue for being belatedly raised.

3E Andaya Ching Espiritu Hefti Galvez Gammad Lainez Lui Madamba Nagera Narvasa
Ong Palangdao Rosales Sanchez Santos Satrain Tabo (2014-2015)

Labor Relations Case Digest - Atty. Joyrich Golangco

the DECS Memorandum No. 10, S. 1998 which requires incumbent teachers to
register as professional teachers.
Respondents filed a complaint contesting their termination as highly irregular and
premature. They admitted that they are indeed non-board passers, however, they also
argued that their security of tenure could not simply be trampled upon for their failure to
register with the Professional Regulation Commission (PRC) or to pass the LET prior to
the deadline and that their outright dismissal was illegal because some of them possessed
civil service eligibilities and special permits to teach. Furthermore, petitioners retention
and acceptance of other teachers who do not also possess the required eligibility showed
evident bad faith in terminating respondents.
LA: Adjudged petitioner guilty of illegal dismissal because it terminated the
services of the respondents on March 31, 2000 which was clearly prior to the
September 19, 2000 deadline fixed by PRC for the registration of teachers as
professional teachers, in violation of the doctrine regarding the prospective
application of laws. Thus, petitioner was ordered to reinstate the respondents or
to pay them separation pay at the rate of month wage for every year of service,
plus limited backwages covering the period from March 31, 2000 to September
30, 2000.

NLRC: Upheld the LAs decision, stating that the grounds relied upon by the
petitioner or dismiss the respondents are not among those enumerated by the
Labor Code and that the respondents are regular employees, who cannot be
removed without just cause.

CA: Upheld both the decisions of the LA and the NLRC. It further held that the
Petitioner should have adopted a contingency plan if in case the respondents still
have not complied with the aforementioned requirements when the deadline has
arrived. The CA also observed that the petitioners ulterior motive for the
termination may have been the result of a confrontation between the principal
and the respondents. However, as regards to Padilla, Palacio, Andalahao and
Decipulo, the CA found them to be merely probationary; therefore, there is no
illegal dismissal to speak of.
ISSUE(S):

(I)

(II)
HELD:
(I)

(II)

Whether the dismissal of the respondents were premature because


it was effected prior to the deadline set by the PRC to acquire their
license.
Whether the respondents are entitled to backwages from March to
19 September 2000, because it is only on such date that they were
already dismissible for cause.

Yes. The Supreme Court agrees with the decisions of the LA, the
NLRC and the CA. It is incumbent upon the Court to afford full
protection to labor.The law has provided a specific timeframe within
which respondents could comply, petitioner has no right to deny
them of this privilege accorded to them by law. In so far as Palacio,
Calibod, Laquio, Santander and Montedramos are concerned, being
dismissed on March 2000 was premature. However, Saile is not
qualified to take the LET, therefore, no prematurity is to speak of on
her end. Petitioners intention and desire not to put the students
education and school operation in jeopardy is neither a decisive
consideration for respondents termination prior to the deadline set
by law. The prejudice that respondents retention would cause to
the schoold operation is only trivial.
Yes. Petitioner cannot possibly presume that respondents could not timely
comply with the requirements of the law. At any rate, we note that
petitioner only assailed the amount of backwages for the first time in its
motion for reconsideration of the Decision of the CA. Thus, the Court
cannot entertain the issue for being belatedly raised. Hence, the award of
limited backwages covering the period from March 31, 2000 to September
30, 2000 as ruled by the Labor Arbiter and affirmed by both the NLRC and
CA is in order.

132. Toyota Motor Phils Corp Workers Association v. NLRC (2007)

DOCTRINE: Entitlement to separation pay of legally dismissed employees


General Rule: when just causes for terminating the services of an employee under
Art. 282 of the Labor Code exist, the employee is not entitled to separation pay. The
reason is that lawbreakers should not benefit from their illegal acts. The dismissed
employee, however, is entitled to "whatever rights, benefits and privileges [s/he]

3E Andaya Ching Espiritu Hefti Galvez Gammad Lainez Lui Madamba Nagera Narvasa
Ong Palangdao Rosales Sanchez Santos Satrain Tabo (2014-2015)

Labor Relations Case Digest - Atty. Joyrich Golangco

may have under the applicable individual or collective bargaining agreement with
the employer or voluntary employer policy or practice" or under the Labor Code
and other existing laws. Hence, despite the dismissal for a valid cause, the employee
retains the right to receive from the employer benefits provided by law, like
accrued service incentive leaves. With respect to benefits granted by the CBA
provisions and voluntary management policy or practice, the entitlement of the
dismissed employees to the benefits depends on the stipulations of the CBA or the
company rules and policies.
Exception: Separation pay is given even though an employee is validly dismissed
when the court finds justification in applying the principle of social justice well
entrenched in the 1987 Constitution. The categorical mandates in the Constitution
for the improvement of the lot of the workers are more than sufficient basis to
justify the award of separation pay in proper cases even if the dismissal be for
cause. But the courts shall not grant separation pay based on social justice when
the cause of dismissal is serious misconduct (which is the first ground for dismissal
under Art. 282) or acts that reflect on the moral character of the employee. Those
who invoke social justice may do so only if their hands are clean and their motives
blameless and not simply because they happen to be poor. This great policy of our
Constitution is not meant for the protection of those who have proved they are not
worthy of it, like the workers who have tainted the cause of labor with the
blemishes of their own character.
FACTS: Petitioner Union filed a Petition for Certification Election among the
Toyota rank and file employees with the NCMB but was denied. But, on appeal,
the DOLE Secretary granted the Unions petition and directed the immediate
holding of the certification election. The Union was certified as the Exclusive
Bargaining Representative of the employees.

Then, the Union submitted its CBA proposals to Toyota, but the latter refused to
negotiate in view of its pending appeal. Subsequent strikes were held which was
declared illegal by the SC.

Anent the grant of severance compensation to legally dismissed union members,


Toyota assails the turn-around by the CA in granting separation pay in its June
20, 2003 Resolution after initially denying it in its February 27, 2003 Decision.
The company asseverates that based on the CA finding that the illegal acts of said

union members constitute gross misconduct, not to mention the huge losses it
suffered, then the grant of separation pay was not proper
ISSUES:
1.
2.

HELD:

WON the Union committed illegal strike (YES)


WON the 227 striking employees entitled to separation pay (NO)

The protest action undertaken by the union is an illegal strike. The Union
contends that the protests or rallies conducted by them are not within the ambit
of strikes as defined in the Labor Code, since they were legitimate exercises of
their right to peaceably assemble and petition the government for redress of
grievances. The fact that the conventional term strike was not used by the
striking employees to describe their common course of action is inconsequential,
since the substance of the situation and not its appearance, will be deemed
controlling. In this case, the protest actions undertaken by the Union officials and
members are not valid and proper exercises of their right to assemble and ask
government for redress of their complaints, especially when they didnt obtain
permit from the City of Manila to hold rallies. Their act constitutes temporary
stoppages of work perpetrated through the concerted action of the employees
who deliberately failed to report for work on the convenient excuse that they will
hold a rally at the BLR and DOLE offices in Intramuros, Manila. Their decision
not to work for two days was designed and calculated to cripple the
manufacturing arm of Toyota and to coerce Toyota to finally acknowledge the
Union as the sole bargaining agent of the company. This is not a legal and valid
exercise of the right of assembly and to demand redress of grievance. Thus,
without the requirements for a valid strike having been complied with, the strike
staged was illegal and all employees who participated in the concerted actions to
have consequently lost their employment status.
The 227 striking employees are not entitled to separation pay due to
serious misconduct. The advertence to the alleged honest belief on the part of
the 227 employees that Toyota committed a breach of the duty to bargain
collectively and an abuse of valid exercise of management prerogative has not
been substantiated by the evidence on record. There can be no good faith in
intentionally incurring absences in a collective fashion from work just to attend

3E Andaya Ching Espiritu Hefti Galvez Gammad Lainez Lui Madamba Nagera Narvasa
Ong Palangdao Rosales Sanchez Santos Satrain Tabo (2014-2015)

Labor Relations Case Digest - Atty. Joyrich Golangco

the DOLE hearings. The Unions strategy was plainly to cripple the operations
and bring Toyota to its knees by inflicting substantial financial damage to the
latter to compel union recognition. The Union officials and members are
supposed to know through common sense that huge losses would befall the
company by the abandonment of their regular work. Toyota lost more than 50M
because of the willful desertion of company operations by the dismissed union
members. In addition, further damage was experienced by Toyota when the
Union again resorted to illegal strikes, when the gates of Toyota were blocked
and barricaded, and the company officials, employees, and customers were
intimidated and harassed. Moreover, they were fully aware of the company rule
on prohibition against concerted action inimical to the interests of the company
and hence, their resort to mass actions on several occasions in clear violation of
the company regulation cannot be excused nor justified. Lastly, they blatantly
violated the assumption/certification Order of the DOLE Secretary, exhibiting
their lack of obeisance to the rule of law. These acts indeed constituted serious
misconduct.
133. BRISTOL MYERS SQUIBB (PHILS.), INC., petitioner, vs. RICHARD NIXON
A. BABAN, respondent.

DOCTRINES: A MEDICAL representative should distribute his employer's


products per company directions or risk termination. The willful breach of the
trust reposed in him by his employer is a cause for the termination of his
employment.

While the State can regulate the right of an employer to select and discharge his
employees, an employer cannot be compelled to continue the employment of an
employee in whom there has been a legitimate loss of trust and confidence.
Since respondent was validly dismissed for a cause other than serious
misconduct or those that negatively reflect on his moral character, the award of
separation pay is justifiable

FACTS: Respondent was hired as petitioners district manager, assigned to


handle the company's clients in Cagayan de Oro-Northern Mindanao area and its
immediate vicinities. His duties included the promotion of nutritional products of
petitioner to medical practitioners, sale to drug outlets and the supervision of
territory managers detailed in his district.

While conducting a field audit in Mindanao, petitioner's auditor, Sheela Torreja,


found (20) packs of "Mamacare" samples in the baggage compartment of a
company car with an accompanying note with political overtones. Apparently,
respondent's father was thanking supporters in the election through distribution
of company sample products.

Upon report of said incident, the company's Medical Sales Director, Ferdinand
Sarfati, issued a Memorandum requiring respondent to explain in writing why he
should not be terminated for the infraction. Respondent admitted that he had
caused the attachment of the notes to the product samples but argued that there
was no unauthorized distribution of the samples since he intended to give them
only to doctors who requested them. To support his claim, he asserted that the
samples found by Ms. Torreja were actually to be given to Dr. Kibtiya Gustahan
and to Rosita Jacoba, a registered midwife, for distribution to the health center.
He admitted that he committed an honest mistake, an irresponsible act to have
succumbed to the suggestion of Dr. Gustahan. He pleaded for consideration for
the lapse, insisting that he has not caused any damage nor injury to the image of
the company as the samples were not, in fact, distributed and that no gain was
derived by him or his family.
Private conferences were held and he was given the chance to submit evidence
and to be assisted by counsel during the conference. Subsequently, he received
under protest the company's memorandum dismissing him from employment.

Respondent filed a complaint for illegal dismissal with a claim for moral and
exemplary damages plus attorney's fees with the Regional Arbitration Branch
No. 10 of the (NLRC) against petitioner Likewise impleaded were the company's
General Manager, Medical Sales Director, HR Director, Personnel Manager,
Auditor and Finance Director.
LA: dismissed complaint but ordered to pay monetary liabilities

NLRC: modified the LAs decision: affirmed the monetary awards, but declared
that dismissal was illegal, ordered the suspension of Baban for 1 month without
pay, ordered the company to reinstate Baban, ordered the payment of moral
damages, exemplary damages and attys fees in favor of Baban

3E Andaya Ching Espiritu Hefti Galvez Gammad Lainez Lui Madamba Nagera Narvasa
Ong Palangdao Rosales Sanchez Santos Satrain Tabo (2014-2015)

Labor Relations Case Digest - Atty. Joyrich Golangco

Upon MR, NLRC modified and set aside its findings: reinstating the LA decision
that termination was valid, affirming award of monetary liabilities, ordering
award of financial assistance in favor of Baban by way of separation pay

CA: reinstated the original NLRC decision, holding that the right of a worker to
security of tenure is constitutionally guaranteed, and finding that the penalty of
dismissal was unjustified, much too harsh and not commensurate with the
alleged infraction
ISSUES: WON respondent was validly dismissed

HELD: YES. Article 282(c) of the Labor Code allows an employer to terminate the
services of an employee for loss of trust and confidence. The right of employers
to dismiss employees by reason of loss of trust and confidence is well established
in jurisprudence.
The first requisite for dismissal on the ground of loss of trust and confidence is
that the employee concerned must be one holding a position of trust and
confidence.

There are two (2) classes of positions of trust: a. Managerial employeesthose


vested with the powers or prerogatives to lay down management policies and to
hire, transfer suspend, lay-off, recall, discharge, assign or discipline employees or
effectively recommend such managerial actions. b. Cashiers, auditors, property
custodians, etcthose who in the normal and routine exercise of their functions,
regularly handle significant amounts of money or property. In this case, since
respondent handles large amounts of petitioner's samples, he is, by law, an
employee with a position of trust, falling under the second class.
The second requisite is that there must be an act that would justify the loss of
trust and confidence.19 Loss of trust and confidence to be a valid cause for
dismissal must be based on a willful breach of trust and founded on clearly
established facts. The basis for the dismissal must be clearly and convincingly
established but proof beyond reasonable doubt is not necessary.20

Respondent's act of stapling a thank you note from his father warrants the loss of
petitioner's trust and confidence. As the supervisor of fellow medical

representatives, he had the duty to set a good example to his colleagues. A higher
standard of confidence was reposed in him.
There is no doubt that respondent willfully breached the trust and confidence
reposed in him by not asking for permission before using company property for
his own or another's benefit, as required in the Company Standards of Business
Conduct.21 Moreover, when respondent failed to turn over the samples left in his
care and stapled the political "thank you" note with the intention of distributing
them to his father's supporters, he had, in effect appropriated company property
for personal gain and benefit.

The CA apparently granted his plea for mercy when it ruled that his action while
censurable did not merit termination. The CA characterized his action as a mere
lapse of human frailty considering the elections were over and that the stapling
of the thank you notes did not give rise to any undue advantage to respondent or
his father. The CA anchored its leniency on Caltex Refinery Employees Association
(CREA) v. National Labor Relations Commission (Third Division).
The CA reliance on Caltex is misplaced. A closer examination of the two cases
reveals that the facts are different. The only similarity is that both respondent
and Clarete had no prior violations. However, unlike Clarete, respondent
qualifies as a confidential employee. It bears emphasis that there is a well-settled
distinction between the treatment of a confidential employee and rank-and-file
personnel, insofar as the application of the doctrine of trust and confidence is
concerned.28 There was also no finding that the value of the goods was minimal
compared to respondent's salary. Another glaring difference between the two
cases is that respondent had people under his supervision and he engaged them
to help commit his infraction.29

The two requisites for dismissal for loss of trust and confidence having been met,
petitioner is well within its rights to dismiss respondent. While the State can
regulate the right of an employer to select and discharge his employees, an
employer cannot be compelled to continue the employment of an employee in
whom there has been a legitimate loss of trust and confidence.30

3E Andaya Ching Espiritu Hefti Galvez Gammad Lainez Lui Madamba Nagera Narvasa
Ong Palangdao Rosales Sanchez Santos Satrain Tabo (2014-2015)

Labor Relations Case Digest - Atty. Joyrich Golangco

However, while We find that the dismissal is valid, We are not deaf to
respondent's plea for mercy. In a line of cases We have held that separation pay
may be awarded as some equitable relief in consideration of the past services
rendered.32 Since respondent was validly dismissed for a cause other than
serious misconduct or those that negatively reflect on his moral character,33 the
award of separation pay is justifiable. This award is merely to coat the bitter
termination experienced by respondent with a little social justice.34 Separation
pay at the rate of one month salary for every year of service is proper.

134. ARMANDO G. YRASUEGUI vs. PHILIPPINE AIRLINES, INC.


DOCTRINE: Qualifying standards are norms that apply prior to and after an
employee is hired. They apply prior to employment because these are the
standards a job applicant must initially meet in order to be hired. They apply
after hiring because an employee must continue to meet these standards while
on the job in order to keep his job. Under this perspective, a violation is not one
of the faults for which an employee can be dismissed pursuant to pars. (a) to (d)
of Article 282; the employee can be dismissed simply because he no longer
qualifies for his job irrespective of whether or not the failure to qualify was
willful or intentional.

FACTS: Petitioner Armando G. Yrasuegui was a former international flight


steward of Philippine Airlines, Inc. (PAL). He stands five feet and eight inches
(58) with a large body frame. The proper weight for a man of his height and
body structure is from 147 to 166 pounds, the ideal weight being 166 pounds, as
mandated by the Cabin and Crew Administration Manual of PAL.

In 1984, due to a weight problem, PAL advised him to go on an extended vacation


leave to address his weight concerns. However, petitioner failed to meet the
companys weight standards, prompting another leave without pay. After
meeting the required weight, petitioner was allowed to return to work. But
petitioners weight problem recurred. He again went on leave without pay. On
April 26, 1989, petitioner weighed 209 pounds, 43 pounds over his ideal weight.
In line with company policy, he was removed from flight duty and was formally
requested to trim down to his ideal weight and report for weight checks on
several dates. He was also told that he may avail of the services of the company
physician should he wish to do so.

On February 25, 1989, petitioner underwent weight check. It was discovered


that he gained, instead of losing, weight. He was overweight at 215 pounds,
which is 49 pounds beyond the limit. Consequently, his off-duty status was
retained. Despite the lapse of a ninety-day period given him to reach his ideal
weight, petitioner remained overweight. Again, he was directed to report every
two weeks for weight checks. Petitioner failed to report for weight checks.
Despite that, he was given one more month to comply with the weight
requirement. As usual, he was asked to report for weight check on different
dates. He was reminded that his grounding would continue pending satisfactory
compliance with the weight standards. Again, petitioner failed to report for
weight checks. On April 17, 1990, petitioner was formally warned that a repeated
refusal to report for weight check would be dealt with accordingly. He was given
another set of weight check dates. Petitioner ignored the directive. On June 26,
1990, petitioner was required to explain his refusal to undergo weight checks.
When petitioner tipped the scale on July 30, 1990, he weighed at 212 pounds.
Clearly, he was still way over his ideal weight of 166 pounds. From then on,
nothing was heard from petitioner until he followed up his case requesting for
leniency on the latter part of 1992. He weighed at 219 pounds on August 20,
1992 and 205 pounds on November 5, 1992.
On November 13, 1992, PAL finally served petitioner a Notice of Administrative
Charge for violation of company standards on weight requirements. On June 15,
1993, petitioner was formally informed by PAL that due to his inability to attain
his ideal weight, and considering the utmost leniency extended to him which
spanned a period covering a total of almost five (5) years, his services were
considered terminated effective immediately.
ISSUES: Whether Petiotioner was illegally dismissed
HELD: No.

I. The obesity of petitioner is a ground for dismissal under Article 282(e) [44] of the
Labor Code.
A reading of the weight standards of PAL would lead to no other conclusion
than that they constitute a continuing qualification of an employee in order to
keep the job. Tersely put, an employee may be dismissed the moment he is
unable to comply with his ideal weight as prescribed by the weight standards.

3E Andaya Ching Espiritu Hefti Galvez Gammad Lainez Lui Madamba Nagera Narvasa
Ong Palangdao Rosales Sanchez Santos Satrain Tabo (2014-2015)

Labor Relations Case Digest - Atty. Joyrich Golangco

The dismissal of the employee would thus fall under Article 282(e) of the Labor
Code.
As explained by the CA:

x x x [T]he standards violated in this case were not mere


orders of the employer; they were the prescribed weights
that a cabin crew must maintain in order to qualify for and keep
his or her position in the company. In other words, they were
standards that establish continuing qualifications for an
employees position. In this sense, the failure to maintain these
standards does not fall under Article 282(a) whose express
terms require the element of willfulness in order to be a ground
for dismissal. The failure to meet the employers qualifying
standards is in fact a ground that does not squarely fall under
grounds (a) to (d) and is therefore one that falls under Article
282(e) the other causes analogous to the foregoing.

By its nature, these qualifying standards are norms that


apply prior to and after an employee is hired. They apply prior
to employment because these are the standards a job applicant
must initially meet in order to be hired. They apply after hiring
because an employee must continue to meet these standards
while on the job in order to keep his job. Under this
perspective, a violation is not one of the faults for which an
employee can be dismissed pursuant to pars. (a) to (d) of Article
282; the employee can be dismissed simply because he no
longer qualifies for his job irrespective of whether or not the
failure to qualify was willful or intentional. x x x[

In fine, We hold that the obesity of petitioner, when placed in the context of his
work as flight attendant, becomes an analogous cause under Article 282(e) of the
Labor Code that justifies his dismissal from the service. His obesity may not be
unintended, but is nonetheless voluntary. As the CA correctly puts it,
[v]oluntariness basically means that the just cause is solely attributable to the
employee without any external force influencing or controlling his actions. This
element runs through all just causes under Article 282, whether they be in the

nature of a wrongful action or omission. Gross and habitual neglect, a recognized


just cause, is considered voluntary although it lacks the element of intent found
in Article 282(a), (c), and (d).
II. The dismissal of petitioner can be predicated on the bona fide occupational
qualification defense.

Employment in particular jobs may not be limited to persons of a particular


sex, religion, or national origin unless the employer can show that sex, religion,
or national origin is an actual qualification for performing the job. The
qualification is called a bona fide occupational qualification (BFOQ).

The Supreme Court of Canada adopted the so-called Meiorin Test in


determining whether an employment policy is justified. Under this test, (1) the
employer must show that it adopted the standard for a purpose rationally
connected to the performance of the job; (2) the employer must establish that
the standard is reasonably necessary to the accomplishment of that work-related
purpose; and (3) the employer must establish that the standard is reasonably
necessary in order to accomplish the legitimate work-related purpose. Similarly,
in Star Paper Corporation v. Simbol, this Court held that in order to justify a
BFOQ, the employer must prove that (1) the employment qualification is
reasonably related to the essential operation of the job involved; and (2) that
there is factual basis for believing that all or substantially all persons meeting the
qualification would be unable to properly perform the duties of the job.
In short, the test of reasonableness of the company policy is used because it is
parallel to BFOQ. BFOQ is valid provided it reflects an inherent quality
reasonably necessary for satisfactory job performance.

The weight standards of PAL are reasonable. A common carrier, from the nature
of its business and for reasons of public policy, is bound to observe extraordinary
diligence for the safety of the passengers it transports. The primary objective of
PAL in the imposition of the weight standards for cabin crew is flight safety. It
cannot be gainsaid that cabin attendants must maintain agility at all times in
order to inspire passenger confidence on their ability to care for the passengers
when something goes wrong. It is not farfetched to say that airline companies,
just like all common carriers, thrive due to public confidence on their safety
records.

3E Andaya Ching Espiritu Hefti Galvez Gammad Lainez Lui Madamba Nagera Narvasa
Ong Palangdao Rosales Sanchez Santos Satrain Tabo (2014-2015)

Labor Relations Case Digest - Atty. Joyrich Golangco

135. DREAMLAND HOTEL RESORT and WESTLEY J. PRENTICE, Petitioners, vs.


STEPHEN B. JOHNSON, Respondent.

FACTS: Dreamland is a corporation duly registered with the Securities and


Exchange Commission. Prentice is its current President and Chief Executive
Officer. It is engaged in the hotel, restaurant and allied businesses.

Respondent Stephen B. Johnson is an Australian citizen who came to the


Philippines as a businessman/investor without the authority to be employed as
the employee/officer of any business as he was not able to secure his Alien
Employment Permit

As a fellow Australian citizen, Johnson was able to convince Prentice to accept his
offer to invest in Dreamland and at the same time provide his services as
Operations Manager of Dreamland with a promise that he will secure an AEP and
Tax Identification Number ["TIN" for brevity] prior to his assumption of work.
Sometime on June 21, 2007, Prentice and Johnson entered into an Employment
Agreement, which stipulates among others, that the [sic] Johnson shall serve as
Operations Manager of Dreamland from August 1, 2007 and shall serve as such
for a period of three (3) years.

Before entering into the said agreement [,] Prentice required the submission of
the AEP and TIN from Johnson. Johnson promised that the same shall be supplied
within one (1) month from the signing of the contract because the application for
the TIN and AEP were still under process. Thus[,] it was agreed that the efficacy
of the said agreement shall begin after one (1) month or on August 1, 2007. x x x.
On or about October 8, 2007, Prentice asked on several occasions the production
of the AEP and TIN from Johnson. Johnson gave excuses and promised that he is
already in possession of the requirements. Believing the word of Johnson,
Dreamland commenced a dry run of its operations.
Johnson worked as a hotel and resort Operations Manager only at that time. He
worked for only about three (3) weeks until he suddenly abandoned his work
and subsequently resigned as Operations Manager starting November 3, 2007.

He never reported back to work despite several attempts of Prentice to clarify his
issues. x x x.4
On the other hand, respondent Stephen B. Johnson (Johnson) averred that:

There is also no truth to the allegation that it was [Johnson] who "offered" and
"convinced" petitioner Prentice to "invest" in and provide his services to
petitioner Dreamland Hotel Resort x x x. The truth of the matter is that it was
petitioners who actively advertised for a resort manager for Dreamland Hotel. x x
x
It was in response to these advertisements that private respondent Johnson
contacted petitioners to inquire on the terms for employment offered. It was
Prentice who offered employment and convinced Johnson to give out a loan,
purportedly so the resort can be completed and operational by August 2007.
Believing the representations of petitioner Prentice, private respondent Johnson
accepted the employment as Resort Manager and loaned money to petitioners
[consisting of] his retirement pay in the amount of One Hundred Thousand US
Dollars (USD 100,000.00) to finish construction of the resort.
As [Johnson] remained unpaid since August 2007 and he has loaned all his
money to petitioners, he asked for his salary after the resort was opened in
October 2007 but the same was not given to him by petitioners

However, in an SMS or text message sent by Prentice to [Johnson] on the same


day at around 8:20 pm, he was informed that " I consider [yo]ur resignation as
immediate". Despite demand, petitioners refused to pay [Johnson] the salaries
and benefits due him.5
On January 31, 2008, Johnson filed a Complaint for illegal dismissal and nonpayment of salaries, among others, against the petitioners.

LA: rendered a Decision6 dismissing Johnsons complaint for lack of merit with
the finding that he voluntarily resigned from his employment and was not
illegally dismissed.

3E Andaya Ching Espiritu Hefti Galvez Gammad Lainez Lui Madamba Nagera Narvasa
Ong Palangdao Rosales Sanchez Santos Satrain Tabo (2014-2015)

Labor Relations Case Digest - Atty. Joyrich Golangco

Dissatisfied, Johnson appealed to the National Labor Relations Commission


(NLRC). The NLRC reversed the LA decision

CA dismissed the petition for lack of proof of authority and affidavit of service of
filing as required by Section 13 of the 1997 Rules of Procedure.
ISSUES: WON Johnson is illegally dismissed

HELD: Yes, The resignation of Johnson,was due to the petitioners acts that he
was constrained to resign. The petitioners cannot expect Johnson to tolerate
working for them without any compensation.

Since Johnson was constructively dismissed, he was illegally dismissed. As to the


reliefs granted to an employee who is illegally dismissed, Golden Ace Builders v.
Talde37 referring to Macasero v. Southern Industrial Gases Philippines38 is
instructive:
Thus, an illegally dismissed employee is entitled to two reliefs: backwages and
reinstatement. The two reliefs provided are separate and distinct. In instances
where reinstatement is no longer feasible because of strained relations between
the employee and the employer, separation pay is granted. In effect, an illegally
dismissed employee is entitled to either reinstatement, if viable, or separation
pay if reinstatement is no longer viable, and backwages.
The normal consequences of respondents illegal dismissal, then, are
reinstatement without loss of seniority rights, and payment of backwages
computed from the time compensation was withheld up to the date of actual
reinstatement. Where reinstatement is no longer viable as an option, separation
pay equivalent to one (1) month salary for every year of service should be
awarded as an alternative. The payment of separation pay is in addition to
payment of backwages.
136. Manila Water v. Del Rosario

DOCTRINE: As a general rule, an employee who has been dismissed for any of
the just causes enumerated under Article 282 of the Labor Code is not entitled to
a separation pay.

In exceptional cases, however, the Court has granted separation pay to a legally
dismissed employee as an act of "social justice" or on "equitable grounds." In
both instances, it is required that the dismissal (1) was not for serious
misconduct; and (2) did not reflect on the moral character of the employee.
FACTS: On 22 October 1979, Del Rosario was employed as Instrument
Technician by Metropolitan Waterworks and Sewerage System (MWSS).
Sometime in 1996, MWSS was reorganized pursuant to Republic Act No. 8041 or
the National Water Crisis Act of 1995, and its implementing guidelines
Executive Order No. 286. Because of the reorganization, Manila Water absorbed
some employees of MWSS including Del Rosario. On 1 August 1997, Del Rosario
officially became an employee of Manila Water.

Sometime in May 2000, Manila Water discovered that 24 water meters were
missing in its stockroom. Upon initial investigation, it appeared that Del Rosario
and his co-employee, a certain Danilo Manguera, were involved in the pilferage
and the sale of water meters to the companys contractor. Consequently, Manila
Water issued a Memorandum dated 23 June 2000, directing Del Rosario to
explain in writing within 72 hours why he should not be dealt with
administratively for the loss of the said water meters.4 In his letterexplanation,5 Del Rosario confessed his involvement in the act charged and
pleaded for forgiveness, promising not to commit similar acts in the future.
On 29 June 2000, Manila Water conducted a hearing to afford Del Rosario the
opportunity to personally defend himself and to explain and clarify his defenses
to the charge against him. During the formal investigation Del Rosario was found
responsible for the loss of the water meters and therefore liable for violating
Section 11.1 of the Companys Code of Conduct.6 Manila Water proceeded to
dismiss Del Rosario from employment on 3 July 2000.
This prompted Del Rosario to file an action for illegal dismissal claiming that his
severance from employment is without just cause. In his Position Paper
submitted before the labor officer, Del Rosario averred that his admission to the

3E Andaya Ching Espiritu Hefti Galvez Gammad Lainez Lui Madamba Nagera Narvasa
Ong Palangdao Rosales Sanchez Santos Satrain Tabo (2014-2015)

Labor Relations Case Digest - Atty. Joyrich Golangco

misconduct charged was not voluntary but was coerced by the company. Such
admission therefore, made without the assistance of a counsel, could not be
made basis in terminating his employment. The company invited the attention of
this Court to the fact that Del Rosario himself confessed his involvement to the
loss of the water meters not only in his letter-explanation, but also during the
formal investigation, and in both instances, pleaded for his employers
forgiveness.

LA: After weighing the positions taken by the opposing parties, including the
evidence adduced in support of their respective cases, the Labor Arbiter issued a
Decision9 dated 30 May 2002 dismissing for lack of merit the complaint filed by
Del Rosario who was, however, awarded separation pay. According to the Labor
Arbiter, Del Rosarios length of service for 21 years, without previous derogatory
record, warrants the award of separation pay.
NLRC: the NLRC dismissed the appeal interposed by Manila Water for its failure
to append a certification against forum shopping in its Memorandum of Appeal.

CA: reversed the NLRC Resolution and held that it committed a grave abuse of
discretion when it dismissed Manila Waters appeal on mere technicality. The
appellate court, however, proceeded to affirm the decision of the Labor Arbiter
awarding separation pay to Del Rosario. Considering that Del Rosario rendered
21 years of service to the company without previous derogatory record, the
appellate court considered the granting of separation pay by the labor officer
justified.
ISSUES: Whether or not separation pay shall be allowed as a measure of social
justice.

HELD: We hold that henceforth separation pay shall be allowed as a measure of


social justice only in those instances where the employee is validly dismissed for
causes other than serious misconduct or those reflecting on his moral character.
Where the reason for the valid dismissal is, for example, habitual intoxication or
an offense involving moral turpitude, like theft or illicit sexual relations with a
fellow worker, the employer may not be required to give the dismissed employee
separation pay, or financial assistance, or whatever other name it is called, on the
ground of social justice.

A contrary rule would, as the petitioner correctly argues, have the effect, of
rewarding rather than punishing the erring employee for his offense. And we do
not agree that the punishment is his dismissal only and that the separation pay
has nothing to do with the wrong he has committed. Of course it has. Indeed, if
the employee who steals from the company is granted separation pay even as he
is validly dismissed, it is not unlikely that he will commit a similar offense in his
next employment because he thinks he can expect a like leniency if he is again
found out. This kind of misplaced compassion is not going to do labor in general
any good as it will encourage the infiltration of its ranks by those who do not
deserve the protection and concern of the Constitution.

The policy of social justice is not intended to countenance wrongdoing simply


because it is committed by the underprivileged. At best, it may mitigate the
penalty but it certainly will not condone the offense. Compassion for the poor is
an imperative of every humane society but only when the recipient is not a rascal
claiming an undeserved privilege. Social justice cannot be permitted to be refuge
of scoundrels any more than can equity be an impediment to the punishment of
the guilty. Those who invoke social justice may do so only if their hands are clean
and their motives blameless and not simply because they happen to be poor. This
great policy of our Constitution is not meant for the protection of those who have
proved they are not worthy of it, like the workers who have tainted the cause of
labor with the blemishes of their own character.27
In the subsequent case of Toyota Motor Phils. Corp. Workers Association
(TMPCWA) v. National Labor Relations Commission,28 we expanded the
exclusions and elucidated that separation pay shall be allowed as a measure of
social justice only in instances where the employee is validly dismissed for
causes other than serious misconduct, willful disobedience, gross and habitual
neglect of duty, fraud or willful breach of trust, commission of a crime against the
employer or his family, or those reflecting on his moral character. In the same
case, we instructed the labor officials that they must be most judicious and
circumspect in awarding separation pay or financial assistance as the
constitutional policy to provide full protection to labor is not meant to be an
instrument to oppress the employers.29 The commitment of the court to the
cause of the labor should not embarrass us from sustaining the employers when
they are right, as here. In fine, we should be more cautious in awarding financial

3E Andaya Ching Espiritu Hefti Galvez Gammad Lainez Lui Madamba Nagera Narvasa
Ong Palangdao Rosales Sanchez Santos Satrain Tabo (2014-2015)

Labor Relations Case Digest - Atty. Joyrich Golangco

assistance to the undeserving and those who are unworthy of liberality of the
law.

Indubitably, the appellate court erred in awarding separation pay to Del Rosario
without taking into consideration that the transgression he committed
constitutes a serious offense. The grant of separation pay to a dismissed
employee is determined by the cause of the dismissal. The years of service may
determine how much separation pay may be awarded. It is, however, not the
reason why such pay should be granted at all.
137: Nacar vs Gallery Frames

DOCTRINE: Interest rates In the absence of an express stipulation as to the


rate of interest that would govern the parties, the rate of legal interest for loads
or forbearance of any money, goods or credits and the rate allowed in judgments
shall no longer be 12% per annum but will now be 6% per annum effective
July 1 2013.
FACTS: Dario Nacar filed a labor case against Gallery Frames and its owner
Felipe Bordey, Jr. Nacar alleged that he was dismissed without cause by Gallery
Frames on January 24, 1997. On October 15, 1998, the Labor Arbiter (LA) found
Gallery Frames guilty of illegal dismissal hence the Arbiter awarded Nacar
P158,919.92 in damages consisting of backwages and separation pay.

Gallery Frames appealed all the way to the Supreme Court (SC). The
Supreme Court affirmed the decision of the Labor Arbiter and the decision
became final on May 27, 2002.

After the finality of the SC decision, Nacar filed a motion before the LA
for recomputation as he alleged that his backwages should be computed from the
time of his illegal dismissal (January 24, 1997) until the finality of the SC decision
(May 27, 2002) with interest. The LA denied the motion as he ruled that the
reckoning point of the computation should only be from the time Nacar was
illegally dismissed )January 24, 1997) until the decision of the LA (October 15,
1998). The LA reasoned that the said date should be the reckoning point because
Nacar did not appeal hence as to him, that decision became final and executory.

ISSUES: Whether the Labor Arbiter is correct.


HELD: No. There are two parts of a decision when it comes to illegal dismissal
cases (referring to cases where the dismissed employee wins, or loses but wins
on appeal). The first part is the ruling that the employee was illegally dismissed.
This is immediately final even if the employer appeals but will be reversed if
employer wins on appeal. The second part is the ruling on the award of
backwages and/or separation pay. For backwages, it will be computed from
the date of illegal dismissal until the date of the decision of the Labor
Arbiter. But if the employer appeals, then the end date shall be extended
until the day when the appellate courts decision shall become final. Hence,
as a consequence, the liability of the employer, if he loses on appeal, will increase
this is just but a risk that the employer cannot avoid when it continued to seek
recourses against the Labor Arbiters decision. This is also in accordance with
Article 279 of the Labor Code.
Anent the issue of award interest in the form of actual or compensatory
damages, the Supreme Court ruled that the old case of Eastern Shipping Lines vs
CA is already modified by the promulgation of the Bangko Sentral ng Pilipinas
Monetary Board Resolution No. 796 which lowered the legal rate of interest from
12% to 6%. Specifically, the rules on interest are now as follows:
1. Monetary Obligations ex. Loans:
a. If stipulated in writing:
a.1. shall run from date of judicial demand (filing of the
case)
a.2. rate of interest shall be that amount stipulated
b. If not stipulated in writing
b.1. shall run from date of default (either failure to pay
upon extra-judicial demand or upon judicial demand
whichever is appropriate and subject to the provisions
of Article 1169 of the Civil Code)
b.2. rate of interest shall be 6% per annum
2. Non-Monetary Obligations (such as the case at bar)
a. If already liquidated, rate of interest shall be 6% per annum,
demandable from date of judicial or extra-judicial demand (Art.
1169, Civil Code)
b. If unliquidated, no interest

3E Andaya Ching Espiritu Hefti Galvez Gammad Lainez Lui Madamba Nagera Narvasa
Ong Palangdao Rosales Sanchez Santos Satrain Tabo (2014-2015)

Labor Relations Case Digest - Atty. Joyrich Golangco

Except: When later on established with certainty.


Interest shall still be 6% per annum demandable from
the date of judgment because such on such date, it is
already deemed that the amount of damages is already
ascertained.
3. Compounded Interest
- This is applicable to both monetary and non-monetary
obligations
- 6% per annum computed against award of damages (interest)
granted by the court. To be computed from the date when the courts
decision becomes final and executory until the award is fully satisfied by
the losing party.
4. The 6% per annum rate of legal interest shall be applied
prospectively:
- Final and executory judgments awarding damages prior to
July 1, 2013 shall apply the 12% rate;
- Final and executory judgments awarding damages on or after
July 1, 2013 shall apply the 12% rate for unpaid obligations
until June 30, 2013; unpaid obligations with respect to said
judgments on or after July 1, 2013 shall still incur the 6% rate.
138. Bani Rural Bank vs De Guzman

FACTS: The respondents were employees of Bani Rural Bank, Inc. and ENOC
Theatre I and II who filed a complaint for illegal dismissal against the petitioners.
The complaint was initially dismissed by LA de Guzman. On appeal, NLRC
reversed LAs decision, and ruled that the respondents had been illegally
dismissed. In a resolution dated March 17, 1995 the NLRC ordered the
petitioners to reinstate the complainants with backwages from the time of
their dismissal until their actual reinstatement.
The parties did not file any motion for reconsideration or appeal. The
March 17, 1995 resolution of the NLRC became final and executory and the
computation of the awards was remanded to the LA for execution purposes.

The computation of the respondents' backwages, under the terms of the March
17 1995 NLRC resolution was remanded to LA Gambito. He fixed the period of
backwages from the respondents' illegal dismissal until August 25 1995 or
the date when the respondents allegedly manifested that they no longer
wanted to be reinstated.
The respondents appealed LA Gambito's computation with the NLRC. In a
Decision dated July 31, 1998, the NLRC modified the terms of the March 17,
1995 resolution. NLRC additionally awarded the payment of separation pay,
in lieu of reinstatement. The NLRC justified the award of separation pay on
account of the strained relations between the parties.
On January 29, 1999, the July 31, 1998 decision of the NLRC lapsed to
finality and became executory.

The recomputation of the monetary awards of the respondents' backwages and


separation pay, according to the decision dated July 31, 1998 was referred to LA
Gambito. In the course of the recomputation, the petitioners filed before LA
Gambito a Motion to Quash Writ of Execution and Suspend Further Execution
they reiterated their position that the respondents backwages should be
computed only up to August 25, 1995, citing the alleged manifestation made by
the respondents, through Samuel de la Cruz, their representative, that the
respondents are not interested on being reinstated, but only in the monetary
award;
LA Gambito computed the respondents backwages only up to August 25, 1995.
Respondent appealed the decision of LA Gambito. NLRC ruled that the
computation of the respondents backwages should be until January 29 1999
which was the date when the July 31, 1998 decision attained finality, which was
affirmed by the CA.
ISSUES: Whether or not the respondents backwages should be computed up to
January 29, 1999 or when the July 31, 1998 decision attained finality?
HELD: YES. Article 279 of the Labor Code, as amended, provides backwages and
reinstatement as basic awards and consequences of illegal dismissal.

3E Andaya Ching Espiritu Hefti Galvez Gammad Lainez Lui Madamba Nagera Narvasa
Ong Palangdao Rosales Sanchez Santos Satrain Tabo (2014-2015)

Labor Relations Case Digest - Atty. Joyrich Golangco

"By jurisprudence derived from this provision, separation pay may [also] be
awarded to an illegally dismissed employee in lieu of reinstatement. Section 4(b),
Rule I of the Rules Implementing Book VI of the Labor Code provides the
following instances when the award of separation pay, in lieu of reinstatement to
an illegally dismissed employee, is proper: (a) when reinstatement is no longer
possible, in cases where the dismissed employee s position is no longer available;
(b) the continued relationship between the employer and the employee is no
longer viable due to the strained relations between them; and (c) when the
dismissed employee opted not to be reinstated, or the payment of se aration
benefits would be for the best interest of the parties involved. In these instances,
separation pay is the alternative remedy to reinstatement in addition to the
award of backwages. The payment of separation pay and reinstatement are
exclusive remedies. The payment of separation pay replaces the legal
consequences of reinstatement to an employee who was illegally dismissed.
For clarity, the bases for computing separation pay and backwages are different.
[U]nder Article 279 of the Labor Code and as held in a catena of cases, an
employee who is dismissed without just cause and without due process is
entitled to backwages and reinstatement or payment of separation pay in lieu
thereof:

The normal consequences of respondents illegal dismissal, then, are


reinstatement without loss of seniority rights, and payment of backwages
computed from the time compensation was withheld up to the date of actual
reinstatement. Where reinstatement is no longer viable as an option, separation
pay equivalent to one (1) month salary for every year of service should be
awarded as an alternative. The payment of separation pay is in addition to
payment of backwages.
The computation of separation pay is based on the length of the employee s
service; and the computation of backwages is based on the actual period when
the employee was unlawfully prevented from working.
The basis of computation of backwages

The computation of backwages depends on the final awards adjudged as a


consequence of illegal dismissal, in that:

First, when reinstatement is ordered, the general concept under Article 279 of
the Labor Code, as amended, computes the backwages from the time of dismissal
until the employees reinstatement. The computation of backwages (and similar
benefits considered part of the backwages) can even continue beyond the
decision of the labor arbiter or NLRC and ends only when the employee is
actually reinstated.42
Second, when separation pay is ordered in lieu of reinstatement (in the event
that this aspect of the case is disputed) or reinstatement is waived by the
employee (in the event that the payment of separation pay, in lieu, is not
disputed), backwages is computed from the time of dismissal until the finality of
the decision ordering separation pay.

Third, when separation pay is ordered after the finality of the decision ordering
the reinstatement by reason of a supervening event that makes the award of
reinstatement no longer possible (as in the case), backwages is computed from
the time of dismissal until the finality of the decision ordering separation pay.

The above computation of backwages, when separation pay is ordered, has been
the Court s consistent ruling. In Session Delights Ice Cream and Fast Foods v.
Court Appeals Sixth Division, we explained that the finality of the decision
becomes the reckoning point because in allowing separation pay, the final
decision effectively declares that the employment relationship ended so that
separation pay and backwages are to be computed up to that point.
We may also view the proper computation of backwages (whether based on
reinstatement or an order of separation pay) in terms of the life of the
employment relationship itself.1wphi1

When reinstatement is ordered, the employment relationship continues. Once


the illegally dismissed employee is reinstated, any compensation and benefits
thereafter received stem from the employee s continued employment. In this
instance, backwages are computed only up until the reinstatement of the

3E Andaya Ching Espiritu Hefti Galvez Gammad Lainez Lui Madamba Nagera Narvasa
Ong Palangdao Rosales Sanchez Santos Satrain Tabo (2014-2015)

Labor Relations Case Digest - Atty. Joyrich Golangco

employee since after the reinstatement, the employee begins to receive


compensation from his resumed employment.

When there is an order of separation pay (in lieu of reinstatement or when the
reinstatement aspect is waived or subsequently ordered in light of a supervening
event making the award of reinstatement no longer possible), the employment
relationship is terminated only upon the finality of the decision ordering the
separation pay. The finality of the decision cuts-off the employment relationship
and represents the final settlement of the rights and obligations of the parties
against each other. Hence, backwages no longer accumulate upon the finality of
the decision ordering the payment of separation pay since the employee is no
longer entitled to any compensation from the employer by reason of the
severance of his employment.
In this case, the change of awards (from reinstatement to separation pay) under
the NLRC s July 31, 1998 not only modified the awards granted, but also changed
the manner the respondents backwages is to be computed. The respondents
backwages can no longer be computed up to the point of reinstatement as there
is no longer any award of reinstatement to speak of.

Thus, the computation of the respondents' backwages must be from the


time of the illegal dismissal from employment until the finality of the decision
ordering the payment of separation pay. It is only when the NLRC rendered its
July 31, 1998 decision ordering the payment of separation pay (which both
parties no longer questioned and which thereafter became final) that the issue of
the respondents' employment with the petitioners was decided with finality,
effectively terminating it. The respondents' backwages, therefore, must be
computed from the time of their illegal dismissal until January 29, 1999, the date
of finality of the NLRC's July 31, 1998 Decision. As a final point, the CA s ruling
must be modified to include legal interest commencing from the finality of the
NLRC's July 31, 1998 decision.
139. Universal Robina Corporation vs Castillo,

DOCTRINE: Separation pay "as a measure of social justice" is allowed in those


instances where the employee is validly dismissed for causes other than serious

misconduct or those reflecting on his moral character. That the award of


separation pay is authorized in the situations dealt with in Article 283 and 284 of
the Labor Code, but not in terminations of employment based on instances
enumerated in Article 282.
FACTS: Respondent Wilfredo Z. Castillo (Castillo) was hired by petitioner
Universal Robina Corporation (URC) as a truck salesman with a monthly salary
of P4,000.00. He rose from the ranks and became a Regional Sales Manager, until
his dismissal. As Regional Sales Manager, respondent was responsible for
planning, monitoring, leading and controlling all activities affecting smooth sales
operation. Among his tasks was to transact, sign and represent the company in
all its dealings with key accounts or customers subject however to his selling
expense budget duly approved by URC Management. Consequently, he is obliged
to give an account of all his dealings or transactions with all his customers to
URC. His area of responsibility covered some parts of Laguna, including Lianas
Supermart (Liana) in San Pablo City, Laguna. On 19 August 2005, URCs Credit
and Collection Department (CCD) Analyst in Silangan, Laguna Branch noted an
outright deduction in the amount of P72,000.00 tagged as Gift Certificate. The
CCD Analyst found the issuance of GCs as unusual. This finding prompted URCs
Corporate Internal Audit (CIA) to conduct a routine audit of the unresolved
accounts of Lianas account receivables. Based on investigation, Ms. Prezy
Manansala, Lianas San Pablo Branch Manager agreed to sponsor a Back to
School Promo which however, was cancelled, that the fact was known to RSM
Castillo as he was the one who signed on the received documents presented.
Nonetheless, Copies of the Charged Invoice Nos. 2189 and 2190 were
marked/stamped paid as these charges were already deducted from their
payment to URC.
The CIA suspected that respondent might have committed an act of fraud against
the company and Lianas for his personal gain. Respondent was asked to explain
in writing why the company should not institute the appropriate disciplinary
action against him for possible violation of Offenses Subject to Disciplinary
Action. He averred that the loss was due to the revisions of Lianas original plan
and cancellation of the raffle draw portion but the expenses caused was for the
use of the cut-case displays, also he denied accepting any gift certificate.
Thereafter, another letter was sent to him directing him to explain why no

3E Andaya Ching Espiritu Hefti Galvez Gammad Lainez Lui Madamba Nagera Narvasa
Ong Palangdao Rosales Sanchez Santos Satrain Tabo (2014-2015)

Labor Relations Case Digest - Atty. Joyrich Golangco

administrative sanctions should be meted against him. Despite his explanations


he was served a written notice of termination. He then filed a complaint for
illegal dismissal against petitioner and its president and chief operating officer
Gokongwei averring that the charges for his dismissal was totally different from
those stated during the investigation. LA decided that respondent was illegally
dismissed and ordered the payment of backwages and separation pay. The NLRC
reversed the decision and denied respondents motion for reconsideration. CA
upheld the dismissal but awarded separation pay due to the reason that he
served the company 23 years and that this was his first and only offense.
ISSUES: WON respondent is illegally terminated from his employment and if in
the negative, whether he is entitled to separation pay

HELD: NO. Indeed, respondent has committed acts constituting willful breach of
trust and confidence reposed on him by URC, , petitioner Castillos ready
admission that he signed the charge invoices even if these were blank clearly
shows his negligence and utter lack or care in the interests of private respondent
URC. As a Regional Sales Manager, petitioner Castillo occupied a position or
responsibility and as such, he should have known that he placed the interests of
the company at a disadvantage by signing the blank charge invoices. The leading
case of Philippine Long Distance Telephone Co. v. NLRC15 enunciated the ruling
that separation pay "as a measure of social justice" is allowed in those instances
where the employee is validly dismissed for causes other than serious
misconduct or those reflecting on his moral character.16 The case of Toyota
Motor Phils. Corp. Workers Association (TMPCWA) v. NLRC17 expanded the
doctrine laid down in PLDT by adding dismissals other than those under Art. 282
of the Labor Code, like willful disobedience, gross and habitual neglect of duty,
fraud or willful breach of trust, and commission of a crime against the employer
or his family which would preclude award of separation pay. As the rule now
stands, the award of separation pay is authorized in the situations dealt with in
Article 283 and 284 of the Labor Code, but not in terminations of employment
based on instances enumerated in Article 282
140. MINETTE BAPTISTA, BANNIE EDSEL SAN MIGUEL, and MA.
FEDAYON, Petitioners, vs.ROSARIO VILLANUEVA, JANETTE ROLDAN, DANILO
OLAYVAR, ONOFRE ESTRELLA, CATALINO LEDDA, MANOLO GUBANGCO,

GILBERT ORIBIANA, CONSTANCIO SANTIAGO, RUTH BAYQUEN, RUBY


CASTANEDA, ALFRED LANDAS, JR., ROSELYN GARCES, EUGENE CRUZ,
MENANDRO SAMSON, FEDERICO MUNOZ and SALVADOR
DIWA, Respondents.

FACTS: Petitioners were former union members of Radio Philippines Network


Employees Union (RPNEU), a legitimate labor organization and the sole and
exclusive bargaining agent of the rank and file employees of Radio Philippines
Network (RPN), while the respondents were the unions elected officers and
members.

On April 26, 2005, on suspicion of union mismanagement, petitioners, together


with some other union members, filed a complaint for impeachment of all the
union officers and members of RPNEU before the Department of Labor and
Employment (DOLE).

Several complaints against petitioners were thereafter filed, one for alleged
violation of the unions Constitution and By-Laws and another for commission
of an act which violates RPNEU Constitution and By-Laws, specifically, Article
IX, Section 2.2 for joining or forming a union outside the sixty (60) days period
and Article IX, Section 2.5 for urging or advocating that a member start an action
in any court of justice or external investigative body against the Union or its
officer without first exhausting all internal remedies open to him or available in
accordance with the CBL before the Chairman of RPNEUs Committee on
Grievance and Investigation. These complaints were, later on, consolidated.

Thereafter, petitioners received a memorandum notice from Jeric Salinas,


Chairman of the Committee, requesting them to answer the complaint and attend
a hearing scheduled on October 3, 2005.10 Petitioners and their group denied the
charges imputed against them and contested the procedure adopted by the
Committee in its investigation.
The Committee submitted their recommendation of expulsion from the union to
RPNEUs Board of Directors, which affirmed such recommendation. Through a
Memorandum, petitioners were served an expulsion notice from the union,
which was set to take effect on December 29, 2005.

3E Andaya Ching Espiritu Hefti Galvez Gammad Lainez Lui Madamba Nagera Narvasa
Ong Palangdao Rosales Sanchez Santos Satrain Tabo (2014-2015)

Labor Relations Case Digest - Atty. Joyrich Golangco

On January 2, 2006, petitioners with the 12 others wrote to RPNEUs President


and Board of Directors that their expulsion from the union was an ultra vires act
because the Committee failed to observe the basic elements of due process
because they were not given the chance to physically confront and examine their
complainants.
In a letter, RPNEUs officers informed their company of the expulsion of
petitioners and the 12 others from the union and requested the management
to serve them notices of termination from employment in compliance with
their CBAs union security clause.15 On February 17, 2006, RPN HRD Manager,
Lourdes Angeles, informed petitioners and the 12 others of suchh termination of
their employment effective March 20, 2006. Aggrieved, petitioners filed
complaints for ULP against the respondents, questioning legality of their
expulsion from the union and their subsequent termination from employment.
LA ruled in favour of petitioners and adjudged the respondents guilty of ULP
pursuant to Article 249 (a) and (b) of the Labor Code.

NLRC set aside the LA decision and dismissed the complaint for ULP for lack of
merit. The NLRC found that petitioners did not first resort to internal remedies
available under its own Constitution and By-Laws in filing the impeachment.

The CA, in its March 9, 2010 Decision, sustained the NLRC decision. The CA
stated that the termination of employment by virtue of a union security clause
was recognized in our jurisdiction. The CA declared that petitioners were
accorded due process before they were removed from office as petitioners were
given the opportunity to explain their case and they actually availed of said
opportunity by submitting letters containing their arguments.
ISSUES: WON the proper procedure was followed in terminating petitioners.

HELD: Petitioners posit that the procedure that should have been followed by
the respondents in resolving the charges against them was Article XVII,
Settlement of Internal Disputes of their Constitution and By-Laws, specifically,
Section 232 thereof, requiring members to put their grievance in writing to be
submitted to their union president, who shall strive to have the parties settle

their differences amicably. Petitioners maintain that any form of grievance would
be referred only to the committee upon failure of the parties to settle amicably.33
The Court is not persuaded.

Based on RPNEUs Constitution and By-Laws, the charges against petitioners


were not mere internal squabbles, but violations that demand proper
investigation because, if proven, would constitute grounds for their expulsion
from the union. As such, Article X, Investigation Procedures and Appeal Process
of RPNEUs Constitution and By-Laws was correctly applied under the
circumstances.
SECTION 1. Charge against any member or officer of the Union
shall be submitted to the Board of Directors in writing, which
shall refer the same, if necessary, to the committee on Grievance
and Investigation. The Committee shall hear any charge and
subsequently, forward its finding and recommendation to the
BOD. The BOD has the power to approve or nullify the
recommendation of the Committee on Grievance and
Investigation based on the merit of the appeal.

Any supposed procedural flaw in the proceedings before the Committee was
deemed cured when petitioners were given the opportunity to be heard. Due
process, as a constitutional precept, is satisfied when a person was notified of the
charge against him and was given an opportunity to explain or defend himself. In
administrative proceedings, the filing of charges and giving reasonable
opportunity for the person so charged to answer the accusations against him
constitute the minimum requirements of due process. It cannot be denied that
petitioners were properly notified of the charges filed against them and were
equally afforded the opportunity to present their side.
It is well-settled that workers and employers organizations shall have the right
to draw up their constitutions and rules to elect their representatives in full
freedom, to organize their administration and activities and to formulate their
programs.

3E Andaya Ching Espiritu Hefti Galvez Gammad Lainez Lui Madamba Nagera Narvasa
Ong Palangdao Rosales Sanchez Santos Satrain Tabo (2014-2015)

Labor Relations Case Digest - Atty. Joyrich Golangco

RPNEUs Constitution and By-Laws expressly mandate that before a party is


allowed to seek the intervention of the court, it is a pre-condition that he should
have availed of all the internal remedies within the organization. Petitioners
were found to have violated the provisions of the unions Constitution and ByLaws when they filed petitions for impeachment against their union officers and
for audit before the DOLE without first exhausting all internal remedies available
within their organization. This act is a ground for expulsion from union
membership. Thus, petitioners expulsion from the union was not a deliberate
attempt to curtail or restrict their right to organize, but was triggered by the
commission of an act, expressly sanctioned by Section 2.5 of Article IX of the
unions Constitution and By-Laws.
Unfortunately, petitioners failed to discharge the burden required to prove the
charge of ULP against the respondents. Aside from their self-serving allegations,
petitioners were not able to establish how they were restrained or coerced by
their union in a way that curtailed their right to self-organization.
141 : BPI EMPLOYEES UNION-DAVAO vs BPI

DOCTRINE: It is management prerogative to farm out any of its activities,


regardless of whether such activity is peripheral or core in nature. What is of
primordial importance is that the service agreement does not violate the
employee's right to security of tenure and payment of benefits to which he is
entitled under the law. Furthermore, the outsourcing must not squarely fall
under labor-only contracting
FACTS
BPI Operations Management Corporation (BOMC) was created pursuant
to CBP Circular No. 1388, 1993, primarily engaged in providing and/or
handling support services for banks and other financial institutions, is a
subsidiary of the BPI operating and functioning as an entirely separate
and distinct entity
A service agreement between BPI and BOMC was initially implemented
in BPIs Metro Manila branches. In this agreement, BOMC undertook to
provide services such as check clearing, delivery of bank statements,
fund transfers, card production, operations accounting and control, and
cash servicing. Not a single BPI employee was displaced and those

performing the functions, which were transferred to BOMC, were given


other assignments.
The service agreement was likewise implemented in Davao City. Later, a
merger between BPI and Far East Bank and Trust Company (FEBTC)
took effect with BPI as the surviving corporation. Thereafter, BPIs
cashiering function and FEBTCs cashiering, distribution and
bookkeeping functions were handled by BOMC. Consequently, 12
former FEBTC employees were transferred to BOMC to complete the
latters service complement. BPI Davaos rank and file collective
bargaining agent, BPIEU-DAVAO-Union objected to the transfer of
the functions and the 12 personnel to BOMC contending that the
functions rightfully belonged to the BPI employees and that the
Union was deprived of membership of former FEBTC personnel
who, by virtue of the merger, would have formed part of the
bargaining unit pursuant to its union shop provision in the CBA. In
contracting out FEBTC functions to BOMC, BPI effectively deprived the
union of the membership of employees handling said functions as well
as curtailed the right of those employees to join the union. Management
prerogative is not absolute.
During the Labor Management Conference between the parties, BPI
invoked management prerogative stating that the creation of the
BOMC was to preserve more jobs and to designate it as an agency to
place employees where they were most needed.
As BPI allegedly ignored the demand of the Union that the matter be
submitted to the grievance machinery, the latter filed a notice of strike
before the NCMB on the following grounds:
a. Contracting out services/functions performed by union
members that interfered with, restrained and/or coerced the
employees in the exercise of their right to self-organization;
b. Violation of duty to bargain; and
c. Union busting.
BPI then filed a petition for assumption of jurisdiction with the SOLE,
who subsequently issued an order certifying the labor dispute to the
NLRC for compulsory arbitration.
NLRC: upheld the validity of the service agreement between BPI and BOMC and
dismissing the charge of ULP. The engagement by BPI of BOMC to undertake

3E Andaya Ching Espiritu Hefti Galvez Gammad Lainez Lui Madamba Nagera Narvasa
Ong Palangdao Rosales Sanchez Santos Satrain Tabo (2014-2015)

Labor Relations Case Digest - Atty. Joyrich Golangco

some of its activities was clearly a valid exercise of its management prerogative.
It further stated that the spinning off by BPI to BOMC of certain services and
functions did not interfere with, restrain or coerce employees in the exercise of
their right to self-organization
CA: affirmed NLRCs resolution considering the ramifications of the corporate
merger, it was well within BPIs prerogatives "to determine what additional tasks
should be performed, who should best perform it and what should be done to
meet the exigencies of business." It pointed out that the Union did not, by the
mere fact of the merger, become the bargaining agent of the merged
employees.

ISSUES: whether or not the act of BPI to outsource the cashiering, distribution
and bookkeeping functions to BOMC is in conformity with the law and the
existing CBA
HELD:

In accordance with Art. 261 of the labor code only gross violations of the
economic provisions of the CBA are treated as ULP. Otherwise, they are
mere grievances. In the present case, the alleged violation of the union
shop agreement in the CBA, even assuming it was malicious and flagrant,
is not a violation of an economic provision in the agreement.
The union failed to take into consideration its recognition of the banks
exclusive rights and prerogatives, likewise provided in the CBA, which
included the hiring of employees, promotion, transfers, and dismissals
for just cause and the maintenance of order, discipline and efficiency in
its operations.
It is incomprehensible how the "reduction of positions in the collective
bargaining unit" interferes with the employees right to self-organization
because the employees themselves were neither transferred nor
dismissed from the service.
In the case at hand, the union has not presented even an iota of
evidence that petitioner bank has started to terminate certain
employees, members of the union. In fact, what appears is that the
Bank has exerted utmost diligence, care and effort to see to it that
no union member has been terminated. In the process of the
consolidation or merger of the two banks which resulted in increased
diversification of functions, some of these non-banking functions

were merely transferred to the BOMC without affecting the union


membership.

CONTRACTING OUT OF SERVICE


It is to be emphasized that contracting out of services is not illegal per se
It is an exercise of business judgment or management prerogative.
Absent proof that the management acted in a malicious or arbitrary
manner, the Court will not interfere with the exercise of judgment
by an employer.
In this case, bad faith cannot be attributed to BPI because its actions
were authorized by CBP Circular No. 1388, Series of 1993
The fact that banks are of a specialized industry must, however, be taken
into account. The competence in determining which banking functions
may or may not be outsourced lies with the BSP. This does not mean that
banks can simply outsource banking functions allowed by the BSP
through its circulars, without giving regard to the guidelines set forth
under D.O. No. 10 issued by the DOLE.
CBP Circular No. 138838 enumerated functions which are ancillary to
the business of banks, hence, allowed to be outsourced.
The subject functions appear to be not in any way directly related to the
core activities of banks. They are functions in a processing center of BPI
which does not handle or manage deposit transactions. Clearly, the
functions outsourced are not inherent banking functions, and, thus,
are well within the permissible services under the circular.
From the very definition of "banks" as provided under the General
Banking Law, it can easily be discerned that banks perform only 2 main
or basic functions deposit and loan functions. Thus, cashiering,
distribution and bookkeeping are but ancillary functions whose
outsourcing is sanctioned under CBP Circular No. 1388 as well as D.O.
No. 10.
In one case, the Court held that it is management prerogative to farm out
any of its activities, regardless of whether such activity is peripheral or
core in nature. What is of primordial importance is that the service
agreement does not violate the employee's right to security of
tenure and payment of benefits to which he is entitled under the
law. Furthermore, the outsourcing must not squarely fall under

3E Andaya Ching Espiritu Hefti Galvez Gammad Lainez Lui Madamba Nagera Narvasa
Ong Palangdao Rosales Sanchez Santos Satrain Tabo (2014-2015)

Labor Relations Case Digest - Atty. Joyrich Golangco

labor-only contracting where the contractor or sub-contractor merely


recruits, supplies or places workers to perform a job, work or service for
a principal
142. INTEGRATED MICROELECTRONICS, INC., Petitioner, vs ADONIS A.
PIONILLA, Respondent.

DOCTRINE: As a general rule, an illegally dismissed employee is entitled to


reinstatement or separation pay, if reinstatement is not viable; and payment of
full backwages. The Court has carved out an exception to the foregoing rule and
thereby ordered the reinstatement of the employee without backwages: (a) the
fact that dismissal of the employee would be too harsh of a penalty; and (b) that
the employer was in good faith in terminating the employee.

FACTS: In 1996, Pionilla was hired by IMI as its production worker. On 2005,
Pionilla received a notice from IMI requiring him to explain the incident where
he was seen escorting a lady to board the company shuttle bus. It was reported
by the bus marshall that the lady was wearing a company ID which serves as a
free pass for shuttle bus passengers even if she was just a job applicant at IMI.
Pionilla admitted that he lent his ID to the lady who turned out to be his relative.
He risked lending her his ID to save on their transportation expenses. He
apologized for his actions.6

A Conscience Committee was formed to investigate the matter. During the


hearing, Pionilla admitted that at the time of the incident, he had two IDs in his
name as he lost his original ID but was able to secure a temporary ID later. IMI
found Pionilla guilty of violating the Company Rules and Regulations which
prohibits the lending of ones ID since the same is considered a breach of its
security rules and carries the penalty of dismissal. Subsequently, Pionilla was
dismissed. He filed a complaint for illegal dismissal with damages against IMI.7
The LA held that Pionilla have been illegally dismissed and ordered IMI to
reinstate him with backwages. It reasoned that Pionilla was harshly penalized
since the misdeed was not tainted with any wrongful intent.

On appeal, the NLRC, reversed the LAs ruling, finding Pionillas dismissal to be
valid. It held that Pionillas act of lending his temporary ID was willful and
intentional.

Pionilla filed a petition for certiorari before the CA which it granted. It found that
while IMIs regulations on company IDs were reasonable, the penalty of dismissal
was too harsh. Pionilla worked with IMI for a period of nine years without any
derogatory record. IMI moved for reconsideration which was denied.
IMI filed a petition for review on certiorari before the Court which was denied.
The Court ruled that the imposition of the penalty of dismissal was too harsh and
incommensurate to the infraction he committed. Hence, the present motion for
reconsideration.
ISSUES: Should Pionilla be reinstated with full backwages.
HELD: The motion for reconsideration is partly granted.

As a general rule, an illegally dismissed employee is entitled to reinstatement or


separation pay, if reinstatement is not viable; and payment of full backwages.
The Court has carved out an exception to the foregoing rule and thereby ordered
the reinstatement of the employee without backwages: (a) the fact that dismissal
of the employee would be too harsh of a penalty; and (b) that the employer was
in good faith in terminating the employee.
Petitioner's guilt was substantially established. Nevertheless, we agree with
respondent Minister's order of reinstating petitioner without backwages instead
of dismissal which may be too drastic. Denial of backwages would sufficiently
penalize her for her infractions. The bank officials acted in good faith. They
should be exempt from the burden of paying backwages. The good faith of the
employer, when clear under the circumstances, may preclude or diminish
recovery of backwages.
143. Golden Ace Builders vs Talde

3E Andaya Ching Espiritu Hefti Galvez Gammad Lainez Lui Madamba Nagera Narvasa
Ong Palangdao Rosales Sanchez Santos Satrain Tabo (2014-2015)

Labor Relations Case Digest - Atty. Joyrich Golangco

FACTS: Jose A. Talde (respondent) was hired in 1990 as a carpenter


by petitioner Golden Ace Builders of which its co-petitioner Arnold Azul (Azul) is
the owner-manager. In February 1999, Azul, alleging the unavailability of
construction projects, stopped giving work assignments to respondent,
prompting the latter to file a complaint for illegal dismissal.

By Decision of January 10, 2001, the Labor Arbiter ruled in favor of


respondent and ordered his immediate reinstatement without loss of seniority
rights and other privileges, and with payment of full backwages, which at that
time was computed at P144,382.23, and the amount of P3,236.37 representing
premium pay for rest days, service incentive leave pay and 13th month pay.
Pending their appeal to the National Labor Relations Commission (NLRC) and in
compliance with the Labor Arbiters Decision, petitioners, through counsel,
advised respondent to report for work in the construction site within 10 days
from receipt thereof. Respondent submitted, however, on May 16, 2001 a
manifestation to the Labor Arbiter that actual animosities existed between him
and petitioners and there had been threats to his life and his familys safety,
hence, he opted for the payment of separation pay. Petitioners denied the
existence of any such animosity. NLRC dismissed petitioners appeal and CA
affirmed the dismissal. The CA decision became final on September 15,2004. As
an agreement could not be forged by the parties on the satisfaction of the
judgment, the matter was referred to the Fiscal Examiner of the NLRC who
recomputed at P562,804.69 the amount due respondent, which was approved by
the Labor Arbiter by Order of July 5, 2005. A writ of execution dated July 8, 2005
was thereupon issued. Finding the amount exorbitant, petitioners filed a motion
for reconsideration with the NLRC, contending that since respondent refused to
report back to work, he should be considered to have abandoned the same,
hence, the recomputation of the wages and benefits due him should not be
beyond May 15, 2001, the date when he manifested his refusal to be
reinstated. NLRC granted the motion. CA set aside the NLRC resolution and
reinstated the previous computation and also held that respondent is entitled to
separation pay.
ISSUES: WON Talde is entitled to separation pay

HELD: YES, The basis for the payment of backwages is different from that for the
award of separation pay. Separation pay is granted where reinstatement is no

longer advisable because of strained relations between the employee and the
employer. Backwages represent compensation that should have been earned but
were not collected because of the unjust dismissal. The basis for computing
backwages is usually the length of the employees service while that for
separation pay is the actual period when the employee was unlawfully prevented
from working. Under the doctrine of strained relations, the payment of separation
pay is considered an acceptable alternative to reinstatement when the latter
option is no longer desirable or viable. On one hand, such payment liberates the
employee from what could be a highly oppressive work environment. On the
other hand, it releases the employer from the grossly unpalatable obligation of
maintaining in its employ a worker it could no longer trust.
In the present
case, the Labor Arbiter found that actual animosity existed between petitioner
Azul and respondent as a result of the filing of the illegal dismissal case. Such
finding, especially when affirmed by the appellate court as in the case at bar, is
binding upon the Court, consistent with the prevailing rules that this Court will
not try facts anew and that findings of facts of quasi-judicial bodies are accorded
great respect, even finality.
Clearly then, respondent is entitled to
backwages and separation pay as his reinstatement has been rendered
impossible due to strained relations. As correctly held by the appellate court, the
backwages due respondent must be computed from the time he was unjustly
dismissed until his actual reinstatement, or from February 1999 until June 30,
2005 when his reinstatement was rendered impossible without fault on his part.
144. Lynvil Fishing Enterprises vs. Ariola, G.R. No. 181974, February 1,
2012

DOCTRINE: The filing of a criminal cases before the prosecutors office


constitutes sufficient basis for a valid termination of employment on the grounds
of serious misconduct and/or loss of confidence

FACTS: Petitioner Lynvil Fishing Enterprises, Inc. (Lynvil) is engaged in deep-sea


fishing. Respondents services were engaged in various capacities: Andres G.
Ariola, captain; Jessie D. Alcovendas, chief mate; Jimmy B. Calinao, chief engineer;
Ismael G. Nubla, cook; Elorde Baez, oiler; and Leopoldo G. Sebullen, bodegero.

3E Andaya Ching Espiritu Hefti Galvez Gammad Lainez Lui Madamba Nagera Narvasa
Ong Palangdao Rosales Sanchez Santos Satrain Tabo (2014-2015)

Labor Relations Case Digest - Atty. Joyrich Golangco

The said employees were engaged on a per trip basis or por viaje
which terminates at the end of each trip. Ariola, Alcovendas and Calinao were
managerial field personnel while the rest of the crew were field personnel.

On Aug. 1, 1998, Lynvil received a report from Ramonito Clarido, one of


its employees, that on July 31, 1998, he witnessed that while on board the
company vessel Analyn VIII, respondents conspired with one another and stole
eight tubs of pampano and tangigue fish and delivered them to another
vessel.

By reason of the report and after initial investigation, Lynvils General


Manager Rosendo S. De Borja (De Borja) summoned respondents to explain
within five (5) days why they should not be dismissed from service. However,
except for Alcovendas and Baez, the respondents refused to sign the receipt of
the notice. Failing to explain as required, respondents employment was
terminated.

Lynvil, through De Borja, filed a criminal complaint against the


dismissed employees for violation of P.D. 532, or the Anti-Piracy and AntiHighway Robbery Law of 1974 before the Office of the City Prosecutor of
Malabon City.
On 12 November 1998, First Assistant City Prosecutor Rosauro
Silverio found probable cause for the indictment of the dismissed employees for
the crime of qualified theft under the Revised Penal Code. Relying on the finding
and Nasipit Lumber Company v. NLRC, 257 Phil. 937 (1989), Lynvil asserted
there was sufficient basis for valid termination of employment of respondents
based on serious misconduct and/or loss of trust and confidence.

FILED FOR ILLEGAL DISMISSAL with claims for backwages, salary


differential reinstatement, service incentive leave, holiday pay and its premium
and 13th month pay from 1996 to1998. They also claimed for moral, exemplary
damages and attorneys fees for their dismissal with bad faith.
Labor Arbiter: judgment was rendered finding that complainants were illegally
dismissed, ordering respondents to jointly and severally pay complainants (a)
separation pay at one half month pay for every year of service; (b) backwages;

(c) salary differential; (d) 13th month pay; and (e) attorneys fees.

LAs grounds for sustaining: The Labor Arbiter found that there was no evidence
showing that the private respondents received the 41 baeras of
pampano as alleged by De Borja in his reply-affidavit; and that no proof was
presented that the 8 baeras of pampano [and tangigue] were missing at the
place of destination.

The Labor Arbiter disregarded the Resolution of Assistant City


Prosecutor Rosauro Silverio on the theft case. He reasoned out that the Labor
Office is governed by different rules for the determination of the validity of the
dismissal of employees.
The Labor Arbiter found that the procedural due process was not
complied with and that the mere notice given to the private respondents fell
short of the requirement of ample opportunity to present the employees side.

NLRC: Reversed LAs decision. Dismissed the case for lack of merit. The
private respondents except Elorde Baez filed a Petition for Certiorari before the
Court of Appeals alleging grave abuse of discretion on the part of NLRC.

CA: The Court of Appeals found merit in the petition and reinstated the
Decision of the Labor Arbiter except as to the award of attorneys fees. The
appellate court held that the allegation of theft did not warrant the dismissal of
the employees since there was no evidence to prove the actual quantities of the
missing kinds of fish loaded to Analyn VIII. It also reversed the finding of the
NLRC that the dismissed employees were merely contractual employees and
added that they were regular ones performing activities, which are usually
necessary or desirable in the business and trade of Lynvil. Finally, it ruled that
the two-notice rule provided by law and jurisprudence is mandatory and noncompliance therewith rendered the dismissal of the employees illegal.
PETITION FOR REVIEW ON CERTIORARI TO SC BY LYNVIL.

ISSUES:

3E Andaya Ching Espiritu Hefti Galvez Gammad Lainez Lui Madamba Nagera Narvasa
Ong Palangdao Rosales Sanchez Santos Satrain Tabo (2014-2015)

Labor Relations Case Digest - Atty. Joyrich Golangco

WON the filing of a criminal cases before the prosecutors office constitutes
sufficient basis for a valid termination of employment on the grounds of serious
misconduct and/or loss of confidence
WON the respondents are regular employees

WON they were validly dismissed and if yes, WON the procedural due process of
dismissal was followed
HELD:

The Supreme Court is not a trier of facts. Under Rule 45, parties may
raise only questions of law, unless the case falls under any of the recognized
exceptions. One of the exceptions is when the findings of facts are conflicting. The
contrariety of the findings of the Labor Arbiter and the NLRC prevents reliance
on the principle of special administrative expertise and provides the reason for
judicial review, at first instance by the appellate court, and on final study through
the present petition.
1. YES. Breach of trust is present in this case. The Court agrees with
the ruling of the Labor Arbiter and Court of Appeals that the quantity of tubs
expected to be received was the same as that which was loaded. However, what
is material is the kind of fish loaded and then unloaded. Sameness is likewise
needed. The Court argued that it couldnt close its eyes to the positive and clear
narration of facts of the three witnesses to the commission of qualified theft.

Proof beyond reasonable doubt of an employees misconduct is not


required when loss of confidence is the ground for dismissal. It is sufficient if the
employer has some basis to lose confidence or that the employer has
reasonable ground to believe or to entertain the moral conviction that the
employee concerned is responsible for the misconduct and that the nature of his
participation therein rendered him absolutely unworthy of the trust and
confidence demanded by his position.

2. YES. In the context of the facts that: (1) the respondents were doing
tasks necessarily to Lynvils fishing business with positions ranging from captain
of the vessel to bodegero; (2) after the end of a trip, they will again be hired for
another trip with new contracts; and (3) this arrangement continued for more

than ten years, the clear intention is to go around the security of tenure of the
respondents as regular employees. And respondents are so by the express
provisions of the second paragraph of Article 280, thus: xxx Provided, That any
employee who has rendered at least one year of service, whether such service is
continuous or broken, shall be considered a regular employee with respect to the
activity in which he is employed and his employment shall continue while such
activity exists.
Jurisprudence, laid two conditions for the validity of a fixed-contract
agreement between the employer and employee: first, the fixed period of
employment was knowingly and voluntarily agreed upon by the parties without
any force, duress, or improper pressure being brought to bear upon the
employee and absent any other circumstances vitiating his consent; or second, it
satisfactorily appears that the employer and the employee dealt with each other
on more or less equal terms with no moral dominance exercised by the former or
the latter. In this case, LYNVIL exercised moral dominance over the respondents.
It is their need for a source of income that they were forced to agree with a por
viaje setting of employment.

3. YES AND NO. Having found that respondents are regular employees
who may be, however, dismissed for cause as we have so found in this case, there
is a need to look into the procedural requirement of due process in Section 2,
Rule XXIII, Book V of the Rules Implementing the Labor Code. It is required that
the employer furnish the employee with two written notices: (1) a written
notice served on the employee specifying the ground or grounds for termination,
and giving to said employee reasonable opportunity within which to explain his
side; and (2) a written notice of termination served on the employee indicating
that upon due consideration of all the circumstances, grounds have been
established to justify his termination. In this case, it is clear that the employees
were not given the final written notices of dismissal.
The Court ruled that since employees were dismissed for just cause, they
were not entitle to separation pay and backwages. However, they were to be
granted nominal damages for failure of the employer to comply with statutory
due process.

3E Andaya Ching Espiritu Hefti Galvez Gammad Lainez Lui Madamba Nagera Narvasa
Ong Palangdao Rosales Sanchez Santos Satrain Tabo (2014-2015)

Labor Relations Case Digest - Atty. Joyrich Golangco

145. CONSOLIDATED BROADCASTING V. OBERIO

DOCTRINE: 'An illegally dismissed employee is entitled to: (1) either


reinstatement if viable or separation pay if reinstatement is no longer viable, and
(2) backwages. If reinstatement is no longer viable considering the strained
relations between petitioner and respondents, payment of separation pay
instead of reinstatement would thus better promote the interest of both parties.'

FACTS: Respondents alleged that they were employed as drama talents by


DYWB-Bombo Radyo, a radio station owned and operated by petitioner
Consolidated Broadcasting System, Inc. They reported for work daily for six days
in a week and were required to record their drama production in advance. Some
of them were employed by petitioner since 1974, while the latest one was hired
in 1997. Their drama programs were aired not only in Bacolod City but also in
the sister stations of DYWB in the Visayas and Mindanao areas.

Sometime in August 1998, petitioner reduced the number of its drama


productions from 14 to 11, but was opposed by respondents. After the
negotiations failed, the latter sought the intervention of the Department of Labor
and Employment (DOLE), which on November 12, 1998, conducted through its
Regional Ofce, an inspection of DWYB station. The results thereof revealed that
petitioner is guilty of violation of labor standard laws, such as underpayment of
wages, 13th month pay, non-payment of service incentive leave pay, and noncoverage of respondents under the Social Security System.
Petitioner contended that respondents are not its employees and refused to
submit the payroll and daily time records despite the subpoena duces tecum
issued by the DOLE Regional Director. Petitioner further argued that the case
should be referred to the NLRC because the Regional Director has no jurisdiction
over the determination of the existence of employer-employee relationship
which involves evidentiary matters that are not veriable in the normal course of
inspection.
Vexed by the respondents complaint, petitioner allegedly pressured and
intimidated respondents. Respondents Oberio and Delta were suspended for
minor lapses and the payment of their salaries were purportedly delayed.

Eventually, on February 3, 1999, pending the outcome of the inspection case with
the Regional Director, respondents were barred by petitioner from reporting for
work; thus, the former claimed constructive dismissal.
April 8, 1999, the DOLE Regional Director issued an order directing petitioner to
pay
respondents
a
total
of
P318,986.74
representing
nonpayment/underpayment of the salary and benets due them. However, on July 8,
1999, the Regional Director reconsidered the April 8, 1999 order and certied
the records of the case to the NLRC, Regional Arbitration Branch VI, for
determination of employer-employee relationship. Respondents appealed said
order to the Secretary of Labor.

On October 12, 1999, respondents led a case for illegal dismissal,


underpayment/non-payment of wages and benets plus damages against
petitioner. On April 10, 2000, the Labor Arbiter dismissed the case without
prejudice while waiting for the decision of the Secretary of Labor on the same
issue of the existence of an employer-employee relationship between petitioner
and respondents.

On appeal to the NLRC, respondents raised the issue of employer-employee


relationship and submitted the following to prove the existence of such
relationship, to wit: time cards, identication cards, payroll, a show cause order
of the station manager to respondent Danny Oberio and memoranda either noted
or issued by said manager. Petitioner, on the other hand, did not present any
documentary evidence in its behalf and merely denied the allegations of
respondents. It claimed that the radio station pays for the drama recorded by
piece and that it has no control over the conduct of respondents.On December 5,
2001, the NLRC rendered a decision holding that respondents were regular
employees of petitioner who were illegally dismissed by the latter. It further held
that respondents complied with the requirements of the rule on forum shopping.
ISSUES 1) Were the respondents employees of petitioner?
2) Was their dismissal was illegal?
HELD:

3E Andaya Ching Espiritu Hefti Galvez Gammad Lainez Lui Madamba Nagera Narvasa
Ong Palangdao Rosales Sanchez Santos Satrain Tabo (2014-2015)

Labor Relations Case Digest - Atty. Joyrich Golangco

1. Respondents employment with petitioner passed the four-fold test on


employer-employee relations, namely:
(1) the selection and engagement of the employee, or the power to hire; (2) the
payment of wages; (3) the power to dismiss; and (4) the power to control the
employee.

Petitioner failed to controvert with substantial evidence the allegation of


respondents that they were hired by the former on various dates from 1974 to
1997. If petitioner did not hire respondents and if it was the director alone who
chose the talents, petitioner could have easily shown, being in possession of the
records, a contract to such effect. However, petitioner merely relied on its
contention that respondents were piece rate contractors who were paid by
results. Note that under Policy Instruction No. 40, petitioner is obliged to execute
the necessary contract specifying the nature of the work to be performed, rates
of pay, and the programs in which they will work. Moreover, project or
contractual employees are required to be apprised of the project they will
undertake under a written contract. This was not complied with by the
petitioner, justifying the reasonable conclusion that no such contracts exist and
that respondents were in fact regular employees.

Moreover, the engagement of respondents for a period ranging from 2 to 25


years and the fact that their drama programs were aired not only in Bacolod City
but also in the sister stations of DYWB in the Visayas and Mindanao areas,
undoubtedly show that their work is necessary and indispensable to the usual
business or trade of petitioner. The test to determine whether employment is
regular or not is the reasonable connection between the particular activity
performed by the employee in relation to the usual business or trade of the
employer. Also, if the employee has been performing the job for at least one
year, even if the performance is not continuous or merely intermittent, the law
deems the repeated and continuing need for its performance as sufcient
evidence of the necessity, if not indispensability of that activity to the business.
Thus, even assuming that respondents were initially hired as project/contractual
employees who were paid per drama or per project/contract, the engagement of
their services for 2 to 25 years justify their classication as regular employees,
their services being deemed indispensable to the business of petitioner.

As to the payment of wages, it was petitioner who paid the same as shown by the
payroll bearing the name of petitioner company in the heading with the
respective salaries of respondents opposite their names. Anent the power of
control, dismissal, and imposition of disciplinary measures, which are indicative
of an employer-employee relationship, the same were duly proven by the
following: (1) memorandum duly noted by Wilfredo Alejaga, petitioners station
manager, calling the attention of the Drama Department to the late submission
of scripts by writers and the tardiness and absences of directors and talents, as
well as the imposable nes of P100 to P200 for future infractions; (2) the
memorandum of the station manager directing respondent Oberio to explain
why no disciplinary action should be taken against him for punching the time
card of a certain Mrs. Fe Oberio who was not physically present in their ofce;
and (3) the station managers memorandum suspending respondent Oberio for
six days for the said infraction which constituted violation of petitioners
network policy. All these, taken together, unmistakably show theexistence of an
employer-employee relationship. Not only did petitioner possess the power of
control over their work but also the power to discipline them through the
imposition of nes and suspension for violation of company rules and policies.

2. Respondents were illegally dismissed. In labor cases, the employer has the
burden of proving that the dismissal was for a just cause; failure to show this
would necessarily mean that the dismissal was unjustied and, therefore, illegal.
To allow an employer to dismiss an employee based on mere allegations and
generalities would place the employee at the mercy of his employer; and the
right to security of tenure, which this Court is bound to protect, would be unduly
emasculated. In this case, petitioner merely contended that it was respondents
who ceased to report to work, and never presented any substantial evidence to
support said allegation. Petitioner therefore failed to discharge its burden, hence,
respondents were correctly declared to have been illegally dismissed.
Furthermore, if doubts exist between the evidence presented by the employer
and the employee, the scales of justice must be tilted in favor of the latter the
employer must afrmatively show rationally adequate evidence that the
dismissal was for a justiable cause. It is a time-honored rule that in
controversies between a laborer and his master, doubts reasonably arising from
the evidence should be resolved in the formers favor. The policy is to extend the

3E Andaya Ching Espiritu Hefti Galvez Gammad Lainez Lui Madamba Nagera Narvasa
Ong Palangdao Rosales Sanchez Santos Satrain Tabo (2014-2015)

Labor Relations Case Digest - Atty. Joyrich Golangco

doctrine to a greater number of employees who can avail of the benets under
the law, which is in consonance with the avowed policy of the State to give
maximum aid and protection of labor.

When a person is illegally dismissed, he is entitled to reinstatement without loss


of seniority rights and other privileges and to his full backwages. In the event,
however, that reinstatement is no longer feasible, or if the employee decides not
to be reinstated, the employer shall pay him separation pay in lieu of
reinstatement. Such a rule is likewise observed in the case of a strained
employer-employee relationship or when the work or position formerly held by
the dismissed employee no longer exists. In sum, an illegally dismissed employee
is entitled to: (1) either reinstatement if viable or separation pay if reinstatement
is no longer viable, and (2) backwages. In the instant controversy, reinstatement
is no longer viable considering the strained relations between petitioner and
respondents. As admitted by the latter, the complaint led before the DOLE
strained their relations with petitioner who eventually dismissed them from
service. Payment of separation pay instead of reinstatement would thus better
promote the interest of both parties
146. Orozco vs CA, Philippine Daily Inquirer(PDI) and Leticia Jimenez
Magsanoc

DOCTRINE: A Newspaper Columnist is not an Employee of the newspaper which


publishes its column
FACTS: In March 1990, PDI engaged the services of petitioner to write a weekly
column for its Lifestyle section. She religiously submitted her articles every
week, except for a six-month stint in New York City when she, nonetheless, sent
several articles through mail. She received compensation of P250.00 later
increased to P300.00 for every column published.
On November 7, 1992, petitioners column appeared in the PDI for the last time.
Petitioner claims that her then editor, Ms. Lita T. Logarta, told her that
respondent Leticia Jimenez Magsanoc, PDI Editor in Chief, wanted to stop
publishing her column for no reason at all and advised petitioner to talk to
Magsanoc herself. Petitioner narrates that when she talked to Magsanoc, the

latter informed her that it was PDI Chairperson Eugenia Apostol who had asked
to stop publication of her column, but that in a telephone conversation with
Apostol, the latter said that Magsanoc informed her (Apostol) that the Lifestyle
section already had many columnists.

On the other hand, PDI claims that in June 1991, Magsanoc met with the Lifestyle
section editor to discuss how to improve said section. They agreed to cut down
the number of columnists by keeping only those whose columns were wellwritten, with regular feedback and following. In their judgment, petitioners
column failed to improve, continued to be superficially and poorly written, and
failed to meet the high standards of the newspaper. Hence, they decided to
terminate petitioners column.
Aggrieved by the newspapers action, petitioner filed a complaint for illegal
dismissal, backwages, moral and exemplary damages, and other money claims
before the NLRC.
LA: Petitioner is an employee of the Company, thus she is entitled to
reinstatement, backwages, 13th month pay and service incentive leave pay it
found out that:

PDI exercised full and complete control over the means and method by which
complainants work that of a regular columnist had to be accomplished. This
control might not be found in an instruction, verbal or oral, given to complainant
defining the means and method she should write her column. Rather, this control
is manifested and ascertained in respondents admitted prerogative to reject any
article submitted by complainant for publication.

By virtue of this power, complainant was helplessly constrained to adopt her


subjects and style of writing to suit the editorial taste of her editor. Otherwise, off
to the trash can went her articles.

Moreover, this control is already manifested in column title, "Feminist


Reflection" allotted complainant. Under this title, complainants writing was
controlled and limited to a womans perspective on matters of feminine interests.
That respondent had no control over the subject matter written by complainant

3E Andaya Ching Espiritu Hefti Galvez Gammad Lainez Lui Madamba Nagera Narvasa
Ong Palangdao Rosales Sanchez Santos Satrain Tabo (2014-2015)

Labor Relations Case Digest - Atty. Joyrich Golangco

is strongly belied by this observation. Even the length of complainants articles


were set by respondents. her actions were controlled by her obligation to
produce an article a week. If complainant did not have to report for work eight
(8) hours a day, six (6) days a week, it is because her task was mainly mental.
Lastly, the fact that her articles were (sic) published weekly for three (3) years
show that she was respondents regular employee, not a once-in-a-blue-moon
contributor who was not under any pressure or obligation to produce regular
articles and who wrote at his own whim and leisure.
PDI appealed the Decision to the NLRC.
NLRC: affirmed LA
CA: Reversed the NLRC.

The Court does not agree with public respondent NLRCs conclusion. First,
private respondent admitted that she was and had never been considered by
petitioner PDI as its employee. Second, it is not disputed that private respondent
had no employment contract with petitioner PDI. In fact, her engagement to
contribute articles for publication was based on a verbal agreement between her
and the petitioners Lifestyle Section Editor. Moreover, it was evident that
private respondent was not required to report to the office eight (8) hours a day.
Further, it is not disputed that she stayed in New York for six (6) months without
petitioners permission as to her leave of absence nor was she given any
disciplinary action for the same. These undisputed facts negate private
respondents claim that she is an employee of petitioner.

Moreover, with regards (sic) to the control test, the public respondent NLRCs
ruling that the guidelines given by petitioner PDI for private respondent to
follow, e.g. in terms of space allocation and length of article, is not the form of
control envisioned by the guidelines set by the Supreme Court. The length of the
article is obviously limited so that all the articles to be featured in the paper can
be accommodated. As to the topic of the article to be published, it is but logical
that private respondent should not write morbid topics such as death because
she is contributing to the lifestyle section. Other than said given limitations, if the
same could be considered limitations, the topics of the articles submitted by
private respondent were all her choices. Thus, the petitioner PDI in deciding to

publish private respondents articles only controls the result of the work and not
the means by which said articles were written.
MR- Denied hence this Petition for review

ISSUES:
1. Whether or not the petition should be dismissed for failure to post appeal
Bond.
2. Whether or not the Petitioner is an employee of PDI and if yes is there illegal
dismissal

HELD:
1. NO, In a Resolution dated April 29, 2005, the Court, without giving due
course to the petition, ordered the Labor Arbiter to clarify the amount of
the award due petitioner and, thereafter, ordered PDI to post the
requisite bond. Upon compliance therewith, the petition would be given
due course. Labor Arbiter Amansec clarified that the award under the
Decision amounted to P15,350.00. Thus, PDI posted the requisite bond
on January 25, 2007.

While the posting of a cash or surety bond is jurisdictional and is a condition sine
qua non to the perfection of an appeal, there is a plethora of jurisprudence
recognizing exceptional instances wherein the Court relaxed the bond
requirement as a condition for posting the appeal.

In the case of NFLU v. Ladrido III, this Court postulated that "private respondents
cannot be expected to post such appeal bond equivalent to the amount of the
monetary award when the amount thereof was not included in the decision of
the labor arbiter." The computation of the amount awarded to petitioner not
having been clearly stated in the decision of the labor arbiter, private
respondents had no basis for determining the amount of the bond to be posted.
2.

NO, We rule for the respondents.

The existence of an employer-employee relationship is essentially a question of


fact. Factual findings of quasi-judicial agencies like the NLRC are generally

3E Andaya Ching Espiritu Hefti Galvez Gammad Lainez Lui Madamba Nagera Narvasa
Ong Palangdao Rosales Sanchez Santos Satrain Tabo (2014-2015)

Labor Relations Case Digest - Atty. Joyrich Golangco

accorded respect and finality if supported by substantial evidence. Considering,


however, that the CAs findings are in direct conflict with those of the Labor
Arbiter and NLRC, this Court must now make its own examination and evaluation
of the facts of this case.
The employment status of a person is defined and prescribed by law and not by
what the parties say it should be.

This Court has constantly adhered to the "four-fold test" to determine whether
there exists an employer-employee relationship between parties. The four
elements of an employment relationship are: (a) the selection and engagement
of the employee; (b) the payment of wages; (c) the power of dismissal; and
(d) the employers power to control the employees conduct.
Of these four elements, it is the power of control which is the most crucial and
most determinative factor, so important, in fact, that the other elements may
even be disregarded. the test is whether the employer controls or has
reserved the right to control the employee, not only as to the work done,
but also as to the means and methods by which the same is accomplished.
Petitioner has misconstrued the "control test," as did the Labor Arbiter and the
NLRC.

Not all rules imposed by the hiring party on the hired party indicate that the
latter is an employee of the former. Rules which serve as general guidelines
towards the achievement of the mutually desired result are not indicative of the
power of control.

Logically, the line should be drawn between rules that merely serve as guidelines
towards the achievement of the mutually desired result without dictating the
means or methods to be employed in attaining it, and those that control or fix the
methodology and bind or restrict the party hired to the use of such means. The
first, which aim only to promote the result, create no employer-employee
relationship unlike the second, which address both the result and the means
used to achieve it. x x x.

The main determinant therefore is whether the rules set by the employer
are meant to control not just the results of the work but also the means and
method to be used by the hired party in order to achieve such results. Thus,
in this case, we are to examine the factors enumerated by petitioner to see if
these are merely guidelines or if they indeed fulfill the requirements of the
control test.

Petitioner believes that respondents acts are meant to control how she executes
her work. We do not agree. A careful examination reveals that the factors
enumerated by the petitioner are inherent conditions in running a newspaper. In
other words, the so-called control as to time, space, and discipline are dictated by
the very nature of the newspaper business itself. We agree with the observations
of the Office of the Solicitor General that:
The Inquirer is the publisher of a newspaper of general circulation which is
widely read throughout the country. As such, public interest dictates that every
article appearing in the newspaper should subscribe to the standards set by the
Inquirer, with its thousands of readers in mind. It is not, therefore, unusual for
the Inquirer to control what would be published in the newspaper. What is
important is the fact that such control pertains only to the end result, i.e., the
submitted articles. The Inquirer has no control over [petitioner] as to the means
or method used by her in the preparation of her articles. The articles are done by
[petitioner] herself without any intervention from the Inquirer.
Petitioner has not shown that PDI, acting through its editors, dictated how she
was to write or produce her articles each week. Aside from the constraints
presented by the space allocation of her column, there were no restraints on her
creativity; petitioner was free to write her column in the manner and style she
was accustomed to and to use whatever research method she deemed suitable
for her purpose. The apparent limitation that she had to write only on subjects
that befitted the Lifestyle section did not translate to control, but was simply a
logical consequence of the fact that her column appeared in that section and
therefore had to cater to the preference of the readers of that section.
The newspapers power to approve or reject publication of any specific article
she wrote for her column cannot be the control contemplated in the "control

3E Andaya Ching Espiritu Hefti Galvez Gammad Lainez Lui Madamba Nagera Narvasa
Ong Palangdao Rosales Sanchez Santos Satrain Tabo (2014-2015)

Labor Relations Case Digest - Atty. Joyrich Golangco

test," as it is but logical that one who commissions another to do a piece of work
should have the right to accept or reject the product. The important factor to
consider in the "control test" is still the element of control over how the work
itself is done, not just the end result thereof.

Where a person who works for another performs his job more or less at his own
pleasure, in the manner he sees fit, not subject to definite hours or conditions of
work, and is compensated according to the result of his efforts and not the
amount thereof, no employer-employee relationship exists.

Aside from the control test, this Court has also used the economic reality test.
The economic realities prevailing within the activity or between the parties
are examined, taking into consideration the totality of circumstances
surrounding the true nature of the relationship between the parties. This is
especially appropriate when, as in this case, there is no written agreement or
contract on which to base the relationship. In our jurisdiction, the benchmark of
economic reality in analyzing possible employment relationships for purposes of
applying the Labor Code ought to be the economic dependence of the worker on
his employer.
Petitioners main occupation is not as a columnist for respondent but as a
womens rights advocate working in various womens organizations. Likewise,
she herself admits that she also contributes articles to other publications. Thus, it
cannot be said that petitioner was dependent on respondent PDI for her
continued employment in respondents line of business.
The inevitable conclusion is that petitioner was not respondent PDIs employee
but an independent contractor, engaged to do independent work.

The instant case presents a parallel to Sonza. Petitioner was engaged as a


columnist for her talent, skill, experience, and her unique viewpoint as a feminist
advocate. How she utilized all these in writing her column was not subject to
dictation by respondent. As in Sonza, respondent PDI was not involved in the
actual performance that produced the finished product. It only reserved the right
to shorten petitioners articles based on the newspapers capacity to
accommodate the same. This fact, we note, was not unique to petitioners

column. It is a reality in the newspaper business that space constraints often


dictate the length of articles and columns, even those that regularly appear
therein.

Furthermore, respondent PDI did not supply petitioner with the tools and
instrumentalities she needed to perform her work. Petitioner only needed her
talent and skill to come up with a column every week. As such, she had all the
tools she needed to perform her work.

Considering that respondent PDI was not petitioners employer, it cannot be held
guilty of illegal dismissal.
147. WILLIAM UY CONSTRUCTION CORP. vs. TRINIDAD

DOCTRINE: The repeated and successive rehiring of project employees do not


qualify them as regular employees, as length of service is not the controlling
determinant of the employment tenure of a project employee, but whether the
employment has been fixed for a specific project or undertaking, its completion has
been determined at the time of the engagement of the employee.
FACTS: Trinidad was a project employee of petitioner as driver for 16 years. He
was terminated allegedly due to lack of projects however, when petitioner
opened up a new project, Trinidad was not rehired. Trinidad filed a complaint for
illegal dismissal and unpaid benefits against petitioner. The Labor Arbiter
dismissed the complaint for unjust dismissal but ordered petitioner to pay
Trinidad P1,500 unpaid service incentive leave based on the 3-year prescriptive
period for money claims but found no basis for the grant of overtime pay, holiday
pay and 13th month pay. The NLRC affirmed the ruling of the Labor Arbiter.
Trinidad appealed. The CA reversed the NLRCs findings.
ISSUES: Whether or not the rehiring of a project employee for several years
automatically entitled him to the status of a regular employee?

HELD: NO. A project employee is distinguished from a regular employee


based on whether or not he has been assigned to carry out a "specific project or
undertaking," with the duration and scope of his engagement specified at the

3E Andaya Ching Espiritu Hefti Galvez Gammad Lainez Lui Madamba Nagera Narvasa
Ong Palangdao Rosales Sanchez Santos Satrain Tabo (2014-2015)

Labor Relations Case Digest - Atty. Joyrich Golangco

time his service is contracted. The length of service used in determining whether
an employee hired on a temporary basis becomes a permanent one is not
applicable to the construction industry since construction firms cannot
guarantee work for its employees beyond the life of each project considering that
the decisions and resources for a project lies on the proponents or owners of the
project. In the instant case, respondents service was contracted by petitioner by
specific projects setting out clearly the duration of his work in his employment
contracts. Thus, respondent remained a project employee regardless of the
number of years and the various projects he worked with petitioner; his series of
employments with petitioner were co-terminus with its projects. His case can be
properly addressed by the Social Security System which should alleviate the
temporary unemployment of construction workers, a problem that is inherent in
the nature of their work.
The Court granted the petition; set aside the decision of the CA and reinstated the
decision of the NLRC affirming the decision of the Labor Arbiter.
148: DM Consunji Inc. (DMCI) vs Jamin

DOCTRINE: The rule that the length of service of a project employee is not the
controlling test of employment tenure but whether or not the employment has
been fixed for a specific project or undertaking the completion or termination of
which has been determined at the time of the engagement of the employee.

Surely, length of time is not the controlling test for project employment.
Nevertheless, it is vital in determining if the employee was hired for a specific
undertaking or tasked to perform functions vital, necessary and indispensable to
the usual business or trade of the employer. Here, respondent had been a project
employee several times over. His employment ceased to be coterminous with
specific projects when he was repeatedly re-hired due to the demands of
petitioners business.
FACTS: Jamin was hired as a laborer and later on became a helper carpenter by
petitioner in its several construction projects. His work spans from 1968 to 1999
(31 years), his last construction project was the construction of SM Manila that
marked his termination and was never re-hired again. Jamin filed a complaint for
illegal dismissal and money claims against DMCI and David Consunji alleging

that such termination was without just cause and that being already 55 yrs old
he had no steady source of income.

DMCI alleged that Jamin was hired on a project-to-project basis. And


every time a project is completed DMCI submits a report to the DOLE of its
employees termination. More so, as evidenced by the project schedules
submitted by DMCI there were gaps in the span of 31 years where Jamin was not
hired for a project and it even went as long as 4 years without rehiring him and
in essence concluding that he is a project employee. Simply stated, DMCI denied
liability for any claim by Jamin. The matter was submitted for Compulsory
Arbitration.

LA: sustained DMCI. Jamin is a project employee. Upon completion of SM Manila,


a notice of termination was served to Jamin and a termination report to DOLERegional Office. LA noted that if Jamin wants to be re-hired he should file an
application.
NLRC: affirmed in toto LAs ruling.

CA: Reversed the compulsory arbitration rulings. Yap is a regular employee on


the grounds that (1) Jamins repeated and successive rehiring in various projects;
and (2) the nature of his work - he was performing activities necessary or
desirable in DMCIs construction business, a key indicator of regular
employment. In this regard, his employment status ripened into regular
employment. In addition, Jamin was part of a work pool making him a regular
employee. The project employment contracts entered into between Jamin and
DMCI are NOT always conclusive of a workers employment status as held in
Liganza vs RBL Corp. Contrary to DMCIs position, there was a failure to submit a
report to DOLE every time Jamin was terminated for a project. CA declared
Jamins disimissal illegal and ordered his reinstatement with backwages and
without loss of seniority.
DMCI: I dont maintain a work pool. It filed MR but was denied.

Note: CA decided on Feb. 26, 2010 and received by DMCI on March 4. The MR
was filed on March 19 or 3 days after the lapse of the 15-day reglementary
period. Hence, when DMCI elevated the case to SC, Jamin outrightly prayed in his

3E Andaya Ching Espiritu Hefti Galvez Gammad Lainez Lui Madamba Nagera Narvasa
Ong Palangdao Rosales Sanchez Santos Satrain Tabo (2014-2015)

Labor Relations Case Digest - Atty. Joyrich Golangco

Comment for its dismissal claiming that CAs decision already became final and
executory.
ISSUES: Whether or not Jamin was a regular employee.
HELD: Affirmative. He is a regular employee.

Jamins employment history with DMCI stands out for his continuous,
repeated and successive rehiring in the companys construction projects. In all
the 38 projects where DMCI engaged Jamins services, the tasks he performed as
a carpenter were indisputably necessary and desirable in DMCIs construction
business. Even if DMCI had no work pool, Jamin still became a regular employee
because of continued rehiring. SC noted Liganza case [refer to DOCTRINE].
As regards the termination reports - To set the records straight, DMCI
indeed submitted reports to the DOLE but as pointed out by Jamin, the
submissions started only in 1992. DMCI explained that it submitted the earlier
reports (1982), but it lost and never recovered the reports. It reconstituted the
lost reports and submitted them to the DOLE in October 1992; thus, the dates
appearing in the reports.
SC absolved David Consunji (president of DMCI) hindi naman raw
napag-usapan yung liability niya sa LA, NLRC at CA.
We Affirm CAs ruling in toto.

149. ARO ET AL. VS NLRC

DOCTRINE: The principal test for determining whether particular employees are
properly characterized as "project employees" as distinguished from "regular
employees" is whether or not the project employees were assigned to carry out a
"specific project or undertaking," the duration and scope of which were specified
at the time the employees were engaged for that project.
The length of service or the re-hiring of construction workers on a project-toproject basis does not confer upon them regular employment status, since their
re-hiring is only a natural consequence of the fact that experienced construction
workers are preferred. Employees who are hired for carrying out a separate job,
distinct from the other undertakings of the company, the scope and duration of

which has been determined and made known to the employees at the time of the
employment , are properly treated as project employees and their services may
be lawfully terminated upon the completion of a project. Should the terms of
their employment fail to comply with this standard, they cannot be
considered project employees.
FACTS: Several employees of private respondent Benthel Development
Corporation, including the petitioners, filed a Complaint for illegal dismissal with
various money claims and prayer for damages against the latter, in the NLRC
Arbitration Branch No. VII in Cebu . Thereafter, Labor Arbiter Carreon rendered
a decision finding private respondent guilty of illegal dismissal and ordering it to
pay its thirty-six (36) employees P446,940.00 as separation pay.

The employees, including the petitioners herein, appealed from the said decision
in the NLRC which affirmed the decision of Labor Arbiter Carreon in its Decision
dated January 12, 1999, with the modification that private respondent pay
backwages computed from the respective dates of dismissal until finality of the
decision.
Private respondent, unsatisfied with the modification made by the NLRC, filed a
motion for reconsideration with the contention that, since it has been found by
the Labor Arbiter and affirmed in the assailed decision that the employees were
project employees, the computation of backwages should be limited to the date
of the completion of the project and not to the finality of the decision. The NLRC,
however, denied the motion ruling that private respondent failed to establish the
date of the completion of the project.
ISSUES: Whether herein petitioners are project employees and therefore entitled only to full
backwages, computed from the date of the termination of their employment until the actual
completion of the work, not up to the finality of the decision.

HELD: It is well-settled in jurisprudence that factual findings of administrative or


quasi-judicial bodies, which are deemed to have acquired expertise in matters
within their respective jurisdictions, are generally accorded not only respect but
even finality, and bind the Court when supported by substantial evidence. This

3E Andaya Ching Espiritu Hefti Galvez Gammad Lainez Lui Madamba Nagera Narvasa
Ong Palangdao Rosales Sanchez Santos Satrain Tabo (2014-2015)

Labor Relations Case Digest - Atty. Joyrich Golangco

Court may take cognizance of and resolve factual issues, only when the findings
of fact and conclusions of law of the Labor Arbiter or the NLRC are inconsistent
with those of the CA. In the present case, the NLRC and the CA have opposing
views.

According to the CA, petitioners are project employees as found by Labor Arbiter
because they were hired for the construction of the Cordova Reef Village Resort
in Cordova, Cebu, which was later on affirmed by the NLRC in its January 12,
1999 decision. The only discrepancy is the Order of the NLRC that petitioners
are entitled to backwages up to the finality of its decision, when as project
employees, private respondents are only entitled to payment of backwages until
the date of the completion of the project. In a later resolution on private
respondent's motion for reconsideration of its January 12, 1999 decision, the
NLRC changed its findings by ruling that petitioners herein were regular
employees and, therefore, entitled to full backwages, until finality of the decision,
citing that petitioners repeated rehiring over a long span of time made them
regular employees.
Article 280 of the Labor Code distinguishes a "project employee" from a
"regular employee," thus:
Article 280. Regular and Casual Employment The provisions of
written agreement to the contrary notwithstanding and
regardless of the oral agreement of the parties, an employment
shall be deemed to be regular where the employee has been
engaged to perform activities which are usually necessary or
desirable in the usual business or trade of the employer, except
where the employment has been fixed for a specific project or
undertaking the completion or termination of which has been
determined at the time of the engagement of the employee or
where the work or service to be performed is seasonal in nature
and the employment is for the duration of the season.

An employment shall be deemed to be casual if it is not covered


by the preceding paragraph: Provided, That, any employee who
has rendered at least one year service, whether such service is

continuous or broken, shall be considered a regular employee


with respect to the activity in which he is employed and his
employment shall continue while such activity exists.

In Hanjin Heavy Industries and Construction Co. Ltd. v. Ibaez, the principal test
for determining whether particular employees are properly characterized as
"project employees" as distinguished from "regular employees" is whether or
not the project employees were assigned to carry out a "specific project or
undertaking," the duration and scope of which were specified at the time the
employees were engaged for that project.
This Court agrees with the findings of the CA that petitioners were project
employees. It is not disputed that petitioners were hired for the construction of
the Cordova Reef Village Resort in Cordova, Cebu. By the nature of the contract
alone, it is clear that petitioners' employment was to carry out a specific
project. Hence, the CA did not commit grave abuse of discretion when it affirmed
the findings of the Labor Arbiter.Therefore, being project employees, petitioners
are only entitled to full backwages, computed from the date of the termination of
their employment until the actual completion of the work.
150. Universal Robina Sugar Milling Corp. v. Acibo (2014)

DOCTRINE: the nature of the employment does not depend solely on the will or
word of the employer or on the procedure for hiring and the manner of
designating the employee. Rather, the nature of the employment depends on the
nature of the activities to be performed by the employee, considering the nature
of the employers business, the duration and scope to be done, and, in some
cases, even the length of time of the performance and its continued existence.

FACTS: The complainants were employees of URSUMCO. They were hired on


various dates (between February 1988 and April 1996) and on different
capacities,8 i.e., drivers, crane operators, bucket hookers, welders, mechanics,
laboratory attendants and aides, steel workers, laborers, carpenters and masons,
among others. At the start of their respective engagements, the complainants

3E Andaya Ching Espiritu Hefti Galvez Gammad Lainez Lui Madamba Nagera Narvasa
Ong Palangdao Rosales Sanchez Santos Satrain Tabo (2014-2015)

Labor Relations Case Digest - Atty. Joyrich Golangco

signed contracts of employment for a period of one (1) month or for a given
season. URSUMCO repeatedly hired the complainants to perform the same duties
and, for every engagement, required the latter to sign new employment contracts
for the same duration of one month or a given season. Subsequently,
complainants filed before the LA complaints for regularization, entitlement to the
benefits under the existing Collective Bargaining Agreement (CBA),and
attorneys fees.
URSUMCO contends: (1) that the employees are contractual or project/seasonal
workers and not regular employees; (2) that the legal standards length of the
employees engagement and the desirability or necessity of the employees work
in the usual trade or business of the employer apply only to regular employees
under paragraph 1, Article 280 of the Labor Code, and, under paragraph 2 of the
same article, to casual employees who are deemed regular by their length of
service; (3) that the employees were specifically engaged for a fixed and
predetermined duration of, on the average, 1 month at a time that coincides with
a particular phase of the companys business operations or sugar milling season;
(4) that by the nature of their engagement, the employees employment legally
ends upon the end of the predetermined period; thus, URSUMCO was under no
legal obligation to rehire them.
The employees maintain that they are regular employees of URSUMCO. They
point out that they have been continuously working for URSUMCO for more than
one year, performing tasks which were necessary and desirable to URSUMCOs
business. Hence, under the above-stated legal parameters, they are regular
employees.
The NLRC declared the complainants as regular URSUMCO employees and
granted their monetary claims under the CBA. The NLRC pointed out that the
complainants performed activities which were usually necessary and desirable
in the usual trade or business of URSUMCO, and had been repeatedly hired for
the same undertaking every season. CA affirmed.
ISSUES: whether the respondents are regular employees of URSUMCO
HELD: Respondents are regular seasonal employees of URSUMCO.

Article 280 of the Labor Code provides for three kinds of employment
arrangements, namely: regular, project/seasonal and casual. Regular
employment refers to that arrangement whereby the employee has been
engaged to perform activities which are usually necessary or desirable in the
usual business or trade of the employer. Under the definition, the primary
standard that determines regular employment is the reasonable connection
between the particular activity performed by the employee and the usual
business or trade of the employer; the emphasis is on the necessity or desirability
of the employees activity. Thus, when the employee performs activities
considered necessary and desirable to the overall business scheme of the
employer, the law regards the employee as regular. By way of an exception,
paragraph 2, Article 280 of the Labor Code also considers regular a casual
employment arrangement when the casual employees engagement has lasted
for at least one year, regardless of the engagements continuity. The controlling
test in this arrangement is the length of time during which the employee is
engaged.
A project employment, on the other hand, contemplates on arrangement
whereby the employment has been fixed for a specific project or undertaking
whose completion or termination has been determined at the time of the
engagement of the employee. Two requirements, therefore, clearly need to be
satisfied to remove the engagement from the presumption of regularity of
employment, namely: (1) designation of a specific project or undertaking for
which the employee is hired; and (2) clear determination of the completion or
termination of the project at the time of the employees engagement. The
services of the project employees are legally and automatically terminated upon
the end or completion of the project as the employees services are coterminous
with the project.

Unlike in a regular employment under Article 280 of the Labor Code, however,
the length of time of the asserted "project" employees engagement is not
controlling as the employment may, in fact, last for more than a year, depending
on the needs or circumstances of the project. Nevertheless, this length of time (or
the continuous rehiring of the employee even after the cessation of the project)
may serve as a badge of regular employment when the activities performed by
the purported "project" employee are necessary and indispensable to the usual

3E Andaya Ching Espiritu Hefti Galvez Gammad Lainez Lui Madamba Nagera Narvasa
Ong Palangdao Rosales Sanchez Santos Satrain Tabo (2014-2015)

Labor Relations Case Digest - Atty. Joyrich Golangco

business or trade of the employer.23 In this latter case, the law will regard the
arrangement as regular employment.

Seasonal employment operates much in the same way as project employment,


albeit it involves work or service that is seasonal in nature or lasting for the
duration of the season.25 As with project employment, although the seasonal
employment arrangement involves work that is seasonal or periodic in nature,
the employment itself is not automatically considered seasonal so as to prevent
the employee from attaining regular status. To exclude the asserted "seasonal"
employee from those classified as regular employees, the employer must show
that: (1) the employee must be performing work or services that are seasonal in
nature; and (2) he had been employed for the duration of the season.26 Hence,
when the "seasonal" workers are continuously and repeatedly hired to perform
the same tasks or activities for several seasons or even after the cessation of the
season, this length of time may likewise serve as badge of regular
employment.27 In fact, even though denominated as "seasonal workers," if these
workers are called to work from time to time and are only temporarily laid off
during the off-season, the law does not consider them separated from the service
during the off-season period. The law simply considers these seasonal workers
on leave until re-employed.28
Casual employment, the third kind of employment arrangement, refers to any
other employment arrangement that does not fall under any of the first two
categories, i.e., regular or project/seasonal.

The Labor Code does not mention another employment arrangement


contractual or fixed term employment (or employment for a term) which, if not
for the fixed term, should fall under the category of regular employment in view
of the nature of the employees engagement, which is to perform an activity
usually necessary or desirable in the employers business.

The Court, for the first time, recognized and resolved the anomaly created by a
narrow and literal interpretation of Article 280 of the Labor Code that appears to
restrict the employees right to freely stipulate with his employer on the duration
of his engagement. In this case, the Court upheld the validity of the fixed-term
employment agreed upon by the employer, Brent School, Inc., and the employee,

Dorotio Alegre, declaring that the restrictive clause in Article 280 "should be
construed to refer to the substantive evil that the Code itself x x x singled out:
agreements entered into precisely to circumvent security of tenure. It should
have no application to instances where [the] fixed period of employment was
agreed upon knowingly and voluntarily by the parties x x x absent any x x x
circumstances vitiating [the employees] consent, or where [the facts
satisfactorily show] that the employer and [the] employee dealt with each other
on more or less equal terms[.]"30 The indispensability or desirability of the
activity performed by the employee will not preclude the parties from entering
into an otherwise valid fixed term employment agreement; a definite period of
employment does not essentially contradict the nature of the employees
duties31 as necessary and desirable to the usual business or trade of the
employer.

Nevertheless, "where the circumstances evidently show that the employer


imposed the period precisely to preclude the employee from acquiring tenurial
security, the law and this Court will not hesitate to strike down or disregard the
period as contrary to public policy, morals, etc."32 In such a case, the general
restrictive rule under Article 280 of the Labor Code will apply and the employee
shall be deemed regular.

Clearly, therefore, the nature of the employment does not depend solely on the
will or word of the employer or on the procedure for hiring and the manner of
designating the employee. Rather, the nature of the employment depends on the
nature of the activities to be performed by the employee, considering the nature
of the employers business, the duration and scope to be done,33 and, in some
cases, even the length of time of the performance and its continued existence.
In light of the above legal parameters laid down by the law and applicable
jurisprudence, the respondents are neither project, seasonal nor fixed-term
employees, but regular seasonal workers of URSUMCO. The following factual
considerations from the records support this conclusion:
1.

the employees were made to perform various tasks that did not at all
pertain to any specific phase of URSUMCOs strict milling operations that
would ultimately cease upon completion of a particular phase in the

3E Andaya Ching Espiritu Hefti Galvez Gammad Lainez Lui Madamba Nagera Narvasa
Ong Palangdao Rosales Sanchez Santos Satrain Tabo (2014-2015)

Labor Relations Case Digest - Atty. Joyrich Golangco

2.

3.

4.

milling of sugar; rather, they were tasked to perform duties regularly


and habitually needed in URSUMCOs operations during the milling
season. The respondents duties as loader operators, hookers, crane
operators and drivers were necessary to haul and transport the
sugarcane from the plantation to the mill; welders, carpenters and utility
workers to ensure the smooth and continuous operation of the mill for
the duration of the milling season, as distinguished from the production
of the sugarcane which involves the planting and raising of the
sugarcane until it ripens for milling. They perform the activities that are
necessary and desirable in sugarcane production. As in the milling of
sugarcane, the plantation workers perform their duties only during the
planting season.
they were regularly and repeatedly hired to perform the same tasks year
after year. This regular and repeated hiring of the same workers (two
different sets) for two separate seasons has put in place, principally
through jurisprudence, the system of regular seasonal employment in
the sugar industry and other industries with a similar nature of
operations.
Under the system, the plantation workers or the mill employees do not
work continuously for one whole year but only for the duration of the
growing of the sugarcane or the milling season. Their seasonal work,
however, does not detract from considering them in regular
employment since in a litany of cases, this Court has already settled that
seasonal workers who are called to work from time to time and are
temporarily laid off during the off-season are not separated from the
service in said period, but are merely considered on leave until reemployment.34 Be this as it may, regular seasonal employees, like the
respondents in this case, should not be confused with the regular
employees of the sugar mill such as the administrative or office
personnel who perform their tasks for the entire year regardless of the
season. The NLRC, therefore, gravely erred when it declared the
respondents regular employees of URSUMCO without qualification and
that they were entitled to the benefits granted, under the CBA, to
URSUMCOS regular employees.
while the URSUMCO assert that the respondents were free to work
elsewhere during the off-season, the records do not support this

assertion. There is no evidence on record showing that after the


completion of their tasks at URSUMCO, the respondents sought and
obtained employment elsewhere

151. GMA NETWORK, INC., Petitioner, vs. CARLOS P. PABRIGA, GEOFFREY F.


ARIAS, KIRBY N. CAMPO, ARNOLD L. LAGAHIT, and ARMANDO A. CATUBIG,
Respondents.

DOCTRINES: In order to safeguard the rights of workers against the arbitrary


use of the word "project" to prevent employees from attaining the status of
regular employees, employers claiming that their workers are project employees
should not only prove that the duration and scope of the employment was
specified at the time they were engaged, but also that there was indeed a project.

The failure of an employer to report to the nearest Public Employment Office the
termination of its workers services every time a project or a phase thereof is
completed indicates that said workers are not project employees.
It is unjustifiable to allow petitioner to hire and rehire workers on fixed terms, ad
infinitum, depending upon its needs, never attaining regular employment status.

FACTS: On July 19 1999 due to the miserable working conditions private


respondents were forced to file a complaint against petitioner before the
National Labor Relations Commission Regional Arbitration Branch No. VII Cebu
City.
Private respondents were hired as television technicians (Pabriga and Arias-on
May 2, 1997, Campo on Dec 1, 1993, Laganit on Feb 11, 1996 and Catubig on
March 2, 1997) and engaged by petitioner to perform the following activities, to
wit:
1) Manning of Technical Operations Center:

(a) Responsible for the airing of local commercials; and

(b) Logging/monitoring of national commercials (satellite)

3E Andaya Ching Espiritu Hefti Galvez Gammad Lainez Lui Madamba Nagera Narvasa
Ong Palangdao Rosales Sanchez Santos Satrain Tabo (2014-2015)

2) Acting as Transmitter/VTR men:

Labor Relations Case Digest - Atty. Joyrich Golangco

(a) Prepare tapes for local airing;


(b) Actual airing of commercials;
(c) Plugging of station promo;

(d) Logging of transmitter reading; and

(e) In case of power failure, start up generator set to resume program;


3) Acting as Maintenance staff;

1) Unfair Labor Practice; 2) Illegal dismissal; and 3) Damages and Attorneys


fees.

LA: dismissed the complaint of respondents for illegal dismissal and unfair labor
practice, but held petitioner liable for 13th month pay.
NLRC: reversed the LAs decision, finding that complainants are regular
employees and are thus entitled to separation pay, 13th month pay, night shift
differential and SIL (but not overtime and holiday pay), plus attys fees of 10% of
the awards
CA: denied petitioners petition for certiorari for lack of merit.
Thus the present Petition for Review on Certiorari

(a) Checking of equipment;

ISSUES:

(b) Warming up of generator;

(c) Filling of oil, fuel, and water in radiator; and


4) Acting as Cameramen

1. Whether respondents are regular or project employees

2. Whether the requisites of a valid fixed term employment are met


HELD:

On 4 August 1999, petitioner received a notice of hearing of the complaint. The


next day, petitioners Engineering Manager, Roy Villacastin, confronted the
private respondents about the said complaint.

On 9 August 1999, private respondents were summoned to the office of


petitioners Area Manager, Susan Alio, to explain why they filed the complaint.
The next day, private respondents were barred from entering and reporting for
work without any notice stating the reasons therefor.
Private respondents, through their counsel, wrote a letter to Alio requesting
that they be recalled back to work. Bienvenido Bustria, petitioners head of
Personnel and Labor Relations Division, sent a reply letter, admitting the nonpayment of benefits. Their request to work was ignored. Thus, private
respondents filed an amended complaint raising the following additional issues:

1. THEY ARE REGULAR EMPLOYEES.

In order to safeguard the rights of workers against the arbitrary use of the word
"project" to prevent employees from attaining the status of regular employees,
employers claiming that their workers are project employees should not only
prove that the duration and scope of the employment was specified at the time
they were engaged, but also that there was indeed a project. The project could
either be (1) a particular job or undertaking that is within the regular or usual
business of the employer company, but which is distinct and separate, and
identifiable as such, from the other undertakings of the company; or (2) a
particular job or undertaking that is not within the regular business of the
corporation. As it was with regard to the distinction between a regular and
casual employee, the purpose of this requirement is to delineate whether or not

3E Andaya Ching Espiritu Hefti Galvez Gammad Lainez Lui Madamba Nagera Narvasa
Ong Palangdao Rosales Sanchez Santos Satrain Tabo (2014-2015)

Labor Relations Case Digest - Atty. Joyrich Golangco

the employer is in constant need of the services of the specified employee. If the
particular job or undertaking is within the regular or usual business of the
employer company and it is not identifiably distinct or separate from the other
undertakings of the company, there is clearly a constant necessity for the
performance of the task in question, and therefore said job or undertaking
should not be considered a project.
The jobs and undertakings of respondents are clearly within the regular or usual
business of the employer company and are not identifiably distinct or separate
from the other undertakings of the company. There is no denying that the
manning of the operations center to air commercials, acting as transmitter/VTR
men, maintaining the equipment, and acting as cameramen are not undertakings
separate or distinct from the business of a broadcasting company.

Petitioners allegation that respondents were merely substitutes or what they


call pinch-hitters (which means that they were employed to take the place of
regular employees of petitioner who were absent or on leave) does not change
the fact that their jobs cannot be considered projects within the purview of the
law. Every industry, even public offices, has to deal with securing substitutes for
employees who are absent or on leave. Such tasks, whether performed by the
usual employee or by a substitute, cannot be considered separate and distinct
from the other undertakings of the company. While it is managements
prerogative to device a method to deal with this issue, such prerogative is not
absolute and is limited to systems wherein employees are not ingeniously and
methodically deprived of their constitutionally protected right to security of
tenure. We are not convinced that a big corporation such as petitioner cannot
device a system wherein a sufficient number of technicians can be hired with a
regular status who can take over when their colleagues are absent or on leave,
especially when it appears from the records that petitioner hires so-called pinchhitters regularly every month.
In affirming the Decision of the NLRC, the CA furthermore noted that if
respondents were indeed project employees, petitioner should have reported the
completion of its projects and the dismissal of respondents in its finished
projects.

Petitioners theory of project employment fails the principal test of


demonstrating that the alleged project employee was assigned to carry out a
specific project or undertaking, the duration and scope of which were specified
at the time the employee is engaged for the project.20
The CA also ruled that even if it is assumed that respondents are project
employees, they would nevertheless have attained regular employment status
because of their continuous rehiring:

Be that as it may, a project employee may also attain the status of a regular
employee if there is a continuous rehiring of project employees after the
stoppage of a project; and the activities performed are usual [and] customary to
the business or trade of the employer. The Supreme Court ruled that a project
employee or a member of a work pool may acquire the status of a regular
employee when the following concur:
1) There is a continuous rehiring of project employees even after cessation of a
project; and
2) The tasks performed by the alleged project employee are vital, necessary and
indispensable to the usual business or trade of the employer.

The circumstances set forth by law and the jurisprudence is present in this case.
In fine, even if private respondents are to be considered as project employees,
they attained regular employment status, just the same.

2. NO. Petitioner interchangeably characterizes respondents service as project


and fixed term employment. These types of employment, however, are not the
same. While the former requires a project, the duration of a fixed-term
employment agreed upon by the parties may be any day certain, which is
understood to be "that which must necessarily come although it may not be
known when."25 The decisive determinant in fixed-term employment is not the
activity that the employee is called upon to perform but the day certain agreed
upon by the parties for the commencement and termination of the employment
relationship.26

3E Andaya Ching Espiritu Hefti Galvez Gammad Lainez Lui Madamba Nagera Narvasa
Ong Palangdao Rosales Sanchez Santos Satrain Tabo (2014-2015)

Labor Relations Case Digest - Atty. Joyrich Golangco

Cognizant of the possibility of abuse in the utilization of fixed-term employment


contracts, we emphasized in Brent that where from the circumstances it is
apparent that the periods have been imposed to preclude acquisition of tenurial
security by the employee, they should be struck down as contrary to public
policy or morals.27 We thus laid down indications or criteria under which "term
employment" cannot be said to be in circumvention of the law on security of
tenure, namely:
1) The fixed period of employment was knowingly and voluntarily agreed upon
by the parties without any force, duress, or improper pressure being brought to
bear upon the employee and absent any other circumstances vitiating his
consent; or
2) It satisfactorily appears that the employer and the employee dealt with each
other on more or less equal terms with no moral dominance exercised by the
former or the latter.28 (Citation omitted.)

These indications, which must be read together, make the Brent doctrine
applicable only in a few special cases wherein the employer and employee are on
more or less in equal footing in entering into the contract. The reason for this is
evident: when a prospective employee, on account of special skills or market
forces, is in a position to make demands upon the prospective employer, such
prospective employee needs less protection than the ordinary worker.

In the case at bar, we find it unjustifiable to allow petitioner to hire and rehire
workers on fixed terms, ad infinitum, depending upon its needs, never attaining
regular employment status. To recall, respondents were repeatedly rehired in
several fixed term contracts from 1996 to 1999. To prove the alleged contracts,
petitioner presented cash disbursement vouchers signed by respondents, stating
that they were merely hired as pinch-hitters. It is apparent that respondents
were in no position to refuse to sign these vouchers, as such refusal would entail
not getting paid for their services. Plainly, respondents as "pinch-hitters" cannot
be considered to be in equal footing as petitioner corporation in the negotiation
of their employment contract.

In sum, we affirm the findings of the NLRC and the Court of Appeals that
respondents are regular employees of petitioner.1wphi1As regular employees,
they are entitled to security of tenure and therefore their services may be
terminated only for just or authorized causes. Since petitioner failed to prove any
just or authorized cause for their termination, we are constrained to affirm the
findings of the NLRC and the CA that they were illegally dismissed.
152. ROY D. P ASOS, Petitioner, vs. PHILIPPINE NATIONAL CONSTRUCTION
CORPORATION, Respondent.

DOCTRINE: Under Article 280 of the Labor Code, as amended, a project


employee is one whose "employment has been fixed for a specific project or
undertaking the completion or termination of which has been determined at the
time of the engagement of the employee or where the work or services to be
performed is seasonal in nature and the employment is for the duration of the
season." Thus, the principal test used to determine whether employees are
project employees is whether or not the employees were assigned to carry out a
specific project or undertaking, the duration or scope of which was specified at
the time the employees were engaged for that project.

FACTS: Petitioner Roy D. Pasos started working for respondent PNCC on April
26, 1996. Petitioner was designated as "Clerk II (Accounting)" and was assigned
to the "NAIA II Project." The contract would end July 25, 1996. Petitioners
employment, however, extended until August 4, 1998. Petitioner was rehired on
even date as "Accounting Clerk (Reliever)" and assigned to the "PCSO Q.I.
Project." It was stated therein that his employment shall end on February 11,
1999. However, said employment did not actually end on February 11, 1999 but
was extended until February 19, 1999.

On February 23, 1999, petitioner was again hired by PNCC as "Accounting Clerk"
and was assigned to the "SM-Project". It did not specify the date when his
employment will end but it was stated therein that it will be "co-terminus with
the completion of the project." Said employment supposedly ended on August 19,
1999. However, it appears that said employment was extended as petitioner was
again appointed as "Accounting Clerk" for "SM Project (Package II)." It did not
state a specific date up to when his extended employment will be, but it provided

3E Andaya Ching Espiritu Hefti Galvez Gammad Lainez Lui Madamba Nagera Narvasa
Ong Palangdao Rosales Sanchez Santos Satrain Tabo (2014-2015)

Labor Relations Case Digest - Atty. Joyrich Golangco

that it will be "co-terminus with the x x x project." It appears that such extension
would eventually end on October 19, 2000.

Despite the termination of his employment on October 19, 2000, petitioner


claims that his superior instructed him to report for work the following day,
intimating to him that he will again be employed for the succeeding SM projects.
For purposes of reemployment, he then underwent a medical examination which
allegedly revealed that he had pneumonitis. Petitioner was advised by PNCCs
physician, Dr. Arthur C. Obena, to take a 14-day sick leave. On November 27,
2000, after serving his sick leave, petitioner claims that he was again referred for
medical examination where it was revealed that he contracted Kochs disease. He
was then required to take a 60-day leave of absence. The following day, he
submitted his application for sick leave but PNCCs Project Personnel Officer, Mr.
R.S. Sanchez, told him that he was not entitled to sick leave because he was not a
regular employee. Petitioner still served a 60-day sick leave and underwent
another medical examination on February 16, 2001. He was then given a clean
bill of health and was given a medical clearance by Dr. Obena that he was fit to
work.

Petitioner claims that after he presented his medical clearance to the Project
Personnel Officer on even date, he was informed that his services were already
terminated on October 19, 2000 and he was already replaced due to expiration of
his contract. This prompted petitioner on February 18, 2003 to file a complaint
for illegal dismissal against PNCC with a prayer for reinstatement and back
wages.
[LA: granted; NLRC: reversed LA; CA: dismissed (lack of merit)]

ISSUES:
1.Whether Pasos was a regular employee
2. Whether the dismissal was illegal

HELD: 1. PASOS was a REGULAR EMPLOYEE

In the instant case, the appointments issued to petitioner indicated that he was
hired for specific projects. This Court is convinced however that although he
started as a project employee, he eventually became a regular employee of PNCC.

Under Article 280 of the Labor Code, as amended, a project employee is one
whose "employment has been fixed for a specific project or undertaking the
completion or termination of which has been determined at the time of the
engagement of the employee or where the work or services to be performed is
seasonal in nature and the employment is for the duration of the season." Thus,
the principal test used to determine whether employees are project employees is
whether or not the employees were assigned to carry out a specific project or
undertaking, the duration or scope of which was specified at the time the
employees were engaged for that project.

In the case at bar, petitioner worked continuously for more than two years after
the supposed three-month duration of his project employment for the NAIA II
Project. While his appointment for said project allowed such extension since it
specifically provided that in case his "services are still needed beyond the
validity of the contract, the Company shall extend his services," there was no
subsequent contract or appointment that specified a particular duration for the
extension. His services were just extended indefinitely until "Personnel Action
Form Project Employment" dated July 7, 1998 was issued to him which
provided that his employment will end a few weeks later or on August 4, 1998.
While for first three months, petitioner can be considered a project employee of
PNCC, his employment thereafter, when his services were extended without any
specification of as to the duration, made him a regular employee of PNCC. And his
status as a regular employee was not affected by the fact that he was assigned to
several other projects and there were intervals in between said projects since he
enjoys security of tenure.
Moreover, if private respondents were indeed employed as "project employees,"
petitioners should have submitted a report of termination to the nearest public
employment office every time their employment was terminated due to
completion of each construction project. The records show that they did not.
2. PASOS ILLEGALLY DISMISSED

Petitioners regular employment was terminated by PNCC due to contract


expiration or project completion, which are both not among the just or
authorized causes provided in the Labor Code, as amended, for dismissing a
regular employee. Thus, petitioner was illegally dismissed.

3E Andaya Ching Espiritu Hefti Galvez Gammad Lainez Lui Madamba Nagera Narvasa
Ong Palangdao Rosales Sanchez Santos Satrain Tabo (2014-2015)

Labor Relations Case Digest - Atty. Joyrich Golangco

153. GAPAYAO VS FULO

FACTS: On 4 November 1097, Jaime Fulo (deceased) died of "acute renal failure
secondary to 1st degree burn secondary electrocution while doing repairs at
the residence and business establishment or petitioner.
Allegedly moved by his Christian faith, petitioner extended some financial
assistance to private respondent. On 16 November 1997, the latter executed an
Affidavit of Desistance liable for the death of her late husband, Jaime Fulo, and
was thereby waiving her right and desisting from filing any criminal or civil
action against petitioner. On 14 January 1998, both parties executed a
Compromise Agreement.

Thereafter, private respondent filed a claim for social security benefits with the
Social Security System (SSS)verification and evaluation, it was discovered that
the deceased was not a registered member of the SSS. Upon the insistence of
private respondent that her late husband had been employed by petitioner from
January 1983 up to his untimely death on 4 November 1997, the SSS conducted a
field investigation to clarify his status of employment.

Consequently, the SSS demanded that petitioner remit the social security
contributions of the deceased. When petitioner denied that thedeceased was his
employee, the SSS required private respondent to presentdocumentary and
testimonial evidence to refute petitioners allegations. Instead of presenting
evidence, private respondent filed a Petition before the SSC on 17 February 2003.
In her Petition, she sought social security coverage and payment of contributions
in order to avail herself of the benefits accruing from the death of her husband.
On 6 May 2003, petitioner filed an Answer13 disclaiming any liabilityon the
premise that the deceased was not the formers employee, but was rather an
independent contractor whose tasks were not subject to petitioners control and
supervision.14 Assuming arguendo that the deceased was petitioners employee,
he was still not entitled to be paid his SSS premiums for the intervening period
when he was not at work, as he was an intermittent worker who [was] only
summoned every now and then as the need [arose].15 Hence, petitioner insisted
that he was under no obligation to report the formers demise to the SSS for
social security coverage.

Subsequently, on 30 June 2003, the SSS filed a Petition-in-Intervention16 before


the SSC, outlining the factual circumstances of the case and praying that
judgment be rendered based on the evidence adduced by the parties.

The SSS is hereby directed to pay petitioner Rosario Fulo theappropriate death
benefit, pursuant to Section 13 of the SS Law, as amended, as well as its
prevailing rules and regulations, and to inform this Commission of its compliance
herewith.,On 18 May 2007, petitioner filed a Motion for Reconsideration,which
was denied. Aggrieved, petitioner appealed to the CA .CA rendered a Decision in
favor of private respondent
ISSUE presented before us is whether or not there exists between the deceased
Jaime Fulo and petitioner an employer-employee relationship that would merit
an award of benefits in favor of private respondent under social security laws.

HELD: YES, Farm workers may be considered regular seasonal employees.


Jurisprudence has identified the three types of employees mentioned in the
provision: (1) regular employees or those who have been engaged to perform
activities that are usually necessary or desirable in the usual business or trade of
the employer; (2) project employees or those whose employment has been fixed
for a specific project or undertaking, the completion or termination of which has
been determined at the time of their engagement, or those whose work or
service is seasonal in nature and is performed for the duration of the season; and
(3) casual employees or those who are neither regular nor project employees.55

Farm workers generally fall under the definition of seasonal employees. We have
consistently held that seasonal employees may be considered as regular
employees.56 Regular seasonal employees are those called to work from time to
time. The nature of their relationship with the employer is such that during the
off season, they are temporarily laid off; but reemployed during the summer
season or when their services may be needed.57 They are in regular employment
because of the nature of their job, and not because of the length of time they have
worked.58
For regular employees to be considered as such, the primary standard used is the
reasonable connection between the particular activity they perform and the
usual trade or business of the employer.61

3E Andaya Ching Espiritu Hefti Galvez Gammad Lainez Lui Madamba Nagera Narvasa
Ong Palangdao Rosales Sanchez Santos Satrain Tabo (2014-2015)

Labor Relations Case Digest - Atty. Joyrich Golangco

A reading of the records reveals that the deceased was indeed a farm worker
who was in the regular employ of petitioner. From year to year, starting January
1983 up until his death, the deceased had been working on petitioners land by
harvesting abaca and coconut, processing copra, and clearing weeds. His
employment was continuous in the sense that it was done for more than one
harvesting season. Moreover, no amount of reasoning could detract from the fact
that these tasks were necessary or desirable in the usual business of petitioner.
154. Millenium Erectors Corporation vs. Magallanes

DOCTRINE: A project employee is one whose employment has been fixed for a
specific project or undertaking, the completion or termination of which has been
determined at the time of the engagement of the employee the service of
project employees are coterminous with the project and may be terminated upon
the end or completion of that project or project phase for which they are hired.
Regular employees, in contrast, enjoy security of tenure and are entitled to hold
on to their work or position until their services are terminated by any of the
modes recognized under the Labor Code.
FACTS: Respondent Virgilio Magallanes started working in 1988 as a utility man
for Laurencito Tiu (Tiu), Chief Executive Officer of Millennium Erectors
Corporation (petitioner), Tius family, and Kenneth Construction
Corporation. He was assigned to different construction projects undertaken by
petitioner in Metro Manila, the last of which was for a building in Libis, Quezon
City. In July of 2004 he was told not to report for work anymore allegedly due to
old age, prompting him to file on August 6, 2004 an illegal dismissal
complaint before the Labor Arbiter. In its Position Paper, petitioner claimed that
respondent was a project employee whom it hired for a building project in Libis
on January 30, 2003, to prove which it submitted the employment contract
signed by him; that on August 3, 2004, respondents services were terminated as
the project was nearing completion; and he was given financial assistance in the
amount of P2,000, for which he signed a quitclaim and waiver.[5] Petitioner
likewise submitted a termination report to the Department of Labor and
Employment (DOLE) dated August 17, 2004. Rebutting respondents claim that
he was employed since 1988, petitioner contended that it was incorporated only
in February 2000, and Kenneth Construction Corporation which was established
in 1989 and dissolved in 2000, was a separate and distinct entity.

LA RULING: In favor of the petitioner and dismissed the complaint, holding that
respondent knew of the nature of his employment as a project employee, he
having executed an employment contract specifying therein the name of and
duration of the project from January 2003 until its completion; and that the
services of respondent were terminated due to the completion of the project as
shown by the termination report submitted to the DOLE. The Labor Arbiter
noted that respondent admitted having been assigned to several building
projects and that he failed to give pertinent details of his dismissal such as who
terminated him, when he was terminated, and what were the overt acts leading
to his dismissal.
NLRC RULING: set aside the LA, holding that respondent was a regular, not a
project employee, as the employment contract he supposedly signed contained
the date of commencement but not a specific date when it would end, contrary to
the rule that the duration and scope of similar contracts should be clearly set
forth therein; and that based on the payrolls petitioner submitted and contrary
to its claim that respondent was hired in January 2003, he had been employed in
2001, not 2003, lending weight to his claim that he had worked for petitioner for
16 years prior to the filing of his complaint. The NLRC thus concluded that while
respondents work as a utility man may not have been necessary or desirable in
the usual business of petitioner as a construction company, that he performed
the same functions continuously for 16 years converted an otherwise casual
employment to regular employment, hence, his termination without just or
authorized cause amounted to illegal dismissal.
CA RULING: Affirmed the NLRCs ruling.
ISSUES: Whether or not respondent was a regular, not a project employee.
HELD: On the merits of the case, the Court finds that, indeed, respondent was a
regular, not a project employee. Saberola v. Suarez reiterates the well-settled
definition of project employee, viz:
A project employee is one whose "employment has been
fixed for a specific project or undertaking, the completion
or termination of which has been determined at the time of
the engagement of the employee or where the work or
service to be performed is seasonal in nature and the
employment is for the duration of the season." (emphasis and
underscoring supplied)

3E Andaya Ching Espiritu Hefti Galvez Gammad Lainez Lui Madamba Nagera Narvasa
Ong Palangdao Rosales Sanchez Santos Satrain Tabo (2014-2015)

Labor Relations Case Digest - Atty. Joyrich Golangco


And Equipment Technical Services v. Court of Appeals emphasizes the difference
between a regular employee and a project employee:
As the Court has consistently held, the service
of project employees are coterminus [sic] with the
project and may be terminated upon the end or completion of
that project or project phase for which they were hired. Regular
employees, in contrast, enjoy security of tenure and are
entitled to hold on to their work or position until their services
are terminated by any of the modes recognized under the Labor
Code. (emphasis and underscoring supplied)

Petitioners various payrolls dating as early as 2001 show that respondent had
been employed by it. As aptly observed by the appellate court, these documents,
rather than sustaining petitioners argument, only serve to support respondents
contention that he had been employed in various projects, if not for 16 years, at
the very least two years prior to his dismissal.
Petitioner having failed to discharge its burden of proving that it terminated the
services of respondent for cause and with due process, the challenged decision
must remain.
155. Caparoso vs CA

DOCTRINE: The Court thus laid down the criteria under which fixed-term
employment could not be said to be in circumvention of the law on security of
tenure, thus:

1. The fixed period of employment was knowingly and voluntarily


agreed upon by the parties without any force, duress, or improper
pressure being brought to bear upon the employee and absent any other
circumstances vitiating his consent; or

2. It satisfactorily appears that the employer and the employee dealt


with each other on more or less equal terms with no moral dominance
exercised by the former or the latter.

FACTS: Composite Enterprises Incorporated (Composite) is engaged in the


distribution and supply of confectioneries to various retail establishments within
the Philippines. Caparoso Quindipan were Composites deliverymen. Caparoso
alleged that he was hired on 8 November 1998 while Quindipan alleged that he
was hired on intermittent basis since 1997. Quindipan further alleged that he
had been working continuously with Composite since August 1998.
Caparoso and Quindipan (petitioners) were dismissed from the service. They
filed a consolidated position paper before the LA charging Composite and
its Personnel Manager Tan of illegal dismissal.

Composite and Tan (respondents) alleged that petitioners were both hired on 11
May 1999 as deliverymen, initially for three months and then on a month-tomonth basis. Respondents alleged that petitioners termination from
employment resulted from the expiration of their contracts of employment on 8
October 1999.
LA ruled that petitioners are regular employees of respondents and that they
have been illegally dismissed.
LA ruled that by the very nature of
respondents business and the nature of petitioners services, there is no doubt
as to the employment status of petitioners.

On appeal, NLRC reversed LAs decision and ruled that the mere fact that the
employees duties are necessary or desirable in the business or trade of the
employer does not mean that they are forbidden from stipulating the period of
employment. The NLRC held that petitioners contracts of employment are valid
and binding between the contracting parties and shall be considered as the law
between them. The NLRC ruled that petitioners are bound by their employment
contracts. It was affirmed by the CA.
ISSUES: 1. Whether petitioners are regular employees of respondents?

3E Andaya Ching Espiritu Hefti Galvez Gammad Lainez Lui Madamba Nagera Narvasa
Ong Palangdao Rosales Sanchez Santos Satrain Tabo (2014-2015)

Labor Relations Case Digest - Atty. Joyrich Golangco

2. Whether respondents are guilty of illegal dismissal.

HELD: 1. NO. Under Article 280 of the Labor Code, a regular employee is (1) one
who is engaged to perform activities that are necessary or desirable in the usual
trade or business of the employer, or (2) a casual employee who has rendered
at least one year of service, whether continuous or broken, with respect to the
activity in which he is employed.

We agree with the Court of Appeals that in this case, the fixed period of
employment was knowingly and voluntarily agreed upon by the parties. The
Court of Appeals noted that there was no indication of force, duress, or
improper pressure exerted on petitioners when they signed the contracts.
Further, there was no proof that respondents were regularly engaged in
hiring workers for work for a minimum period of five months to prevent
the regularization of their employees. Petitioners Employment is akin to
Probationary Employment

However, even if an employee is engaged to perform activities that are necessary


or desirable in the usual trade or business of the employer, it does not preclude
the fixing of employment for a definite period.

At most, petitioners employment for less than six months can be considered
probationary. Article 281 of the Labor Code provides:

The Court thus laid down the criteria under which fixed-term employment could
not be said to be in circumvention of the law on security of tenure, thus:

Petitioners were hired on 11 May 1999, initially for three months. After the
expiration of their contracts, petitioners were hired on a month-to-month basis.
Their contracts of employment ended on 8 October 1999. Hence, they were
employed for a total of five months. Their employment did not even exceed six
months to entitle them to become regular employees.

The clause in said article indiscriminately and completely ruling out all written
or oral agreements conflicting with the concept of regular employment as
defined therein should be construed to refer to the substantive evil that the Code
itself has singled out: agreements entered into precisely to
circumvent security of tenure. It should have no application to instances where a
fixed period of employment was agreed upon knowingly and voluntarily by the
parties, without any force, duress or improper pressure being brought to bear
upon the employee and absent any other circumstances vitiating his consent, or
where it satisfactorily appears that the employer and employee dealt with each
other on more or less equal terms with no moral dominance whatever being
exercised by the former over the latter.

1. The fixed period of employment was knowingly and voluntarily


agreed upon by the parties without any force, duress, or improper
pressure being brought to bear upon the employee and absent any other
circumstances vitiating his consent; or
2. It satisfactorily appears that the employer and the employee dealt
with each other on more or less equal terms with no moral dominance
exercised by the former or the latter.

Art. 281. Probationary Employment. - Probationary employment shall not exceed


six (6) months from the date the employee started working, unless it is covered by
an apprenticeship agreement stipulating a longer period. The services of an
employee who has been engaged on a probationary basis may be terminated for a
just cause or when he fails to qualify as a regular employee in accordance with
reasonable standards made known by the employer to the employee at the time of
his engagement. An employee who is allowed to work after a probationary period
shall be considered a regular employee.

We cannot accept petitioners bare allegations that Caparoso was hired on 8


November 1998 while Quindipan was hired on intermittent basis since 1997.
Petitioners failed to substantiate their allegations. The payslips submitted by
petitioners to prove their prior employment with respondents are handwritten
and indicate only the date and amount of pay. They do not even indicate the
name of the employer. The printed payslips during the period of the contracts
indicate not only the name of the employer but also the breakdown of
petitioners net pay.

3E Andaya Ching Espiritu Hefti Galvez Gammad Lainez Lui Madamba Nagera Narvasa
Ong Palangdao Rosales Sanchez Santos Satrain Tabo (2014-2015)

Labor Relations Case Digest - Atty. Joyrich Golangco

2. NO. Petitioners terms of employment are governed by their fixed-term


contracts. Petitioners fixed-term employment contracts had expired. They were
not illegally dismissed from employment.
156. Sps Lim v. Legazpi Hope Christian School, et al.

DOCTRINE: for a private school teacher to acquire permanent status in


employment, the following requisites must concur: (1) the teacher is a full-time
teacher; (2) the teacher must have rendered three consecutive years of service;
and (3) such service must have been satisfactory.

FACTS: Petitioner-spouses Alwyn Ong Lim and Evelyn Lukang Lim were hired in
June 1999. Alwyn was assigned to teach Mathematics, Geometry, Algebra and
Trigonometry
subjects
in
the
high
school
department
of Legazpi Hope Christian School. Evelyn, on the other hand, was assigned to
teach Chinese Language 1 and 2 and Chinese Math subjects in the elementary
department of the same school.
Respondent Helen Sia, head teacher of the schools Chinese department,
verbally informed petitioners that their employment with the school were to be
terminated, without giving the reasons therefor. Thus, petitioners filed their
complaints for illegal dismissal and monetary claims against the school and its
officials. Respondent Ramon Sia, Vice Chairman of the schools Board of Directors,
sent a letter to the petitioners stating that their three-year probation had expired and
that the school management had decided to discontinue their employment.
Before the Labor Arbiter, respondents claimed that petitioners were
merely part-time teachers and thus they can be dismissed even without
waiting for the three-year probation period to lapse, as they never acquired
permanent status.
The Labor Arbiter ruled in favor of petitioners. Respondents appealed the
decision of the Labor Arbiter to the NLRC. The NLRC found that petitioners were
only part-time teachers who did not acquire permanent status; hence, their
dismissal was legal.
ISSUES: Whether the Spouses Lims dismissals were legal.

HELD: In resolving the issue of whether or not petitioners were hired as


permanent teaching personnel, it is relevant to first determine whether
petitioners are part-time or full-time teachers. Petitioners work schedules
are 7:30am to 9:30 am for Evelyin and 7am to 12nn for Alwyn.
In University of Sto. Tomas v. NLRC, the Court ruled that for a private
school teacher to acquire permanent status in employment, the following
requisites must concur: (1) the teacher is a full-time teacher; (2) the teacher
must have rendered three consecutive years of service; and (3) such service
must have been satisfactory.
The burden is on petitioners to prove their affirmative allegation that
they are permanent teaching personnel. However, there is not enough evidence
on record to show that their total working day is devoted to the school. There is
no showing of what the regular work schedule of a regular teacher in respondent
school is. What is clear in the records is that Evelyn and Alwyn spent two hours
and four hours, respectively, but not the entire working day, at the respondent
school. They do not meet requirements of the Manual of regulations for private
schools. Hence, the petitioners are part-time teachers. Being part-time teachers,
in accordance with University of Sto. Tomas v. NLRC, they cannot acquire
permanent status.
In this case, the contracts of employment of the petitioners were not
presented. It is in fact claimed that they have no written contracts, and such
contracts are not disclosed by the records in the instant case. However on
record, attached as part of a pleading of petitioners, is a copy of the TEACHERS
GUIDELINES of respondent school which, in part, state:
STATUS & PRIVILEGES OF TEACHERS
1. New Teachers
a.
New Teachers are on probation for three (3) years,
within the duration, they must submit a letter of reapplication for each school year. After the expiration
date of the contract, a new one must be signed if it is
sent to you.
b. A full time new teacher is under 10-month contract
only. If his/her performance is satisfactory, he/she will
be rehired and will be entitled to receive the salaries
for the 2-month summer vacation. (Emphasis supplied.)
xxxx

3E Andaya Ching Espiritu Hefti Galvez Gammad Lainez Lui Madamba Nagera Narvasa
Ong Palangdao Rosales Sanchez Santos Satrain Tabo (2014-2015)

Labor Relations Case Digest - Atty. Joyrich Golangco

Considering that petitioners were new teachers, then in accordance with


the above-quoted guidelines their unwritten contracts were considered to be for
one school year at a time, they being required to submit a letter of re-application
for each school year. After the end of each school year, the school did not have any
obligation to give them any teaching loads, they being part-time teachers. That
respondents did not give any teaching assignment to the petitioners after the
school year 2001-2002 did not amount to an actionable violation of petitioners
right. It did not amount to illegal dismissal. In view of the foregoing finding that
petitioners were not illegally dismissed, there is also no basis to order their
reinstatement and the payment of damages and attorneys fees to them.
157. DM Consunji Inc.(DMCI) vs Jamin

DOCTRINE: Once a project or work pool employee has been: (1) continuously,
as opposed to intermittently, rehired by the same employer for the same tasks or
nature of tasks; and (2) these tasks are vital, necessary and indispensable to the
usual business or trade of the employer, then the employee must be deemed a
regular employee.
FACTS: Petitioner DMCI, a construction company, hired respondent Estelito L.
Jamin as laborer. Since his initial hiring, his employment contract had been
renewed a number of times. On March 20, 1999, his work at DMCI was
terminated due to the completion of the SM Manila project. Jamin filed a
complaint[5] for illegal dismissal, with several money claims (including attorneys
fees), against DMCI and its President/General Manager, David M. Consunji. Jamin
alleged that DMCI terminated his employment without a just and authorized
cause at a time when he was already 55 years old and had no independent source
of livelihood. He claimed that he rendered service to DMCI continuously for
almost 31 years. DMCI denied liability. It argued that it hired Jamin on a projectto-project basis, from the start of his engagement in 1968 until the completion of
its SM Manila project onMarch 20, 1999 where Jamin last worked. With the
completion of the project, it terminated Jamins employment. It alleged that it
submitted a report to the Department of Labor and Employment (DOLE)
everytime it terminated Jamins services. The LA dismissed the complaint and
ruled that Jamin was a project employee whose services had been terminated
due to the completion of the project he was assigned to. The labor arbiter added

that everytime DMCI rehired Jamin, it entered into a contract of employment


with him because that Jamin had to file an application if he wanted to be re-hired.
NLRC affirmed the decision of the LA and denied respondents motion for
reconsideration. On appeal with the CA, the appellate court reversed the decision
and held that Jamin was a regular employee. . It based its conclusion on: (1)
Jamins repeated and successive rehiring in DMCIs various projects; and (2) the
nature of his work in the projects he was performing activities necessary or
desirable in DMCIs construction business. Contrary to DMCIs submission and
the labor arbiters findings, the CA noted that DMCI failed to submit a report to
the DOLE Regional Office everytime Jamins employment was terminated, as
required by DOLE Policy Instructions No. 20. The CA opined that DMCIs failure
to submit the reports to the DOLE is an indication that Jamin was not a project
employee. By reasons of such findings, CA declared his dismissal as illegal as it
was without a valid cause and without due process.

ISSUES: WON Jamin is a regular employee and if in the affirmative, whether his
dismissal from employment illegal

HELD: YES. To reiterate, Jamins employment history with DMCI stands out for
his continuous, repeated and successive rehiring in the companys construction
projects. In all the 38 projects where DMCI engaged Jamins services, the tasks he
performed as a carpenter were indisputably necessary and desirable in DMCIs
construction business. He might not have been a member of a work pool as DMCI
insisted that it does not maintain a work pool, but his continuous rehiring and
the nature of his work unmistakably made him a regular employee.
In Maraguinot, Jr. v. NLRC,[43] the Court held that once a project or work pool
employee has been: (1) continuously, as opposed to intermittently, rehired by
the same employer for the same tasks or nature of tasks; and (2) these tasks are
vital, necessary and indispensable to the usual business or trade of the
employer, then the employee must be deemed a regular employee. Surely,
length of time is not the controlling test for project employment. Nevertheless, it
is vital in determining if the employee was hired for a specific undertaking or
tasked to perform functions vital, necessary and indispensable to the usual
business or trade of the employer. Here, [private] respondent had been a project
employee several times over. His employment ceased to be coterminous with

3E Andaya Ching Espiritu Hefti Galvez Gammad Lainez Lui Madamba Nagera Narvasa
Ong Palangdao Rosales Sanchez Santos Satrain Tabo (2014-2015)

Labor Relations Case Digest - Atty. Joyrich Golangco

specific projects when he was repeatedly re-hired due to the demands of


petitioners business.
158. Yolanda Mercado vs. AMA Computer College Paranaque.

DOCTRINE: The Labor Code is supplemented with respect to the period of the
probation by special rules found in the Manual of Regulations for Private Schools.
The Company standards should be made known to the teachers on Probationary
Status at the start of their probationary period, or at the very least, at the start of
the semester or the trimester during which the probationary standards are to be
applied.

FACTS: For the school year 2000-2001, AMACC implemented new faculty
screening guidelines, set forth in its Guidelines on the Implementation of AMACC
Faculty Plantilla. Under the new screening guidelines, teachers were to be hired
or maintained based on extensive teaching experience, capability, potential, high
academic qualifications and research background. The performance standards
under the new screening guidelines were also used to determine the present
faculty members entitlement to salary increases. The petitioners failed to
obtain a passing rating based on the performance standards; hence AMACC
did not give them any salary increase. Because of AMACCs action on the
salary increases, the petitioners filed a complaint with the Arbitration Branch of
the NLRC on July 25, 2000, for underpayment of wages, non-payment of overtime
and overload compensation, 13th month pay, and for discriminatory practices. On
September 7, 2000, the petitioners individually received a memorandum from
AMACC, through Human Resources Supervisor Mary Grace Beronia, informing
them that with the expiration of their contract to teach, their contract would no
longer be renewed. The petitioners amended their labor arbitration complaint to
include the charge of illegal dismissal against AMACC. In their Position Paper,
the petitioners claimed that their dismissal was illegal because it was made in
retaliation for their complaint for monetary benefits and discriminatory
practices against AMACC. The petitioners also contended that AMACC failed to
give them adequate notice; hence, their dismissal was ineffectual.
AMACC contended in response that the petitioners worked under a contracted
term under a non-tenured appointment and were still within the three-year
probationary period for teachers. Their contracts were not renewed for the

following term because they failed to pass the Performance Appraisal System for
Teachers (PAST) while others failed to comply with the other requirements for
regularization, promotion, or increase in salary. This move, according to AMACC,
was justified since the school has to maintain its high academic standards.
LA Ruling: Petitioners had been illegally dismissed. The LA ruled that Article
281 of the Labor Code on probationary employment applied to the case; that
AMACC allowed the petitioners to teach for the first semester of school year
2000-200; that AMACC did not specify who among the petitioners failed to pass
the PAST and who among them did not comply with the other requirements of
regularization, promotions or increase in salary; and that the petitioners
dismissal could not be sustained on the basis of AMACCs vague and general
allegations without substantial factual basis.

NLRC Ruling: Affirmed LAs ruling. The NLRC, however, observed that the
applicable law is Section 92 of the Manual of Regulations for Private Schools
(which mandates a probationary period of nine consecutive trimesters of
satisfactory service for academic personnel in the tertiary level where collegiate
courses are offered on a trimester basis), not Article 281 of the Labor Code
(which prescribes a probationary period of six months) as the LA ruled. Despite
this observation, the NLRC affirmed the LAs finding of illegal dismissal since the
petitioners were terminated on the basis of standards that were only introduced
near the end of their probationary period. The NLRC ruled that the new
screening guidelines for the school year 2000-20001 cannot be imposed on the
petitioners and their employment contracts since the new guidelines were not
imposed when the petitioners were first employed in 1998.
CA Ruling: Dismissed petitioners complaint for illegal dismissal. The CA ruled
that under the Manual for Regulations for Private Schools, a teaching personnel
in a private educational institution (1) must be a full time teacher; (2) must have
rendered three consecutive years of service; and (3) such service must be
satisfactory before he or she can acquire permanent status. The CA noted that the
petitioners had not completed three (3) consecutive years of service (i.e. six
regular semesters or nine consecutive trimesters of satisfactory service) and
were still within their probationary period; their teaching stints only covered a
period of two (2) years and three (3) months when AMACC decided not to renew
their contracts on September 7, 2000. The CA disagreed with the NLRCs ruling

3E Andaya Ching Espiritu Hefti Galvez Gammad Lainez Lui Madamba Nagera Narvasa
Ong Palangdao Rosales Sanchez Santos Satrain Tabo (2014-2015)

Labor Relations Case Digest - Atty. Joyrich Golangco

that the new guidelines for the school year 2000-20001 could not be imposed on
the petitioners and their employment contracts. Finally, the CA found that the
petitioners were hired on a non-tenured basis and for a fixed and predetermined
term based on the Teaching Contract exemplified by the contract between the
petitioner Lachica and AMACC.
ISSUES: Whether or not the teachers probationary status be disregarded simply
because the contracts were fixed term.

HELD: The provision on employment on probationary status under the Labor


Code is a primary example of the fine balancing of interests between labor and
management that the Code has institutionalized pursuant to the underlying
intent of the Constitution. On the one hand, employment on probationary status
affords management the chance to fully scrutinize the true worth of hired
personnel before the full force of the security of tenure guarantee of the
Constitution comes into play. Based on the standards set at the start of the
probationary period, management is given the widest opportunity during the
probationary period to reject hirees who fail to meet its own adopted but
reasonable standards. These standards, together with the just] and authorized
causes for termination of employment the Labor Code expressly provides, are the
grounds available to terminate the employment of a teacher on probationary
status. For example, the school may impose reasonably stricter attendance or
report compliance records on teachers on probation, and reject a probationary
teacher for failing in this regard, although the same attendance or compliance
record may not be required for a teacher already on permanent status. At the
same time, the same just and authorizes causes for dismissal under the Labor
Code apply to probationary teachers, so that they may be the first to be laid-off if
the school does not have enough students for a given semester or
trimester. Termination of employment on this basis is an authorized cause
under the Labor Code. Labor, for its part, is given the protection during the
probationary period of knowing the company standards the new hires have to
meet during the probationary period, and to be judged on the basis of these
standards, aside from the usual standards applicable to employees after they
achieve permanent status. Under the terms of the Labor Code, these standards
should be made known to the teachers on probationary status at the start of their
probationary period, or at the very least under the circumstances of the present
case, at the start of the semester or the trimester during which the probationary

standards are to be applied. Of critical importance in invoking a failure to meet


the probationary standards, is that the school should show as a matter of due
process how these standards have been applied. This is effectively the second
notice in a dismissal situation that the law requires as a due process guarantee
supporting the security of tenure provision, and is in furtherance, too, of the
basic rule in employee dismissal that the employer carries the burden of
justifying a dismissal. These rules ensure compliance with the limited security of
tenure guarantee the law extends to probationary employees. Given the clear
constitutional and statutory intents, we cannot but conclude that in a situation
where the probationary status overlaps with a fixed-term contract not
specifically used for the fixed term it offers, Article 281 should assume primacy
and the fixed-period character of the contract must give way. This conclusion is
immeasurably strengthened by the petitioners and the AMACCs hardly
concealed expectation that the employment on probation could lead to
permanent status, and that the contracts are renewable unless the petitioners fail
to pass the schools standards. If the school were to apply the probationary
standards (as in fact it says it did in the present case), these standards must not
only be reasonable but must have also been communicated to the teachers at the
start of the probationary period, or at the very least, at the start of the period
when they were to be applied. These terms, in addition to those expressly
provided by the Labor Code, would serve as the just cause for the termination of
the probationary contract. As explained above, the details of this finding of just
cause must be communicated to the affected teachers as a matter of due
process.
AMACC, by its submissions, admits that it did not renew the
petitioners contracts because they failed to pass the Performance Appraisal
System for Teachers (PAST) and other requirements for regularization that the
school undertakes to maintain its high academic standards. The evidence is
unclear on the exact terms of the standards, although the school also admits that
these were standards under the Guidelines on the Implementation of AMACC
Faculty Plantilla put in place at the start of school year 2000-2001.
Petition Granted. Given the period that has lapsed and the inevitable change of
circumstances that must have taken place in the interim in the academic world
and at AMACC, which changes inevitably affect current school operations, we
hold that - in lieu of reinstatement - the petitioners should be paid separation
pay computed on a trimestral basis from the time of separation from service up
to the end of the complete trimester preceding the finality of this Decision.

3E Andaya Ching Espiritu Hefti Galvez Gammad Lainez Lui Madamba Nagera Narvasa
Ong Palangdao Rosales Sanchez Santos Satrain Tabo (2014-2015)

Labor Relations Case Digest - Atty. Joyrich Golangco

159. BRENT SCHOOL, INC., and REV. GABRIEL DIMACHE, petitioners,


vs.RONALDO ZAMORA, the Presidential Assistant for Legal Affairs, Office of
the President, and DOROTEO R. ALEGRE, respondents.

DOCTRINE: Article 280 of the Labor Code clearly appears to have been, as
already observed, to prevent circumvention of the employee's right to be secure
in his tenure, the clause in said article indiscriminately and completely ruling out
all written or oral agreements conflicting with the concept of regular
employment as defined therein should be construed to refer to the substantive
evil that the Code itself has singled out: agreements entered into precisely to
circumvent security of tenure. It should have no application to instances where a
fixed period of employment was agreed upon knowingly and voluntarily by the
parties, without any force, duress or improper pressure being brought to bear
upon the employee and absent any other circumstances vitiating his consent, or
where it satisfactorily appears that the employer and employee dealt with each
other on more or less equal terms with no moral dominance whatever being
exercised by the former over the latter.

FACTS: The root of the controversy at bar is an employment contract in virtue of


which Doroteo R. Alegre was engaged as athletic director by Brent School, Inc. at
a yearly compensation of P20,000.00. The contract fixed a specific term for its
existence, five (5) years, i.e., from July 18, 1971, the date of execution of the
agreement, to July 17, 1976. Subsequent subsidiary agreements dated March 15,
1973, August 28, 1973, and September 14, 1974 reiterated the same terms and
conditions, including the expiry date, as those contained in the original contract
of July 18, 1971. Some three months before the expiration of the stipulated
period, or more precisely on April 20,1976, Alegre was given a copy of the report
filed by Brent School with the Department of Labor advising of the termination of
his services effective on July 16, 1976. The stated ground for the termination was
"completion of contract, expiration of the definite period of employment."
Alegre protested the announced termination of his employment. He argued that
although his contract did stipulate that the same would terminate on July 17,
1976, since his services were necessary and desirable in the usual business of his
employer, and his employment had lasted for five years, he had acquired the
status of a regular employee and could not be removed except for valid cause.
The Regional Director considered Brent School's report as an application for

clearance to terminate employment (not a report of termination), and accepting


the recommendation of the Labor Conciliator, refused to give such clearance and
instead required the reinstatement of Alegre, as a "permanent employee," to his
former position without loss of seniority rights and with full back wages. The
Director pronounced "the ground relied upon by the respondent (Brent) in
terminating the services of the complainant (Alegre) . . . (as) not sanctioned by
P.D. 442," and, quite oddly, as prohibited by Circular No. 8, series of 1969, of the
Bureau of Private Schools.
ISSUES: Whether or not the provisions of the Labor Code have anathematized
"fixed period employment" or employment for a term.

HELD: NO, Respondent Alegre's contract of employment with Brent School


having lawfully terminated with and by reason of the expiration of the agreed
term of period thereof, he is declared not entitled to reinstatement. It is plain
then that when the employment contract was signed between Brent School and
Alegre, it was perfectly legitimate for them to include in it a stipulation fixing the
duration thereof Stipulations for a term were explicitly recognized as valid by
this Court.

Article 280 of the Labor Code, under a narrow and literal interpretation, not only
fails to exhaust the gamut of employment contracts to which the lack of a fixed
period would be an anomaly, but would also appear to restrict, without
reasonable distinctions, the right of an employee to freely stipulate with his
employer the duration of his engagement, it logically follows that such a literal
interpretation should be eschewed or avoided. The law must be given a
reasonable interpretation, to preclude absurdity in its application. Outlawing the
whole concept of term employment and subverting to boot the principle of
freedom of contract to remedy the evil of employer's using it as a means to
prevent their employees from obtaining security of tenure is like cutting off the
nose to spite the face or, more relevantly, curing a headache by lopping off the
head.
Accordingly, and since the entire purpose behind the development of legislation
culminating in the present Article 280 of the Labor Code clearly appears to have
been, as already observed, to prevent circumvention of the employee's right to
be secure in his tenure, the clause in said article indiscriminately and completely

3E Andaya Ching Espiritu Hefti Galvez Gammad Lainez Lui Madamba Nagera Narvasa
Ong Palangdao Rosales Sanchez Santos Satrain Tabo (2014-2015)

Labor Relations Case Digest - Atty. Joyrich Golangco

ruling out all written or oral agreements conflicting with the concept of regular
employment as defined therein should be construed to refer to the substantive
evil that the Code itself has singled out: agreements entered into precisely to
circumvent security of tenure. It should have no application to instances where a
fixed period of employment was agreed upon knowingly and voluntarily by the
parties, without any force, duress or improper pressure being brought to bear
upon the employee and absent any other circumstances vitiating his consent, or
where it satisfactorily appears that the employer and employee dealt with each
other on more or less equal terms with no moral dominance whatever being
exercised by the former over the latter. Unless thus limited in its purview, the
law would be made to apply to purposes other than those explicitly stated by its
framers; it thus becomes pointless and arbitrary, unjust in its effects and apt to
lead to absurd and unintended consequences.

Paraphrasing Escudero, respondent Alegre's employment was terminated upon


the expiration of his last contract with Brent School on July 16, 1976 without the
necessity of any notice. The advance written advice given the Department of
Labor with copy to said petitioner was a mere reminder of the impending
expiration of his contract, not a letter of termination, nor an application for
clearance to terminate which needed the approval of the Department of Labor to
make the termination of his services effective. In any case, such clearance should
properly have been given, not denied.
160: Pure Foods Corporation vs NLRC

DOCTRINE: Contractual employment is NOT violative of the constitution (i.e.


Security of Tenure) if the following criteria are present:

1) The fixed period of employment was knowingly and voluntarily agreed upon
by the parties without any force, duress, or improper pressure being brought to
bear upon the employee and absent any other circumstances vitiating his
consent; or
2) It satisfactorily appears that the employer and the employee dealt with each
other on more or less equal terms with no moral dominance exercised by the
former or the latter.

FACTS: Private respondents (a total of 906) were contractual workers of


petitioner for a 5-month contract at its tuna cannery plant in GenSan.
Respondents activities consisted in the receiving, skinning, loining, packing, and
casing-up of tuna fish, which were then exported by the petitioner. After the
expiration of their respective contracts of employment in June and July 1991,
their services were terminated. They forthwith executed a Release and
Quitclaim stating that they had no claim whatsoever against the petitioner.
Respondent filed before NLRC Sub-Regional Branch GenSan, a complaint for
illegal dismissal.

LA: dimissed the complaint. Being contractual employees they could not avail of
the law on security of tenure. Their termination was justified. LA observed
petitioner is correct that if there would be reinstatement it would burden the
company in reemploying more than 10,000 contractual employees (the number
of contractual employees PureFood has as of July 1990).
Respondents appealed before the NLRC

NLRC: initially it affirmed LA but upon respondents MR it SET ASIDE LAs ruling.
Respondents were declared regular employees It declared that the contract of
employment for five months was a clandestine scheme employed by [the
petitioner] to stifle [private respondents] right to security of tenure and should
therefore be struck down and disregarded for being contrary to law, public
policy, and morals. Hence, their dismissal on account of the expiration of their
respective contracts was illegal. It ordered reinstatement without loss of
seniority and benefits or reparation pat of 1 month or month pay for every
year of service whichever is higher.
Pure Foods argues that respondents cannot claim anymore (they are
estopped) as they EXPRESSLY conformed with the 5-month contact and that they
fell within the exception in Article 280 of LC: [E]xcept where the employment
has been fixed for a specific project or undertaking the completion or
termination of which has been determined at the time of the engagement of the
employee. In the instant case, the private respondents were employed for a
period of five months only. In any event, private respondents' prayer for
reinstatement is well within the purview of the Release and Quitclaim they had
executed wherein they unconditionally released the petitioner from any and all

3E Andaya Ching Espiritu Hefti Galvez Gammad Lainez Lui Madamba Nagera Narvasa
Ong Palangdao Rosales Sanchez Santos Satrain Tabo (2014-2015)

Labor Relations Case Digest - Atty. Joyrich Golangco

other claims which might have arisen from their past employment with the
petitioner.

In its Comment, the Office of the Solicitor General (OSG) advances the
argument that the private respondents were regular employees, since they
performed activities necessary and desirable in the business or trade of the
petitioner. The period of employment stipulated in the contracts of employment
was null and void for being contrary to law and public policy, as its purpose was
to circumvent the law on security of tenure. The expiration of the contract did
not, therefore, justify the termination of their employment.
ISSUES: Whether or not respondents are considered regular employee.
HELD: Affirmative.

ART. 280. Regular and Casual Employment.-- The


provisions of written agreement to the contrary
notwithstanding and regardless of the oral argument
of the parties, an employment shall be deemed to be
regular where the employee has been engaged to
perform activities which are usually necessary or
desirable in the usual business or trade of the
employer, except where the employment has been
fixed for a specific project or undertaking the
completion or termination of which has been
determined at the time of the engagement of the
employee or where the work or services to be
performed is seasonal in nature and the employment
is for the duration of the season.

An employment shall be deemed to be casual


if it is not covered by the preceding
paragraph; Provided, That, any employee who has
rendered at least one year of service, whether such
service is continuous or broken, shall be considered a
regular employee with respect to the activity in which
he is employed and his employment shall continue
while such activity exists.

Thus, the two kinds of regular employees are (1) those who are engaged
to perform activities which are necessary or desirable in the usual business or
trade of the employer; and (2) those casual employees who have rendered at
least one year of service, whether continuous or broken, with respect to the
activity in which they are employed.
Indisputably, they were performing activities, which were necessary and
desirable in petitioners business or trade.

Contrary to petitioner's submission, the private respondents could not be


regarded as having been hired for a specific project or undertaking. The term
specific project or undertaking under Article 280 of the Labor Code
contemplates an activity which is not commonly or habitually performed or such
type of work which is not done on a daily basis but only for a specific duration of
time or until completion; the services employed are then necessary and desirable
in the employers usual business only for the period of time it takes to complete
the project.

The fact that the petitioner repeatedly and continuously hired workers
to do the same kind of work as that performed by those whose contracts had
expired negates petitioners contention that those workers were hired for a
specific project or undertaking only.
Now on the validity of private respondents' five-month contracts of
employment. In the leading case of Brent School, Inc. v. Zamora laid down
criteria under which term employment cannot be said to be in circumvention of
the law on security of tenure:

1) The fixed period of employment was knowingly and voluntarily agreed upon
by the parties without any force, duress, or improper pressure being brought to
bear upon the employee and absent any other circumstances vitiating his
consent; or
2) It satisfactorily appears that the employer and the employee dealt with each
other on more or less equal terms with no moral dominance exercised by the
former or the latter.

3E Andaya Ching Espiritu Hefti Galvez Gammad Lainez Lui Madamba Nagera Narvasa
Ong Palangdao Rosales Sanchez Santos Satrain Tabo (2014-2015)

Labor Relations Case Digest - Atty. Joyrich Golangco

None of these criteria had been met in the present case. As pointed out by the
private respondents: First, Cannery workers are never on equal terms with their
employers. Almost always, they agree to any terms of an employment contract
just to get employed considering that it is difficult to find work given their
ordinary qualifications. Second, their freedom to contract is empty and hollow
because theirs is the freedom to starve if they refuse to work as casual or
contractual workers.

This scheme of the petitioner was apparently designed to prevent the


private respondents and the other casual employees from attaining the status
of a regular employee. It was a clear circumvention of the employees right to
security of tenure and to other benefits like minimum wage, cost-of-living
allowance, sick leave, holiday pay, and 13th month pay. Indeed, the petitioner
succeeded in evading the application of labor laws. Also, it saved itself from the
trouble or burden of establishing a just cause for terminating employees by the
simple expedient of refusing to renew the employment contracts.

The execution by the private respondents of a Release and Quitclaim


did not preclude them from questioning the termination of their
services. Generally, quitclaims by laborers are frowned upon as contrary to
public policy and are held to be ineffective to bar recovery for the full measure of
the workers rights. The reason for the rule is that the employer and the
employee do not stand on the same footing

The NLRC was, thus, correct in finding that the private respondents were
regular employees. However, since reinstatement is no longer possible because
the petitioner's tuna cannery plant had, admittedly, been closed in November
1994, the proper award is separation pay equivalent to one month pay or onehalf month pay for every year of service, whichever is higher
161. ZUELLIG FREIGHT AND CARGO SYSTEMS, Petitioner, vs. NATIONAL
LABOR RELATIONS COMMISSION AND RONALDO V. SAN
MIGUEL, Respondents.

DOCTRINE: The mere change in the corporate name is not considered under the
law as the creation of a new corporation; hence, the renamed corporation

remains liable for the illegal dismissal of its employee separated under that
guise.

FACTS: San Miguel brought a complaint for unfair labor practice, illegal
dismissal, non-payment of salaries and moral damages against petitioner,
formerly known as Zeta Brokerage Corporation.2 He alleged that he had been a
checker/customs representative of Zeta; he and other employees of Zeta were
informed that Zeta would cease operations; and that all affected employees,
including him, would be separated. After a month, Zeta informed him of his
termination. H e reluctantly accepted his separation pay.

San Miguel contended that the amendments of the articles of incorporation of


Zeta were for the purpose of changing the corporate name, broadening the
primary functions, and increasing the capital stock; and that such amendments
could not mean that Zeta had been thereby dissolved.3 Petitioner countered that
San Miguels termination from Zeta had been for a cause authorized by the Labor
Code; its predecessor-in-interest had complied with the requirements for
termination due to the cessation of business operations; it had no obligation to
employ San Miguel in the exercise of its valid management prerogative.
LA held that San Miguel had been illegally dismissed. It reasoned that there was
merely a change of business name and primary purpose and upgrading of stocks
of the corporation. Zuellig and Zeta are therefore legally the same person and
entity. As such, the termination of complainants services allegedly due to
cessation of business operations of Zeta is deemed illegal. NLRC affirmed the
decision of the LA. It denied petitioners motion for reconsideration. CA
dismissed the petition for certiorari and affirmed NLRCs decision.
ISSUES: WON the closure of the business operation of Zeta had not been bona
fide, thereby resulting in the illegal dismissal of San Miguel.
HELD: The petition for review on certiorari is denied for its lack of merit.

The Labor Arbiter, the NLRC, and the CA were united in concluding that the
cessation of business by Zeta was not a bona fide closure to be regarded as a
valid ground for the termination of employment of San Miguel within the ambit

3E Andaya Ching Espiritu Hefti Galvez Gammad Lainez Lui Madamba Nagera Narvasa
Ong Palangdao Rosales Sanchez Santos Satrain Tabo (2014-2015)

Labor Relations Case Digest - Atty. Joyrich Golangco

of Article 283 of the Labor Code. The amendments of the articles of incorporation
of Zeta to change the corporate name to Zuellig Freight and Cargo Systems, Inc.
did not produce the dissolution of the former as a corporation. The effect of the
change of name was not a change of the corporate being, and has no effect on its
property, rights, or liabilities.

as a demotion. As it turned out, however, on November 9, 2006, the petitioner


had already filed a complaint for constructive dismissal9 against RSC, Sarte, Gadia
and Alex (respondents).

162. Peckson vs Robinsons Supermarket

Respondents: The respondents in their position paper denied the correctness of


the organizational chart presented by the petitioner. They maintained that her
transfer was not a demotion since the Provincial Coordinator occupied a Level
5 position like the Category Buyer, with the same work conditions, salary and
benefits. But while both positions had no significant disparity in the required
skill, experience and aptitude, the position of Category Buyer demanded the
traits of punctuality, diligence and attentiveness because it is a frontline position
in the day-to-day business operations of RSC which the petitioner, unfortunately,
did not possess.

Zeta and petitioner remained one and the same corporation. The change of name
did not give petitioner the license to terminate employees of Zeta like San Miguel
without just or authorized cause. Petitioner, despite its new name, was the mere
continuation of Zeta's corporate being, and still held the obligation to honor all of
Zeta's obligations, one of which was to respect San Miguel's security of tenure.
The dismissal of San Miguel from employment on the pretext that petitioner,
being a different corporation, had no obligation to accept him as its employee,
was illegal and ineffectual.

FACTS: The petitioner first joined the Robinsons Supermarket Corporation (RSC)
as a Sales Clerk on November 3, 1987. On October 26, 2006, she was holding the
position of Category Buyer when respondent Roena Sarte (Sarte), RSCs Assistant
Vice-President for Merchandising, reassigned her to the position of Provincial
Coordinator, effective November 1,
2006.5 Claiming that her new assignment was a demotion because it was n
on-supervisory and clerical in nature, the petitioner refused to turn over her
responsibilities to the new Category Buyer, or to accept her new responsibilities
as Provincial Coordinator. Jody Gadia (Gadia) and Ruby Alex (Alex) were
impleaded because they were corporate officers of the RSC. In a memorandum to
the petitioner dated November 13, 2006,6 the RSC, through Sarte, demanded an
explanation from her within 48 hours for her refusal to accept her new
assignment despite written and verbal demands. Sarte cited a company rule,
Offenses Subject to Disciplinary Action No. 4.07, which provided that
[d]isobedience, refusal or failure to do assigned task or to obey
superiors/officials orders/instructions, or to follow established procedures or
practices without valid reason would be meted the penalty of suspension. In her
one-paragraph reply submitted on November 27, 2006,8 the petitioner stated
that she could not accept the position of Provincial Coordinator since she saw it

Petitioners arguments: The petitioner argued before the LA that the true
organizational chart of the RSC showed that the position of Category Buyer was
one level above that of the Provincial Coordinator, and that moreover, the job
description of a Provincial Coordinator was largely clerical and did not require
her to analyze stock levels and order points, or source new local and
international suppliers, or monitor stock level per store and recommend items
for replenishment, or negotiate better items and discounts from suppliers, duties
which only a Category Buyer could perform. She also claimed that she was
instructed to file a courtesy resignation in exchange for a separation pay of onehalf salary per year of service.

The respondents also raised the petitioners record of habitual tardiness as far
back as 1999, as well as poor performance rating in 2005. In addition to her
performance rating of 2.8 out of 4.0 in 2005 equivalent to below
expectation, the petitioner was found to be tardy in June and July 2005, 13 times,
and for the entire 2005, 57 times; that she was suspended twice in 2006 for 20
instances of tardiness and absences from July to September 2006 alone.13 We
also note that the petitioner was suspended for seven (7) days in September and
October 2005 for deliberately violating a company policy after she was seen
having lunch with a company supplier.

3E Andaya Ching Espiritu Hefti Galvez Gammad Lainez Lui Madamba Nagera Narvasa
Ong Palangdao Rosales Sanchez Santos Satrain Tabo (2014-2015)

Labor Relations Case Digest - Atty. Joyrich Golangco

LA dismissed the complaint and ruled that job reassignment or classification is a


strict prerogative of the employer, and that the petitioner cannot refuse her
transfer from Category Buyer to Provincial Coordinator since both positions
commanded the same salary structure, high degree of responsibility and
impeccable honesty and integrity. Upholding the employers right not to retain
an employee in a particular position to prevent losses or to promote profitability,
the LA found no showing of any illegal motive on the part of the respondents in
reassigning the petitioner.

NLRC affirmed the LA ruling and reiterated the settled rule that management
may transfer an employee from one office to another within the business
establishment, provided there is no demotion in rank or diminution of salary,
benefits, and other privileges, and the action is not motivated by discrimination
or bad faith or effected as a form of punishment without sufficient cause. It ruled
that the respondents were able to show that the petitioners transfer was not
unreasonable, inconvenient or prejudicial, but was prompted by her failure to
meet the demands of punctuality, diligence, and personal attention of the
position of Category Buyer; that management wanted to give the petitioner a
chance to improve her work ethic, but her obstinate refusal to assume her new
position has prejudiced respondent RSC, even while she continued to receive her
salaries and benefits as Provincial Coordinator. CA also affirmed the ruling.
ISSUES: WON the transfer of the petitioner is valid

HELD: YES, Concerning the transfer of employees, these are the following
jurisprudential guidelines: (a) a transfer is a movement from one position to
another of equivalent rank, level or salary without break in the service or a
lateral movement from one position to another of equivalent rank or salary; (b)
the employer has the inherent right to transfer or reassign an employee for
legitimate business purposes; (c) a transfer becomes unlawful where it is
motivated by discrimination or bad faith or is effected as a form of punishment
or is a demotion without sufficient cause; (d) the employer must be able to show
that the transfer is not unreasonable, inconvenient, or prejudicial to the
employee. It is the employers prerogative, based on its assessment and
perception of its employees qualifications, aptitudes, and competence, to move
them around in the various areas of its business operations in order to ascertain
where they will function with maximum benefit to the company. An employees

right to security of tenure does not give him such a vested right in his position as
would deprive the company of its prerogative to change his assignment or
transfer him where he will be most useful. When his transfer is not
unreasonable, nor inconvenient, nor prejudicial to him, and it does not involve a
demotion in rank or a diminution of his salaries, benefits, and other privileges,
the employee may not complain that it amounts to a constructive dismissal.

The managerial prerogative to transfer personnel must be exercised without


grave abuse of discretion, bearing in mind the basic elements of justice and fair
play. Having the right should not be confused with the manner in which that
right is exercised. Thus, it cannot be used as a subterfuge by the employer to rid
himself of an undesirable worker. In particular, the employer must be able to
show that the transfer is not unreasonable, inconvenient or prejudicial to the
employee; nor does it involve a demotion in rank or a diminution of his salaries,
privileges and other benefits. Should the employer fail to overcome this burden
of proof, the employees transfer shall be tantamount to constructive dismissal,
which has been defined as a quitting because continued employment is rendered
impossible, unreasonable or unlikely; as an offer involving a demotion in rank
and diminution in pay. Likewise, constructive dismissal exists when an act of
clear discrimination, insensibility or disdain by an employer has become so
unbearable to the employee leaving him with no option but to forego with his
continued employment.
In the case at bar, we agree with the appellate court that there is substantial
showing that the transfer of the petitioner from Category Buyer to Provincial
Coordinator was not unreasonable, inconvenient, or prejudicial to her.
163. Gatbonton vs. NLRC, MAPUA Institute of Technology

DOCTRINE: Preventive suspension is a disciplinary measure for the protection


of the companys property pending investigation of any alleged malfeasance or
misfeasance committed by the employee. The employer may place the worker
concerned under preventive suspension if his continued employment poses a
serious and imminent threat to the life or property of the employer or of his coworkers. However, when it is determined that there is no sufficient basis to
justify an employees preventive suspension, the latter is entitled to the payment
of salaries during the time of preventive suspension.

3E Andaya Ching Espiritu Hefti Galvez Gammad Lainez Lui Madamba Nagera Narvasa
Ong Palangdao Rosales Sanchez Santos Satrain Tabo (2014-2015)

Labor Relations Case Digest - Atty. Joyrich Golangco


FACTS: Petitioner Renato S. Gatbonton is an associate professor of respondent
Mapua Institute of Technology (MIT), Faculty of Civil Engineering. Some time in
November 1998, a civil engineering student of respondent MIT filed a lettercomplaint against petitioner for unfair/unjust grading system, sexual
harassment and conduct unbecoming of an academician.
Pending investigation of the complaint, respondent MIT, through its
Committee on Decorum and Investigation placed petitioner under a 30-day
preventive suspension effective January 11, 1999. The committee believed that
petitioners continued stay during the investigation affects his performance as a
faculty member, as well as the students learning; and that the suspension will
allow petitioner to prepare himself for the investigation and will prevent his
influences to other members of the community.

Thus, petitioner filed with the NLRC a complaint for illegal


suspension, damages and attorneys fees. Petitioner questioned the validity of
the administrative proceedings with the Regional Trial Court of Manila in a
petition for certiorari but the case was terminated on May 21, 1999 when the
parties entered into a compromise agreement wherein respondent MIT agreed to
publish in the school organ the rules and regulations implementing Republic Act
No. 7877 (R.A. No. 7877) or the Anti-Sexual Harassment Act; disregard the
previous administrative proceedings and conduct anew an investigation on the
charges against petitioner. Petitioner agreed to recognize the validity of
the published rules and regulations, as well as the authority of respondent to
investigate, hear and decide the administrative case against him
On June 18, 1999, the Labor Arbiter rendered a decision stating that the
thirty day preventive suspension of complainant is hereby declared to be
illegal. Accordingly, respondents are directed to pay his wages during the period
of his preventive suspension.
Respondents appealed to the NLRC. Granted. The LAs decision was set
aside. December 13, 1999, petitioner filed a special civil action for certiorari with
the CA.On November 10, 2000, the CA promulgated the assailed decision
affirming the NLRC decision. Petitioner filed a motion for reconsideration, which
the CA denied in its Resolution dated January 16, 2001.

PETITION FOR REVIEW ON CERTIORARI with the following arguments: his


preventive suspension does not find any justification in the Mapua Rules and
Regulations considering that at the time of his preventive suspension on January
11, 1999, the rules have not been promulgated yet as it was published only on
February 23, 1999. Petitioner also contests the lack of award of damages in his
favor.
ISSUES:
WON the preventive suspension was legal and valid
WON petitioner is entitled to damages

HELD: 1. NO. R.A. No. 7877 imposed the duty on educational or training
institutions to promulgate rules and regulations in consultation with and jointly
approved by the employees or students or trainees, through their duly
designated representatives, prescribing the procedures for the investigation of
sexual harassment cases and the administrative sanctions therefor. Petitioners
preventive suspension was based on respondent MITs Rules and Regulations for
the Implemention of the Anti-Sexual Harassment Act of 1995, or R.A. No. 7877.

It must be noted however, that respondent published said rules and


regulations only on February 23, 1999. In Taada vs. Tuvera, it was ruled that:
Administrative rules and regulations must also be published if their purpose is to
enforce or implement existing law pursuant also to a valid delegation.
The Mapua Rules is one of those issuances that should be published for
its effectivity, since its purpose is to enforce and implement R.A. No. 7877, which
is a law of general application.

Moreover, even assuming that the Mapua Rules are applicable, the Court
finds that there is no sufficient basis to justify his preventive suspension. Under
the Labor Code, petitioners preventive suspension finds no valid justification. As
provided in Section 8, Rule XXIII, Book V of the Omnibus Rules Implementing the
Labor Code:
Sec. 8. Preventive Suspension. The employer may place the worker
concerned under preventive suspension if his continued employment poses a
serious threat to the life or property of the employer or of his co-workers.

3E Andaya Ching Espiritu Hefti Galvez Gammad Lainez Lui Madamba Nagera Narvasa
Ong Palangdao Rosales Sanchez Santos Satrain Tabo (2014-2015)

Labor Relations Case Digest - Atty. Joyrich Golangco

As previously stated, there is nothing on record which shows that


respondent MIT imposed the preventive suspension on petitioner as his
continued employment poses a serious threat to the life or property of the
employer or of his co-workers; therefore, his preventive suspension is not
justified.

2. NO. While petitioners preventive suspension may have been


unjustified, this does not automatically mean that he is entitled to moral or other
damages. In Primero vs. Intermediate Appellate Court, this Court held that " an
award (of moral damages) cannot be justified solely upon the premise
(otherwise sufficient for redress under the Labor Code) that the employer fired
his employee without just cause or due process. Additional facts must be
pleaded and proven to warrant the grant of moral damages under the Civil
Code, these being, to repeat, that the act of dismissal was attended by bad
faith or fraud, or was oppressive to labor, or done in a manner contrary to
morals, good customs, or public policy; and of course, that social humiliation,
wounded feelings, grave anxiety, etc., resulted therefrom."

The records of this case are bereft of any evidence showing that
respondent MIT acted in bad faith or in a wanton or fraudulent manner in
preventively suspending petitioner, thus, the Labor Arbiter was correct in not
awarding any damages in favor of petitioner.
164.TAMSON'S ENTERPRISES, INC., NELSON LEE, LILIBETH ONG and
JOHNSON NG vs. COURT OF APPEALS and ROSEMARIE L. SY, Respondents.

DOCTRINE: The power of the employer to terminate an employee on probation


is not without limitations. First, this power must be exercised in accordance with
the specific requirements of the contract. Second, the dissatisfaction on the part
of the employer must be real and in good faith, not feigned so as to circumvent
the contract or the law; and third, there must be no unlawful discrimination in
the dismissal. In termination cases, the burden of proving just or valid cause for
dismissing an employee rests on the employer.23

FACTS: Sy was hired by Tamsons as Assistant to the President. Despite the title,
she did not act as such because she was directed to act as payroll officer, though
she actually worked as a payroll clerk.5

Four days before she completed her sixth month of working in Tamsons, Ng, the
Sales Project Manager, called her to a meeting with him and Lee. They informed
Sy that her services would be terminated due to inefficiency. She was asked to
sign a letter of resignation and quitclaim.
During her pre-employment interview, Lee had nice comments about her good
work experience and educational background. She was assured of a long-term
employment with benefits. Throughout her employment, she earnestly
performed her duties, had a perfect attendance record, worked even during
brownouts and typhoons, and would often work overtime just to finish her work.

Sy claimed that her dismissal was highly suspicious as it took place barely four
(4) days prior to the completion of her six-month probationary period. The
petitioners did not show her any evaluation or appraisal report regarding her
alleged inefficient performance. Petitioners asserted that before Sy was hired,
she was apprised that she was being hired as a probationary employee for six
months subject to extension as a regular employee conditioned on her meeting
the standards of permanent employment set by the company. When she was
formally informed that her employment would end, Sy started threatening the
families of the petitioners with bodily harm. They pointed out that the
unpredictable attitude of Sy was one of the reasons for her not being considered
for regular employment.
Sy filed a case for illegal dismissal with claims for back wages, unpaid salary,
service incentive leave, overtime pay, 13th month pay, and moral and exemplary
damages, and attorneys fees.

ELA rendered a decision in favor of Sy, stating that a termination,


notwithstanding the probationary status, must be for a just cause. As there was
an absence of evidence showing just cause and due process, he found Sys
termination to be arbitrary and illegal.
NLRC reversed the ELAs finding that Sy was terminated without just cause and
without due process and dismissed the case.8

3E Andaya Ching Espiritu Hefti Galvez Gammad Lainez Lui Madamba Nagera Narvasa
Ong Palangdao Rosales Sanchez Santos Satrain Tabo (2014-2015)

Labor Relations Case Digest - Atty. Joyrich Golangco

CA rendered the assailed decision reversing the NLRC. It held that at the time Sy
was engaged as a probationary employee she was not informed of the standards
that she should meet to become a regular employee. Where an employee hired
on probationary basis was not informed of the standards that would qualify her
as a regular employee, she was deemed to have been hired from day one as a
regular employee. As a regular employee, she was entitled to security of tenure
and could be dismissed only for a just cause and after due compliance with
procedural due process. The CA added that the petitioners did not observe due
process in dismissing Sy.

ISSUES: Whether the termination of Sy, a probationary employee, was valid or


not.
HELD: The Court finds the petition devoid of merit.

The pertinent law governing the present case is Article 281 of the Labor Code.
There is no dispute that Sys employment with Tamsons was probationary in
character. Although a probationary or temporary employee with a limited
tenure, she was still entitled to a security of tenure.

Labor is given the protection during the probationary period of knowing the
company standards the new hires have to meet during the probationary period,
and to be judged on the basis of these standards. These standards should be
made known to the [employees] on probationary status at the start of their
probationary period, or xxx during which the probationary standards are to be
applied. The employer should show as a matter of due process how these
standards have been applied. In this case, the justification given by the
petitioners for Sys dismissal was her alleged failure to qualify by the companys
standard. Other than the general allegation that said standards were made
known to her at the time of her employment, however, no evidence,
documentary or otherwise, was presented to substantiate the same. Neither was
there any performance evaluation presented to prove that indeed hers was
unsatisfactory. Thus, this Court is in full accord with the ruling of the CA. This
case is bereft of any proof like an evaluation or assessment report which would
support private respondents claim that she failed to comply with the standards
in order to become a regular employee.

One of the conditions before an employer can terminate a probationary


employee is dissatisfaction on the part of the employer which must be real and in
good faith, not feigned so as to circumvent the contract or the law.

In its attempt to justify Sys dismissal, the petitioners relied on the ruling that the
constitutional protection ends on the expiration of the probationary period when
the parties are free to either renew or terminate their contract of employment.
Nonetheless, the power of the employer to terminate an employee on probation
is not without limitations. First, this power must be exercised in accordance with
the specific requirements of the contract. Second, the dissatisfaction on the part
of the employer must be real and in good faith, not feigned so as to circumvent
the contract or the law; and third, there must be no unlawful discrimination in
the dismissal. In termination cases, the burden of proving just or valid cause for
dismissing an employee rests on the employer.23
The dissatisfaction on the part of the petitioners was at best self-serving and
dubious as they could not present concrete and competent evidence establishing
her alleged incompetence. Failure on the part of the petitioners to discharge the
burden of proof is indicative that the dismissal was not justified.
The standards under which she would qualify as a regular employee not having
been communicated to her at the start of her probationary period, Sy qualified as
a regular employee.

Even on the assumption that Sy indeed failed to meet the standards set by them
and made known to the former at the time of her engagement, still, the
termination was flawed for failure to give the required notice to Sy as stated
under Section 2, Rule I, Book VI of the Implementing Rules. In this case, the
petitioners failed to comply with the requirement of a written notice. Sy was
merely verbally informed that her employment would be terminated. Being a
regular employee whose termination was illegal, Sy is entitled to the twin relief
of reinstatement and backwages. Likewise, having been compelled to come to
court and to incur expenses to protect her rights and interests, the award of
attorneys fees is in order.28

3E Andaya Ching Espiritu Hefti Galvez Gammad Lainez Lui Madamba Nagera Narvasa
Ong Palangdao Rosales Sanchez Santos Satrain Tabo (2014-2015)

WHEREFORE, the petition is DENIED.

Labor Relations Case Digest - Atty. Joyrich Golangco

165. Hacienda Primera Development Corp vs. Villegas

DOCTRINE: There are two grounds to legally terminate a probationary employee.


It may be done either: a) for a just cause; or b) when the employee fails to qualify
as a regular employee in accordance with reasonable standards made known by
the employer to the employee at the start of the employment.
FACTS:Hacienda Primera Development Corporation (Hacienda) hired Michael S.
Villegas as General Manager of Amorita Resort. He was hired as a probationary
employee for three (3) months. The standards upon which his regular
employment would be confirmed were not stated in the contract.

Respondent started working for petitioner on January 1, 2007. On March


14, 2007, he received a call from Paramount Consultancy and Management
telling him to report back to Manila. There, he learned that his services were
terminated. He, thus, asked for a written notice of termination, but did not
receive any. Hence, the complaint for illegal dismissal.
Hacienda, on the other hand, stated that respondent was hired as
probationary employee. It explained that respondents services were terminated
because he failed to qualify for regular employment. Specifically, it claimed that
respondent failed to conceptualize and complete financial budgets, sales
projection, room rates, website development, and marketing plan in coordination
with the Sales and Marketing Manager.

LA: Respondent was illegally dismissed. Ordered Hacienda to reinstate


complainant to his former position without loss of seniority rights and other
benefits and to pay complainant his backwages from the time he was dismissed
on March 15, 2007, until his actual reinstatement either physically or by payroll;
NLRC: Complaint of Illegal Dismissal is dismissed but ordered Hacienda to pay
respondent his salaries for the unexpired portion of his employment contract.
CA: LAs decision reinstated with the modification that since reinstatement is no

longer possible due to strained relations between the parties, a separation pay of
one month for every year of service is hereby decreed.
ISSUES: W/N theres illegal dismissal

HELD: YES. The Labor Code provides that: Art. 281. Probationary
Employment.Probationary employment shall not exceed six (6) months from
the date the employee started working, unless it is covered by an apprenticeship
agreement stipulating a longer period. The services of an employee who has
been engaged on a probationary basis may be terminated for a just cause or
when he fails to qualify as a regular employee in accordance with reasonable
standards made known by the employer to the employee at the time of his
engagement.
The Labor Code, Implementing Rules of Book VI, Rule I, Section 6
also state that: Sec. 6. Probationary employment. There is
probationary employment where the employee, upon his
engagement, is made to undergo a trial period during which the
employer determines his fitness to qualify for regular employment,
based on reasonable standards made known to him at the time of
engagement.

Petitioner Hacienda fails to specify the reasonable standards by which


respondents alleged poor performance was evaluated, much less to prove that
such standards were made known to him at the start of his employment. Thus, he
is deemed to have been hired from day one as a regular employee. Due process
dictates that an employee be apprised beforehand of the condition of his
employment and of the terms of advancement therein.
166. UNIVERSIDAD DE STA. ISABEL (USI) vs MARVIN-JULIAN L. SAMBAJON,
JR.

DOCTRINES: It is the Manual of Regulations for Private Schools, and not the
Labor Code, that determines whether or not a faculty member in an educational
institution has attained regular or permanent status.

3E Andaya Ching Espiritu Hefti Galvez Gammad Lainez Lui Madamba Nagera Narvasa
Ong Palangdao Rosales Sanchez Santos Satrain Tabo (2014-2015)

Labor Relations Case Digest - Atty. Joyrich Golangco

In a situation where the probationary status overlaps with a fixed-term contract


not specifically used for the fixed term it offers, Article 281 should assume
primacy and the fixed-period character of the contract must give way.

Notwithstanding the limited engagement of probationary employees, they are


entitled to constitutional protection of security of tenure during and before the
end of the probationary period. The services of an employee who has been
engaged on probationary basis may be terminated for any of the following: (a) a
just or (b) an authorized cause; and (c) when he fails to qualify as a regular
employee in accordance with reasonable standards prescribed by the employer.

[A probationary employee illegally dismissed is entitled to continue (his threeyear) probationary period. In the presence of distrained relations, he is entitled
to backwages confined to the probationary period, plus pro-rated 13th month
pay.]
FACTS: USI is a non-stock, non-profit religious educational institution in Naga
City. It hired Marvin-Julian Sambajon, Jr. (respondent) as a full-time college
faculty member of the Department of Religious Education with the rank of
Assistant Professor on probationary status, as evidenced by an Appointment
Contract effective November 1, 2002-March 30, 2003.

After the contract expired, petitioner continued to give teaching loads for the two
semesters of SY 2003-2004 and 2004-2005.
In June 2003, after respondent completed his course in Master of Arts in
Education, major in Guidance and Counseling, he submitted the corresponding
Special Order from (CHED) and his credentials for salary adjustment/increase.
His salary was increased, starting October 1-15, 2004. He was likewise re-ranked
from Assistant Professor to Associate Professor.
Respondent wrote a letter dated October 15, 2004 addressed to USI President,
Sr. Ma. Asuncion G. Evidente, D.C., where he argued that his salary increase
should be made effective as of June 2003. The school administration replied by
explaining its policy on re-ranking of faculty members, that re-ranking is done

every two years, and those undergoing probationary period and those on parttime basis of employment are not covered by this provision.

Respondent reiterated his demand for retroactive application of the salary


adjustment while USI was insistent in its position. To resolve the issue, a
dialogue was held between respondent and Sr. Evidente. Respondent claimed
that Sr. Evidente told him that the school administration had decided to shorten
his probationary period to two years on the basis of his satisfactory performance.
This was categorically denied by Sr. Evidente, saying that respondents
appointment as Associate Professor did not affect his status as a probationary
employee and that USI was not and did not exercise its prerogative to shorten his
probationary period to only two years.
On February 26, 2005, respondent received his letter of termination which stated
that upon expiration of his full-time probationary appointment on March 31,
2005, it will not be renewed.
Respondent filed a complaint for illegal dismissal against the petitioner.

LA: ruled that there was no just or authorized cause in the termination of
respondents probationary employment; consequently, petitioner was found
liable for illegal dismissal. Respondent school is hereby directed to pay
complainant full backwages covering the period/duration of the 1st semester of
academic year 2005-2006. Reinstatement being rendered moot by the expiration
of the probationary period, respondent school is directed to pay complainant
separation pay in lieu of reinstatement computed at one (1) months pay for
every year of service. An award of 10% attorneys fees in favor of complainant is
also held in order.
Petitioner appealed to the NLRC raising the issue of the correct interpretation of
Section 92 of the Manual of Regulations for Private Schools and DOLE-DECSCHED-TESDA Order No. 01, series of 1996, and alleging grave abuse of discretion
committed by the Labor Arbiter in ruling on a cause of action/issue not raised by
the complainant (respondent) in his position paper.

3E Andaya Ching Espiritu Hefti Galvez Gammad Lainez Lui Madamba Nagera Narvasa
Ong Palangdao Rosales Sanchez Santos Satrain Tabo (2014-2015)

Labor Relations Case Digest - Atty. Joyrich Golangco

NLRC: affirmed LA and held that respondent had acquired a permanent status
pursuant to Sections 91, 92 and 93 of the 1992 Manual of Regulations for Private
Schools, in relation to Article 281 of the Labor Code, as amended. NLRC based its
findings on the fact that of the five appointment contracts on record, only the
first and third were signed by respondent. The third appointment contract, for
the second semester of SY 2003-2004, reads in part:
"I am pleased to inform you that you are designated and commissioned
to be an Apostle of Love and Service, Unity and Peace as you dedicate
and commit yourself in the exercise of your duties and responsibilities
as a:

FULL-TIME FACULTY MEMBER


of the Religious Education Department from November 1, 2003 to March
31, 2004.
Unless otherwise renewed in writing this designation automatically
terminates as of the date expiration above stated without further
notice."

NLRC held that since respondent did not sign a contract for the first and second
semesters of SY 2004-2005, it must be concluded that the complainant has
acquired permanent status, based on Art 281 which provides that "an employee
who is allowed to work after a probationary period shall be considered a regular
employee."
It also held that Section 92 of the revised Manual of Regulations for Private
Schools merely provides for the maximum lengths of the probationary periods of
academic personnel of private schools and that the periods provided therein are
not requirements for the acquisition of permanent status.

CA: sustained the conclusion of the NLRC that respondent had already acquired
permanent status when he was allowed to continue teaching after the expiration
of his first appointment-contract on March 30, 2003. It also directed
respondents reinstatement and the payment of full back wages.
ISSUES

1.
2.
3.
4.

WON respondent had already acquired permanent status


2. WON the subject appointment contracts are fixed-term contracts
WON there was illegal dismissal
Relief and damages

HELD:
1. NO. The probationary employment of teachers in private schools is not
governed purely by the Labor Code. The Labor Code is supplemented with
respect to the period of probation by special rules found in the Manual of
Regulations for Private Schools. On the matter of probationary period, Section 92
of the 1992 Manual of Regulations for Private Schools regulations states:
Section 92. Probationary Period. Subject in all instances to compliance with the
Department and school requirements, the probationary period for academic
personnel shall not be more than (3) consecutive years of satisfactory service for
those in the elementary and secondary levels, (6) consecutive regular semesters
of satisfactory service for those in the tertiary level, and (9) consecutive
trimesters of satisfactory service for those in the tertiary level where collegiate
courses are offered on a trimester basis.
Thus, it is the Manual of Regulations for Private Schools, and not the Labor
Code, that determines whether or not a faculty member in an educational
institution has attained regular or permanent status. Section 93 of the 1992
Manual of Regulations for Private Schools provides that full-time teachers who
have satisfactorily completed their probationary period shall be considered
regular or permanent.

Since it was explicitly provided in the third contract that unless renewed in
writing respondents appointment automatically expires at the end of the
stipulated period of employment, the CA erred in concluding that simply because
the word "probationary" no longer appears below the designation (Full-Time
Faculty Member), respondent had already become a permanent employee.
It bears stressing that full-time teaching primarily refers to the extent of services
rendered by the teacher to the employer school and not to the nature of his

3E Andaya Ching Espiritu Hefti Galvez Gammad Lainez Lui Madamba Nagera Narvasa
Ong Palangdao Rosales Sanchez Santos Satrain Tabo (2014-2015)

Labor Relations Case Digest - Atty. Joyrich Golangco

appointment. Its significance lies in the rule that only full-time teaching
personnel can acquire regular or permanent status.

In this case, petitioner applied the maximum three-year probationary period


equivalent to six consecutive semesters provided in the Manual of Regulations.
The circumstance that respondents services were hired on semester basis did
not negate the applicable probationary period, which is 3 school years or 6
consecutive semesters. In Magis Young Achievers Learning Center37 the Court
explained the three years probationary period rule in this wise:

The common practice is for the employer and the teacher to enter into a contract,
effective for one school year. At the end of the school year, the employer has the
option not to renew the contract, particularly considering the teachers
performance. If the contract is not renewed, the employment relationship
terminates. If the contract is renewed, usually for another school year, the
probationary employment continues. Again, at the end of that period, the parties
may opt to renew or not to renew the contract. If renewed, this second renewal
of the contract for another school year would then be the last year since it
would be the third school year of probationary employment. At the end of this
third year, the employer may now decide whether to extend a permanent
appointment to the employee, primarily on the basis of the employee having met
the reasonable standards of competence and efficiency set by the employer. For
the entire duration of this three-year period, the teacher remains under
probation. Upon the expiration of his contract of employment, being simply on
probation, he cannot automatically claim security of tenure and compel the
employer to renew his employment contract. It is when the yearly contract is
renewed for the third time that Section 93 of the Manual becomes operative, and
the teacher then is entitled to regular or permanent employment status.
2. NO. Petitioner argues that respondents probationary period expires after each
semester he was contracted to teach and hence it was not obligated to renew his
services at the end of the fifth semester (March 2005) of his probationary
employment. Plainly, petitioner considered the subject appointment contracts as
fixed-term contracts such that it can validly dismiss respondent at the end of
each semester for the reason that his contract had expired.

The Court finds no merit in petitioners interpretation of the Manual of


Regulations, supplemented by DOLE-DECS-CHED-TESDA Order No. 01, series of
1996.

The fixed-term character of employment essentially refers to the period agreed


upon between the employer and the employee; employment exists only for the
duration of the term and ends on its own when the term expires. In a sense,
employment on probationary status also refers to a period because of the
technical meaning "probation" carries in Philippine labor law a maximum
period of six months, or in the academe, a period of three years for those engaged
in teaching jobs. Their similarity ends there, however, because of the overriding
meaning that being "on probation" connotes, i.e., a process of testing and
observing the character or abilities of a person who is new to a role or job.

To be sure, nothing is illegitimate in defining the school-teacher relationship in


this manner. The school, however, cannot forget that its system of fixed-term
contract is a system that operates during the probationary period and for this
reason is subject to the terms of Article 281 of the Labor Code. Unless this
reconciliation is made, the requirements of this Article on probationary status
would be fully negated as the school may freely choose not to renew contracts
simply because their terms have expired. The inevitable effect of course is to
wreck the scheme that the Constitution and the Labor Code established to
balance relationships between labor and management.

Given the clear constitutional and statutory intents, we cannot but conclude that
in a situation where the probationary status overlaps with a fixed-term
contract not specifically used for the fixed term it offers, Article 281 should
assume primacy and the fixed-period character of the contract must give
way. This conclusion is immeasurably strengthened by the petitioners and the
AMACCs hardly concealed expectation that the employment on probation could
lead to permanent status, and that the contracts are renewable unless the
petitioners fail to pass the schools standards.

3. YES. Notwithstanding the limited engagement of probationary employees, they


are entitled to constitutional protection of security of tenure during and before
the end of the probationary period.41 The services of an employee who has been

3E Andaya Ching Espiritu Hefti Galvez Gammad Lainez Lui Madamba Nagera Narvasa
Ong Palangdao Rosales Sanchez Santos Satrain Tabo (2014-2015)

Labor Relations Case Digest - Atty. Joyrich Golangco

engaged on probationary basis may be terminated for any of the following: (a) a
just or (b) an authorized cause; and (c) when he fails to qualify as a regular
employee in accordance with reasonable standards prescribed by the
employer.42

Thus, while no vested right to a permanent appointment had as yet accrued in


favor of respondent since he had not completed the prerequisite three-year
period (six consecutive semesters) necessary for the acquisition of permanent
status as required by the Manual of Regulations for Private Schools43 -- which has
the force of law44 -- he enjoys a limited tenure. During the said probationary
period, he cannot be terminated except for just or authorized causes, or if he fails
to qualify in accordance with reasonable standards prescribed by petitioner for
the acquisition of permanent status of its teaching personnel.

In a letter dated February 26, 2005, petitioner terminated the services of


respondent stating that his probationary employment as teacher will no longer
be renewed upon its expiry on March 31, 2005, respondents fifth semester of
teaching. No just or authorized cause was given by petitioner. Prior to this,
respondent had consistently achieved above average rating based on evaluation
by petitioners officials and students. He had also been promoted to the rank of
Associate Professor after finishing his masters degree course on his third
semester of teaching. Clearly, respondents termination after five semesters of
satisfactory service was illegal.

Respondent therefore is entitled to continue his three-year probationary period,


such that from March 31, 2005, his probationary employment is deemed
renewed for the following semester (1st semester of SY 2005-2006). However,
given the discordant relations that had arisen from the parties dispute, it can be
inferred with certainty that petitioner had opted not to retain respondent in its
employ beyond the three-year period.
4. We adhere to our disposition in Magis Young Achievers Learning Center45:

Finally, we rule on the propriety of the monetary awards. Petitioner, as


employer, is entitled to decide whether to extend respondent a
permanent status by renewing her contract beyond the three-year

period. Given the acrimony between the parties which must have been
generated by this controversy, it can be said unequivocally that
petitioner had opted not to extend respondent's employment beyond
this period. Therefore, the award of backwages as a consequence of the
finding of illegal dismissal in favor of respondent should be confined to
the three-year probationary period. xxx Moreover, respondent is also
entitled to receive her 13th month pay (pro-rated).

Petitioner Universidad de Sta. Isabel is hereby DIRECTED to PAY respondent


Marvin-Julian L. Sambajon, Jr. back wages corresponding to his full monthly
salaries for one semester (1st semester of SY 2005-2006) and pro-rated 13th
month pay.
167. UNIVAC DEVELOPMENT vs SORIANO

DOCTRINE: The power of the employer to terminate a probationary employee is


subject to three limitations, namely: (1) it must be exercised in accordance with
the specific requirements of the contract; (2) the dissatisfaction on the part of the
employer must be real and in good faith, not feigned so as to circumvent the
contract or the law; and (3) there must be no unlawful discrimination in the
dismissal.
FACTS: The case stemmed from the Complaint5 for Illegal Dismissal filed by
respondent against petitioner, the companys Chairperson Sadamu Watanabe
(Watanabe), and the Head of the Engineering Department Johnny Castro
(Castro). Admittedly, respondent was hired on August 23, 2004 by petitioner on
probationary basis as legal assistant of the company with a monthly salary
of P15,000.00. Respondent claimed that on February 15, 2005, or eight (8) days
prior to the completion of his six months probationary period, Castro allegedly
informed him that he was being terminated from employment due to the
companys cost-cutting measures. He allegedly asked for a thirty-day notice but
his termination was ordered to be effective immediately. Thus, he was left with
no choice but to leave the company.

Petitioner, on the other hand, denied the allegations of respondent and claimed
instead that prior to his employment; respondent was informed of the standards

3E Andaya Ching Espiritu Hefti Galvez Gammad Lainez Lui Madamba Nagera Narvasa
Ong Palangdao Rosales Sanchez Santos Satrain Tabo (2014-2015)

Labor Relations Case Digest - Atty. Joyrich Golangco

required for regularization. Petitioner also supposedly informed him of his


duties and obligations which included safekeeping of case folders, proper
coordination with the companys lawyers, and monitoring of the status of the
cases filed by or against the company. Petitioner recalled that on January 5, 2005,
a company meeting was held where respondent allegedly expressed his intention
to leave the company because he wanted to review for the bar examinations. It
was also in that meeting where he was informed of his unsatisfactory
performance in the company. Thus, when respondent did not report for work on
February 16, 2005, petitioner assumed that he pushed through with his plan to
leave the company. In other words, petitioner claimed that respondent was not
illegally dismissed from employment; rather, he in fact abandoned his job by his
failure to report for work.
ISSUES: whether respondent was illegally dismissed from employment by
petitioner
HELD: Yes. It is undisputed that respondent was hired as a probationary
employee. As such, he did not enjoy a permanent status. Nevertheless, he is
accorded the constitutional protection of security of tenure which means that he
can only be dismissed from employment for a just cause or when he fails to
qualify as a regular employee in accordance with reasonable standards made
known to him by the employer at the time of his engagement.

It is primordial that at the start of the probationary period, the standards for
regularization be made known to the probationary employee. In this case, as held
by the CA, petitioner failed to present adequate evidence to substantiate its claim
that respondent was apprised of said standards. It is evident from the LA and
NLRC decisions that they merely relied on surmises and presumptions in
concluding that respondent should have known the standards considering his
educational background as a law graduate. Equally important is the requirement
that in order to invoke "failure to meet the probationary standards" as a
justification for dismissal, the employer must show how these standards have
been applied to the subject employee. In this case, aside from its bare allegation,
it was not shown that a performance evaluation was conducted to prove that his
performance was indeed unsatisfactory.

Indeed, the power of the employer to terminate a probationary employee is


subject to three limitations, namely: (1) it must be exercised in accordance with
the specific requirements of the contract; (2) the dissatisfaction on the part of the
employer must be real and in good faith, not feigned so as to circumvent the
contract or the law; and (3) there must be no unlawful discrimination in the
dismissal. In this case, not only did petitioner fail to show that respondent was
apprised of the standards for regularization but it was likewise not shown how
these standards had been applied in his case.
Pursuant to well-settled doctrine, petitioners failure to specify the reasonable
standards by which respondents alleged poor performance was evaluated as
well as to prove that such standards were made known to him at the start of his
employment, makes respondent a regular employee. In other words, because of
this omission on the part of petitioner, respondent is deemed to have been hired
from day one as a regular employee.

To justify the dismissal of an employee, the employer must, as a rule, prove that
the dismissal was for a just cause and that the employee was afforded due
process prior to dismissal. We find no reason to depart from the CA conclusion
that respondents termination from employment is without just and valid
ground. Neither was due process observed, making his termination illegal. He is,
therefore, entitled to the twin relief of reinstatement and backwages granted
under the Labor Code. However, as aptly held by the CA, considering the strained
relations between petitioner and respondent, separation pay should be awarded
in lieu of reinstatement. This Court has consistently ruled that if reinstatement is
no longer feasible, backwages shall be computed from the time of illegal
dismissal until the date the decision becomes final. Separation pay, on the other
hand, is equivalent to at least one month pay, or one month pay for every year of
service, whichever is higher (with a fraction of at least six months being
considered as one whole year), computed from the time of employment or
engagement up to the finality of the decision.
Having been forced to litigate in order to seek redress of his grievances,
respondent is entitled to the payment of attorneys fees equivalent to 10% of his
monetary award. Pursuant to prevailing jurisprudence, legal interest shall be

3E Andaya Ching Espiritu Hefti Galvez Gammad Lainez Lui Madamba Nagera Narvasa
Ong Palangdao Rosales Sanchez Santos Satrain Tabo (2014-2015)

Labor Relations Case Digest - Atty. Joyrich Golangco

imposed on the monetary awards herein granted at the rate of 6% per annum
from date of termination until full payment.
168. Abbott vs Alcaraz

DOCTRINE: In all cases of probationary employment, the employer shall make


known to the employee the standards under which he will qualify as a regular
employee at the time of his engagement. Where no standards are made known to
the employee at that time, he shall be deemed a regular employee.
FACTS: Petitioner caused the publication in a major broadsheet newspaper of
its need for a Medical and Regulatory Affairs Manager. Alcaraz - who was then a
Regulatory Affairs and Information Manager at Aventis Pasteur Philippines,
Incorporated (another pharmaceutical company like Abbott) showed interest
and submitted her application. Abbott formally offered Alcaraz the position. In
Abbotts offer sheet it was stated that Alcaraz was to be employed on a
probationary basis. Later that day, she accepted the said offer. Alcaraz signed an
employment contract which stated, inter alia, that she was to be placed on
probation for a period of six (6) months. Petitioner Misa, Abbotts HR Director,
sent Alcaraz an e-mail which contained an explanation of the procedure for
evaluating the performance of probationary employees and further indicated
that Abbott had only one evaluation system for all of its employees.
Alcaraz was called to a meeting where she was informed that she failed
to meet the regularization standards for the position of Regulatory Affairs
Manager. Thereafter, Walsh requested Alcaraz to tender her resignation, else
they be forced to terminate her services. Petitioner personally handed to Alcaraz
a letter stating that her services had been terminated which detailed the reasons
for Alcarazs termination, such as failure to manage his time and to train his staff
effectively.
Alcaraz then filed an illegal dismissal case. She claimed that she should
have already been considered as a regular and not a probationary employee
given Abbotts failure to inform her of the reasonable standards for her
regularization upon her engagement. On the contrary, petitioners maintained
that Alcaraz was validly terminated from her probationary employment given
her failure to satisfy the prescribed standards for her regularization which were
made known to her at the time of her engagement.

LA dismissed respondents complaint stating that she was informed by


the petitioner of the standards and as such for his failure to comply with the
standards, he was validly dismissed. . NLRC reversed LAs ruling, which was
affirmed by the CA.
ISSUES: 1. whether or not Alcaraz was sufficiently informed of the reasonable
standards to qualify her as a regular employee?
2. whether or not Alcaraz was validly terminated from her employment?

HELD: 1. Yes. Section 6(d), Rule I, Book VI of the Implementing Rules of the
Labor Code provides:
d) In all cases of probationary employment, the employer shall make
known to the employee the standards under which he will qualify as a regular
employee at the time of his engagement. Where no standards are made known to
the employee at that time, he shall be deemed a regular employee.

Keeping with these rules, an employer is deemed to have made known


the standards that would qualify a probationary employee to be a regular
employee when it has exerted reasonable efforts to apprise the employee of what
he is expected to do or accomplish during the trial period of probation. This goes
without saying that the employee is sufficiently made aware of his probationary
status as well as the length of time of the probation.
The exception to the foregoing is when the job is self-descriptive in
nature, for instance, in the case of maids, cooks, drivers, or messengers.

A punctilious examination of the records reveals that Abbott had indeed


complied with the above-stated requirements. This conclusion is largely impelled
by the fact that Abbott clearly conveyed to Alcaraz her duties and responsibilities
as Regulatory Affairs Manager prior to, during the time of her engagement, and
the incipient stages of her employment. On this score, the Court finds it apt to
detail not only the incidents which point out to the efforts made by Abbott but
also those circumstances which would show that Alcaraz was well-apprised of
her employers expectations that would, in turn, determine her regularization:

3E Andaya Ching Espiritu Hefti Galvez Gammad Lainez Lui Madamba Nagera Narvasa
Ong Palangdao Rosales Sanchez Santos Satrain Tabo (2014-2015)

Labor Relations Case Digest - Atty. Joyrich Golangco

a) Abbott caused the publication in a major broadsheet newspaper of its need for
a Regulatory Affairs Manager, indicating therein the job description for as well as
the duties and responsibilities attendant to the aforesaid position; this prompted
Alcaraz to submit her application to Abbott on
(b) In Abbotts offer sheet, it was stated that Alcaraz was to be employed on a
probationary status;
(c) Alcaraz signed an employment contract which specifically stated, inter alia,
that she was to be placed on probation for a period of six (6) months;

(d) On the day Alcaraz accepted Abbotts employment offer, Bernardo sent her
copies of Abbotts organizational structure and her job description through email;

(e) Alcaraz was made to undergo a pre-employment orientation where Almazar


informed her that she had to implement Abbotts Code of Conduct and office
policies on human resources and finance and that she would be reporting
directly to Walsh;
(f) Alcaraz was also required to undergo a training program as part of her
orientation;

(g) Alcaraz received copies of Abbotts Code of Conduct and Performance


Modules from Misa who explained to her the procedure for evaluating the
performance of probationary employees; she was further notified that Abbott
had only one evaluation system for all of its employees; and

(h) Moreover, Alcaraz had previously worked for another pharmaceutical


company and had admitted to have an "extensive training and background" to
acquire the necessary skills for her job.

2. YES. Considering the totality of the above-stated circumstances, it cannot,


therefore, be doubted that Alcaraz was well-aware that her regularization would
depend on her ability and capacity to fulfill the requirements of her position as

Regulatory Affairs Manager and that her failure to perform such would give
Abbott a valid cause to terminate her probationary employment.

While there lies due cause to terminate Alcarazs probationary


employment for her failure to meet the standards required for her regularization,
and while it must be further pointed out that Abbott had satisfied its statutory
duty to serve a written notice of termination, the fact that it violated its own
company procedure renders the termination of Alcarazs employment
procedurally infirm, warranting the payment of nominal damages.

Records show that Abbotts PPSE procedure mandates, inter alia, that
the job performance of a probationary employee should be formally reviewed
and discussed with the employee at least twice: first on the third month and
second on the fifth month from the date of employment. Abbott is also required
to come up with a Performance Improvement Plan during the third month
review to bridge the gap between the employees performance and the standards
set, if any. In addition, a signed copy of the PPSE form should be submitted to
Abbotts HRD as the same would serve as basis for recommending the
confirmation or termination of the probationary employment.
In this case, it is apparent that Abbott failed to follow the above-stated
procedure in evaluating Alcaraz. For one, there lies a hiatus of evidence that a
signed copy of Alcarazs PPSE form was submitted to the HRD. It was not even
shown that a PPSE form was completed to formally assess her performance.
Neither was the performance evaluation discussed with her during the third and
fifth months of her employment. Nor did Abbott come up with the necessary
Performance Improvement Plan to properly gauge Alcarazs performance with
the set company standards.
169. Colegio de Santisimo Rosario v. Rojo

DOCTRINE: It is incumbent upon the school to have not only set reasonable
standards to be followed by said teachers in determining qualification for regular
employment, the same must have also been communicated to the teachers at the
start of the probationary period, or at the very least, at the start of the period
when they were to be applied. These terms, in addition to those expressly
provided by the Labor Code, would serve as the just cause for the termination of

3E Andaya Ching Espiritu Hefti Galvez Gammad Lainez Lui Madamba Nagera Narvasa
Ong Palangdao Rosales Sanchez Santos Satrain Tabo (2014-2015)

Labor Relations Case Digest - Atty. Joyrich Golangco

the probationary contract. The law is clear that in all cases of probationary
employment, the employer shall convey to the employee the standards under
which he will qualify as a regular employee at the time of his engagement. Where
no standards are made known to the employee at that time, he shall be deemed a
regular employee.
FACTS: Petitioner Colegio del Santisimo Rosario (CSR) hired respondent as a
high school teacher on probationary basis for the school years 1992-1993, 199319947 and 1994-1995.
On April 5, 1995, CSR, through petitioner Sr. Zenaida S. Mofada, OP (Mofada),
decided not to renew respondents services.

Thus, on July 13, 1995, respondent filed a Complaint for illegal dismissal. He
alleged that since he had served three consecutive school years which is the
maximum number of terms allowed for probationary employment, he should be
extended permanent employment. Citing paragraph 75 of the 1970 Manual of
Regulations for Private Schools (1970 Manual), respondent asserted that "fulltime teachers who have rendered three (3) consecutive years of satisfactory
services shall be considered permanent."

On the other hand, petitioners argued that respondent knew that his Teachers
Contract for school year 1994-1995 with CSR would expire on March 31,
1995. Accordingly, respondent was not dismissed but his probationary contract
merely expired and was not renewed. Petitioners also claimed that the "three
years" mentioned in paragraph 75 of the 1970 Manual refer to "36 months," not
three school years. And since respondent served for only three school years of 10
months each or 30 months, then he had not yet served the "three years" or 36
months mentioned in paragraph 75 of the 1970 Manual.
The LA ruled that "three school years" means three years of 10 months, not
12 months. Considering that respondent had already served for three
consecutive school years, then he has already attained regular employment
status. Thus, the non-renewal of his contract for school year 1995-1996
constitutes illegal dismissal.
The LA also found petitioners guilty of bad faith when they treated respondents
termination merely as the expiration of the third employment contract and when
they insisted that the school board actually deliberated on the non-renewal of

respondents employment without submitting admissible proof of his alleged


regular performance evaluation.

On appeal, the NLRC affirmed the LAs Decision with modification. It held that
after serving three school years, respondent had attained the status of
regular employment especially because CSR did not make known to
respondent the reasonable standards he should meet. The NLRC also agreed
with the LA that respondents termination was done in bad faith. It held that
respondent is entitled to reinstatement, if viable; or separation pay, if
reinstatement was no longer feasible, and backwages.
ISSUES: Whether Respondent was a probationary employee.

HELD: Respondent was a regular employee. It is incumbent upon the school


to have not only set reasonable standards to be followed by said teachers in
determining qualification for regular employment, the same must have
also been communicated to the teachers at the start of the probationary
period, or at the very least, at the start of the period when they were to be
applied. These terms, in addition to those expressly provided by the Labor Code,
would serve as the just cause for the termination of the probationary contract.
The law is clear that in all cases of probationary employment, the employer
shall convey to the employee the standards under which he will qualify as a
regular employee at the time of his engagement. Where no standards are
made known to the employee at that time, he shall be deemed a regular
employee.
In this case, glaringly absent from petitioners evidence are the reasonable
standards that respondent was expected to meet that could have served as
proper guidelines for purposes of evaluating his performance. Nowhere in the
Teachers Contract could such standards be found. Neither was it mentioned that
the same were ever conveyed to respondent. Even assuming that respondent
failed to meet the standards set forth by CSR and made known to the former at
the time he was engaged as a teacher on probationary status, still, the
termination was flawed for failure to give the required notice to respondent.
Curiously, despite the absence of standards, Mofada mentioned the existence of
alleged performance evaluations in respondents case. The Court is, however, in a
quandary as to what could have been the basis of such evaluation, as no

3E Andaya Ching Espiritu Hefti Galvez Gammad Lainez Lui Madamba Nagera Narvasa
Ong Palangdao Rosales Sanchez Santos Satrain Tabo (2014-2015)

Labor Relations Case Digest - Atty. Joyrich Golangco

evidence were adduced to show the reasonable standards with which


respondents performance was to be assessed or that he was informed
thereof. Notably too, none of the supposed performance evaluations were
presented. These flaws violated respondents right to due process. As such, his
dismissal is, for all intents and purposes, illegal.
It should be pointed out that absent any showing of unsatisfactory performance
on the part of respondent, it can be presumed that his performance was
satisfactory, especially taking into consideration the fact that even while he was
still more than a year into his probationary employment, he was already
designated Prefect of Discipline. In such capacity, he was able to uncover the
existence of a drug syndicate within the school and lessen the incidence of drug
use therein. Yet despite respondents substantial contribution to the school,
petitioners chose to disregard the same and instead terminated his services;
while most of those who were involved in drug activities within the school were
punished with a slap on the wrist as they were merely made to write letters
promising that the incident will not happen again.
170. Philippine Daily Inquirer vs Leon m. Magtibay

DOCTRINE: A probationary employee, as understood under Article 282 (now


Article 281) of the Labor Code, is one who is on trial by an employer during
which the employer determines whether or not he is qualified for permanent
employment. A probationary appointment is made to afford the employer an
opportunity to observe the fitness of a probationer while at work, and to
ascertain whether he will become a proper and efficient employee. It is well
settled that the employer has the right or is at liberty to choose who will be hired
and who will be denied employment. In that sense, it is within the exercise of the
right to select his employees that the employer may set or fix a probationary
period within which the latter may test and observe the conduct of the former
before hiring him permanently
FACTS: PDI hired Magtibay, on contractual basis, to assist, for a period of five
months as a regular phone operator. Before the expiration of Magtibays
contractual employment, he and PDI agreed to a fifteen-day contract extension,
under the same conditions as the existing contract. After the expiration of
Magtibays contractual employment, as extended, PDI announced the creation

and availability of a new position for a second telephone operator who would
undergo probationary employment. Apparently, it was PDIs policy to accord
regular employees preference for new vacancies in the company. Thus, Ms.
Regina M. Layague, a PDI employee and member of respondent PDI Employees
Union (PDIEU), filed her application for the new position. However, she later
withdrew her application, paving the way for outsiders or non-PDI employees,
like Magtibay in this case, to apply. After the usual interview for the second
telephone operator slot, PDI chose to hire Magtibay on a probationary basis for a
period of six (6) months. A week before the end the agreed 6-month
probationary period, PDI officer Benita del Rosario handed Magtibay his
termination paper, grounded on his alleged failure to meet company standards.
Aggrieved, Magtibay immediately filed a complaint for illegal dismissal and
damages before the Labor Arbiter. PDIEU later joined the fray by filing a
supplemental complaint for unfair labor practice. Magtibay anchored his case by
contending that he has become a regular employee by operation of law because
he worked for PDI for 10 months and that there was no basis of his termination
because he was not apprised at the beginning of his employment of the
performance standards of the company. On the other hand, PDI denied all of
defendants allegations and alleged that the period covered by the contractual
employment cannot be counted with or tacked to the period for probation. LA
favored PDI and dismissed Magtibays complaint. According to the Labor Arbiter,
upon termination of the period stated therein, the contractual employment was
also effectively terminated, implying that Magtibay was merely on a
probationary status when his services were terminated. PDIEU and MAgtibay
appealed, and the NLRC reversed the ruling and also denied petitioners MR thus
prompting the respondents to file a petition for certiorari with the CA which
denied due course to PDIs petition.
Note: Magtibays violations; (1) he repeatedly violated the company rule
prohibiting unauthorized persons from entering the telephone operators room;
(2) he intentionally omitted to indicate in his application form his having a
dependent child; and (3) he exhibited lack of sense of responsibility by locking
the door of the telephone operators room on March 10, 1996 without switching
the proper lines to the company guards so that incoming calls may be answered
by them.

3E Andaya Ching Espiritu Hefti Galvez Gammad Lainez Lui Madamba Nagera Narvasa
Ong Palangdao Rosales Sanchez Santos Satrain Tabo (2014-2015)

Labor Relations Case Digest - Atty. Joyrich Golangco

ISSUES: WON Magtibays termination is a violation of his right to due process


and security of tenure as an employee of PDI for having employed therein for
more than 6 months

HELD: No,Within the limited legal six-month probationary period, probationary


employees are still entitled to security of tenure. It is expressly provided in the
afore-quoted Article 281 that a probationary employee may be terminated only
on two grounds: (a) for just cause, or (b) when he fails to qualify as a regular
employee in accordance with reasonable standards made known by the
employer to the employee at the time of his engagement. It is on record that
Magtibay committed obstinate infractions of company rules and regulations,
which in turn constitute sufficient manifestations of his inadequacy to meet
reasonable employment norms. The suggestion that Magtibay ought to have been
made to understand during his briefing and orientation that he is expected to
obey and comply with company rules and regulations strains credulity for
acceptance. All employees, be they regular or probationary, are expected to
comply with company-imposed rules and regulations, else why establish them in
the first place. Probationary employees unwilling to abide by such rules have no
right to expect, much less demand, permanent employment. Unlike under the
first ground for the valid termination of probationary employment which is for
just cause, the second ground does not require notice and hearing. Due process
of law for this second ground consists of making the reasonable standards
expected of the employee during his probationary period known to him at the
time of his probationary employment. By the very nature of a probationary
employment, the employee knows from the very start that he will be under close
observation and his performance of his assigned duties and functions would be
under continuous scrutiny by his superiors.
171. ALCIRA V. NLRC

DOCTRINE: Our computation of the 6-month probationary period is reckoned


from the date of appointment up to the same calendar date of the 6th month
following.(italics supplied)

In short, since the number of days in each particular month was irrelevant,
petitioner was still a probationary employee when respondent Middleby opted
not to regularize him on November 20, 1996.

FACTS: Respondent Middleby Philippines Corporation (Middleby) hired


petitioner as engineering support services supervisor on a probationary basis for
six months. Apparently unhappy with petitioners performance, respondent
Middleby terminated petitioners services. The parties, presenting their
respective copies of Alciras appointment paper, claimed conflicting starting
dates of employment: May 20, 1996 according to petitioner and May 27, 1996
according to respondent. Both documents indicated petitioners employment
status as probationary (6 mos.) and a remark that after five months
(petitioners) performance shall be evaluated and any adjustment in salary shall
depend on (his) work performance.
Petitioner asserts that, on November 20, 1996, in the presence of his co-workers
and subordinates, a senior officer of respondent Middleby in bad faith withheld
his time card and did not allow him to work. Considering this as a dismissal
after the lapse of his probationary employment, petitioner filed on November
21, 1996 a complaint in the National Labor Relations Commission (NLRC) against
respondent Middleby contending that he had already become a regular employee
as of the date he was illegally dismissed. In their defense, respondents claim that,
during petitioners probationary employment, he showed poor performance in
his assigned tasks, incurred ten absences, was late several times and violated
company rules on the wearing of uniform. Since he failed to meet company
standards, petitioners application to become a regular employee was
disapproved and his employment was terminated.
LA dismissed the complaint and held that:
(1) respondents were able to prove that petitioner was apprised of the standards
for becoming a regular employee;
(2) respondent Mamaradlos affidavit showed that petitioner did not perform
well in his assigned work and his attitude was below par compared to the
companys standard required of him and
(3) petitioners dismissal on November 20, 1996 was before his regularization,
considering that, counting from May 20, 1996, the six-month probationary
period ended on November 20, 1996.

3E Andaya Ching Espiritu Hefti Galvez Gammad Lainez Lui Madamba Nagera Narvasa
Ong Palangdao Rosales Sanchez Santos Satrain Tabo (2014-2015)

Labor Relations Case Digest - Atty. Joyrich Golangco


NLRC and CA affirmed the ruling.
ISSUES: WON Alcira was illegally dismissed

HELD: NO, 1. Petitioner insists that he already attained the status of a regular
employee when he was dismissed on November 20, 1996 because, having started
work on May 20, 1996, the six-month probationary period ended on November
16, 1996. According to petitioners computation, since Article 13 of the Civil Code
provides that one month is composed of thirty days, six months total one
hundred eighty days. As the appointment provided that petitioners status was
probationary (6 mos.) without any specific date of termination, the 180th day
fell on November 16, 1996. Thus, when he was dismissed on November 20, 1996,
he was already a regular employee.

Petitioners contention is incorrect. In CALS Poultry Supply Corporation, et. al. vs.
Roco, et. al.,1[11] this Court dealt with the same issue of whether an employment
contract from May 16, 1995 to November 15, 1995 was within or outside the sixmonth probationary period. We ruled that November 15, 1995 was still within
the six-month probationary period. We reiterate our ruling in CALS Poultry
Supply:
(O)ur computation of the 6-month probationary period is reckoned from the
date of appointment up to the same calendar date of the 6th month
following.(italics supplied)

In short, since the number of days in each particular month was irrelevant,
petitioner was still a probationary employee when respondent Middleby opted
not to regularize him on November 20, 1996.

2. We hold that respondent Middleby substantially notified petitioner of the


standards to qualify as a regular employee when it apprised him, at the start of
his employment, that it would evaluate his supervisory skills after five months.
3. Although we can regard petitioners severance from work as dismissal, the
same cannot be deemed illegal. As found by the labor arbiter, the NLRC and the

Court of Appeals, petitioner (1) incurred ten absences (2) was tardy several
times (3) failed to wear the proper uniform many times and (4) showed inferior
supervisory skills. Petitioner failed to satisfactorily refute these substantiated
allegations. Taking all this in its entirety, respondent Middleby was clearly
justified to end its employment relationship with petitioner.
172. MERCADO vs. AMA COMPUTER COLLEGE-PARAAQUE CITY, INC.
(AMACC)

DOCTRINE: If the employer were to apply probationary standards, these


standards must not only be reasonable but must have also been communicated
to the teachers at the start of the probationary period, or at the very least, at the
start of the period when they were to be applied. These terms, in addition to
those expressly provided by the Labor Code, would serve as the just cause for the
termination of the probationary contract. The details of this nding of just cause
must be communicated to the affected teachers as a matter of due process.
FACTS: AMACC is an educational institution engaged in computer-based
education in the country. The petitioners were faculty members who started
teaching at AMACC on May 25, 1998. The petitioners executed individual
Teachers Contracts for each of the trimesters that they were engaged to teach,
with the following common stipulation:

1. POSITION. The TEACHER has agreed to accept a non-tenured appointment to


work in the College of xxx effective xxx to xxx or for the duration of the last term
that the TEACHER is given a teaching load based on the assignment duly
approved by the DEAN/SAVP-COO.

For the school year 2000-2001, AMACC implemented new faculty screening
guidelines, set forth in its Guidelines on the Implementation of AMACC Faculty
Plantilla. Under the new screening guidelines, teachers were to be hired or
maintained based on extensive teaching experience, capability, potential, high
academic qualications and research background. The performance standards
under the new screening guidelines were also used to determine the present
faculty members entitlement to salary increases. The petitioners failed to obtain

3E Andaya Ching Espiritu Hefti Galvez Gammad Lainez Lui Madamba Nagera Narvasa
Ong Palangdao Rosales Sanchez Santos Satrain Tabo (2014-2015)

Labor Relations Case Digest - Atty. Joyrich Golangco

a passing rating based on the performance standards; hence AMACC did not give
them any salary increase.
Because of AMACCs action on the salary increases, the petitioners led a
complaint with the Arbitration Branch of the NLRC on July 25, 2000, for
underpayment of wages, non-payment of overtime and overload compensation,
13th month pay, and for discriminatory practices.
On September 7, 2000, the petitioners individually received a memorandum
from AMACC informing them that with the expiration of their contract to teach,
their contract would no longer be renewed.

The petitioners amended their labor arbitration complaint to include the charge
of illegal dismissal against AMACC. Petitioners claimed that their dismissal was
illegal because it was made in retaliation for their complaint for monetary
benets and discriminatory practices against AMACC. The petitioners also
contended that AMACC failed to give them adequate notice; hence, their
dismissal was ineffectual.

AMACC contended in response that the petitioners worked under a contracted


term under a non-tenured appointment and were still within the three-year
probationary period for teachers. Their contracts were not renewed for the
following term because they failed to pass the Performance Appraisal System for
Teachers (PAST) while others failed to comply with the other requirements for
regularization, promotion, or increase in salary. This move, according to AMACC,
was justied since the school has to maintain its high academic standards.

The LA ruled that Article 281 of the Labor Code on probationary employment
applied to the case; that AMACC allowed the petitioners to teach for the rst
semester of school year 2000-2001; that AMACC did not specify who among the
petitioners failed to pass the PAST and who among them did not comply with the
other requirements of regularization, promotions or increase in salary; and that
the petitioners dismissal could not be sustained on the basis of AMACCs "vague
and general allegations" without substantial factual basis. On appeal, the NLRC
denied AMACCs appeal for lack of merit and afrmed in toto the LAs ruling.

The CA effectively found reasonable basis for AMACC not to renew the
petitioners contracts. To the CA, the petitioners were not actually dismissed;
their respective contracts merely expired and were no longer renewed by
AMACC because they failed to satisfy the schools standards for the school year
2000-2001 thatmeasured their tness and aptitude to teach as regular faculty
members. The CA emphasized that in the absence of any evidence of bad faith on
AMACCs part, the court would not disturb or nullify its discretion to set
standards and to select for regularization only the teachers who qualify, based on
reasonable and non- discriminatory guidelines.
The CA disagreed with the NLRCs ruling that the new guidelines for the school
year 2000-20001 could not be imposed on the petitioners and their employment
contracts. The appellate court opined that AMACC has the inherent right to
upgrade the quality of computer education it offers to the public; part of this
pursuit is the implementation of continuing evaluation and screening of its
faculty members for academic excellence. The CA noted that the nature of
education AMACC offers demands that the school constantly adopt progressive
performance standards for its faculty to ensure that they keep pace with the
rapid developments in the eld of information technology.
ISSUES: Was there illegal dismissal despite the fact that the petitioners where
probationary employees and employed under term to term contracts?

HELD: Yes. The existence of the term-to-term contracts covering the petitioners
employment is not disputed, nor is it disputed that they were on probationary
status not permanent or regular status from the time they were employed on
May 25, 1998 and until the expiration of their Teaching Contracts on September
7, 2000. This case brings to the fore the essential question of which, between the
two factors affecting employment, should prevail given AMACCs position that
the teachers contracts expired and it had the right not to renew them. In other
words, should the teachers probationary status be disregarded simply because
the contracts were xed-term?
On the one hand, employment on probationary status affords management the
chance to fully scrutinize the true worth of hired personnel before the full force
of the security of tenure guarantee of the Constitution comes into play.37 Based

3E Andaya Ching Espiritu Hefti Galvez Gammad Lainez Lui Madamba Nagera Narvasa
Ong Palangdao Rosales Sanchez Santos Satrain Tabo (2014-2015)

Labor Relations Case Digest - Atty. Joyrich Golangco

on the standards set at the start of the probationary period, management is given
the widest opportunity during the probationary period to reject hires who fail to
meet its own adopted but reasonable standards.

Labor, for its part, is given the protection during the probationary period of
knowing the company standards the new hires have to meet during the
probationary period, and to be judged on the basis of these standards, aside from
the usual standards applicable to employees after they achieve permanent status.
Under the terms of the Labor Code, these standards should be made known to
the teachers on probationary status at the start of their probationary period. Of
critical importance in invoking a failure to meet the probationary standards, is
that the school should show as a matter of due process how these standards
have been applied. This is effectively the second notice in a dismissal situation
that the law requires as a due process guarantee supporting the security of
tenure provision, and is in furtherance, too, of the basic rule in employee
dismissal that the employer carries the burden of justifying a dismissal. These
rules ensure compliance with the limited security of tenure guarantee the law
extends to probationary employees.

Understood in the above sense, the essentially protective character of


probationary status for management can readily be appreciated. But this same
protective character gives rise to the countervailing but equally protective rule
that the probationary period can only last for a specic maximum period and
under reasonable, well-laid and properly communicated standards. Otherwise
stated, within the period of the probation, any employer move based on the
probationary standards and affecting the continuity of the employment must
strictly conform to the probationary rules.

Under the given facts where the school year is divided into trimesters, the school
apparently utilizes its xed- term contracts as a convenient arrangement
dictated by the trimestral system and not because the workplace parties really
intended to limit the period of their relationship to any xed term and to nish
this relationship at the end of that term. If we pierce the veil, so to speak, of the
parties so-called xed-term employment contracts, what undeniably comes out
at the core is a xed-term contract conveniently used by the school to dene and
regulate its relations with its teachers during their probationary period.1avvphi1

Given the clear constitutional and statutory intents, we cannot but conclude that
in a situation where the probationary status overlaps with a xed-term contract
not specically used for the xed term it offers, Article 281 should assume
primacy and the xed-period character of the contract must give way. This
conclusion is immeasurably strengthened by the petitioners and the AMACCs
hardly concealed expectation that the employment on probation could lead to
permanent status, and that the contracts are renewable unless the petitioners fail
to pass the schools standards.
If the school were to apply the probationary standards (as in fact it says it did in
the present case), these standards must not only be reasonable but must have
also been communicated to the teachers at the start of the probationary period,
or at the very least, at the start of the period when they were to be applied. These
terms, in addition to those expressly provided by the Labor Code, would serve as
the just cause for the termination of the probationary contract. As explained
above, the details of this nding of just cause must be communicated to the
affected teachers as a matter of due process.
AMACC, by its submissions, admits that it did not renew the petitioners
contracts because they failed to pass the Performance Appraisal System for
Teachers (PAST) and other requirements for regularization that the school
undertakes to maintain its high academic standards. The evidence is unclear on
the exact terms of the standards, although the school also admits that these were
standards under the Guidelines on the Implementation of AMACC Faculty
Plantilla put in place at the start of school year 2000-2001.

While we can grant that the standards were duly communicated to the
petitioners and could be applied beginning the 1st trimester of the school year
2000-2001, glaring and very basic gaps in the schools evidence still exist. The
exact terms of the standards were never introduced as evidence; neither does the
evidence show how these standards were applied to the petitioners. Without
these pieces of evidence (effectively, the nding of just cause for the non-renewal
of the petitioners contracts), we have nothing to consider and pass upon as valid
or invalid for each of the petitioners. Inevitably, the non-renewal (or effectively,
the termination of employment of employees on probationary status) lacks the
supporting nding of just cause that the law requires and, hence, is illegal.

3E Andaya Ching Espiritu Hefti Galvez Gammad Lainez Lui Madamba Nagera Narvasa
Ong Palangdao Rosales Sanchez Santos Satrain Tabo (2014-2015)

Labor Relations Case Digest - Atty. Joyrich Golangco


173. Concepcion vs Minex Import Corporation
DOCTRINE: Neither a criminal prosecution nor a conviction beyond reasonable
doubt for the crime is a requisite for the validity of the dismissal. Nonetheless,
the dismissal for a just or lawful cause must still be made upon compliance with
the requirements of due process under the Labor Code; otherwise, the employer
is liable to pay nominal damages as indemnity to the dismissed employee.

FACTS: The petitioner Lolita Cocepcion was hired by respondent Minex ImportExport Corporation, the latter is engaged in the retail of semi-precious stones,
selling them in kiosk or stalls installed in various shopping centers within Metro
Manila. On October 23, 1997, respondent Vina Mariano, an Assistant Manager of
Minex, assigned the petitioner to the SM Harrison Plaza kiosk with the
instruction to hold the keys of the kiosk. Together with Cristina Calung and Lida
Baquilar as sales girls

On November 9, 1997 the kiosk had a total sales of 50, 912.00 which the
petitioner wrapped in a plastic bag and deposited in the drawer of the locked
wooden cabinet of the kiosk. The next day petitioner phoned Vina Mariano and
reported that such money was missing. Later while giving a detailed statement
on the theft to the security investigator of Harrison Plaza, Vina and Sylvia
Mariano, her supervisors arrived with the policemen and immediately placed the
petitioner in arrest. The police investigated her. She was detained for a day, from
11:30 am of November 10, 1997 until 11:30 am of November 11, 1997, being
released only because the inquest prosecutor instructed so.
On November 12, 1997, the petitioner complained against the respondents for
illegal dismissal in the DOLE, On November 14, 1997, Minex, through Vina, filed a
complaint for qualified theft against the petitioner in the Office of the City
Prosecutor in Manila.

After the preliminary investigation, the Assistant Prosecutor rendered a


resolution dated February 4, 1998 finding probable cause for qualified theft and
recommending the filing of an information against the petitioner, Thus, she was

charged with qualified theft in the RTC in Manila, docketed as Criminal Case No.
98-165426.
The petitioner appealed by petition for review to the DOJ, but the DOJ Secretary
denied her petition for review on July 4, 2001.

As to the petitioners complaint for illegal dismissal, Labor Arbiter Jose G. de Vera
Declared that there is illegal dismissal and ordered the reinstatement of the
petitioner with full backwages, moral damages and attorneys fees
NLRC: reversed the decision of LA, declaring that the petitioner had not been
dismissed, but had abandoned her job after being found to have stolen the
proceeds of the sales; and holding that even if she had been dismissed, her
dismissal would be justifiable for loss of trust and confidence in the light of the
finding of probable cause by the DOJ and the City Prosecutor and the filing of the
information for qualified theft against her. And deleted the award of damages,
MR denied
CA: affirmed NLRC mainly because of the DOJ Secretarys finding of probable
cause for qualified theft, MR denied
ISSUES:
1. whether or not there is a just cause for her dismissal
2. whether or not due process of law was complied with

HELD:
1, Yes, To dismiss an employee, the law requires the existence of a just and valid
cause. Article 282 of the Labor Code enumerates the just causes for termination
by the employer: (a) serious misconduct or willful disobedience by the
employee of the lawful orders of his employer or the latters representative in
connection with the employees work; (b) gross and habitual neglect by the
employee of his duties; (c) fraud or willful breach by the employee of the trust
reposed in him by his employer or his duly authorized representative; (d)
commission of a crime or offense by the employee against the person of his
employer or any immediate member of his family or his duly authorized
representative; and (e) other causes analogous to the foregoing.

3E Andaya Ching Espiritu Hefti Galvez Gammad Lainez Lui Madamba Nagera Narvasa
Ong Palangdao Rosales Sanchez Santos Satrain Tabo (2014-2015)

Labor Relations Case Digest - Atty. Joyrich Golangco


The petitioner however argues, that there was no evidence at all upon
which Minex could validly dismiss her considering that she had not yet been
found guilty beyond reasonable doubt of the crime of qualified theft.

The petitioners argument is not novel. It has been raised and rejected
many times before on the basis that neither conviction beyond reasonable doubt
for a crime against the employer nor acquittal after criminal prosecution was
indispensable. Nor was a formal charge in court for the acts prejudicial to the
interest of the employer a pre-requisite for a valid dismissal.
It is unfair to require an employer to first be morally certain of the guilt
of the employee by awaiting a conviction before terminating him when there is
already sufficient showing of the wrongdoing. Requiring that certainty may
prove too late for the employer, whose loss may potentially be beyond repair.
Here, no less than the DOJ Secretary found probable cause for qualified theft
against the petitioner. That finding was enough to justify her termination for loss
of confidence. To repeat, her responsibility as the supervisor tasked to oversee
the affairs of the kiosk, including seeing to the secure handling of the sales
proceeds, could not be ignored or downplayed. The employers loss of trust and
confidence in her was directly rooted in the manner of how she, as the
supervisor, had negligently handled the large amount of sales by simply leaving
the amount inside the cabinet drawer of the kiosk despite being aware of the
great risk of theft. At the very least, she could have resorted to the SOP of first
seeking guidance from the main office on how to secure the amount if she could
not deposit in the bank due to that day being a Sunday.

2, NO, Section 2 (d) of Rule I of the Implementing Rules of Book VI of the Labor
Code, viz:
Section 2. Security of tenure. xxx

xxx
(d) In all cases of termination of employment, the following standards of
due process shall be substantially observed:

For termination of employment based on just causes as defined in


Article 282 of the Labor Code:
(i) A written notice served on the employee specifying the
ground or grounds for termination, and giving said employee
reasonable opportunity within which to explain his side.
(ii) A hearing or conference during which the employee
concerned, with the assistance of counsel if he so desires is given
opportunity to respond to the charge, present his evidence, or
rebut the evidence presented against him.
(iii) A written notice of termination served on the
employee, indicating that upon due consideration of all the
circumstances, grounds have been established to justify his
termination. (emphasis supplied)
xxx

We answer the query in the negative, in the affidavit of the petitioner it provides
that after she was released by the policemen she contracted Sylvia Mariano and
informed her that since she was innocent of the charges, she would report back
to work but Sylvia Mariano shouted at the petitioner and told the latter that it no
longer wanted to see her face. This led to the filling of the illegal dismissal case

The petitioner plainly demonstrated how quickly and summarily her


dismissal was carried out without first requiring her to explain anything in her
defense as demanded under Section 2 (d) of Rule I of the Implementing Rules of
Book VI of the Labor Code. Instead, the respondents forthwith had her arrested
and investigated by the police authorities for qualified theft. This, we think, was a
denial of her right to due process of law, consisting in the opportunity to be
heard and to defend herself, In fact, their decision to dismiss her was already
final even before the police authority commenced an investigation of the theft,
the finality being confirmed by no less than Sylvia Mariano herself telling the
petitioner during their phone conversation following the latters release from
police custody on November 11, 1997 that she (Sylvia) no longer wanted to see
her.

3E Andaya Ching Espiritu Hefti Galvez Gammad Lainez Lui Madamba Nagera Narvasa
Ong Palangdao Rosales Sanchez Santos Satrain Tabo (2014-2015)

Labor Relations Case Digest - Atty. Joyrich Golangco


The fair and reasonable opportunity required to be given to the
employee before dismissal encompassed not only the giving to the employee of
notice of the cause and the ability of the employee to explain, but also the chance
to defend against the accusation.

In view of the foregoing, we impose on the respondents the obligation to pay to


the petitioner an indemnity in the form of nominal damages of P30,000.00,
Under the Civil Code, nominal damages is adjudicated in
order that a right of the plaintiff, which has been violated or
invaded by the defendant, may be vindicated or recognized, and
not for the purpose of indemnifying the plaintiff for any loss
suffered by him.

174. GRAND ASIAN SHIPPING LINES, INC., FRANCISCO and HOW, vs. GALVEZ,
ET AL.
DOCTRINE: While plain accusations are not sufficient to justify the dismissal of
rank and file employees, the mere existence of a basis for believing that managerial
employees have breached the trust reposed on them by their employer would
suffice to justify their dismissal.
FACTS: Grand Asian Shipping Lines, Inc. (GASLI), a domestic corp. is engaged in
transporting LPG from Petron Corp.s refinery in Bataan to its plant in Pasig and
Cavite. How and Francisco are GASLIs President & Gen. Mgr. respectively.
GASLIs vessel M/T Dorothy Uno is manned by respondents: Galvez (Captain),
Sales (Chief Mate) Gruta (Chief Engineer) while the rest are the vessels
crewmembers. One of the vessels oiler, Abis reported to GASLI that respondents
conspired to commit illegal activities by over declaring the consumed fuel of the
vessel and the saved fuel oil was siphoned and sold to other vessels out at sea
during nighttime. The proceeds of the sale were then divided among themselves.
After investigation of the alleged pilferage, GASLI found that the fuel
consumption of M/T Dorothy Uno for the period June 1999 to Feb. 2000 was
overstated by 6,954.3 liters equivalent to P74,737.86. A formal complaint for
qualified theft was filed with the CIDG then to the RTC. Meanwhile, GASLI placed

respondents under preventive suspension and later terminated them (with the
exception of Sales) for serious misconduct, willful breach of trust and
commission of a crime or offense. Respondents filed with the NLRC complaints
for illegal suspension and dismissal, underpayment/non-payment of salaries,
wages, overtime and other benefit pay, indemnity for damages and attorneys
fees.

The Labor Arbiter found the dismissal of the 21 complainants illegal, holding that
the filing of a criminal case against them did not justify their termination from
employment. Petitioners were ordered to reinstate complainants with full
backwages, and to pay jointly and severally, each complainants the amount of
their money claims, damages due to attending bad faith in terminating them,
double indemnity prescribed by RA 8188 in view of violation of the Minimum
Wage Law and attorneys fee in the total amount of P7,104,483.84. Petitioners
filed a Notice of Appeal with a Very Urgent Motion to Reduce Bond before the
NLRC posting a cash bond of P500,000 which was denied. However, in spite of
the earlier denial to reduce the bond, the NLRC reduced the amount of the appeal
bond to P1.5M and gave due course to petitioners appeal finding it meritorious
for having presented sufficient evidence to show just cause for terminating
complainants except for Sales who was declared illegally terminated absent any
record that the latter received any notice of suspension, administrative hearing,
or termination.
The NLRC reversed the Labor Arbiters decision as to all complainants and struck
down the monetary awards given except for Sales which was ordered to be paid
backwages of P124,115.10 plus 10% thereof as attorneys fee and his
reinstatement to his former position w/o loss of seniority rights & other benefits.
After a Motion for Reconsideration filed by petitioners, the NLRC reversed its
ruling with regard to Sales, finding that the latter was not illegally dismissed but
it was he who abandoned his work thus, the monetary award as well as the
reinstatement order in favor of Sales was ordered deleted in the Decision.
The CA reversed; found the NLRC to have committed grave abuse of discretion
for entertaining the appeal despite the insufficiency of petitioners appeal bond.
The CA annulled and set aside the NLRC Resolution and reinstated the Labor
Arbiters Decision; finding petitioners evidence inadequate to support the

3E Andaya Ching Espiritu Hefti Galvez Gammad Lainez Lui Madamba Nagera Narvasa
Ong Palangdao Rosales Sanchez Santos Satrain Tabo (2014-2015)

Labor Relations Case Digest - Atty. Joyrich Golangco

charges against respondents and their termination and ruling further that Sales
was illegally terminated. Petitioners motion for recon was denied.
ISSUES: Whether or not respondents termination and dismissal valid?

HELD: YES with regard to Galvez and Gruta and NO with regard to the other
respondents.

While there is reasonable ground to believe that respondents were responsible


for the pilferage of the fuel oil in M/T Dorothy Uno which renders them
unworthy of the trust and confidence reposed to them, petitioners failed to
substantiate adequately the charges against respondents. "The quantum of proof
which the employer must discharge is substantial evidence. x x x Substantial
evidence is that amount of relevant evidence as a reasonable mind might accept
as adequate to support a conclusion, even if other minds, equally reasonable,
might conceivably opine otherwise." (AMA Computer College-East Rizal v.
Ignacio, G. R. No. 178520, 2009). Aside from the Affidavits containing the
uncorroborated accusations of Abis that respondents committed pilferage and
the report made by Jade Sea Land Inspection Services which declared the actual
usage and amount of fuel consumed for a particular voyage, there are no other
sufficient evidence to show that respondents participated in the commission of a
serious misconduct or an offense against their employer.
On the ground of respondents termination for loss of trust and confidence,
distinction should be made between managerial and rank and file employees.
"With respect to rank-and-file personnel, loss of trust and confidence, as ground
for valid dismissal, requires proof of involvement in the alleged events x x x while
for managerial employees, the mere existence of a basis for believing that such
employee has breached the trust of his employer would suffice for his
dismissal."(Velez v. Shangri-las Edsa Plaza Hotel, 535 Phil. 12, 27 (2006). The
Court found some basis for the loss of confidence reposed on Galvez and Gruta
who are considered managerial employees. Galvez as the ships captain was
involved in the governance, care and management of the vessel while Gruta as
the chief engineer was tasked to take charge of the technical operations of the
vessel. They were vested with authority to execute management policies so they
hold positions of responsibility over the activities in the vessel requiring full

trust and confidence of their employer since they were entrusted with the
custody, handling and care of company property and exercise authority over it.
The fact of an overstatement of fuel consumption and loss of considerable
amount of fuel oil was unrefuted. Their failure to account for the loss of company
property betrays the trust reposed and expected of them. They had violated
petitioners trust and for which their dismissal is justified on the ground of
breach of confidence.
As for the other respondents, they are ordinary rank-and-file employees thus,
their involvement in the pilferage of the vessels fuel as well as their participation
in the alleged theft must be proved. This renders their dismissal illegal, thus,
entitling them to reinstatement plus full backwages, inclusive of allowances and
other benefits, computed from the time of their dismissal up to the time of actual
reinstatement. In the case of Sales, there was no evidence of his dismissal from
employment. Respondents are not entitled to their money claims; Galvez and
Gruta as managerial employees are excluded from payment of holiday pay,
service incentive leave pay, holiday & restday pay while the remaining
respondents cannot be classified as field personnel entitled to similar pay, in
accordance with Art. 82 of the Labor Code.

The Court annulled and set aside the Decision and Resolution of the CA; Galvez
and Gruta were declared dismissed from employment for just cause while Sales
was considered not dismissed from employment. The other respondents were
declared illegally terminated hence, GASLI was ordered to reinstate them to their
former position or its equivalent w/o loss of seniority rights and to pay them full
backwages inclusive of allowances and other benefits computed from the time of
dismissal up to actual reinstatement & 13th month pay for the period of their
illegal dismissal. Ten percent of the monetary award was ordered added as
attorneys fees. 6% interest per annum was imposed on all monetary awards
from date of finality of the Courts decision until full payment. How and Francisco
were absolved from liability adjudged against GASLI.
175. Mirant Philippines Corp. vs Joesilito Caro

DOCTRINES:
1. Article 4 of LC: all doubts should be resolved in favor of labor.

3E Andaya Ching Espiritu Hefti Galvez Gammad Lainez Lui Madamba Nagera Narvasa
Ong Palangdao Rosales Sanchez Santos Satrain Tabo (2014-2015)

Labor Relations Case Digest - Atty. Joyrich Golangco

2. Employees offense must be commensurate with the sanction


imposed.

FACTS: Respondent Caro was hired by petitioner as Procurement Supervisor at


the Logistics and Purchasing Department that falls within the nature of a
confidential employee. He has rendered service for 10 years without any record
of any violation in the company and his monthly salary amounts to Php39k.
Pursuant to RA 9165 or CDDA of 2002 (Drugs Act), the company conducted a
random drug testing (the Company Policy) among its employees on November 3,
2004. Caro was even informed during lunchtime (few hours before the testing)
that he was selected to participate in the testing. Caro left his office immediately
alleging that he received a phone call from his wifes male colleague that a bomb
explosion happened near the workstation of his wife in Tel Aviv. He left the office
to confirm such incident at the Israeli Embassy. He was able to go back at around
6pm in his office and alleged that he was only able to read the text messages
from the drug committee that he is obligated to go thru such testing because his
phone battery was drained all through out the afternoon. He tried to call the
Drug Committee and even offered that he would subject himself to the testing a
day after the scheduled testing. On Nov. 8, 2004 a Show Cause Notice was sent to
Caro requiring him to explain why he should not be charged with UNJUST
REFUSAL to submit to random drug testing and to submit supporting evidence
to prove that he was in fact at the Israeli Embassy. On January 2005,
the petitioners Investigating Panel issued a Report (after a hearing was
conducted and after Caro was able to present his evidence) finding that Caro is
guilty of unjustified refusal to go thru random drug testing and recommended 4
weeks suspension without pay; more importantly the Panel noted that the term
unjust refusal is ambiguous and it finds proper not to implement dismissal but
instead doubled the companys normal 2-week suspension without pay as to
manifest the gravity of his disobedience. However, a month after the Report, the
VP for Operations Mr. Sliman, terminated him alleging that Caros avoidance
was synonymous with refusal and it became unjustified as the latter was not
able to determine precisely the one who called him, nor procured evidence that
he was at the Israeli Embassy.

LA: Caro was illegally dismissed. Ordered payment of Php250k backwages plus
Php3M as moral and exemplary damages. Caros omission is only a failure to be
tested and not unjustified refusal.

NLRC: Caros offer to be tested a day after would affect the integrity and accuracy
of the specimen and that he could use remedial measures to metabolize or
eradicate whatever drugs he may have ingested, if any. There was falsity in
Caros claim. The Report was merely recommendatory and it was still the
Managements decision that should prevail. The dismissal was VALID and he was
afforded DUE PROCESS.
CA: disagreed with NLRC. Even if there was a clear disobedience on the part of
Caro such penalty of dismissal was too harsh. There must be REASONABLE
PROPORTIONALITY, on the one hand, the willful disobedience by the employee,
on the other hand, the penalty imposed.

ISSUES: Whether or not Caros omission amounts to UNJUST REFUSAL that


warrants his termination.

HELD: Negative, It is beyond debate that petitioner Corporations enforcement


of its Anti-Drugs Policy is an exercise of its management prerogative. It is also a
conceded fact that respondent "failed" to take the random drug test as scheduled,
and under the said company policy, such failure metes the penalty of termination
for the first offense. A plain, simple and literal application of the said policy to the
omission of respondent would have warranted his outright dismissal from
employment --- BUT this should NOT be the case. There was illegal dismissal.
First. The Company Policy was not clear on what constitutes "unjustified
refusal" when the subject drug policy prescribed that an employees "unjustified
refusal" to submit to a random drug test shall be punishable by the penalty of
termination for the first offense. To be sure, the term "unjustified refusal" could
not possibly cover all forms of "refusal" as the employees resistance, to be
punishable by termination, must be "unjustified." The Investigating Panel, LA,
NLRC and the CA had different takes on the meaning of unjust refusal --- hence,
it safe to say that there is no concrete definition provided for in the company
policy, it is vague. Article 4 of the LC provides that in case of doubt as to the
interpretation of the LC including its rules and regulations, it shall be resolved in
favor of labor.
Second. The penalty imposed fell short of being reasonable. Recognizing
the ambiguity the CA was correct to adopt Investigating Panels Report. Check
abovementioned DOCTRINE.

3E Andaya Ching Espiritu Hefti Galvez Gammad Lainez Lui Madamba Nagera Narvasa
Ong Palangdao Rosales Sanchez Santos Satrain Tabo (2014-2015)

Labor Relations Case Digest - Atty. Joyrich Golangco

176. Bluer Than Blue Joint Ventures Co. vs. Glyza Esteban

DOCTRINE: "It is not the job title but the actual work that the employee
performs that determines whether he or she occupies a position of trust and
confidence." In this case, while respondent's position was denominated as Sales
Clerk, the nature of her work included inventory and cashiering, a function that
clearly falls within the sphere of rank-and-file positions imbued with trust and
confidence. Loss of trust and confidence to be a valid cause for dismissal must be
work related such as would show the employee concerned to be unfit to continue
working for the employer and it must be based on a willful breach of trust and
founded on clearly established facts.

FACTS: The respondent was employed as a sales clerk and assigned at the
petitioners boutique. Her primary tasks were attending to all customer needs,
ensuring efficient inventory, coordinating orders from clients, cashiering and
reporting to the accounting department. The petitioner learned that some of
their employees had access to their POS system with the use of a universal
password given to them by a certain Elmer Flores, who in turn learned of the
password from the respondent. The petitioner then conducted an investigation
and asked the petitioner to explain why she should not be disciplinarily dealt
with. During the investigation the respondent was placed under preventive
suspension. In her explanation, Esteban admitted that she used the universal
password three times on the same day in December 2005, after she learned of it
from two other employees who she saw browsing through the petitioners sales
inquiry. She inquired how the employees were able to open the system and she
was told that they used the "123456" password. After investigation the
petitioner terminated the respondent on the grounds of loss of trust or
confidence. Subsequently, Esteban filed a complaint for illegal dismissal, illegal
suspension, holiday pay, rest day and separation pay.
LA: Ruled in favor of Esteban and found that she was illegally dismissed. The LA
also awarded separation pay, backwages, unpaid salary during her preventive
suspension and attorneys fees.

NLRC: Reversed the decision of the LA and dismissed the case for illegal
dismissal

CA: Reinstated LAs decision. The position occupied by Esteban was that of a
rank-and-file employee and she is neither a supervisor, manager nor a cashier;
thus, she does not hold a position of trust and confidence.

ISSUES:
(1) Whether or not Estaban holds a position of responsibility, trust and
confidence.

(2) Whether or not Estebans acts constitute just cause to terminate her
employment with the company on the ground of loss of trust and confidence.
HELD:
(1) Loss of trust and confidence is premised on the fact that the employee
concerned holds a position of responsibility, trust and confidence. The
employee must be invested with confidence on delicate matters, such as the
custody, handling, care and protection of the employers property and
funds. With respect to rank-and-file personnel, loss of trust and confidence
as ground for valid dismissal requires proof of involvement in the alleged
events in question, and that mere uncorroborated assertions and
accusations by the employer will not be sufficient. Esteban is, no doubt, a
rank-and-file employee. The question now is whether she occupies a
position of trust and confidence.
Among the fiduciary rank-and-file employees are cashiers, auditors,
property custodians, or those who, in the normal exercise of their functions,
regularly handle significant amounts of money or property. These
employees, though rank-and-file, are routinely charged with the care and
custody of the employers money or property, and are thus classified as
occupying positions of trust and confidence. In this case, Esteban was a sales
clerk. Her duties, however, were more than that of a sales clerk. Aside from
attending to customers and tending to the shop, Esteban also assumed
cashiering duties.

(2) Loss of trust and confidence to be a valid cause for dismissal must be work
related such as would show the employee concerned to be unfit to continue

3E Andaya Ching Espiritu Hefti Galvez Gammad Lainez Lui Madamba Nagera Narvasa
Ong Palangdao Rosales Sanchez Santos Satrain Tabo (2014-2015)

Labor Relations Case Digest - Atty. Joyrich Golangco

working for the employer and it must be based on a willful breach of trust
and founded on clearly established facts. Such breach is willful if it is done
intentionally, knowingly, and purposely, without justifiable excuse as
distinguished from an act done carelessly, thoughtlessly, heedlessly or
inadvertently. The loss of trust and confidence must spring from the
voluntary or willful act of the employee, or by reason of some blameworthy
act or omission on the part of the employee However, as ruled above, such
breach must have been done intentionally, knowingly, and purposely, and
without any justifiable excuse, and not simply something done carelessly,
thoughtlessly, heedlessly or inadvertently. To the Courts mind, Estebans
lapse is, at best, a careless act that does not merit the imposition of the
penalty of dismissal. The Court is not saying that Esteban is innocent of any
breach of company policy. That she relayed the password to another
employee is likewise demonstrative of her mindless appreciation of her
duties as a sales clerk in the petitioners employ. But absent any showing
that her acts were done with "moral perverseness" that would justify the
claimed loss of trust and confidence attendant to her job, the Court must
sustain the conclusion that Esteban was illegally dismissed.
177. MANILA JOCKEY CLUB, INC. v. TRAJANO (2013)

DOCTRINE: Loss of the employers trust and confidence is a just cause under
Article 282 (c), a provision that ideally applies only to cases involving an
employee occupying a position of trust and confidence, or to a situation where
the employee has been routinely charged with the care and custody of the
employers money or property. But the loss of trust and confidence, to be a valid
ground for dismissal, must be based on a willful breach of trust and confidence
founded on clearly established facts. "A breach is willful, if it is done
intentionally, knowingly and purposely, without justifiable excuse, as
distinguished from an act done carelessly, thoughtlessly, heedlessly or
inadvertently. It must rest on substantial grounds and not on the employers
arbitrariness, whims, caprices or suspicion; otherwise, the employee would
eternally remain at the mercy of the employer."30 An ordinary breach is not
enough. Moreover, the loss of trust and confidence must be related to the
employees performance of duties.

FACTS: MJCI had employed Trajano as a selling teller of betting tickets since. On
April 25, 1998, two regular bettors gave her their respective lists of bets (rota)
and money for the bets for Race 14. Although the bettors suddenly left her, she
entered their bets in the selling machine and segregated the tickets for pick up by
the two bettors upon their return. Before closing time, one of the bettors
(requesting bettor) returned and asked her to cancel one of his bets
worth P2,000.00. Since she was also operating the negative machine on that day,
she obliged and immediately cancelled the bet as requested. She gave the
remaining tickets and the P2,000.00 to the requesting bettor, the money
pertaining to the canceled bet. When Race 14 was completed, she counted the
bets received and the sold tickets. She found that the bets and the tickets
balanced. But then she saw in her drawer the receipt for the canceled ticket, but
the canceled ticket was not inside the drawer. Thinking she could have given the
canceled ticket to the requesting bettor, she immediately looked for him but
could not find him. It was only then that she remembered that there were two
bettors who had earlier left their bets with her. Thus, she went to look for the
other bettor (second bettor) to ask if the canceled ticket was with him. When she
located the second bettor, she showed him the receipt of the canceled ticket to
counter-check the serial number with his tickets.
Thereafter, the second bettor returned to Trajano and told her that it was one of
his bets that had been canceled, instead of that of the requesting bettor. To
complicate things, it was also the same bet that had won Race 14. Considering
that the bet was for a daily double, the second bettor only needed to win Race 15
in order to claim dividends. At that point, she realized her mistake, and explained
to the second bettor that the cancellation of his ticket had not been intentional,
but the result of an honest mistake on her part. She offered to personally pay the
dividends should the second bettor win Race 15, which the latter accepted. When
Race 15 was completed, the second bettor lost. She was thus relieved of the
obligation to pay any winnings to the second bettor.4
To her surprise, the reliever-supervisor later approached Trajano and told her to
submit a written explanation about the ticket cancellation incident. The next day
(April 26, 1998), she submitted the handwritten explanation. Later that day,
when she received an inter-office correspondence informing her that she was
being placed under preventive suspension effective for an unstated period of

3E Andaya Ching Espiritu Hefti Galvez Gammad Lainez Lui Madamba Nagera Narvasa
Ong Palangdao Rosales Sanchez Santos Satrain Tabo (2014-2015)

Labor Relations Case Digest - Atty. Joyrich Golangco

time. At the end of thirty days of her suspension, Trajano reported for work. But
she was no longer admitted. She then learned that she had been dismissed when
she read a copy of an inter-office correspondence about her termination posted
in a selling station of MJCI. Thus, Trajano instituted a complaint for illegal
dismissal against MJCI in DOLE;

On its part, MJCI averred that on April 25, 1998, it received a letter from Jun
Carpio, the Field Officer of the Games and Amusement Board, calling its attention
to a complaint against Trajano brought by a certain bettor named "Tito" who had
reported the cancellation of his ticket that had already won the first leg (Race 14)
of the daily double bet; that it acted on the complaint by placing her under
preventive suspension upon her submission of a written explanation11 and after
the conduct of preliminary investigation on the matter; that on June 5, 1998, it
invited her to a clarificatory meeting in the presence of MJCI Raceday Union
President Miguel Altonaga; and that it terminated her services on the next day
"for cause due to unauthorized cancellation of ticket."
ISSUES:

1. WON there was just cause when MJCI dismissed Trajano from the service;

2. WON MJCI complied with the due process requirement when it effected the
dismissal of Trajano.

HELD: No just cause for Trajanos dismissal. Loss of the employers trust and
confidence is a just cause under Article 282 (c), a provision that ideally applies
only to cases involving an employee occupying a position of trust and confidence,
or to a situation where the employee has been routinely charged with the care
and custody of the employers money or property. But the loss of trust and
confidence, to be a valid ground for dismissal, must be based on a willful breach
of trust and confidence founded on clearly established facts. "A breach is willful,
if it is done intentionally, knowingly and purposely, without justifiable excuse, as
distinguished from an act done carelessly, thoughtlessly, heedlessly or
inadvertently. It must rest on substantial grounds and not on the employers
arbitrariness, whims, caprices or suspicion; otherwise, the employee would
eternally remain at the mercy of the employer." An ordinary breach is not
enough.

Moreover, the loss of trust and confidence must be related to the employees
performance of duties. Loss of confidence, as a just cause for termination of
employment, is premised on the fact that the employee concerned holds a
position of responsibility, trust and confidence. He must be invested with
confidence on delicate matters such as the custody, handling, care and protection
of the employers property and/or funds. But in order to constitute a just cause
for dismissal, the act complained of must be "work-related" such as would show
the employee concerned to be unfit to continue working for the employer.
As a selling teller, Trajano held a position of trust and confidence. The nature of
her employment required her to handle and keep in custody the tickets issued
and the bets made in her assigned selling station. The bets were funds belonging
to her employer. Although the act complained of the unauthorized cancellation
of the ticket (i.e., unauthorized because it was done without the consent of the
bettor) was related to her work as a selling teller, MJCI did not establish that
the cancellation of the ticket was intentional, knowing and purposeful on her
part in order for her to have breached the trust and confidence reposed in her by
MJCI, instead of being only out of an honest mistake.

Still, to justify the supposed loss of its trust and confidence in Trajano, MJCI
contends that the unauthorized cancellation of the ticket could have greatly
prejudiced MJCI for causing damage to both its income and reputation. We
consider the contention of MJCI unwarranted. As the records indicate, MJCIs
prejudice remained speculative and unrealized. To dismiss an employee based
on speculation as to the damage the employer could have suffered would be an
injustice. Furthermore, the loss of trust and confidence as a ground for the
dismissal of an employee must also be shown to be genuine. Loss of confidence
should not be simulated in order to justify what would otherwise be, under the
provisions of law, an illegal dismissal. It should not be used as a subterfuge for
causes which are illegal, improper and unjustified. It must be genuine, not a mere
afterthought to justify an earlier action taken in bad faith.

Anent the second issue, there was a compliance with the second requirement for
a hearing or conference. It is undeniable that Trajano was accorded the real
opportunity to respond to the complaint against her, for she did submit her
written explanation on April 26, 1998 and was invited to the final clarificatory
meeting set on June 5, 1998 in the presence of the MJCI Raceday Union President.

3E Andaya Ching Espiritu Hefti Galvez Gammad Lainez Lui Madamba Nagera Narvasa
Ong Palangdao Rosales Sanchez Santos Satrain Tabo (2014-2015)

Labor Relations Case Digest - Atty. Joyrich Golangco

Nor was it necessary at all for Trajano to be able to confront the complainant
against her. In Muaje-Tuazon v. Wenphil Corporation,43 the Court has clarified
that the opportunity to confront a witness is not demanded in company
investigations of the administrative sins of an employee. Confrontation of
witnesses is required only in adversarial criminal prosecutions, and not in
company investigations for the administrative liability of the employee.
Additionally, actual adversarial proceedings become necessary only for
clarification, or when there is a need to propound searching questions to
witnesses who give vague testimonies. This is not an inherent right, and in
company investigations, summary proceedings may be conducted.

As for the last procedural requirement of giving the second notice, the posting of
the notice of termination at MJCIs selling stations did not satisfy it, and the fact
that Trajano was eventually notified of her dismissal did not cure the infirmity.
While personal service of the notice of termination on the employee is not
required, Section 2 (d), Rule I of the Implementing Rules of Book VI of the Labor
Code mandates that such notice be served on Trajano at her last known address.
178. REXIE A. HORMILLOSA, Petitioner, vs. COCA-COLA BOTTLERS PHILS.,
INC., represented by its Iloilo Plant Human Resource Head, ROBERTO
RICHARD H. DOLAR, Respondent.

DOCTRINE: The requisites for a valid dismissal on the ground of loss of trust and
confidence are as follows: (1) the employee concerned must be one holding a
position of trust and confidence; and (2) there must be an act that would justify
the loss of trust and confidence.
FACTS: On November 1, 1996, Hormillosa was employed as a route salesman by
Coca-Cola Bottlers Phils., Inc. (CBPI).

Sometime in the early part of 1999, the then CBPI District Sales Supervisor, Raul
S. Tiosayco III (Tiosayco), conducted a verification and audit of the accounts
handled by Hormillosa. He discovered transactions in violation of CCBPI
Employee Code of Disciplinary Rules and Regulations, specifically "Fictitious
sales transactions; Falsification of company
records/data/documents/invoices/reports; fictitious issuances of
TCS/COL(Temporary Credit Sales/Container on Loan); non-issuance or mis-

issuance of invoices and receipts as well as commercial documents to dealers;


forgery; misuse, abuse or defalcation of funds form market development
program." On March 8, 1999, Tiosayco issued a memorandum to Hormillosa
informing him that he was being placed on grounded status and would be
subjected to an investigation.

On March 15, 1999 Tiosayco issued memorandum directing Hormillosa to report


for a question-and-answer investigation relative to the findings. Hormilla asked
for deferment. On March 16, 1999, another memorandum was issued. Hormilla
again asked for postponement. On March 17, 1999, another memorandum was
issued. Hormilla instead sent a letter informing Tiosayco that the investigation
was already "moot and academic" on the pretense that he had already filed a case
against CBPI for Unfair Labor Practice (ULP).

On March 22, 1999, Tiosayco submitted his findings and recommendations to the
Regional Sales Manager, proposing the termination of Hormillosa. CBPI gave
credence to the report and approved his recommendation. Subsequently, a
termination letter was issued informing Hormillosa that he was being terminated
effective March 29, 1999.
On May 24, 1999, Hormillosa filed a complaint for ULP (harassment due to union
activities and union busting), Illegal Dismissal, Illegal Deduction, Illegal
Grounding, Non-payment of Commission, Non-payment of 13th Month pay,
Violation of CBA, Damages, and Attorneys Fees against CBPI before the SubRegional Arbitration Branch No. VI (SRAB). Thereafter, a preliminary conference
was conducted and both parties were directed to file their respective position
papers.
Hormillosa averred in his position paper that prior to his dismissal, he was a
member of the Board of Directors of CBPIs employees union and he became its
secretary on March 7, 1999. As secretary, he sent a copy of the new list of union
officers to the management with a warning that if CBPI would not stop harassing
the members of the union, it would declare a strike.
LA Lagoc: Termination was proper. Hormillosa was just using the union to
thwart managements exercise of its legal prerogative.

3E Andaya Ching Espiritu Hefti Galvez Gammad Lainez Lui Madamba Nagera Narvasa
Ong Palangdao Rosales Sanchez Santos Satrain Tabo (2014-2015)

Labor Relations Case Digest - Atty. Joyrich Golangco

NLRC: Ordered remand of case to give Hormillosa the opportunity to confront


the witnesses and evidence against him.

LA Acosta: Dismissal was illegal because the witnesses of CBPI did not appear in
the hearings as ordered, it had no other alternative but to give Hormillosa the
"benefit of the doubt" and decide the case in his favor.
NLRC: Upheld LA Acosta. No substantial evidence that Hormillosa falsified and
issued fictitious invoices and CBPI failed to "unleash the burden of proof" to
justify his termination.

CA: Dismissal was valid. The NLRC decision did not contain a recital of the facts
of the case, applicable laws or rules and the conclusions and reasons therefor.
ISSUES: Whether Hormillosas employment was validly terminated

HELD: YES, The rule is that, in labor cases, substantial evidence or such relevant
evidence as a reasonable mind might accept as sufficient to support a conclusion
is required. The CA was correct when it ruled that Hormillosas employment was
validly terminated under paragraph (c) of Art. 282. There was substantial
evidence to justify his dismissal.
In Bristol Myers Squibb (Phils.), Inc. v. Baban, the Court discussed the requisites
for a valid dismissal on the ground of loss of trust and confidence as follows:
The first requisite for dismissal on the ground of loss of trust
and confidence is that the employee concerned must be one
holding a position of trust and confidence. Verily, We must first
determine if respondent holds such a position.

There are two (2) classes of positions of trust. The first class
consists of managerial employees. They are defined as those
vested with the powers or prerogatives to lay down
management policies and to hire, transfer suspend, lay-off,
recall, discharge, assign or discipline employees or effectively
recommend such managerial actions. The second class consists
of cashiers, auditors, property custodians, etc. They are defined
as those who in the normal and routine exercise of their

functions, regularly handle significant amounts of money or


property.

The second requisite is that there must be an act that would


justify the loss of trust and confidence. Loss of trust and
confidence to be a valid cause for dismissal must be based on a
willful breach of trust and founded on clearly established facts.
The basis for the dismissal must be clearly and convincingly
established but proof beyond reasonable doubt is not necessary.

Hormillosa, being a route salesman, falls under the second class. By selling soft
drink products and collecting payments for the same, he was considered an
employee who regularly handled significant amounts of money and property in
the normal and routine exercise of his functions. We agree that route salesmen
are likely individualistic personnel who roam around selling softdrinks, deal with
customers and are entrusted with large asset and funds and property of the
employer. There is a high degree of trust and confidence reposed on them, and
when confidence is breached, the employer may take proper disciplinary action
on them.
With regard to the second requisite for dismissal on the ground of loss of trust
and confidence, the Court finds that Hormillosa committed acts which warranted
his dismissal from employment.

Hormillosa cannot deny that fact that he issued sales invoices to Arnold Store, a
store unregistered or unaccredited with CBPI. He transacted with the said store
using the account of Virgie Bucaes, proprietor of Virgies Eatery. Bucaes, who had
an outlet profile with CBPI, was assigned with Control No. 0027069.22
Hormillosa extended credit to Arnold Store, an unknown customer to CBPI, as
documented by two credit sales invoices, Invoice Nos. 79872 and 79873,
amounting to P5,600.00 and P4,806.00respectively. By doing so, he gave a false
and misleading representation that the account was that of Bucaes. CBPI had a
set of rules and regulations, one of which was that only those outlets, which had
outlet control, were entitled to enjoy credit from CBPI. Salesmen were not
allowed to extend credit to those who had no outlet numbers or outlet profiles
from CBPI. Evidently, Hormillosa disregarded and disobeyed the company rules.

3E Andaya Ching Espiritu Hefti Galvez Gammad Lainez Lui Madamba Nagera Narvasa
Ong Palangdao Rosales Sanchez Santos Satrain Tabo (2014-2015)

Labor Relations Case Digest - Atty. Joyrich Golangco

The loss of trust and confidence must be based on willful breach of the trust
reposed in the employee by the employer. Ordinary breach will not suffice; it
must be willful. Such breach is willful if it is done intentionally, knowingly, and
purposely, without justifiable excuse as distinguished from an act done
carelessly, thoughtlessly, heedlessly, or inadvertently.

In the case at bench, Hormillosa's act of issuing sales invoices to Arnold Store
could not have been performed without intent and knowledge on his part as such
act could not have been done without planning or merely through negligence.
Hence, the breach was willful.
179. Eric V. Chuanico vs. Legacy Consolidated Plans, Inc.

DOCTRINE: Settled is the rule, however, that under Article 282(c) of the Labor
Code, the breach of trust must be willful. Ordinary breach will not be enough. A
breach is willful if it is done intentionally and knowingly without any justifiable
excuse, as distinguished from an act done carelessly, thoughtlessly or
inadvertently. Willful breach was not proved in this case.

FACTS: On January 3, 2002 Legacy Plans Philippines, Inc. (Legacy Plans)hired


petitioner Eric V. Chuanico (Atty. Chuanico) as Assistant Vice- President for legal
services. He was to serve as in-house counsel for the company and its
subsidiaries under the supervision of Atty. Christine A. Cruz (Atty. Cruz), the
Senior Vice-President for Legal Affairs. In the same year, Legacy Plans merged
with Consolidated Plans Philippines, Inc. to become Legacy Consolidated Plans,
Inc.

On October 17, 2002 Atty. Cruz wrote Atty. Chuanico a memorandum, requiring
him to explain why no administrative action should be taken against him for
mishandling two cases.

In the first case he was supposed to draft an answer to a complaint for Bank of
East Asia (a Legacy Consolidated affiliate) but he belatedly drafted a haphazard
one that he gave to the handling lawyers without coursing it to his superior. In
his defense, Atty. Chuanico said that he was given only one day within which to
finish the draft. While admitting that his superior had no opportunity to review it
for lack of time, he denied that the answer had been haphazardly done.

In the second case, Atty. Chuanico was required to prepare a complaint-affidavit


for the Rural Bank of Paraaque (also a Legacy Consolidated affiliate) against a
certain De Rama but he failed to do so. Atty. Chuanico replied that the case had
not actually been turned over to him. It was originally assigned to Atty. Dennis
Amparo who later said that the complaint-affidavit could not be prepared
because the Rural Bank had no witness.

On December 5, 2002 Legacy Consolidated dismissed Atty. Chuanico with effect


on December 20, 2002 for serious misconduct, willful disobedience to lawful
orders, gross and habitual neglect of duties, and willful breach of trust.7This
prompted him to file a complaint for illegal dismissal with claims for his unpaid
December 2002 salary and 13th-month pay plus moral and exemplary damages
and attorneys fees.

LA: Rendered decision finding respondent guilty of illegal dismissal and


awarded Atty. Chuanico with full backwages from December 20, 2002 and
separation pay in lieu of reinstatement computed at one month pay for every
year of service inclusive of the period when the case was pending. LA found that
Atty. Chuanico actually drafted an answer for Bank of East Asia but the
companys two new lawyers did not like it and chose to file one that they
themselves prepared. But since Legacy Consolidated neither bothered to present
Atty. Chuanicos draft answer nor explained why it regarded the same as
haphazardly done, it failed to prove its case. As to the second charge, the LA gave
credence to Atty. Dennis Amparos sworn statement that it was to Atty. Cruz, not
to Atty. Chuanico, that he personally turned over the cases he was handling. In
one of these, the case for the Rural Bank, he had been unable to prepare a
complaint affidavit against De Rama for failure of the bank to find a willing
witness against her.
NLRC: Affirmed LA. The NLRC held that Legacy Consolidated failed to present
evidence to prove that Atty. Chuanico violated rules or his superiors some
company order. His employer gave him no notice of these alleged violations that
were supposedly willful.
CA: Held that the NLRC committed grave abuse of discretion in holding Legacy
Consolidated guilty of illegal dismissal of Atty. Chuanico.
ISSUES: WON CA Atty Chuanico was illegally dismissed

3E Andaya Ching Espiritu Hefti Galvez Gammad Lainez Lui Madamba Nagera Narvasa
Ong Palangdao Rosales Sanchez Santos Satrain Tabo (2014-2015)

Labor Relations Case Digest - Atty. Joyrich Golangco

HELD: To be a valid cause for dismissal, the loss of trust must be based on a
willful breach of such trust and founded on clearly established facts. The
company charged him with having mishandled two things that were assigned to
him, the drafting of an answer in one and the preparation of a complaint affidavit
in the other. It failed to present proof, however, of such mishandling.
The Court held in CAPANELA v. National Labor Relations Commission that the
factual findings of quasi-judicial bodies, which are triers of facts on matters
within their expertise, should be considered, when supported by substantial
evidence, binding and conclusive on appellate courts. Here the LA and the NLRC
were in better positions to assess and evaluate the credibility of the parties'
claims and the weight to which the irrespective evidence is entitled.

Settled is the rule, however, that under Article 282( c) of the Labor Code, the
breach of trust must be willful. Ordinary breach will not be enough. A breach is
willful if it is done intentionally and knowingly without any justifiable excuse, as
distinguished from an act done carelessly, thoughtlessly or inadvertently. Willful
breach was not proved in this case.
180. Duncan Association and Tecson vs Glaxo
DOCTRINE: Constructive dismissal is defined as a quitting, an involuntary
resignation resorted to when continued employment becomes impossible,
unreasonable, or unlikely; when there is a demotion in rank or diminution in
pay; or when a clear discrimination, insensibility or disdain by an employer
becomes unbearable to the employee.

FACTS:
Petitioner Tecson was hired by respondent Glaxo as medical
representative after Tecson had undergone training and orientation. Tecson
signed a contract of employment which stipulates, among others, that he agrees
to study and abide by existing company rules; to disclose to management any
existing or future relationship by consanguinity or affinity with co-employees or
employees of competing drug companies and should management find that such
relationship poses a possible conflict of interest, to resign from the company. The
Employee Code of Conduct of Glaxo similarly provides the same.
Tecson was initially assigned to market Glaxos products in
the Camarines Sur-Camarines Norte sales area.

Subsequently, Tecson entered into a romantic relationship with Bettsy,


an employee of Astra Pharmaceuticals a competitor of Glaxo. Bettsy was Astras
Branch Coordinator. Even before they got married, Tecson received several
reminders from his District Manager regarding the conflict of interest which his
relationship with Bettsy might engender. Still, love prevailed, and Tecson
married Bettsy.
Tecsons superiors reminded him that he and Bettsy should decide
which one of them would resign from their jobs. Tecson requested for time to
comply with the company policy against entering into a relationship with an
employee of a competitor company.
Glaxo transferred Tecson to the Butuan City-Surigao City-Agusan del Sur
sales area. Tecson asked Glaxo to reconsider its decision, but his request was
denied. Tecson defied the transfer order and continued acting as medical
representative in the Camarines Sur-Camarines Norte sales area.
Parties failed to resolve the issue at the grievance machinery level, they
submitted the matter for voluntary arbitration. Glaxo offered Tecson a
separation pay but he declined the offer. NCMB rendered its decision declaring as
valid Glaxos policy on relationships between its employees and persons
employed with competitor companies, and affirming Glaxos right to transfer
Tecson to another sales territory. This was affirmed by the CA.
Petitioners contend that Glaxos policy against employees marrying
employees of competitor companies violates the equal protection clause of the
Constitution because it creates invalid distinctions among employees on account
only of marriage. He also argued that he was constructively dismissed by his
transfer, diminution of pay and prohibiting him to attend seminars.
ISSUES: 1) Whether Glaxos policy against its employees marrying employees
from competitor companies is valid?
(2) Whether Tecson was constructively dismissed?

HELD: 1. Yes. Glaxos policy prohibiting an employee from having a relationship


with an employee of a competitor company is a valid exercise of management
prerogative. Glaxo has a right to guard its trade secrets, manufacturing formulas,
marketing strategies and other confidential programs and information from
competitors, especially so that it and Astra are rival companies in the highly
competitive pharmaceutical industry.

3E Andaya Ching Espiritu Hefti Galvez Gammad Lainez Lui Madamba Nagera Narvasa
Ong Palangdao Rosales Sanchez Santos Satrain Tabo (2014-2015)

Labor Relations Case Digest - Atty. Joyrich Golangco

The prohibition against personal or marital relationships with employees of


competitor companies upon Glaxos employees is reasonable under the
circumstances because relationships of that nature might compromise the
interests of the company. In laying down the assailed company policy, Glaxo only
aims to protect its interests against the possibility that a competitor company
will gain access to its secrets and procedures.

That Glaxo possesses the right to protect its economic interests cannot be
denied. No less than the Constitution recognizes the right of enterprises to adopt
and enforce such a policy to protect its right to reasonable returns on
investments and to expansion and growth.
The challenged company policy does not violate the equal protection clause
of the Constitution as petitioners erroneously suggest. It is a settled principle
that the commands of the equal protection clause are addressed only to the state
or those acting under color of its authority.

The assailed company policy which forms part of respondents Employee


Code of Conduct and of its contracts with its employees, such as that signed by
Tecson, was made known to him prior to his employment. Tecson, therefore,
was aware of that restriction when he signed his employment contract and when
he entered into a relationship with Bettsy. Since Tecson knowingly and
voluntarily entered into a contract of employment with Glaxo, the stipulations
therein have the force of law between them and, thus, should be complied with
in good faith. He is therefore estopped from questioning said policy.

2. NO. The Court finds no merit in petitioners contention that Tecson was
constructively dismissed when he was transferred from the Camarines NorteCamarines Sur sales area to the Butuan City-Surigao City-Agusan del Sur sales
area, and when he was excluded from attending the companys seminar on new
products which were directly competing with similar products manufactured by
Astra.
Constructive dismissal is defined as a quitting, an involuntary
resignation resorted to when continued employment becomes impossible,
unreasonable, or unlikely; when there is a demotion in rank or diminution in
pay; or when a clear discrimination, insensibility or disdain by an employer
becomes unbearable to the employee. None of these conditions are present in the
instant case. The record does not show that Tecson was demoted or unduly

discriminated upon by reason of such transfer. Glaxo properly exercised its


management prerogative in reassigning Tecson to the Butuan City sales area.
181. Star Paper Corporation et al. v. Simbol

DOCTRINE: A requirement that a woman employee must remain unmarried


could be justified as a bona fide occupational qualification, or BFOQ, where
the particular requirements of the job would justify the same, but not on the
ground of a general principle, such as the desirability of spreading work in the
workplace. A requirement of that nature would be valid provided it reflects an
inherent quality reasonably necessary for satisfactory job performance.
FACTS: Petitioner was the employer of the three respondents. Under the policy
of Star Paper the employees are:
1.
2.

New applicants will not be allowed to be hired if in case he/she has


a relative, up to the 3rd degree of relationship, already employed by
the company.

In case of two of our employees (singles, one male and another


female) developed a friendly relationship during the course of their
employment and then decided to get married, one of them should
resign to preserve the policy stated above.

Respondents Wilfreda Comia and Ronaldo Simbol both got married to their
fellow employees, Howard Comia and Alma Dayrit, respectively. As to
respondent Estrella, she alleges that she had a relationship with co-worker
Zuiga who misrepresented himself as a married but separated man. After he got
her pregnant, she discovered that he was not separated. Thus, she severed her
relationship with him to avoid dismissal due to the company policy.
On November 30, 1999, she met an accident and was advised by the doctor at
the Orthopedic Hospital to recuperate for twenty-one (21) days. She returned to
work on December 21, 1999 but she found out that her name was on-hold at the
gate. She was denied entry. She was directed to proceed to the personnel office
where one of the staff handed her a memorandum. The memorandum stated that
she was being dismissed for immoral conduct. She refused to sign the
memorandum because she was on leave for twenty-one (21) days and has not
been given a chance to explain. The management asked her to write an
explanation. However, after submission of the explanation, she was

3E Andaya Ching Espiritu Hefti Galvez Gammad Lainez Lui Madamba Nagera Narvasa
Ong Palangdao Rosales Sanchez Santos Satrain Tabo (2014-2015)

Labor Relations Case Digest - Atty. Joyrich Golangco

nonetheless dismissed by the company. Due to her urgent need for money, she
later submitted a letter of resignation in exchange for her thirteenth month pay.
The respondents allege that they were forced to resign as a result of the
implementation of the said assailed company policy.
Respondents later filed a complaint for unfair labor practice, constructive
dismissal, separation pay and attorneys fees. The Labor Arbiter and the NLRC
ruled in favor of petitioner. The decision was appealed to the Court of Appeals
which reversed the decision.
ISSUES: Whether the prohibition to marry in the contract of employment is valid.

HELD: No, it is not, for lack of a bona fide occupational qualification or


BFOQ.
A requirement that an employee must remain unmarried could be justified as a
bona fide occupational qualification, or BFOQ, where the particular
requirements of the job would justify the same, but not on the ground of a
general principle, such as the desirability of spreading work in the workplace. A
requirement of that nature would be valid provided it reflects an inherent
quality reasonably necessary for satisfactory job performance.

It is significant to note that in the case at bar, respondents were hired after they
were found fit for the job, but were asked to resign when they married a coemployee. Petitioners failed to show how the marriage of Simbol, then a Sheeting
Machine Operator, to Alma Dayrit, then an employee of the Repacking Section,
could be detrimental to its business operations. Neither did petitioners explain
how this detriment will happen in the case of Wilfreda Comia, then a Production
Helper in the Selecting Department, who married Howard Comia, then a helper
in the cutter-machine. The policy is premised on the mere fear that employees
married to each other will be less efficient. If we uphold the questioned rule
without valid justification, the employer can create policies based on an
unproven presumption of a perceived danger at the expense of an employees
right
to
security
of
tenure.

Petitioners contend that their policy will apply only when one employee marries
a co-employee, but they are free to marry persons other than co-employees. The
questioned policy may not facially violate Article 136 of the Labor Code but it

creates a disproportionate effect and under the disparate impact theory, the only
way it could pass judicial scrutiny is a showing that it is reasonable despite the
discriminatory, albeit disproportionate, effect. The failure of petitioners to prove
a legitimate business concern in imposing the questioned policy cannot prejudice
the employees right to be free from arbitrary discrimination based upon
stereotypes of married persons working together in one company.

Lastly, the absence of a statute expressly prohibiting marital discrimination in


our jurisdiction cannot benefit the petitioners. The protection given to labor in
our jurisdiction is vast and extensive that we cannot prudently draw inferences
from the legislatures silence that married persons are not protected under our
Constitution and declare valid a policy based on a prejudice or stereotype. Thus,
for failure of petitioners to present undisputed proof of a reasonable business
necessity, we rule that the questioned policy is an invalid exercise of
management prerogative. Corollary, the issue as to whether respondents Simbol
and Comia resigned voluntarily has become moot and academic.
In the case of Estrella, the petitioner failed to adduce proof to justify her
dismissal. Hence, the Court ruled that it was illegal.
182. ACE PROMOTION AND MARKETING CORPORATION, petitioner, vs.
REYNALDO URSABIA, respondent.

FACTS: Sometime in August, 1994, petitioner Ace Promotion hired respondent


Reynaldo Ursabia as a company driver assigned to pick up the products of Nestle
Philippines, Inc., for promotion and marketing.

On July 6, 2001, respondent failed to report for work. Petitioner, through its area
supervisor, Gerry Garcia, issued a Memorandum dated July 9, 2001. Ursabia was
asked to explain in writing within 24 hours why no disciplinary measure be
taken against him for Violation of Company Rule (Abandonment of Work) last Jul.
06, 2001.
Ursabia reported back to work on July 9, 2001. He was personally served with
the memorandum but refused to acknowledge the same, hence, it was sent
through registered mail to respondents last known address.

3E Andaya Ching Espiritu Hefti Galvez Gammad Lainez Lui Madamba Nagera Narvasa
Ong Palangdao Rosales Sanchez Santos Satrain Tabo (2014-2015)

Labor Relations Case Digest - Atty. Joyrich Golangco

The following day, July 10, 2001, Garcia noticed some damage on the vehicle
assigned to respondent, hence, he issued another Memorandum which was
served through registered mail. Laman ng memo:
1.
2.

Ursabia was required to explain why he should not be terminated for


destruction of company vehicle.
The vehicle was checked and found that the right front wheel was
deflated; the sliding door was slightly damaged; disconnected wirings; a
piece of paper inserted on the distribution cap.

Sometime in July 2001, an anonymous note was discovered among the stocks of
petitioner containing the words "(Good news) be careful and save youre (sic)
life because there's a time to come everybody x x x will die." Upon
examination of PNP crime lab, it allegedly showed that the handwriting of
respondent has significant similarities with the said handwritten note.
On August 6, 2001, respondent went to petitioner's office and was served with a
termination letter.
Again, respondent refused to receive the same prompting petitioner serve it by
registered mail to respondent's last known address.
Meanwhile, the petitioner filed two criminal cases for Malicious Mischief and
Grave Threats against the respondent. Displeased with his termination,
respondent filed a complaint for illegal dismissal and non-payment of other
monetary benefits.

Labor Arbiter rendered a decision in favor of respondent holding that the latter
was illegally dismissed for lack of hearing. On appeal, the NLRC reversed the
decision of the Labor Arbiter on the ground of abandonment, hence, legal ung
dismissal. The Court of Appeals set aside the decision of the NLRC and held that
respondent was illegally dismissed on the ground that there is no abandonment
and the criminal cases were dismissed. MR was denied, hence the instant
petition.

ISSUES: Whether there exists a just cause to dismiss respondent and whether he
was accorded procedural due process.
HELD: We agree with the Court of Appeals that respondent cannot be dismissed
for abandonment. To constitute a just and valid ground for dismissal,
abandonment requires the deliberate and unjustified refusal of the employee to
resume his employment. Two elements must be present, namely: (1) the failure
to report for work or absence without valid or justifiable reason, and (2) a clear
intention to sever the employer-employee relationship. The second element is
more determinative of the intent and must be evinced by overt acts. Mere
absence, not being sufficient, the burden of proof rests upon the employer to
show that the employee clearly and deliberately intended to discontinue his
employment without any intention of returning.

Abandonment- conduct of respondent after he failed to report for work on July 6,


2001, shows that he had no intention to sever his employment with petitioner.
He went to work on July 9 hahit na nagloiter lng sya outside the company
premises, it shows his intention to make his services available. He formally
reported for work on Aug. 6. All these show that respondent never really wanted
to quit his job. He may be guilty of going on absence without leave, but not
abandonment because the totality of his acts shows a clear intention to
return to work.
Damage sa company car- cannot justify dismissal. Termination is
disproportionate sa infraction considering na hindi naprove ung extent ng
damage at walang substantial evidence to establish guilt.

Anonymous note- di naprove na threat nga iyon at si respondent ang may gawa.

HOWEVER, respondent should be dismissed for willful disobedience of the


memoranda issued by petitioner. To be validly dismissed on the ground of willful
disobedience requires the concurrence of at least two requisites: (1) the
employee's assailed conduct must have been willful or intentional, the
willfulness being characterized by a wrongful and perverse attitude; and (2) the
order violated must have been reasonable, lawful, made known to the employee
and must pertain to the duties which he had been engaged to discharge.

3E Andaya Ching Espiritu Hefti Galvez Gammad Lainez Lui Madamba Nagera Narvasa
Ong Palangdao Rosales Sanchez Santos Satrain Tabo (2014-2015)

Labor Relations Case Digest - Atty. Joyrich Golangco

The failure of respondent to answer the July 9 and 10, 2001 memoranda of
petitioner is clearly intentional. He reported to and loitered outside petitioner's
premises but never made any oral or written reply to the said memoranda. This
shows respondent's wrongful and perverse attitude to defy the reasonable
orders which undoubtedly pertain to his duties as an employee of petitioner.

Respondent's absence without leave for almost a month aggravated his


infractions. While in some cases, the length of service of the employee is
considered to mitigate the imposable penalty, we cannot apply the same ruling in
the instant case. Respondent had worked with petitioner for almost seven years
yet he did not give the courtesy, if not gratitude due it by complying with its
directives and explaining his conduct either verbally or in writing. Indeed, to
hold that there is no just cause to terminate respondent would demoralize the
rank and file employees who religiously comply with the lawful orders of their
employer. It may encourage respondent to do even worse and will render a
mockery of the rules of discipline that employees are required to observe. In
protecting the rights of the laborer, courts cannot authorize the oppression or
self-destruction of the employer.
Dismissal was for just cause pero walang due process since di sya nabigyan ng
notice. Nominal damages was awarded.

183. THE COCA-COLA EXPORT CORPORATION vs. CLARITA P. GACAYAN

DOCTRINES: When the breach of trust or loss of confidence theorized upon is


not borne by clearly established facts, such dismissal on the ground of loss and
confidence cannot be countenanced.

In order to constitute serious misconduct which will warrant the dismissal of an


employee, it is not sufficient that the act or conduct complained of has violated
some established rules or policies. It is equally important and required that the
act or conduct must have been done with wrongful intent.
Petitioner (employer) has the management prerogative to discipline its
employees and to impose appropriate penalties on erring workers pursuant to
company rules and regulations. [The] Court upholds these management
prerogatives so long as they are exercised in good faith for the advancement of

the employers interest and not for the purpose of defeating or circumventing the
rights of the employees under special laws and valid agreements. Xxx However,
petitioners rules and regulations cannot preclude the State from inquiring
whether the strict and rigid application or interpretation thereof would be harsh
to the employee.
FACTS: Respondent Clarita P. Gacayan was holding the position of Senior
Financial Accountant of petitioner when she was dismissed for alleged loss of
trust and company.

Under petitioners company policy, one of the benefits enjoyed by its employees
was the reimbursement of meal and transportation expenses incurred while
rendering overtime work. This reimbursement was allowed only when the
employee worked overtime for at least four hours on a Saturday, Sunday or
holiday, and for at least two hours on weekdays. The maximum amount allowed
to be reimbursed was P150.00. It was in connection with this company policy
that petitioner called the attention of respondent and required her to explain the
alleged alterations in three receipts which she submitted to support her claim for
reimbursement of meal expenses, to wit:
1. McDonalds Receipt No. 875493 dated October 1, 1994 for P111.00 - date of
issuance of the receipt was altered. The receipt was actually issued for a meal
bought on October 2, 1994 and not on October 1, 1994.
2. Shakeys Pizza Parlor Receipt No. 122658 dated November 20, 1994
for P174.06; - the receipt was actually for three orders of Bunch of Lunch, and
not for Buddy Pack (+ mojos)
3. Shakeys Pizza Parlor Receipt No. 41274 dated July 19, 1994 for P130.50 which contained an annotation "w/ CAV 50% only P130.50." Such annotation
meant that respondent was claiming only half of the total amount indicated in
the receipt as the said meal was supposedly shared with another employee,
Corazon A. Varona. Said employee, however, denied that she ordered and shared
the food covered by the receipt in question.

Petitioner sent respondent a letter directing her to explain why she should not be
subjected to disciplinary sanctions for violating Section II, No. 15, paragraph (d)
of the companys rules and regulations which punishes with dismissal the

3E Andaya Ching Espiritu Hefti Galvez Gammad Lainez Lui Madamba Nagera Narvasa
Ong Palangdao Rosales Sanchez Santos Satrain Tabo (2014-2015)

Labor Relations Case Digest - Atty. Joyrich Golangco

submission of any fraudulent item of expense. Respondent submitted her


explanation and denied any personal knowledge in the commission of the
alterations in the subject receipts.

Petitioner sent respondent a memorandum inviting her to a hearing and formal


investigation on January 17, 1995, to give her an opportunity to explain the
issues against her. Respondent was also advised that she was free to bring along
a counsel of her choice. At said hearing, she was extensively questioned
regarding the alterations on the receipts.
Respondent did not attend the January 23, 1995 hearing, citing her doctors
advice to rest since she was suffering from "severe mixed migraine and muscle
contraction headache."

On January 24, 1995, respondent filed an application for leave from January 13 to
February 3; and on January 31, filed for another leave for the period February 624, 1995.
On February 23, 1995, petitioner sent another notice to respondent informing
her of the re-setting of the continuation of the formal investigation on March 15,
advising her that it would be her last opportunity to fully explain her side.
However, respondent did not attend said hearing. Petitioner then concluded the
formal investigation.

Petitioner dismissed respondent for fraudulently submitting tampered and/or


altered receipts in support of her petty cash reimbursements in gross violation of
the companys rules and regulations.
Respondent filed a complaint for illegal dismissal, non-payment of service
incentive leave, sick leave and vacation leave with prayer for reinstatement,
payment of backwages as well as for damages and attorneys fees, against
petitioner with the NLRC. In her position paper, respondent averred that,
assuming arguendo that she altered the receipts in question, dismissal was too
harsh a penalty for her. Petitioner maintained that respondent was dismissed for
cause, that of "tampering official receipts to substantiate her claim for (meal)
reimbursement which reflects her questionable integrity and honesty."

LA: dismissed the complaint, holding that the termination was valid because
there was due process and that the termination for repeatedly submitting
fraudulent items of expense was in violation of respondents [petitioners]
company rules and regulations which consequently resulted in loss of trust and
confidence.
NLRC: affirmed LA decision; denied the motion for reconsideration

CA: reversed and set aside the NLRC resolutions, holding that the penalty of
dismissal imposed on respondent was too harsh. It directed petitioner to
immediately reinstate respondent to her former position, if possible, or a
substantially equivalent position without loss of seniority rights and with full
backwages.

ISSUES: WON respondents repeated submission of altered or tampered receipts


to support her claim for reimbursement constitutes a betrayal of the employers
trust and confidence and a serious misconduct, thus, giving cause for the
termination of her employment with petitioner.
HELD:NO. After examining the records of the case, this Court finds that
respondents dismissal from employment was not grounded on any of the just
causes enumerated under Article 282 of the Labor Code.

At the outset, it is important to note that the term "trust and confidence" is
restricted to managerial employees. In Samson v. National Labor Relations
Commission, the Court, citing Section 2(b), Rule I, Book III of the Omnibus Rules
Implementing the Labor Code, enumerated the conditions for one to be properly
considered a managerial employee:
(1) Their primary duty consists of the management of the establishment in which
they are employed or of a department or sub-division thereof;
(2) They customarily and regularly direct the work of two or more employees
therein; [and]
(3) They have the authority to hire or fire other employees of lower rank; or
their suggestions and recommendations as to the hiring and firing and as to the
promotion or any other change of status of other employees are given particular
weight.

3E Andaya Ching Espiritu Hefti Galvez Gammad Lainez Lui Madamba Nagera Narvasa
Ong Palangdao Rosales Sanchez Santos Satrain Tabo (2014-2015)

Labor Relations Case Digest - Atty. Joyrich Golangco

In the instant case, respondent was the Senior Financial Accountant with the Job
Description of a Financial Project Analyst. Respondent, among others, "provides
support in the form of financial analyses and evaluation of alternative strategies
or action plans to assist management in strategic and operational decisionmaking, x x x liaises with the Bottler to comply with Corporate Bottler financial
reporting requirements and to ensure Bottlers plans are aligned with TCCECs, x
x x and assists management on various initiatives on ad hoc basis."

In Nokom v. National Labor Relations Commission,44 this Court set the guidelines
for the application of the doctrine of loss of confidence
(a) Loss of confidence should not be simulated;
(b) It should not be used as a subterfuge for causes which are improper, illegal or
unjustified;
(c) It may not be arbitrarily asserted in the face of overwhelming evidence to the
contrary; and
(d) It must be genuine, not a mere afterthought to justify earlier action taken in
bad faith.
In the instant case, the basis for terminating the employment of respondent was
for gross violation of the companys rules and regulations, as specified in the
termination letter. No mention was made regarding petitioners alleged loss of
trust and confidence in respondent. Neither was there any explanation nor
discussion of the alleged sensitive and delicate position of respondent requiring
the utmost trust of petitioner.

It bears emphasizing that the right of an employer to dismiss its employees on


the ground of loss of trust and confidence must not be exercised arbitrarily. For
loss of trust and confidence to be a valid ground for dismissal, it must be
substantial and founded on clearly established facts. Loss of confidence must
not be used as a subterfuge for causes which are improper, illegal or unjustified;
it must be genuine, not a mere afterthought, to justify earlier action taken in bad
faith. Because of its subjective nature, this Court has been very scrutinizing in
cases of dismissal based on loss of trust and confidence because the same can
easily be concocted by an abusive employer. Thus, when the breach of trust or
loss of confidence theorized upon is not borne by clearly established facts,

as in the instant case, such dismissal on the ground of loss and confidence
cannot be countenanced.
In the instant case, it was only in the Reply to Respondents Comment that
petitioner made mention of another ground for the dismissal of respondent, that
of serious misconduct. Such allegation appears to be a mere afterthought, being
tardily raised only in the Reply.
In Marival Trading, Inc. v. National Labor Relations Commission, we held, thus:

Misconduct has been defined as improper or wrong conduct. It is the


transgression of some established and definite rule of action, a forbidden act, a
dereliction of duty, willful character, and implies wrongful intent and not mere
error of judgment. The misconduct to be serious must be of such grave and
aggravated character and not merely trivial and unimportant. Such misconduct,
however serious, must nevertheless be in connection with the employees work
to constitute just cause for his separation. Thus, for misconduct or improper
behavior to be a just cause for dismissal, (a) it must be serious; (b) must relate to
the performance of the employees duties; and (c) must show that the employee
has become unfit to continue working for the employer. Indeed, an employer
may not be compelled to continue to employ such person whose continuance in
the service would be patently inimical to his employers business.
In this light, the alleged infractions of respondent could hardly be considered
serious misconduct. It is well to stress that in order to constitute serious
misconduct which will warrant the dismissal of an employee, it is not
sufficient that the act or conduct complained of has violated some
established rules or policies. It is equally important and required that the
act or conduct must have been done with wrongful intent. Such is, however,
lacking in the instant case.

While this Court does not condone respondents act of submitting altered and/or
tampered receipts to support her claim for reimbursement, we nevertheless
agree with the finding of the CA that, under the attendant facts, the dismissal
meted out on respondent appears to be too harsh a penalty.

3E Andaya Ching Espiritu Hefti Galvez Gammad Lainez Lui Madamba Nagera Narvasa
Ong Palangdao Rosales Sanchez Santos Satrain Tabo (2014-2015)

Labor Relations Case Digest - Atty. Joyrich Golangco

As respondents employer, petitioner has the right to regulate, according to its


discretion and best judgment, work assignments, work methods, work
supervision, and work regulations, including the hiring, firing and discipline of its
employees. Indeed, petitioner has the management prerogative to discipline its
employees, like herein respondent, and to impose appropriate penalties on
erring workers pursuant to company rules and regulations. This Court upholds
these management prerogatives so long as they are exercised in good faith for
the advancement of the employers interest and not for the purpose of defeating
or circumventing the rights of the employees under special laws and valid
agreements.
In the instant case, petitioner alleged that under its rules and regulations,
respondents submission of fraudulent items of expense is punishable by
dismissal. However, petitioners rules cannot preclude the State from
inquiring whether the strict and rigid application or interpretation thereof
would be harsh to the employee. Even when an employee is found to have
transgressed the employers rules, in the actual imposition of penalties upon the
erring employee, due consideration must still be given to his length of service
and the number of violations committed during his employ. Respondent had no
previous record in her 9 years of service; this would have been her first
offense. Respondent had also been a recipient of various commendations
attesting to her competence and diligence in the performance of her duties, not
only from petitioner, but also from petitioners counterparts in Poland5 and
Thailand. Respondent also countered that she acted in good faith and with no
wrongful intent when she submitted the receipts in support of her claim for
reimbursement of meal allowance. According to respondent, only the dates or
items were altered on the receipts. She did not claim more than what was
allowed as meal expense for the days that she rendered overtime work. She
believed that the submission of receipts was simply for records-keeping, since
she actually rendered overtime work on the dates that she claimed for meal
allowance. All told, this Court holds that the penalty of dismissal imposed on
respondent is unduly oppressive and disproportionate to the infraction which
she committed. A lighter penalty would have been more just.
SIDE NOTE: RELIEFS After a finding of illegal dismissal herein, we apply the
foregoing provision entitling respondent Clarita P. Gacayan to reinstatement

without loss of seniority rights and other privileges and full backwages, inclusive
of allowances and other benefits or their monetary equivalent computed from
the time the compensation was not paid up to the time of her reinstatement.
Thus, the award of backwages by the Court of Appeals is in order. However, the
Court of Appeals period of computation of the award of backwages must be
modified. The Court of Appeals ruled that:
Coca Cola Export Corporation is hereby directed to immediately reinstate
[respondent] to her former position, if possible, otherwise, to a substantially
equivalent position without loss of seniority rights and with full backwages,
based on her last monthly salary, to be computed from the date of her dismissal
from the service up to the date of finality of this decision, without any
qualifications or deductions. No costs.
184. DOMINGO vs RAYALA

DOCTRINE: Chairman of the NLRC shall hold office during good behavior until
he or she reaches the age of sixty-five, unless sooner removed for cause as
provided by law or becomes incapacitated to discharge the duties of the office
FACTS On November 16, 1998, Ma. Lourdes T. Domingo (Domingo), then
Stenographic Reporter III at the NLRC, filed a Complaint for sexual harassment
against Rayala before Secretary Bienvenido Laguesma of the Department of
Labor and Employment (DOLE).The committee constituted found Rayala guilty
of the offense charged. Secretary Laguesma submitted a copy of the Committee
Report and Recommendation to the OP, but with the recommendation that the
penalty should be suspension for six (6) months and one (1) day, in accordance
with AO 250.

On May 8, 2000, the OP issued AO 119, disagreeing with the recommendation


that respondent be meted only the penalty of suspension for six (6) months and
one (1) day considering the circumstances of the case because of the nature of
the position of Reyala as occupying the highest position in the NLRC, being its
Chairman. Long digest by Ernani Tadili.It was ordered that Rayala be dismissed
from service for being found guilty of grave offense of disgraceful and immoral
conduct. Rayala filed Motions for Reconsideration until the case was finally
referred to the Court of Appeals for appropriate action. The CA found Reyala

3E Andaya Ching Espiritu Hefti Galvez Gammad Lainez Lui Madamba Nagera Narvasa
Ong Palangdao Rosales Sanchez Santos Satrain Tabo (2014-2015)

Labor Relations Case Digest - Atty. Joyrich Golangco

guilty and imposed the penalty of suspension of service for the maximum period
of one (1) year.
Domingo filed a Petition for Review before the SC

ISSUES: Whether rayala was validly removed or dismissed

HELD: No.It is noteworthy that under AO 250, sexual harassment amounts to


disgraceful and immoral conduct Thus, any finding of liability for sexual
harassment may also be the basis of culpability for disgraceful and immoral
conduct.
With the foregoing disquisitions affirming the finding that Rayala committed
sexual harassment, we now determine the proper penalty to be imposed.

Rayala attacks the penalty imposed by the OP. He alleges that under the
pertinent Civil Service Rules, disgraceful and immoral conduct is punishable by
suspension for a period of six (6) months and one (1) day to one (1) year. He also
argues that since he is charged administratively, aggravating or mitigating
circumstances cannot be appreciated for purposes of imposing the penalty.

Under AO 250, the penalty for the first offense is suspension for six (6) months
and one (1) day to one (1) year, while the penalty for the second offense is
dismissal. On the other hand, Section 22(o), Rule XVI of the Omnibus Rules
Implementing Book V of the Administrative Code of 1987 and Section 52 A(15) of
the Revised Uniform Rules on Administrative Cases in the Civil Service both provide
that the first offense of disgraceful and immoral conduct is punishable by
suspension of six (6) months and one (1) day to one (1) year. A second offense is
punishable by dismissal.
Under the Labor Code, the Chairman of the NLRC shall hold office during good
behavior until he or she reaches the age of sixty-five, unless sooner removed
for cause as provided by law or becomes incapacitated to discharge the duties
of the office.

In this case, it is the President of the Philippines, as the proper disciplining


authority, who would determine whether there is a valid cause for the removal of

Rayala as NLRC Chairman. This power, however, is qualified by the phrase "for
cause as provided by law." Thus, when the President found that Rayala was
indeed guilty of disgraceful and immoral conduct, the Chief Executive did not
have unfettered discretion to impose a penalty other than the penalty provided
by law for such offense. As cited above, the imposable penalty for the first offense
of either the administrative offense of sexual harassment or for disgraceful and
immoral conduct is suspension of six (6) months and one (1) day to one (1) year.
Accordingly, it was error for the Office of the President to impose upon
Rayala the penalty of dismissal from the service, a penalty which can only
be imposed upon commission of a second offense.
Even if the OP properly considered the fact that Rayala took advantage of his
high government position, it still could not validly dismiss him from the service.
Under the Revised Uniform Rules on Administrative Cases in the Civil
Service, taking undue advantage of a subordinate may be considered as an
aggravating circumstance and where only aggravating and no mitigating
circumstances are present, the maximum penalty shall be imposed. Hence, the
maximum penalty that can be imposed on Rayala is suspension for one (1) year.
185. Philippine Aeolus Automotive United Corporation vs NLRC

DOCTRINE: On serious misconduct warranting dismissal of an employee, has


ruled that for misconduct or improper behavior to be a just cause for dismissal
(a) it must be serious; (b) must relate to the performance of the employees
duties; and, (c) must show that the employee has become unfit to continue
working for the employer.

Gross negligence implies a want or absence of or failure to exercise slight care or


diligence, or the entire absence of care. It evinces a thoughtless disregard of
consequences without exerting any effort to avoid them. The negligence, to
warrant removal from service, should not merely be gross but also habitual.
FACTS: Petitioner Philippine Aeolus Automotive United Corporation (PAAUC) is
a corporation duly organized and existing under Philippine laws, petitioner
Francis Chua is its President while private respondent Rosalinda C. Cortez was a
company nurse of petitioner corporation until her termination. On 5 October

3E Andaya Ching Espiritu Hefti Galvez Gammad Lainez Lui Madamba Nagera Narvasa
Ong Palangdao Rosales Sanchez Santos Satrain Tabo (2014-2015)

Labor Relations Case Digest - Atty. Joyrich Golangco

1994 a memorandum was issued by Ms. Myrna Palomares, Personnel Manager of


petitioner corporation, addressed to private respondent Rosalinda C. Cortez
requiring her to explain within 48 hours why no disciplinary action should be
taken against her (a) for throwing a stapler at Plant Manager William Chua, her
superior, and uttering invectives against him; (b) for losing the amount
of P1,488.00 entrusted to her by Plant Manager Chua to be given to Mr. Fang of
the CLMC Department; and, (c) for asking a co-employee to punch-in her time
card thus making it appear that she was in the office in the morning of 6
September 1994 when in fact she was not. The memorandum however was
refused by private respondent although it was read to her and discussed with her
by a co-employee. She did not also submit the required explanation, so that while
her case was pending investigation the company placed her under preventive
suspension for 30 days. While Cortez was still under preventive suspension,
another memorandum was issued by petitioner corporation giving her seventytwo (72) hours to explain why no disciplinary action should be taken against her
for allegedly failing to process the ATM applications of her 9 co-employees with
the Allied Banking Corporation. Private respondent also refused to receive the
second memorandum although it was read to her by a co-employee. Meanwhile,
private respondent submitted a written explanation with respect to the loss of
the P1,488.00 and the punching-in of her time card by a co-employee. 1994 a
third memorandum was issued to private respondent, this time informing her of
her termination from the service on grounds of gross and habitual neglect of
duties, serious misconduct and fraud or willful breach of trust. Private
respondent then filed with the LA a complaint for illegal dismissal, non-payment
of annual service incentive leave and 13th month pay. LA held that Cortez was
validly and legally terminated. On appeal, NLRC reversed LA decision and
ordered petitioner to be reinstated, hence this petition.
ISSUES: WON private respondent was legally terminated by petitioner? And if in
the negative, should she be reinstated?
HELD: No, The Supreme Court, in a litany of decisions on serious misconduct
warranting dismissal of an employee, has ruled that for misconduct or improper
behavior to be a just cause for dismissal (a) it must be serious; (b) must relate to
the performance of the employees duties; and, (c) must show that the employee
has become unfit to continue working for the employer. The act of private

respondent in throwing a stapler and uttering abusive language upon the person
of the plant manager may be considered, from a lay man's perspective, as a
serious misconduct. However, in order to consider it a serious misconduct that
would justify dismissal under the law, it must have been done in relation to the
performance of her duties as would show her to be unfit to continue working for
her employer. The acts complained of, under the circumstances they were done,
did not in any way pertain to her duties as a nurse. Her employment
identification card discloses the nature of her employment as a nurse and no
other. Also, the memorandum informing her that she was being preventively
suspended pending investigation of her case was addressed to her as a nurse.

As regards the third alleged infraction, i.e., the act of private respondent in asking
a co-employee to punch-in her time card, although a violation of company rules,
likewise does not constitute serious misconduct. Firstly, it was done by her in
good faith considering that she was asked by an officer to perform a task outside
the office, which was for the benefit of the company, with the consent of the plant
manager. Secondly, it was her first time to commit such infraction during her five
(5)-year service in the company. Finally, the company did not lose anything by
reason thereof as the offense was immediately known and corrected.
On alleged infraction No. 4, The mere delay/failure to open an ATM account for
nine employees is not sufficient, by itself, to support a conclusion that Rosalinda
is guilty of gross and habitual neglect of duties. First, petitioner did not show that
opening an ATM is one of her primary duties as company nurse. Second,
petitioner failed to show that Rosalinda intentionally, knowingly, and purposely
delayed the opening of ATM accounts for petitioners employees.

Gross negligence implies a want or absence of or failure to exercise slight care or


diligence, or the entire absence of care. It evinces a thoughtless disregard of
consequences without exerting any effort to avoid them. The negligence, to
warrant removal from service, should not merely be gross but
also habitual. Likewise, the ground "willful breach by the employee of the trust
reposed in him by his employer" must be founded on facts established by the
employer who must clearly and convincingly prove by substantial evidence the
facts and incidents upon which loss of confidence in the employee may fairly be

3E Andaya Ching Espiritu Hefti Galvez Gammad Lainez Lui Madamba Nagera Narvasa
Ong Palangdao Rosales Sanchez Santos Satrain Tabo (2014-2015)

Labor Relations Case Digest - Atty. Joyrich Golangco

made to rest. All these requirements prescribed by law and jurisprudence are
wanting in the case at bar.

186. PHARMACIA and UPJOHN, INC. (now PFIZER PHILIPPINES, INC.),


ASHLEY MORRIS, ALEDA CHU, JANE MONTILLA & FELICITO
GARCIA, Petitioners, vs. RICARDO P. ALBAYDA, JR., Respondent.

FACTS: Respondent was designated by petitioner Pharmacia as District Sales


Manager assigned to District XI in the Western Visayas area. During the period of
his assignment, respondent settled in Bacolod City.
Pursuant to a Memorendum announcing the sales force structure for the year
2000, respondent was reassigned as District Sales Manager to District XII in the
Northern Mindanao area. One of the key areas covered in District XII is Cagayan
de Oro City.

Respondent opposed the reassignment in a letter to Pharmacias Vice-President


for Sales and Marketing and in a subsequent letter to Pharmacias National Sales
and External Business Manager because of inconvenience and dislocation from
his family. However the same was denied explaining their need of respondents
expertise to build the business in Cagayan de Oro. In reply thereto, respondent
argued that the issues he raised regarding the loss of his family income, the
additional cost of housing and other additional expenses he will incur in
Mindanao was not settled.

Thereafter, Jane B. Montilla, Pharmacias Human Resource Manager, met with


respondent to discuss his situation. After the meeting, Montilla sent respondent a
memorandum wherein his request to continue to work in Western Visayas was
denied as there was no vacant position in those areas. Respondent was also given
an option to be assigned in Metro Manila as a position in the said territory had
recently opened. Montilla gave respondent until June 2, 2000 to decide on
whether to accept a post in Cagayan de Oro City or in Manila.
In a letter dated June 8, 2000, respondent told Montilla that he will be airing his
grievance before the NLRC. Montilla sent respondent another memorandum

stating that it is in the best interest of the company for respondent to report to
the Makati office to assume his new area of assignment.

In a memorandum dated June 15, 2000, Montilla stated that contrary to the
opinion of respondent, respondent is entitled to Relocation Benefits and
Allowance pursuant to the companys Benefits Manual and directed respondent
to report for work in Manila within 5 working days from receipt of the
memorandum.

In another memorandum Montilla stated that she had not heard from respondent
since his June 8, 2000 letter and that he has not replied to their last
memorandum. Respondent was warned that the same would be a final notice for
him to report for work in Manila within 5 working days from receipt of the
memo; otherwise, his services will be terminated on the basis of being AWOL.
On July 13, 2000, Montilla sent respondent a memorandum notifying him of their
decision to terminate his services after he repeatedly refused to report for work
despite due notice on the basis of AWOL and insubordination pursuant to
Article 282 of the Labor Code which shall be effective on July 19, 2000.

Respondent filed a Complaint for constructive dismissal. LA dismissed the case.


NLRC dismissed the appeal. CA rendered a Decision ruling in favor of
respondent.
ISSUES: WON respondent was validly dismissed.

HELD:*Quinote at sinustain lang ng SC ung decisions ng LA at NLRC kasi ang


main issue dito ay tungkol sa factual findings ng NLRC affirming those of the LA,
are entitled to great weight and will not be disturbed if they are supported by
substantial evidence. Ung CA kasi nireverse ung findings ng LA at ng NLRC.
Pinagalitan lng ng SC ung CA sa case na ito.

Transfer was held to be a valid exercise of management prerogative. LA


explained that the reassignment of respondent was not a demotion as he will
also be assigned as a District Sales Manager in Mindanao or in Metro Manila and
that the notice of his transfer did not indicate that his emoluments will be

3E Andaya Ching Espiritu Hefti Galvez Gammad Lainez Lui Madamba Nagera Narvasa
Ong Palangdao Rosales Sanchez Santos Satrain Tabo (2014-2015)

Labor Relations Case Digest - Atty. Joyrich Golangco

reduced. The NLRC affirmed in toto the findings of the LA. The NLRC ruled that
petitioners restructuring move was a valid exercise of its management
prerogative and authorized under the employment contract of respondent. There
is nothing in the records to prove that the restructuring move of respondent
company was done with ill motives or with malice and bad faith purposely to
constructively terminate complainants employment. Such misinterpretation or
misguided supposition by complainant is belied by the fact that respondents
officers had in several communications officially sent to complainant, expressly
recognized complainants expertise and capabilities as a top sales man and
manager for which reason the respondent company needs his services and skills
to energize the low-performing areas in order to maximize business
opportunities and to afford complainant an opportunity for further growth and
development. Further, respondent agreed, as evidenced by the employment
contract, during the period of employment, to be assigned to any work or
workplace for such period as may be determined by the company and whenever
the operations thereof require such assignment.
The LA ruled that because of respondents adamant refusal to be reassigned,
petitioners had valid grounds to terminate his employment. Clearly, the
complainant had abandoned his work by reason of his being on AWOL as a
consequence of vigorous objection to his transfer to either Cagayan de Oro or
Metro Manila. The long period of absence of complainant without official leave
from April to July 19, 2000 is more than sufficient ground to dismiss him. The
refusal of complainant to accept his transfer of assignment is a clear willful
disobedience of the lawful order of his employer and a ground to terminate his
services under Article 282, par. (a) of the Labor Code, as amended. The series of
chances given complainant to report for work, coupled by his adamant refusal to
report to his new assignment, is a conclusive indication of willful disobedience of
the lawful orders of his employer.
NLRC also ruled that respondent was guilty of insubordination, thus:

Apparently, complainant, by his unjustified acts of refusing to be transferred


either to Mindanao or Manila for personal reasons, absent any bad faith or malice
on the part of respondents, has deliberately ignored and defied lawful orders of

his employer. An employee who refuses to be transferred, when such transfer is


valid, is guilty of insubordination.

Based on the foregoing, this Court rules that the findings of the LA and the NLRC
are supported by substantial evidence. The LA clearly outlined the steps taken by
petitioners and the manner by which respondent was eventually dismissed. The
NLRC, for its part, explained why respondent was guilty of insubordination. No
abuse of discretion can, therefore, be attributed to both agencies, and the CA was
certainly outside its mandate in reversing such findings.

Further, this Court has long stated that the objection to the transfer being
grounded solely upon the personal inconvenience or hardship that will be caused
to the employee by reason of the transfer is not a valid reason to disobey an
order of transfer. Such being the case, respondent cannot adamantly refuse to
abide by the order of transfer without exposing himself to the risk of being
dismissed. Hence, his dismissal was for just cause in accordance with Article
282(a) of the Labor Code.
Lastly, while it is understandable that respondent does not want to relocate his
family, this Court agrees with the NLRC when it observed that such
inconvenience is considered an "employment" or "professional" hazard which
forms part of the concessions an employee is deemed to have offered or
sacrificed in the view of his acceptance of a position in sales.
187. JERUSALEM VS. HOCK

Keywords: credit card scam, VISA


DOCTRINE: For breach of trust and confidence to become a valid ground for the
dismissal of an employee, the cause of loss of trust and confidence must be
related to the performance of the employees duties.
FACTS

James Ben L. Jerusalem (James) was employed by Keppel Monte Bank


(Keppel) as Assistant Vice-President. He was assigned as Head of the
newly created VISA Credit Card Department. The bank subsequently re-

3E Andaya Ching Espiritu Hefti Galvez Gammad Lainez Lui Madamba Nagera Narvasa
Ong Palangdao Rosales Sanchez Santos Satrain Tabo (2014-2015)

Labor Relations Case Digest - Atty. Joyrich Golangco

organized the VISA Credit Card Department and reduced it to a mere


unit.
Carrying the same rank, James was reassigned as Head of the Marketing
and Operations of the Jewelry Department. The VISA Credit Card Unit
was then headed by Senior Vice President Roberto Borromeo (Roberto)
and supported by Marciana C. Gerena (Marciana), Rosario R. Ronquillo
and Aileen Alcantara as Unit Head, Processor and Bookkeeper,
respectively.
In or about May 1999, James received from Jorge Javier (Jorge) a sealed
envelope said to be containing VISA Card application forms. Jorge is a
Keppel Visa Card Holder since December 1998. James immediately
handed over the envelope with accomplished application forms to the
VISA Credit Card Unit. All in all, the VISA credit card applications
referred by Jorge which James forwarded to the VISA Credit Card Unit
numbered 67, all of which were subsequently approved. As it turned out,
all the accounts under these approved applications became past due.
Marciana sent a letter to Jorge asking the latter to assist the bank in the
collection of his referred VISA accounts which have already an
accumulated principal balance of P6,281,443.90 excluding interest and
service fees in the amount of P1,157,490.08.
On the same date, James upon knowing the status of the accounts
referred by Jorge, sent a Memorandum to Roberto recommending the
filing of a criminal case for estafa against Jorge. James even warned
Keppel that immediate action should be taken while Jorge is still in the
country.
James received a Notice to Explain from Keppels Vice President for
Operations, Sunny Yap (Sunny), why no disciplinary action should be
taken against him for referring/endorsing fictitious VISA card
applicants. The said referrals resulted in substantial financial losses to
Keppel.
On August 23, 2000, James submitted his written explanation to Sunny.
He pointed out that he had no participation in the processing of the VISA
card applications since he was no longer connected with the VISA Credit
Card Unit at the time of such transactions. He explained that he can only
endorse the applications referred by Jorge to the VISA Credit Card Unit
because he was already transferred to Jewelry Department, as Head.

On September 26, 2000, the Manager for Human Resources Department,


handed to James a Notice of Termination informing the latter that he
was found guilty of breach of trust and confidence for knowingly and
maliciously referring, endorsing and vouching for VISA card applicants
who later turned out to be impostors resulting in financial loss to
Keppel.
This prompted James to file before the Labor Arbiter a complaint for
illegal dismissal, illegal confiscation of car with prayer for the payment
of vacation/sick leaves, 13th month pay, damages, attorneys fees and
full backwages against Keppel on October 9, 2000.
o LA: Keppel is guilty of illegal dismissal
o NLRC: affirmed LA
o CA: set aside the NLRCs resolution and dismissed Jerusalems
complaint

ISSUES: whether Keppel legally terminated Jamess employment on the ground


of willful breach of trust and confidence.
HELD: NO

(Refer To Art. 282 of the Labor Code)

Article 282(c) of the Labor Code prescribes two separate and distinct
grounds for termination of employment, namely: (1) fraud; or (2) willful
breach by the employee of the trust reposed in him by his employer or
duly authorized representative.
"Law and jurisprudence have long recognized the right of employers to
dismiss employees by reason of loss of trust and confidence." As
provided for in Article 282, an employer may terminate an employees
employment for fraud or willful breach of trust reposed in him. "But, in
order to constitute a just cause for dismissal, the act complained of
must be work-related such as would show the employee
concerned to be unfit to continue working for the employer."\
The burden of establishing facts as bases for an employers loss of
confidence in an employee facts which reasonably generate belief by
the employer that the employee was connected with some misconduct
and the nature of his participation therein is such as to render him

3E Andaya Ching Espiritu Hefti Galvez Gammad Lainez Lui Madamba Nagera Narvasa
Ong Palangdao Rosales Sanchez Santos Satrain Tabo (2014-2015)

Labor Relations Case Digest - Atty. Joyrich Golangco

unworthy of trust and confidence demanded of his position is on the


employer.
Proof beyond reasonable doubt is not required. It is sufficient that
there must only be some basis for such loss of confidence or that
there is reasonable ground to believe, if not to entertain, the moral
conviction that the concerned employee is responsible for the
misconduct and that the nature of his participation therein
rendered him absolutely unworthy of trust and confidence
demanded by his position
In this case, there is no doubt that James held a position of trust and
confidence as Assistant Vice-President of the Jewelry Department.
Loss of trust and confidence, to be a valid cause for dismissal, must be
based on a willful breach of trust and founded on clearly established
facts. The basis for the dismissal must be clearly and convincingly
established but proof beyond reasonable doubt is not necessary."
Keppels evidence against James fails to meet this standard.
From the findings of both the Labor Arbiter and the NLRC it is clear that
James did nothing wrong when he handed over to Marciana the
envelope containing the applications of persons under the referred
accounts of Jorge who were later found to be fictitious. As the records
now stand, James was no longer connected with the VISA Credit Card
Unit when the 67 applications for VISA card were approved. His act
therefore of forwarding the already accomplished applications to
the VISA Credit Card Unit is proper as he is not in any position to act
on them. The processing and verification of the identities of the
applicants would have been done by the proper department, which is
the VISA Credit Card Unit. Therefore, it is incumbent upon Marciana as
Unit Head to have performed her duties. As correctly observed by the
Labor Arbiter, Keppel had gone too far in blaming James for the
shortcomings and imprudence of Marciana. The invocation of Keppel
of the loss of trust and confidence as ground for Jamess termination
has therefore no basis at all.
Having shown that Keppel failed to discharge its burden of proving
that Jamess dismissal is for a just cause, we have no other recourse
but to declare that such dismissal based on the ground of loss of
trust and confidence was illegal. This is in consonance with the

constitutional guarantee of security of tenure.

188. Reno Foods vs. NLM-Katipunan

DOCTRINE: There is no legal or equitable justification for awarding financial


assistance to an employee who was dismissed for stealing company property.
Social justice and equity are not magical formulas to erase the unjust acts
committed by the employee against his employer. While compassion for the
poor is desirable, it is not meant to coddle those who are unworthy of such
consideration.

FACTS: Petitioner Reno Foods, Inc. (Reno Foods) is a manufacturer of canned


meat products of which Vicente Khu is the president and is being sued in that
capacity. Respondent Nenita Capor (Capor) was an employee of Reno Foods
until her dismissal on October 27, 1998.

During a standard operating procedure of the company in searching its


employees, the guard on duty found six Reno canned goods wrapped in nylon
leggings inside Capors fabric clutch bag. The only other contents of the bag were
money bills and a small plastic medicine container.
Petitioners accorded Capor several opportunities to explain her side,
often with the assistance of the union officers of Nagkakaisang Lakas ng
Manggagawa (NLM) Katipunan. In fact, after petitioners sent a Notice of
Termination to Capor, she was given yet another opportunity for reconsideration
through a labor-management grievance conference held on November 17, 1999.
Unfortunately, petitioners did not find reason to change its earlier decision to
terminate Capors employment with the company.
On December 8, 1998, petitioners filed a complaint-affidavit against
Capor for qualified theft. Consequently, an Information was filed against Capor
docketed as Criminal Case No. 207-58-MN.

Meanwhile, the Nagkakaisang Lakas ng Manggagawa (NLM) Katipunan


filed on behalf of Capor a complaint for illegal dismissal and money claims
against petitioners. The complaint prayed that Capor be paid her full backwages
as well as moral and exemplary damages.

3E Andaya Ching Espiritu Hefti Galvez Gammad Lainez Lui Madamba Nagera Narvasa
Ong Palangdao Rosales Sanchez Santos Satrain Tabo (2014-2015)

Labor Relations Case Digest - Atty. Joyrich Golangco

LA: Capor was validly dismissed for serious misconduct which is a just cause for
termination. Under Article 232 of the Labor Code, an employer may terminate
the services of an employee for just cause, such as serious misconduct. In this
case, the Labor Arbiter found that theft of company property is tantamount to
serious misconduct; as such, Capor is not entitled to reinstatement and
backwages, as well as moral and exemplary damages.
NLRC affirmed the ruling but awarded financial assistance in the form of a
separation pay. CA affirmed the NLRC decision.
ISSUES: WON Capor is entitled to financial assistance

HELD: In Nicolas v. National Labor Relations Commission, we held that a


criminal conviction is not necessary to find just cause for employment
termination. Otherwise stated, an employees acquittal in a criminal case,
especially one that is grounded on the existence of reasonable doubt, will not
preclude a determination in a labor case that he is guilty of acts inimical to the
employers interests.

Criminal cases require proof beyond reasonable doubt while labor disputes
require only substantial evidence, which means such relevant evidence as
a reasonable mind might accept as adequate to justify a conclusion. The evidence
in this case was reviewed by the appellate court and two labor tribunals
endowed with expertise on the matter the Labor Arbiter and the NLRC. They
all found substantial evidence to conclude that Capor had been validly dismissed
for dishonesty or serious misconduct. It is settled that factual findings of quasijudicial agencies are generally accorded respect and finality so long as these are
supported by substantial evidence. In the instant case, we find no compelling
reason to doubt the common findings of the three reviewing bodies.
We find no justification for the award of separation pay to Capor. This
award is a deviation from established law and jurisprudence.

The law is clear. Separation pay is only warranted when the cause for
termination is not attributable to the employees fault, such as those provided in
Articles 283 and 284 of the Labor Code, as well as in cases of illegal dismissal in
which reinstatement is no longer feasible. It is not allowed when an employee is
dismissed for just cause, such as serious misconduct.

Jurisprudence has classified theft of company property as a serious misconduct


and denied the award of separation pay to the erring employee. We see no
reason why the same should not be similarly applied in the case of Capor. She
attempted to steal the property of her long-time employer. For committing such
misconduct, she is definitely not entitled to an award of separation pay.
189. FE LA ROSA, et al. v. AMBASSADOR HOTEL

DOCTRINE: Constructive dismissal occurs when there is cessation of work


because continued employment is rendered impossible, unreasonable or
unlikely; when there is a demotion in rank or diminution in pay or both; or when
a clear discrimination, insensibility, or disdain by an employer becomes
unbearable to the employee.
FACTS: Petitioners Fe La Rosa, Ofelia Velez, Cely Domingo, Jona Natividad and
Edgar De Leon (La Rosa, et al.), were employees of respondent Ambassador
Hotel. La Rosa, et al. led before the National Labor Relations Commission
(NLRC) several complaints for illegal dismissal, illegal suspension, and illegal
deductions against the hotel and its manager. La Rosa, et al. alleged that after
ling their complaints with the Department of Labor, the latter inspected the
hotels premises. The hotel was thereafter found to have been violating labor
standards laws. Consequently, after such incident, the management of the hotel
retaliated by suspending and/or constructively dismissing them by drastically
reducing their work days through the adoption of a work reduction/rotation
scheme. The hotel however countered that such reduction/rotation scheme was
an exercise of its management prerogative due to business losses.

The labor arbiter found the hotel and its manager guilty of illegal dismissal. The
hotel appealed to the NLRC but the latter afrmed the labor arbiters ruling with
modication. The hotel appealed and prayed for the issuance of an injunctive
writ before the Court of Appeals. The appellate court reversed the NLRC decision
and dismissed the petitioners complaints, stating that there was no constructive
dismissal.
ISSUES: Whether or not La Rosa et al. were constructively dismissed

HELD: The records fail, however, to show any documentary proof that the work
reduction scheme was adopted due to Ambassadors business reverses. The

3E Andaya Ching Espiritu Hefti Galvez Gammad Lainez Lui Madamba Nagera Narvasa
Ong Palangdao Rosales Sanchez Santos Satrain Tabo (2014-2015)

Labor Relations Case Digest - Atty. Joyrich Golangco

hotels memorandum dated April 5, 2000 (sic, should be 2002) informing La


Rosa et al. of the adoption of a two-day work scheme effective April 5, 2002 made
no mention why such scheme was being adopted. Neither do the records show
any documentary proof that the hotel suffered nancial losses to justify its
adoption of the said scheme to stabilize its operations.
What is undisputed, as found by both the labor arbiter and the NLRC and
admitted by respondent itself, is that the complaints for violation of labor
standards laws were led by La Rosa et al. against Ambassador Hotel at the
DOLE-NCR, some of which complaints were partially settled; and that almost
immediately after the partial settlement of the said complaints, the work
reduction/rotation scheme was implemented.

Case law holds that constructive dismissal occurs when there is cessation of
work because continued employment is rendered impossible, unreasonable or
unlikely; when there is a demotion in rank or diminution in pay or both; or when
a clear discrimination, insensibility, or disdain by an employer becomes
unbearable to the employee. The hotels sudden, arbitrary and unfounded
adoption of the two-day work scheme which greatly reduced La Rosa, et al.s
salaries renders it liable for constructive dismissal. Upon the other hand, La Rosa
et al.'s immediate ling of complaints for illegal suspension and illegal dismissal
after the implementation of the questioned work scheme, which scheme was
adopted soon after petitioners complaints against respondent for violation of
labor standards laws were found meritorious, negates respondents claim of
abandonment. An employee who takes steps to protest his dismissal cannot by
logic be said to have abandoned his work.
190. Maribago Bluewater Beach Resort vs Nito Dual

DOCTRINE: Serious misconduct which is a just cause for termination under the
law. Theft committed by an employee is a valid reason for his dismissal by the
employer.

FACTS: Petitioner Maribago is a corporation operating a resort hotel and


restaurant in Barangay Maribago, Lapu-lapu city. It hired the respondent Dual as
a waiter and promoted him later as outlet cashier of its Poolbar/Allegro
restaurant

On 9 January 2005, around 6:30 p.m., a group of Japanese guests and their
companions dined at Allegro. Captain waiter Alvin Hiyas (Hiyas for brevity) took
their dinner orders comprising of six (6) sets of lamb and six (6) sets of fish. As
per company procedure, Hiyas forwarded one copy of the order slip to the
kitchen and another copy to respondent. Pursuant to the order slip, fourteen (14)
sets of dinner were prepared by the chef. Hiyas and waiter Genaro Mission, Jr.
(Mission for brevity) served twelve (12) set dinners to the guests, and another
two (2) sets to their guides free of charge a total of 14 sets of dinner
After dinner, at around 9:00 p.m., the guests asked for their bill. Since Hiyas was
attending to other guests, he gave a signal to Mission to give the bill. Mission
asked respondent Dual for the sales transaction receipt and presented this to the
guests. The guests paid the amount indicated on the receipt and thereafter left in
a hurry.
The receipt printed at 10:40 p.m. shows that only P3,036.00 was remitted by
cashier Dual corresponding to six (6) sets of dinner.

In view of the discrepancy between the order slip and the receipt issued,
petitioner Maribago, through its Human Resource Development (HRD) manager,
issued memoranda, all dated 12 January 2005, requiring respondent Dual, Alvin
Hiyas, Ernesto Avenido and Basilio Alcoseba to explain why they should not be
penalized for violating House Rule 4.1 (dishonesty in any nature).
On 14 January 2005, the concerned employees were requested to attend a
clarificatory hearing to be conducted on 15 January 2005. The hearing was
attended by respondent Dual, Human Resource Manager Ignacio Hermias, Jr.,
Chief Security Officer Roland Cubillan, Captain Waiter Hiyas, Chef Arman,
Bartender Avenido, Room Service Waiter Alcoseba, Butcher Ryan Alegrado, John
Marollana, and union officials. This was followed by another clarificatory
hearing conducted on 16 January 2005. It was in the 16 January 2005 hearing
that waiter Mission gave his testimony.
After the investigation, respondent Dual was found guilty of dishonesty for his
fabricated statements and for asking one of the waiters (Mission) to corroborate

3E Andaya Ching Espiritu Hefti Galvez Gammad Lainez Lui Madamba Nagera Narvasa
Ong Palangdao Rosales Sanchez Santos Satrain Tabo (2014-2015)

Labor Relations Case Digest - Atty. Joyrich Golangco

his allegations. He was terminated per memorandum dated 22 January 2005.


Alcoseba was also terminated for dishonesty based on his admission that he
altered the order slip

Respondent: Dual confirms that the orders were for 6 sets of lamb and 6 sets of
fish dinner. However he alleges that 4 sets were cancelled and 2 sets were given
to the guides for free. That such cancellation was confirmed with Alsoceba and
Hiyas. That upon confirming of the cancellation with Alsoceba the latter said that
6 sets were cancelled since some of the guests did not take dinner. And upon
verification with the chief waiter Hiyas the latter said that the orders were
indeed cancelled and placed in the utensil section. Satisfied he issued the
transaction receipt for 6 orders. He also argued that MIsson received the receipt
in the amount of P 3,036 for 6 sets and did not complain that the entry was
incorrect.

Petitioner: the transaction receipt handed to Misson amounted to P10,100 which


the guest alledgedly paid 10,500 with the instructions to return only P200.00. the
rest can be kept by the waiter as a tip. That Misson handed the money to dual
and the later gave misson 200 to give to the guest. Then it was discovered that
only 3,036 was entered by dual in the cash register. The rest of the payment was
missing. And the original transaction receipt for 10,100 was likewise missing and
in its place the transaction receipt of 3,036 was registered. Upon verification it
was found out that the order slip was tampered by Alcoseba to make it appear
that only 6 dinners were ordered. that on 14 January 2005, Dual and Alcoseba
tried to convince Mission to say that he altered the order slip from twelve (12)
sets of dinner to six (6) sets. Mission did not report for work and did not attend
the 15 January 2005 clarificatory hearing since he could not in conscience tell a
lie. At past 11:00 p.m. of 15 January 2005, Dual met Mission and tried again to
convince him to say that only six (6) sets of dinner were ordered. Mission
reported on 16 January 2005 and attended the hearing that day. Dual was not
present.

On 3 February 2005, Dual filed a complaint for unfair labor practice, illegal
dismissal, non-payment of 13th month and separation pay, and damages before
the NLRC, Regional Arbitration Branch No. VII, Cebu City.

LA: There was Illegal Dismissal and ordered the payment of separation pay in
lieu of reinstatement

NLRC: set aside the decision of LA stating that the act of DUAL in depriving
Maribago of its lawful revenue is tantamount fraud which warrants the dismissal
from service. Falsification of commercial documents as a means to malverse
company funds constitutes fraud against the company, MR:denied
CA: reversed the decision and resolution of the NLRC. Finding no sufficient valid
cause to justify respondents dismissal, the Court of Appeals ordered petitioner
to pay respondent full backwages and separation pay, MR: denied
ISSUES: Whether or not there is illegal dismissal

HELD: NO, After a full review of the case, we are constrained to reverse the Court
of Appeals.
The law requires that an employer shall not terminate the services of an
employee except for a just or authorized cause. Otherwise, an employee unjustly
dismissed from work is entitled to reinstatement and full backwages. The law
further requires that the burden of proving the cause for termination rests with
the employer.

In this case, we are in agreement that petitioners evidence proved that


respondent is guilty of dishonesty and of stealing money entrusted to him as
cashier. Instead of reporting P10,100.00 as payment by the guests for their
dinner, respondent cashier only reported P3,036.00 as shown by the receipt
which he admitted to have issued. The receipt which bears his name NITO was
printed at 22:40 (10:40 p.m.) or 1 hour and 40 minutes after the guests had left
at 9:00 p.m. Two other receipts were issued for the same amount at 22:39:55
and 22:40:01. Moreover, respondents claim that he received P3,100.00 only
and gave Mission P64.00 as change is not shown by the receipt that he issued.
The issued receipt does not show that change was given. In addition, the amount
indicated in the receipt does not coincide with Duals contention that only four
(4) dishes were cancelled and two (2) dishes were given free of charge. If such
were the case, then the amount charged to the guests should have been for eight

3E Andaya Ching Espiritu Hefti Galvez Gammad Lainez Lui Madamba Nagera Narvasa
Ong Palangdao Rosales Sanchez Santos Satrain Tabo (2014-2015)

Labor Relations Case Digest - Atty. Joyrich Golangco

(8) sets of dinner and not six (6) sets. As established during the clarificatory
hearing, twelve (12) sets of dinner were served to guests and two (2) dinner sets
were given to the tour guides free of charge. It is clearly indicated in the altered
order slip that six (6) out of the twelve (12) sets of dinner were cancelled.

The allegation of Dual that six (6) dinner sets were indeed cancelled as evidenced
by the dishes he allegedly saw in the utensil station is negated by the testimonies
of the kitchen staff (Chef Armand Galica, Butcher Alegrado and Dessert-in-charge
John Marollano) that twelve (12) set meals were served and consumed. These
testimonies coincide with the claim of waiters Hiyas and Mission that fourteen
(14) sets of dinner were served. The serving of food eliminates the argument of
cancellation.
The alibi of cancellation has no leg to stand on. The standard operating
procedure of Maribago dictates that in cases of cancellation, the order slip has to
be countersigned by the attending waiter (which in this case should have been
Chief Waiter Hiyas) but such was not so in this case.

Respondents acts constitute serious misconduct which is a just cause for


termination under the law. Theft committed by an employee is a valid reason for
his dismissal by the employer.
191. CENTURY CANNING CORP., PO JR. and RONQUILLO, vs. RAMIL

DOCTRINE: The right of an employer to dismiss an employee on the ground that it


has lost its trust and confidence in him must not be exercised arbitrarily and
without just cause. Loss of trust and confidence, to be a valid cause for dismissal,
must be based on a willful breach of trust and founded on clearly established facts.
FACTS: Respondent Ramil was technical specialist of Century Canning Corp.
(Century for brevity) for 6 years, in charge of preparation of Purchase
Requisition (PR) and Capital Expenditure (CAPEX) forms for the purchase of
products and services needed by different departments of Century. Ramil
prepared a CAPEX form for external fax modems and terminal server per order
of the Technical Operations and endorsed it to the secretary of EVP Po for
signature. The next day, Ramil retrieved the form signed by Po and submitted it

to the Purchasing Officer but the purchase was put on hold as the latter
suspected that Pos signature was forged and that some details in the CAPEX
form was left blank. But since the requested equipment were badly needed,
Ramil was ordered to make another CAPEX form which was transmitted to the
Purchasing Dept. Ramil was later subjected to investigation by the company on
suspicion that he committed forgery, suspended, and later on was terminated for
loss of trust and confidence. Ramil filed a complaint against Century, Po Jr., &
Ronquillo for illegal dismissal, non-payment of overtime pay, separation pay,
moral and exemplary damages and attorneys fees in the NLRC.
The Labor Arbiter dismissed Ramils complaint for lack of merit.

Aggrieved, Ramil appealed to the NLRC which set aside the ruling of the Labor
Arbiter and declared Ramils dismissal to be illegal holding that petitioner failed
to show clear & convincing evidence that Ramil was responsible for the forgery
of Pos signature in the CAPEX form and directed petitioner to reinstate Ramil
with full backwages and seniority rights and privileges. However, on petitioners
motion for reconsideration, the NLRC reversed itself and rendered a new
decision, this time upholding the Labor Arbiters dismissal of the complaint.
Frustrated, Ramil filed a petition for certiorari with the CA.
The CA rendered judgment in favor of Ramil reinstating the earlier decision of
the NLRC and ordered petitioners to reinstate Ramil without loss of seniority
rights and privileges, pay full backwages from the time of his termination up to
the finality of its decision. The CA remanded the case to the Labor Arbiter for the
computation of backwages of Ramil. Petitioners motion for reconsideration was
denied.
ISSUES: Whether or not respondent can be validly terminated by his employer
for loss of trust and confidence?

HELD: NO. While the Court had previously held that employers are allowed a
wider latitude of discretion in terminating employees performing functions that
require the employers full trust and confidence and the mere existence of basis
for believing that the employee has breached the trust of the employer is
sufficient (Atlas Fertilizer Corp. vs NLRC, G.R. No. 120030, 1997), this does not

3E Andaya Ching Espiritu Hefti Galvez Gammad Lainez Lui Madamba Nagera Narvasa
Ong Palangdao Rosales Sanchez Santos Satrain Tabo (2014-2015)

Labor Relations Case Digest - Atty. Joyrich Golangco

mean that the said basis may be arbitrary and unfounded. The right of an
employer to dismiss an employee on the ground that it has lost its trust and
confidence in him must not be exercised arbitrarily and without just cause
(Pepsi-Cola Products Phils., Inc. v. NLRC, 374 Phil. 196, 205 (1999). Loss of trust
and confidence, to be a valid cause for dismissal, must be based on a willful
breach of trust and founded on clearly established facts. Labor Code, Article 282
of the Labor Code provides: Termination by employer. - An employer may
terminate an employment for any of the following causes: xxx c) Fraud or willful
breach by the employee of the trust reposed in him by his employer or his duly
authorized representative. Petitioner based Ramil's dismissal on its
unsubstantiated suspicions and conclusion that since Ramil was the custodian
and in charge of preparing the CAPEX form, he had the motive to commit the
forgery. But if Ramil retrieved the form the following day with the signature of
Po, it can be correctly inferred that he is not the forger because had the CAPEX
form been returned to him without Po's signature, any officer of Century could
have readily noticed the lack of signature, and could have easily attested that the
form was unsigned when it was released to Ramil. Since the equipment would be
for the use of Century, he would not be benefited by the purchase of the
equipment. The Court however found that it would be best to award separation
pay instead of reinstatement, in view of the strained relations between Century
and Ramil because the latter was dismissed due to loss of trust and confidence
and it would be impractical to reinstate an employee whom the employer does
not trust, and whose task is to handle and prepare delicate documents. The Court
further HELD: Under the doctrine of strained relations, the payment of
separation pay has been considered an acceptable alternative to reinstatement
when the latter option is no longer desirable or viable. On the one hand, such
payment liberates the employee from what could be a highly oppressive work
environment; on the other hand, the payment releases the employer from the
grossly unpalatable obligation of maintaining in its employ a worker it could no
longer trust.
The Court denied the petition and affirmed with modification the Decision and
Resolution of the CA in that the order of reinstatement was deleted and in lieu
thereof, Century was directed to pay Ramil separation pay. The case was
remanded to the Labor Arbiter to compute Ramils full backwages, inclusive of
allowances and other benefits or their monetary equivalent, computed from the

date of his dismissal up to the finality of the decision, and separation pay in lieu
of reinstatement equivalent to one month salary for every year of service
computed from the time of his engagement by Century up to the finality of this
Courts decision.
192. Tongko vs The Manufacturers Life Insurance Co. (MANULIFE)
DOCTRINE: When there is no showing of a clear, valid and legal cause for the
termination of employment, the law considers the matter a case of illegal
dismissal and the burden is on the employer to prove that the termination was
for a valid or authorized cause. This burden of proof appropriately lies on the
shoulders of the employer and not on the employee because a worker's job has
some of the characteristics of property rights and is therefore within the
constitutional mantle of protection.

FACTS: Greg Tongko initially started his professional relationship with


Manulife as a mere agent manifested by the Career Agents Agreement he
executed with Manulife. The agreement indicates that no Er-Ee relationship
exists between the parties and that the company may terminate this agreement.
Luck and fortune were both on the side of Tongko as became a Branch Manager
of the Northern Branch and his yearly gross earnings were as high as Php8M.

In Nov. 2001, De Dios, then the Pres. and CEO of Manulife, sent a letter
to Tongko. In essence, the letter talks about the comments regarding the poor
performance of the Northern Branch, the failure to meet the objectives of the
company to hire more agents to bolster the manpower development and the
latter part of the letter simply ordered Tongko to meet these objectives and
ordered that an Assistant would now be helping Tongko in his workload. The
letter also talked about the comments allegedly uttered by Tongko which can
be detrimental to the company, some of which are as follows: that Tongko said
that some Managers were unhappy with their earnings AND that these managers
wanted to just go back to being agents where they can earn more.
However, in another letter sent a month after the first one, De Dios
issued a Notice of Termination of Tongkos Agency Agreement on the ground the
grounds of gross and habitual neglect of duties, inefficiency, as well as willful
disobedience of the lawful orders of Manulifethat despite several meetings

3E Andaya Ching Espiritu Hefti Galvez Gammad Lainez Lui Madamba Nagera Narvasa
Ong Palangdao Rosales Sanchez Santos Satrain Tabo (2014-2015)

Labor Relations Case Digest - Atty. Joyrich Golangco

between Tongko and Manulife officials. Tongko filed a complaint for illegal
dismissal alleging that he is a regular employee and the Control Test is present in
this case. Manulife posits that no Ee-Er relationship exists and even assuming
arguendo that there is, there is a willful disobedience on the part of Tongko that
warrants his dismissal.
LA: No Ee-Er relationship.

NLRC: Reversed LA. There is Ee-Er relationship. Evidence of control test can be
found in the records: different codes of conducts such as Agent Code of Conduct,
Manulife Financial Code of Conduct, Manulife Financial Code of Conduct
Agreement, which serve as the foundations of the power of control wielded by
Tongko over complainant that is further manifested in the various administrative
task he is required to perform. These codes of conduct corroborate and reinforce
the display of respondents power of control in the November Letter. Tongko was
illegally dismissed. In lieu of reinstatement, Manulife is ordered to pay Tongko
separation pay and his backwages.
CA: Manulife did not exercise control over Tongko. No Ee-Er relationship.
ISSUES:

1.
2.

Is Tongko a regular employee? YUP


Did Tongko willfully disobey the order/letter? NOPE

HELD:
1.
SC used "control test" which constitutes the most important index of the
existence of the employer-employee relationship that is, whether the employer
controls or has reserved the right to control the employee not only as to the
result of the work to be done but also as to the means and methods by which the
same is to be accomplished. Stated otherwise, an employer-employee
relationship exists where the person for whom the services are performed
reserves the right to control not only the end to be achieved but also the means
to be used in reaching such end.

i.e. Code of Conduct, November Letter, Agent Agreement which requires


Tongko to render EXCLUSIVE service to Manulife, and lastly being integrated into
the management structure Manulife exercised control of his actions.

2.
Tongko was illegally dismissed. NO WILLFULL DISOBEDIENCE.
Manulife's petition that it failed to cite a single iota of evidence to support its
claims. Manulife did not even point out which order or rule that Tongko
disobeyed. More importantly, Manulife did not point out the specific acts that
Tongko was guilty of that would constitute gross and habitual neglect of duty or
disobedience. Manulife merely cited Tongko's alleged "laggard performance,"
without substantiating such claim, and equated the same to disobedience and
neglect of duty.
The law mandates that the burden of proving the validity of the
termination of employment rests with the employer. Failure to discharge this
evidentiary burden would necessarily mean that the dismissal was not justified,
and, therefore, illegal. Unsubstantiated suspicions, accusations and conclusions
of employers do not provide for legal justification for dismissing employees. In
case of doubt, such cases should be resolved in favor of labor, pursuant to the
social justice policy of our labor laws and Constitution. To repeat, mere
conjectures cannot work to deprive employees of their means of livelihood. Thus,
it must be concluded that Tongko was illegally dismissed. Check Doctrine.
193. School of Holy Spirit of QC vs. Corazon Taguiam

DOCTRINE: Under Article 282 of the Labor Code, gross and habitual neglect of
duties is a valid ground for an employer to terminate an employee. Gross
negligence implies a want or absence of or a failure to exercise slight care or
diligence, or the entire absence of care. It evinces a thoughtless disregard of
consequences without exerting any effort to avoid them. Habitual neglect implies
repeated failure to perform ones duties for a period of time, depending upon the
circumstances. Notably, respondents negligence, although gross, was not
habitual. In view of the considerable resultant damage, however, we are in
agreement that the cause is sufficient to dismiss respondent. As a result of gross
negligence in the present case, petitioners lost its trust and confidence in
respondent.

3E Andaya Ching Espiritu Hefti Galvez Gammad Lainez Lui Madamba Nagera Narvasa
Ong Palangdao Rosales Sanchez Santos Satrain Tabo (2014-2015)

Labor Relations Case Digest - Atty. Joyrich Golangco

FACTS:Respondent Corazon Taguiam was the class adviser of a Grade 5 class of


petitioner school. After obtaining permission from the principal, they were
allowed to use the school swimming pool for their year-end activity. With this,
respondent Taguiam distributed the parents/guardians permit forms to the
students. The permit form of student Chiara Mae was unsigned. But because the
mother personally brought her to the school with her packed lunch and
swimsuit, Taguiam concluded that the mother allowed her to join. Before the
activity started, respondent warned the pupils who did not know how to swim to
avoid the deeper area. However, while the pupils were swimming, two of them
sneaked out. Respondent went after them to verify where they were going.
Unfortunately, while respondent was away, Chiara Mae drowned. When
respondent returned, the maintenance man was already administering
cardiopulmonary resuscitation on Chiara Mae. She was still alive when
respondent rushed her to the General Malvar Hospital where she was
pronounced dead on arrival. The petitioner school conducted a clarificatory
hearing to which respondent attended and submitted her Affidavit of
Explanation. A month later, petitioner school dismissed respondent on the
ground of gross negligence resulting to loss of trust and confidence. Respondent
in turn filed a complaint against the school for illegal dismissal, with a prayer for
reinstatement with full backwages and other money claims, damages and
attorneys fees.
LA: Declared that respondent was validly terminated for gross neglect of duty
NLRC: Affirmed the dismissal of the complaint.
CA: Ruled in favor of the respondent.

ISSUES: Whether or not respondents dismissal on the ground of gross


negligence resulting to loss of trust and confidence was valid

HELD: Yes. Under Article 282of the Labor Code, gross and habitual neglect of
duties is a valid ground for an Employer to terminate an employee. Gross
negligence implies a want or absence of or a failure to Exercise slight care or
diligence, or the entire absence of care. It evinces a thoughtless disregard of
consequences without exerting any effort to avoid them. Habitual neglect implies
repeated failure to perform ones duties for a period of time, depending upon the
circumstances. The SC concluded that respondent had been grossly negligent.
First, it is undisputed that Chiara Maes permit form was unsigned. Yet,

respondent allowed her to join the activity because she assumed that Chiara
Maes mother has allowed her to join it by personally bringing her to the school
with her packed lunch and swimsuit. Second, it was respondents responsibility
as Class Adviser to supervise her class in all activities sanctioned by the school.
Thus, she should have coordinated with the school to ensure that proper
safeguards, such as adequate first aid and sufficient adult personnel, were
present during their activity. She should have been mindful of the fact that with
the number of pupils involved, it would be impossible for her by herself alone to
keep an eye on each one of them.
Notably, respondents negligence, although gross, was not habitual. In view of the
considerable resultant damage, however, the SC agreed that the cause is
sufficient to dismiss respondent. Indeed, the sufficiency of the evidence as well as
the resultant damage to the employer should be considered in the dismissal of
the employee. In this case, the damage went as far as claiming the life of a child.
As a result of gross negligence in the present case, petitioners lost its trust and
confidence in respondent. Loss of trust and confidence to be a valid ground for
dismissal must be based on a willful breach of trust and founded on clearly
established facts. s a teacher who stands in loco parentis to her pupils,
respondent should have made sure that the children were protected from all
harm while in her company Respondent should have known that leaving the
pupils in the swimming pool area all by themselves may result in an accident. A
simple reminder not to go to the deepest part of the pool was insufficient to cast
away all the serious dangers that the situation presented to the children,
especially when respondent knew that Chiara Mae cannot swim. Dismally,
respondent created an unsafe situation which exposed the lives of all the pupils
concerned to real danger. This is a clear violation not only of the trust and
confidence reposed on her by the parents of the pupils but of the school itself.
194. JOHN HANCOCK LIFE INSURANCE CORPORATION and/or MICHAEL
PLAXTON, petitioners, vs. JOANNA CANTRE DAVIS, respondent.

DOCTRINE: For an employee to be validly dismissed for a cause analogous to


those enumerated in Article 282, the cause must involve a voluntary and/or
willful act or omission of the employee. Theft committed by an employee against

3E Andaya Ching Espiritu Hefti Galvez Gammad Lainez Lui Madamba Nagera Narvasa
Ong Palangdao Rosales Sanchez Santos Satrain Tabo (2014-2015)

Labor Relations Case Digest - Atty. Joyrich Golangco

a person other than his employer, if proven by substantial evidence, is a cause


analogous to serious misconduct.
FACTS: Respondent Joanna Cantre Davis was agency administration officer of
petitioner John Hancock Life Insurance Corporation.

On October 18, 2000, Patricia Yuseco, petitioner's corporate affairs manager,


discovered that her wallet was missing. She immediately reported the loss of her
credit cards to AIG and BPI Express. To her surprise, she was informed that
"Patricia Yuseco" had just made substantial purchases using her credit cards in
various stores in the City of Manila. She was also told that a proposed transaction
in Abenson's-Robinsons Place was disapproved because "she" gave the wrong
information upon verification.
Because loss of personal property among its employees had become rampant in
its office, petitioner sought the assistance of the National Bureau of Investigation
(NBI). The NBI, in the course of its investigation, obtained a security video from
Abenson's showing the person who used Yuseco's credit cards. Yuseco and other
witnesses positively identified the person in the video as respondent.
Consequently, the NBI and Yuseco filed a complaint for qualified theft against
respondent in the office of the Manila city prosecutor. But because the affidavits
presented by the NBI (identifying respondent as the culprit) were not properly
verified, the city prosecutor dismissed the complaint due to insufficiency of
evidence.
Meanwhile, petitioner placed respondent under preventive suspension and
instructed her to cooperate with its ongoing investigation. Instead of doing so,
however, respondent filed a complaint for illegal dismissal alleging that
petitioner terminated her employment without cause.
LA: Dismissal valid, respondent committed serious misconduct
NLRC: Affirmed LA.

CA: Granted petition for certiorari. The labor arbiter and NLRC merely adopted
the findings of the NBI regarding respondent's culpability. Because the affidavits
of the witnesses were not verified, they did not constitute substantial evidence.

ISSUES: Whether petitioner substantially proved the presence of valid cause for
respondent's termination.
HELD: YES, In this case, petitioner dismissed respondent based on the NBI's
finding that the latter stole and used Yuseco's credit cards. But since the theft
was not committed against petitioner itself but against one of its employees,
respondent's misconduct was not work-related and therefore, she could not be
dismissed for serious misconduct.

Nonetheless, Article 282(e) of the Labor Code talks of other analogous causes or
those which are susceptible of comparison to another in general or in specific
detail. For an employee to be validly dismissed for a cause analogous to those
enumerated in Article 282, the cause must involve a voluntary and/or willful act
or omission of the employee.
A cause analogous to serious misconduct is a voluntary and/or willful act or
omission attesting to an employee's moral depravity. Theft committed by an
employee against a person other than his employer, if proven by substantial
evidence, is a cause analogous to serious misconduct.

The labor arbiter and the NLRC relied not only on the affidavits of the NBI's
witnesses but also on that of respondent. They likewise considered petitioner's
own investigative findings. Clearly, they did not merely adopt the findings of the
NBI but independently assessed evidence presented by the parties. Their
conclusion (that there was valid cause for respondent's separation from
employment) was therefore supported by substantial evidence.
195. SPI Technologies vs. Mapua

DOCTRINE: The Court remains steadfast on its stand that the determination of
the continuing necessity of a particular officer or position in a business
corporation is a management prerogative, and the courts will not interfere
unless arbitrary or malicious action on the part of management is shown. Indeed,
an employer has no legal obligation to keep more employees than are necessary
for the operation of its business. In the instant case however, we find our
intrusion indispensable, to look into matters which we would otherwise consider
as an exercise of management prerogative. "Management prerogative" are not

3E Andaya Ching Espiritu Hefti Galvez Gammad Lainez Lui Madamba Nagera Narvasa
Ong Palangdao Rosales Sanchez Santos Satrain Tabo (2014-2015)

Labor Relations Case Digest - Atty. Joyrich Golangco

magic words uttered by an employer to bring him to a realm where our labor
laws cannot reach.
FACTS: Victoria K. Mapua (Mapua) alleged that she was hired in 2003 by SPI
Technologies, Inc. (SPI) and was the Corporate Developments
Research/Business Intelligence Unit Head and Manager of the company.
Subsequently in August 2006, the then Vice President and Corporate
Development Head, Peter Maquera (Maquera) hired Elizabeth Nolan (Nolan) as
Mapuas supervisor. Sometime in October 2006, the hard disk on Mapuas laptop
crashed, causing her to lose files and data. Mapua informed Nolan and her
colleagues that she was working on recovering the lost data and asked for their
patience for any possible delay on her part in meeting deadlines.
On November 13, 2006, Mapua retrieved the lost data with the assistance of
National Bureau of Investigation Anti-Fraud and Computer Crimes Division. Yet,
Nolan informed Mapua that she was realigning Mapuas position to become a
subordinate of co-manager Sameer Raina (Raina) due to her missing a work
deadline. Nolan also disclosed that Mapuas colleagues were "demotivated" [sic]
because she was "taking things easy while they were working very hard," and
that she was "frequently absent, under timing, and coming in late every time
[Maquera] goes on leave or on vacation." Mapua noticed that her colleagues
began to ostracize and avoid her. Nolan and Raina started giving out majority of
her research work and other duties under Healthcare and Legal Division to the
rank-and-file staff. Mapua lost about 95% of her work projects and job
responsibilities. Mapua consulted these work problems with SPIs Human
Resource Director, Lea Villanueva (Villanueva), and asked if she can be
transferred to another department within SPI.
On February 28, 2007, Mapua allegedly saw the new table of organization of the
Corporate Development Division which would be renamed as the Marketing
Division. The new structure showed that Mapuas level will be again downgraded
because a new manager will be hired and positioned between her rank and
Rainas. On March 21, 2007, Raina informed Mapua over the phone that her
position was considered redundant and that she is terminated from
employment effective immediately. Villanueva notified Mapua that she should
cease reporting for work the next day. Her laptop computer and company mobile
phone were taken right away and her office phone ceased to function. Mapua

filed with the Labor Arbiter a complaint for illegal dismissal, claiming
reinstatement or if deemed impossible, for separation pay. Afterwards, she went
to a meeting with SPI, where she was given a second termination letter,14 the
contents of which were similar to the first one. Mapua received through mail, a
third Notice of Termination dated March 21, 2007 but the date of effectivity of
the termination was changed from March 21 to April 21, 2007. It further stated
that her separation pay will be released on May 20, 2007 and a notation was
inscribed, "refused to sign and acknowledge" with unintelligible signatures of
witnesses. On May 13, 2007, a recruitment advertisement17 of SPI was published
in the Philippine Daily Inquirer (Inquirer advertisement, for brevity). It listed all
vacancies in SPI, including a position for Marketing Communications Manager
under Corporate Support the same group where Mapua previously belonged.

Mapua suspected that this advertisement was for SPI because the writing style
used was similar to Rainas. She also claimed that SPI is the only BPO office in
Paraaque City at that time. Thereafter, she applied for the position under the
pseudonym of "Jeanne Tesoro". On the day of her interview with Prime
Manpowers consultant, Ms. Portia Dimatulac (Dimatulac), the latter allegedly
revealed to Mapua that SPI contracted Prime Manpowers services to search for
applicants for the Corporate Development Manager position. Because of these
developments, Mapua was convinced that her former position is not redundant.
According to her, she underwent psychiatric counseling and incurred medical
expenses as a result of emotional anguish, sleepless nights, humiliation and
shame from being jobless. She also averred that the manner of her dismissal was
unprofessional and incongruous with her rank and stature as a manager as other
employees have witnessed how she was forced to vacate the premises on the
same day of her termination. SPI denied contracting the services of Prime
Manpower for the hiring of a Corporate Development Manager and emphasized
that Prime Manpower did not even state the name of its client in the Jobstreet
website.
LA: Declared Mapuas termination as illegal, the redundancy of her position
being in want of factual basis.
NLRC: Reversed and set aside LAs decision. the NLRC held that "[t]he
determination of whether [Mapuas] position as Corporate Development

3E Andaya Ching Espiritu Hefti Galvez Gammad Lainez Lui Madamba Nagera Narvasa
Ong Palangdao Rosales Sanchez Santos Satrain Tabo (2014-2015)

Labor Relations Case Digest - Atty. Joyrich Golangco

Manager is redundant is not for her to decide. It essentially and necessarily lies
within the sound business management." As early as August 28, 2006, Ernest Cu,
SPIs Chief Executive Officer, announced the corporate changes in the company.
A month earlier, the officers held their Senior Management Strategic Planning
Session with the theme, "Transformation" or re-invention of SPI purposely to
create an organizational structure that is streamlined, clear and efficient. In fact,
Nolan and Raina, Mapuas superiors were actually doing her functions with the
assistance of the pool of analysts, as attested to by Villanueva.

was given to her. To correct the oversight, a copy of the original letter was sent to
her through mail.

HELD: Yes. Please see Article 283 LC. Expounding on the above requirements of
written notice and separation pay, this Court in Asian Alcohol Corporation v.
NLRC pronounced that for a valid implementation of a redundancy program, the
employer must comply with the following requisites: (1) written notice served
on both the employee and the DOLE at least one month prior to the intended
date of termination; (2) payment of separation pay equivalent to at least one
month pay or at least one month pay for every year of service, whichever is
higher; (3) good faith in abolishing the redundant position; and (4) fair and
reasonable criteria in ascertaining what positions are to be declared redundant.

Also, crucial to the determination of the effective date of termination was that
Mapua was very specific as regards what happened immediately after: "Ms.
Villanueva had Ms. Mapuas assigned laptop computer and cellphone
immediately taken by Human Resources supervisor, Ms. Dhang Rondael. Within
about an hour, Ms. Mapuas landline phone ceased to function after Ms.
Villanuevas and Mr. Rainas announcement." Her company I.D. was taken away
from her that very same day.

CA: Declared illegal Mapuas termination.


ISSUES: WON Mapuas Termination was illegal

Anent the first requirement which is written notice served on both the employee
and the DOLE at least one month prior to the intended date of termination, SPI
had discharged the burden of proving that it submitted a notice to the DOLE on
March 21, 2007, stating therein that the effective date of termination is on April
21, 2007. It is, however, quite peculiar that two kinds of notices were served to
Mapua. One termination letter stated that its date of effectivity is on the same
day, March 21, 2007. The other termination letter sent through mail to Mapuas
residence stated that the effective date of her termination is on April 21, 2007.

Explaining the discrepancy, SPI alleged that the company served a notice to
Mapua on March 21, 2007, which stated that the effective date of termination is
on April 21, 2007. However she refused to acknowledge or accept the letter.
Later on, Mapua requested for a copy of the said letter but due to inadvertence
and oversight, a draft of the termination letter bearing a wrong effectivity date

Our question is, after Mapua initially refused to accept the letter, why did SPI
make a new letter instead of just giving her the first one which the Court notes
was already signed and witnessed by other employees? Curiously, there was
neither allegation nor proof that the original letter was misplaced or lost which
would necessitate the drafting of a new one. SPI did not even explain in the
second letter that the same was being sent in lieu of the one given to her. Hence,
SPI must shoulder the consequence of causing the confusion brought by the
variations of termination letters given to Mapua.

To counter these statements, SPI merely stated that before the effective date of
Mapuas termination on April 21, 2007, she no longer reported for work. To this
Court, this is insufficient rebuttal to the precise narrative of Mapua.

On the issue of the validity of the redundancy program, the memorandum made
no mention that the position of the Corporate Development Manager or any
other position would be abolished or deemed redundant. Also connected with
the evidence negating redundancy was SPIs publication of job vacancies after
Mapua was terminated from employment. SPI maintained that the CA erred
when it considered Mapuas self-serving affidavit as regards the Prime
Manpower advertisement because the allegations therein were based on
Mapuas unfounded suspicions. Also, the failure of Mapua to present a sworn
statement of Dimatulac renders the formers statements hearsay. Even if we
disregard Mapuas affidavit as regards the Prime Manpower advertisement, SPI
admitted that it caused the Inquirer advertisement for a Marketing
Communications Manager position. Even if we disregard Mapuas affidavit as

3E Andaya Ching Espiritu Hefti Galvez Gammad Lainez Lui Madamba Nagera Narvasa
Ong Palangdao Rosales Sanchez Santos Satrain Tabo (2014-2015)

Labor Relations Case Digest - Atty. Joyrich Golangco

regards the Prime Manpower advertisement, SPI admitted that it caused the
Inquirer advertisement for a Marketing Communications Manager position.

Furthermore, on the assumption that the functions of a Marketing


Communications Manager are different from that of a Corporate Development
Manager, it was not even discussed why Mapua was not considered for the
position. While SPI had no legal duty to hire Mapua as a Marketing
Communications Manager, it could have clarified why she is not qualified for that
position. In fact, Mapua brought up the subject of transfer to Villanueva and
Raina several times prior to her termination but to no avail. There was even no
showing that Mapua could not perform the duties of a Marketing
Communications Manager.
The Court does not agree with the rationalization of the NLRC that "[i]f it were
true that her position was not redundant and indispensable, then the company
must have already hired a new one to replace her in order not to jeopardize its
business operations. The fact that there is none only proves that her position was
not necessary and therefore superfluous."

What the above reasoning of the NLRC failed to perceive is that "[o]f primordial
consideration is not the nomenclature or title given to the employee, but the
nature of his functions." "It is not the job title but the actual work that the
employee performs." Also, change in the job title is not synonymous to a change
in the functions. A position cannot be abolished by a mere change of job title. In
cases of redundancy, the management should adduce evidence and prove that a
position which was created in place of a previous one should pertain to functions
which are dissimilar and incongruous to the abolished office.
196. Arabit vs Jardine Pacific

FACTS: Petitioners were former regular employees of respondent Jardine The


petitioners were also officers and members of MB Finance Employees
Association-FFW Chapter (the Union), a legitimate labor union and the sole
exclusive bargaining agent of the employees of Jardine. On the claim
of financial losses, Jardine decided to reorganize and implement a redundancy
program among its employees. The petitioners were among those affected by the

redundancy program. Jardine thereafter hired contractual employees to


undertake the functions these employees used to perform.
The Union filed a notice of strike with NCMB, questioning the
termination of employment of the petitioners who were also union officers. The
Union alleged unfair labor practice on the part of Jardine, as well as
discrimination in the dismissal of its officers and members.
Negotiations ensued between the Union and Jardine under the auspices
of the NCMB, and both parties eventually reached an amicable settlement. In the
settlement, the petitioners accepted their redundancy pay without prejudice to
their right to question the legality of their dismissal with the NLRC. Jardine paid
the petitioners a separation package
Petitioners and the Union filed a complaint against Jardine with the
NLRC for illegal dismissal and unfair labor practice.
LA: The petitioners alleged before the LA that their dismissal was illegal and
was tainted with bad faith as their positions were not superfluous. They argued
that if their positions had really been redundant, then Jardine should have not
hired contractual workers to replace them.
Respondent admitted that it hired contractual employees to replace petitioners
in their previous posts. Jardine reasoned out that no bad faith took place since
the hiring of contractual employees was a valid exercise of
its management prerogative
LA held that the hiring of contractual employees to replace the petitioners
directly contradicts the concept of redundancy which involves the trimming
down of the workforce because a task is being carried out by too many people.
The LA explained that the companys action was a circumvention of the right of
the petitioners to security of tenure
NLRC: affirmed LAs decision.
CA :reversed LA and NLRCs decision. According to the CA, the hiring of
contractual employees is a management prerogative that Jardine has the right to
exercise. In the absence of any showing of malice or arbitrariness on the part of
Jardine in implementing its redundancy program, the courts must not interfere
with the companys exercise of a bona fide management decision
ISSUES: 1. Whether or not the petitioners were illegally dismissed?
2. Whether or not there is a valid redundancy?

3E Andaya Ching Espiritu Hefti Galvez Gammad Lainez Lui Madamba Nagera Narvasa
Ong Palangdao Rosales Sanchez Santos Satrain Tabo (2014-2015)

Labor Relations Case Digest - Atty. Joyrich Golangco

HELD:1. YES. Redundancy exists where the services of an employee are in


excess of what is reasonably demanded by the actual requirements of the
enterprise. A position is redundant where it is superfluous, and superfluity of a
position or positions may be the outcome of a number of factors, such as over
hiring of workers, decreased volume of business, or dropping of a particular
product line or service activity previously manufactured or undertaken by the
enterprise.
Retrenchment, on the other hand, is used interchangeably with the term
"lay-off." It is the termination of employment initiated by the employer through
no fault of the employees and without prejudice to the latter, resorted to by
management during periods of business recession, industrial depression, or
seasonal fluctuations, or during lulls occasioned by lack of orders, shortage of
materials, conversion of the plant for a new production program or the
introduction of new methods or more efficient machinery, or of automation.
Simply put, it is an act of the employer of dismissing employees because of losses
in the operation of a business, lack of work, and considerable reduction on the
volume of his business, a right consistently recognized and affirmed by this Court
From this perspective, it is illogical for Jardine to terminate the
petitioners employment and replace them with contractual employees. The
replacement effectively belies Jardines claim that the petitioners positions were
abolished due to superfluity. Redundancy could have been justified if the
functions of the petitioners were transferred to other existing employees of the
company.
To dismiss the petitioners and hire new contractual employees as
replacements necessarily give rise to the sound conclusion that the petitioners
services have not really become in excess of what Jardines business requires. To
replace the petitioners who were all regular employees with contractual ones
would amount to a violation of their right to security of tenure.

2. NO. We recognize that management has the prerogative to characterize an


employees services as no longer necessary or sustainable, and therefore
properly terminable.
This Court laid down the principle that the employer must use fair and
reasonable criteria in the selection of employees who will be dismissed from
employment due to redundancy. Such fair and reasonable criteria may include

the following, but are not limited to: (a) less preferred status (e.g. temporary
employee); (b) efficiency; and (c) seniority. The presence of these criteria used
by the employer shows good faith on its part and is evidence that the
implementation of redundancy was painstakingly done by the employer in order
to properly justify the termination from the service of its employees.
The records are bereft of indications that Jardine employed clear criteria
when it decided who among its employees, who held similar positions as the
petitioners, should be removed from their posts because of redundancy. Jardine
never bothered to explain how and why the petitioners were the ones dismissed.
Aside from the guidelines for the selection of employees who will be
terminated, the Court, in Asian Alcohol Corp. v. NLRC, also laid down guidelines
for redundancy to be characterized as validly undertaken by the employer. The
Court ruled:
For the implementation of a redundancy program to be valid, the
employer must comply with the following requisites: (1) written notice served
on both the employees and the Department of Labor and Employment at least
one month prior to the intended date of retrenchment; (2) payment of separation
pay equivalent to at least one month pay or at least one month pay for every year
of service, whichever is higher; (3) good faith in abolishing the redundant
positions; and (4) fair and reasonable criteria in ascertaining what positions are
to be declared redundant and accordingly abolished.
Admittedly, Jardine complied with guidelines 1 and 2 of the guidelines in
Asian Alcohol. Jardine informed the Department of Labor and Employment of the
petitioners separation from the service due to redundancy on April 30, 1999,
one month before their terminations effectivity. Also, the petitioners were given
their individual separation packages, composed of their severance pay, plus their
grossed up transportation allowance.
Guidelines 3 and 4 of Asian Alcohol, however, are different matters.
These last two guidelines are interrelated to ensure good faith in abolishing
redundant positions; the employer must clearly show that it used fair and
reasonable criteria in ascertaining what positions are to be declared redundant.
Jardine never undertook the 3rd and 4rth guidelines. Jardine was never
able to explain in any of its pleadings why the petitioners positions were
redundant. It never even attempted to discuss the attendant facts and
circumstances that led to the conclusion that the petitioners positions had

3E Andaya Ching Espiritu Hefti Galvez Gammad Lainez Lui Madamba Nagera Narvasa
Ong Palangdao Rosales Sanchez Santos Satrain Tabo (2014-2015)

Labor Relations Case Digest - Atty. Joyrich Golangco

become superfluous and unnecessary to Jardines business requirements. Thus,


we can only speculate on what actually happened.
To sum up, based on the guidelines set by the Court we find that at two
levels, Jardine failed to set the required fair and reasonable criteria in the
termination of the petitioners employment, leading to the conclusion that the
termination from the service was arbitrary and in bad faith.

197. PHILIPPINE CARPET v. TAGYAMON

Keywords: Invalid Retrenchment , Stare Decisis, Laches


DOCTRINE: When there is no proof that the retrenchment is valid, it follows that
the dismissal is illegal
FACTS

Philippine Carpet Manufacturing Corporation (PCMC) is a corporation


registered in the Philippines engaged in the business of manufacturing
wool and yarn carpets and rugs. Respondents were its regular and
permanent employees, but were affected by petitioners retrenchment
and voluntary retirement programs.
o Reason for retrenchment: slump in the market demand for
PCMCs products; The long-term effects of September 11 and
the war in the Middle East have greatly affected the viability of
PCMCs business
On March 15, 2004, Tagyamon, Luna, Badayos, Dela Cruz, and
Comandao9 received a uniformly worded Memorandum of dismissal
As to Marcos, Ilao, and Nemis, PCMC claimed that they availed of the
companys voluntary retirement program and, in fact, voluntarily
executed their respective Deeds of Release, Waiver, and Quitclaim.
Claiming that they were aggrieved by PCMCs decision to terminate their
employment, respondents filed separate complaints for illegal dismissal
against PCMC, Mr. Patricio Lim and Mr. David Lim.
Respondents primarily relied on the Supreme Courts decision in Philcea
v. Sto Tomas (Philcea case), as to the validity of the companys
retrenchment program. They further explained that PCMC did not, in

fact, suffer losses shown by its acts prior to and subsequent to their
termination. They also insisted that their acceptance of separation pay
and signing of quitclaim is not a bar to the pursuit of illegal dismissal
case.
PCMC, for its part, defended its decision to terminate the services of
respondents being a necessary management prerogative. It pointed out
that as an employer, it had no obligation to keep in its employ more
workers than are necessary for the operation of his business. Thus, there
was an authorized cause for dismissal. Petitioners also stressed that
respondents belatedly filed their complaint as they allowed almost three
years to pass making the principle of laches applicable. Considering that
respondents accepted their separation pay and voluntarily executed
deeds of release, waiver and quitclaim, PCMC invoked the principle of
estoppel on the part of respondents to question their separation from
the service.
o LA: dismissed the complaint for lack of merit
o NLRC: sustained LAs decision and added the application of the
principle of laches for respondents inaction for an
unreasonable period.
o CA: reversed LA and NLRC. Laches is not applicable because the
case was instituted prior to the expiration of the prescriptive
period set by law which is four years. It also applied the
doctrine of stare decisis. As to Ilao, Nemis and Marcos, while
acknowledging their voluntary resignation, the CA found the
same not a bar to the illegal dismissal case because they did so
on the mistaken belief that PCMC was losing money

ISSUES: Whether or not the respondents were illegally dismissed


HELD: YES

LACHES

Laches has been defined as the failure or neglect for an unreasonable and
unexplained length of time to do that which by exercising due diligence,
could or should have been done earlier, thus, giving rise to a presumption
that the party entitled to assert it either has abandoned or declined to

3E Andaya Ching Espiritu Hefti Galvez Gammad Lainez Lui Madamba Nagera Narvasa
Ong Palangdao Rosales Sanchez Santos Satrain Tabo (2014-2015)

Labor Relations Case Digest - Atty. Joyrich Golangco

assert it.
Laches is a doctrine in equity while prescription is based on law. Our
courts are basically courts of law not courts of equity. Thus, laches
cannot be invoked to resist the enforcement of an existing legal right
Thus, where the claim was filed within the [four-year] statutory period,
recovery therefore cannot be barred by laches. Courts should never
apply the doctrine of laches earlier than the expiration of time limited
for the commencement of actions at law
An action for reinstatement by reason of illegal dismissal is one based on
an injury to the complainants rights which should be brought within
four years from the time of their dismissal pursuant to Article 1146 of
the Civil Code. Respondents complaint filed almost 3 years after their
alleged illegal dismissal was still well within the prescriptive period. To
be sure, laches may be applied only upon the most convincing evidence of
deliberate inaction, for the rights of laborers are protected under the
social justice provisions of the Constitution and under the Civil Code.

STARE DECISIS; VALIDITY OF DISMISSAL

Under the doctrine of stare decisis, when a court has laid down a
principle of law as applicable to a certain state of facts, it will adhere to
that principle and apply it to all future cases in which the facts are
substantially the same, even though the parties may be different.
This case and the Philcea case involve the same period which is March to
April 2004; the issuance of Memorandum to employees informing them
of the implementation of the cost reduction program; the
implementation of the voluntary retirement program and retrenchment
program, except that this case involves different employees; the
execution of deeds of release, waiver, and quitclaim, and the acceptance
of separation pay by the affected employees.

IN PHILCEA CASE

Respondents failed to adduce clear and convincing evidence to prove the


confluence of the essential requisites for a valid retrenchment of its
employees. We believe that respondents acted in bad faith in

terminating the employment of the members of petitioner Union.


Contrary to the claim of respondents that the Corporation was
experiencing business losses, respondent in fact, amassed substantial
earnings from 1999 to 2003.
If respondent Corporation were to be believed that it had to retrench
employees due to the debilitating slump in demand for its products
resulting in severe losses, how could it justify the purchase
of P20,000,000.00 worth of machinery and equipment? There is likewise
no justification for the hiring of more than 100 new employees, more
than the number of those who were retrenched, as well as the order
authorizing full blast overtime work for six hours daily. All these are
inconsistent with the intransigent claim that respondent Corporation
was impelled to retrench its employees precisely because of low demand
for its products and other external causes.
Moreover, respondent Corporation failed to exhaust all other means to
avoid further losses without retrenching its employees, such as utilizing
the latter's respective forced vacation leaves. Respondents also failed to
use fair and reasonable criteria in implementing the retrenchment
program, and instead chose to retrench 77 of the members of petitioner
out of the dismissed 88 employees. Worse, respondent Corporation
hired new employees and even rehired the others who had been
"retrenched."
Respondent Corporation increased its net sales by more
than P8,000,000.00. Respondents failed to prove that there was a
drastic or severe decrease in the product sales or that it suffered
severe business losses within an interval of three (3) months from
January 2004 to March 9, 2004 when Diaz issued said Memorandum.
In contrast, in this case, the retrenchment effected by respondent
Corporation is invalid due to a substantive defect, non-compliance
with the substantial requirements to effect a valid retrenchment; it
necessarily follows that the termination of the employment of
petitioner Union's members on such ground is, likewise, illegal.

SC (case herein): In short, we adopt the Courts earlier findings that there
was no valid ground to terminate the employees

3E Andaya Ching Espiritu Hefti Galvez Gammad Lainez Lui Madamba Nagera Narvasa
Ong Palangdao Rosales Sanchez Santos Satrain Tabo (2014-2015)

Waivers, Releases and Quitclaims

Labor Relations Case Digest - Atty. Joyrich Golangco

As the ground for termination of employment was illegal, the


quitclaims are deemed illegal as the employees consent had been
vitiated by mistake or fraud. The law looks with disfavor upon
quitclaims and releases by employees pressured into signing by
unscrupulous employers minded to evade legal responsibilities. The
circumstances show that petitioners misrepresentation led its
employees, specifically respondents herein, to believe that the
company was suffering losses which necessitated the implementation
of the voluntary retirement and retrenchment programs, and eventually
the execution of the deeds of release, waiver and quitclaim
The Court understands that such a risk of not receiving anything
whatsoever, coupled with the probability of not immediately getting any
gainful employment or means of livelihood in the meantime, constitutes
enough pressure upon anyone who is asked to sign a release and
quitclaim in exchange of some amount of money which may be way
below what he may be entitled to based on company practice and policy
or by law

NOTE: Ito wala sa case ito pero sinearch ko na rin sa PHILCEA case baka tanungin.
REQUISITES FOR VALID RETRENCHMENT:
1.

2.
3.

that the retrenchment is reasonably necessary and likely to prevent


business losses which, if already incurred, are not merely de minimis,
but substantial, serious, actual and real, or if only expected, are
reasonably imminent as perceived objectively and in good faith by the
employer;
that the employer served written notice both to the employees and to
the Department of Labor and Employment at least one month prior to
the intended date of retrenchment;
that the employer pays the retrenched employees separation pay
equivalent to one month pay or at least - month pay for every year of
service, whichever is higher;

4.

5.

that the employer exercises its prerogative to retrench employees in


good faith for the advancement of its interest and not to defeat or
circumvent the employees' right to security of tenure;
that the employer used fair and reasonable criteria in ascertaining who
would be dismissed and who would be retained among the employees,

198. SANOH FULTON PHILS., INC. and MR. EDDIE JOSE vs. EMMANUEL
BERNARDO and SAMUEL TAGHOY
DOCTRINE: In termination cases either by retrenchment or closure, the burden
of proving that the termination of services is for a valid or authorized cause rests
upon the employer.17 Not every loss incurred or expected to be incurred by an
employer can justify retrenchment. The employer must prove, among others,
that the losses are substantial and that the retrenchment is reasonably necessary
to avert such losses.

FACTS: Sanoh is a domestic corporation engaged in the manufacture of


automotive parts and wire condensers for home appliances. Its Wire Condenser
Department employed 61 employees. Respondents belonged to this department.
Sanoh decided to phase out the Wire Condenser Department. HR informed the 17
employees, 16 of whom belonged to the Wire Condenser Department, of
retrenchment. All 17 employees are union members. A grievance conference was
held where the affected employees were informed of the following grounds for
retrenchment: 1) Lack of local market; 2) Competition from imported products;
3) Phasing out of Wire Condenser Department.3

Two conciliation conferences were held but the parties failed to reach an
amicable settlement. Two separate complaints for illegal dismissal were filed by
17 employees. Sanoh filed a petition for declaration of the partial closure of its
Wire Condenser Department and valid retrenchment of the 17 employees.
During the course of the proceedings before the Labor Arbiter, 14 of the 17
employees executed individual quitclaims. Hence, their interest in the cases was
dismissed with prejudice. Only 3 employees, Bernardo, Taghoy, and Santos
persisted.

3E Andaya Ching Espiritu Hefti Galvez Gammad Lainez Lui Madamba Nagera Narvasa
Ong Palangdao Rosales Sanchez Santos Satrain Tabo (2014-2015)

Labor Relations Case Digest - Atty. Joyrich Golangco

The complainants alleged that there was no valid cause for retrenchment and in
effecting retrenchment, there was a violation of the "first in-last out" and "last infirst out" (LIFO) policy embodied in the CBA. Sanoh asserted that retrenchment
was a valid exercise of management prerogative.
The LA5 dismissed the complaint for illegal dismissal. It ordered Respondent
company however to pay the separation pay. NLRC affirmed in toto the decision
of the LA.

The CA overturned the findings of the LA and the NLRC, and ruled that Sanoh
failed to prove the existence of substantial losses that would justify a valid
retrenchment. The CA also upheld the quitclaim executed by Santos, thus he was
deemed to have released Sanoh from his monetary claims. The company is found
guilty of illegal dismissal and is ordered to reinstate Bernardo and Taghoy with
full backwages. Where reinstatement is no longer feasible, respondent company
is ordered to pay backwages plus separation pay.
ISSUES: Whether or not the retrenchment was valid.

HELD: We are in full accord with the decision of the Court of Appeals.

To justify retrenchment, Sanoh invokes as grounds serious business losses


resulting in the closure of the Wire Condenser Department, to which
respondents belonged; and that its decision to close the Wire Condenser
Department is within its right even in the absence of business losses as long as it
is done in good faith. Sanohs two-tiered argument rests on the application of
Article 283 of the Labor Code.

Retrenchment to prevent losses and closure not due to serious business losses
are two separate authorized causes for terminating the services of an employee.
Closure of business as an authorized cause for termination of employment aims
to prevent further financial drain upon an employer who cannot pay anymore his
employees since business has already stopped. On the other hand, retrenchment
is reduction of personnel usually due to poor financial returns so as to cut down
on costs of operations in terms of salaries and wages to prevent bankruptcy of
the company.

For retrenchment, the three (3) basic requirements are: (a) proof that the
retrenchment is necessary to prevent losses or impending losses; (b) service of
written notices to the employees and to the Department of Labor and
Employment at least one (1) month prior to the intended date of retrenchment;
and (c) payment of separation pay equivalent to one (1) month pay, or at least
one-half (1/2) month pay for every year of service, whichever is higher.14 Also,
jurisprudence has set the standards for losses which may justify retrenchment:
(1) the losses incurred are substantial and not de minimis; (2) the losses are
actual or reasonably imminent; (3) the retrenchment is reasonably necessary
and is likely to be effective in preventing the expected losses; and (4) the alleged
losses, if already incurred, or the expected imminent losses sought to be
forestalled, are proven by sufficient and convincing evidence.15
On the other hand, the law authorizes termination of employment due to
business closure, regardless of the underlying reasons and motivations therefor.
However, the closure/cessation of business must be bona fide, i.e., its purpose is
to advance the interest of the employer and not to defeat or circumvent the
rights of employees under the law or a valid agreement.16

In termination cases either by retrenchment or closure, the burden of proving


that the termination of services is for a valid or authorized cause rests upon the
employer.17 Not every loss incurred or expected to be incurred by an employer
can justify retrenchment. The employer must prove, among others, that the
losses are substantial and that the retrenchment is reasonably necessary to avert
such losses. In this case, there was no valid retrenchment. Nor was there a
closure of business.

A lull caused by lack of orders or shortage of materials must be of such nature as


would severely affect the continued business operations of the employer to the
detriment of all and sundry if not properly addressed.19 Sanoh asserts that
cancelled orders of wire condensers led to the phasing out of the Wire Condenser
Department which triggered retrenchment. Sanoh presented the letters of
cancellation given by Matsushita and Sanyo as evidence of cancelled orders. The
evidence presented by Sanoh barely established the connection between the
cancelled orders and the projected business losses that may be incurred by

3E Andaya Ching Espiritu Hefti Galvez Gammad Lainez Lui Madamba Nagera Narvasa
Ong Palangdao Rosales Sanchez Santos Satrain Tabo (2014-2015)

Labor Relations Case Digest - Atty. Joyrich Golangco

Sanoh. Sanoh failed to prove that these cancelled orders would severely impact
on their production of wire condensers.

On the other hand, respondents refutations of the employers reason for


retrenchment were supported by documentary evidence. Respondents explained
that Matsushita had four (4) outstanding orders of condensers of refrigerators
and the order of Matsushita had been increased from 500 to 1600 units.With
respect to the Sanyo account, respondent assert that Sanyo had sufficient stocks
for three (3) months which explained why it did not order from Sanyo. However,
Sanyo resumed making orders.23 Respondents added that despite the
cancellation of some orders by Matsushita and Sanyo, the additional orders made
by Concepcion Industries and Uni-Magma more than compensated the losses
incurred on the cancelled orders.24
Sanoh would then argue that it did not even have to prove business losses when
it decided to close down the Wire Condenser Department because the law
recognizes the right of management to cease business operations. Sanoh made a
categorical statement that the Wire Condenser Department was totally closed.
The documentary evidence presented by respondents, however, negate Sanohs
statement. In other words, Sanoh lacked bona fides even in its assertion that
Wire Condenser Department had closed down. Respondents disclose that this
department had gone full blast in its operations, even with substantial overtime
operations immediately after their dismissal was effected. Moreover,
respondents assert that Sanoh still hired employees after the so-called
retrenchment.
As the Wire Condenser Department is still in operation and no business losses
were proven by Sanoh, the dismissal of respondents was unlawful.
WHEREFORE the petition is DENIED.

199. Andrada vs NLRC

DOCTRINE: Redundancy exists when the number of employees is in excess of


what is reasonably necessary to operate the business. The declaration of
redundant positions is a management prerogative. The determination that the

employees services are no longer necessary or sustainable and therefore


properly terminable is an exercise of business judgment by the employer. It is
however not enough for a company to merely declare that positions have
become redundant. It must produce adequate proof of such redundancy to
justify the dismissal of the affected employees.

FACTS: Petitioners Ruben Andrada, Jovencio Poblete, Filamer Alfonso, Harvey


Cayetano, Vicente Mantala, Jr., Bernaldo delos Santos, and Joven Pabustan were
hired on various dates from 1995 up to 1997 and worked as architects,
draftsmen, operators, engineers, and surveyors in the Subic Legend Resorts and
Casino, Inc. (Legend) Project Development Division on various projects.

On January 6, 1998, Legend sent notice to the DOLE of its intention to


retrench and terminate the employment of thirty-four (34) of its employees,
which include petitioners, in the Project Development Division. Legend
explained that it would be retrenching its employees on a last-in-first-out basis
on the strength of the updated status report of its Project Development Division,
as follows: (1) shelving of the condotel project until economic conditions in the
Philippines improve.
The following day, on January 7, 1998, Legend sent the 34 employees their
respective notices of retrenchment, stating the same reasons for their
retrenchment. It also offered the employees the following options, to wit:

1.
Temporary retrenchment/lay-off for a period not to exceed six months
within which we shall explore your possible reassignment to other departments or
affiliates, after six months and redeployment and/or matching are unsuccessful,
permanent retrenchment takes place and separation pay is released.
2.
Permanent retrenchment and payment of separation pay
and other benefits after the thirty (30) days notice has lapsed; or
3.
Immediate retrenchment and payment of separation pay,
benefits and one months salary in lieu of notice to allow you to look for
other employment opportunities.
Legend gave said employees a period of one week or until January 14,
1998 to choose their option, with option number 2 (permanent retrenchment) as

3E Andaya Ching Espiritu Hefti Galvez Gammad Lainez Lui Madamba Nagera Narvasa
Ong Palangdao Rosales Sanchez Santos Satrain Tabo (2014-2015)

Labor Relations Case Digest - Atty. Joyrich Golangco

the default choice in case they failed to express their preferences. After the
employees made their choices, they also expressed their reservation that their
choice should not be deemed as waiver of their rights granted under the Labor
Code or their right to question the validity of their retrenchment should their
separation benefits not be settled by January 30, 1998.
Curiously, on the same day, the Labor and Employment Center of the
Subic Bay Metropolitan Authority advertised that Legend International Resorts,
Inc. was in need of employees for positions similar to those vacated by
petitioners.
Afterwards, on February 6, 1998, Legend informed the retrenched
employees of their permanent retrenchment and/or their options. Legend paid
the retrenched employees their salaries up to February 6, 1998, separation pay,
pro-rated 13th-month pay, ex-gratia, meal allowance, unused vacation leave
credits, and tax refund. Petitioners, in turn, signed quitclaims but reserved their
right to sue Legend.

Subsequently, on March 3, 1998, 14 of the 34 retrenched employees


filed before the Regional Arbitration Branch of the National Labor Relations
Commission (NLRC) in San Fernando City, Pampanga, a complaint for illegal
dismissal because Legend, after giving retrenchment as the reason for their
termination, created new positions similar to those they had just
vacated. Legend, on the other hand, invoked management prerogative when it
terminated the retrenched employees; and said that complainants voluntarily
signed quitclaims so that they were already barred from suing Legend.
LA: Illegal dismissal. The documents submitted by Legend to justify the
retrenchment of its personnel were insufficient because the documents failed to
show that Legend was suffering from actual losses or that there was redundancy
in the positions occupied by petitioners. The LA also attributed bad faith on the
part of Legend when it advertised openings for positions similar to those
occupied by the retrenched employees at the same time the retrenchment
program was being implemented.
NLRC: No illegal dismissal.

CA: Valid dismissal, not due to retrenchment but because of redundancy. The CA

ratiocinated that Legend had validly terminated the employment of its


employees since it had proven that complainants positions were superfluous
and that there was an oversupply of employees; more than what its projects
needed.

ISSUES: W/N respondents were illegally dismissed


If not, W/N there was a valid retrenchment
HELD: 1. YES and NO. There was an illegal dismissal, because of the invalid
retrenchment. Under the Labor Code, retrenchment and redundancy are
authorized causes for separation from service. However, to protect labor,
dismissals due to retrenchment or redundancy are subject to strict requirements
under Article 283 of the Labor Code. (Pls refer to Art 283, para di na humaba.)
Retrenchment is an exercise of managements prerogative to terminate the
employment of its employees en masse, to either minimize or prevent losses, or
when the company is about to close or cease operations for causes not due to
business losses. In the present case, Legend glaringly failed to show its financial
condition prior to and at the time it enforced its retrenchment program. It failed
to submit audited financial statements regarding its alleged financial
losses. Though Legend complied with the notice requirements and the payment
of separation benefits to the retrenched employees, its failure to establish the
basis for the retrenchment of its employees constrained the Court to declare the
retrenchment illegal.

Contrary to what the CA said, evidence presented by Legend are


insufficient to justify the redundancy. The required quantum of proof, which is
substantial evidence, has not been met. Legend did not even bother to illustrate
or explain in detail how and why it considered petitioners positions superfluous
or unnecessary.
SC discussion on redundancy and retrenchment:

Retrenchment and redundancy are two different concepts; they are not
synonymous and therefore should not be used interchangeably.

3E Andaya Ching Espiritu Hefti Galvez Gammad Lainez Lui Madamba Nagera Narvasa
Ong Palangdao Rosales Sanchez Santos Satrain Tabo (2014-2015)

Labor Relations Case Digest - Atty. Joyrich Golangco

Redundancy exists where the services of an employee are in


excess of what is reasonably demanded by the actual requirements of
the enterprise. A position is redundant where it is superfluous, and
superfluity of a position or positions may be the outcome of a number
of factors, such as over hiring of workers, decreased volume of
business, or dropping of a particular product line or service activity
previously manufactured or undertaken by the enterprise.

Retrenchment, on the other hand, is used interchangeably


with the term lay-off. It is the termination of employment initiated
by the employer through no fault of the employees and without
prejudice to the latter, resorted to by management during periods of
business recession, industrial depression, or seasonal fluctuations, or
during lulls occasioned by lack of orders, shortage of materials,
conversion of the plant for a new production program or the
introduction of new methods or more efficient machinery, or of
automation. Simply put, it is an act of the employer of dismissing
employees because of losses in the operation of a business, lack of
work, and considerable reduction on the volume of his business, a
right consistently recognized and affirmed by this Court.
200. ASUFRIN vs SAN MIGUEL

DOCTRINE: It is not enough for a company to merely declare that it has become
overmanned. It must produce adequate proof that such is the actual situation to
justify the dismissal of the affected employees for redundancy.
In selecting employees to be dismissed, a fair and reasonable criteria must be
used, such as but not limited to (a) less preferred status, e.g. temporary
employee; (b) efficiency; and (c) seniority In the case at bar,

FACTS: Coca Cola Plant, then a department of respondent San Miguel Beer
Corporation (SMC), hired petitioner as a utility/miscellaneous worker in
February 1972. On November 1, 1973, he became a regular employee paid on
daily basis as a Forklift Operator. On November 16, 1981, he became a monthly
paid employee promoted as Stock Clerk.

Sometime in 1984, the sales office and operations at the Sum-ag, Bacolod
City Sales Office were reorganized. Several positions were abolished including
petitioners position as Stock Clerk. After reviewing petitioners qualifications,
he was designated warehouse checker at the Sum-ag Sales Office.
On April 1, 1996, respondent SMC implemented a new marketing system
known as the pre-selling scheme at the Sum-ag Beer Sales Office. As a
consequence, all positions of route sales and warehouse personnel were declared
redundant. Respondent notified the DOLE Director of Region VI that 22
personnel of the Sales Department of the Negros Operations Center would be
retired effective March 31, 1995.

Respondent SMC thereafter wrote a letter to petitioner informing him that,


owing to the implementation of the pre-selling operations scheme, all positions
of route and warehouse personnel will be declared redundant and the Sum-ag
Sales Office will be closed effective April 30, 1996. Thus, from April 1,
1996 to May 15, 1996, petitioner reported to respondents Personnel
Department at the Sta. Fe Brewery, pursuant to a previous directive.

Thereafter, the employees of Sum-ag sales force were informed that they
can avail of respondents early retirement package pursuant to the retrenchment
program, while those who will not avail of early retirement would be redeployed
or absorbed at the Brewery or other sales offices. Petitioner opted to remain and
manifested to Acting Personnel Manager Salvador Abadesco his willingness to be
assigned to any job, considering that he had three children in college.
Petitioner was surprised when he was informed by the Acting Personnel
Manager that his name was included in the list of employees who availed of the
early retirement package. Petitioners request that he be given an assignment in
the company was ignored by the Acting Personnel Manager.
Petitioner thus filed a complaint for illegal dismissal.

ISSUES: whether or not the dismissal of petitioner is based on a just and


authorized cause.

HELD: NO. In the case at bar, petitioner was dismissed on the ground of
redundancy, one of the authorized causes for dismissal.

3E Andaya Ching Espiritu Hefti Galvez Gammad Lainez Lui Madamba Nagera Narvasa
Ong Palangdao Rosales Sanchez Santos Satrain Tabo (2014-2015)

Labor Relations Case Digest - Atty. Joyrich Golangco

The determination that employees services are no longer necessary or


sustainable and, therefore, properly terminable is an exercise of business
judgment of the employer. The wisdom or soundness of this judgment is not
subject to discretionary review of the Labor Arbiter and the NLRC, provided
there is no violation of law and no showing that it was prompted by an arbitrary
or malicious act. In other words, it is not enough for a company to merely
declare that it has become overmanned. It must produce adequate proof that
such is the actual situation to justify the dismissal of the affected employees for
redundancy.
Persuasive as the explanation proffered by respondent may be to justify the
dismissal of petitioner, a number of disturbing circumstances, however, leave us
unconvinced.

First, of the 23 SMC employees assigned at the Sum-ag Sales


Office/Warehouse, 9 accepted the offer of SMC to avail of the early retirement
whose separation benefits was computed at 250% of their regular pay. The rest,
including petitioner, did not accept the offer. Out of the remaining fourteen
14, only petitioner clearly manifested, through several letters,[ his desire to be
redeployed to the Sta. Fe Brewery or any sales office and for any position not
necessarily limited to that of a warehouse checker. In short, he was even willing
to accept a demotion just to continue his employment. Meanwhile, other
employees who did not even write a letter to SMC were redeployed to the Sta. Fe
Brewery or absorbed by other offices/outlets outside Bacolod City.
Second, petitioner was in the payroll of the Sta. Fe Brewery and assigned to
the Materials Section, Logistics Department, although he was actually posted at
the Sum-ag Warehouse. Thus, even assuming that his position in the Sum-ag
Warehouse became redundant, he should have been returned to the Sta. Fe
Brewery where he was actually assigned and where there were vacant positions
to accommodate him.

Third, it appears that despite respondents allegation that it ceased and


closed down its warehousing operations at the Sum-ag Sales Office, actually it is
still used for warehousing activities and as a transit point where buyers and
dealers get their stocks. Indeed, the Sum-ag Office is strategically situated on the
southern part of Bacolod City making it convenient for dealers from the southern
towns of Negros Occidental to get their stocks and deposit their empty bottles in

the said warehouse, thereby decongesting the business activities at the Sta. Fe
Brewery.

Fourth, in selecting employees to be dismissed, a fair and reasonable criteria


must be used, such as but not limited to (a) less preferred status, e.g. temporary
employee; (b) efficiency; and (c) seniority. In the case at bar, no criterion
whatsoever was adopted by respondent in dismissing petitioner. Furthermore, as
correctly observed by the NLRC, respondent has not shown how the cessation of
operations of the Sum-ag Sales Office contributed to the ways and means of
improving effectiveness of the organization with the end in view of efficiency and
cutting distribution overhead and other related costs. Respondent, thus, clearly
resorted to sweeping generalization[s] in dismissing complainant. Indeed,
petitioners predicament may have something to do with an incident where he
incurred the ire of an immediate superior in the Sales Logistics Unit for exposing
certain irregularities committed by the latter.
petitioners dismissal is declared illegal, and respondent is ordered to reinstate
him to his former or equivalent position, with full backwages computed
from April 1, 1996 up to his actual reinstatement. Respondent is likewise
ordered to pay petitioner the sum equivalent to ten percent (10%) of his total
monetary award as attorneys fees.

201. VICENTE SY, TRINIDAD PAULINO, 6BS TRUCKING CORPORATION, and


SBT TRUCKING CORPORATION vs. HON. COURT OF APPEALS and JAIME
SAHOT

DOCTRINES: In order to validly terminate employment on the ground of disease,


Book VI, Rule I, Section 8 of the Omnibus Implementing Rules of the Labor Code
must be observed. (The foregoing rule requires a certification by competent
public health authority that the disease is of such nature or at such a stage that it
cannot be cured within a period of six (6) months even with proper medical
treatment.)

As this Court stated in Triple Eight integrated Services, Inc. vs. NLRC, the
requirement for a medical certificate under Article 284 of the Labor Code cannot
be dispensed with; otherwise, it would sanction the unilateral and arbitrary

3E Andaya Ching Espiritu Hefti Galvez Gammad Lainez Lui Madamba Nagera Narvasa
Ong Palangdao Rosales Sanchez Santos Satrain Tabo (2014-2015)

Labor Relations Case Digest - Atty. Joyrich Golangco

determination by the employer of the gravity or extent of the employees illness


and thus defeat the public policy in the protection of labor.

not until the year 1994, when SBT Trucking Corporation was established, and
only then did respondent Sahot become an employee of the company.

FACTS: Sometime in 1958, private respondent Jaime Sahot5 started working as a


truck helper for petitioners family-owned trucking business named Vicente Sy
Trucking. In 1965, he became a truck driver of the same family business,
renamed T. Paulino Trucking Service, later 6Bs Trucking Corporation in 1985,
and thereafter known as SBT Trucking Corporation since 1994. For 36 years,
private respondent continuously served the trucking business of petitioners.

LA: held that there was no illegal dismissal in Sahots case. Private respondent
had failed to report to work. He also held that petitioners and private respondent
were industrial partners before January 1994. He ordered petitioners to pay
"financial assistance" of P15,000 to Sahot for having served the company as a
regular employee since January 1994.

An employee who is terminated because of disease is entitled to "separation pay


equivalent to at least one month salary or to one-half month salary for every year
of service, whichever is greater.

In April 1994, Sahot then 59 years old, incurred absences as he was suffering
from various ailments, particularly the pain in his left thigh, which greatly
affected the performance of his task. He inquired about his medical and
retirement benefits with the (SSS) but discovered that his premium payments
had not been remitted by his employer.

In May 1994, he filed a week-long leave. On May 27th, he was medically


examined and treated for EOR, presleyopia, hypertensive retinopathy G II, HPM,
UTI, Osteoarthritis and heart enlargement. On said grounds, Belen Paulino of the
SBT Trucking Service management told him to file a formal request for extension
of his leave. Sahot applied for extension of his leave for the whole month of June,
1994. It was at this time when petitioners allegedly threatened to terminate his
employment should he refuse to go back to work. Subsequently, he was
dismissed from work, effective June 30, 1994.
Sahot filed a complaint for illegal dismissal with prayer for the recovery of
separation pay and attorneys fees against Vicente Sy and Trinidad Paulino-Sy,
Belen Paulino, Vicente Sy Trucking, T. Paulino Trucking Service, 6Bs Trucking
and SBT Trucking.

Petitioners contended that private respondent was not illegally dismissed as a


driver because he was in fact petitioners industrial partner. They add that it was

Petitioners further claimed that from the expiration of his leave until June 30,
private respondent never reported back to work nor did he file an extension of
his leave. Instead, he filed the complaint for illegal dismissal against the trucking
company and its owners.

NLRC: declared that private respondent was an employee, not an industrial


partner, since the start. It held that Sahot did not abandon his job but was
terminated on account of his illness, pursuant to Article 284. It ordered
petitioners to pay private respondent separation pay at the rate of P2,080 per
year for 29 years of service.
CA: held that private respondent was indeed an employee of petitioners since
1958. It also increased the amount of separation pay at the rate of P2,080 per
year for 36 years of service from 1958 to 1994.
ISSUES:
1.
2.
3.

HELD:

WON there was an employer-employee relationship


WON private respondent was validly dismissed by petitioners
WON private respondent is entitled to separation pay

1. YES. SC did not entertain petitioners contention that complainant was an


industrial partner, since a computation of the complainants age shows that he
was only 23 years old when he started working with respondent as truck helper.

3E Andaya Ching Espiritu Hefti Galvez Gammad Lainez Lui Madamba Nagera Narvasa
Ong Palangdao Rosales Sanchez Santos Satrain Tabo (2014-2015)

Labor Relations Case Digest - Atty. Joyrich Golangco

Moreover, not one of the circumstances in a contract of partnership is present in


this case. In contrast, the requisites for an Er-Ee relationship exist.)

2. NO, Article 284 of the Labor Code authorizes an employer to terminate an


employee on the ground of disease. However, in order to validly terminate
employment on this ground, Book VI, Rule I, Section 8 of the Omnibus
Implementing Rules of the Labor Code requires:

Sec. 8. Disease as a ground for dismissal- Where the employee suffers from a
disease and his continued employment is prohibited by law or prejudicial to his
health or to the health of his co-employees, the employer shall not terminate his
employment unless there is a certification by competent public health authority
that the disease is of such nature or at such a stage that it cannot be cured within
a period of six (6) months even with proper medical treatment. If the disease or
ailment can be cured within the period, the employer shall not terminate the
employee but shall ask the employee to take a leave. The employer shall reinstate
such employee to his former position immediately upon the restoration of his
normal health. (Italics supplied).
As this Court stated in Triple Eight integrated Services, Inc. vs. NLRC,31 the
requirement for a medical certificate under Article 284 of the Labor Code cannot
be dispensed with; otherwise, it would sanction the unilateral and arbitrary
determination by the employer of the gravity or extent of the employees illness
and thus defeat the public policy in the protection of labor.

In the case at bar, the employer clearly did not comply with the medical
certificate requirement before Sahots dismissal was effected. In the same case of
Sevillana vs. I.T. (International) Corp., we ruled:
Since the burden of proving the validity of the dismissal of the employee rests on
the employer, the latter should likewise bear the burden of showing that the
requisites for a valid dismissal due to a disease have been complied with. In the
absence of the required certification by a competent public health authority, this
Court has ruled against the validity of the employees dismissal. It is therefore
incumbent upon the private respondents to prove by the quantum of evidence
required by law that petitioner was not dismissed, or if dismissed, that the

dismissal was not illegal; otherwise, the dismissal would be unjustified. This
Court will not sanction a dismissal premised on mere conjectures and suspicions,
the evidence must be substantial and not arbitrary and must be founded on
clearly established facts sufficient to warrant his separation from work.32
In addition, petitioners failed to furnish private respondent of the two written
notices prior to dismissal, even only for record purposes. What management did
was to threaten the employee with dismissal, then actually implement the threat
when the occasion presented itself because of private respondents painful left
thigh. All told, both the substantive and procedural aspects of due process were
violated. Clearly, therefore, Sahots dismissal is tainted with invalidity.

3. YES. An employee who is terminated because of disease is entitled to


"separation pay equivalent to at least one month salary or to one-half month
salary for every year of service, whichever is greater xxx."34 Following the
formula set in Art. 284 of the Labor Code, his separation pay was computed by
the appellate court at P2,080 times 36 years (1958 to 1994) or P74,880. We
agree with the computation, after noting that his last monthly salary was
P4,160.00 so that one-half thereof is P2,080.00. Finding no reversible error nor
grave abuse of discretion on the part of appellate court, we are constrained to
sustain its decision. To avoid further delay in the payment due the separated
worker, whose claim was filed way back in 1994, this decision is immediately
executory. Otherwise, six percent (6%) interest per annum should be charged
thereon, for any delay, pursuant to provisions of the Civil Code.
202. Union Motor Corporation vs NLRC

DOCTRINE: We note that the company rules do not require that the notice of an
employees absence and the reasons therefor be in writing and for such notice to
be given to any specific office and/or employee of the petitioner. Hence, the
notice may be verbal; it is enough then that an officer or employee of the
petitioner, competent and responsible enough to receive such notice for and in
behalf of the petitioner, was informed of such absence and the corresponding
reason.
FACTS: Respondent Alejandro Etis was hired by petitioner as an automotive
mechanic at the service department in the latters Paco Branch. In 1994, he was

3E Andaya Ching Espiritu Hefti Galvez Gammad Lainez Lui Madamba Nagera Narvasa
Ong Palangdao Rosales Sanchez Santos Satrain Tabo (2014-2015)

Labor Relations Case Digest - Atty. Joyrich Golangco

transferred to the Caloocan City Branch, where his latest monthly salary
was P6,330.00. During his employment, he was awarded the Top Technician
for the month of May in 1995 and Technician of the Year (1995). He also became
a member of the Exclusive P40,000.00 Club and received the Model Employee
Award in the same year. One day, he called Rosita dela Cruz, company nurse, and
informed her that he had to take a sick leave as he had a painful and unbearable
toothache. The next day, he again phoned Dela Cruz and told her that he could
not report for work because he still had to consult a doctor. Due to tooth
inflammation he was referred to Dr. Pamor for a scheduled tooth extraction.
Upon instructions from the management, Dumugan, a security guard visited the
private respondent and confirmed that the latter was ill. The tooth extraction
was rescheduled because the inflammation has not yet subsided, and again Etis
was not able to report to work.The petitioner issued a Memorandum through the
manager of its Human Resources Department, terminating the services of the
respondent for having incurred more than 5 consecutive absences without
proper notification. The petitioner considered the consecutive absences of the
respondent as abandonment of office under Section 6.1.1, Article III of the
Company Rules. After the tooth extraction and recovery of Etis he reported to
work but was denied entry into the companys premises and was informed that
his employment was terminated. An arbitration was sought before the NCMB,
during its pendency, respondent wrote petitioner asking reconsideration for his
dismissal but was denied, and so was the complaint with the NCMB. That led
respondent to file an illegal dismissal before the LA which denied the complaint.
LA ruled that the respondents failure to report for work for 10 days without an
approved leave of absence was equivalent to gross neglect of duty, and that his
claim that he had been absent due to severe toothache leading to a tooth
extraction was unsubstantiated. Aggrieved by the decision respondent appealed
with the NLRC and reversed the LAs decision and ordered respondents
reinstatement. Petitioner then filed a certiorari with the CA which agreed with
the decision of the NLRC, hence this petition,
ISSUES: WON respondent Etis was legally terminated for failing to notify
petitioner of his absence

HELD: No, We agree with the rulings of the NLRC and the CA. We note that the
company rules do not require that the notice of an employees absence and the
reasons therefor be in writing and for such notice to be given to any specific

office and/or employee of the petitioner. Hence, the notice may be verbal; it is
enough then that an officer or employee of the petitioner, competent and
responsible enough to receive such notice for and in behalf of the petitioner, was
informed of such absence and the corresponding reason. The evidence on record
shows that the respondent informed the petitioner of his illness through the
company nurse. The security guard who was dispatched by the petitioner to
verify the information received by the company nurse, confirmed the
respondents illness. We find and so hold that the respondent complied with the
requisite of giving notice of his illness and the reason for his absences to the
petitioner. While the records do not reveal that the respondent filed the required
leave of absence for the period during which he suffered from a toothache, he
immediately reported for work upon recovery, armed with medical certificates
to attest to the cause of his absence. The respondent could not have anticipated
the cause of his illness, thus, to require prior approval would be unreasonable.
While it is true that the petitioner had objected to the veracity of the medical
certificates because of lack of notarization, it has been said that verification of
documents is not necessary in order that the said documents could be
considered as substantial evidence.

Dismissal is the ultimate penalty that can be meted to an employee. Thus, it


must be based on just cause and must be supported by clear and convincing
evidence. To effect a valid dismissal, the law requires not only that there be just
and valid cause for termination; it, likewise, enjoins the employer to afford the
employee the opportunity to be heard and to defend himself. To warrant removal
from service, the negligence should not merely be gross but also habitual. Gross
negligence implies a want or absence of or failure to exercise slight care or
diligence, or the entire absence of care. It evinces a thoughtless disregard of
consequences without exerting any effort to avoid them
203. Villaruel vs. Yeo Han Guan

DOCTRINE:An employee who resigns from work due to an illness is not entitled
to separation pay under article 284. A plain reading of Article 284 clearly
presupposes that it is the employer who terminates the services of the employee
found to be suffering from any disease and whose continued employment is
prohibited by law or is prejudicial to his health as well as to the health of his co-

3E Andaya Ching Espiritu Hefti Galvez Gammad Lainez Lui Madamba Nagera Narvasa
Ong Palangdao Rosales Sanchez Santos Satrain Tabo (2014-2015)

Labor Relations Case Digest - Atty. Joyrich Golangco

employees. It does not contemplate a situation where it is the employee who


severs his or her employment ties.

FACTS: On February 15, 1999, herein petitioner filed with the NLRC, National
Capital Region, Quezon City a Complaint [3] for payment of separation pay
against Yuhans Enterprises.

Subsequently, in his Amended Complaint and Position Paper [4] dated


December 6, 1999, petitioner alleged that in June 1963, he was employed as a
machine operator by Ribonette Manufacturing Company, an enterprise engaged
in the business of manufacturing and selling PVC pipes and is owned and
managed by herein respondent Yeo Han Guan. Over a period of almost twenty
(20) years, the company changed its name four times. Starting in 1993 up to the
time of the filing of petitioner's complaint in 1999, the company was operating
under the name of Yuhans Enterprises. Despite the changes in the company's
name, petitioner remained in the employ of respondent. Petitioner further
alleged that on October 5, 1998, he got sick and was confined in a hospital; on
December 12, 1998, he reported for work but was no longer permitted to go back
because of his illness; he asked that respondent allow him to continue working
but be assigned a lighter kind of work but his request was denied; instead, he
was offered a sum of P15,000.00 as his separation pay; however, the said amount
corresponds only to the period between 1993 and 1999; petitioner prayed that
he be granted separation pay computed from his first day of employment in June
1963, but respondent refused. Aside from separation pay, petitioner prayed for
the payment of service incentive leave for three years as well as attorney's fees.
On the other hand, respondent averred in his Position Paper [5] that
petitioner was hired as machine operator from March 1, 1993 until he stopped
working sometime in February 1999 on the ground that he was suffering from
illness; after his recovery, petitioner was directed to report for work, but he
never showed up. Respondent was later caught by surprise when petitioner filed
the instant case for recovery of separation pay. Respondent claimed that he
never terminated the services of petitioner and that during their mandatory
conference, he even told the latter that he could go back to work anytime but
petitioner clearly manifested that he was no longer interested in returning to
work and instead asked for separation pay.

LA ruled in favor of complainant and awarded separation pay in the amount of


91,445 pesos and service incentive leave equivalent to 15 days salary in the
amount of 3015 pesos. NLRC affirmed the decision and dismissed the appeal of
respondent. CA deleted the award of separation pay and affirmed the award of
service incentive leave pay.

ISSUES: WON petitioner Villaruel is entitled to separation pay under article 284

HELD: A plain reading of Article 284 clearly presupposes that it is the employer
who terminates the services of the employee found to be suffering from any
disease and whose continued employment is prohibited by law or is prejudicial
to his health as well as to the health of his co-employees. It does not contemplate
a situation where it is the employee who severs his or her employment ties. This
is precisely the reason why Section 8, [14] Rule 1, Book VI of the Omnibus Rules
Implementing the Labor Code, directs that an employer shall not terminate the
services of the employee unless there is a certification by a competent public
health authority that the disease is of such nature or at such a stage that it cannot
be cured within a period of six (6) months even with proper medical treatment.
Hence, the pivotal question that should be settled in the present case is
whether respondent, in fact, dismissed petitioner from his employment.
A perusal of the Decisions of the Labor Arbiter and the NLRC would
show, however, that there was no discussion with respect to the
abovementioned issue. Both lower tribunals merely concluded that petitioner is
entitled to separation pay under Article 284 of the Labor Code without any
explanation. The Court finds no convincing justification, in the Decision of the
Labor Arbiter on why petitioner is entitled to such pay. In the same manner, the
NLRC Decision did not give any rationalization as the gist thereof simply
consisted of a quoted portion of the appealed Decision of the Labor Arbiter.
On the other hand, the Court agrees with the CA in its observation of the
following circumstances as proof that respondent did not terminate petitioner's
employment: first, the only cause of action in petitioner's original complaint is
that he was offered a very low separation pay; second, there was no allegation
of illegal dismissal, both in petitioner's original and amended complaints and
position paper; and, third, there was no prayer for reinstatement.

3E Andaya Ching Espiritu Hefti Galvez Gammad Lainez Lui Madamba Nagera Narvasa
Ong Palangdao Rosales Sanchez Santos Satrain Tabo (2014-2015)

Labor Relations Case Digest - Atty. Joyrich Golangco

In consonance with the above findings, the Court finds that petitioner
was the one who initiated the severance of his employment relations with
respondent. It is evident from the various pleadings filed by petitioner that he
never intended to return to his employment with respondent on the ground that
his health is failing. Indeed, petitioner did not ask for reinstatement. In fact, he
rejected respondent's offer for him to return to work. This is tantamount to
resignation.
204. ELSA S. MALIG-ON vs EQUITABLE GENERAL SERVICES, INC
DOCTRINE: The rule in termination cases is that the employer bears the burden
of proving that he dismissed his employee for a just cause, and, when the
employer claims that the employee resigned from work, the burden is on the
employer to prove that he did so willingly.

FACTS: Petitioner Elsa Malig-on (Malig-on) claimed that on March 4, 1996


respondent Equitable General Services, Inc. (the company) hired her as janitress
in its janitorial services. The company paid her P250.00 per day for a nine-hour
work. After six years or on February 15, 2002 Malig-ons immediate supervisor
told her that the company would be assigning her to another client. But it never
did despite several follow-ups that she made. 8 months later or on October 15,
2002 the company told Malig-on that she had to file a resignation letter before it
would reassign her. She complied but the company reneged on its undertaking,
prompting Malig-on to file a complaint against it for illegal dismissal.

The company denied Malig-ons allegations. It claimed that she just stopped
reporting for work on February 16, 2002 without giving any
reason. Consequently, the company wrote her two letters, first on August 23,
2002 and again on September 2, 2002, asking her to explain her continued
absence. On October 15, 2002 Malig-on showed up at the companys office and
submitted her resignation letter.
LA: the resignation made is valid and binding

NLRC: reversed. The company constructively dismissed Malig-on, and ordered


her reinstatement with full backwages

CA: reversed NLRC and reinstated LA

ISSUES: Whether or not there is a valid resignation

HELD: NO, The rule in termination cases is that the employer bears the burden of
proving that he dismissed his employee for a just cause, and, when the employer
claims that the employee resigned from work, the burden is on the employer to
prove that he did so willingly. Whether that is the case would largely depend on
the circumstances surrounding such alleged resignation. Those circumstances
must be consistent with the employees intent to give up work.
Here, the company claims that Malig-on voluntarily resigned, gave a
letter of resignation that she wrote with her own hand, used the vernacular
language, and signed it. But these are not enough. They merely prove that she
wrote that letter, a thing that she did not deny. She was quick to point out that
she wrote it after being told that she needed to resign so she could be cleared for
her next assignment.

According to the company, Malig-on simply dropped out of sight one day on
February 16, 2002 for no reason at all. Eight months later or on October 15,
2002 she appeared at the companys office and tendered her resignation. To the
companys surprise, three days later or on October 18, 2002 she went to the
NLRC office and filed her complaint against the company for illegal
dismissal. Clearly, however, these circumstances do not sound consistent with
resignation freely made.
First, when Malig-on reportedly dropped out of sight and the company
had no idea about the reason for it, the natural and right thing for it to do was
investigate why she had suddenly vanished. Indeed, the company needed to
write Malig-on immediately and ask her to explain in writing why she should
not be considered to have abandoned her job so the company may be cleared of
its responsibility as employer. This did not happen here.

Second, if Malig-on had abandoned her work and had no further interest
in it, there was no reason for her to suddenly show up at her former place of

3E Andaya Ching Espiritu Hefti Galvez Gammad Lainez Lui Madamba Nagera Narvasa
Ong Palangdao Rosales Sanchez Santos Satrain Tabo (2014-2015)

Labor Relations Case Digest - Atty. Joyrich Golangco

work after eight months and file her resignation letter. Her action would make
sense only if, as she claimed, she had been on floating status for over six months
and the company promised to give her a new assignment if she would go through
the process of resigning and reapplying.

And, third, that Malig-on went to the NLRC to file a complaint for unjust
dismissal just three days after she filed her alleged resignation letter is
inconsistent with genuine resignation, It would make sense only if, as Malig-on
claims, the company tricked her into filing for resignation upon a promise to give
her a new work assignment and failed to deliver such promise.
The company evidently placed Malig-on on floating status after being
relieved as janitress in a clients workplace. But, as the Court has repeatedly
ruled, such act of off-detailing Malig-on was not the equivalent of dismissal so
long as her floating status did not continue beyond a reasonable time. But, when
it ran up to more than six months, the company may be considered to have
constructively dismissed her from work, that is, as of August 16, 2002.Thus, her
purported resignation on October 15, 2002 could not have been legally possible.

205. CHIANG KAI SHEK V. TORRES


Keywords: Leakage; HEKASI quiz
Doctrine: There is constructive dismissal when:

FACTS:

there is cessation of work, because continued employment is rendered


impossible, unreasonable or unlikely, as an offer involving a demotion in
rank or a diminution in pay and other benefits.
there is an act of clear discrimination, insensibility, or disdain by an
employer becomes so unbearable on the part of the employee that it
could foreclose any choice by him except to forego his continued
employment.

Chiang Kai Shek College is a private educational institution that offers


elementary to college education to the public. Individual petitioner
Carmelita Espino is the Vice-President of the school.
Respondent had been employed as a grade school teacher of the school
from July 1970 until 31 May 2003. The manner of her severance from
employment is the matter at hand.
Respondent was accused of leaking a copy of a special quiz given to
Grade 5 students of HEKASI
Petitioners came to know about the leakage from one of the teachers of
HEKASI 5, Ms. Benabese. Ms. Benabese narrated that after giving a
special quiz, she borrowed the book of one of her students, Aileen
Anduyan, for the purpose of making an answer key. When she opened
Aileens book, a piece of paper fell. Said paper turned out to be a copy of
the same quiz she had just given and the same already contained
answers.
Mrs. Koo, who was in charge of the HEKASI area confronted respondent,
who had initially denied leaking the test paper but later on admitted it.
Ganito para madali, sabi ni Mrs Torres sobrang busy siya sa pagchecheck
ng workbooks, tapos may hiniram si Mrs Anduyan (co-teacher and
mother nung nakitaan ng leakage) hindi niya namalayan na kasama pala
dun yung paper ng quiz for HEKASI and that Mrs. Anduyan took the
same without her consent and without her knowledge. She is very
apologetic sabi niya hindi daw talaga intentional yun hindi rin naman
niya akalain a gagawin ni Mrs. Anduyan yun since faculty member din
sya
As expected syempre dineny ni Mrs. Anduyan ito, sabi niya willingly na
pinahiram ni Mrs. Torres sakanya yung quiz for HEKASI, then pinasilip
lang niya sa anak niya tapos sinagutan ng anak niya.
The Investigating Committee held a meeting and found Mrs. Torres and
Mrs. Anduyan guilty of committing a grave offense of the school policies
by leaking a special quiz. The Committee decided to impose the penalty
of one-month suspension without pay on respondent and forfeiture of
all the benefits scheduled to be given on Teachers Day.
According to petitioners, their Investigating Committee had actually
decided to terminate respondent and had in fact prepared a
memorandum of termination, but respondent allegedly pleaded through

3E Andaya Ching Espiritu Hefti Galvez Gammad Lainez Lui Madamba Nagera Narvasa
Ong Palangdao Rosales Sanchez Santos Satrain Tabo (2014-2015)

Labor Relations Case Digest - Atty. Joyrich Golangco

a letter for a change of punishment from termination to suspension and


that she will resign at the end of the year, to which the school acceded
(Mrs. Torres was suspended and continued her employment, however
near the end of the school year respondents counsel sent a letter to
petitioners demanding it to pay Mrs. Torres backwages, bonus, damages,
etc.
Respondent then filed a complaint for constructive dismissal and illegal
suspension with the Labor Arbiter, alleging that she was forced and
pressured to submit the written request for a change of penalty and
commitment to resign at the end of the school year. She was threatened
by the school management with immediate dismissal from service if she
did not submit the written statement. She claimed that she was not
formally charged with any offense and she was not served a copy of the
notice of the schools decision to terminate her services.
Petitioners contention: Mrs. Torres voluntarily resigned and she was
accorded due process when a formal investigation was conducted, which
gave her the opportunity to defend herself.
o LA: dismissed the complaint (no constructive dismissal)
o NLRC: affirmed LA (but pay separation pay out of social justice)
o CA: reversed NLRC (there was constructive dismissal)

ISSUES: whether or not the schools act of imposing the penalty of suspension
instead of immediate dismissal from service at the behest of the erring employee,
in exchange for the employees resignation at the end of the school year,
constitutes constructive dismissal.

HELD: NO

On 30 August 2002, the Investigating Committee found respondent


guilty of leaking a copy of the special quiz. Based on this infraction alone,
Chiang Kai Shek College would have been justified to validly terminate
respondent from service. As Associate Justice Antonio T. Carpio
emphasized, academic dishonesty is the worst offense a teacher can
make because teachers caught committing academic dishonesty
lose their credibility as educators and cease to be role models for
their students. More so that under Chiang Kai Shek College Faculty

Manual, leaking and selling of test questions is classified as a grave


offense punishable by dismissal/termination.
We do not find anything irregular with respondents handwritten letter.
The letter came about because respondent was faced with an imminent
dismissal and opted for an honorable severance from employment. That
respondent voluntarily resigned is a logical conclusion. Justice Arturo D.
Brion correctly observed that respondents infraction and the inevitable
and justifiable consequence of that infraction, i.e., termination of
employment, induced her to resign or promise to resign by the end of
the school year.
Given the indications of voluntary resignation, we rule that there is no
constructive dismissal in this case. There is constructive dismissal
when there is cessation of work, because continued employment is
rendered impossible, unreasonable or unlikely, as an offer
involving a demotion in rank or a diminution in pay and other
benefits. Aptly called a dismissal in disguise or an act amounting to
dismissal but made to appear as if it were not, constructive dismissal
may, likewise, exist if an act of clear discrimination, insensibility, or
disdain by an employer becomes so unbearable on the part of the
employee that it could foreclose any choice by him except to forego
his continued employment.
There was here no discrimination committed by petitioners. While
respondent did not tender her resignation wholeheartedly,
circumstances of her own making did not give her any other option.
With due process, she was found to have committed the grave offense of
leaking test questions. Dismissal from employment was the justified
equivalent penalty. Having realized that, she asked for, and was granted,
not just a deferred imposition of, but also an acceptable cover for the
penalty.
Respondents profession, the gravity of her infraction, and the fact that
she waited until the close of the school year to challenge her impending
resignation demonstrate that respondent had bargained for a graceful
exit and is now trying to renege on her obligation. Respondent should
not be rewarded for reneging on her promise to resign at the end of the
school year. Otherwise, employers placed in similar situations would no
longer extend compassion to employees. Compromise agreements, like

3E Andaya Ching Espiritu Hefti Galvez Gammad Lainez Lui Madamba Nagera Narvasa
Ong Palangdao Rosales Sanchez Santos Satrain Tabo (2014-2015)

Labor Relations Case Digest - Atty. Joyrich Golangco

that in the instant case, which lean towards desired liberality that favor
labor, would be discouraged.

206. VICTORINO OPINALDO, Petitioner, vs. NARCISA RAVINA, Respondent.

FACTS: Respondent Narcisa Ravina (Ravina) is the general manager and sole
proprietor of St. Louisse Security Agency (the Agency). Petitioner Victorino
Opinaldo (Opinaldo) is a security guard who had worked for the Agency until his
alleged illegal dismissal by respondent on December 22, 2006.
Agency hired the services of petitioner on October 5, 2005 and detailed him to
PAIJR Furniture Accessories (PAIJR) in Mandaue City.

In a letter, the owner of PAIJR submitted a written complaint to


respondent requesting him to relieve Opinaldo as he is no longer physically fit to
perform his duties and responsibilities as a company guard because of his health
condition.
Acceding to PAIJRs request, respondent relieved petitioner from his work and
required the latter to submit a medical certificate to prove that he is physically
and mentally fit for work as security guard.

On September 6, 2006, respondent reassigned petitioner to Gomez Construction


at Mandaue City. After working for two weeks for Gomez Construction and upon
receipt of his salary for services rendered within the said two-week period,
petitioner ceased to report for work. The records show that petitioners post at
Gomez Construction was the last assignment given to him by respondent.

On November 7, 2006, petitioner filed a complaint against respondent before the


DOLE Regional Office for underpayment of salary and nonpayment of other labor
standard benefits. The parties reached a compromise agreement. On November
27, 2006, petitioner signed a Quitclaim and Release before the Regional Office for
the amount ofP5,000.
After almost four weeks from the settlement of the case, petitioner returned to
respondents office on December 22, 2006.

Petitioner claims that when he asked respondent to sign an SSS Sickness


Notification which he was going to use in order to avail of the discounted fees for
a medical check-up, respondent allegedly refused and informed him that he
was no longer an employee of the Agency. Respondent allegedly told him that
when he signed the quitclaim and release form at the DOLE Regional Office, she
already considered him to have quit his employment.

Respondent, on the other hand, counterclaims that she did not illegally dismiss
petitioner and that it was a valid exercise of management prerogative that he
was not given any assignment pending the submission of the required
medical certificate of his fitness to work.
On January 26, 2007, petitioner filed a Complaint for Illegal Dismissal. Labor
Arbiter rendered a Decision holding respondent and the Agency liable for illegal
dismissal since respondents failed to establish that complainant was dismissed
for valid causes. There is no evidence that complainant was suffering from
physical illness which will explain his lack of assignment and that there is no
admissible proof that Ravina required complainant to submit a medical
certificate.
On appeal, NLRC affirmed the decision of the Labor Arbiter and dismissed the
appeal for lack of merit. The NLRC ruled that there was no just and authorized
cause for dismissal and held that without a certification from a competent
public authority that petitioner suffers from a disease of such nature or
stage that cannot be cured within a period of six (6) months even with
proper medical attendance, respondents are not justified in refusing
petitioners presence in the workplace.

The CA ruled for respondent and reversed and set aside the decision and
resolution of the NLRC and held that it was petitioner himself who failed to
report for work and therefore severed his employment with the Agency and that
petitioners claims relative to his alleged illegal dismissal were not substantiated.
ISSUES: WON respondent is liable for illegal dismissal.

HELD: Respondent harps that she could not be held liable for illegal dismissal
because, in the first place, she did not dismiss petitioner. Respondent maintains

3E Andaya Ching Espiritu Hefti Galvez Gammad Lainez Lui Madamba Nagera Narvasa
Ong Palangdao Rosales Sanchez Santos Satrain Tabo (2014-2015)

Labor Relations Case Digest - Atty. Joyrich Golangco

that she merely refused to give petitioner any work assignment until the
submission of a medical certificate.

Jurisprudence is replete with cases recognizing the right of the employer to have
free reign and enjoy sufficient discretion to regulate all aspects of employment,
including the prerogative to instill discipline in its employees and to impose
penalties, including dismissal, upon erring employees. However, the exercise of
management prerogative is not unlimited as it issubject to limitations provided
by law, collective bargaining agreements, and general principles of fair play and
justice. Hence, in the exercise of its management prerogative, an employer must
ensure that the policies, rules and regulations on work-related activities of the
employees must always be fair and reasonable and the corresponding penalties,
when prescribed, commensurate to the offense involved and to the degree of the
infraction.

In the case at bar, we recognize that respondents act of requiring petitioner to


undergo a medical examination and submit a medical certificate is a valid
exercise of management prerogative. This is further justified in view of the lettercomplaint from one of respondents clients, PAIJR. Petitioners job as security
guard naturally requires physical and mental fitness.
There is lack of evidence on record which establishes that respondent informed
petitioner that his failure to submit the required medical certificate will result in
his lack of work assignment. It is a basic principle of labor protection in this
jurisdiction that a worker cannot be deprived of his job without satisfying the
requirements of due process. The due process requirement in the deprivation of
ones employment is transcendental that it limits the exercise of the management
prerogative of the employer to control and regulate the affairs of the business.
In the case at bar, all that respondent employer needed to prove was that
petitioner employee was notified that his failure to submit the required medical
certificate will result in his lack of work assignment and eventually the
termination of his employment as a security guard. There is no iota of evidence
in the records, save for the bare allegations of respondent, that petitioner was
notified of such consequence for non-submission. In truth, the facts of the case
clearly show that respondent even reassigned petitioner to Gomez Construction
from his PAIJR post despite the non-submission of a medical certificate.

Had respondent exercised the rules of fair play, petitioner would have had the
option of complying or not complying with the medical certificate requirement
having full knowledge of the consequences of his actions.
207. Willi Hahn Enterprises v. Maghuyop

DOCTRINE: The failure of petitioner to pursue the termination proceedings


against respondent and to make her pay for the shortage incurred did not cast
doubt on the voluntary nature of her resignation. A decision to give a graceful
exit to an employee rather than to file an action for redress is perfectly within the
discretion of an employer. It is not uncommon that an employee is permitted to
resign to save face after the exposure of her malfeasance.
FACTS: Sometime in 1982, respondent Lilia Maghuyop was hired by petitioner
Willi Hahn as nanny of one of his sons. In 1986, she was employed as salesclerk
of Willi Hahn Enterprises (Ali Mall, Cubao branch), an authorized dealer of
sporting goods, guns and ammunitions. In 1996, she was promoted as store
manager of its branch in Shoe Mart (SM) Cebu, with a monthly salary of
P8,240.00.
On February 25, 1998, petitioner conducted an Inventory Report and
discovered that its SM Cebu branch incurred stock shortages and nonremittances in the total amount of P27,727.39. In the latter part of July 1998,
petitioner decided to terminate the services of respondent, however,
before he could do so, the latter tendered her resignation. Believing the
good faith of respondent in resigning, petitioner decided not to file charges
against her anymore.
On the other hand, respondent claimed that on July 22, 1998, while she was
in SM Cebu branch, she was approached by Tony Abu and Cesar Araneta who
ordered her to close shop and to write a letter to Mr. and Mrs. Hahn thanking
them for the years she had been in their employ and for all the benefits she
received from them. She refused to obey the order, but Tony Abu typed the letter
of resignation and asked her to sign the same. Respondent admitted that she
read and affixed her signature on the letter. Thereafter, she was allegedly told to

3E Andaya Ching Espiritu Hefti Galvez Gammad Lainez Lui Madamba Nagera Narvasa
Ong Palangdao Rosales Sanchez Santos Satrain Tabo (2014-2015)

Labor Relations Case Digest - Atty. Joyrich Golangco

pack her belongings and to vacate the housing unit provided by the company for
her family.

On August 25, 1998, respondent filed a complaint with the NLRC, alleging
that she should be awarded backwages, separation pay, salary for July 16-22,
1998 which was withheld by petitioner, proportionate 13th month pay, damages
and attorneys fees.

The LA ruled in favor of respondent, but dismissed the charge for illegal
dismissal. The appeal to the NLRC was denied for lack of merit. The CA reversed
the decision and held that the attendant circumstances put to doubt the
voluntariness of the execution of the resignation letter. Petitioner was
ordered to pay respondent her unpaid salary from July 16 to 22, 1998, full back
wages (inclusive of basic pay, 13th month pay, allowances and monetary value of
all benefits) computed from July 22, 1998 up to the finality of this decision,
separation pay equivalent to one (1) month pay for every year of service, moral
damages of P10,000.00 and 10% attorneys fees.
ISSUES: Whether respondent voluntarily resigned as manager of the SM Cebu
branch.

HELD: Yes, her resignation was voluntary. The letter is simple, candid and direct
to the point. We find no merit in respondents claim that being a mere clerk, she
did not realize the consequences of her resignation. Although she started as
nanny to the son of petitioner Willi Hahn, she has risen to being the
manager and officer-in-charge of the Willi Hahn Enterprises in SM Cebu
branch.
The failure of petitioner to pursue the termination proceedings against
respondent and to make her pay for the shortage incurred did not cast
doubt on the voluntary nature of her resignation. A decision to give a
graceful exit to an employee rather than to file an action for redress is perfectly
within the discretion of an employer. It is not uncommon that an employee is
permitted to resign to save face after the exposure of her malfeasance. Under the
circumstances, the failure of petitioner to file action against the respondent
should be considered as an act of compassion for one who used to be a trusted
employee and a close member of the household.

Respondents unsubstantiated and self-serving claim that she was coerced


into signing the resignation letter does not deserve credence. It is a basic rule
in evidence that the burden of proof is on the part of the party who makes
the allegations. Respondent failed to discharge this burden.
Moreover, the Court of Appeals finding that respondent had no motive to
resign because the charges of dishonesty were not fully substantiated has no
basis. Had the separation of respondent been for dismissal due to loss of trust
and confidence, substantial evidence of the shortages and non-remittances
would have been indispensable. Such, is not the case here considering her
voluntary resignation.

The rule that the filing of a complaint for illegal dismissal is inconsistent
with resignation, is not applicable to the instant case. The filing of an illegal
dismissal case by respondent was evidently a mere afterthought. It was
filed not because she wanted to return to work but to claim separation pay and
backwages.

Settled is the rule that factual findings of labor officials who are deemed to
have acquired expertise in matters within their respective jurisdiction are
generally accorded not only respect, but even finality, and bind the Supreme
Court when supported by substantial evidence. The findings of both the Labor
Arbiter and the NLRC are amply supported by the required quantum of
evidence, i.e., evidence as a reasonable mind might accept as adequate to support
a conclusion.
Hence, The Court find no reason to deviate from the conclusion of both the
NLRC and the Labor Arbiter that respondent, having tendered a voluntary
resignation was not illegally dismissed.
208. Skippers United Pacific, Inc. vs. Doza
DOCTRINE: Article 285 of the Labor Code recognizes termination by the employee
of the employment contract by serving written notice on the employer at least
one (1) month in advance. Given that provision, the law contemplates the
requirement of a written notice of resignation. In the absence of a written
resignation, it is safe to presume that the employer terminated the seafarers.

3E Andaya Ching Espiritu Hefti Galvez Gammad Lainez Lui Madamba Nagera Narvasa
Ong Palangdao Rosales Sanchez Santos Satrain Tabo (2014-2015)

Labor Relations Case Digest - Atty. Joyrich Golangco

FACTS: Petitioner deployed De Gracia, Lata and Aprosta to work on board


the vessel MV Wisdom Star.On December 3 1998, Skippers alleges that De Garcia
smelling strongly of alcohol, went to the cabin of Gabriel Oleszek, MV Wisdom
Stars Master. Skippers claims that he was rude and shouted noisily to the
master. De Gracia left the masters cabin after a few minutes and was heard
shouting very loudly somewhere down the corridors. The incident was
evidenced by the Captains Report sent on said date.
Furthermore, Skippers also claim that on January 22, 1999, Aprosta, De Gracia,
Lata and Daza arrived in the masters cabin and demanded immediate
repatriation because they were not satisfied with the ship. De Gracia, et al.
threatened that they may become crazy any moment and demanded for all
outstanding payments due to them. The incident is evidenced by a telex of
Cosmoship MV Wisdom to skippers but had conflicting dates. De Gracia claims
that Skippers failed to remit their respective allotments, compelling them to vent
their grievances with the Romanian Seafarers Union. On January 28, 1999, the
Filipino seafarers were unceremoniously discharged and immediately
repatriated. Upon arrival in the Philippines, they filed a complaint for illegal
dismissal with the LA.
LA: Dismissed action for lack of merit because the seafarers voluntarily preterminated their employment contracts by demanding for immediate
repatriation due to dissatisfaction with the ship.

NLRC: Dismissed De Gracia, et al.s appeal for lack of merit and affirmed the LAs
decision. They also considered De Gracia, et al.s claim for home allotment for
December 1998 unsubstantiated.

CA: Granted De Gracia, et al.s petition and reversed the decisions of the Labor
Arbiter and NLRC. The telex message was "a self-serving document that does not
satisfy the requirement of substantial evidence, or that amount of relevant
evidence which a reasonable mind might accept as adequate to justify the
conclusion that petitioners indeed voluntarily demanded their immediate
repatriation."
ISSUES: Whether or not the seafarers demand for immediate repatriation can be
considered an act of voluntary resignation considering the telex message sent.

HELD: For a worker's dismissal to be considered valid, it must comply with both
procedural and substantive due process. The legality of the manner of dismissal
constitutes procedural due process, while the legality of the act of dismissal
constitutes substantive due process. Procedural due process in dismissal cases
consists of the twin requirements of notice and hearing. The employer must
furnish the employee with two written notices before the termination of
employment can be effected: (1) the first notice apprises the employee of the
particular acts or omissions for which his dismissal is sought; and (2) the second
notice informs the employee of the employer's decision to dismiss him. Before
the issuance of the second notice, the requirement of a hearing must be complied
with by giving the worker an opportunity to be heard. It is not necessary that an
actual hearing be conducted. Substantive due process, on the other hand,
requires that dismissal by the employer be made under a just or authorized
cause under Articles 282 to 284 of the Labor Code. In this case, there was no
written notice furnished to De Gracia, et al., regarding the cause of their
dismissal. Cosmoship furnished a written notice (telex) to Skippers, the local
manning agency, claiming that De Gracia, et al., were repatriated because the
latter voluntarily pre-terminated their contracts. This telex was given credibility
and weight by the Labor Arbiter and NLRC in deciding that there was pretermination of the employment contract "akin to resignation" and no illegal
dismissal. However, as correctly ruled by the CA, the telex message is "a biased
and self-serving document that does not satisfy the requirement of substantial
evidence." If, indeed, De Gracia, et al., voluntarily pre-terminated their contracts,
then De Gracia, et al., should have submitted their written resignations.
Article 285 of the Labor Code recognizes termination by the employee of the
employment contract by "serving written notice on the employer at least one (1)
month in advance." Given that provision, the law contemplates the requirement
of a written notice of resignation. In the absence of a written resignation, it is safe
to presume that the employer terminated the seafarers. In addition, the telex
message relied upon by the Labor Arbiter and NLRC bore conflicting dates of 22
January 1998 and 22January 1999, giving doubt to the veracity and authenticity
of the document. In 22 January 1998, DeGracia, et al., were not even employed
yet by the foreign principal.

3E Andaya Ching Espiritu Hefti Galvez Gammad Lainez Lui Madamba Nagera Narvasa
Ong Palangdao Rosales Sanchez Santos Satrain Tabo (2014-2015)

Labor Relations Case Digest - Atty. Joyrich Golangco


209. JONATHAN V. MORALES vs. HARBOUR CENTRE PORT TERMINAL, INC.
G.R. No. 174208 January 25, 2012
DOCTRINE: In cases of a transfer of an employee, the rule is settled that the
employer is charged with the burden of proving that its conduct and action are
for valid and legitimate grounds such as genuine business necessity and that the
transfer is not unreasonable, inconvenient or prejudicial to the employee. If the
employer cannot overcome this burden of proof, the employees transfer shall be
tantamount to unlawful constructive dismissal.

FACTS: On 16 May 2000, petitioner Morales was hired by respondent HCPTI as


an Accountant and Acting Finance Ofcer. He was regularized and later on
promoted to Division Manager of the Accounting Department. Subsequent to
HCPTIs transfer to its new ofces at Vitas, Tondo, Manila, Morales received an
inter-ofce memorandum, reassigning him to Operations Cost Accounting,
tasked with the duty of "monitoring and evaluating all consumables requests,
gears and equipment" related to the corporations operations and of interacting
with its sub-contractor, Bulk Fleet Marine Corporation.
Because of this, Morales wrote Singson (HCPTIs new Administration Manager),
protesting that his reassignment was a clear demotion since the position to
which he was transferred was not even included in HCPTIs plantilla. In response
to Morales grievance that he had been effectively placed on oating status,
Singson issued an inter-ofce memorandum to the effect that "transfer of
employees is a management prerogative" and that HCPTI had "the right and
responsibility to nd the perfect balance between the skills and abilities of
employees to the needs of the business."
For the whole of the ensuing month Morales was absent from work and/or tardy.
Singson issued to Morales another inter-ofce memorandum denominated as a
First Warning. The memorandum reminded Morales that, as an employee of
HCPTI, he was subject to its rules and regulations and could be disciplinarily
dealt with pursuant to its Code of Conduct. In view of the absences Morales
continued to incur, HCPTI issued a Second Warning and a Notice to Report for
Work and Final Warning.

In the meantime, Morales led a complaint against HCPTI, Filart and Singson, for
constructive dismissal, moral and exemplary damages as well as attorneys fees.
HCPTI, Filart and Singson led their position paper, arguing that Morales
abandoned his employment and was not constructively dismissed. They averred
that: Morales admitted his inability to handle his tasks at the corporations
Accounting Department; that as a consequence, HCPTI reassigned Morales from
managerial accounting to operations cost accounting as an exercise of its
management prerogative to assign its employees to jobs for which they are best
suited; and, that despite the justication in Singsons reply to his protest against
his reassignment, Morales chose to stop reporting for work.
LA: Dismissed for lack of merit Morales complaint for constructive dismissal,
claiming that Morales reassignment was a valid exercise of HCPTIs management
prerogative.
NLRC: LA decision reversed and set aside, nding that Morales reassignment
was a clear demotion despite lack of showing of diminution of salaries and
benets.
CA: Reversed the NLRCs Decision, upon the following ndings and conclusions:

(a) Morales reassignment to Operations Cost Accounting was a valid


exercise of HCPTIs prerogative to transfer its employees as the exigencies of the
business may require;
(b) the transfer cannot be construed as constructive dismissal since it
entailed no demotion in rank, salaries and benets;
(c) rather than being terminated, Morales refused his new assignment
by taking a leave of absence from 4 to 17 April 2003 and disregarding HCPTIs
warnings and directives to report back for work.
ISSUES: W/N Change in designation/ position constituted constructive dismissal.

HELD: Yes. Constructive dismissal exists where there is cessation of work


because "continued employment is rendered impossible, unreasonable or
unlikely, as an offer involving a demotion in rank or a diminution in pay" and
other benets. Aptly called a dismissal in disguise or an act amounting to
dismissal but made to appear as if it were not, constructive dismissal may,
likewise, exist if an act of clear discrimination, insensibility, or disdain by an

3E Andaya Ching Espiritu Hefti Galvez Gammad Lainez Lui Madamba Nagera Narvasa
Ong Palangdao Rosales Sanchez Santos Satrain Tabo (2014-2015)

Labor Relations Case Digest - Atty. Joyrich Golangco

employer becomes so unbearable on the part of the employee that it could


foreclose any choice by him except to forego his continued employment.
In cases of a transfer of an employee, the rule is settled that the employer is
charged with the burden of proving that its conduct and action are for valid and
legitimate grounds such as genuine business necessity and that the transfer is
not unreasonable, inconvenient or prejudicial to the employee. If the employer
cannot overcome this burden of proof, the employees transfer shall be
tantamount to unlawful constructive dismissal.

Our perusal of the record shows that HCPTI miserably failed to discharge the
foregoing onus. While there was a lack of showing that the transfer or
reassignment entailed a diminution of salary and benets, one fact that must not
be lost sight of was that Morales was already occupying the position of Division
Manager at HCPTIs Accounting Department as a consequence of his promotion
to said position.
1. Morales demotion is evident from the fact that his reassignment
entailed a transfer from a managerial position to one which was not even
included in the corporations plantilla. For an employee newly charged with
functions which even the CA recognized as pertaining to the Operations
Department, it also struck a discordant chord that Morales was, just the same,
directed by HCPTI to report to Filart, its Vice- President for Finance with whom
he already had a problematic working relationship.
2. By itself, HCPTIs claim of reorganization is bereft of any supporting
evidence in the record. Having pointed out the matter in his 31 March 2003
written protest, Morales was able to prove that HCPTIs existing plantilla did not
include an Operations Cost Accounting Department and/or an Operations Cost
Accountant. As the party belatedly seeking to justify the reassignment due to the
supposed reorganization of its corporate structure, HCPTI, in contrast, did not
even bother to show that it had implemented a corporate reorganization and/or
approved a new plantilla of positions which included the one to which Morales
was being transferred.
3.Although much had been made about Morales supposed refusal to
heed his employers repeated directives for him to return to work, our perusal of
the record also shows that HCPTIs theory of abandonment of employment
cannot bear close scrutiny. The Inter-Ofce Memorandum labeled as a Second

Warning was sent to Morales thru the JRS Express only two (2) days after
summons were served on HCPTI, Filart and Singson. Sent to Morales after the
parties initial conference before the Labor Arbiter, there was obviously even less
reason for HCPTIs letter denominated as Notice to Report for Work and Final
Warning. As a just and valid ground for dismissal, at any rate, abandonment
requires the deliberate, unjustied refusal of the employee to resume his
employment, without any intention of returning. Since an employee like Morales
who takes steps to protest his dismissal cannot logically be said to have
abandoned his work, it is a settled doctrine that the ling of a complaint for
illegal dismissal is inconsistent with abandonment of employment.
210. SHS PERFORATED MATERIALS, INC., v DIAZ (2010)
DOCTRINES: Although management prerogative refers to the right to regulate
all aspects of employment, it cannot be understood to include the right to
temporarily withhold salary/wages without the consent of the employee. To
sanction such an interpretation would be contrary to Article 116 and Art 113.

There is constructive dismissal if an act of clear discrimination, insensibility, or


disdain by an employer becomes so unbearable on the part of the employee that
it would foreclose any choice by him except to forego his continued employment.
It exists where there is cessation of work because continued employment is
rendered impossible, unreasonable or unlikely, as an offer involving a demotion
in rank and a diminution in pay.
FACTS: Diaz was hired by petitioner SHS as Manager for Business Development
on probationary status from July 18, 2005 to January 18, 2006, with a monthly
salary of P100,000.00. As Hartmannnshenn, SHSs president, was often abroad,
his instructions to Diaz were either sent thru e-mail or telephone calls. Hence,
there was no close supervision as to Diazs work.

Hartmannshenn expressed his dissatisfaction over Diazs poor performance; that


he allegedly failed to make any concrete business proposal or implement any
specific measure to improve the productivity of the SHS office and plant or
deliver sales. In numerous emails, respondent acknowledged his poor
performance and offered to resign from the company. Diaz denied having sent or
received any of the messages. Meanwhile, Hartmannshenn ordered the company

3E Andaya Ching Espiritu Hefti Galvez Gammad Lainez Lui Madamba Nagera Narvasa
Ong Palangdao Rosales Sanchez Santos Satrain Tabo (2014-2015)

Labor Relations Case Digest - Atty. Joyrich Golangco

not to release Diazs salary for having failed to immediately communicate with
the former.

Diaz then tendered his resignation letter with a demand of his unpaid salary
which is being withheld illegally. Hartmannshenn then accepted respondents
resignation and informed him that his salary would be released upon explanation
of his failure to report to work, and proof that he did, in fact, work for the period
in question. He demanded that respondent surrender all company property and
information in his possession. Respondent agreed to these exit conditions
through email. Instead of complying with the said conditions, however, Diaz sent
another email to Hartmannshenn and Schumacher appealing for the release of
his salary. They refused his demand while Diaz alleged that he was rudely
treated during their meeting and that he was forced to accept a certain amount
instead of his accrued wages and to stop working for SHS.
Diaz filed a Complaint for illegal dismissal; non-payment of salaries/wages and
13th month pay with prayer for reinstatement and full backwages, among others.
The LA ruled against the petitioners. The LA found that respondent was
constructively dismissed because the withholding of his salary was contrary to
Article 116 of the Labor Code as it was not one of the exceptions for allowable
wage deduction by the employer under Article 113 of the Labor Code. He had no
other alternative but to resign because he could not be expected to continue
working for an employer who withheld wages without valid cause.

The NLRC, reversed the said decision holding that that the withholding of
respondents salary was a valid exercise of management prerogative. The act was
deemed justified as it was reasonable to demand an explanation for failure to
report to work and to account for his work accomplishments. The NLRC held that
the respondent voluntarily resigned as evidenced by the language used in his
resignation letter and demand letters.
The CA reversed NLRCs decision. It held that withholding respondents salary
was not a valid exercise of management prerogative as there is no such thing as a
management prerogative to withhold wages temporarily. The malicious
withholding of respondents salary made it impossible or unacceptable for
respondent to continue working, thus, compelling him to resign. The
respondents immediate filing of a complaint for illegal dismissal could only

mean that his resignation was not voluntary. As a probationary employee


entitled to security of tenure, respondent was illegally dismissed.

ISSUES: WON Diaz was constructively dismissed

HELD: YES. What made it impossible, unreasonable or unlikely for respondent to


continue working for SHS was the unlawful withholding of his salary. For said
reason, he was forced to resign. It is of no moment that he served his resignation
letter on November 30, 2005, the last day of the payroll period and a nonworking holiday, since his salary was already due him on November 29, 2005,
being the last working day of said period. In fact, he was then informed that the
wages of all the other SHS employees were already released, and only his was
being withheld. What is significant is that the respondent prepared and served
his resignation letter right after he was informed that his salary was being
withheld. It would be absurd to require respondent to tolerate the unlawful
withholding of his salary for a longer period before his employment can be
considered as so impossible, unreasonable or unlikely as to constitute
constructive dismissal. Even granting that the withholding of respondents salary
on November 30, 2005, would not constitute an unlawful act, the continued
refusal to release his salary after the payroll period was clearly unlawful. The
petitioners claim that they prepared the check ready for pick-up cannot undo
the unlawful withholding.
It is worthy to note that in his resignation letter, respondent cited petitioners
illegal and unfair labor practice as his cause for resignation. As correctly noted
by the CA, respondent lost no time in submitting his resignation letter and
eventually filing a complaint for illegal dismissal just a few days after his salary
was withheld. These circumstances are inconsistent with voluntary resignation
and bolster the finding of constructive dismissal. Respondent was constructively
dismissed and, therefore, illegally dismissed.
211. San Miguel Properties Philippines vs Gucaban

DOCTRINE: In resignation, the intent of the employee to relinquish must concur


with the overt act of relinquishment; hence, the acts of the employee before and
after the alleged resignation must be considered in determining whether he in
fact intended to terminate his employment. In illegal dismissal cases,
fundamental is the rule that when an employer interposes the defense of

3E Andaya Ching Espiritu Hefti Galvez Gammad Lainez Lui Madamba Nagera Narvasa
Ong Palangdao Rosales Sanchez Santos Satrain Tabo (2014-2015)

Labor Relations Case Digest - Atty. Joyrich Golangco

resignation, on him necessarily rests the burden to prove that the employee
indeed voluntarily resigned.
FACTS: Gwendellyn Rose Gucaban (Gucaban) was initially engaged as a
construction management specialist by petitioner San Miguel Properties
Philippines, Inc. (SMPI) in 1991., she, by her satisfactory performance on the job,
was promoted in 1994 and 1995, respectively, to the position of technical
services manager, and then of project development manager. As project
development manager, she also sat as a member of the companys management
committee. She had been in continuous service in the latter capacity until her
severance from the company in February 1998.

In her complaint for illegal dismissal filed on June 26, 1998, Gucaban
alleged that her separation from service was practically forced upon her by
management. She claimed that on January 27, 1998, she was informed by SMPIs
President and Chief Executive Officer, Federico Gonzalez (Gonzalez), that the
company was planning to reorganize its manpower in order to cut on costs, and
that she must file for resignation or otherwise face termination. Three days later,
the Human Resource Department allegedly furnished her a blank resignation
form which she refused to sign. From then on, she had been hounded by
Gonzalez to sign and submit her resignation letter.

Gucaban that she had been kept off from all the meetings of the management
committee, and that on February 12, 1998, she received an evaluation report
signed by Gonzalez showing that for the covered period she had been negligent
and unsatisfactory in the performance of her duties. She found said report to be
unfounded and unfair, because no less than the companys Vice-President for
Property Management, Manuel Torres (Torres), in a subsequent memorandum,
had actually vouched for her competence and efficiency on the job It was
supposedly the extreme humiliation and alienation that impelled her to submit a
signed resignation letter on February 18, 1998.
SMPI argued that it truly encountered a steep market decline in 1997 that
necessitated cost-cutting measures and streamlining of its employee structure
which, in turn, would require the abolition of certain job positions; Gucabans
post as project development manager was one of such positions. As a measure of
generosity, it allegedly proposed to Gucaban that she voluntarily resign from

office in consideration of a financial package. Gucaban, however, did not


communicate her acceptance of the offer and, instead, she allegedly conferred
with the Human Resource Department and negotiated to augment her benefits
package.
LA: Dismissed the case for illegal dismissal for lack of merit. Resignation was
voluntary on the part of Gucaban.
NLRC: There was illegal dismissal.
CA: Affirmed the NLRC.
ISSUES:

W/N the resignation was voluntary

HELD: NO. Resignation the formal pronouncement or relinquishment of a


position or office is the voluntary act of an employee who is in a situation
where he believes that personal reasons cannot be sacrificed in favor of the
exigency of the service, and he has then no other choice but to disassociate
himself from employment. The intent to relinquish must concur with the overt
act of relinquishment; hence, the acts of the employee before and after the
alleged resignation must be considered in determining whether he in fact
intended to terminate his employment. In illegal dismissal cases, fundamental is
the rule that when an employer interposes the defense of resignation, on him
necessarily rests the burden to prove that the employee indeed voluntarily
resigned.

It is not difficult to see that, shortly prior to and at the time of Gucabans
alleged resignation, there was actually no genuine corporate restructuring plan in
place as yet. In other words, although the company might have been suffering
from losses due to market decline as alleged, there was still no concrete plan for
a corporate reorganization at the time Gonzalez presented to Gucaban the
seemingly last available alternative options of voluntary resignation and
termination by abolition of her office. Certainly, inasmuch as the necessity of
corporate reorganization generally lies within the exclusive prerogative of
management, Gucaban at that point had no facility to ascertain the truth behind
it, and neither was she in a position to question it right then and there. Indeed,
she could not have chosen to file for resignation had SMPI not broached to
her the possibility of her being terminated from service on account of the

3E Andaya Ching Espiritu Hefti Galvez Gammad Lainez Lui Madamba Nagera Narvasa
Ong Palangdao Rosales Sanchez Santos Satrain Tabo (2014-2015)

supposed reorganization.

Labor Relations Case Digest - Atty. Joyrich Golangco

Separation pay was awarded in lieu of the reinstatement because of the


lapse of 10 years since the constructive dismissal and because of the strained
relations between the parties.
212. BMG RECORDS (PHILS.), INC. and JOSE YAP, JR., vs. AIDA C. APARECIO
and NATIONAL LABOR RELATIONS COMMISSION

DOCTRINE: Once an employee resigns and his resignation is accepted, he no


longer has any right to the job. If the employee later changes his mind, he must
ask for approval of the withdrawal of his resignation from his employer, as if he
were re-applying for the job. It will then be up to the employer to determine
whether or not his service would be continued. If the employer accepts said
withdrawal, the employee retains his job. If the employer does not x x x the
employee cannot claim illegal dismissal for the employer has the right to
determine who his employees will be.
FACTS: Petitioner BMG is engaged in the business of selling various audio
records nationwide. It hired private respondent Aparecio as one of the promo
girls

Aparecio filed a complaint against BMG and its Branch Manager, Yap, Jr., for
illegal dismissal and non-payment of overtime pay, holiday pay, premium pay for
rest day, 13th month pay, service incentive leave, and separation pay.5
She alleged: She was illegally dismissed. Before said date, she was asked by
respondent to resign and execute a letter of resignation with the promise that
she will be given all her benefits. She executed a letter-resignation even without
proper accounting. She induced and compelled to submit a letter of resignation
believing on respondent's promise and assurance to pay all the benefits due her.
After executing said resignation letter, the respondent did not make good its
promise.

Petitioners narrate that Aparecio was initially performing well as an employee


but as years passed by she seemed to be complacent in the performance of her

job. Aparecio and two other promo girls, Soco and Mutya, intimated to their
supervisor that they were intending to resign and were requesting for some
financial assistance. BMG made it clear that, as a company policy, an employee
who resigns from service is not entitled to financial assistance, but considering
the length of their service and due to humanitarian consideration it would accede
to the request after they secure their respective clearances. The three employees
tendered their resignations, which were accepted. Attached to petitioners'
Position Paper7 were the sworn statements of Jose Yap, Jr. and Evangeline A.
Magno, supervisor of BMG.
LA dismissed Aparecio's complaint. Since the letter of resignation showed no
signs that it was made through duress or compulsion, it was concluded that the
severance of her employment in BMG was brought about by her resignation and
not by the illegal dismissal supposedly committed by the latter.

NLRC found that Aparecio was illegally dismissed from service.It held that
resignation is inconsistent with the filing of the complaint.
CA affirmed in toto the judgment of the NLRC. It held that the [petitioners] failed
to support [their] claim that [Aparecio's] resignation was made out of her own
volition.
ISSUES: Whether or not respondent resigned voluntarily.
HELD: The petition is meritorious.

Aparecio submits that fraud, undue influence, intimidation, and/or mistake were
attendant upon her resignation from BMG. As her consent was allegedly vitiated,
the act of resigning became involuntary; hence, petitioners are guilty of illegal
dismissal. The argument is not tenable. This Court finds nothing to support
Aparecio's allegation that fraud was employed on her to resign. Fraud exists only
when, through insidious words or machinations, the other party is induced to act
and without which, the latter would not have agreed to.25 Fraud and
misrepresentation are never presumed; it must be proved by clear and
convincing evidence and not mere preponderance of evidence.26

3E Andaya Ching Espiritu Hefti Galvez Gammad Lainez Lui Madamba Nagera Narvasa
Ong Palangdao Rosales Sanchez Santos Satrain Tabo (2014-2015)

Labor Relations Case Digest - Atty. Joyrich Golangco

Aparecio alleged that her resignation was wrongfully obtained when petitioners
did not keep the promise of giving her employment benefits and financial
assistance without any deductions. Without a showing of the nature and extent of
such "inducement," however, such submission fails to establish that there was in
fact a deception on the part of petitioners.

Aparecio did not adduce any competent evidence to prove that force or threat
was applied by petitioners. For intimidation to vitiate consent, the following
requisites must be present: (1) that the intimidation caused the consent to be
given; (2) that the threatened act be unjust or unlawful; (3) that the threat be
real or serious, there being evident disproportion between the evil and the
resistance which all men can offer, leading to the choice of doing the act which is
forced on the person to do as the lesser evil; and (4) that it produces a wellgrounded fear from the fact that the person from whom it comes has the
necessary means or ability to inflict the threatened injury to his person or
property. In the instant case, not one of these essential elements was amply
proven by Aparecio.
Petitioners correctly point out that the NLRC finding, which the CA erroneously
affirmed, of a "strong and irresistible economic pressure originating from
[petitioners] if only to push [Aparecio] into accepting the offer" is not supported
by any evidence in the records but is merely based on conjectures and
guesswork. Resignation is the voluntary act of an employee who is in a situation
where one believes that personal reasons cannot be sacrificed in favor of the
exigency of the service, and one has no other choice but to dissociate oneself
from employment. It is a formal pronouncement or relinquishment of an office,
with the intention of relinquishing the office accompanied by the act of
relinquishment. As the intent to relinquish must concur with the overt act of
relinquishment, the acts of the employee before and after the alleged resignation
must be considered in determining whether in fact, he or she intended to sever
from his or her employment.30
The acceptance by petitioners of Aparecio's resignation rendered the same
effective.31 Upon such acceptance, it may not be unilaterally withdrawn without
the consent of petitioners.32 When the employee later signified the intention of
continuing his or her work, it was already up to the employer to accept the

withdrawal of his or her resignation. The mere fact that the withdrawal was not
accepted does not constitute illegal dismissal, the acceptance of the withdrawal
of the resignation being the employer's sole prerogative.
Certainly, what transpired here was caused by an employee's error of judgment
and not by the employer's application of means vitiating the consent to resign.
WHEREFORE, the petition is GRANTED.

213. SKM ART CRAFT CO vs BAUCA

DOCTRINE: The bona fide suspension of the operations of a business or


undertaking for a period not exceeding six (6) months, shall not terminate
employment.

In all such cases, the employer shall reinstate the employee to his former
position without loss of seniority rights if he indicates his desire to resume his
work not later than one (1) month from the resumption of operations of his
employer

FACTS: The 23 respondents in G.R. No. 171282 were employed by petitioner


SKM Art Craft Corporation which is engaged in the handicraft business. On April
18, 2000, around 1:12 a.m., a fire occurred at the inspection and
receiving/repair/packing area of petitioners premises in Intramuros, Manila.
The fire investigation report4 stated that the structure and the beach rubber
building were totally damaged. Also burned were four container vans and a
trailer truck. The estimated damage was P22 million.
On May 8, 2000, petitioner informed respondents that it will suspend its
operations for six months, effective May 9, 2000.5

On May 16, 2000, only eight days after receiving notice of the suspension of
petitioners operations, the 23 respondents (and other co-workers) filed a
complaint for illegal dismissal,. They alleged that there was discrimination in
choosing the workers to be laid off and that petitioner had discovered that most
of them were members of a newly-organized union.6

3E Andaya Ching Espiritu Hefti Galvez Gammad Lainez Lui Madamba Nagera Narvasa
Ong Palangdao Rosales Sanchez Santos Satrain Tabo (2014-2015)

Labor Relations Case Digest - Atty. Joyrich Golangco

Petitioner denied the claim of illegal dismissal and said that Article 2867 of the
Labor Code allows the bona fide suspension of a business or undertaking for a
period not exceeding six months. Petitioner claimed that the fire cost it millions
in losses and that it is impossible to resume its normal operations for a
significant period of time.8

the Labor Arbiter ruled that respondents were illegally dismissed and ordered
petitioner to reinstate them and pay them back wages of P59,918.41 each, the
amount being subject to further computation up to the date of their actual
reinstatement. The Labor Arbiter ruled that the fire that burned a part of
petitioners premises may validate the suspension of respondents employment,
but the suspension must not exceed six months. Since petitioner failed to recall
respondents after the lapse of six months, the Labor Arbiter held that
respondents were illegally dismissed.

National Labor Relations Commission (NLRC) set aside the Labor Arbiters
Decision and ruled that there was no illegal dismissal. The NLRC ordered that
respondents be reinstated to their former positions but it deleted the award of
back wages. The NLRC noted that the fire caused millions in damages to
petitioner. Thus, petitioners suspension of operations is valid under Article 286
of the Labor Code. It was not meant to remove respondents because they were
union members. The NLRC added that the illegal dismissal complaint filed by
respondents was premature for it was filed during the six-month period of
suspension of operations. .
The CA ruled that petitioner failed to prove that its suspension of operations is
bona fide . The CA noted that the proof of alleged losses the list of items and
materials allegedly burned was not even certified or signed by petitioners
accountant or comptroller. And even if the suspension of operations is
considered bona fide, the CA said that respondents were not reinstated after six
months. Thus, respondents are deemed to have been illegally dismissed. The CA
also noted that petitioners manifestation that it is willing to admit the
respondents if they return to work was belatedly made after almost one year
from the expiration of the suspension of operations.
ISSUES: WON the respondents were illegally dismissed

HELD: while we agree with the NLRC that the suspension of petitioners
operation is valid, the Labor Arbiter and the CA are correct that respondents
were illegally dismissed since they were not recalled after six months, after the
bona fide suspension of petitioners operations.

It is admitted that petitioners premises was burned on April 18,


2000.22 Petitioner also submitted pictures23 of its premises after the fire, the
certification24 by the Barangay Chairman that petitioners factory was burned,
and the fire investigation report25 of the Bureau of Fire Protection. To prove the
damages, petitioner submitted a list26 of burned machines, its inventory27 for
April 2000 and the fire investigation report which stated that the estimated
damage isP22 million.

We therefore agree with the NLRC that petitioners suspension of operations is


valid because the fire caused substantial losses to petitioner and damaged its
factory. On this point, we disagree with the CA that petitioner failed to prove that
its suspension of operations is bona fide. The list of materials burned was not the
only evidence submitted by petitioner. It was corroborated by pictures and the
fire investigation report, and they constitute substantial evidence of petitioners
losses.
Under Article 286 of the Labor Code, the bona fide suspension of the operations
of a business or undertaking for a period not exceeding six months shall not
terminate employment. Article 286 provides,

ART. 286. When employment not deemed terminated. The bona fide
suspension of the operations of a business or undertaking for a period not
exceeding six (6) months, or the fulfillment by the employee of a military or civic
duty shall not terminate employment.
In all such cases, the employer shall reinstate the employee to his former
position without loss of seniority rights if he indicates his desire to resume his
work not later than one (1) month from the resumption of operations of his
employer or from his relief from the military or civic duty.

3E Andaya Ching Espiritu Hefti Galvez Gammad Lainez Lui Madamba Nagera Narvasa
Ong Palangdao Rosales Sanchez Santos Satrain Tabo (2014-2015)

Labor Relations Case Digest - Atty. Joyrich Golangco

The NLRC correctly noted that the complaint for illegal dismissal filed by
respondents was premature since it was filed only eight days after petitioner
announced that it will suspend its operations for six months. In Nippon Housing
Phil., Inc. v. Leynes,28 we said that a complaint for illegal dismissal filed prior to
the lapse of said six months is generally considered as prematurely filed.

On 27 January 2006, respondent filed a complaint for underpayment of wages,


non-payment of legal and special holiday pay, premium pay for rest day and
underpayment of ECOLA before DOLE. The hearing officer recommended the
dismissal of the complaint since the claims were already paid.

Under Article 286 of the Labor Code, the bona fide suspension of the operation of
a business or undertaking for a period not exceeding six months shall not
terminate employment. Consequently, when the bona fide suspension of the
operation of a business or undertaking exceeds six months, then the employment
of the employee shall be deemed terminated. By the same token and applying
said rule by analogy, if the employee was forced to remain without work or
assignment for a period exceeding six months, then he is in effect constructively
dismissed.

Petitioner denied dismissing respondent. It admitted that it relieved respondent


from his last assignment on 10 December 2005; however, on 27 January 2006 it
sent respondent a notice requiring him to report to the head office within 72
hours from receipt of said notice. It further alleged that it had informed
respondent that he had been absent without official leave for the month of
January 2006, and that his failure to report within 72 hours from receipt of the
notice would mean that he was no longer interested to continue his employment.

In this case, however, we agree with the Labor Arbiter and the CA that
respondents were already considered illegally dismissed since petitioner failed
to recall them after six months, when its bona fide suspension of operations
lapsed. We stress that under Article 286 of the Labor Code, the employment will
not be deemed terminated if the bona fide suspension of operations does not
exceed six months. But if the suspension of operations exceeds six months, the
employment will be considered terminated. In Valdez v. NLRC,29 we explained:

On 16 June 2006, respondent filed a complaint for illegal dismissal and payment
of separation pay against petitioner before the Conciliation and Mediation Center
of NLRC. On 14 July 2006, respondent filed another complaint for illegal
dismissal, underpayment of salaries and non-payment of full backwages before
the NLRC. He claimed that on various dates, he went to petitioners office to
follow-up his next assignment, but still he was not given a new assignment. He
argued that if an employee is on floating status for more than six months, such
employee is deemed illegally dismissed.

214. EMERITUS SECURITY AND MAINTENANCE SYSTEMS, INC. vs. JANRIE C.


DAILIG

LA: declared that complainant has been illegally dismissed; ordered the company
to reinstate complainant and to pay him backwages.

DOCTRINES

A floating status of a security guard, such as respondent, for more than


six months constitutes constructive dismissal.
Reinstatement is the general rule, while the award of separation pay is
the exception.

FACTS: Respondent was employed as a security guard assigned to petitioner's


various clients, the last of which was Panasonic. On 10 December 2005,
respondent was relieved from his post.

NLRC: affirmed LAs decision; denied MR. But it pointed out that computation of
backwages should be reckoned from 10 June 2006 and not 10 Dec 2005.
CA: affirmed LA and NLRC but set aside the reinstatement order and instead
ordered the payment of separation pay, invoking the doctrine of strained
relations between the parties.
ISSUES
(1) Whether respondent was illegally dismissed by respondent; and
(2) If he was, whether respondent is entitled to separation pay, instead of
reinstatement.

3E Andaya Ching Espiritu Hefti Galvez Gammad Lainez Lui Madamba Nagera Narvasa
Ong Palangdao Rosales Sanchez Santos Satrain Tabo (2014-2015)

Labor Relations Case Digest - Atty. Joyrich Golangco

HELD
(1) YES. Respondent was on floating status from 10 December 2005 to 16 June
2006 or more than six months. Petitioners allegation of sending respondent a
notice requiring him to report for work, is unsubstantiated, and thus, selfserving.

The Court agrees with the ruling of the LA, NLRC and CA that a floating status of a
security guard, such as respondent, for more than six months constitutes
constructive dismissal. In Nationwide Security and Allied Services, Inc. v.
Valderama,8 the Court HELD:
x x x the temporary inactivity or "floating status" of security guards should
continue only for six months. Otherwise, the security agency concerned could be
liable for constructive dismissal. The failure of petitioner to give respondent a
work assignment beyond the reasonable six-month period makes it liable for
constructive dismissal. x x x
(2) NO. Article 279 of the Labor Code of the Philippines mandates the
reinstatement of an illegally dismissed employee. Thus, reinstatement is the
general rule, while the award of separation pay is the exception. (The
circumstances warranting the grant of separation pay, in lieu of reinstatement,
are laid down by the Court in Globe-Mackay Cable and Radio Corporation v.
NLRC, thus: Over time, the following reasons have been advanced by the Court
for denying reinstatement under the facts of the case and the law applicable
thereto; that reinstatement can no longer be effected in view of the long passage
of time or because of the realities of the situation; or that it would be inimical to
the employers interest; or that reinstatement may no longer be feasible; or, that
it will not serve the best interests of the parties involved; or that the company
would be prejudiced by the workers continued employment; or that it will not
serve any prudent purpose as when supervening facts have transpired which
make execution on that score unjust or inequitable or, to an increasing extent,
due to the resultant atmosphere of antipathy and antagonism or strained
relations or irretrievable estrangement between the employer and the
employee.)
In this case, petitioner claims that it complied with the reinstatement order of
the Labor Arbiter. On 23 January 2008, petitioner sent respondent a notice
informing him of the Labor Arbiters decision to reinstate him. Accordingly, in

February 2008, respondent was assigned by petitioner to Canlubang Sugar


Estate, Inc. and to various posts thereafter.

However, respondent points out that he was not reinstated by petitioner


Emeritus Security and Maintenance Systems, Inc. but was employed by another
company, Emme Security and Maintenance Systems, Inc. (Emme). xxx
Considering petitioner's undisputed claim that Emeritus and Emme are one and
the same, there is no basis in respondent's allegation that he was not reinstated
to his previous employment.

Considering that (1) petitioner reinstated respondent in compliance with the


LAs decision, and (2) there is no ground, particularly strained relations between
the parties, to justify the grant of separation pay, the Court of Appeals erred in
ordering the payment thereof, in lieu of reinstatement.

LA decision is reinstated BUT backwages should be computed from 10 June 2006


when respondent was illegally dismissed up to the time he was reinstated in
February 2008.
215. Nippon Housing Phil. Inc v Maiah Angela Leynes

DOCTRINE: that "off-detailing" is not equivalent to dismissal, so long as such


status does not continue beyond a reasonable time and that it is only when such
a "floating status" lasts for more than six months that the employee may be
considered to have been constructively dismissed.

FACTS: Petitioner NHPI ventured into building management, providing such


services as handling of the lease of condominium units, collection of dues and
compliance with government regulatory requirements. Having gained the Bay
Gardens Condominium Project of the Bay Gardens Condominium Corporation
(BGCC) as its first and only building maintenance client, NHPI hired respondent
Maiah Angela Leynes as Property Manager. Tasked with surveying the
requirements of the government and the client for said project, the formulation
of house rules and regulations and the preparation of the annual operating and
capital expenditure budget, Leynes was also responsible for the hiring and
deployment of manpower, salary and position determination as well as the

3E Andaya Ching Espiritu Hefti Galvez Gammad Lainez Lui Madamba Nagera Narvasa
Ong Palangdao Rosales Sanchez Santos Satrain Tabo (2014-2015)

Labor Relations Case Digest - Atty. Joyrich Golangco

assignment of the schedules and responsibilities of employees. Later on, Leynes


had a misunderstanding with Engr. Cantuba, building engineer, regarding the
extension of the latters working hours, they instructed the security guards to bar
Cantuba from entering into the project and to tell him to report to NPHIs main
office in Makati. Also, a letter was sent to Cantuba through the HRD apprising
him of his insubordination and disrespectful conduct. Cantubas reply in turn
accused Leynes of pride, conceit and poor managerial skills. VP decided on the
matter and said through a memorandum that such was a simple personal
difference and instructed Leynes to allow Cantuba to report back to work.
Disappointed, Leynes sought an emergency LOA and possible resignation. The
position of Property Manager was offered to Engr. Jose, however, Leynes sent a
letter to HRD expressing her intention to return to work and call off her planned
resignation. Upon learning of a having a substitute she filed a protest and
eventually she was asked to report to NPHIs main office while she was on
floating status. She then filed a complaint for illegal dismissal against NPHI
before the NLRC, while the case was pending the HRD of NPHI sent DOLE and
respondent a notice of termination of service due to redundancy, and Leynes was
then offered an amount for her unpaid wages, 13th month pay, tax refund and
SILP. LA rendered a decision that putting Leynes on a floating status was
tantamount to termination without just cause and due process. NLRC reversed
the decision. CA again reversed the decision and ruled that there was
constructive dismissal.
ISSUES: a) WON the petitioners acted in bad faith on placing respondent in a
floating status of which is tantamount to constructive dismissal b) WON there
was redundancy of position

HELD: a) No. Far from being the indication of bad faith the CA construed the
same to be, these factual antecedents suggest that NHPIs immediate hiring of
Engr. Jose as the new Property Manager for the Project was brought about by
Leynes own rash announcement of her intention to resign from her
position. Although she subsequently changed her mind and sent a letter by
telefax announcing the reconsideration of her planned resignation and her
intention to return to work, Leynes evidently had only herself to blame for
precipitately setting in motion the events which led to NHPIs hiring of her own
replacement. Acting on Leynes letter protesting against the hiring of her
replacement and reiterating her lack of intention to resign from her position, the

record, moreover, shows that NHPI simply placed her on floating status until
such time that another project could be secured for her.

Considering that even labor laws discourage intrusion in the employers


judgment concerning the conduct of their business, courts often decline to
interfere in their legitimate business decisions, absent showing of illegality, bad
faith or arbitrariness. Indeed, the right of employees to security of tenure does
not give them vested rights to their positions to the extent of depriving
management of its prerogative to change their assignments or to transfer them.
The rule is settled, however, that "off-detailing" is not equivalent to dismissal, so
long as such status does not continue beyond a reasonable time and that it is only
when such a "floating status" lasts for more than six months that the employee
may be considered to have been constructively dismissed. A complaint for illegal
dismissal filed prior to the lapse of said six-month and/or the actual dismissal of
the employee is generally considered as prematurely filed.

b) Yes. With no other client aside from BGCC for the building management side of
its business, we find that NHPI was acting well within its prerogatives when it
eventually terminated Leynes services on the ground of redundancy. One of the
recognized authorized causes for the termination of employment, redundancy
exists when the service capability of the workforce is in excess of what is
reasonably needed to meet the demands of the business enterprise. A redundant
position is one rendered superfluous by any number of factors, such as
overhiring of workers, decreased volume of business, dropping of a particular
product line previously manufactured by the company or phasing out of service
activity priorly undertaken by the business. It has been held that the exercise of
business judgment to characterize an employees service as no longer necessary
or sustainable is not subject to discretionary review where, as here, it is
exercised there is no showing of violation of the law or arbitrariness or malice on
the part of the employer. An employer has no legal obligation to keep more
employees than are necessary for the operation of its business.
216. MAYON HOTEL & RESTAURANT, PACITA O. PO and/or JOSEFA PO
LAM, petitioners,
vs. ROLANDO ADANA, CHONA BUMALAY, ROGER BURCE, EDUARDO
ALAMARES, AMADO ALAMARES, EDGARDO TORREFRANCA, LOURDES
CAMIGLA, TEODORO LAURENARIA, WENEFREDO LOVERES, LUIS GUADES,

3E Andaya Ching Espiritu Hefti Galvez Gammad Lainez Lui Madamba Nagera Narvasa
Ong Palangdao Rosales Sanchez Santos Satrain Tabo (2014-2015)

Labor Relations Case Digest - Atty. Joyrich Golangco

AMADO MACANDOG, PATERNO LLARENA, GREGORIO NICERIO, JOSE


ATRACTIVO, MIGUEL TORREFRANCA, and SANTOS BROOLA, respondents.

HELD:Of the sixteen employees, only the 7 employees filed a case for illegal
dismissal.

Due to the expiration and non-renewal of the lease contract for the rented space
occupied by the said hotel and restaurant at Rizal Street, the hotel operations of
the business were suspended on March 31, 1997. The operation of the
restaurant was continued in its new location at Elizondo Street, Legazpi City,
while waiting for the construction of a new Mayon Hotel & Restaurant at
Pearanda Street, Legazpi City. Only 9 of the 16 employees continued
working in the Mayon Restaurant at its new site.

Whether respondents are still working for petitioners is a factual question. And
the records are unequivocal that since April 1997, when petitioner Mayon
suspended its hotel operations and transferred its restaurant operations in
Elizondo Street, respondents have not been permitted to work for
petitioners.and were subsequently laid-off when the Elizondo Street operations
closed. Since then, respondents have not been permitted to work nor recalled,
even after the construction of the new premises at Pearanda Street and the
reopening of the hotel operations with the restaurant in this new site. Such
findings were not disputed.

FACTS:Petitioner Mayon Hotel & Restaurant is a single proprietor business


registered in the name of Pacita O. Po, whose mother, Josefa Po Lam, manages the
establishment. The hotel and restaurant employed about sixteen employees.

On various dates of April and May 1997, the 16 employees filed complaints for
underpayment of wages and other money claims against petitioners.

Executive Labor Arbiter rendered a Joint Decision in favor of the employees. The
Labor Arbiter awarded substantially all of respondents' money claims. The Labor
Arbiter also held that based on the evidence presented, Josefa Po Lam is the
owner/proprietor of Mayon Hotel & Restaurant and the proper respondent in
these cases.
On appeal, NLRC reversed the decision of the Labor Arbiter and all the
complaints were dismissed. MR was denied.
CA reversed the NLRC decision which had dismissed all of respondents'
complaints, and reinstated the Joint Decision of the Labor Arbiter which ruled
that respondents were illegally dismissed and entitled to their money claims. MR
was denied.

ISSUES: WON respondents were illegally dismissed or merely temporarily laidoff.

It is petitioners contention that when the illegal dismissal case was filed by
respondents on April 1997, they had as yet no cause of action.

As found by the LA: more than six months had lapsed without petitioner
having resumed operation of the hotel. After more than one year from the
temporary closure of Mayon Hotel and the temporary transfer to another site of
Mayon Restaurant, the building which petitioner Josefa alleged where the hotel
and restaurant will be transferred has been finally constructed and the same is
operated as a hotel with bar and restaurant nevertheless, none of respondents
herein who were employed at Mayon Hotel and Restaurant which was also
closed on April 30, 1998 was/were recalled by petitioner to continue their
services

It was a serious error that the NLRC did not inquire as to the legality of the
cessation of employment. Article 286 of the Labor Code is clear there is
termination of employment when an otherwise bona fide suspension of work
exceeds six (6) months. The cessation of employment for more than six months
was patent and the employer has the burden of proving that the termination was
for a just or authorized cause.

While the closure of the hotel operations in April of 1997 may have been
temporary, we hold that the evidence on record belie any claim of petitioners
that the lay-off of respondents on that same date was merely temporary. There

3E Andaya Ching Espiritu Hefti Galvez Gammad Lainez Lui Madamba Nagera Narvasa
Ong Palangdao Rosales Sanchez Santos Satrain Tabo (2014-2015)

Labor Relations Case Digest - Atty. Joyrich Golangco

is substantial evidence that petitioners intended the termination to be


permanent.
First, respondents filed the complaint for illegal dismissal immediately
after the closure of the hotel operations in Rizal Street, notwithstanding the
alleged temporary nature of the closure of the hotel operations.

Second, invoking Article 286 to assert that the employer-employee relationship


was merely suspended, petitioner made no mention of any intent to recall
these respondents to work upon completion of the new premises.
Third, The vehemence of petitioners' accusation of mismanagement against
respondents, is inconsistent with the desire to recall them to work.

Fourth, petitioners' memorandum on appeal averred that the case was filed
because of the fact that the source of their livelihood, whether legal or
immoral, was stopped on March 31, 1997, when the owner of the building
terminated the Lease Contract."

Fifth, petitioners had inconsistencies in their pleadings in referring to the


closure (temporary cessation of operations due to non-renewal of lease; closure
due to financial losses)
Sixth, the uncontroverted finding of the LA that petitioner terminated
respondents, by not employing them when they transferred to its new site on
Pearanda Street.

These factors may be inconclusive individually, but when taken together, they
lead us to conclude that petitioners really intended to dismiss all respondents
and merely used the termination of the lease (on Rizal Street premises) as a
means by which they could terminate their employees.
Moreover, even assuming arguendo that the cessation of employment on April
1997 was merely temporary, it became dismissal by operation of law when
petitioners failed to reinstate respondents after the lapse of six (6) months,
pursuant to Article 286 of the Labor Code.

While we recognize the right of the employer to terminate the services of an


employee for a just or authorized cause, the dismissal of employees must be
made within the parameters of law and pursuant to the tenets of fair play. Under
the circumstances, the award of damages was proper. As a rule, moral damages
are recoverable where the dismissal of the employee was attended by bad faith
or fraud or constituted an act oppressive to labor, or were done in a manner
contrary to morals, good customs or public policy. We believe that the dismissal
of the respondents was attended with bad faith and meant to evade the lawful
obligations imposed upon an employer.
217. Serrano vs Severino

Doctrine: Since employees paid on commission basis are not excluded from SIL
entitlement, the same must be included in computing their retirement benefit,
plus 1/12 of their 13th month pay and other benefits.

FACTS: Rodolfo Serrano was hired on September 28, 1992 as bus conductor by
Severino Santos Transit.After 14 years of service petitioner applied for optional
retirement from the company whose representative advised him that he must
first sign the already prepared Quitclaim before his retirement pay could be
released. As petitioners request to first go over the computation of his
retirement pay was denied, he signed the Quitclaim on which he wrote U.P.
(under protest) after his signature, indicating his protest to the amount of
P75,277.45 which he received, computed by the company at 15 days per year of
service.

Petitioner soon after filed a complaint before the Labor Arbiter. Petitioners
contention: company erred in its computation since under R.A. 7641,
(Retirement Pay Law), his retirement pay should have been computed at 22.5
days per year of service to include the cash equivalent of the 5-day service
incentive leave (SIL) and 1/12 of the 13th month pay which the company did not

Respondents contention: The Quitclaim signed by petitioner barred his claim


and, in any event, its computation was correct since petitioner was not entitled to
the 5-day SIL and pro rated 13th month pay for, as a bus conductor, he was paid
on commission basis. Noting that the retirement differential pay amounted to

3E Andaya Ching Espiritu Hefti Galvez Gammad Lainez Lui Madamba Nagera Narvasa
Ong Palangdao Rosales Sanchez Santos Satrain Tabo (2014-2015)

Labor Relations Case Digest - Atty. Joyrich Golangco

only P1,431.15, explained that in the computation of petitioners retirement pay,


five months were inadvertently not included because some index cards
containing his records had been lost.
LA: ruled in favor of petitioner, awarding him P116,135.45 as retirement pay
differential, and 10% of the total monetary award as attorneys fees
NLRC: reversed the Labor Arbiters ruling and dismissed petitioners complaint.
Citing R & E Transport, Inc. v. Latag, the NLRC held that since petitioner was paid
on purely commission basis, he was excluded from the coverage of the laws on
13th month pay and SIL pay, hence, the 1/12 of the 13th month pay and the 5day SIL should not be factored in the computation of his retirement pay.

ISSUES: Whether or not petitioners SIL and pro-rated 13th month pay should be
included in the computation of his retirement benefit, considering that he is paid
on a commission basis.
HELD: Yes.R.A. 7641 which was enacted on December 9, 1992 amended Article
287 of the Labor Code by providing for retirement pay to qualified private sector
employees in the absence of any retirement plan in the establishment. The
pertinent provision of said law reads:
Section 1. Article 287 of Presidential Decree No. 442, as amended, otherwise
known as the
Labor Code of the Philippines, is hereby amended to read as follows:
xxxx
In the absence of a retirement plan or agreement providing for retirement
benefits of employees in the establishment, an employee upon reaching the
age of sixty (60) years or more, but not beyond sixtyfive (65) years which is
hereby declared the compulsory retirement age, who has served at least five
(5) years in the said establishment, may retire and shall be entitled to
retirement pay equivalent to at least onehalf (1/2) month salary for every
year of service, a fraction of at least six
(6) months being considered as one whole year. Unless the parties provide
for broader inclusions, the term one-half (1/2) month salary shall mean
fifteen (15) days plus one-twelfth (1/12) of the 13th month pay and the cash
equivalent of not more than five (5) days of service incentive leaves. Retail,
service and agricultural establishments or operations employing not more

than (10) employees or workers are exempted from the coverage of this
provision. x x x x (emphasis and underscoring supplied) Further, the Implementing
Rules of said law provide:

RULE II Retirement Benefits


SECTION 1. General Statement on Coverage. This Rule shall apply to all
employees in the private sector, regardless of their position, designation or
status and irrespective of the method by which their wages are paid, except
to those specifically exempted under Section 2 hereof.
SECTION 2 Exemptions. This Rule shall not apply to the following employees:
2.1 Employees of the National Government and its political subdivisions,
including
Government-owned and/or controlled corporations, if they are covered by the Civil
Service
Law and its regulations.
2.2 Domestic helpers and persons in the personal service of another.
2.3 Employees of retail, service and agricultural establishment or
operations regularly employing not more than ten (10) employees. As used in
this sub-section;
xxxx
SECTION 5
Retirement Benefits.
5.1 In the absence of an applicable agreement or retirement plan, an
employee who retires pursuant to the Act shall be entitled to retirement pay
equivalent to at least one-half () month salary for every year of service, a fraction
of at least six (6) months being considered as one whole year.
5.2 Components of One-half () Month Salary. For the purpose of
determining the minimum retirement pay due an employee under this Rule, the
term one-half month salary shall include all of the following:
(a) Fifteen (15) days salary of the employee based on his latest salary rate. As
used herein, the term salary includes all remunerations paid by an employer
to his employees for services rendered during normal working days and
hours, whether such payments are fixed or ascertained on a time, task, piece
of commission basis, or other method of calculating the same, and includes the
fair and reasonable value, as determined by the Secretary of Labor and
Employment, of food, lodging or other facilities customarily furnished by the

3E Andaya Ching Espiritu Hefti Galvez Gammad Lainez Lui Madamba Nagera Narvasa
Ong Palangdao Rosales Sanchez Santos Satrain Tabo (2014-2015)

Labor Relations Case Digest - Atty. Joyrich Golangco

employer to his employees. The term does not include cost of living allowances,
profit-sharing payments and other monetary benefits which are not considered as
part of or integrated into the regular salary of the employees.
(b) The cash equivalent of not more than five (5) days of service incentive
leave;
(c) One-twelfth of the 13th month pay due the employee.
(d) All other benefits that the employer and employee may agree upon that should
be included in the computation of the employees retirement pay. x x x x (emphasis
supplied)
Admittedly, petitioner worked for 14 years for the bus company which did
not adopt any retirement scheme. Even if petitioner as bus conductor was
paid on commission basis then, he falls within the coverage of R.A. 7641
and its implementing rules. As thus correctly ruled by the Labor Arbiter,
petitioners retirement pay should include the cash equivalent of the 5-day SIL
and 1/12 of the 13th month pay. The affirmance by the appellate court of the
reliance by the NLRC on R & E Transport, Inc. is erroneous. In said case, the Court
held that a taxi driver paid according to the boundary system is not entitled to
the 13th month and the SIL pay, hence, his retirement pay should be computed
on the sole basis of his salary.
For purposes, however, of applying the law on SIL, as well as on retirement, the
Court notes that there is a difference between drivers paid under the boundary
system and conductors who are paid on commission basis. In practice, taxi
drivers do not receive fixed wages. They retain only those sums in excess of the
boundary or fee they pay to the owners or operators of the vehicles.
Conductors, on the other hand, are paid a certain percentage of the bus earnings
for the day. It bears emphasis that under P.D. 851 or the SIL Law, the exclusion
from its coverage of workers who are paid on a purely commission basis is only
with respect to field personnel.

The more recent case of Auto Bus Transport Systems, Inc., v. Bautista clarifies that
an employee who is paid on purely commission basis is entitled to SIL:
The same is true with respect to the phrase those who are engaged on task
or contract basis, purely commission basis. Said phrase should be related
with field personnel, applying the rule on ejusdem generis that general and

unlimited terms are restrained and limited by the particular terms that they follow.
Hence, employees engaged on task or contract basis or paid on purely
commission basis are not automatically exempted from the grant of service
incentive leave, unless, they fall under the classification of field personnel.

Note: field personnel shall refer to non-agricultural employees who


regularly perform heir duties away from the principal place of business or
branch office of the employer and whose actual hours of work in the field
cannot be determined with reasonable certainty. (Art. 82 Labor Code) If
required to be at specific places at specific times, employees including drivers
cannot be said to be field personnel despite the fact that they are performing
work away from the principal office of the employee.

218. JEROME M. DAABAY, PETITIONER, vs. COCA-COLA BOTTLERS PHILS.,


INC., RESPONDENT.

DOCTRINE: Even private respondents assertion that, at the time of her lawful
dismissal, she was already qualified for retirement does not aid her case because
the fact remains that private respondent was already terminated for cause
thereby rendering nugatory any entitlement to mandatory or optional
retirement pay that she might have previously possessed.
And although retirement benefits, where not mandated by law, may still be
granted by agreement of the employees and their employer or as a voluntary act
of the employer,38 there is no proof that any of these incidents attends the instant
case.
FACTS: The case stems from a complaint for illegal dismissal, illegal suspension,
unfair labor practice and monetary claims filed by Daabay against respondent
and three officers of the company.4The employment of Daabay with Coca-Cola as
Sales Logistics Checker was terminated by the company following receipt of
information from one Cesar Sorin that Daabay was part of a conspiracy that
allowed the pilferage of company property.6

3E Andaya Ching Espiritu Hefti Galvez Gammad Lainez Lui Madamba Nagera Narvasa
Ong Palangdao Rosales Sanchez Santos Satrain Tabo (2014-2015)

Labor Relations Case Digest - Atty. Joyrich Golangco

Coca-Cola then served upon Daabay a Notice to Explain with Preventive


Suspension, which required him to explain in writing his participation in the
scheme. Daabay submitted an Explanation wherein he denied any participation
in the reported pilferage.9
A formal investigation on the matter ensued. Eventually, Coca-Cola served upon
Daabay a Notice of Termination that cited pilferage, serious misconduct and loss
of trust and confidence as grounds. At the time of his dismissal, Daabay had been
a regular employee of Coca-Cola for eight years.

Daabay then filed the subject labor complaint. Executive Labor Arbiter (ELA)
rendered his Decision12 in favor of Daabay who was illegally dismissed
because his participation in the alleged conspiracy was not proved by substantial
evidence. ELA ordered the payment to Daabay of backwages and separation pay
or retirement benefits, as may be applicable.

Respondents appealed ELAs Decision to the NLRC. Daabay filed a separate


appeal to ask for his reinstatement without loss of seniority rights, the payment
of backwages instead of separation pay or retirement benefits, and an award of
litigation expenses, moral and exemplary damages and attorneys fees. The NLRC
reversed the finding of illegal dismissal. It held that several documents such
as checkers receipts and sales invoices that made the fraudulent scheme possible
were signed by Daabay.16 The NLRC also found fault in Daabay for his failure to
detect the pilferage, considering that the "timely recording and monitoring as
security control for the outgoing [sic] of company products are necessarily
connected with the functions, duties and responsibilities reposed in him as Sales
Logistics Checker."17 Notwithstanding its ruling on the legality of the dismissal,
the NLRC awarded retirement benefits in favor of Daabay.
The CA agreed with Coca-Cola that the award of retirement benefits lacked
basis considering that Daabay was dismissed for just cause.
ISSUES: Whether or not Daabay should be entitled to the retirement benefits that
were awarded by the NLRC

HELD: NO. Daabay was declared by the NLRC to have been lawfully dismissed by
Coca-Cola on the grounds of serious misconduct, breach of trust and loss of
confidence. In the case of Philippine Airlines, Inc. v. NLRC31 on the issue of
whether an employee who is dismissed for just cause may still claim retirement
benefits equally applies to this case; We HELD: Even private respondents
assertion that, at the time of her lawful dismissal, she was already qualified for
retirement does not aid her case because the fact remains that private
respondent was already terminated for cause thereby rendering nugatory any
entitlement to mandatory or optional retirement pay that she might have
previously possessed.32

Financial assistance, or whatever name it is called, as a measure of social justice


is allowed only in instances where the employee is validly dismissed for causes
other than serious misconduct or those reflecting on his moral character.34 A
contrary rule would have the effect of rewarding rather than punishing the
erring employee for his offense. Considering that Daabay was dismissed on the
grounds of serious misconduct, breach of trust and loss of confidence, the award
based on equity was unwarranted.

Even the NLRCs reliance on the alleged admission by Coca-Cola in its motion to
reduce bond that Daabay is entitled to retirement benefits is misplaced. The
supposed admission by Coca-Cola was based on the following: xxx In support of
its motion to reduce bond, Coca-cola seeks leniency for its failure to include in
the posting of the bond the monetary award for [Daabays] retirement benefits
which, as directed by the ELA, should be computed in accordance with the latest
CBA prior to his termination. Coca-Cola explains that the amount of the
retirement benefits has not been determined and there is a need to compute the
same on appeal. x x x.37
It is patent that the statements made by Coca-Cola were in light of ELA ruling
that Daabay was illegally dismissed. Furthermore, any admission was only for
the purpose of explaining the non-inclusion of the amount of retirement benefits
in the computation of the appeal bond posted with the NLRC. Coca-Colas
statements should be taken in such context, and could not be deemed to bind the
company even after the NLRC had reversed the finding of illegal dismissal. And
although retirement benefits, where not mandated by law, may still be granted

3E Andaya Ching Espiritu Hefti Galvez Gammad Lainez Lui Madamba Nagera Narvasa
Ong Palangdao Rosales Sanchez Santos Satrain Tabo (2014-2015)

Labor Relations Case Digest - Atty. Joyrich Golangco

by agreement of the employees and their employer or as a voluntary act of the


employer,38 there is no proof that any of these incidents attends the instant case.
219. Piero vs. NLRC

DOCTRINE: An employee who is dismissed for cause is generally not entitled to


any financial assistance. Equity considerations, however, provide an exception.
Equity has been defined as justice outside law, being ethical rather than jural and
belonging to the sphere of morals than of law. It is grounded on the precepts of
conscience and not on any sanction of positive law, for equity finds no room for
application where there is law.

FACTS: Private respondent Dumaguete Cathedral College, Inc., an educational


institution, is the employer of the faculty and staff members comprising the labor
union DUCACOFSA-NAFTEU. On December 19, 1986, DUCACOFSA (then
affiliated with the National Alliance of Teachers and Allied Workers NATAW)
and private respondent entered into a Collective Bargaining Agreement (CBA)
effective for 3 years. Upon the expiration of their CBA in 1989, the parties failed
to conclude another CBA which led DUCACOFSA (now affiliated with NAFTEU) to
file a notice of strike with the Department of Labor and Employment (DOLE) on
the ground of refusal to bargain. On November 4, 1991, DUCACOFSA-NAFTEU
conducted a strike in the premises of private respondent without submitting to
the DOLE the required results of the strike vote obtained from the members of
the union. Private respondent filed a complaint to declare the strike illegal and
dismiss the union officers including herein petitioner who was the president of
the union.
LA rendered a decision in favor of private respondent declaring the strike illegal
and the union officers to have lost their employment status effective October
28,1994 (the date of the decision).
Pending the NLRC appeal, the officers were allowed to return to work by virtue
of a memorandum of agreement entered into between the union and private
respondent without prejudice to the outcome of the appeal.
NLRC affirmed the LA decision and added that the union had no personality to
hold a strike because it was not a legitimate labor organization.

CA affirmed the decision. Piero brought the petition before the SC.

ISSUES: WON the strike was illegal, If yes, should Piero be dismissed

HELD: YES, The union is a legitimate labor organization as settled by the NLRC
decision in Case No. V-0432-93, however the union failed to hold a strike vote
and submit its result to the DOLE prior to the holding of a strike. In the case at
bar, DUCACOFSA-NAFTEU failed to prove that it obtained the required strikevote among its members and that the results thereof were submitted to the
DOLE.

The strike was therefore correctly declared illegal, for non-compliance


with the procedural requirements of Article 263 of the Labor Code, and Piero
properly dismissed from service. Pursuant to Article 264 of the Labor Code, any
union officer who knowingly participates in an illegal strike and any worker or
union officer who knowingly participates in the commission of illegal acts during
a strike may be declared to have lost his employment status.

The Court notes that petitioner Piero turned 60 years old and retired
on March 1, 1996 after 29 years of service, rendering his dismissal from service
moot and academic. However, in view of the propriety of his termination as a
consequence of the illegal strike, he is no longer entitled to payment of
retirement benefits because he lost his employment status effective as of the date
of the decision of the Labor Arbiter October 28, 1994. Although meriting
termination of employment, Pieros infraction is not so reprehensible nor
unscrupulous as to warrant complete disregard of his long years of
service. Moreover, he has no previous derogatory records. Weighed on the
scales of justice, conscience and reason tip in favor of granting financial
assistance to support him in the twilight of his life after long years of service.
Under the circumstances, social and compassionate justice dictate that
petitioner Piero be awarded financial assistance equivalent to one-half (1/2)
months pay for every year of service computed from his date of employment up
to October 28, 1994 when he was declared to have lost his employment
status. Indeed, equities of this case should be accorded due weight because labor
law determinations are not only secundum rationem but also secundum
caritatem.

3E Andaya Ching Espiritu Hefti Galvez Gammad Lainez Lui Madamba Nagera Narvasa
Ong Palangdao Rosales Sanchez Santos Satrain Tabo (2014-2015)

Labor Relations Case Digest - Atty. Joyrich Golangco

220. Sta. Catalina College and Sr. Lorenzo Oranza vs NLRC and Hilaria
Tercero

DOCTRINE: Private respondents re-employment as a new employee would


mean a demotion in rank and privileges, retirement benefits, for example, as her
entire previous fifteen (15) years of service with petitioner, would simply be
considered as non-existent.
FACTS: In June 1955, Hilaria was hired as an elementary school teacher at the
Sta. Catalina College in Bian, Laguna. In 1970, she applied for and was granted a
one year leave of absence without pay on account of the illness of her
mother. After the expiration in 1971 of her leave of absence, she had not been
heard from by petitioner school.

In the meantime, she was employed as a teacher at the San Pedro


Parochial School during school year 1980-1981 and at the Liceo de San Pedro,
Bian, Laguna during school year 1981-1982. In 1982, she applied anew at
petitioner school which hired her with a monthly salary of P6,567.95.
On March 22, 1997, during the 51st Commencement Exercises of petitioner
school, Hilaria was awarded a Plaque of Appreciation for thirty years of service
and P12,000.00 as gratuity pay.

On May 31, 1997, Hilaria reached the compulsory retirement age of


65. Retiring pursuant to Article 287 of the Labor Code, as amended by Republic
Act 7641, petitioner school pegged her retirement benefits at P59,038.35,
computed on the basis of fifteen years of service from 1982 to 1997. Her
service from 1955 to 1970 was excluded in the computation, petitioner school
having asserted that she had, in 1971, abandoned her employment.

From the P59,038.35 retirement benefits was deducted the amount


of P28,853.09 representing reimbursement of the employers contribution to her
retirement benefits under the Private Education Retirement Annuity Association
(PERAA) which Hilaria had already received. Deducted too was the amount
of P12,000.00 representing the gratuity pay which was given to her. The
remaining balance of the retirement benefits due her thus amounted
to P18,185.26.

Hilaria insisted, however, that her retirement benefits should be


computed on the basis of her thirty years of service, inclusive of the period from
1955 to 1970; and that the gratuity pay earlier given to her should not be
deducted therefrom. She thus concluded that she was entitled to P190,539.90.
However the petitioner school argued that after 1971, Hilaria abandoned
her job for not re-applying as a teacher after the expiration of her leave of
absence. Hence her retirement benefits should only be computed from 1982, the
year when she went back to teach in said school.
Hence the case filed in the NLRC for non-payment of retirement benefits.

LA: Sustained the Schools petitioner.

NLRC: Decided that the retirement benefits should be computed from 1955.

CA: Upheld the NLRC ruling stating that petitioners failed to prove that Hilaria
abandoned her job from 1971 onwards, hence entitling her to receive the
retirement benefits to be computed from 1955.
ISSUES:
1955

W/N Hilarias retirement benefit should be computed from

HELD: No. Hilaria cannot be credited for her services in 1955-1970 in the
determination of her retirement benefits. For, after her one year leave of
absence expired in 1971 without her requesting for extension thereof, as in fact
she had not been heard from until she resurfaced in 1982 when she reapplied
with petitioner school, she abandoned her teaching position as in fact she was
employed elsewhere in the interim and effectively relinquished the retirement
benefits accumulated during the said period.

For a valid finding of abandonment, two factors must be present: (1) the
failure to report for work, or absence without valid or justifiable reason; and (2)
a clear intention to sever employer-employee relationship, with the second
element as the more determinative factor, being manifested by some overt acts.

3E Andaya Ching Espiritu Hefti Galvez Gammad Lainez Lui Madamba Nagera Narvasa
Ong Palangdao Rosales Sanchez Santos Satrain Tabo (2014-2015)

Labor Relations Case Digest - Atty. Joyrich Golangco

It is not disputed that the approved one year leave of absence without pay
of Hilaria expired in 1971, without her, it bears repeating, requesting for
extension thereof or notifying petitioner school if and when she would resume
teaching. Nor is it disputed that she was rehired only in 1982 after filing anew an
application, without her proffering any explanation for her more than a decade of
absence. Under the circumstances, abandonment of work at petitioner
school in 1971 is indubitably manifest.
That Hilaria was in 1997 given a plaque of appreciation for thirty years of
service to the school and awarded P12,000.00 as gratuity pay should not be
taken against petitioners, for acknowledgment of the total number of years of her
service, which was discontinuous, should not obliterate the fact that she
abandoned her employment in 1971, albeit she was rehired in 1982.
221. PANTRANCO NORTH EXPRESS, INC. v. NATIONAL LABOR RELATIONS
COMMISSION and URBANO SUIGA

DOCTRINE: Providing in a CBA for compulsory retirement for employees earlier


than the optional or mandatory retirement age is legal and enforceable so long as
the parties agree to be governed by such CBA. The law presumes that employees
know what they want and what is good for them absent any showing that fraud
or intimidation was employed to secure their consent thereto.
FACTS: Private respondent was hired by petitioner in 1964 as a bus conductor.
He eventually joined the Pantranco Employees Association-PTGWO. He
continued the petitioner's employ until August 12, 1989, when he was retired at
the age of fifty- two(52) after having rendered twenty five years' service. The
basis of his retirement was the compulsory retirement provision of the collective
bargaining agreement between the petitioner and the aforenamed union. Private
respondent received P49,300.00 as retirement pay.
Private respondent filed a complaint for illegal dismissal against petitioner with
the Sub-Regional Arbitration Branch of the respondent Commission in Dagupan
City.
LA & NLRC: Private respondent was illegally dismissed and ordered for his
reinstatement.

ISSUES:1. Who has jurisdiction over a case involving the question/dispute? The
labor arbiter or the arbitrators authorized by the CBA?
2. Is a Collective Bargaining Agreement provision allowing compulsory
retirement before age 60 but after twenty five years of service legal and
enforceable?

HELD: 1. Labor Arbiter. It cannot be said that the "dispute" is between the
union and petitioner company because both have previously agreed upon the
provision on "compulsory retirement" as embodied in the CBA. Also, it was only
private respondent on his own who questioned the compulsory retirement. Thus,
the case is properly denominated as a "termination dispute" which comes under
the jurisdiction of labor arbiters.
2. Private respondent's compulsory retirement is not illegal dismissal.

Compulsory retirement in the CBA: Upon reaching the age of sixty (60) years or
upon the completing twenty-five (25) years of service to the COMPANY,
whichever comes first, and the employee shall be compulsorily retired and paid
the retirement benefits herein provided.

Art. 287 of the Labor Code as worded permits employers and employees to fix
the applicable retirement age at below 60 years. Moreover, providing for early
retirement does not constitute diminution of benefits. In almost all countries
today, early retirement, i.e., before age 60, is considered a reward for services
rendered since it enables an employee to reap the fruits of his labor
particularly retirement benefits, whether lump-sum or otherwise at an earlier
age, when said employee, in presumably better physical and mental condition,
can enjoy them better and longer. As a matter of fact, one of the advantages of
early retirement is that the corresponding retirement benefits, usually consisting
of a substantial cash windfall, can early on be put to productive and profitable
uses by way of income-generating investments, thereby affording a more
significant measure of financial security and independence for the retiree who,
up till then, had to contend with life's vicissitudes within the parameters of his
fortnightly or weekly wages. Thus we are now seeing many CBA's with such early
retirement provisions. And the same cannot be considered a diminution of
employment benefits.

3E Andaya Ching Espiritu Hefti Galvez Gammad Lainez Lui Madamba Nagera Narvasa
Ong Palangdao Rosales Sanchez Santos Satrain Tabo (2014-2015)

Labor Relations Case Digest - Atty. Joyrich Golangco

A CBA incorporates the agreement reached after negotiations between employer


and bargaining agent with respect to terms and conditions of employment. A CBA
is not an ordinary contract. As a labor contract within the contemplation of
Article 1700 of the Civil Code of the Philippines which governs the relations
between labor and capital, it is not merely contractual in nature but impressed
with public interest, thus it must yield to the common good. As such, it must be
construed liberally rather than narrowly and technically, and the courts must
place a practical and realistic construction upon it, giving due consideration to
the context in which it is negotiated and purpose which it is intended to serve.
Being a product of negotiation, the CBA between the petitioner and the union
intended the provision on compulsory retirement to be beneficial to the
employees-union members, including herein private respondent. When private
respondent ratified the CBA with the union, he not only agreed to the CBA but
also agreed to conform to and abide by its provisions. Thus, it cannot be said that
he was illegally dismissed when the CBA provision on compulsory retirement
was applied to his case.

Republic Act No. 7641, known as "The Retirement Pay Law," which went
into effect on January 7, 1993, although passed many years after the
compulsory retirement of herein private respondent, nevertheless sheds light on
the present discussion when it amended Art. 287 of the Labor Code, to make it
read as follows:

"Art. 287.
Retirement Any employee may be retired upon reaching the
retirement age established in the collective bargaining agreement or other
applicable employment contract. xxx
xxx
xxx
In the absence of a retirement plan or agreement providing for retirement
benefits of employees in the establishment, an employee upon reaching the age
of sixty (60) years or more, but not beyond sixty-five
(65) years which is
hereby declared the compulsory retirement age, who has served at least five (5)
years in the said establishment may retire"
.
.
.
.

The aforequoted provision makes clear the intention of the spirit of the law to
give employers and employees a free hand to determine and agree upon the
terms and conditions of retirement. Providing in a CBA for compulsory
retirement of employees after twenty-five (25) years of service is legal and

enforceable so long
as the parties agree to be governed by such CBA. The
law presumes that employees know what they want and what is good for them
absent any showing that fraud or intimidation was employed to secure their
consent thereto.

On this point then, public respondent committed a grave abuse of discretion in


affirming the decision of the labor arbiter. The compulsory retirement of private
respondent effected in accordance with the CBA is legal and binding.
222. R and E Transport vs Latag
DOCTRINE: Taxi drivers do not receive fixed wages, but retain only those sums
in excess f the boundary or fee they pay to the owners or operators of their
vehicles.[34] Thus, the basis for computing their benefits should be
the average daily income
FACTS: Pedro Latag was a regular employee of La Mallorca Taxi since March 1,
1961. When the company ceased its operations and was transferred to R&E
Transport Inc. Latag is paid under the boundary system and was receiving an
average daily salary of 500.00 as a taxi driver. In 1995 he got sick and was forced
to apply for partial disability with the SSS which was granted. When he
recovered on September 1998 and reported back to work but he was no longer
allowed to continue on account of his old age. With this he asked the
administrative officer of petitioner (R&E) for his retirement pay pursuant to RA
7641 but was ignored. Thereafter he filed a case for payment of his retirement
pay before the NLRC. He however died on April 30, 1999 and his wife (Avelina
Latag) substituted him.
LA: Pay Avelina Latag retirement pay of his deceased husband in the amount of
277,500.00 for 37 years of working, 23 years under La Mallorca and 14 years
under R&E and finding that the 2 companies is one and the same entity.

Thereafter R&E offered respondent (Avelina Latag) 38,500 which she accepted
and thereafter she was made to sign a prepared quitclaim and release and joint
motion to dismiss the case. After a day or 2 the respondent received a copy of the
decision of the LA. On June 24 2000 R&E filed a quitclaim and motion to dismiss

3E Andaya Ching Espiritu Hefti Galvez Gammad Lainez Lui Madamba Nagera Narvasa
Ong Palangdao Rosales Sanchez Santos Satrain Tabo (2014-2015)

Labor Relations Case Digest - Atty. Joyrich Golangco

however the LA issued a decision stating: WHEREFORE, the decision stands and
the Labor Arbitration Associate of this Office is directed to prepare the Writ of
Execution in due course.
NLRC: dismissed for failure to post cash or surety bond: MR modified the
decision of LA, the retirement pay should only be based on 14 years or the years
worked under R&E
CA: affirmed LA, 37 Years

ISSUES:
1. How many years should be used in computing the retirement pay
2. Whether or not the doctrine of piercing the corporate veil has application to
the case
3. Whether or not the quitclaim is valid

HELD:
1. 14 years only, After a careful and painstaking review of the evidence on
record, it was found out that The labor arbiters conclusion -- that Mallorca Taxi
and R & E Transport, Inc., are one and the same entity -- is negated by the
documentary evidence presented by petitioners.
1) R & E Transport, Inc., was established only in 1978;
2) Honorio Enriquez, its president, was not a stockholder of La Mallorca
Taxi;
3) none of the stockholders of the La Mallorca hold stocks in the R&E.
In the face of such evidence, it must be ruled that R&E and La Mallorca is not
of the same entity and the respondent is only entitled to 14 years of retirement
benefits.
2. No. basic is the rule that the corporate veil may be pierced only if it becomes a
shield for fraud, illegality or inequity committed against a third person.
In Philippine National Bank v. Andrada Electric & Engineering Company, we said:
x x x Any application of the doctrine of piercing the corporate veil
should be done with caution. A court should be mindful of the milieu where it is
to be applied. It must be certain that the corporate fiction was misused to such an
extent that injustice, fraud, or crime was committed against another, in disregard

of its rights. The wrongdoing must be clearly and convincingly established; it


cannot be presumed. Otherwise, an injustice that was never unintended may
result from an erroneous application. Xxx

Piercing the veil of corporate fiction may be allowed only if the following
elements concur:
(1) control -- not mere stock control, but complete domination -- not only of
finances, but of policy and business practice in respect to the transaction
attacked, must have been such that the corporate entity as to this transaction had
at the time no separate mind, will or existence of its own;
(2) such control must have been used by the defendant to commit a fraud or a
wrong to perpetuate the violation of a statutory or other positive legal duty, or a
dishonest and an unjust act in contravention of plaintiffs legal right; and
(3) the said control and breach of duty must have proximately caused the injury
or unjust loss complained of.

Respondent has not shown by competent evidence that one taxi company
had stock control and complete domination over the other or vice versa. In fact,
no evidence was presented to show the alleged renaming of La Mallorca Taxi to
R & E Transport, Inc. The seven-year gap between the time the former closed
shop and the date when the latter came into being also casts doubt on any
alleged intention of petitioners to commit a wrong or to violate a statutory duty.
This lacuna in the evidence compels us to reverse the Decision of the CA
affirming the labor arbiters finding of fact that the basis for computing Pedros
retirement pay should be 37 years, instead of only 14 years.

3. No, Courts have stepped in to annul questionable transactions, especially


where there is clear proof that a waiver, for instance, was wangled from an
unsuspecting or a gullible person; or where the agreement or settlement was
unconscionable on its face. A quitclaim is ineffective in barring recovery of the full
measure of a workers rights, and the acceptance of benefits therefrom does not
amount to estoppel. Moreover, a quitclaim in which the consideration is
scandalously low and inequitable cannot be an obstacle to the pursuit of a
workers legitimate claim.

3E Andaya Ching Espiritu Hefti Galvez Gammad Lainez Lui Madamba Nagera Narvasa
Ong Palangdao Rosales Sanchez Santos Satrain Tabo (2014-2015)

Labor Relations Case Digest - Atty. Joyrich Golangco

Art. 287. Retirement. - x x x In the absence of a retirement plan or agreement


providing for retirement benefits of employees in the establishment, an
employee upon reaching the age of sixty (60) years or more, but not beyond
sixty-five (65) years which is hereby declared the compulsory retirement age,
who has served at least five (5) years in said establishment, may retire and shall
be entitled to retirement pay equivalent to at least one-half (1/2) month salary for
every year of service, a fraction of at least six (6) months being considered as one
whole year.
Unless the parties provide for broader inclusions, the term one half-month
salary shall mean fifteen (15) days plus one-twelfth (1/12) of the 13th month pay
and the cash equivalent of not more than five (5) days of service incentive leaves.
(Italics supplied)

Since Pedro was paid according to the boundary system, he is not entitled
to the 13th month and the service incentive pay. hence, his retirement pay should
be computed on the sole basis of his salary.
It is accepted that taxi drivers do not receive fixed wages, but retain only
those sums in excess of the boundary or fee they pay to the owners or
operators of their vehicles. Thus, the basis for computing their benefits should be
the average daily income. In this case, the CA found that Pedro was earning an
average of five hundred pesos (P500) per day. We thus compute his retirement pay
as follows: P500 x 15 days x 14 years of service equals P105,000. Compared with
this amount, the P38,850 he received, which represented just over one third of
what was legally due him, was unconscionable.

from government service and paid her retirement gratuity. Obusan continued
her employment at PNB, now a privately organized bank. PNB, through a Board
Resolution approved the PNB Regular Retirement Plan (PNB-RRP) duly
recognized by the Philnabank Employees Association, the union of PNB rankand-file employees in the CBA with PNB which provides: Section 1, Article VI.
Normal Retirement. The normal retirement date of a Member shall be the day he
attains sixty (60) years of age, regardless of length of service or has rendered
thirty (30) years of service, regardless of age, whichever of the said conditions
comes first. A Member who has reached the normal retirement date shall have to
compulsorily retire and shall be entitled to receive the retirement benefits under
the Plan. Obusan was informed of her last day of employment when she would
reach the mandatory retirement age of 60 years but Obusan questioned her
compulsory retirement and filed a complaint for illegal dismissal and unfair labor
practice claiming that she had a vested right to be retired only at 65 years old
pursuant to civil service regulations since this was the retirement age at the time
she was hired.
The Labor Arbiter dismissed the complaint upholding the validity of the PNBRRP and its provisions on compulsory retirement at age 60; PNB having ceased
to be a government corp. and the law now applicable to the bank is the Labor
Code which allows PNB to establish its own retirement plan provided it is in
accord with law.
The NLRC affirmed the assailed decision of the Labor Arbiter in toto and
dismissed the appeal.

DOCTRINE: Retirement plans allowing employers to retire employees who have


not yet reached the compulsory retirement age of 65 years are not per se
repugnant to the constitutional guaranty of security of tenure.

The CA dismissed Obusans petition ratiocinating that the PNB-RRPs lowering


the compulsory retirement age to 60 years is not violative of Article 287 of the
Labor Code of the Philippines, as amended, despite the issuance of the plan years
after Obusan was hired.

FACTS: Obusan was hired by PNB in 1979 when the bank was still a governmentowned and controlled corporation whose retirement program for employees was
under the GSIS. When PNB was privatized in 1996, Obusan was deemed retired

HELD: YES. Article 287 of the Labor Code, as amended by RA 7641, which took
effect on January 7, 1993, provides: ART. 287. Retirement. Any employee may

223. OBUSAN vs. PHILIPPINE NATIONAL BANK

ISSUES: Whether or not PNB can unilaterally lower the compulsory retirement
age to 60 years in violation of Obusans alleged right to retire at the age of 65?

3E Andaya Ching Espiritu Hefti Galvez Gammad Lainez Lui Madamba Nagera Narvasa
Ong Palangdao Rosales Sanchez Santos Satrain Tabo (2014-2015)

Labor Relations Case Digest - Atty. Joyrich Golangco

be retired upon reaching the retirement age established in the collective


bargaining agreement or other applicable employment contract. Under this
provision, the retirement age is primarily determined by the existing agreement
or employment contract. Obusan when hired in 1979 was governed by civil
service laws and the compulsory retirement age imposed by law was at 65 years.
When PNB ceased to be a government-owned & controlled corporation as a
result of its privatization, all of its officers and employees were deemed retired
from the government service thats why they received their respective
retirement gratuities. When she continued her employment at PNB which
became a privately organized bank, her membership in the PNB-RRP was made
automatic, to wit: Section 1. Membership. Membership in the Plan shall be
automatic for all full-time regular and permanent officers and employees of the
Bank as of the effectivity date of the Plan. For employees hired after the
effectivity of this Plan, their membership shall be effective on "Date Entered
Bank." PNB-RRP was registered with the BIR and, later, was recognized by the
Philnabank Employees Association in the CBA it entered with PNB. PNB
disseminated to all its officers and employees the terms and conditions and the
guidelines for its implementation, opened for scrutiny, giving the employees
every opportunity to question the plan should it be not beneficial to the
employees as compared to the mandate of Art. 287 of the Labor Code. Obusan,
who claimed to be the President of the PNB Supervisors and Officers Association,
did not express their dissent or disagreement with its provision but only
questioned the compulsory retirement age provision of the PNB-RPP when she
was compulsorily retired. The Court held that the PNB-RRP is a valid exercise of
PNBs prerogative to provide a retirement plan for all its employees. Besides, the
PNB-RRP is solely and exclusively funded by PNB, and no financial burden is
imposed on the employees for their retirement benefits.

The Court denied the petition and affirmed the assailed Decision and Resolution
of the CA. No costs.
224. Kimberly-Clark Philippines Inc. vs Dimayuga

DOCTRINE:
It is settled that retirement benefits must be specifically be
granted by existing laws, under a CBA or an established employer policy.

FACTS: Respondents Nora Dimayuga, Rosemarie Gloria and Maricar de Guia


were all employees of petitioner. Nora and Rosemarie tendered their resignation
effective October 21, 2002 months prior to the early retirement package offered
by petitioner due to cost-cutting which took effect November 10-30, 2002. On the
other hand, Maricar tendered her resignation on Nov. 4, 2002 that is to take
effect December 1, 2002 due to career advancement. Even if Nora and Rosemarie
were no longer employees at the time the retirement package was offered, they
pleaded with petitioner that such be retroactively extended to them to which
petitioner acceded.

After Nora and Rosemarie retired and received more than a million
pesos each and after executing a release and quitclaim, the petitioner offered an
economic assistance to all monthly-paid employees on a regular status as of
November 16, 2002. Subsequently, petitioner announced that it would grant
200K as lump sum retirement pay in addition to the early retirement package
benefit to those who signed up for early retirement and those who would sign up
until Jan. 22, 2003.
The 200K lump sum pay and economic assistance were not given to the
petitioner hence they filed a complaint before the NLRC RAB claiming
entitlement to such benefits.

LA: dismissed the complaint holding that Nora and Rosemarie were not entitled
to the 200K as they have ceased their employment at the time it was offered BUT
held that Maricar was entitled to it because her resignation took effect Dec. 1 and
such pay was already offered.
NLRC: modified LA. Nora and Rosemarie are both entitled to the 200K plus
economic assistance. Affirmed decision regarding Maricars entitlement.
CA: affirmed NLRC. The general principles of fair play and justice dictate that
petitioner extend to them the same benefits in consideration of their long years
of service.

ISSUES: Whether or not respondents were entitled to the 200k lump sum pay
and economic assistance offered by the petitioner.

3E Andaya Ching Espiritu Hefti Galvez Gammad Lainez Lui Madamba Nagera Narvasa
Ong Palangdao Rosales Sanchez Santos Satrain Tabo (2014-2015)

Labor Relations Case Digest - Atty. Joyrich Golangco

HELD: Negative. It is settled that retirement benefits must be specifically be


granted by existing laws, under a CBA or an established employer policy. In this
case, NO LAW, NO CBA provision or an ESTABLISHED COMPANY POLICY existed
during respondents employment entitling them to the 200K pay. Hence
petitioner was not obliged to grant them such pay. The offer of the lump sum
retirement pay was an incentive to those employees who would voluntarily avail
of its early retirement scheme due to cost-cutting or retrenchment and that
respondents RESIGNED and NOT RETRENCHED.

Even if petitioner acceded to the request of Nora and Rosemarie, this


was out of pure generosity and it is thus absurd to fault petitioner for acceding to
such a request out of compassion by directing it to pay additional benefits to
resigned employees who are not entitled thereto. Neither Nora and Rosemarie
entitled to the economic assistance as they were NO monthly employees who
are under regular status as of Nov. 2002, they having resigned earlier or on Oct.
2002. The SC noted that this economic assistance is a form of a BONUS given in
lieu of the performance-based annual salary increase --- a bonus or grant that
highly DEPENDS on the petitioners FINANCIAL CAPABILITY and in essence it is
well within petitioners MANAGEMENT PREROGATIVES.
With regard the quitclaims executed, these are usually frowned upon
being contrary to public policy BUT where the person making the waiver has
done so VOLUNTARILY with full understanding thereof and the consideration in
the quitclaim is credible and reasonable, then such is valid and binding between
the parties. More than a million pesos each for Nora and Rosemarie are
conscionable.
Lastly, Maricar is also not entitled to the 200K because again, she
resigned and did not avail of the retirement package and because she never
really intended to retire based on her reason for resigning -- career
advancement. No incentives should be extended to her just like Nora and
Rosemarie.
225. Magdadaro vs PNB

DOCTRINE: Retirement is the result of a bilateral act of the parties, a voluntary


agreement between the employer and the employee whereby the latter, after

reaching a certain age, agrees to sever his or her employment with the former.
Retirement is provided for under Article 287 of the Labor Code, as amended by
Republic Act No. 7641, or is determined by an existing agreement between the
employer and the employee.

FACTS: Marcelino A. Magdadaro was employed by PNB since 1968. He was the
Senior Assistant Manager of Visayas division when he filed his application for
early retirement under Special Separation Incentive Program (SSIP) and he
stated that his preferred effective date of retirement is Dec. 31, 1999. PNB
approved the application for early retirement but made it effective on December
31, 1998. Petitioner protested the acceleration of his retirement. He received,
under protest, his retirement and separation benefits amounting to P908,950.44.
Petitioner filed a complaint for illegal dismissal and payment of moral, exemplary
and actual damages.
LA: PNB had the discretion and prerogative to set the effective date of retirement
under the SSIP. The insistence on the date of effectivity of petitioners retirement
was not tantamount to illegal dismissal and there was no dismissal to speak of
because petitioner voluntarily availed of the SSIP. Still, the Labor Arbiter granted
petitioners preferred date of retirement and awarded him additional retirement
benefits.

NLRC: Considered petitioners retirement on 31 December 1998 as tantamount


to illegal dismissal. The NLRC ruled that while it recognized respondents
prerogative to change petitioners retirement date, management prerogative
should be exercised with prudence and without malice
CA: NLRC acted with grave abuse of discretion in affirming the decision of the
Labor Arbiter, while at the same time finding that petitioners retirement was
tantamount to illegal dismissal. The Court of Appeals held that petitioner
voluntarily applied for the SSIP. The Court of Appeals ruled that petitioner could
not claim to have been illegally dismissed just because the date of effectivity of
his retirement did not conform to his preferred retirement date.
ISSUES: Whether petitioner was illegally dismissed from employment.

3E Andaya Ching Espiritu Hefti Galvez Gammad Lainez Lui Madamba Nagera Narvasa
Ong Palangdao Rosales Sanchez Santos Satrain Tabo (2014-2015)

Labor Relations Case Digest - Atty. Joyrich Golangco

HELD: Retirement is the result of a bilateral act of the parties, a voluntary


agreement between the employer and the employee whereby the latter, after
reaching a certain age, agrees to sever his or her employment with the former.
Retirement is provided for under Article 287 of the Labor Code, as amended by
Republic Act No. 7641, or is determined by an existing agreement between the
employer and the employee. In this case, respondent offered the SSIP to
overhaul the bank structure and to allow it to effectively compete with local peer
and foreign banks. SSIP was not compulsory on employees. Petitioner
voluntarily availed of the SSIP. He accomplished the application form and
submitted it. He only questioned the approval of his retirement on a date earlier
than his preferred retirement date.
The Labor Arbiter ruled that petitioner was not illegally dismissed from the
service. Even the NLRC ruled that petitioner could no longer withdraw his
application for early retirement under the SSIP. However, the NLRC ruled that
respondent could not accelerate the petitioners retirement date. The NLRC
ruled that it could not imagine how petitioners continued employment until 31
December 1999 would impair the delivery of bank services and attribute bad
faith on respondent when it accelerated petitioners retirement.

We do not agree. Whether petitioners early retirement within the SSIP period
will improve or impair the delivery of bank services is a business decision
properly within the exercise of management prerogative. We see no grave abuse
of discretion on the part of respondent in the exercise of this management
prerogative. The exercise of management prerogative is valid provided it is not
performed in a malicious, harsh, oppressive, vindictive or wanton manner or out
of malice or spite.
226.. ROBERTO R. SERRANO vs. COURT OF APPEALS, NATIONAL LABOR
RELATIONS COMMISSION, MAERSK-FILIPINAS CREWING, INC. and A.P.
MOLLER

DOCTRINE: Petitioner's cause of action accrues only upon respondent's definite


denial of his claim.

FACTS: From 1974 to 1991, respondent Maersk-Filipinas Crewing, Inc., the local
agent of respondent foreign corporation A.P. Moller, deployed petitioner Serrano
as a seaman to Liberian, British and Danish ships. As petitioner was on board a
ship most of the time, respondent Maersk offered to send portions of petitioner's
salary to his family in the Philippines by money order. From 1977 to 1978, he
instructed Maersk to send money orders to his family. Respondent Maersk
deducted the amounts of these money orders totalling HK$4,600.00 and
1,050.00 Sterling Pounds from petitioner's salary, as well as various amounts
for Danish Social Security System (SSS), welfare contributions, ship club, and SSS
Medicare.
In 1978, upon learning that his family failed to receive the money orders he sent
through Maersk, petitioner demanded payment of the amounts deducted from
his salary. Respondent Maersk assured him that they would look into the matter.
He followed up his money claims several times but would always be told to
return as Maersk needed time to verify its records and to bring up the matter
with A.P. Moller.
Finally, in October 1993, he wrote to Maersk demanding immediate payment of
the total amount of the money orders deducted from his salary from 1977 to
1978. Respondent A.P. Moller replied to petitioner that they keep accounting
documents only for a certain number of years, thus data on his money claims
from 1977 to 1978 were no longer available. Likewise, it was claimed that it had
no outstanding money orders.

In April 1994, petitioner filed a complaint for collection of the total amount of the
unsent money orders and illegal salary deductions against Maersk in the POEA.
The case was transferred to the NLRC where LA ruled that the deductions were
lawful but ordered the refund of the money order payments which were not
transmitted.
NLRC: reversed and set aside LAs decision and dismissed the case on the ground
of prescription. Citing Art. 291 of the Labor Code, it held that the cause of action
accrued in 1977 and 1978 but complainant filed a complaint only in 1994. MR
denied.
CA: dismissed petition for having been filed out of time.

3E Andaya Ching Espiritu Hefti Galvez Gammad Lainez Lui Madamba Nagera Narvasa
Ong Palangdao Rosales Sanchez Santos Satrain Tabo (2014-2015)

Labor Relations Case Digest - Atty. Joyrich Golangco

ISSUES: Whether or not the claim of the petitioner has prescribed.

HELD: NO. The pivotal question is when petitioner's cause of action accrued for
this will determine the reckoning date of the three-year prescriptive period.

Petitioner contends that his cause of action accrued only in 1993 when
respondent A.P. Moller wrote to him that its accounting records showed it had no
outstanding money orders and that his case was considered outdated. Thus, the
three (3) year prescriptive period should be counted from 1993 and not 1978
and since his complaint was filed in 1994, he claims that it has not prescribed.
We agree. Petitioner's cause of action accrued in November 1993 upon
respondent Maersk's definite denial of his money claims (following this Court's
ruling in the similar case of Baliwag Transit, Inc. v. Ople - - - x x x).

Petitioner repeatedly demanded payment from respondent Maersk but


respondent Maersk warded off these demands by saying that it would look into
the matter until years passed by. In October 1993, Serrano finally demanded in
writing payment of the unsent money orders. Then and only then was the claim
categorically denied by respondent A.P. Moller in its letter dated November 22,
1993. Following the Baliwag Transit ruling, petitioner's cause of action accrued
only upon respondent A.P. Moller's definite denial of his claim in November
1993. Having filed his action five (5) months thereafter or in April 1994, we hold
that it was filed within the three-year (3) prescriptive period provided in Article
291 of the Labor Code.
227. INTERCONTINENTAL BROADCASTING CORPORATION, Petitioner, vs.
IRENEO PANGANIBAN, Respondent.

DOCTRINE: The prescription of an action is interrupted by (a) the filing of an


action, (b) a written extrajudicial demand by the creditor, and (c) a written
acknowledgment of the debt by the debtor. On this point, the Court ruled that
although the commencement of a civil action stops the running of the statute of
prescription or limitations, its dismissal or voluntary abandonment by plaintiff
leaves the parties in exactly the same position as though no action had been
commenced at all.

FACTS: Ireneo Panganiban (respondent) was employed as Assistant General


Manager of the Intercontinental Broadcasting Corporation (petitioner) from May
1986 until his preventive suspension on August 26, 1988. Respondent resigned
from his employment on September 2, 1988. On April 12, 1989, respondent filed
with the Regional Trial Court of Quezon City, Branch 93, Civil Case No. Q-89-2244
against the members of the Board of Administrators (BOA) of petitioner alleging,
among others, non-payment of his unpaid commissions. A motion to dismiss was
filed by Joselito Santiago, one of the defendants, on the ground of lack of
jurisdiction, as respondent's claim was a labor money claim, but this was denied.
Thus, Santiago filed a petition for certiorari with the CA, and in a Decision dated
October 29, 1991, the CA granted Santiago's petition for lack of jurisdiction and
set aside the RTC's Orders dated October 19, 1990 and November 23, 1990.
Thereafter, respondent was elected by the BOA as Vice-President for Marketing
in July 1992. He resigned in April 1993.

On July 24, 1996, respondent filed against petitioner a complaint for illegal
dismissal, separation pay, retirement benefits, unpaid commissions, and
damages. The Labor Arbiter (LA) ordered respondent's reinstatement with full
backwages, and the payment of his unpaid commission in the amount of
P2,521,769.77, damages and attorney's fees. Petitioner appealed to the National
Labor Relations Commission (NLRC) but due to petitioner's failure to post a
bond, the appeal was dismissed. Petitioner filed a motion for reconsideration of
the NLRC's dismissal, which was denied. Petitioner then filed a petition with this
Court but the same was referred to the CA in view of the ruling in St. Martin
Funeral Home v. National Labor Relations Commission. On July 30, 1999, the CA
rendered its Decision granting the petition. The claims of private respondent for
reinstatement, backwages and benefits in conjunction with his employment from
1986 to 1988 have prescribed. Respondent filed a motion for reconsideration of
the CA Decision, and on August 21, 2001, the CA rendered the assailed
Resolution. Petitioner sought reconsideration of the CA Resolution, but it was
denied per the assailed Resolution dated January 9, 2002.
ISSUES: Whether or not respondent's claim for unpaid commissions in the
amount of P2,521,769.77 has already prescribed.

3E Andaya Ching Espiritu Hefti Galvez Gammad Lainez Lui Madamba Nagera Narvasa
Ong Palangdao Rosales Sanchez Santos Satrain Tabo (2014-2015)

Labor Relations Case Digest - Atty. Joyrich Golangco

HELD: YES. The applicable law in this case is Article 291 of the Labor Code which
provides that "all money claims arising from employer-employee relations
accruing during the effectivity of this Code shall be filed within three (3) years
from the time the cause of action accrued; otherwise they shall be forever
barred." The term "money claims" covers all money claims arising from an
employer-employee relation.

DOCTRINE: Promissory estoppel may arise from the making of a promise, even
though without consideration, if it was intended that the promise should be
relied upon, as in fact it was relied upon, and if a refusal to enforce it would
virtually sanction the perpetration of fraud or would result in other injustice. The
principle of promissory estoppel is a recognized exception to the three-year
prescriptive period enunciated in Article 291 of the Labor Code.

ART. 1155. The prescription of actions is interrupted when they are filed before
the Court, when there is a written extrajudicial demand by the creditors, and
when there is any written acknowledgment of the debt by the debtor.

FACTS: On September 27, 2002, respondent Alabanza filed a complaint against


petitioners Arts 21 and Hashimoto for and in behalf of her husband for nonpayment of salaries, separation pay and 13th month pay.

Like other causes of action, the prescriptive period for money claims is subject to
interruption, and in the absence of an equivalent Labor Code provision for
determining whether the said period may be interrupted, Article 1155 of the
Civil Code may be applied, to wit:

Thus, the prescription of an action is interrupted by (a) the filing of an action, (b)
a written extrajudicial demand by the creditor, and (c) a written
acknowledgment of the debt by the debtor. On this point, the Court ruled that
although the commencement of a civil action stops the running of the statute of
prescription or limitations, its dismissal or voluntary abandonment by plaintiff
leaves the parties in exactly the same position as though no action had been
commenced at all.

Hence, while the filing of Civil Case No. Q-89-2244 could have interrupted the
running of the three-year prescriptive period, its consequent dismissal by the CA
in CA-G.R. SP No. 23821 due to lack of jurisdiction effectively canceled the tolling
of the prescriptive period within which to file his money claim, leaving
respondent in exactly the same position as though no civil case had been filed at
all. The running of the three-year prescriptive period not having been
interrupted by the filing of Civil Case No. Q-89-2244, respondent's cause of action
had already prescribed on September 2, 1991, three years after his cessation of
employment on September 2, 1988. Consequently, when respondent filed his
complaint for illegal dismissal, separation pay, retirement benefits, and damages
in July 24, 1996, his claim, clearly, had already been barred by prescription.
228. Accessories Specialist Inc., a.k.a. Arts 21 Corporation vs. Alabanza

In order to make out a claim of promissory estoppel, a party bears the burden of
establishing the following elements: (1) a promise was reasonably expected to
induce action or forbearance; (2) such promise did, in fact, induce such action or
forbearance; and (3) the party suffered detriment as a result.

Respondents husband was the Vice-President, Manager and Director of Arts 21


and had been with the company from 1975 to 1997. He was compelled by the
owner, Hashimoto, to file his involuntary resignation on October 17, 1997 on the
ground that Arts 21 allegedly suffered losses. Respondents husband demanded
payment of his money claims upon resignation but was told that rank and file
employees will be paid first and thus waited for his turn. Respondents husband
made several demands but Arts 21 just kept on assuring him that he will be paid
his money claims. Respondents husband died on August 5, 2002 with his claims
still unpaid.

Petitioners invoke Art. 291 of the Labor Code and contend that respondents
husband voluntarily resigned in October, 1997, thus the cause of action has
already prescribed since the case was filed in 2002 only, beyond the three-yearperiod within which money claims should be filed.

The Labor Arbiter rendered a decision ordering petitioner to pay respondent


over P4M. Petitioners filed an appeal along with a motion to reduce bond,
attaching receipts for cash bond amounting to P290K and appeal fee for P170.00.
The motion was denied and petitioners were given 10 days within which to file
the required bond. Petitioners filed a motion for reconsideration which the NLRC
denied ordering the dismissal of the appeal for non-perfection thereof due to
non-compliance with the bond requirement. The resolution became final and

3E Andaya Ching Espiritu Hefti Galvez Gammad Lainez Lui Madamba Nagera Narvasa
Ong Palangdao Rosales Sanchez Santos Satrain Tabo (2014-2015)

Labor Relations Case Digest - Atty. Joyrich Golangco

executory and a writ of execution was issued by the Labor Arbiter upon motion
by respondent. Petitioners filed a petition for certiorari with the Court of Appeals
praying for the issuance of a TRO and a writ of preliminary injunction. The
petition was dismissed.
ISSUES: WON the cause of action of respondent has already prescribed/

HELD: NO. Based on the findings of facts of the Labor Arbiter, it was petitioner
Arts 21 which was responsible for the delay in the institution of the complaint.
When petitioners husband filed his resignation he immediately asked for the
payment of his money claims. However, the management of Arts 21 promised
him that he would be paid immediately after the claim of the rank-and-file
employees had been paid. Jones relied on this representation.

Promissory estoppel may arise from the making of a promise, even


though without consideration, if it was intended that the promise should be
relied upon, as in fact it was relied upon, and if a refusal to enforce it would
virtually sanction the perpetration of fraud or would result in other injustice. The
principle of promissory estoppel is a recognized exception to the three-year
prescriptive period enunciated in Article 291 of the Labor Code.
In order to make out a claim of promissory estoppel, a party bears the
burden of establishing the following elements: (1) a promise was reasonably
expected to induce action or forbearance; (2) such promise did, in fact, induce
such action or forbearance; and (3) the party suffered detriment as a result. All
the requisites are present in this case. The Court, therefore, finds ample
justification not to follow the prescriptive period imposed under Art. 291 of the
Labor Code. Great injustice will be committed if respondents claims will be
brushed aside on a mere technicality, especially when it was petitioners own
action that prevented respondent from interposing the claims within the
required period.
229. Autobus Transport System vs. Bautista

DOCTRINE: Applying Article 291 of the Labor Code in light of this peculiarity of
the service incentive leave, we can conclude that the three (3)-year prescriptive
period commences, not at the end of the year when the employee becomes
entitled to the commutation of his service incentive leave, but from the time

when the employer refuses to pay its monetary equivalent after demand of
commutation or upon termination of the employees services, as the case may be.

FACTS: Antonio Bautista has been employed by petitioner Auto Bus Transport
Systems, Inc. (Autobus), as driver-conductor with travel routes ManilaTuguegarao via Baguio, Baguio- Tuguegarao via Manila and Manila-Tabuk via
Baguio. Respondent was paid on commission basis, seven percent (7%) of the
total gross income per travel, on a twice a month basis.
On 03 January 2000, while respondent was driving Autobus No. 114 along Sta.
Fe, Nueva Vizcaya, the bus he was driving accidentally bumped the rear portion
of Autobus No. 124, as the latter vehicle suddenly stopped at a sharp curve
without giving any warning.

Respondent averred that the accident happened because he was compelled by


the management to go back to Roxas, Isabela, although he had not slept for
almost twenty-four (24) hours, as he had just arrived in Manila from Roxas,
Isabela. Respondent further alleged that he was not allowed to work until he fully
paid the amount of P75,551.50, representing thirty percent (30%) of the cost of
repair of the damaged buses and that despite respondents pleas for
reconsideration, the same was ignored by management. After a month,
management sent him a letter of termination.
Thus, on 02 February 2000, respondent instituted a Complaint for Illegal
Dismissal with Money Claims for nonpayment of 13th month pay and service
incentive leave pay against Autobus.

Petitioner, on the other hand, maintained that respondents employment was


replete with offenses involving reckless imprudence, gross negligence, and
dishonesty. To support its claim, petitioner presented copies of letters, memos,
irregularity reports, and warrants of arrest pertaining to several incidents
wherein respondent was involved.
LA: Found that the complaint has no leg to stand on. It is hereby DISMISSED.
However, still based on the above-discussed premises, the respondent must pay
to the complainant the following:

3E Andaya Ching Espiritu Hefti Galvez Gammad Lainez Lui Madamba Nagera Narvasa
Ong Palangdao Rosales Sanchez Santos Satrain Tabo (2014-2015)

Labor Relations Case Digest - Atty. Joyrich Golangco

a. his 13th month pay from the date of his hiring to the date of his dismissal,
presently computed at P78,117.87;
b. his service incentive leave pay for all the years he had been in service with the
respondent, presently computed at P13,788.05.

NLRC: Modified the assailed decision by deleting the award for 13th month pay
to the complainant.
CA: Affirmed the NLRC
ISSUES: Whether or not respondent is entitled to service incentive leave;
Whether or not the three (3)-year prescriptive period provided under Article
291 of the Labor Code, as amended, is applicable to respondents claim of service
incentive leave pay.

HELD: Respondent is entitled to service incentive leave, the disposition of the


first issue revolves around the proper interpretation of Article 95 of the Labor
Code vis--visSection 1(D), Rule V, Book III of the Implementing Rules and
Regulations of the Labor Code which provides:
Art. 95. RIGHT TO SERVICE INCENTIVE LEAVE
(a) Every employee who has rendered at least one year of
service shall be entitled to a yearly service incentive leave of
five days with pay.
Book III, Rule V: SERVICE INCENTIVE LEAVE
SECTION 1. Coverage. This rule shall apply to all employees except:

(d) Field personnel and other employees whose performance is


unsupervised by the employer including those who are engaged
on task or contract basis, purely commission basis, or those who
are paid in a fixed amount for performing work irrespective of
the time consumed in the performance thereof; . . .
A careful perusal of said provisions of law will result in the conclusion that the
grant of service incentive leave has been delimited by the Implementing Rules
and Regulations of the Labor Code to apply only to those employees not explicitly
excluded by Section 1 of Rule V. According to the Implementing Rules, Service
Incentive Leave shall not apply to employees classified as "field personnel." The

phrase "other employees whose performance is unsupervised by the employer"


must not be understood as a separate classification of employees to which
service incentive leave shall not be granted. Rather, it serves as an amplification
of the interpretation of the definition of field personnel under the Labor Code as
those "whose actual hours of work in the field cannot be determined with
reasonable certainty."
The same is true with respect to the phrase "those who are engaged on task or
contract basis, purely commission basis." Said phrase should be related with "field
personnel," applying the rule on ejusdem generis that general and unlimited term
are restrained and limited by the particular terms that they follow. Hence,
employees engaged on task or contract basis or paid on purely commission basis
are not automatically exempted from the grant of service incentive leave, unless,
they fall under the classification of field personnel.

According to Article 82 of the Labor Code, "field personnel" shall refer to nonagricultural employees who regularly perform their duties away from the
principal place of business or branch office of the employer and whose actual
hours of work in the field cannot be determined with reasonable certainty. This
definition is further elaborated in theBureau of Working Conditions (BWC),
Advisory Opinion to Philippine Technical-Clerical Commercial Employees
Association which states that:

As a general rule, [field personnel] are those whose performance of their


job/service is not supervised by the employer or his representative, the
workplace being away from the principal office and whose hours and
days of work cannot be determined with reasonable certainty; hence,
they are paid specific amount for rendering specific service or
performing specific work. If required to be at specific places at specific
times, employees including drivers cannot be said to be field personnel
despite the fact that they are performing work away from the principal
office of the employee.

At this point, it is necessary to stress that the definition of a "field personnel" is


not merely concerned with the location where the employee regularly performs
his duties but also with the fact that the employees performance is unsupervised

3E Andaya Ching Espiritu Hefti Galvez Gammad Lainez Lui Madamba Nagera Narvasa
Ong Palangdao Rosales Sanchez Santos Satrain Tabo (2014-2015)

Labor Relations Case Digest - Atty. Joyrich Golangco

by the employer. As discussed above, field personnel are those who regularly
perform their duties away from the principal place of business of the
employer and whose actual hours of work in the field cannot be determined with
reasonable certainty. Thus, in order to conclude whether an employee is a field
employee, it is also necessary to ascertain if actual hours of work in the field can
be determined with reasonable certainty by the employer. In so doing, an inquiry
must be made as to whether or not the employees time and performance are
constantly supervised by the employer.

In the application of this section of the Labor Code, the pivotal question to be
answered is when does the cause of action for money claims accrue in order to
determine the reckoning date of the three-year prescriptive period.

It is of judicial notice that along the routes that are plied by these bus
companies, there are its inspectors assigned at strategic places who
board the bus and inspect the passengers, the punched tickets, and the
conductors reports. There is also the mandatory once-a-week car barn
or shop day, where the bus is regularly checked as to its mechanical,
electrical, and hydraulic aspects, whether or not there are problems
thereon as reported by the driver and/or conductor. They too, must be
at specific place as [sic] specified time, as they generally observe prompt
departure and arrival from their point of origin to their point of
destination. In each and every depot, there is always the Dispatcher
whose function is precisely to see to it that the bus and its crew leave the
premises at specific times and arrive at the estimated proper time.
These, are present in the case at bar. The driver, the complainant herein,
was therefore under constant supervision while in the performance of
this work. He cannot be considered a field personnel.

To properly construe Article 291 of the Labor Code, it is essential to ascertain the
time when the third element of a cause of action transpired. Stated differently, in
the computation of the three-year prescriptive period, a determination must be
made as to the period when the act constituting a violation of the workers right
to the benefits being claimed was committed. For if the cause of action accrued
more than three (3) years before the filing of the money claim, said cause of
action has already prescribed in accordance with Article 291.

As observed by the Labor Arbiter and concurred in by the Court of Appeals:

We agree in the above disquisition. Therefore, as correctly concluded by the


appellate court, respondent is not a field personnel but a regular employee who
performs tasks usually necessary and desirable to the usual trade of petitioners
business. Accordingly, respondent is entitled to the grant of service incentive
leave.
Article 291 of the Labor Code states that all money claims arising from employeremployee relationship shall be filed within three (3) years from the time the
cause of action accrued; otherwise, they shall be forever barred.

It is settled jurisprudence that a cause of action has three elements, to wit, (1) a
right in favor of the plaintiff by whatever means and under whatever law it arises
or is created; (2) an obligation on the part of the named defendant to respect or
not to violate such right; and (3) an act or omission on the part of such defendant
violative of the right of the plaintiff or constituting a breach of the obligation of
the defendant to the plaintiff.

Correspondingly, it can be conscientiously deduced that the cause of action of an


entitled employee to claim his service incentive leave pay accrues from the
moment the employer refuses to remunerate its monetary equivalent if the
employee did not make use of said leave credits but instead chose to avail of its
commutation. Accordingly, if the employee wishes to accumulate his leave
credits and opts for its commutation upon his resignation or separation from
employment, his cause of action to claim the whole amount of his accumulated
service incentive leave shall arise when the employer fails to pay such amount at
the time of his resignation or separation from employment.
Applying Article 291 of the Labor Code in light of this peculiarity of the service
incentive leave, we can conclude that the three (3)-year prescriptive period
commences, not at the end of the year when the employee becomes entitled to
the commutation of his service incentive leave, but from the time when the
employer refuses to pay its monetary equivalent after demand of commutation
or upon termination of the employees services, as the case may be.

3E Andaya Ching Espiritu Hefti Galvez Gammad Lainez Lui Madamba Nagera Narvasa
Ong Palangdao Rosales Sanchez Santos Satrain Tabo (2014-2015)

Labor Relations Case Digest - Atty. Joyrich Golangco

The above construal of Art 291, vis--vis the rules on service incentive leave, is in
keeping with the rudimentary principle that in the implementation and
interpretation of the provisions of the Labor Code and its implementing
regulations, the workingmans welfare should be the primordial and paramount
consideration. The policy is to extend the applicability of the decree to a greater
number of employees who can avail of the benefits under the law, which is in
consonance with the avowed policy of the State to give maximum aid and
protection to labor.
230. PLDT v. Pingol (Sept 8, 2010)
DOCTRINE: The Labor Code has no specific provision on when a claim for illegal
dismissal or a monetary claim accrues. Thus, the general law on prescription
applies. Article 1150 of the Civil Code states that the time for prescription for all
kinds of actions, when there is no special provision which ordains otherwise,
shall be counted from the day they may be brought. The day the action may be
brought is the day a claim starts as a legal possibility.
The prescription of an action is interrupted by (a) the filing of an action,
(b) a written extrajudicial demand by the creditor, and (c) a written
acknowledgment of the debt by the debtor.

FACTS: In 1979, respondent Roberto Pingol was hired by petitioner PLDT as a


maintenance technician. On April 13, 1999, while still under the employ of PLDT,
Pingol was admitted at The Medical City, Mandaluyong City, for paranoid
personality disorder due to financial and marital problems. On May 14, 1999, he
was discharged from the hospital. Thereafter, he reported for work but
frequently absented himself due to his poor mental condition. From September
16, 1999 to December 31, 1999, Pingol was absent from work without official
leave.
According to PLDT, notices were sent to him with a stern warning that
he would be dismissed from employment if he continued to be absent without
official leave pursuant to PLDT Systems Practice which provides that Absence
without authorized leaves for seven (7) consecutive days is subject to
termination from the service. Despite the warning, he failed to show up for
work. On January 1, 2000, PLDT terminated his services on the grounds of
unauthorized absences and abandonment of office.

On March 29, 2004, four years later, Pingol filed a Complaint for
Constructive Dismissal and Monetary Claims against PLDT. In his complaint, he
alleged that he was hastily dismissed from his employment on January 1, 2000.
In response, PLDT filed a motion to dismiss claiming, among others, that
respondents cause of action had already prescribed as the complaint was filed 4
years and 3 months after his dismissal.
Pingol, however, countered that in computing the prescriptive period,
the years 2001 to 2003 must not be taken into account. He explained that from
2001 to 2003, he was inquiring from PLDT about the financial benefits due him
as an employee who was no longer allowed to do his work, but he merely got
empty promises. It could not, therefore, result in abandonment of his claim.
LA ruled that the action was barred by prescription. NLRC and CA held
that the cause of action has not yet prescribed as Pingol was not formally
dismissed on January 1, 2000 or his monetary claims categorically denied by
petitioner.
ISSUES: WON Pingols claim has already prescribed.

HELD: The actions have prescribed. Judicial admissions made by parties in the
pleadings, or in the course of the trial or other proceedings in the same case are
conclusive and so does not require further evidence to prove them. These
admissions cannot be contradicted unless previously shown to have been made
through palpable mistake or that no such admission was made.
In the case at bench, Pingol himself alleged the date January 1, 2000 as
the date of his dismissal in his complaint filed on March 29, 2004, exactly four (4)
years and three (3) months later. Respondent never denied making such
admission or raised palpable mistake as the reason therefor. Thus, the petitioner
correctly relied on such allegation in the complaint to move for the dismissal of
the case on the ground of prescription.
The Labor Code has no specific provision on when a claim for illegal
dismissal or a monetary claim accrues. Thus, the general law on prescription
applies. Article 1150 of the Civil Code states that the time for prescription for all
kinds of actions, when there is no special provision which ordains otherwise,
shall be counted from the day they may be brought.
The day the action may be brought is the day a claim starts as a legal
possibility. In the present case, January 1, 2000 was the date that respondent

3E Andaya Ching Espiritu Hefti Galvez Gammad Lainez Lui Madamba Nagera Narvasa
Ong Palangdao Rosales Sanchez Santos Satrain Tabo (2014-2015)

Labor Relations Case Digest - Atty. Joyrich Golangco

Pingol was not allowed to perform his usual and regular job as a maintenance
technician. Respondent Pingol cited the same date of dismissal in his complaint
before the LA. Thus, the complaint filed had already prescribed.
The prescription of an action is interrupted by (a) the filing of an action,
(b) a written extrajudicial demand by the creditor, and (c) a written
acknowledgment of the debt by the debtor. In this case, respondent Pingol never
made any written extrajudicial demand. Neither did petitioner make any written
acknowledgment of its alleged obligation. Thus, the claimed follow-ups could
not have validly tolled the running of the prescriptive period. It is worthy to note
that respondent never presented any proof to substantiate his allegation of
follow-ups.
231. PEAFRANCIA
SARMIENTO

TOURS

AND

TRAVEL

TRANSPORT,

INC.

v.

DOCTRINE: Closure of business is the reversal of fortune of the employer


whereby there is a complete cessation of business operations and/or an actual
locking-up of the doors of the establishment, usually due to financial losses.
Closure of business, as an authorized cause for termination of employment, aims
to prevent further financial drain upon an employer who can no longer pay his
employees since business has already stopped.
A change of ownership in a business concern is not proscribed by law. Where the
charge of ownership is done in bad faith, or is used to defeat the rights of labor,
the successor-employer is deemed to have absorbed the employees and is held
liable for the transgressions of his or her predecessor.

FACTS: Since October 1993, until his alleged termination on October 30, 2002,
respondent Joselito Sarmiento (Sarmiento) worked as a bus inspector of
petitioner Peafrancia Tours and Travel Transport, Inc. (petitioner), earning a
daily wage of P198.00. In his complaint for illegal dismissal filed on November
26, 2002, Sarmiento prayed for his reinstatement, and charged petitioner with
underpayment of wages; non-payment of overtime, holiday pay, premium pay for
holiday and rest day, service incentive leave pay, 13th month pay, and separation
pay; unfair labor practice; damages; and attorneys fees. Meanwhile, respondent
Ricardo Catimbang (Catimbang) also worked for petitioner as a bus inspector

from February 1997 until his termination on October 30, 2002. He was also paid
a daily wage of P198.00. He averred that petitioner was guilty of union-busting,
and prayed for reinstatement with payment of full backwages, benefits, damages,
and attorneys fees.

Both Sarmiento and Catimbang (respondents) averred that they were


required to work seven (7) days a week, and that they had no rest day and
worked even during the holidays, except Good Friday, Christmas Eve, and New
Years Eve. Sometime in the first week of October 2002, they received notices
of termination on the ground of petitioners alleged irreversible business
losses.
In the middle of October 2002, a meeting was called by petitioners
President and General Manager, Bonifacio Cu, wherein respondents were
introduced to Alfredo Perez, the owner of ALPS Transportation, as the new
owner of petitioner, having allegedly bought the same. On October 30, 2002,
respondents received their last pay with a letter informing them that their
application with the company had been held in abeyance. Sarmiento was
paid P26,730.00 as separation pay and P4,686.00 as 13th month pay; while
Catimbang was paid P17,820.00 as separation pay and P4,851.00 as 13th month
pay. Respondents, however, learned that, several days after their
termination, Bonifacio Cu continued to operate petitioner bus company.
Traversing the complaint, petitioner admitted that respondents were
among its bus inspectors. It asseverated, however, that due to severe business
losses, petitioner made the painful decision to stop its operation and sell the
business enterprise to the Perez family of ALPS Transportation. It alleged that
due notice was given to the Department of Labor and Employment, and that all
its employees were duly notified and were paid their corresponding separation
pay, as well as their 13th month pay. The new owners maintained the business
name of petitioner, and the management of petitioner was entrusted to the new
owners in October 2002, with Edilberto Perez as Vice-President for Finance and
Operations. Subsequently, several memoranda were issued by Edilberto Perez in
behalf of petitioner. Petitioner argued that the matter of rehiring respondents
rested on the sound discretion of its new owners, and the latter could not be
compelled to absorb petitioners former employees since the same was not part
of the deal. Petitioner alleged that respondents submitted their application for

3E Andaya Ching Espiritu Hefti Galvez Gammad Lainez Lui Madamba Nagera Narvasa
Ong Palangdao Rosales Sanchez Santos Satrain Tabo (2014-2015)

Labor Relations Case Digest - Atty. Joyrich Golangco

reemployment but, after evaluation, the new owners opted not to hire
respondents.
The Labor Arbiter dismissed the case. NLRC reversed. CA affirmed the NLRCs
Decision. A Petition for Certiorari was filed before the Supreme Court.

continues to own and operate petitioner, or even to show that Willy Deterala is
actually in charge of petitioners business. Petitioner did not confront this issue
head-on, and its failure to do so is fatal to its cause. Petitioner having failed to
discharge its burden of submitting sufficient and convincing evidence required
by law, we hold that respondents were illegally dismissed.

HELD: Yes, there were. Closure of business is the reversal of fortune of the
employer whereby there is a complete cessation of business operations and/or
an actual locking-up of the doors of the establishment, usually due to financial
losses. Closure of business, as an authorized cause for termination of
employment, aims to prevent further financial drain upon an employer who can
no longer pay his employees since business has already stopped.

DOCTRINE: Tthe rule is still that the doctrine of piercing the corporate veil
applies only when the corporate fiction is used to defeat public convenience,
justify wrong, protect fraud, or defend crime. In the absence of malice, bad faith,
or a specific provision of law making a corporate officer liable, such corporate
officer cannot be made personally liable for corporate liabilities. Neither Article
212[e] nor Article 273 (now 272) of the Labor Code expressly makes any
corporate officer personally liable for the debts of the corporation.

ISSUES: Whether Sarmiento and Catimbang were illegally dismissed.

Closure or cessation of operation of the establishment is an authorized


cause for terminating an employee, as provided in Article 283 of the Labor Code.

On this ground, petitioner terminated the employment of


respondents. However, what petitioner apparently made was a transfer of
ownership. It is true that, as invoked by petitioner, in Manlimos, et al. v. NLRC, et
al., we held that a change of ownership in a business concern is not proscribed by
law. Lest petitioner forget, however, we also held therein that the sale or
disposition must be motivated by good faith as a condition for exemption from
liability. Thus, where the charge of ownership is done in bad faith, or is used to
defeat the rights of labor, the successor-employer is deemed to have absorbed
the employees and is held liable for the transgressions of his or her predecessor.
But, in this case, there is no successor-employer because there was
no actual change of ownership. We sustain the uniform factual finding of
both the NLRC and the CA that no actual sale transpired and, as such, there
is no closure or cessation of business that can serve as an authorized cause
for the dismissal of respondents.

The fact remains that the Cu family continues to operate petitioners


business. Despite the alleged recent sale to SCBC, represented by Willy Deterala,
petitioner failed to refute the allegations of respondents that the Cu family still

232. Antonio Carag v NLRC

FACTS: Complainant NAFLU is the sole and exclusive bargaining agent


representing all rank and file employees of Mariveles Apparel Corporation
(MAC). It averred that on July 8, 1993, without notice of any kind filed in
accordance with pertinent provisions of the Labor Code, [MAC], for reasons
known only by itself ceased operations with the intention of completely closing
its shop or factory. Such intention was manifested in a letter, allegedly claimed
by MAC as its notice filed only on the same day that the operations closed. That at
the time of closure, employees who have rendered one to two weeks work were
not paid their corresponding salaries/wages, which remain unpaid. LA
Ortiguerra summoned the parties for possible settlement. In their complaint,
they moved to implead Carag and David, Chairman of the Board and President of
MAC, respectively, as to guarantee the satisfaction of any judgement award. Atty.
Pastores, as counsel for respondents, submitted a position paper and stated that
complainants should not have impleaded Carag and David because MAC is
actually owned by a consortium of banks. Carag and David own shares in MAC
only to qualify them to serve as MAC's officers. By way of controversion, they
stated that the allegation of complainants that there was illegal closure is
misplaced because MAC was only on a temporary shutdown. LA Ortiguerra
granted the motion to implead Carag and David and held that they be solidarily
liable with MAC to the complainants. MAC, Carag, and David filed their
Memorandum before the NLRC. Carag, through a separate counsel, filed an

3E Andaya Ching Espiritu Hefti Galvez Gammad Lainez Lui Madamba Nagera Narvasa
Ong Palangdao Rosales Sanchez Santos Satrain Tabo (2014-2015)

Labor Relations Case Digest - Atty. Joyrich Golangco

appeal as well before the NLRC, which however, dismissed the case. Respondents
filed separate petitions for certiorari before the SC and referred the consolidated
cases to the appellate court. CA rendered a decision affirming the decision of LA
Ortiguerra.
ISSUES: WON Carag and David, acting merely as officers of MAC, and not its
owner, should be held liable for the claims asserted by the respondents herein
against MAC

HELD: The rule is that a director is not personally liable for the debts of the
corporation, which has a separate legal personality of its own. Section 31 of the
Corporation Code lays down the exceptions to the rule, as follows:

Liability of directors, trustees or officers. - Directors or trustees who wilfully and


knowingly vote for or assent to patently unlawful acts of the corporation or who
are guilty of gross negligence or bad faith in directing the affairs of the
corporation or acquire any personal or pecuniary interest in conflict with their
duty as such directors or trustees shall be liable jointly and severally for all
damages resulting therefrom suffered by the corporation, its stockholders or
members and other persons.

Section 31 makes a director personally liable for corporate debts if he wilfully


and knowingly votes for or assents to patently unlawful acts of the corporation.
Section 31 also makes a director personally liable if he is guilty of gross
negligence or bad faith in directing the affairs of the corporation. Complainants
did not allege in their complaint that Carag wilfully and knowingly voted for or
assented to any patently unlawful act of MAC. Complainants did not present any
evidence showing that Carag wilfully and knowingly voted for or assented to any
patently unlawful act of MAC. Neither did Arbiter Ortiguerra make any finding to
this effect in her Decision.Complainants did not also allege that Carag is guilty of
gross negligence or bad faith in directing the affairs of MAC. Complainants did
not present any evidence showing that Carag is guilty of gross negligence or bad
faith in directing the affairs of MAC. Neither did Arbiter Ortiguerra make any
finding to this effect in her Decision.

To hold a director personally liable for debts of the corporation, and thus pierce
the veil of corporate fiction, the bad faith or wrongdoing of the director must be
established clearly and convincingly. Bad faith is never presumed. Bad faith does
not connote bad judgment or negligence. Bad faith imports a dishonest purpose.
Bad faith means breach of a known duty through some ill motive or interest. Bad
faith partakes of the nature of fraud. Neither does bad faith arise automatically
just because a corporation fails to comply with the notice requirement of labor
laws on company closure or dismissal of employees. The failure to give notice is
not an unlawful act because the law does not define such failure as unlawful.
Such failure to give notice is a violation of procedural due process but does not
amount to an unlawful or criminal act. Such procedural defect is called illegal
dismissal because it fails to comply with mandatory procedural requirements,
but it is not illegal in the sense that it constitutes an unlawful or criminal act.
Personal liability of corporate directors, trustees or officers attaches only when
(1) they assent to a patently unlawful act of the corporation, or when they are
guilty of bad faith or gross negligence in directing its affairs, or when there is a
conflict of interest resulting in damages to the corporation, its stockholders or
other persons; (2) they consent to the issuance of watered down stocks or when,
having knowledge of such issuance, do not forthwith file with the corporate
secretary their written objection; (3) they agree to hold themselves personally
and solidarily liable with the corporation; or (4) they are made by specific
provision of law personally answerable for their corporate action.
233: BPI vs BPI Union-Davao Chapter

DOCTRINE:
In the hierarchy of labor rights, unionism is favored over
security of tenure.

FACTS: Pursuant to the Article and Plan of Merger all assets and liabilities of Far
East Bank and Trust Company (FEBTC) were transferred to and absorbed by BPI
as the surviving corporation. Petitioner hired FEBTC employees, including those
in its different branches across the country, were hired by BPI as its own
employees, and with their status recognized and salaries maintained.

Respondent Union is the exclusive bargaining agent of BPIs rank-andfile employees in Davao City. The former FEBTC rank-and-file employees in

3E Andaya Ching Espiritu Hefti Galvez Gammad Lainez Lui Madamba Nagera Narvasa
Ong Palangdao Rosales Sanchez Santos Satrain Tabo (2014-2015)

Labor Relations Case Digest - Atty. Joyrich Golangco

Davao City did not belong to any labor union at the time of the merger. Prior to
the merger, respondent Union invited FEBTC employees to a meeting regarding
the Union Shop Clause existing between BPI and respondent union. Respondent
argues that the absorbed employees, by virtue of the Union Shop Clause, join the
respondent Union as a condition for their employment under BPI. FEBTC
employees refused to join hence it was brought before the management but to no
avail until it was submitted for Voluntary Arbitration.

LA: ruled in favor of petitioner BPIs interpretation that the former FEBTC
employees were not covered by the Union Security Clause of the CBA between
the Union and the Bank on the ground that the said employees were not new
employees who were hired and subsequently regularized, but were absorbed
employees by operation of law because the former employees of FEBTC can
be considered assets and liabilities of the absorbed corporation. The
Voluntary Arbitrator concluded that the former FEBTC employees could not be
compelled to join the Union, as it was their constitutional right to join or not to
join any organization.

CA: reversed LA. CA agreed with LA that absorbed employees are different from
new employees. (their only difference absorbed employees length of service
is continued from FEBTC to BPI, no gap) But the CA is more persuaded that the
similarities between the two outweigh the difference between them, to wit: (a)
they have a new employer; (b) new working conditions; (c) new terms of
employment and; (d) new company policy to follow. As such, they should be
considered as new employees for purposes of applying the provisions of the
CBA regarding the union-shop clause.

To rule otherwise would definitely result to a very awkward and unfair


situation wherein the absorbed employees shall be in a different if not, better
situation than the existing BPI employees. The existing BPI employees by virtue
of the union-shop clause are required to pay the monthly union dues, remain as
members in good standing of the union otherwise, they shall be terminated from
the company, and other union-related obligations. On the other hand, the
absorbed employees shall enjoy the fruits of labor of the petitioner-union and
its members for nothing in exchange. Certainly, this would disturb industrial
peace in the company that is the paramount reason for the existence of the CBA
and the union.

A closed-shop agreement is an agreement whereby an employer binds


himself to hire only members of the contracting union who must continue to
remain members in good standing to keep their jobs. It is THE MOST PRIZED
ACHIEVEMENT OF UNIONISM. IT ADDS MEMBERSHIP AND COMPULSORY
DUES. By holding out to loyal members a promise of employment in the closedshop, it wields group solidarity.
ISSUES:

1. May a corporation invoke its merger with another corporation as a valid


ground to exempt its absorbed employees from the coverage of a union shop
clause contained in its existing CBA with its own certified labor union

2. Whether or not the Union Shop agreement violated the constitutional right of
security of tenure of the FEB employees absorbed by BPI.

HELD: 1. NO. The Articles of Merger and Plan of Merger did not contain any
specific stipulation with respect to the employment contracts of existing
personnel of then on-surviving entity which is FEBTC. Unlike the Voluntary
Arbitrator, this Court cannot uphold the reasoning that the general stipulation
regarding transfer of FEBTC assets and liabilities to BPI as set forth in the
Articles of Merger necessarily includes the transfer of all FEBTC employees into
the employ of BPI and neither BPI nor the FEBTC employees allegedly could do
anything about it. Even if it is so, it does not follow that the absorbed employees
should not be subject to the terms and conditions of employment obtaining in the
surviving corporation. An employee is NOT an asset or liability.
The rule is that unless expressly assumed, labor contracts such as
employment contracts and collective bargaining agreements are not enforceable
against a transferee of an enterprise, labor contracts being in personam, thus
binding only between the parties. A labor contract merely creates an action in
personam and does not create any real right, which should be respected by third
parties. This conclusion draws its force from the right of an employer to select
his employees and to decide when to engage them as protected under our
Constitution, and the same can only be restricted by law through the exercise of
the police power.

3E Andaya Ching Espiritu Hefti Galvez Gammad Lainez Lui Madamba Nagera Narvasa
Ong Palangdao Rosales Sanchez Santos Satrain Tabo (2014-2015)

Labor Relations Case Digest - Atty. Joyrich Golangco

2. NO. As a general rule, the State protects the workers right to security of tenure.
An employees services can only be terminated upon just and authorized causes.
In this case, the presence of a Union Shop Clause in the CBA between BPI and BPI
Union must be respected. Failure of an employee to join the union pursuant to
the clause is an authorized cause for BPI not to continue employing the employee
concerned and BPI must respect that provision of the CBA. In the hierarchy of
labor rights, unionism is favored over security of tenure. A contrary
interpretation of the Union Shop Clause would dilute its efficacy and put the
certified union that is supposedly being protected thereby at the mercy of
management. Nevertheless, the FEB employees are still entitled to the twin
notice rule this is to afford them ample opportunity to whether or not join the
union.
234. Fernandez, et al. v. Newfield Staff Solutions, Inc.

DOCTRINE: "A corporation, being a juridical entity, may act only through its
directors, officers and employees. Obligations incurred by them, acting as such
corporate agents, are not theirs but the direct accountabilities of the corporation
they represent. True, solidary liability may at times be incurred but only when
exceptional circumstances warrant such as, generally, in the following cases:
1. When directors and trustees or, in appropriate cases, the officers of a
corporation -

(a) vote for or assent to patently unlawful acts of the


corporation;
(b) act in bad faith or with gross negligence in directing the
corporate affairs;
xxxx

In labor cases, for instance, the Court has held corporate directors and
officers solidarily liable with the corporation for the termination of
employment of employees done with malice or in bad faith."
FACTS: Respondent Newfield Staff Solutions, Inc. (Newfield) hired Fernandez as
Recruitment Manager starting September 30, 2008 with a salary of P50,000 and
an allowance of P6,000 per month. It was provided in the employment

agreement that Fernandez will receive a loyalty bonus of P60,000 and life
insurance worth P500,000 upon reaching six months of employment with
Newfield. Newfield also hired Beltran as probationary Recruitment Specialist
starting October 7, 2008 with a salary of P15,000 and an allowance of P2,000 per
month. Her employment contract provided that Beltran will receive a 10% salary
and allowance increase upon reaching 12 months of employment with Newfield.

Petitioners guaranteed to perform their tasks for six months and breach
of this guarantee would make them liable for liquidated damages of P45,000. It
was further provided in their employment agreements that if they want to
terminate their employment agreements after the "guaranteed period of
engagement," they should send a written notice 45 days before the effective date
of termination. They should also surrender any equipment issued to them and
secure a clearance. If they fail to comply, Newfield can refuse to issue a clearance
and to release any amount due them.
On October 17, 2008, respondent Arnold "Jay" Lopez, Jr., Newfields
General Manager, asked petitioners to come to his office and terminated their
employment on the ground that they failed to perform satisfactorily. Lopez,
Jr. ordered them to immediately turn over the records in their possession to their
successors.

A week later, petitioners received Lopez, Jr.s return-to-work


letters dated October 22, 2008. The letters stated that they did not report since
October 20, 2008 without resigning, in violation of their employment
agreements. They were directed to report and explain their failure to file
resignation letters.
Fernandez countered with a demand letter dated November 11, 2008. She
claimed that her salary of P36,400 from September 30 to October 17, 2008 and
mobile phone expenses of P3,000 incurred in furtherance of Newfields business
were not paid. She also said that she was able to hire one team leader and 12
agents in three weeks, but Newfield still found her performance unsatisfactory
and told her to file her resignation letter. Thus, she referred the matter to her
lawyer. She threatened to sue unless Newfield responds favorably. Beltran for
her part also sent a demand letter. Her demand letter dated November 17, 2008
is similar to Fernandezs letter except for the amount of the claim for unpaid
salary which is P7,206.80.

3E Andaya Ching Espiritu Hefti Galvez Gammad Lainez Lui Madamba Nagera Narvasa
Ong Palangdao Rosales Sanchez Santos Satrain Tabo (2014-2015)

Labor Relations Case Digest - Atty. Joyrich Golangco

When they failed to receive favourable action from respondents,


petitioners filed on December 9, 2008, a complaint for illegal dismissal,
nonpayment of salary and overtime pay, reimbursement of cell phone billing,
moral and exemplary damages and attorneys fees against respondents. The
Labor Arbiter ruled that petitioners dismissal was illegal. Affirmed by the NLRC.
Reversed by the CA.
ISSUES: Whether petitioners were illegally dismissed.

HELD: Yes, they were. The CA erred in ruling that the meeting on October 17,
2008 did not prove that petitioners were dismissed. Neither were petitioners
guilty of abandonment. One, petitioners were absent because Lopez, Jr. had
fired them. Thus, we cannot fault them for refusing to comply with the
return-to-work letters and responding instead with their demand letters.
Neither can they be accused of being AWOL or of breaching their employment
agreements. Indeed, as stated above, respondents cannot claim that no evidence
shows that petitioners were forced not to report for work. Two, petitioners
protest of their dismissal by sending demand letters and filing a complaint for
illegal dismissal with prayer for reinstatement convinces us that petitioners have
no intention to sever the employment relationship. Employees who take steps to
protest their dismissal cannot logically be said to have abandoned their work. A
charge of abandonment is totally inconsistent with the immediate filing of a
complaint for illegal dismissal. The filing thereof is proof enough of ones desire
to return to work, thus negating any suggestion of abandonment. Petitioners
were illegally dismissed since there is no just cause for their dismissal.

The dispositive portion of the Labor Arbiters decision, as affirmed and


modified by the NLRC, stated that "respondents are ordered to pay" petitioners.
This gives the impression that Lopez, Jr. is solidarily liable with Newfield. In
Grandteq Industrial Steel Products, Inc. v. Estrella, we discussed how corporate
agents incur solidary liability, as follows:
There is solidary liability when the obligation expressly so states,
when the law so provides, or when the nature of the obligation so requires.
In MAM Realty Development Corporation v. NLRC, the solidary liability of
corporate officers in labor disputes was discussed in this wise:

"A corporation, being a juridical entity, may act only through its directors,
officers and employees. Obligations incurred by them, acting as such corporate
agents, are not theirs but the direct accountabilities of the corporation they
represent. True, solidary liability may at times be incurred but only when
exceptional circumstances warrant such as, generally, in the following cases:
1. When directors and trustees or, in appropriate cases, the officers of a
corporation -

(a) vote for or assent to patently unlawful acts of the


corporation;

(b) act in bad faith or with gross negligence in directing the


corporate affairs;
xxxx

In labor cases, for instance, the Court has held corporate directors and
officers solidarily liable with the corporation for the termination of
employment of employees done with malice or in bad faith."
Bad faith does not connote bad judgment or negligence; It Imports
dishonest purpose or some moral obliquity and conscious doing of wrong; it
means breach of a known duty through some motive or interest or ill will; it
partakes of the nature of fraud. To sustain such a finding, there should be
evidence on record that an officer or director acted maliciously or in bad faith in
terminating the employee.
But here, the Labor Arbiter and NLRC have not found Lopez, Jr.
guilty of malice or bad faith. Thus, there is no basis to hold Lopez, Jr.
solidarily liable with Newfield. Payment of the judgment award is the direct
accountability of Newfield.

235. SME bank vs Peregrin


DOCTRINE: Security of tenure is a constitutionally guaranteed right. Employees
may not be terminated from their regular employment except for just or
authorized causes under the Labor Code2 and other pertinent laws. A mere
change in the equity composition of a corporation is neither a just nor an

3E Andaya Ching Espiritu Hefti Galvez Gammad Lainez Lui Madamba Nagera Narvasa
Ong Palangdao Rosales Sanchez Santos Satrain Tabo (2014-2015)

Labor Relations Case Digest - Atty. Joyrich Golangco

authorized cause that would legally permit the dismissal of the corporations
employees en masse

FACTS:
Respondent employees were employees of Small and
Medium Enterprise
Bank,
Incorporated
(SME
Bank).
Originally,
the principal shareholders and corporate directors of the bank were Eduardo M.
Agustin, Jr. (Agustin) and Peregrin de Guzman, Jr. (De Guzman). SME Bank
experienced financial difficulties. To remedy the situation, the bank officials
proposed its sale to Abelardo Samson (Samson).
Accordingly, negotiations ensued, and a formal offer was made to
Samson. Samson then sent formal letters to Agustin and De Guzman, demanding
the following as preconditions for the sale of SME Banks shares of stock:chanr
4. You shall guarantee the peaceful turn over of all assets as well as the
peaceful transition of management of the bank and shall terminate/retire the
employees we mutually agree upon, upon transfer of shares in favor of our
groups nominees;

Agustin and De Guzman accepted the terms and conditions proposed by


Samson. Simeon Espiritu (Espiritu), then the general manager of SME Bank,
persuaded the employees to tender their resignations, with the promise that they
would be rehired upon reapplication. His directive was allegedly done at the
behest of petitioner Olga Samson. Relying on this representation, the respondent
employees tendered their resignation.
Agustin and De Guzman signified their conformity to the Letter
Agreements and sold 86.365% of the shares of stock of SME Bank to spouses
Abelardo and Olga Samson. Spouses Samson then became the principal
shareholders of SME Bank, while Aurelio Villaflor, Jr. was appointed bank
president. As it turned out, respondent employees were not rehired.

Respondent-employees demanded the payment of their respective


separation pays, but their requests were denied. Aggrieved by the loss of
their jobs, respondent employees filed illegal dismissal and money claims and
sued SME Bank, spouses Abelardo and Olga Samson and Aurelio Villaflor
(Samson Group)

The LA ruled that the buyer of an enterprise is not bound to absorb its
employees, unless there is an express stipulation to the contrary. However, he
also found that respondent employees were illegally dismissed, because they had
involuntarily executed their resignation letters after relying on representations
that they would be given their separation benefits and rehired by the new
management. Accordingly, the labor arbiter decided the case against Agustin
and De Guzman, but dismissed the Complaint against the Samson Group. NLRC
and CA affirmed decision of LA.
ISSUES: 1. Whether or not the respondent employees were illegally dismissed?
2. Whether or not SME, de Guzman and Agustin were liable?
HELD:

1. YES. Petitioner bank also argues that, there being a transfer of the business
establishment, the innocent transferees no longer have any obligation to
continue employing respondent employees, and that the most that they can do is
to give preference to the qualified separated employees; hence, the employees
were
validly
dismissed.57cralawlibrary
The argument is misleading and unmeritorious. Contrary to petitioner banks
argument, there was no transfer of the business establishment to speak of,
but merely a change in the new majority shareholders of the corporation.
There are two types of corporate acquisitions: asset sales and stock sales. In
asset sales, the corporate entity sells all or substantially all of its assets to
another entity. In stock sales, the individual or corporate shareholders sell a
controlling
block
of
stock to
new
or
existing
shareholders.

In asset sales, the rule is that the seller in good faith is authorized to dismiss the
affected employees, but is liable for the payment of separation pay under the
law.63 The buyer in good faith, on the other hand, is not obliged to absorb the
employees affected by the sale, nor is it liable for the payment of their
claims.64 The most that it may do, for reasons of public policy and social justice, is
to give preference to the qualified separated personnel of the selling firm.

3E Andaya Ching Espiritu Hefti Galvez Gammad Lainez Lui Madamba Nagera Narvasa
Ong Palangdao Rosales Sanchez Santos Satrain Tabo (2014-2015)

Labor Relations Case Digest - Atty. Joyrich Golangco

In contrast with asset sales, in which the assets of the selling corporation are
transferred to another entity, the transaction in stock sales takes place at the
shareholder level. Because the corporation possesses a personality separate and
distinct from that of its shareholders, a shift in the composition of its
shareholders will not affect its existence and continuity. Thus, notwithstanding
the stock sale, the corporation continues to be the employer of its people and
continues to be liable for the payment of their just claims. Furthermore, the
corporation or its new majority shareholders are not entitled to lawfully dismiss
corporate employees absent a just or authorized cause.
In the case at bar, the Letter Agreements show that their main object is the
acquisition by the Samson Group of 86.365% of the shares of stock of SME
Bank. Hence, this case involves a stock sale, whereby the transferee acquires the
controlling shares of stock of the corporation. Thus, following the rule in stock
sales, respondent employees may not be dismissed except for just or authorized
causes under the Labor Code.

2. Yes. None of the parties dispute that SME Bank was the employer of
respondent employees. The fact that there was a change in the composition of its
shareholders did not affect the employer-employee relationship between the
employees and the corporation, because an equity transfer affects neither the
existence nor the liabilities of a corporation.

precondition in the Letter Agreements as to the termination or retirement of


SME Banks employees. However, instead of going through the proper procedure,
the bank manager induced respondent employees to resign or retire from their
respective employments, while promising that they would be rehired by the new
management. Fully relying on that promise, they tendered courtesy resignations
or retirements and eventually found themselves jobless. Clearly, this sequence of
events constituted a gross circumvention of our labor laws and a violation of the
employees constitutionally guaranteed right to security of tenure. We therefore
rule that, as Agustin and De Guzman are corporate directors who have acted in
bad faith, they may be held solidarily liable with SME Bank for the satisfaction of
the
employees
lawful
claims.
As to spouses Samson, we find that nowhere in the records does it appear that
they were either corporate directors or officers of SME Bank at the time the
illegal termination occurred, except that the Samson Group had already taken
over as new management when Simeon, Jr. was constructively dismissed. Not
being corporate directors or officers, spouses Samson were not in legal control of
the bank and consequently had no power to dismiss its employees.
*** NOTING FOLLOWS***

Unless they have exceeded their authority, corporate officers are, as a


general rule, not personally liable for their official acts, because a corporation, by
legal fiction, has a personality separate and distinct from its officers,
stockholders and members. However, this fictional veil may be pierced whenever
the corporate personality is used as a means of perpetuating a fraud or an illegal
act, evading an existing obligation, or confusing a legitimate issue. In cases of
illegal dismissal, corporate directors and officers are solidarily liable with the
corporation, where terminations of employment are done with malice or in bad
faith.

There is no question that both Agustin and De Guzman were corporate


directors of SME Bank. An analysis of the facts likewise reveals that the dismissal
of the employees was done in bad faith. Motivated by their desire to dispose of
their shares of stock to Samson, they agreed to and later implemented the

3E Andaya Ching Espiritu Hefti Galvez Gammad Lainez Lui Madamba Nagera Narvasa
Ong Palangdao Rosales Sanchez Santos Satrain Tabo (2014-2015)

You might also like